You are on page 1of 781

CONCEPTUAL

APPROACH TO TAXES
TAX YEAR 2016
6th EDITION

Income Tax
Sales tax
Capital Value Tax
Federal Excise Act & Rules, 2005

By:

NADEEM BUTT
Chartered Accountant
FCA, FPFA, B.Com.

AMAN PUBLICATIONS COMPANY


ALL RIGHTS RESERVED BY THE AUTHOR

No responsibility shall be taken for any mistake, error or


omission regarding any material contained in this book.

6TH Edition: 2016

AUTHOR
NADEEM BUTT
Chartered Accountant
FCA, FPFA, B.Com.

PRICE Rs. 600/-


SALIENT FEATURES OF THIS EDITION

Use of easy & understandable language.

Covers the syllabus of CA Module C, CAM-IV, ICMAP Stage IV & M.Com.

Helpful for the students & teachers of CA - Final, ACCA (PAK), Higher National Diploma
(HND), MBA, MPA, PIPFA & LLB.

Added various examples & solved practice questions to demonstrate the theory.

Reference of relevant provisions of income tax & sales tax law has been given.

Added after each chapter past papers topic wise theoretical questions of ICMA Stage IV from
tax year 2003 to 2015 & CA Module C / Stage-II CAF-06 from tax year 2001 to 2015.

Added past papers numerical questions of ICMA Stage IV from tax year 2003 to 2015 & CA
Module C / Stage-II CAF-06 from tax year 2001 to 2015.

Includes the amendments incorporated by the Finance Act, 2015.

Useful for all who want to learn, teach & practice Income tax & Sales tax.

FACILITY FOR ANY QUESTION OR QUERY FROM THE AUTHOR AT:

Ph No: 042-35408635-6 and 042-35408638-9

Cell: 0333 / 0301 - 4245240

E-mail: info@nbandco.com and


nadeembutt@nbandco.com
PREFACE TO THE SIXTH EDITION

With the grace of Almighty ALLAH and prayers of my family, friends, and students, I am presenting
the Sixth Edition of Conceptual approach to taxes. This book is primarily for the students of CAF
- of ICAP, Stage IV of ICMAP and M. Com.

I hope this book will serve the purpose of the students, teachers & other persons related with the
taxation. The book primarily consists of notes on each chapter with examples, multiple choice
questions, practical problems with solutions and topic wise last year questions from tax year 2003
to 2015.

I am grateful to all those who contributed a lot in the completion of this book. My special thanks
are due to my staff.

All positive criticism with suggestions for the improvement will be entertained.

Nadeem Butt
Chartered Accountant

October 01, 2015


THE AUTHOR

Mr. Nadeem Butt qualified as a Chartered Accountant in 1998 & after working with a well
known professional firm as Tax Manager has started his own professional firm under the
name and style Nadeem & Co. Chartered Accountants in 2007 & is practicing as Fellow
member of ICAP mainly in audit, income tax & sales tax for more than 200 various
corporate and non-corporate clients in Pakistan. He is also the life time member of Lahore
Tax Bar Association and Audit & Tax advisor of All Pakistan Cottage Industry & Small
Traders Association.

Member, (special invitees) Taxation Committee of Lahore Chamber of Commerce & Industry
(LCCI).

17 years post qualification teaching experience in the subjects of Taxation..

Visiting faculty member of the following professional educational institutions in providing


coaching for the exams of Taxation:

1. College of Accountancy & Professional Studies(CAPS) Module C & F

2. Institute of Cost & Management Accountants of Pakistan (ICMAP) Stage IV

3. Institute of Chartered Accountants of Pakistan (ICAP) Module C

4. Rise School of Accountancy Module C

5. Premier DLC (A project of Beacon house) AFC- CA

6. The University of Lahore M. Com.

7. SKANS School of Accountancy Module C

8. Punjab Group of Colleges M. Com.

9. GC University, Lahore Business Taxation Coaching

Appointed as auditor on behalf of the tax department by the Federal Board of Revenue for
Special Audit Under section 4A of the repealed Income Tax Ordinance of various corporate
and non-corporate entities.

Two students of Mr. Nadeem Butt (FCA) got 1st & 2ndpositions all over the Pakistan by
scoring 98% & 97% marks in Taxation exam of ICMAP.

Student of Mr. Nadeem Butt (FCA) got the first gold medal in the history of SKANS School
of Accountancy, Lahore in Financial Accounting Module C exam of ICAP.
INCOME TAX
For CA & ICMAP Students
S. No. CHAPTERS Page No.
1 Taxation System 1
2 Ethics in tax Laws 9
3 Constitutional Provisions 17
4 Preliminary 27
5 Introduction & geographical source of income 59
6 Computation of taxable income 71
7 Income from Salary 85
8 Income from property 111
9 Income from business 125
10 Assets and depreciation 147
11 Method of accounting & records 171
12 Capital gains 183
13 Income from other sources 205
14 Losses 219
15 Tax credits 237
16 Common rules 255
17 Returns and Assessments 265
18 Appeals & Revisions 287
19 Income Tax Authorities 301
20 Exemptions other than Covered in Respective Chapters 319
Solved Past Papers Income Tax Numericals of CA Module
21
C - (2001 to 2015) 323
For CA Mod F & ICMAP Students
S. No. CHAPTERS Page No.
22 Final Tax Regime & Minimum Tax 383
23 Deduction / Payment of Tax 407
24 Offences and prosecutions 455
25 Insurance business 465
26 Oil, natural gas & other mineral deposits 473
27 Banking business 481
28 Solved Past Papers Income Tax Numericals of ICMAP Stage IV 489
(2003 to 2015)
SALES TAX
For CA & ICMAP Students
S. No. CHAPTERS Page No.

1 Preliminary 535
2 Registration 549
3 Sales Tax Returns 561
4 Records & Books 569
5 Scope and Payment of Tax 577
6 Practice questions with solutions 603
7 Solved Past Papers Sales Tax Numericals of ICAP - (2003 to 2015) 615

For CA Mod F & ICMAP Students


S. No. CHAPTERS Page No.

8 Recovery of Arrears & Refund 637


9 Offences & Penalties 649
10 Appeals 659
11 Sales Tax Special Procedure Rules, 2007 665
12 Sales Tax Special Procedure (Withholding) Rules, 2007 685
13 Solved Past Papers Sales Tax Numericals of ICMAP Stage IV
(2003 to 2015) 691
14 Third, fifth and sixth Schedules 705
15 Eighth and Ninth Schedules 715
CAPITAL VALUE TAX & FEDERAL
EXCISE DUTY
For CA Mod F & ICMAP Students
S. No. CHAPTERS Page No.

(A) CAPITAL VALUE TAX 729

(B) FEDERAL EXCISE ACT & RULES, 2005

1 Definition 733
2 Levy, Collection & Payment of Duty 737
3 Offences & Penalties 745
4 Federal Excise Rules, 2005 751
Syllabus CAF-6 OF ICAP
Objective
The aim of this paper is to develop basic knowledge and understanding in the core areas of Income Tax and its
chargeability as envisaged in the Income Tax Ordinance 2001 and the Income Tax Rules 2002 (relevant to the
syllabus), Sales Tax Act 1990 and the Sales Tax Rules (relevant to the syllabus).

Grid Weighting
Objective, system and historical background, constitutional provisions and ethics 10
Income tax 65
Sales tax 25
Total 100

Syllabus
Contents Level Learning Outcome
Ref

A Basic concepts of taxation


1 Objectives 1 LO 1.1.1: Comprehend the main objectives of taxation
LO 1.1.2: Justify Taxation as means of development

2 Basic concept 1 LO 1 2.1: Understands the implication of direct and


indirect taxation

3 System of taxation in Pakistan 1 LO 1.3.1: Comprehend different kinds of taxes and


(Income Tax, Sales Tax, Federal their scope
Excise Tax, Capital Value Tax,
Customs)
4 Historical background 1 LO 1.4.1: State the history of taxation in sub-continent

B Constitutional provisions
1 Federal financial procedures 1 LO 2.1.1: Demonstrate familiarity with the Federal
(Article 78 to 88 of the Constitution Consolidated Fund and Public Account
of Pakistan) LO 2.1.2: Demonstrate familiarity with the expenditure
that can be charged upon Federal Consolidated Fund
2 Provincial financial procedures 1 LO 2.2.1: Demonstrate familiarity with the Provincial
(Article 118 to 127 of the Consolidated Fund and Public Account
Constitution of Pakistan) LO 2.2.2: Demonstrate familiarity with the expenditure
that can be charged upon Provincial Consolidated
Fund

i
Syllabus
Contents Level Learning Outcome
Ref

3 Distribution of revenues between 1 LO 2.3.1: Demonstrate familiarity with the formation of


Federation and the Provinces National Finance Commission and its main function.
(Article 160 to 165A of the LO 2.3.2: Demonstrate familiarity with the taxes that
Constitution of Pakistan) can be raised under the authority of Parliament
LO 2.3.3: Demonstrate familiarity with the powers of
provincial assemblies in respect of professional tax
LO 2.3.4: Demonstrate familiarity with the exemption
available to federal and provincial governments
LO 2.3.5: Demonstrate familiarity with the tax on
corporation owned by federal and provincial
government
4 Federal legislative lists relating to 1 LO 2.4.1: Enlist the revenue collection mentioned at
revenue generating measures S. No. 43 to 53 in Fourth Schedule attached to the
(related part of Fourth Schedule to Constitution
the Constitution of Pakistan)
C Ethics
1 Ethics for tax legislation 2 LO 3.1.1: Describe how canons of taxation developed
by economist are relevant for legislators while
formulating tax policies
2 Ethics for taxpayers 2 LO 3.2.1: Understand the right and purpose of state to
tax its citizen
LO 3.2.2: Understand morality behind compliance
with tax laws
3 Ethics for tax implementing 2 LO 3.3.1: Understand the powers Vs. ethical
authorities responsibilities of tax implementation authorities
LO 3.3.2: Understand pillars of tax administration,
namely, fairness, transparency, equity and
accountability
4 Tax evasion and avoidance 2 LO 3.4.1: Explain with simple examples the basic
difference between evasion and avoidance of tax

ii
Syllabus
Contents Level Learning Outcome
Ref

D Income Tax
1 Chapter I Preliminary (concepts 1 LO 4.1.1: Describe the definitions given in section 2
of terms defined section 2 sub- sub-section 1, 5, 5A, 6, 7, 9, 10, 11A, 19, 19C, 20, 21,
section 1, 5, 5A, 6, 7, 9, 10, 11A, 22, 23, 29, 29A, 29C, 36, 37, 38, 41, 44A, 46, 47, 49,
50, 51, 52, 53, 68
19, 19C, 20, 21, 22, 23, 29, 29A,
LO 4.1.2: Describe other definitions covered under
29C, 36, 37, 38, 41, 44A, 46, 47,
relevant sections
49, 50, 51, 52, 53, 68) LO 4.1.3: Apply definitions on simple scenarios
2 Chapter II Charge of tax 2 LO 4.2.1: Explain the chargeability of tax with simple
(excluding section 7) examples
3 Chapter III Tax on Taxable 2 LO 4.3.1: Compute taxable income and tax thereon
income (Excluding Section 29A, 30 relating to salary, income from property, income from
and 31) business, capital gain, dividend, profit on debt, ground
rent, rent from sub-lease, income from provision of
amenities, utilities or any other services connected
with rented building and consideration for vacating the
possession of building
4 Chapter IV (Part I, II and III) 1 LO 4.4.1: Understand and apply on simple scenarios
Common rules (Excluding Sections provisions for income of joint owner, apportionment of
78 and 79) deductions, fair market value and receipt of income
LO 4.4.2: Explain using simple examples the
provisions relating to tax year
LO 4.4.3: Explain with simple examples the provisions
relating to disposal and acquisition of assets, cost and
consideration received
5 Chapter V Part I Central 2 LO 4.5.1: Describe with simple examples the meaning
concepts of persons, resident and non-resident persons and
associates
6 Chapter V Part II Div I and II 2 LO 4.6.1: Describe with simple examples the
Individuals (Excluding Section principles of taxation of individuals
88A)
7 Chapter V Part III Association of 2 LO 4.7.1: Describe with simple examples the
persons principles of taxation of association of persons
8 Chapter VII Part II Taxation of 2 LO 4.8.1: Understand the applicability of tax on
foreign-source income of residents foreign salary income, credit against foreign tax and
treatment of foreign loss of a resident on simple
scenarios.
9 Chapter X Part I Returns 2 LO 4.9.1: Identify persons required to furnish a return
of income
LO 4.9.2: Identify persons not required to furnish a
return of income
LO 4.9.3: Identify persons required to furnish wealth
statements
LO 4.9.4: List the contents of wealth statement
10 Chapter X Part II Assessments 1 LO 4.10.1: Understand the meaning of assessment by
Commissioner and power of Commissioner to conduct
audit

iii
Syllabus
Contents Level Learning Outcome
Ref

11 Chapter X Part III Appeals 1 LO 4.11.1: List the appellate bodies


LO 4.11.2: Explain using simple examples the
circumstances when appeal to the Commissioner
(Appeals) is made and the pre-conditions applicable
LO 4.11.3: Explain the provisions relating to decision
of appeals by Commissioner (Appeals) using simple
examples
12 Chapter X Part VIII Records, 2 LO 4.12.1: Understand the provisions relating to
Information Collection and Audit records to be kept by the taxpayers
(Section174 and 177) LO 4.12.2: Describe the provisions relating to audit by
Commissioner
13 The Income Tax Rules related to 1 LO 4.13.1: Apply rules relevant to learning outcomes
the above chapters of the Income specified against each topic on well explained
Tax Ordinance 2001 shall also be scenarios
examined
14 First and Second Schedule 2 LO 4.14.1: Apply clauses relevant to learning
attached to the Income Tax outcomes specified against each topic on well
Ordinance 2001 explained scenario

iv
Syllabus
Contents Level Learning Outcome
Ref

E Sales Tax
Sales Tax Act 1990
1 Chapter I Preliminary (concepts 2 LO 5.1.1: Describe the definitions given in section 2
of terms defined Section 2 sub- sub-section 3, 5AA, 9, 11, 14, 16, 17, 20, 21, 22A, 25,
sections 3, 5AA, 9, 11, 14, 16, 17, 27, 28, 29A, 33, 35, 39, 40, 41, 43, 44, 46
20, 21, 22A, 25, 27, 28, 29A, 33, LO 5.1.2: Describe other definitions covered under
35, 39, 40, 41, 43, 44, 46) relevant sections
LO 5.1.3: Apply definitions on simple scenarios
2 Chapter II Scope and payment of 2 LO 5.2.1: Understand the application sales tax law on
tax taxable supplies including zero rated and exempt
supplies
LO 5.2.2: State the determination, time and manner of
sales tax liability and payment using simple examples
3 Chapter III Registration LO 5.3.1: State the requirement and procedure of
registration
4 Chapter IV Book keeping and LO 5.4.1: List the record to be kept by a registered
invoicing requirements person
LO 5.4.2: State the requirements of tax invoice
LO 5.4.3: Explain the retention period of record using
simple examples.
5 Chapter V Returns LO 5.5.1: Understand the various types of returns
required to be filed by registered and un-registered
persons.
Sales Tax Rules, 2006
1 Chapter I Registration, 2 LO 6.1.1: Explain the requirement and procedure of
Compulsory registration and De- registration, compulsory registration and
registration deregistration using simple examples
2 Chapter II Filing of return 2 LO 6.2.1: Explain the requirement and procedure of
filing of return using simple examples
3 Chapter III Credit and Debit And 2 LO 6.3.1: Explain the requirement and procedure of
Destruction of Goods issuing debit and credit notes using simple examples
LO 6.3.2: State the procedure for destruction of goods
4 Chapter IV Apportionment of 2 LO 6.4.1: Explain the requirement and procedure of
Input Tax apportionment of input tax using simple examples

v
Syllabus ICMAP Stage IV
Introduction:
This course covers Income Tax Ordinance, 2001, the Income Tax Rules, 2002 and other Tax Laws such
as the Sales Tax Act, 1990, Customs Act, 1969 and Federal Excise Act and Rules, 2005, as amended
to date.
Objectives:
To provide the students with an in-depth knowledge of Tax Laws, enabling them to apply in decision-
making process in different business situations.
Outcomes:
On completion of this course, students should be able to:
identify and interpret principal types of taxation, such as direct taxes on individuals, income,
business individuals, Association of persons, registered and unregistered firms and companies,
trading profit and capital gains, and indirect taxes such as sales tax, customs duty and central excise
duty,
describe features of the direct and indirect taxes,
describe record-keeping, filing and tax payment requirements of principal types of taxation, relating
to business,
compute for recommendations to the management on issues, pertaining to tax liabilities of company
or firm, arising from income generation and capital gains,
compute and advise on tax liabilities of individuals, arising from income receipts, capital gains,
business or professions and other sources, and
Identify foreign tax obligations, situations and apply appropriate methods for relieving from such tax.

Indicative grid

PART SYLLABUS CONTENT AREA WEIGHTAGE

INCOME TAX
A 50%
1. Income Tax Ordinance, 2001
2. Income Tax Rules, 2002
INDIRECT TAX (SALES TAX)
3. Sales Tax Act, 1990
B 30%
4. Sales Tax Rules
5. Sales Tax Special Procedures
6. Federal Excise Act and Rules, 2005
INDIRECT TAX (OTHER INDIRECT TAXES)
7. Sindh Sales Tax on Services Act, 2011
8. Sindh Sales Tax on Services Rules, 2011
9. The Punjab Sales Tax Ordinance 2000
C 10. Punjab Sales Tax on Services Rules, 2012 20%
11. Islamabad Capital Territory (Tax on Services) Ordinance,
2001
12. The North-west Frontier Province Sales Tax Ordinance,
2000
13. The Balochistan Sales Tax Ordinance, 2000
TOTAL 100%

Note: The weightage shown against each section indicates, study time required for the topics in that
section. This weightage does not necessarily specify the number of marks to be allocated to that section
in the examination.

vi
Detailed Contents of Syllabus of ICMAP Stage IV
PART-A
INCOME TAX
1. The Income Tax Ordinance 2001

Definitions; charge to tax; tax on taxable income (computation of income from salary, property,
business, capital gains (as per Capital Gain Tax Ordinance 2012), other sources, exemptions, losses,
deductible allowances, tax credits); common rules (general, tax year, assets); provisions governing
persons (concept, individuals, AOP, companies); special industries (insurance, oil & gas and other
mineral deposits); international (geographical source of income, taxation of foreign source of income of
residents, taxation of non-residents, double taxation); anti-avoidance; procedures (returns,
assessments, appeals, collection and recovery of tax, payments and deductions, refunds, records and
audit, penalty, offence and prosecutions, additional tax); administration (general, transitional advance
tax provisions, miscellaneous); Schedules (first schedule, second schedule, third schedule, sixth
schedule, seventh schedule). Special provisions regarding depreciation, initial allowance, intangibles,
pre-commencement expenses, scientific research expenditures, employees training and facilities, profit
on debt, financial costs and lease payment, bad debts, provisioning regarding consumer loans, profit on
non-performing debts, transfer to participating reserve and tax accounting. Nature and areas of tax
management; deduction of tax at source; advance payment of tax; minimum tax.
2. Income Tax Rules, 2002

Definitions related to the rules; heads of income; income of residents; tax of non-residents; transfer
pricing; records and books of accounts; certificates; advance tax collection or deduction, payment,
statements of tax collected or deducted. Income tax recovery rules, registration of income tax
practitioners, Recognized terminal benefits funds.

vii
PART-B
SALES TAX
3. The Sales Tax Act, 1990
Chapter No. I, II, III, IV, V VII, VIII and IX of the Act, as amended up-to-date covering; definitions; scope
and payment of tax; registration; book-keeping and invoicing requirements; returns; offences and
penalties, appeals and recovery of arrears.
4. The Sales Tax Rules 2006
Definitions related to rules, registration, compulsory registration and de-registration, filing of returns,
credit and debit note destruction of goods, apportionment of input tax, refund, supply of zero-rated
goods to diplomats, diplomatic missions, privileged persons and privileged organizations, taxpayers
authorized representatives, alternative dispute resolution, special procedure for issuance of electronic
sales tax invoices between buyers and sellers.
5. Sales Tax Special Procedure Rules 2007
Payment of Sales Tax by Retailers, providing or Rendering Services Subject to Tax under Provincial
Laws, Refund claim by the Persons Engaged in Making Zero Rates, payment of Sales Tax by Importers.
Payment of Tax by Steel Melters and Ship Breakers, sales tax special procedure (withholding) Rules
2007.

INDIRECT TAX

6. Federal Excise Act and Rules, 2005

Federal Excise Act, 2005, Sections 2 to 19, 31 & 33 to 39, Federal Excise Rules, 2005, Rules 7 to 10,
15 to 17, & 32 to 34. Capital value tax: section 7 of the Finance Act 1989 as amended up-to-date.
Notifications, rules general orders and circulars, issued under the above-mentioned laws

PART C

PROVINCIAL SALES TAX

7. Sindh Sales Tax on Services Act, 2011

Preliminary, Scope of Tax, Payment and Collection of Tax on Taxable Services, Registration & De-
Registration, Book Keeping and Audit Proceedings, Returns

8. Sindh Sales Tax on Services Rules, 2011


Definitions, Registration and De-Registration, Filling of Returns, Adjustment of Input Tax And Tax paid,
Procedure For Collection of Sales tax on services.

9. The Punjab sales Tax ordinance 2000

Preliminary, Scope of Tax, Payment & Collection of Tax on Taxable Services, Registration & De-
Registration, Book Keeping And Audit Proceedings, Returns, First schedule (Classification of Services),
Second Schedule (Taxable Services)

10. Punjab sales Tax on services Rules, 2012

Definition, Registration & De- Registration, Filling of Return, Adjustment of Tax.

11. Islamabad Capital Territory (Tax on Services) Ordinance, 2001

12. The NorthWest Frontier Province Sales Tax Ordinance, 2000

13. The Balochistan Sales Tax Ordinance, 2000

viii
Syllabus M.Com.
COURSE DESCRIPTION:
This course is designed to give student an understanding of the structure of taxation system in Pakistan
and the policy factors essential to the application of tax system in business enterprise. This
comprehensive course will enable students to know how the taxation system of the Pakistan allows
them to take effective business decisions.it will also guide them to apply current tax rules in their
organization for effective usage of resources,
LEARNING OBJECTIVES:
After studying this course the student should be able:
a) to understand the various terms with concepts used under the income tax and sales tax law in
Pakistan;
b) to compute income, total income and taxable income and tax thereon of individuals, AOPs /
partnerships and corporations after taking into account the different exemptions and reliefs
available;
c) to compute the advance tax liability under section 147 and withholding tax provisions under
specific sections of the Income tax Ordinance, 2001;
d) to understand how income tax returns are filed, how assessment under various sections framed
and ultimately the procedure to file appeals under the Income tax Ordinance, 2001; and
e) to file the various sales tax returns and how to compute the sales tax liability / refund of various
persons in Pakistan.
Course contents:
INCOME TAX ORDINANCE 2001 AND INCOME TAX RULES 2002
Overview, scheme and scope
Comprehension of basic rules and concepts
Basic principles of construction, relevant definitions / concepts
1. Assessment [section 2(5) ]
2. Business[section 2,(9) ]
3. Deductible allowance[section 2(16) ]
4. Dividend [section 2(19) ]
5. Employment [section 2(22) ]
6. Income [section 2(29) ]
7. Permanent establishment [section 2(41) ]
8. Persons [section 2(42) ]
9. Taxable income [section 2(64) Read with total income under section 2(69)]
10. Taxpayer [section 2(66) ]
11. Tax year [[section 2 (68) ]
TAX ADMINISTRATION
[COVERED IN 19TH CHAPTER]
1. Tax authorities [section 207]
2. Circulars, orders and directions issued by the FBR [Section 206,213 and 214]
3. Advance rulings [section 206 A]
Provisions governing persons [Section 80 to 84]
Central concepts
Person
Resident and non / resident persons
Associates
ix
Tax on taxable income

1. Heads of income [section 11 ]


2. Income for tax purposes [section 9 and 10 ]
3. Tax payable on taxable income [first Schedule to the Ordinance ]

Salary

[Sections 12 to 14 along with all other operating sections read with rule 3 to 7 income tax rules, 2002]

1. Basis of assessment
2. Residence
3. Employee share schemes
4. Perquisites and benefits in kind
5. Taxation at source
6. Pension, gratuity and receipts from provident fund
7. Exemptions and tax concession

Income from property

1 Basis of charge
2 Non / adjustable amounts received in relation to buildings

Profits chargeable to tax as income from business

[Sections 18 to 36, relevant clauses of Second Schedule and Rules 10 TO 12 and 13]

1 Principles regarding chargeable income


2 General principles for all allowable deductions
3 Deductions not allowed
4 Depreciation
5 Special provisions for allowable deductions in computing profits chargeable to tax as income
from business
6 Intangibles
7 Pre /commencement expenditure
8 Scientific research expenditure
9 Employee training and facilities
10 Profit on debt, financial costs and lease payments
11 Bad debts
12 Profit on the non-performing debts of a banking company or development finance institution
13 Transfer to participatory reserve
14 Relief for losses
15 Exemptions and tax concessions

Capital gains

[Sections 2(10) read with 37, section 76, 77, 78, 79, 95 to 97 and various exemption clauses]

1 Principles for computing capital gains


2 Deduction of losses
Income from other sources [Section 39, 40, 55(1), 67, 89,101(6), 111]

1. Description of other sources


2. Allowable deductions

x
Tax credits [Sections 61 to 65]

Set off and carry forward of losses [Sections 56 to 59B]

A. Principles relating to inter-head adjustments


B. Rules for carrying forward and subsequent adjustments

Minimum tax [Sections 113 to113B]

Advance tax and withholding tax regime [Sections 14, 231A to 236B]

Default surcharge [Section 205 and 205A]

Procedure for filing of income tax returns and assessment [Sections 114 to 126]

Appellate system [Sections 127 to 136]

xi
SALES TAX ACT 1990 OR REFORMED GST WITH RELEVANT RULES / NOTIFICATIONS

Overview, scheme and scope

Comprehension of basis rules and concepts relevant definitions/ concepts

1 Due date [section 2(9)]


2 Exempt supply [section 2(11)]
3 Goods [section 2(12)]
4 Input tax [section 2(14) ]
5 Registered person [section 2(25)]
6 Supply [section 2(33) ]
7 Tax [section 2(34)]
8 Taxable activity [section 2(41)]
9 Taxable supply [section 2(43)]
10 Tax period [section 2(43)]
11 Time of supply [section 2(44)]

Scope and payment of tax

[Sections 3 to13 with relevant rules 19 to 25]

Scope of chargeability, mode of collection etc.


Zero rating
Change in the rate of tax
Time and manner of payment
Determination of tax liability
Tax credit not allowed
Debit and credit note
Excess amount to be carried forward or refunded
Assessment of tax
Short-paid amounts recoverable without notice
Exemption

Return

[Sections 26, 26AA, section 27 to 29 with relevant Rules 13 to 18]

Monthly return
Turnover tax return
Retail tax return
Special return
Final return
Return deemed to have been made

xii
Taxation System Chapter-01

Chapter

1 TAXATION SYSTEM

Sr. No. TOPICS COVERED

1. History of tax laws in Pakistan

2. Introduction to different taxation laws of Pakistan

3. Basics of taxation laws

4. Objectives of tax laws

(FOR CAF-6, MODULE F & ICMAP STUDENTS)


1. HISTORY OF TAX LAWS IN PAKISTAN

1.1 History of tax laws in Pakistan

In undivided India (now consisting of Pakistan, Bangladesh and India) income tax was introduced for the first
time in the year 1860. It was introduced by Income Tax Act, 1860 and exactly the same pattern was followed
that was prevailing in those days in the United Kingdom. This Act came into force on July 31, 1860 and
continued for only five (5) years upto 01-08-1865 when it was completely withdrawn. A major characteristic of
this Act was that the agricultural income from land, above the rental value of RS.600 per annum, was taxable.

In Pakistan, Federal Government is empowered to levy and collect tax on the income of a person other than
the income taxable in the domain of the respective Provincial Governments.

Later on some of the provinces imposed general income tax on traders being as Income Tax Act of 1886. This
Act of 1886 was a great improvement on earlier enactments. Its basic scheme, by and large, survives till today.
It introduced the definition of agricultural income which is almost the same as in the income tax ordinance
2001. This Act continued in force for 32 years till 1918.

The 1918 Act consolidated a number of wartime amendments. A graduated super tax on income over
Rs.50,000 and on the undistributed profits of the corporation and other entities was introduced by the Super
Tax Act of 1917 and continued in force through modifications by the Super Tax Act of 1920. The Income Tax
Act and the Super Tax Act were later on consolidated in another Act i.e. the Income Tax Act of 1922.

After independence from British rule on 14-08-1947, the Pakistan Government adopted the Income tax Act,
1922, as amended upto that date. The provisions of the Act were extended to the whole of Pakistan except the
special areas. The Income tax Act 1922 continued for 57 years till 1979. Due to a number of changes the
Government has faced difficulties in its implementation the Government introduced Income tax Ordinance,
1979.
The job of improving the law continued after the promulgation this Ordinance through National Tax Reforms
Commission in 1985. The commission suggested that Income Tax Ordinance 1979 should be replaced by
Income Tax Ordinance, 2001. This new Income Tax Ordinance was promulgated on 13-09-2001 and it has
became effective from 01-072002. The Central Board of Revenue (now FBR) has claimed that the new
Ordinance is a justifiable, pragmatic, easy to understand and in accordance with the global environment. All
the income tax returns for the income earned from 01-07-2002 onwards are being taxed under this law.

Conceptual Approach to Taxes 1


Taxation System Chapter-01

The tabular presentation of laws in Pakistan is as under:

SR. Name of prevailing Law Remarks

1. Income tax Act, 1860 Repealed in 1865

2. Income tax Act, 1886 Effective till 1917

3. Income tax Act, 1918 including Super Tax Act, 1917 Enforced till 1920

4. Income tax Act, 1922 (Merged Income Tax Act, 1918 with Effective till 30-06-1979
Super Tax Act, 1917)

5. Income tax Ordinance, 1979 Effective till 30-06-2002

6. Income tax Ordinance, 2001 Enforced from 01-07-2002

2. INTRODUCTION TO DIFFERENT TAXATION LAWS OF PAKISTAN

2.1 Brief overview of different direct and indirect taxes


Federal taxes in Pakistan like most of the taxation systems in the world are classified into two broad
categories, viz., direct and indirect taxes. A broad description regarding the nature of administration of these
taxes is explained below:
DIRECT TAXES

SR. Name of prevailing Law Remarks

1. Income tax Ordinance, 2001 Being as direct tax the income Tax Ordinance, 2001, tax is
levied on the taxable income of a taxpayer earned during a
tax year computed by applying the specified tax rates as
applicable to respective Taxpayer.
For the purpose of the charge of tax and the computation of
total income, all income is classified under the following five
heads:

Income from salary


Income from property
Income from business
Capital gains; and

Income from other sources

2. Income Support Act, 2013 Although effective from tax year 2013 on every individual
whose net moveable wealth as per wealth statement
exceeds from Rs. 1 Million, he has to pay income support
levy @ 0.5%. However this Act has ultimately repealed
retrospectively through Finance Act, 2014.

3. Capital Value Tax Capital Value Tax on different transaction such as transfer
of immoveable property, transfer of rights and acquisition of
shares of listed Companies etc.

2 Conceptual Approach to Taxes


Taxation System Chapter-01

INDIRECT TAXES
Following are the indirect Taxes under the Pakistani Taxation System.

SR. Name of prevailing Law Remarks

1. Customs Act, 1969 Goods imported and exported from Pakistan are liable to Customs
duties as prescribed through code or otherwise in Pakistan
Customs Tariff. Customs duties in the form of import duties and
export duties constitute a major part of the total tax receipts. The
rate structure of customs duty is determined by a large number of
socio-economic factors. However, the general scheme envisages
higher rates on luxury items as well as on less essential goods.
The import tariff has been given an industrial bias by keeping the
duties on industrial plants and machinery and raw material lower
than those on consumer goods.

2. Federal Excise Act, 2005 Federal Excise duties (FEDs) are leviable on a limited number of
goods produced or manufactured, and services provided or
rendered in Pakistan. On most of the items FED is charged on the
basis of value or retail price. Some items are, however, chargeable
to duty on the basis of weight or quantity. Classification of goods is
done in accordance with the Harmonized Commodity Description
and Coding system which is being used all over the world. All
exports are liable to 0% FED.

3. Sales Tax Act, 1990 Sales tax is a value added tax system. Being as indirect tax
collectable from whole supply chain i.e. importers, manufacturers,
wholesalers (including dealers and distributors) and retailers with
certain exceptions. Therefore , the sales tax is a multi stage tax
payable at standard rate of 17% u/s 3 of the Sales tax Act, 1990
on:
Goods imported into Pakistan;
All taxable supplies by a registered person in respect of any
taxable activity carried on by him;
VAT is a percentage tax levied on the price each registered person
charges for goods or taxable services rendered by him.
VAT normally utilizes as system of tax credit (being as input tax
adjustment) to place the ultimate and read burden on tax on the
final consumer and to relieve the intermediaries from any tax
burden except the final consumer.
Further there are also the concepts of minimum tax and Final tax
under the sales tax Act on specific persons or class of persons or
sectors as the case may be.

2.2 Tax reliefs in cross border transactions


In cross border Transactions, Pakistan taxation system provides following types of reliefs:
Unilateral Relief
A person resident in Pakistan is entitled to a relief in tax on any income earned abroad, if such income has
already been subjected to tax outside Pakistan. Proportionate relief is allowed on such income at an average
rate of tax in Pakistan or abroad, whichever is lower. It is important to state that foreign source income of non
residents in Pakistan is not taxable in Pakistan under section 11(6) of the Income tax Ordinance, 2001.
Agreement for avoidance of double taxation and fiscal evasion with respect to Taxes
The Government of Pakistan has so far signed agreements to avoid double taxation with more than 50
countries including almost all the developed countries of the world. These agreements lay down the ceilings on
tax rates applicable to different types of income arising in Pakistan. They also lay down some basic principles
of taxation which cannot be modified unilaterally.

Conceptual Approach to Taxes 3


Taxation System Chapter-01

3. BASICS OF TAXATION LAWS


3.1 Basics of tax laws
Adam Smiths in his famous book Wealth of Nations has elaborated following canons of Taxation:
Equality
Tax payments should be proportional to income and applied equally to all concerned areas.
Certainty
Tax liabilities should be clear and certain.
Convenience of payment
Taxes should be collected at a time and in a manner convenient for taxpayer.
Economy of collection
Taxes should not be expensive to collect and should not discourage business.
3.2 Principles for levy of tax
Accordingly, following are broader principles for levy of taxes:
The Benefit Principle
This principle holds the individuals should be taxed in proportion to the benefits they receive from the
governments and that taxes should be paid by those people who receive the direct benefit of the
government programs and projects out of the taxes paid.
The Ability-to-Pay Principle
This principle holds that taxes should relate with the peoples income or the ability to pay, that is, people
with greater income or wealth and can afford to pay more taxes should be taxed at a higher rate than
people with less income or wealth.
The Equal-Distribution Principle
This principle that income, wealth, and transaction should be taxed at a fixed percentage; that is, people
who earn more and buy more should pay more taxes, but will not pay a higher rate of taxes.
3.3 Structure of taxes
A tax is proportional
Meaning that the government takes an amount of money from a person which is indirect proportion to
his income. Rashid income is Rs 100,000 and the employer is deducting 10% of his salary for tax. After
a year his income increases to Rs 150,000 and employer deducts 12% of his salary for tax. The said tax
is proportional.
A tax is regressive
Meaning that the governments takes a larger percentage of a persons income per tax, while he is
receiving a lower income. Rashid s salary Rs 100,000 and employer is deducintg 15% of his salary for
tax which is contrary to our given example in number 1.
A tax is progressive
Meaning that the government takes a larger percentage of his salary for tax due to his high salary.
Rashid has a monthly income of Rs 300,000 and employer deducted 20% of his salary for tax. The tax
amount is proportionately equal to someones status in the society. A rich man should pay more than a
poor man.
3.4 Characteristics of tax laws
Following are major characteristics of a Taxation system:
It is enforced contribution.
Its payment is not voluntary nature, and the imposition is not dependent upon the will of the person
taxed.
It is generally payable in cash.
This means that payment by cheques, promissory notes, or in kind should not be accepted.
It is proportionate in character.

4 Conceptual Approach to Taxes


Taxation System Chapter-01

Payment of taxes should be based on the ability to pay principle; the higher income of the tax payer the
bigger amount of the tax paid.

It is levied (to impose; collect) on income, Person or property.


There are taxes that are imposed or levied on acts, rights or privileges.

It is levied by the state which has jurisdiction over the person or property.
As a general rule, only persons, properties, acts, right or transaction within the jurisdiction of the taxing
state are subject for taxation.
It is levied by the law making body of the state.
This means that a prior law must be enacted first by the Parliament in Pakistan.
It is levied for public purposes.
Taxes or imposed to support the government for implementation of projects and programs.
Fiscal adequacy
Means that the sources of revenue taken as a whole should be sufficient to meet the expanding
expenditures of the government regardless of business, export taxes, trade balances, and problems of
economic adjustment. Revenues should be capable expanding or contracting annually in response to
variations of public expenditures.
Equality or Theoretical Justice.
Means the taxes levied must be based upon the ability of the citizen to pay.
Administrative Feasibility.
This principle connotes that in a successful tax system, such tax should be clear and plain to taxpayers,
capable of enforcement by an adequate and well-trained staff of public office, convenient as to the time
and manner payment, and not unduly burdensome upon on discouraging to business activity.
Consistency or Compatibility with Economic Goals.
This refers to the tax laws that should be consistent with economic goals or programs of the
government. These are the basic services intended for the masses.
3.5 Forms of escape from taxation
Shifting
It is one way of passing the burden of tax from one person to another.
For example: Taxes paid by the manufacturer may be shifted to the consumer by adding the
amount of the tax paid to price of the product.
Kinds of Shifting
Forward shifting occurs when the burden of the tax is transferred from a factor of the production
to the factor of distribution.
Backward shifting occurs when the burden of tax is transferred from the consumer to the
producer or manufacturer.
Onward shifting occurs when tax is shifted to two or more times either forward or backward.
Capitalization
This refers to the reduction in the price of the tax object to the capitalized value of future taxes
which the purchaser expects to be called upon to pay.
For example: A reduction made by the seller on the price of the real estate, in anticipation of the
future tax to be shouldered by the future buyer.
Transformation occurs when the manufacturer or producer upon whom the tax has been
imposed pays the tax and endeavor to recoup (make up for) himself by improving his process
of production
Tax Exemption is the grant of immunity or freedom from a financial charge or obligation or
burden to which others are subjected.
Grounds for tax exemption:

Conceptual Approach to Taxes 5


Taxation System Chapter-01

Contract, wherein the government is the contracting party.


Public policy
Reciprocity

3.6 Strategies of taxation management


Tax practitioners and Taxpayer normally adopts any of the following technique to lessen Tax burden:
Tax Evasion is the practice by the taxpayer whereby revenue results are understated to defeat at or
lessen the amount for tax. This is also known as tax evasion.
Tax avoidance is non-declaration of income and non-payment of due taxes, this is called Tax
avoidance, it is called Tax Avoidance and is an offence.
Tax Planning is the exploitation/use by the taxpayer of legally permissible methods, Tax credits, tax
rebates, tax reductions in order to avoid or reduce tax liability. This is also known as tax minimization.

3.7 Example

Explain which type of Tax strategy is being employed by following persons and what are its legal
consequences:

Tax Law Objective


Mr. Babar has earned Turnover of Rs. 5 (M), however, he kept Tax avoidance, Criminal Act, He
its cash in his bank locker and hide that from Tax authorities. He cannot buy any asset or settle
paid all its related expenses from cash. liabilities unless he declare this
income
Mr. Tanveer earned income of Rs. 6(M), however, he declared Tax evasion, understatement is
only so much of income which is verifiable from the banks i.e. similar to Tax avoidance
4(M), remaining amount he has hidden in a separate bank
account
Mr. Yousaf has earned Rs. 7(M), however, he recorded Tax planning, it is legally
expenses incurred against such receipts, and accordingly, he permissible and appreciable Act.
offered the remaining income of Rs 3(M) for Taxes. He paid
salary to his brother to the extent it remain taxable below the tax
rate of 10%. In such way that he get reduction in tax rate.

4. OBJECTIVES OF TAX LAWS

4.1 Definition of taxation and its revenue objectives


Taxation is defined in many ways, common definition are as under:
It is the process by which the sovereign, through its law making body, raises revenues in order use it for
expenses of government.
It is a means of government in increasing its revenue under the authority of the law, purposely used to
promote welfare and protection of its citizenry.
It is the collection of the share of individual and organizational income by a government under the
authority of the law.

4.2 Non-revenue objectives


Taxes are primary revenue yielding tools of the Government of modern ages. The government levy taxes in
order to achieve following objectives:
For collection of revenue to run and administer the Government;
To use as a tool for implementation of its policies; and
For fair distribution of wealth.
Aside from purely financing government operational expenditures, taxation is also utilized as a tool to carry out
the national objective of social and economic development.

6 Conceptual Approach to Taxes


Taxation System Chapter-01

To strengthen anemic enterprises by granting them tax exemptions or other conditions or incentives for
growth;
To protect local industries against foreign competition by increasing local import taxes;
As a bargaining tool in trade negotiations with other countries;

To counter the effects of inflation or depression;


To reduce inequalities in the distribution of wealth;
To promote science and invention, finance educational activities or maintain and improve the efficiency
of local forces;
To implement laws which eliminate discrimination among various elements in the markets / businesses.

4.3 Example

Explain what are the objectives of following tax laws:

Sr. Tax Law Objective


1. Tax on Salary income (including income from other Revenue Collection
heads of income) under various sections of the
Income Tax Ordinance, 2001
2. Any amount transferred otherwise than banking Documentation of economy
channel will be deemed as income u/s 39
3. Tax on moveable assets of the taxpayers Fair distribution of wealth
4. Higher taxes on import of luxury Goods Reduction in imports of unnecessary
goods and create good balance of Trade
5. Allowability of expenditure of Research & Promotion of Research & developments
developments
6. Zero rating on Exports, reduced rates of taxes on Promotion of Exports
imports
7. Tax credit on donations to approved institutions u/c 61 To promote culture of payment of
of Part 1 of the 2nd Schedule to the Income tax donation to only organised and regulated
Ordinance, 2001 institutions
8. Tax credit on investments in listed Companies shares Promote investments in listed
u/s 61 companies
9. Tax exemptions to software exports u/c 133 of Part 1 Promote software Industry
of the 2nd Schedule to the Income tax Ordinance,
2001

4.4 Taxes as mean for development


Taxes are one of the main mean for development. This is not because that revenue collected by the state is
used on developmental projects rather taxes can be used in many a ways to bring development in the country.
Some examples are as under:
The Government can declare some areas as free zone, industrial zone, economic zone and provide tax
incentives to such areas. These under developed area will become focus of businessman/industrialist.
This will bring employment, opportunities and prosperity in these un-developed areas.
Taxing the rich at higher rate whereas taxing the low income group at low tax rates
Imposition of high custom duty rates on luxury items or kind of items which are also manufactured in
Pakistan. This promotes local manufacturers and industry.

Tax credits on charity/donations to promote welfare activities.


Tax exemptions to charity organization /educational institutions to promote these activities.
Tax incentives for agro based projects to promote agriculture.

Conceptual Approach to Taxes 7


Taxation System Chapter-01

CA MOD C PAST PAPERS THEORECTICAL QUESTIONS


Q.NO.9 Spring 2015 State any five ways through which taxes can be used for development of the country.
Q.NO.10 Spring 2015 Briefly explain any three indirect taxes applicable in Pakistan.
Q.NO.7 Autumn 2014 The primary objective of a taxation system is to collect revenue. You are required to list the other
objectives (non-revenue) which a taxation system can achieve.

8 Conceptual Approach to Taxes


Ethics in Tax Laws Chapter-02

Chapter

2 ETHICS IN TAX LAWS

Sr. No. TOPICS COVERED

1. Ethics

2. Ethics for Legislators

3. Ethics for Administrators

4. Ethics for Practitioners

5. Ethics for taxpayers

(FOR CAF-6, MODULE F & ICMAP STUDENTS)


1. ETHICS
1.1 Ethics meanings and application
The word ethics is derived from the Greek word ethos, which means "character," and from the Latin word
mores, which means "customs." Aristotle was one of the first great philosophers to study ethics. To him, ethics
was more than a moral, religious, or legal concept. He believed that the most important element in ethical
behaviour is knowledge that actions are accomplished for the betterment of the common good. He asked
whether actions performed by individuals or groups are good both for an individual or a group and for society.
To determine what is ethically good for the individual and for society, Aristotle said, it is necessary to possess
three virtues of practical wisdom: temperance, courage, and justice.
2. ETHICS FOR TAX LEGISLATORS
2.1 Ethics for tax legislators
Although we have already discussed the objectives of Taxation however the same are repeated here at the
cost of repetition.
For collection of revenue to run and administer the Government;

To use as a tool for implementation of its policies; and


For fair distribution of wealth.
Non-revenue objectives
Aside from purely financing government operational expenditures, taxation is also utilized as a tool to carry out
the national objective of social and economic development.
To strengthen anemic enterprises by granting them tax exemptions or other conditions or incentives for
growth;
To protect local industries against foreign competition by increasing local import taxes;
As a bargaining tool in trade negotiations with other countries;
To counter the effects of inflation or depression;
To reduce inequalities in the distribution of wealth;
To promote science and invention, finance educational activities or maintain and improve the efficiency
of local forces;
To implement laws which eliminate discrimination among various elements in the markets / businesses.

Conceptual Approach to Taxes 9


Ethics in Tax Laws Chapter-02

In the second Chapter, we discussed the provisions of Constitution of Pakistan which empowers the legislators
to legislate for levy of Taxes on the masses. These powers are not unfettered and it should carry some ethical
and rational basis. This matter is further discussed as under:

2.2 Ethics and canons of taxation


According to Hugh Dalton, "a tax is a compulsory contribution imposed by a public authority, irrespective of the
exact amount of service rendered to the taxpayer in return, and not imposed as penalty for any legal offence."
As per above definition, Taxes are compulsory contribution. Secondly, its compliance is must for a citizen
living in that prescribed Jurisdiction otherwise State has right of enforcement its laws including recovery of tax
through prescribed coercive measures. Now, state right to tax its masses should be based upon some rational
basis. A good tax system is one which is designed on the basis of an appropriate set of principles (rules).
The tax system should strike a balance between the interest of the taxpayer and that of tax authorities. Adam
Smith was the first economist to develop a list of Canons of Taxation. These canons are still regarded as
characteristics or features of a good tax system.

2.3 Canons of taxation


What are Canons of Taxation? Canons of Taxation are the main basic principles (i.e. rules) set to build a 'Good
Tax System'. The canons of taxation are as under:
(i) Canon of Equity: The word equality here does not mean that everyone should pay the exact, equal
amount of tax. What equality really means here is that the rich people should pay more taxes and the
poor pay less. This is because the amount of tax should be in proportion to the abilities of the tax payer.
It is one of the fundamental concepts to bring social equality in the country.
The canon of equality states that there should be justice, in the form of equality, when it comes to
paying taxes. Not only does it bring social justice, it is also one of the primary means for reaching the
equal distribution of wealth in an economy.
(ii) Canon of Certainty: The tax payers should be well-aware of the purpose, amount and manner of the
tax payment. Everything should be made clear, simple and absolutely certain for the benefit of the
taxpayer. The canon of certainty is considered a very important guidance rule when it comes to
formulating the tax laws and procedures in a country. The canon of certainty ensures that the taxpayer
should have full knowledge about his tax payment, which includes the amount to be paid, the mode it
should be paid in and the due-date. It is believed that if the canon of certainty is not present, it leads to
tax evasion.
(iii) Canon of Convenience: Canon of convenience can be understood as an extension of canon of
certainty. Where canon of certainty states that the taxpayer should be well-aware of the amount,
manner and mode of paying taxes, the canon of convenience states that all this should easy,
convenient and taxpayer-friendly. The time and manner of payment must be convenient for the tax
payer so that he is able to pay his taxes in due time. If the time and manner of the payment is not
convenient, then it may lead to tax evasion and corruption.
(iv) Canon of Economy: The whole purpose of collecting taxes is to generate revenue for the company.
This revenue, in turn, is spent on public welfare projects. The canon of economy keeping in view the
above-mentioned purpose states that the cost of collecting taxes should be as minimum as possible.
There should not be any leakage in the way. In this way, a large amount of the collections will go
directly to the treasury, and therefore, will be spent in the government projects for the welfare of the
economy, country and the people. On the other hand, if the canon of economy isnt applied and the
overall cost of collecting taxes is unreasonably high, the collected amount will not be sufficient in the
end.
(v) Canon of Productivity: By virtue of the canon of productivity, it is better to have fewer taxes with large
revenues, rather than more taxes with lesser amounts of revenue. It is always considered better to
impose the only taxes that are able to produce larger returns. More taxes tend to create panic, chaos
and confusion among the taxpayers and it is also against the canon of certainty and convenience to
some extent.
(vi) Canon of Elasticity: An ideal system of taxation should consist of those types of taxes that can easily
be adjusted. Taxes, which can be increased or decreased, according to the demand of the revenue, are
considered ideal for the system. An example of such a tax can be the income tax, which is considered
very much ideal in accordance with the canon of elasticity. This example can also be taken in
accordance with the canon of equality. Flexible taxes are more suited for bringing social equality and
achieving equal distribution of wealth. Since they are elastic and easily adjustable, many government
objectives can be achieved through them.

10 Conceptual Approach to Taxes


Ethics in Tax Laws Chapter-02

(vii) Canon of Simplicity: The system of taxation should be made as simple as possible. The entire
process should be simple, non-technical and straightforward. Along with the canon of certainty, where
the amount, time duration and manner of payment is made certain, the canon of simplicity avoids cases
of corruption and tax evasion if the entire method is made simple and easy.
(viii) Canon of Diversity: Canon of diversity refers to diversifying the tax sources in order to be more
prudent and flexible. Being heavily dependent on a single tax source can be detrimental for the
economy. Canon of diversity states that it is better to collect taxes from multiple sources rather than
concentrating on a single tax source. Otherwise, the economy is more likely to be confined, and hence,
its growth will be limited as well.
(ix) Canon of Flexibility: Canon of flexibility means that the entire tax system should be flexible enough
that the taxes can easily be increased or lowered, in accordance with the government needs. This
flexibility ensures that whenever the government requires additional revenue, it can be generated
without much hassle. Similarly, when the economy isnt booming, lowering taxes shouldnt be a problem
either.
Conclusion:
So these are the nine (9) canons of taxation that are used as the fundamentals for any taxation system and
study about taxation principles. Although Mr. Adam Smith originally presented the first four canons. Later, in
order to better suit to modern economies and for the sake of evolution as well, more canons were introduced.
2.4 Responsibilities of the tax legislators
The tax structure is a part of economic organisation of a society and therefore fit in its overall economic
environment. No tax system that does not satisfy above canons of Taxation can be termed a good one.
Moreover, the state should pursue that the primary aim of the tax should be to raise revenue for public
services, however, People should be asked to pay taxes according to their ability to pay and assessment of
their taxable capacity should be made primarily on the basis of income and property. May it be added here that
tax should not be discriminatory in any aspect between individuals and also between various groups.
3. ETHICS FOR TAX ADMINISTRATORS
3.1 Ethics for tax administrators introduction
Federal Board of Revenue is empowered under the law to monitor, assess, levy, collect taxes according to the
tax legislations. There are a number of occasions whereby they possess any of the following descretionary
powers.
Asses taxes;
Collect Revenue;
Seize Property;
Attatch Bank A/cs;
Commence legal (criminal/civil) proceedings against the taxpayer
Such descretionary powers may be misused and can become abusive powers as exercise of that power can
result in the following against the taxpayer:
Loss of property and income;
Imprisonment
So, these power can result in the loss of some of the fundamental human rights of the taxpayer. Ethics tend to
bring these powers within the principles of good and morality.

Example:
Mr. Asif is running a textile unit and income tax amounting to Rs. 15(M) is assessed against him. His bank
accounts balance is Rs. 10M, however, he has to fulfil his exports orders. In case he fails to fulfil his orders,
he will loose his clients and that orders. Moreover, he has to face SBP penal action for non export.
Considering his present critical financial position, Mr. Asif believes that tax recovery proceedings by recovery
from bank account (Attachment of bank account) will entail to an irreparable loss to his organisation. So he
requested to Commissioner Inland Revenue for allowing him to pay the due tax in instalments.
Now Commissioner Inland Revenue has power to allow him instalments (but to be paid with default
surcharge) or recover this tax directly from his bank account, unless stay order provided by the taxpayer from
the Commissioner Inland Revenue (Appeals) or honourable High Court. Justice and equity demands that his
request should be entertained; if not so then stay order as discussed shall be in the field. However the
allowablility of instalments will result into for the continuation and prosperity of business that eventually result
in payment of better taxes in future whereas recovery of tax will jeopardise his business operation.

Conceptual Approach to Taxes 11


Ethics in Tax Laws Chapter-02

Example:
Income Tax Ordinance, sales Tax law, Federal excise law empower tax authorities to select cases for Audit
under various sections of the respective laws. This power can be misused by selecting some cases while
leaving many unaudited even in the presence of power of amendment in assessment under section 122 of
the Income Tax Ordinance, 2001. Thus, despite the law provides unfettered powers however, such powers
should be exercised on some ethical and rational basis.

3.2 Pillars of tax administration


In order to safeguard the interest of taxpayers and avoid abuse of powers by he Tax administration. Following
four pillars of Tax administration are defined:
1. Fairness
Strive to be impartial, fair, neutral and consistent in administering the law without regard to race, social
or economic circumstance;

2. Transparency
All Proceedings must be transparent and must be seen as transparent.
3. Equity
Best Tax administration is not that which collects most revenue rather it depends how this revenue
generation is accomplished. Whether all stakeholders are taxed fairly or tax is collected from poor
/salaried class after failing to collect taxes from entrepreneurs/businesses. Thus, equity demands that
tax administrator should not achieve its objectives in an irrational manner.
4. Accountability
There must be a strong system of accountability for wrong doers which should curb corruption, nepotism
and maladministration.
Under the four pillars, some of the ethical issues facing Tax administration:
1. Acceptance of gifts;
2. Conflict of Interest;
3. Selective application of the law/ or inconsistency in applying the law;
4. Political influence;
5. Confidentiality/secrecy;
6. Discretion;
7. Corruption;
8. Lack of Autonomy
In order to avoid pitfalls of the abusive use of discretion, seven principles for structuring discretion are defined
which are as under:
Open plans,
Open policy statements,
Open rules,
Open findings,
Open reasons,
Open precedents and
Fair informal procedure
3.3 Responsibilities of the tax implementing authorities
The tax administrator shall perform the following responsibilities:-
1. implement the tax administration reforms;
2. promote voluntary tax compliance and to make the tax administration a service oriented organization
and to implement comprehensive policies and programs for the education and facilitation of taxpayers,
stakeholders and employees, etc.;

12 Conceptual Approach to Taxes


Ethics in Tax Laws Chapter-02

3. adopt modern effective tax administration methods, information technology systems and policies in
order to consolidate assessments, improve processes, organize registration of tax payers, widen the tax
base, and make departmental remedies more efficient including enforcement of, or reduction or
remission in, duty, penalty or tax, in accordance with the relevant law for the time being in force;
4. improve the productivity through a comprehensive and effective human resource strategy;
5. identify and select through Internal Job Posting process the employees for designated jobs;
6. grant additional allowances or any other incentives and rewards to the employees and members of the
Board;
7. take appropriate measures including internal controls to combat corruption within the organizations
under the Board and provide checks to ensure the integrity of employees that is verified periodically
through applicable procedure which shall be made one of the criterion for promotion and incentives;
8. re-designate existing posts within its jurisdiction, prepare job description of any post and create posts as
per rules;
9. direct or advise, where necessary, investigation or inquiry into suspected duty tax evasion, tax and
commercial fraud, money-laundering, financial crimes cases and to coordinate with the relevant law
enforcement agencies;
10. introduce and maintain a system of accountability of performance, competence and conduct of the
employees.
11. implement international obligations pursuant to a treaty, resolution or any international commitment;
12. create a surplus pool of employees as and when required;
13. make regulations, policies, programs, strategies in order to carry out the purposes of this Act;
14. regulate and enter into any agreement, contract, understanding, with any international organization or
institution or donor agency or counterpart entity with approval of the FG;
15. set up mechanism and processes that facilitate removal of grievances and complaints of the tax payers;
16. enable electronic communication in respect of all taxation matters such as e-filing, e-payments, e-
notice, e-notification, digital imaging, protocols or agreements as may be prescribed; and
4. ETHICS FOR TAX PRACTITIONERS

4.1 Ethics for tax practitioners


There are five fundamental principles of Ethics for tax practitioners are set out below:
(i) Integrity. The principle of integrity imposes an obligation on all tax practitioners to be straightforward
and honest in all professional and business relationships. Integrity also implies fair dealing and
truthfulness. Tax practitioners should not be associated with the reports, returns, communications or
other information where they believe that the information;
(a) Contains a materially false or misleading statement;
(b) Contains statements or information furnished recklessly; or
(c) Omits or obscures information required to be included where such omission or obscurity would
be misleading.
(ii) Objectivity. The principle of objectivity imposes an obligation on all tax practitioners not to compromise
their professional or business judgment because of bias, conflict or interest or the undue influence of
others. A Tax practitioner may be exposed to situations that may impair objectivity. It is impracticable to
define and prescribe all such situations. Relationships that bias or unduly influence the professional
judgment of the tax practitioners should be avoided.
(iii) Professional competence and due care. The principle of professional competence and due care
imposes the following obligations on tax practitioners;
(a) To maintain professional knowledge and skill at such a level that clients or employer receives a
competent professional service; and
(b) To act diligently and in accordance with applicable technical and professional standards, when
providing professional services.
Competent professional service requires the exercise of sound judgment in applying professional
knowledge and skill in the performance of such service. Professional competence may be divided into
two separate phases;

Conceptual Approach to Taxes 13


Ethics in Tax Laws Chapter-02

(a) Attainment of professional competence; and


(b) Maintenance of professional competence.
(iv) Confidentiality. The principle of confidentiality imposes an obligation on all tax practitioners to refrain
from:
(a) Disclosing to third parties the clients business confidential information acquired as a result of
professional and business relationships without proper and specific authority or unless there is a
legal or professional right or duty to disclose, and
(b) Using confidential information acquired as a result of professional and business relationships
should to their personal advantage or the advantage of third parties.
A tax practitioner should maintain confidentiality even in a social environment. The tax practitioner
should be alert to the possibility of inadvertent disclosure, particularly in circumstances involving long
association with a business associate or a close or immediate family member.
(v) Professional behavior. The principle of professional behavior imposes an obligation on all tax
practitioners to comply with relevant laws and regulations and avoid any action that may bring discredit
to the profession. This includes actions which a reasonable and informed third party, having knowledge
of all relevant information, would conclude negativity affects the good reputation of the profession.
5. ETHICS FOR TAX PAYERS

5.1 Three approaches of tax compliance


There are three approaches to ethics for tax compliance which are as under:
Utilitarianism is a theory in normative ethics holding that the proper course of action is the one that
maximizes utility, usually defined as maximizing total benefit and reducing suffering or the negatives. In
simple words, it is the way one helps himself in the art of decision-making.
Deontology, It is described as "duty" or "obligation" or "rule"-based ethics, because rules "bind you to
your duty."[
Virtue ethics, focus less on lying in any particular instance and instead consider what a decision to tell
a lie or not tell a lie said about one's character and moral behavior. As such, the morality of lying would
be determined on a case-by-case basis, which would be based on factors such as personal benefit,
group benefit, and intentions.

5.2 Ethics and morality for taxation compliance


For Taxpayers following utilitarian approach, the most important economic goals are to ensure that goods and
services are available to allow everyone to have a decent life, and to ensure that these resources are
distributed widely enough for all or most people to enjoy them. So these taxpayers are ready to pay taxes even
on highest rate. Moreover, their compliance level will be high as the need availability of resources for vast
majority of masses and country.
On the other hand, taxpayers liking the deontologist ethical approach lay down absolute duties. Such duty
includes respect to other peoples property rights. This could be interpreted to mean that there should be no
tax at all, because tax is the forcible transfer of property away from taxpayers. On the other hand, the duty to
respect property rights could be used to argue that any social resources one used should be paid for, even if
one did not ask for those resources to be provided. Thus in order not to be a thief, anyone who uses a public
hospital, or even a public road, should make sure that he or she pays tax to cover their use. So in deontologist
approach, taxes are paid as duty to pay off against the facilities used by the taxpayers.
Virtue ethics can be a bit more helpful on the question of the justice of taxation. One should use ones talents
to the full. Financial incentives can encourage people to use their talents, but very high taxation dampens down
those incentives by reducing take-home pay. Another virtue is charity, either in cash or in time. The more take-
home pay people have, the more likely it is that they will feel able to afford charitable donations; and the higher
peoples pay rates, the easier it will be for them to take time away from paid work to perform charity work or
other forms of civic service, as school trustee or Mutwali of Masjid for example. A third virtue is independence.
It is good to earn what one needs rather than to depend on subsidies from others. Lower rates of taxation
make independence more easily achievable.
Tax can be used for all sorts of purposes, and it is often clear what ethicists of any particular kind would say
about these purposes. We can start with the provision of law and order and the more extensive public services
such as healthcare and education. Utilitarians will approve of taxation for these things because they allow
more goods and services to be produced, and they also allow more non-materialistic desires to be satisfied.
Virtue ethicists will approve because these services enhance peoples opportunities to use their talents and to
lead flourishing lives.

14 Conceptual Approach to Taxes


Ethics in Tax Laws Chapter-02

When we turn to aid to the poor, utilitarians will approve because transferring resources from rich to poor
increases the happiness of the poor more than it reduces the happiness of the rich. Virtue ethicists will approve
because with redistribution the poor can be helped to flourish and develop virtues, and because looking after
the less fortunate is itself a virtue (although voluntary charity may be a greater virtue than forced payment).
And deontologists can recognize a duty to care for the poor.
A taxation addressing the needs of all these ethical thoughts can get better compliance. Morality for Taxpayers
to pay taxes is very justified as state is responsible to provide infrastructure for a decent life. Moreover, State is
responsible for providing facilities to the masses then it is the duty of the masses to pay taxes for it. State also
provide level playing field to all the concerned so that talent can be explored at full. So it is necessary that
taxes should be paid to provide facilities, to control law & order situation, infrastructural development etc.

5.3 The conduct of taxpayers


Most taxpayers pay their taxes, without fuss. But not all taxpayers act in this way. So lastly lets look at whether
three other forms of behavior can be ethically acceptable: tax evasion, tax avoidance and tax planning. We
already discussed these strategies in the following manner:
Tax evasion is the illegal evasion of taxes by individuals, corporations and trusts. Tax evasion often
entails taxpayers deliberately misrepresenting the true state of their affairs to the tax authorities to
reduce their tax liability and includes dishonest tax reporting, such as declaring less income, profits or
gains than the amounts actually earned, or overstating deductions. Tax evasion is an activity commonly
associated with the informal economy.
Tax avoidance is the legal use of tax laws to reduce one's tax burden. Both tax evasion and avoidance
can be viewed as forms of tax noncompliance, as they describe a range of activities that intend to
subvert a state's tax system, although such classification of tax avoidance is not indisputable, given that
avoidance is lawful, within self-creating systems.
Tax Planning is the exploitation/use by the taxpayer of legally permissible methods, tax credits, tax
rebates and tax reductions in order to avoid or reduce tax liability. This is also known as tax
minimization.
A utilitarian, concerned with aggregate welfare, might be quite relaxed about tax planning. After all, when tax is
avoided, wealth is not destroyed: it is merely kept in the private sector instead of being transferred to the public
sector. The main utilitarian concern would probably be that it would result in an unintended distribution of the
tax burden, as some of the burden would be shifted from the rich onto people on modest incomes who cannot
afford clever tax lawyers. That would reduce their satisfaction more than it would increase the satisfaction of
the better-off people who have reduced their tax burdens. But that loss to the poor might not happen.
A virtue ethicist would unlike tax planning. It is, after all, hardly virtuous to exploit rules knowing that one is
exploiting them in unintended ways to redistribute the disadvantage away from oneself. A deontologist would
not positively favour tax planning, but might not condemn it either. Deontologists can easily argue for a duty to
obey the law: yet obeying the law is something the tax planner takes care to do, in his own special way.

Conceptual Approach to Taxes 15


Ethics in Tax Laws Chapter-02

CA MOD C PAST PAPERS THEORECTICAL QUESTIONS


Q.NO.8 Autumn 2014 Briefly explain the ethical responsibilities of the tax implementing authorities.

16 Conceptual Approach to Taxes


Constitutional Provisions Chapter-03

Chapter

3 CONSTITUTIONAL PROVISIONS

Under Article of TOPICS COVERED


Constitution

78 to 88 Federal Financial Procedures


118 to 127 Provincial Financial Procedures
161 to 165A Distribution of Revenues between Federation and Provinces

Federal Legislative List

(FOR CAF-6, MODULE F & ICMAP STUDENTS)


1. FEDERAL FINANCIAL PROCEDURES
1.1 Introduction
Constitution of Pakistan is prime source for all legislations in Pakistan. It distributes powers among Federation
and Provinces. It provides procedures for levy and collection of taxes along-with procedures for use and
exploitations of funds received from taxes or by the Federation from any other source. This chapter has been
divided into the following three main areas:
Federal Financial Procedures
Provincial Financial Procedures
Distribution of Revenues between Federation & Provinces
The detailed explanation of the above areas is as under.
1.2 Federal consolidated fund and public account [Under Article 78]
All revenues received by the Federal Government, all loans raised by that Government and all moneys
received by it in repayment of any loan, shall form part of a consolidated fund, to be known as the Federal
Consolidated Fund.
All other moneys:
received by or on behalf of the Federal Government; or
received by or deposited with the Supreme Court or any other court established under the authority of
the Federation;
Shall be credited to the Public Account of the Federation.
1.3 Custody , etc., of federal consolidated fund and public account [Under Article 79]
The custody of the Federal Consolidated Fund, the payment or moneys into that Fund, the withdrawal of
moneys there from, the custody of other moneys received by or on behalf of the Federal Government, their
payment into, and withdrawal from, the Public Account of the Federation, and all matters connected with or
ancillary to the matters aforesaid shall be regulated by Act of Majlis-e-Shoora (Parliament) or, until provision in
that behalf is so made, by rules made by the President.
1.4 Annual budget statement [Under Article 80]
The Federal Government shall, in respect of every financial year, cause to be laid before the National
Assembly a statement of the estimated receipts and expenditure of the Federal Government for that year, in
this Part, referred to as the Annual Budget Statement.

Conceptual Approach to Taxes 17


Constitutional Provisions Chapter-03

The Annual Budget Statement shall show separately:


the sums required to meet expenditure described by the Constitution as expenditure charged upon the
Federal Consolidated Fund; and
The sums required to meet other expenditure proposed to be made from the Federal Consolidated
Fund; and
shall distinguish expenditure on revenue account from other expenditure.
1.5 Expenditure charged upon federal consolidated fund [Under Article 81]
The following expenditure shall be expenditure charged upon the Federal Consolidated Fund:-
the remuneration payable to the President and other expenditure relating to his office, and the
remuneration payable to-
the Judges of the Supreme Court and the Islamabad High Court;
the Chief Election Commissioner;
the Chairman and the Deputy Chairman;
the Speaker and the Deputy Speaker of the National Assembly;
the Auditor-General;
The administrative expenses, including the remuneration payable to officers and servants, of the
Supreme Court, the Islamabad High Court, the department of the Auditor-General, the Office of the
Chief Election Commissioner and of the Election Commission and the Secretariats of the Senate and
the National Assembly;
All debt charges for which the Federal Government is liable, including interest, sinking fund charges, the
repayment or amortisation of capital, and other expenditure in connection with the raising of loans, and
the service and redemption of debt on the security of the Federal Consolidated Fund;
Any sums required to satisfy any judgment, decree or award against Pakistan by any court or tribunal;
and
Any other sums declared by the Constitution or by Act of Majlis-e-Shoora (Parliament) to be so charged.
1.6 Procedure relating to annual budget statement [Under Article 82]
So much of the Annual Budget Statement as relates to expenditure charged upon the Federal
Consolidated Fund may be discussed in, but shall not be submitted to the vote of, the National
Assembly.
So much of the Annual Budget Statement as relates to other expenditure shall be submitted to the
National Assembly in the form of demands for grants, and the Assembly shall have power to assent to,
or to refuse to assent to, any demand, or to assent to any demand subject to a reduction of the amount
specified therein;
Provided that, for a period of ten years from the commencing day or the holding of the second general election
to the National Assembly, whichever occurs later, a demand shall be deemed to have been assented to
without any reduction of the amount specified therein, unless, by the votes of a majority of the total
membership of the Assembly, it is refused or assented to subject to a reduction of the amount specified
therein.
No demand for a grant shall be made except on the recommendation of the Federal Government.
1.7 Authentication of schedule of authorised expenditure [Under Article 83]
The Prime Minister shall authenticate by his signature a schedule specifying:
the grants made or deemed to have been made by the National Assembly under Article 82, and
the several sums required to meet the expenditure charged upon the Federal Consolidated Fund but not
exceeding, in the case of any sum, the sum shown in the statement previously laid before the National
Assembly.
The schedule so authenticated shall be laid before the National Assembly, but shall not be open to discussion
or vote thereon.
Subject to the Constitution, no expenditure from the Federal Consolidated Fund shall be deemed to be duly
authorised unless it is specified in the schedule so authenticated and such schedule is laid before the National
Assembly as required above.

18 Conceptual Approach to Taxes


Constitutional Provisions Chapter-03

1.8 Supplementary and excess grants [Under Article 84]


If in respect of any financial year it is found:
that the amount authorized to be expended for a particular service for the current financial year is
insufficient, or that a need has arisen for expenditure upon some new service not included in the Annual
Budget Statement for that year; or
that any money has been spent on any service during a financial year in excess of the amount granted
for that service for that year;
the Federal Government shall have power to authorize expenditure from the Federal Consolidated
Fund, whether the expenditure is charged by the Constitution upon that Fund or not, and shall cause to
be laid before the National Assembly a Supplementary Budget Statement or, as the case may be, an
Excess Budget Statement, setting out the amount of that expenditure, and the provisions of Articles 80
to 83 shall apply to those statements as they apply to the Annual Budget Statement.
1.9 Votes on account [Under Article 85]
Notwithstanding anything contained in the foregoing provisions relating to financial matters, the National
Assembly shall have power to make any grant in advance in respect of the estimated expenditure for a part of
any financial year, not exceeding four months, pending completion of the procedure prescribed in Article 82 for
the voting of such grant and the authentication of the schedule of authorized expenditure in accordance with
the provisions of Article 83 in relation to the expenditure.
1.10 Power to authorise expenditure when assembly stands dissolved [Under Article 86]

Notwithstanding anything contained in the foregoing provisions relating to financial matters, at any time when
the National Assembly stands dissolved, the Federal Government may authorize expenditure from the Federal
Consolidated Fund in respect of the estimated expenditure for a period not exceeding four months in any
financial year, pending completion of the procedure prescribed in Article 82 for the voting of grants and the
authentication of the schedule of authorized expenditure in accordance with the provisions of Article 83 in
relation to the expenditure.
1.11 Secretariats of Majlis-e-Shoora (Parliament) [Under Article 87]
Each House shall have a separate Secretariat: Provided that nothing in this clause shall be construed as
preventing the creation of posts common to both Houses.
Majlis-e-Shoora (Parliament) may by law regulate the recruitment and the conditions of service of persons
appointed to the Secretarial staff of either House.
Until provision is made by Majlis-e-Shoora (Parliament) under clause (2), the Speaker or, as the case may be,
the Chairman may, with the approval of the President, make rules regulating the recruitment and the conditions
of service, of persons appointed to the secretarial staff of the National Assembly or the Senate.
1.12 Finance committees [Under Article 88]
The expenditure of the National Assembly and the Senate within authorised appropriations shall be controlled
by the National Assembly or, as the case may be, the Senate acting on the advice of its Finance Committee.
The Finance Committee shall consist of the Speaker or, as the case may be, the Chairman, the Minister of
Finance and such other members as may be elected thereto by the National Assembly or, as the case may be,
the Senate.
The Finance Committee may make rules for regulating its procedure.
2 PROVINCIAL FINANCIAL PROCEDURES
2.1 Introduction
Provincial financial Procedures are almost the same as Federal Financial Procedures; however, these are
discussed in detail as under:
2.2 Provincial consolidated fund and public account [Under Article 118]
All revenues received by the Provincial Government, all loans raised by that Government, and all revenues
received by the Provincial Government, all loans raised by that Government, and all moneys received by it in
repayment of any loan, shall form part of a consolidated fund, to be known as the Provincial Consolidated
Fund.
All other moneys:
received by or on behalf of the Provincial Government; or

Conceptual Approach to Taxes 19


Constitutional Provisions Chapter-03

Received by or deposited with the High Court or any other court established under the authority of the
Province;
shall be credited to the Public Account of the Province.
2.3 Custody, etc., of provincial consolidated fund and public account [Under Article 119]
The custody of the Provincial Consolidated Fund, the payment of moneys into that Fund, the withdrawal of
moneys there from, the custody of other moneys received by or on behalf of the Provincial Government, their
payment into, and withdrawal from, the Public Account of the Province, and all matters connected with or
ancillary to the matters aforesaid, shall be regulated by Act of the Provincial Assembly or, until provision in that
behalf is so made, by rules made by the Governor
2.4 Annual budget statement [Under Article 120]
The Provincial Government shall, in respect of every financial year, cause to be laid before the Provincial
Assembly a statement of the estimated receipts and expenditure of the Provincial Government for that year, in
this Chapter referred to as the Annual Budget Statement.
The Annual Budget Statement shall show separately:
The sums required to meet expenditure described by the Constitution as expenditure charged upon the
Provincial Consolidated Fund; and
The sums required to meet other expenditure proposed to be made from the Provincial Consolidated Fund;
and shall distinguish expenditure on revenue account from other expenditure.
2.5 Expenditure charged upon provincial consolidated fund [Under Article 121]
The following expenditure shall be expenditure charged upon the Provincial Consolidated Fund:
The remuneration payable to the Governor and other expenditure relating to his office, and the remuneration
payable to:
the Judges of the High Court; and
the Speaker and Deputy Speaker of the Provincial Assembly;
the administrative expenses, including the remuneration payable to officers and servants, of the High
Court and the Secretariat of the Provincial Assembly;
all debt charges for which the Provincial Government is liable, including interest, sinking fund charges,
the repayment or amortization of capital, and other expenditure in connection with the raising of loans,
and the service and redemption of debt on the security of the Provincial Consolidation Fund;
any sums required to satisfy any judgment, decree or award against the Province by any Court or
tribunal; and
Any other sums declared by the Constitution or by Act of the Provincial Assembly to be so charged.
2.6 Procedure relating to annual budget statement [Under Article 122]
So much of the Annual Budget Statement as relates to expenditure charged upon the Provincial Consolidated
Fund may be discussed in, but shall not be submitted to the vote of, the Provincial Assembly.
So much of the Annual Budget Statement as relates to other expenditure shall be submitted to the Provincial
Assembly in the form of demands for grants, and that Assembly shall have power to assent to, or to refuse to
assent to, any demand, or to assent to any demand subject to a reduction of the amount specified therein:
No demand for a grant shall be made except on the recommendation of the Provincial Government.
2.7 Authentication of schedule of authorised expenditure [Under Article 123]
The Chief Minister shall authenticate by his signature a schedule specifying:
the grants made or deemed to have been made by the Provincial Assembly under Article 122, and
The several sums required to meet the expenditure charged upon the Provincial Consolidated Fund but
not exceeding, in the case of any sum, the sum shown in the statement previously laid before the
Assembly.
The schedule so authenticated shall be laid before the Provincial Assembly, but shall not be open to
discussion or vote thereon.
Subject to the Constitution, no expenditure from the Provincial Consolidated Fund shall be deemed to be duly
authorized unless it is specified in the schedule so authenticated and such schedule is laid before the
Provincial Assembly as required above.

20 Conceptual Approach to Taxes


Constitutional Provisions Chapter-03

2.8 Supplementary and excess grants [Under Article 124]


If in respect of any financial year it is found:
that the amount authorized to be expended for a particular service for the current financial year is
insufficient, or that a need has arisen for expenditure upon some new service not included in the Annual
Budget Statement for that year; or
that any money has been spent on any service during a financial year in excess of the amount granted
for that service for that year;
the Provincial Government shall have power to authorize expenditure from the Provincial Consolidated Fund,
whether the expenditure is charged by the Constitution upon that Fund or not, and shall cause to be laid before
the Provincial Assembly a Supplementary Budget Statement or, as the case may be, an Excess Budget
Statement, setting out the amount of that expenditure, and the provisions of Article 120 to 123 shall apply to
those statements as they apply to the Annual Budget Statement.
2.9 Votes on account [Under Article 125]
Notwithstanding anything contained in the foregoing provisions relating to financial matters, the Provincial
Assembly shall have power to make any grant in advance in respect of the estimated expenditure for a part of
any financial year, not exceeding three months, pending completion of the procedure prescribed in Article 122
for the voting of such grant and the authentication of the schedule of expenditure in accordance with the
provisions of Article 123 in relation to the expenditure.
2.10 Power to authorise expenditure when assembly stands dissolved [Under Article 126]
Notwithstanding anything contained in the foregoing provisions relating to financial matters, at any time when
the Provincial Assembly stands dissolved, the Provincial Government may authorize expenditure from the
Provincial Consolidated Fund in respect of the estimated expenditure for a period not exceeding four months in
any financial year, pending completion of the procedure prescribed in Article 122 for the voting of grants and
the authentication of the schedule of authorized expenditure in accordance with the provisions of Article 123 in
relation to the expenditure.
2.11 Provisions relating to National Assembly, etc., to apply to Provincial Assembly, etc. [Under Article
127]
Subject to the Constitution, the provisions of clauses (2) to (8) of Article 53, clauses (2) and (3) of Article 54,
Article 55, Articles 63 to 67, Article 69, Article 77, Article 87 and Article 88 shall apply to and in relation to a
Provincial Assembly or a committee or members thereof or the Provincial Government, but so that:
Any reference in those provisions to Majlis-e-Shoora (Parliament), a House or the National Assembly
shall be read as a reference to the Provincial Assembly;
Any reference in those provisions to the President shall be read as a reference to the Governor of the
Province;
any reference in those provisions to the Federal Government shall be, read as a reference to the
Provincial Government;
any reference in those provisions to the Prime Minister shall be read as a reference to the Chief
Minister.
any reference in those provisions to a Federal Minister shall be read as a reference to a Provincial
Minister.
any reference in those provisions to the National Assembly of Pakistan shall be read as a reference to
the Provincial Assembly in existence immediately before the commencing day.
3. DISTRIBUTION OF REVENUES BETWEEN FEDERATION AND PROVINCES
3.1 Introduction
It is essential to know who authorises which revenues. Federation can only tax to the extent constitution
authorise to legislate for collection of revenues. Similarly, Provinces can only legislate for levy of taxes to the
extent it is clarified in the Constitution of Pakistan. This part of this chapter describes the mechanism for
determination of distribution of revenue among Federation and Provinces.
3.2 National Finance Commission [Under Article 160]
Within six months of the commencing day and thereafter at intervals not exceeding five years, the President
shall constitute a National Finance Commission consisting of the Minister of Finance of the Federal
Government, the Ministers of Finance of the Provincial Governments, and such other persons as may be
appointed by the President after consultation with the Governors of the Provinces

Conceptual Approach to Taxes 21


Constitutional Provisions Chapter-03

It shall be the duty of the National Finance Commission to make recommendations to the President as to:
the distribution between the Federation and the Provinces of the net proceeds of the taxes mentioned in
clause (3);
the making of grants-in-aid by the Federal Government to the Provincial Governments;
the exercise by the Federal Government and the Provincial Governments of the borrowing powers
conferred by the Constitution; and
Any other matter relating to finance referred to the Commission by the President.
The taxes referred above are the following taxes raised under the authority of Majlis-e-Shoora (Parliament),
namely:
taxes on income, including corporation tax, but not including taxes on income consisting of remuneration
paid out of the Federal Consolidated Fund;
taxes on the sales and purchases of goods imported, exported, produced, manufactured or consumed;
export duties on cotton, and such other export duties as may be specified by the President;
export duties on cotton, and such other export duties as may be specified by the President;
such duties of exercise as may be specified by the President; and
Such other taxes as may be specified by the President.
The share of the Provinces, in each Award of National Finance Commission shall not be less than the share
given to the Provinces in the previous Award.
The Federal Finance Minister and Provincial Finance Ministers shall monitor the implementation of the Award
biannually and lay their reports before both Houses of Majlis-e-Shoora (Parliament) and the Provincial
Assemblies.
As soon as may be after receiving the recommendation, of the National Finance Commission, the President
shall, by Order, specify, in accordance with the recommendations of the Commission under paragraph two (2)
above, the share of the net proceeds of the taxes mentioned in above which is to be allocated to each
Province, and that share shall be paid to the Government of the Province concerned, and, notwithstanding the
provision of Article 78 shall not form part of the Federal Consolidated Fund.
The recommendations of the National Finance Commission, together with an explanatory memorandum as to
the action taken thereon, shall be laid before both Houses and the Provincial Assemblies.
At any time before an Order as above is made, the President may, by Order, make such amendments or
modifications in the law relating to the distribution of revenues between the Federal Government and the
Provincial Governments as he may deem necessary or expedient.
The President may, by Order, make grants-in-aid of the revenues of the Provinces in need of assistance and
such grants shall be charged upon the Federal Consolidated Fund.
3.3 Natural gas and hydro-electric power [Under Article 161]
Notwithstanding the provisions of Article 78:
the net proceeds of the Federal duty of excise on natural gas levied at well-head and collected by the
Federal Government and of the royalty collected by the Federal Government, shall not form part of the
Federal Consolidated Fund and shall be paid to the Province in which the well-head of natural gas is
situated.
the net proceeds of the Federal duty of excise on oil levied at well-head and collected by the Federal
Government, shall not form part of the Federal Consolidated Fund and shall be paid to the Province in
which the well-head of oil is situated.
The net profits earned by the Federal Government, or any undertaking established or administered by the
Federal Government from the bulk generation of power at a hydro-electric station shall be paid to the Province
in which the hydro-electric station is situated.
Explanation: for the purposes of this clause "net profits" shall be computed by deducting from the revenues
accruing from the bulk supply of power from the bus-bars of a hydro-electric station at a rate to be determined
by the Council of Common Interests, the operating expenses of the station, which shall include any sums
payable as taxes, duties, interest or return on investment, and depreciations and element of obsolescence,
and over-heads, and provision for reserves.

22 Conceptual Approach to Taxes


Constitutional Provisions Chapter-03

3.4 Prior sanction of President required to Bills affecting taxation in which provinces are interested
[Under Article 162]
No Bill or amendment which imposes or varies a tax or duty the whole or part of the net proceeds whereof is
assigned to any province, or which varies the meaning of the expression "agricultural income" as defined for
the purposes of the enactments relating to income-tax, or which affects the principles on which under any of
the foregoing provisions of this Chapter moneys are or may be distributable to provinces, shall be introduced
or moved in the National Assembly except with the previous sanction of the President.
3.5 Provincial taxes in respect of professions, etc [Under Article 163]
A Provincial Assembly may by Act impose taxes, not exceeding such limits as may from time to time be fixed
by Act of Majlis-e-Shoora (Parliament), on persons engaged in professions, trades, callings or employments,
and no such Act of the Assembly shall be regarded as imposing a tax on income.
3.6 Grants out of consolidated fund [Under Article 164]
The Federation or a Province may make grants for any purpose, notwithstanding that the purpose is not one
with respect to which Majlis-e-Shoora (Parliament) or, as the case may be, a Provincial Assembly may make
laws.
3.7 Exemption of certain public property from taxation [Under Article 165]
The Federal Government shall not, in respect of its property or income, be liable to taxation under any Act of
Provincial Assembly and, subject to clause (2), a Provincial Government shall not, in respect of its property or
income, be liable to taxation under Act of Majlis-e-Shoora (Parliament) or under Act of the Provincial Assembly
of any other Province.
If a trade or business of any kind is carried on by or on behalf of the Government of a Province outside that
Province, that Government may, in respect of any property used in connection with that trade or business or
any income arising from that trade or business, be taxed under Act of Majlis-e-Shoora (Parliament) or under
Act of the Provincial Assembly of the Province in which that trade or business is carried on.
Nothing in this Article shall prevent the imposition of fees for services rendered.
3.8 Power of Majlis-e-Shoora (Parliament) to impose tax on the income of certain corporations, etc.
[Under Article 165A]
For the removal of doubt, it is hereby declared that Majlis-e-Shoora (Parliament) has, and shall be deemed
always to have had, the power to make a law to provide for the levy and recovery of a tax on the income of a
corporation, company or other body or institution established by or under a Federal law or a Provincial law or
an existing law or a corporation, company or other body or institution owned or controlled, either directly or
indirectly, by the Federal Government or a Provincial Government, regardless of the ultimate destination of
such income.
All orders made, proceedings taken and acts done by any authority or person, which were made, taken or
done, or purported to have been made, taken or done, before the commencement of the Constitution
(Amendment) Order 1985, in exercise of the powers derived from any law referred to in above para, or in
execution of any orders made by any authority in the exercise or purported exercise of powers as aforesaid,
shall, notwithstanding any judgment of any court or tribunal, including the Supreme Court and a High Court, be
deemed to be and always to have been validly made, taken or done and-shall not be called in question in any
court, including the Supreme Court and a High Court, on any ground whatsoever.
Every judgment or order of any court or tribunal, including the Supreme Court and a High Court, which is
repugnant to the provisions of above paras shall be, and shall be deemed always to have been, void and of no
effect whatsoever.

Conceptual Approach to Taxes 23


Constitutional Provisions Chapter-03

4. FEDERAL LEGISLATIVE LIST


4.1 Introduction
Federal Legislative List defines the areas whereby Federal Government can legislate to collect Revenue. This
is a long list, however, we herein discuss the areas which relates to Taxation.
4.2 Powers of the Federation to legislate on taxes
Following entries in the Federal legislative list as contained in the Constitution of Pakistan relates to taxes:

Entry No Taxes which can be imposed by the Federation

47. Taxes on income other than agricultural income;

48. Taxes on corporations.

49. Taxes on the sales and purchases of goods imported, exported, produced, manufactured
or consumed, except sales tax on services.

50. Taxes on the capital value of the assets, not including taxes on immovable property.

51. Taxes on mineral oil, natural gas and minerals for use in generation of nuclear energy.

52. Taxes and duties on the production capacity of any plant, machinery, undertaking,
establishment or installation in lieu of any one or more of them.

53. Terminal taxes on goods or passengers carried by railway, sea or air; taxes on their fares
and freights.

Keeping in view the above provisions, following laws are enacted by the Federal Government:

Legislative powers of Federation Laws enacted there under

Taxes on income other than agricultural income; Income Tax Ordinance, 2001
Taxes on corporations.
Taxes on mineral oil, natural gas and minerals for
use in generation of nuclear energy.

Taxes on the sales and purchases of goods Sales Tax Act, 1990, Federal Excise Act, 2005,
imported, exported, produced, manufactured or Customs Act, 1969
consumed, except sales tax on services Taxes and
duties on the production capacity of any plant,
machinery, undertaking, establishment or
installation in lieu of any one or more of them.

Taxes on the capital value of the assets, not Income Support Levy, 2013 (repealed through
including taxes on immovable property. Finance Act of 2014) & Capital Value Tax levied
through Finance Act, 1989

4.3 Powers of the Provinces to legislate on taxes


All taxes other than the mentioned in above list of Federal legislative list as contained in the Constitution of
Pakistan are covered in the scope of legislation of Provinces. Accordingly, various types of taxes are
introduced by the Provinces are as under:
Agriculture income Tax
Sales Tax on services
Taxes on transfer of immoveable property
Professional Tax
Tax on luxury Houses
Tax on registration of luxury Vehicles etc.
Property tax

24 Conceptual Approach to Taxes


Constitutional Provisions Chapter-03

CA MOD C PAST PAPERS THEORECTICAL QUESTIONS


Q.NO.9 (a) Autumn 2014 List the taxes which can be imposed by the Federal Government.

Q. NO. 9 (b) Autumn 2014 Briefly describe the duties of National Finance Commission.

Conceptual Approach to Taxes 25


Constitutional Provisions Chapter-03

26 Conceptual Approach to Taxes


Preliminary Chapter-04

Chapter

4 PRELIMINARY

Covered under this chapter:


- Section 1 & 3 of the Income Tax Ordinance, 2001
- All definitions u/s 2 of the Income Tax Ordinance, 2001.
- MCQs with solutions
- ICMAP & CA Mod C past papers theoretical questions
- Past papers of ICMAP stage IV and CAF-6 students

(FOR CAF-6 AND ICMAP STUDENTS)


Short title, extent and commencement [U/S 1]

This Ordinance may be called the Income Tax Ordinance, 2001.


It extends to the whole of Pakistan.
It shall come into force on such date as the Federal Government may, by notification in official Gazette, appoint
{1.7.2002}.

Ordinance to override other laws [U/S 3]

The provisions of this Ordinance shall apply notwithstanding anything to the contrary contained in any other law for the time
being in force.

Definitions [U/S 2]
"Accumulated profits" [U/s 2(1)] in relation to distribution or payment of a dividend, include-
(a) any reserve made up wholly or partly of any allowance, deduction, or exemption admissible under this Ordinance;
(b) all profits of the company including income and gains of a trust up to the date of such distribution or such payment, as
the case may be; and
(c) includes all profits of the company including income and gains of a trust up to the date of its liquidation;
Explanation: From the above it is clear that the term "accumulated profits" out of which companies may distribute or pay
dividends include reserve made up wholly or partly of any allowance, deduction, or exemption available under this
Ordinance, advance or loan to a shareholder and profits where distribution is made on liquidation.
"Appellate Tribunal" [U/s 2(2)] means the Appellate Tribunal Inland Revenue established u/s130;

Explanation: The Tribunal, established u/s130 of the Ordinance enjoys jurisdiction to hear cases of income tax, sales tax
and federal excise. The Customs Tribunal, however, remains separate and independent.
Tribunal is the final fact finding forum. Its decisions on law point are also final if not further contested or entertained by High
Court u/s133 of the Ordinance.
"Approved gratuity fund" [U/s 2(3)] means a gratuity fund approved by the Commissioner Inland Revenue in accordance
with Part III of the Sixth Schedule;
"Approved Annuity Plan" [U/s 2(3A)] means an Annuity Plan approved by Securities and Exchange Commission of
Pakistan (SECP) under Voluntary Pension System Rules, 2005 and offered by a Life Insurance Company registered with the
SECP under Insurance Ordinance, 2000;
"Approved Income Payment Plan" [U/s 2(3B)] means an income Payment Plan approved by Securities and Exchange
Commission of Pakistan (SECP) under Voluntary Pension System Rules, 2005 and offered by a Pension Fund Manager
registered with the SECP under Voluntary Pension System Rules, 2005;

Conceptual Approach to Taxes 27


Taxation System Chapter-01

"Approved Pension Fund" [U/s 2(3C)] means Pension Fund approved by Securities and Exchange Commission of
Pakistan (SECP) under Voluntary Pension System Rules, 2005, and managed by a Pension Fund Manager registered with
the SECP under Voluntary Pension System Rules, 2005;

Explanation of u/s 2(3A) (3B) and (3C): In terms of section 63, eligible persons are allowed tax credit for depositing an
amount in the fund, which is lesser of 25% of their taxable income a person joining at the age of 41 or above is allowed from
1st July 2006 to additional contribution of 2% for the first 10 years for each year exceeding 41 but his contribution should not
exceed 50% of his taxable income or up to Rs. 500,000. This regime allows tax credit to contributions and exemption to
investment income and then taxes the benefits at the time of premature or excessive withdrawals, Existing asset
management companies and life insurance companies are eligible to apply for licenses to set up pension funds and EFU
Life Insurance Company and three leading asset management companies including Arif Habib Investments and Atlas Asset
Management Company are running such schemes.
"Approved Employment Pension or Annuity Scheme" [U/s 2(3D)] means any employment related retirement scheme
approved under this Ordinance, which makes periodical payment to a beneficiary i.e. pension or annuity such as approved
superannuation fund, public sector pension scheme and Employees Old-Age Benefit Scheme;
"Approved Occupational Savings Scheme" [U/s 2(3E)] means any approved gratuity fund or recognized provident fund;

Explanation of u/s 2(3D) and (3E): The schemes include pension scheme and Employee Old-Age Benefit Scheme. No
approval for the second type of scheme is mentioned in Schedule to the Ordinance.
As regards clause (3E), it exclusively defines the terms to mean any approved gratuity or recognised provident fund. In other
words this has to be read in conjunction with section 2(3B), 2(3C) and Sixth Schedule to this Ordinance.

"Approved superannuation fund" [U/s 2(4)]: means a superannuation fund, or any part of a superannuation fund,
approved by the Commissioner Inland Revenue in accordance with Part II of the Sixth Schedule;

Explanation: It exclusively defines statutory superannuation funds that are approved by the Commissioner Inland Revenue
in accordance with Part II of the Sixth Schedule to the Ordinance read with rules 91 to 121.

"Assessment" [U/s 2(5)] includes provisional assessment, re-assessment and amended assessment, and the cognate
expressions shall be construed accordingly;

Explanation: The amended definition of the expression "assessment" has historic background based on judicial
pronouncements that the original assessment order, reassessment orders and final assessment orders are really but steps
in a series of judicial proceedings all connected on intrinsic unity and are regarded as one legal proceeding.

"Assessment year" [U/s 2(5A)] means assessment year as defined in the repealed Ordinance;

Explanation: It is a fixed period of twelve months starting from 1st July and ending on 30th June.
"Asset management company" [U/s 2(5B)] means an asset management company as defined in the Non-Banking
Finance Companies and Notified Entities Regulations, 2007;

Explanation: The asset management companies have the following features:-


1. The company should be a public company under the Companies Ordinance, 1984.
2. It has a paid-up capital of not less than Rs. 30 million,
3. Any director, officer or employee of such company who has not been convicted of fraud or breach of trust or
adjudicated as insolvent or had suspended payment or has compounded with his creditors.
4. The promoters and directors of such company are, in the opinion of corporate law authority, persons of means and
integrity having special knowledge and experience of matters which the company may have to deal with.

"Association of persons" [U/s 2(6)] "AOP" includes a firm, a Hindu undivided family, any artificial juridical person and
anybody of persons formed under a foreign law, but does not include a company;

Explanation: Now the Ordinance treats all kinds of bodies of persons except companies as AOP. It means that Punjab Bar
Council, which is an artificial juridical person, is to be treated as an AOP.

The taxation of AOPs is elaborated in section 92 and 93. If there is a change in the constitution of an AOP, section 98A will
apply. The CBR has clarified that not all AOPs are obliged to deduct tax u/s153 as clause (c) sub-section (9) of the said
section providing that only AOP "constituted by or under law" should act as withholding agent. The concept of AOP under
section 2(6) read with section 80 is different from one provided in section 153(9)(c).

"Banking company" [U/s 2(7)] means a banking company as defined in the Banking Companies Ordinance, 1962 and
includes anybody corporate which transacts the business of banking in Pakistan;

28 Conceptual Approach to Taxes


Preliminary Chapter-04

Explanation: Previously only specific statutory bodies like Pakistan Industrial Credit Investment Corporation (PICIC),
ADBF, National Bank of Pakistan, Band of Punjab, formed under specific law and doing banking business, were covered
under this definition, whereas now all bodies corporate, if engaged in banking business in Pakistan, will be considered
banking companies and will be subjected to higher rate of tax, till the time the rate of tax of public and banking companies
come at par.
The body corporate simplicitor and body corporate formed by or under a law are two different categories.
However, the expression "banking business" remains the same in both the enactments meaning by that only those bodies
corporate will be covered which are engaged in the banking business. The "banking business" is not an ordinary dictionary
expression. It has specific, technical meaning under the relevant law i.e. the Banking Companies Ordinance of 1962, which
reads as under:
"Banking Company" means any company which transacts the business of banking in Pakistan.
Banking means accepting, for the purpose of lending or investment, of deposits of money from the public, repayable on
demand or otherwise, and withdraw-able by cheque, draft, or otherwise.
Any company which is engaged in the manufacture of goods or carries on any trade and which accepts deposits of money
from the public merely for the purpose of financing its business as such manufacturer or trader shall not be deemed to
transact the business of banking within the meaning of this clause.
"BOARD" [U/s 2(8)] means the Central Board of Revenue established under the Central Board of Revenue Act, 1924 and
on the commencement of Federal Board of Revenue Act, 2007, the Federal Board of Revenue established u/s 3 thereof;
"Bonus shares" [U/s 2(9)] includes bonus units in a unit trust;

Explanation: This is an inclusive definition which retains the generally accepted meaning of the word "bonus shares" but
includes in its ambit "bonus units' in a unit trust. The purpose is to treat profit distribution by NIT and other unit trusts at par
with stock dividend issued by companies. Bonus shares shall now to be taxed separately in the hands of shareholders @
5% due to inclusion of the same in the income as provided in section 2(29) of the Ordinance.

"Business" [U/s 2(10)] includes any trade, commerce, manufacture, profession, vocation or adventure or concern in the
nature of trade, commerce, manufacture, profession or vocation, but does not include employment;

Explanation: The Legislature has merged profession or vocation in the definition of '"business." This definition is not
exhaustive. It includes some specific categories but specifically excludes "employment" meaning by that professionals like
doctors, lawyers, accountants, engineers etc. while deriving income from employment will not be charged to tax under the
head "Income from Business" although they derive emoluments from rendering of professional services.
This verifies the principle laid down in 2000 PTD (Trib.) 457 that professional receipts (hospital share) derived by a doctor in
addition to emoluments as employee are not salary.

"Capital asset" [U/s 2(11)] Capital asset means property of any kind held by a person, whether or not connected with a
business, but does not include the following:
(a) Any stock-in-trade, consumable stores or raw materials held for the purpose of business;
(b) Any depreciable and intangible property; or
(c) Any movable property held for personal use by the person or any member of the persons family dependent on the
person but including the following as stated in section 38(5):
A painting, sculpture, drawing or other work of art, Jewellery, a rare manuscript, folio or book, a postage stamp or first day
cover, a coin or medallion; or an antique.
"Charitable purpose" [U/s 2(11A)] includes relief of the poor, education, medical relief and the advancement of any other
object of general public utility;
"Chief Commissioner Inland Revenue" [U/s 2(11B)] means a person appointed as Chief Commissioner Inland Revenue
u/s208 and includes a Regional Commissioner of Income Tax and a Director-General of Income Tax and Sales Tax;
Explanation: This definition was inserted after merger of income tax, sales tax and federal excise into one unified Inland
Revenue Service and after this insertion, consequently, the definition of "Regional Commissioner" appearing in u/s 2(46A)
was deleted.
"Collective investment scheme" [U/s 2(11C)] Collective Investment Scheme means a closed end fund and open-end
scheme as assigned under the Non-Banking Finance Companies (Establishment and Regulation) Rules, 2003;

Explanation: Closed End Fund means an investment Company or a closed end scheme;

Conceptual Approach to Taxes 29


Taxation System Chapter-01

Open end Scheme means a scheme constituted by way of a trust deed that continuously offer for sale its units as
specified in the constituted document that entitle the holder of such units on demand to receive his proportionate share of
net assets of the scheme less any applicable charges.
"Company" [U/s 2(12)] "company" means -
(i) a company as defined in the Companies Ordinance, 1984;
(ii) a body corporate formed by or under any law in force in Pakistan;
(iii) a modaraba;
(iv) a body incorporated by or under the law of a country outside Pakistan relating to incorporation of companies;
(v) a co-operative society, a finance society or any other society;
(va) a non-profit organization
(vb) a trust, an entity or a body of persons established or constituted by or under any law for the time being in force;
(vi) a foreign association, whether incorporated or not, which the Board has, by general or special order, declared to be a
company for the purposes of this Ordinance;
(vii) a Provincial Government;
(viii) a Local Government in Pakistan; or
(ix) a Small Company;

Explanation: This is a referral definition adopting the same meaning as given in section 80. For the interpretation of the
expressions "formed by" or "under any law" see (2000) 82 TAX 52 (H.C.Lah.) = 2000 PTD 3388. A company is defined in the
Companies Ordinance, 1984, as under:
"company" means a company formed and registered under this Ordinance or an existing company;
"Commissioner Inland Revenue" [U/s 2(13)] means a person appointed as Commissioner Inland Revenue and includes
any other authority vested with all or any of the powers and functions of the CIR;

Explanation: The nomenclature of Commissioner Inland Revenue has been changed as a consequence of merger of
income tax, sales tax and federal excise into Inland Revenue Service.
The law after the said amendment envisaged two types of Commissioner Inland Revenues; one appointed as
Commissioner Inland Revenue u/s 208 and the other being taxation officer who was vested with powers of Commissioner
Inland Revenue u/s 209(2). A taxation officer [defined u/s 2(65)] exercising delegated powers u/s 210, however, is not
covered in this definition as explained in section 211(1). The second part of definition only covers those officers who enjoy
original jurisdiction of a case u/s 209(2) and not as delegates u/s 210 and thus to be treated as Commissioner Inland
Revenue as envisaged in section 209(4).
"CIR (Appeals)" [U/s 2(13A)] means a person appointed as a Commissioner Inland Revenue (Appeals) u/s 208;

Explanation: In order to cater for the amendments in income tax, sales tax and federal excise laws, merging all the three
into Inland Revenue. Commissioner Inland Revenue Inland Revenue (Appeals) now hears appeals for income tax, sales tax
and federal excise matters. Commissioner Inland Revenue of Appeals is the first appellate authority. His appointment,
functions and jurisdiction are governed u/s 127 to 129, 208 and 209.
The Board's instructions issued u/s 206 or otherwise are not binding on Commissioner Inland Revenue (Appeals). He is
bound to follow the orders of Income Tax Appellate Tribunal (ITAT) and higher courts as explained in [1996] 73 TAX 132
(Trib)].
Consumer goods [U/s 2(13AA)] means goods that are consumed by the end consumer rather than used in the
production of another good;
"Contribution to an Approved Pension Fund" [U/s 2(13B)] means contribution as defined in rule 2(j) of the Voluntary
Pension System Rules, 2005;

Explanation: At the same time a proviso is added that total tax credit available for the contribution made to approved
employment pension or annuity scheme and approved pension fund under Voluntary Pension System Rules, 2005 should
not exceed the limit prescribed or specified in section 63.
"Co-operative society" [U/s 2(14)] means a co-operative society registered under the Co-operative Societies Act, 1925 or
under any other law for the time being in force in Pakistan for the registration of co-operative societies;

Explanation: This refers to such cooperative societies that are governed under the Cooperative Societies Act of 1925 or
under any other law in force for the registration of cooperative bodies in Pakistan.
The courts have consistently held that cooperative societies are not companies within the ambit of section 16(2)(b) of the
Repealed Ordinance - [now section 80(2)(b)].

30 Conceptual Approach to Taxes


Preliminary Chapter-04

Section 3(e) of the Corporative Societies Act, 1925 says: "society" means a society registered or deemed to be registered
under the Act."
Section 2(e) of Corporative Societies Act defines "registered" to mean a society registered or deemed to be registered under
this Act."
The following enactments for registration of various cooperative societies and regulation of their affairs exist:
Cooperative Societies Act, 1925; (ii) Punjab Amendment Act I of 1992; (iii) Co-operative Societies Act, 1912; (iv) Co-
operative Societies Rules, 1927; (v) Sindh Co-operative Societies Reforms Rules, 1973; (vi) Multi-Unit Co-operative
Societies Act, 1942; (vii) Co-operative Development Board Ordinance, 1962; (viii) Co-operative Farming Act, 1976; (ix)
Co-operative Societies (Reforms) Ordinance, 1980; and (x) Sindh Co-operative Farming Societies.
"Debt" [Section 2(15)] means any amount owing, including accounts payable and the amounts owing under promissory
notes, bills of exchange, debentures, securities, bonds or other financial instruments;

Explanation: It is exclusively defined to mean any amount owing, including accounts payable and the sums owing under
promissory notes, bills of exchange, debentures, securities, bonds and other financial instruments.
The various terms used in this definition clause are defined in their respective law as under:
Section 4 of Negotiable Instruments Act, 1881: "Promissory not" - A "promissory note" is an instrument in writing (not
being a bank-note or a currency-note) containing an unconditional undertaking, signed by the maker, to pay on demand or at
a fixed or determinable future time a certain sum of money only to, or to the order of, a certain person, or to the bearer of the
instrument.
Section 5 of Negotiable Instruments Act, 1881: "Bill of exchange" - A "bill of exchange" is an instrument in writing
containing an unconditional order, signed by the maker, directing a certain person to pay on demand or at a fixed or
determinable future time a certain sum of money only to, or to the order of, a certain person or to the bearer of the
instrument.
An order to pay out of a particular fund is not unconditional within the meaning of this section; but an unqualified order to
pay, coupled with-
(a) an indication of a particular fund out of which the drawee is to reimburse himself or a particular account to be debited
to the amount, or
(b) a statement of the transaction which gives rise to the note or bill, is unconditional.
Where the payee is a fictitious or non-existing person the bill of exchange may be treated as payable to bearer.
Section 2(a) of the Securities Act of 1920: "security - means a marketable deed or document that endeavours to secure
against pecuniary loss, e.g., bearer bonds, stock certificates, treasury bills etc. Currency notes are not covered under this
definition.
Section 2(34) of the Companies Ordinance, 1984: "security' - means any share, scrip, debenture, participation term
certificate, modaraba certificate, musharika certificate, term finance certificate bond, pre-organization certificate or such
other instrument as the Federal Government may, by notification in the official gazette, specify for the purpose.
Section 2(12) of the Companies Ordinance, 1984: "Debenture" - includes debenture stock, bonds, term finance
certificates and any other securities, other than a share of company, whether constituting a charge on the assets of the
company or not.
"Deductible allowance" [U/s 2(16)] means an allowance that is deductible from total income;

Explanation: This definition has been introduced with reference to specific deductions for Zakat and Workers' Welfare Fund
(WWF).
"Depreciable asset" [U/s 2(17)] "depreciable asset" means any tangible movable property, immovable property (other than
unimproved land), or structural improvement to immovable property, owned by a person that -
(a) has a normal useful life exceeding one year;
(b) is likely to lose value as a result of normal wear and tear, or obsolescence; and
(c) is used wholly or partly by the person in deriving income from business chargeable to tax,
but shall not include any tangible movable property, immovable property, or structural improvement to immovable
property in relation to which a deduction has been allowed under another section of this Ordinance for the entire cost
of the property or improvement in the tax year in which the property is acquired or improvement made by the person.
Developmental REIT Scheme [U/s 2(17A)] means Developmental REIT Scheme as defined under the Real Estate
Investment Trust Regulations, 2015;
"Disposal" [U/s 2(18)] A person who holds an asset shall be treated as having made a disposal of the asset at the time
when the asset is sold, exchanged, transferred or distributed or cancelled, redeemed, relinquished, destroyed, lost, expired

Conceptual Approach to Taxes 31


Taxation System Chapter-01

or surrendered, transmitted by succession or under a will, in case of a business asset applied to personal use or discarded
or ceased to be used in business.

Explanation of section 2(18) "disposal" However, the language is restrictive and does not convey this meaning. If
definition of the term is only restricted for the purpose of section 75, then this clause is redundant.

"Dividend" [U/s 2(19)] Includes any:


(a) Distribution by a company of accumulated profits to its shareholders, whether capitalised or not;
(b) Distribution by a company, to its shareholders of debentures, debenture-stock or deposit certificate in any form, to the
extent to which the company possesses accumulated profits;
(c) Distribution made to the shareholders of a company on its liquidation, to the extent to which the distribution is
attributable to the accumulated profits of the company immediately before its liquidation;
(d) Distribution by a company to its shareholders on the reduction of its capital, to the extent to which the company
possesses accumulated profits;
(e) payment by a private company or trust of any sum by way of advance or loan to a shareholder or any payment by any
such company or trust on behalf, or for the individual benefit, of any such shareholder, to the extent to which the
company or trust, in either case, possesses accumulated profits;
(f) Remittance of after tax profit of a branch of a foreign company operating in Pakistan;
but does not include -
(i) A distribution made in respect of any share for full cash consideration, or redemption of debentures or debenture
stock, where the holder of the share or debenture is not entitled in the event of liquidation to participate in the surplus
assets;
(ii) Any advance or loan made to a shareholder by a company in the ordinary course of its business, where the lending of
money is a substantial part of the business of the company;
(iii) any dividend paid by a company which is set off by the company against the whole or any part of any sum previously
paid by it and treated as a dividend to the extent to which it is so set off; and
(iv) Remittance of after tax profit by a branch of Petroleum Exploration and Production (E and P) foreign company,
operating in Pakistan.

Explanation: PE of a non-resident is taxable on attributable profits in Pakistan u/s 105 or in terms of applicable provisions of
a double taxation treaty, if available. A Branch and head office is not independent or separate entities as are a holding or a
subsidiary company.

Example: Iqbal Industries (Pvt.) Ltd. had paid up capital of Rs.100,000 divided into 10,000 shares of Rs.10 each and
accumulated profit of Rs.50,000 at the time of liquidation. The official liquidator realized Rs.380,000 out of which Rs.230,000
was paid to the creditors. Remaining was paid to shareholders. Mr. Amir had 500 shares at the time of liquidation.
Required: Calculate (a) the amount received by Amir and (b) how much out of this amount is to be treated as dividend
income.
Solution:
Total amount received by Amir (150,000 x 500 / 10,000) 7,500
Amount treated as dividend to the extent of accumulated profit
(50,000 x 500 / 10,000) 2,500
Example: Under what circumstances advance or loan to a shareholder by a private company would be treated as 'dividend'
with reference to the Income tax Ordinance, 2001.
Solution: Loan or advance to the extent of accumulated profits paid to a shareholder by a private company as defined in the
Companies Ordinance, 1984 or by a trust shall be treated as dividend.
However, if the company is involved in the business of money lending then loan or advance in the ordinary course of
business shall not be treated as dividend.
"Eligible Person" [U/s 2(19A)] for the purpose of Voluntary Pension System Rules, 2005, means an individual Pakistani
who holds a valid National Tax Number or Computerised National Identity Card or National Identity Card for Overseas
Pakistanis issued by the National Database and Registration Authority:
Provided that the total tax credit available for the contribution made to approved employment pension or annuity scheme and
approved pension fund under Voluntary Pension System Rules, 2005, should not exceed the limit prescribed or specified in
section 63.

32 Conceptual Approach to Taxes


Preliminary Chapter-04

Explanation: For the purpose of voluntary pension scheme, the condition of computerized National Identity Cards or
National Identity Card for Overseas Pakistanis is provided in case of non-availability of National Tax Number.
"Definitions in Electronic Transactions Ordinance" [U/s 2(19B)] the expressions "addressee", "automated",
"electronic", "electronic signature", "information", "information system", "originator" and "transaction", shall have
the same meanings as are assigned to them in the Electronic Transactions Ordinance, 2002;
"Electronic record" [U/s 2(19C)] includes the contents of communications, transactions and procedures under this
Ordinance, including attachments, annexes, enclosures, accounts, returns, statements, certificates, applications, forms,
receipts, acknowledgements, notices, orders, judgments, approvals, notifications, circulars, rulings, documents and any
other information associated with such communications, transactions and procedures, created, sent, forwarded, replied to,
transmitted, distributed, broadcast, stored, held, copied, downloaded, displayed, viewed, read, or printed, by one or several
electronic resources and any other information in electronic form;
"Electronic resource" [U/s 2(19D)] includes telecommunication systems, transmission devices, electronic video or audio
equipment, encoding or decoding equipment, input, output or connecting devices, data processing or storage systems,
computer systems, servers, networks and related computer programs, applications and software including databases, data
warehouses and web portals as may be prescribed by the Board from time to time, for the purpose of creating electronic
record;
"Telecommunication system" [U/s 2(19E)] includes a system for the conveyance, through the agency of electric,
magnetic, electro-magnetic, electro-chemical or electro-mechanicals energy, of speech, music and other sounds, visual
images and signals serving for the impartation of any matter otherwise than in the form of sounds or visual images and also
includes real time online sharing of any matter in manner and mode as may be prescribed by the Board from time to time.";
Explanation of u/s 2(19B) to (19E): The insertion of three expressions, namely, "electronic record", "electronic resource"
and "telecommunication system" should have been under their respective alphabetical place, a pattern universally applied in
section 2.
These definitions are necessitated by the fact that the law now requires mandatory e-filing by the companies. These
expressions take into account the mode of keeping electronic record and method of its communication through electronic
modes.
"Employee" [U/s 2(20)] means any individual engaged in employment;
Explanation: This is an exclusive definition which says that "employee" means any individual engaged in employment.
"Employer" [U/s 2(21)] means any person who engages and remunerates an employee;
Explanation: This is an exclusive definition that says that "employer" means a person who engages and remunerates an
employee.`
"Employment [U/s 2(22)] includes -
(a) a directorship or any other office involved in the management of a company;
(b) a position entitling the holder to a fixed or ascertainable remuneration; or
(c) the holding or acting in any public office;
Explanation: The legislature wants to treat certain persons as employees although in their case the relationship of
"employer" and "employee" (master and servant) is not in existence. Such persons draw remuneration by way of holding
office and not through employment, but for the purpose of this Ordinance they are to be treated as employees. These
include amongst others the President, MNAs, MPAs and part-time directors of companies etc.
Fast moving consumer goods [U/s 2(22A)] means consumer goods which are supplied in retail marketing as per daily
demand of a consumer;
"Fee for technical services" [U/s 2(23)] means any consideration, whether periodical or lump sum, for the rendering of
any managerial, technical or consultancy services including the services of technical or other personnel, but does not
include-
(a) consideration for services rendered in relation to a construction, assembly or like project undertaken by the recipient;
or
(b) consideration which would be income of the recipient chargeable under the head "Salary";
Explanation: The key words in this definition are "managerial", "technical" or "consultancy" services, including services of
"technical" or "other" personnel. The contracts for services are distinguishable on the parameters of "technical" and "non-
technical" services, although the line of demarcation may be very thin and difficult to be drawn in certain circumstances.
Filer [U/s 2(23A)] means a taxpayer whose name appears in the active taxpayers list issued by the Board from time
to time or is holder of a taxpayers card.
"Financial institution" [U/s 2(24)] means an institution as defined under the Companies Ordinance, 1984 as follows;

Conceptual Approach to Taxes 33


Taxation System Chapter-01

Financial institution includes,


(a) a company or an institution whether established under any special enactment and operating within or outside
Pakistan which transacts the business of banking or any associated or ancillary business through its branches;
(b) a modaraba, leasing company, investment bank, venture capital company, financing company, housing finance
company, a non-banking finance company; and
(c) such other institution or companies authorised by law to undertake any similar business, as the federal Government
may, by notification in the office Gazette, specify for the purpose;
Explanation: "It is clear that only sub-clause (c) is applicable as far as this Ordinance is concerned. The company or an
institution which transacts the business of banking or an ancillary business, a Moradabad, a leasing company, investment
bank, a non-banking finance company are separately defined in the Ordinance, therefore, clause (a) and (b) of section
2(15A) cannot apply. The language of section 2(24) is also clear that it only refers to those institutions that are notified for
the purpose of section 2(15A)(c) of the Companies Ordinance, 1984, as "financial institution".
"Finance society" [U/s 2(25)] includes a co-operative society which accepts money on deposit or otherwise for the
purposes of advancing loans or making investments in the ordinary course of business;

Explanation: This is an inclusive definition that in addition to finance societies established under the relevant law covers all
such cooperative societies that accept money on deposit or otherwise for the purpose of advancing loans or making
investment in the ordinary course of their business.
"Firm" [U/s 2(26)] "firm" means the relation between persons who have agreed to share the profits of a business carried on
by all or any of them acting for all;

Explanation: This has the meaning as defined in section 80 which is the same as contained in the Partnership Act of 1932:
"Foreign-source income" [U/s 2(27)] An amount shall be foreign-source income to the extent to which it is not Pakistan-
source income.

Explanation: It means income as defined u/s 101(16). Section 101 has to be kept in mind to determine whether any income
is foreign-source or not. The concept of geographical source of income has been provided in the Ordinance to distinguish
between incomes accruing or arising in Pakistan or treated to be so and those having no Pakistani connection.
"House Building Finance Corporation" [U/s 2(28)] means the Corporation constituted under the House Building Finance
Corporation Act, 1952;

Explanation: This refers to statutory corporation viz. House Building Finance Corporation (HBFC) established under the
relevant Act, 1952.
Imputable income [U/s 2(28A)] in relation to an amount subject to final tax means the income which would have
resulted in the same tax, had this amount not been subject to final tax;
"Income" [U/s 2(29)] includes any amount chargeable to tax under this Ordinance, any amount subject to collection or
deduction of tax at source covered under final tax regime, any amount treated as income under any provision of this
Ordinance and any loss of income;

Explanation: The Ordinance describes different sources of income and prescribes various modes of computation under
each head. The definition in this clause is inclusive and not exhaustive. It includes all kinds of profits and gains or receipts
that are chargeable to tax under this Ordinance.
In the expression "income" the amounts subjected to collection of tax u/s 148, 150, 152(1), 153, 154, 156, 156A, 233, 233A,
234(5), 236M and 236N were included. The word "deduction" has also been added to preclude the possibility of any benefit
by interpreting that only where some tax is collected income will be chargeable to tax and not otherwise. Likewise after
section 234 the words "any amount treated as income under any provision of this Ordinance" are added to cover those
incomes which are artificially made taxable under the Ordinance.
Example: Mr. A is a filer and being as shareholder of S Ltd. In tax year 2016 he received net dividend of Rs.18,000 and
bonus shares of having day end (ex-bonus) value on first day of closure of books Rs.50,000. Calculate his taxable income
and tax liability for tax year 2016.
Solution:
Mr. A
Tax year 2016
Computation of taxable income and tax liability: Rs.
Dividend (Final Tax Regime) (18,000 x 100/90) 20,000
Bonus shares (Final Tax Regime) (at first day end price) 50,000
34 Conceptual Approach to Taxes
Preliminary Chapter-04

Computation of tax liability:


Tax on Dividend @ 10% 2,000
Tax on Bonus issue @ 5% 2,500
Less: tax deducted on dividend 2,000
Less: tax deducted on bonus issue u/s 236M 2,500
Balance tax payable -
"Income year" [U/s 2(29A)] means income year as defined in the repealed Ordinance;

Explanation: This definition is retained as still used and is relevant under a number of provisions of the new Ordinance.
"Individual Pension Account" [U/s 2(29B)] means an account maintained by an eligible person with a Pension Fund
Manager approved under the Voluntary Pension System Rules, 2005;

Explanation; See detailed note u/s 2(3A)


"Industrial undertaking" [U/s 2(29C)] means-
(a) an undertaking which is set up in Pakistan and which employs,-
(i) 10 or more persons in Pakistan and involves the use of electrical energy or any other form of energy which is
mechanically transmitted and is not generated by human or animal energy; or
(ii) 20 or more persons in Pakistan and does not involve the use of electrical energy or any other form of energy
which is mechanically transmitted and is not generated by human or animal energy and which is engaged in,-
(i) the manufacture of goods or materials or the subjection of goods or materials to any process which
substantially changes their original condition; or
(ii) ship-building; or
(iii) generation, conversion, transmission or distribution of electrical energy, or the supply of hydraulic
power; or
(iv) the working of any mine, oil-well or any other source of mineral deposits; and
(b) any other industrial undertaking which the Board may by notification in the official Gazette, specify;
Explanation: It may create adverse impact for exemption clauses where originally no intention was manifested by the
legislature for restrictive scope of the expression. Since the Federal Board of Revenue (FBR) enjoys power to declare any
other concern as industrial undertaking, it is hoped hardship cases, if any, would be dealt sympathetically and judiciously.
"Intangible" [U/s 2(30)] "intangible" means any patent, invention, design or model, secret formula or process, copyright,
trade mark, scientific or technical knowledge, computer software, motion picture film, export quotas, franchise, licence,
intellectual property, or other like property or right, contractual rights and any expenditure that provides an advantage or
benefit for a period of more than one year (other than expenditure incurred to acquire a depreciable asset or unimproved
land).

Explanation: This refers to assets mentioned in section 24, viz. all shades of intellectual property, or other like property or
right that provides advantage or benefit for a period of more than one year. This is an exclusive definition and therefore
eliminates any other possible meaning of the term "intangible." Since the word "intangible" is defined in section 2, it covers
the entire Ordinance unless there is anything repugnant in the subject or context.
"Investment company" [U/s 2(30A)] means an investment company as defined in the Non-Banking Finance Companies
Rules, 2003;
Explanation: "Investment company" was not defined in Investment Companies Advisors Rules, 971. Rather, certain
conditions were laid down for the companies intending to carry on the business of investment. They were as under:
Eligibility for registration
A company proposing to commence business as an investment company shall be eligible for registration under these rules if
it fulfils or complies with the following conditions or requirements, namely:-
- that such company is registered as a public limited company under the Companies Ordinance, 1984;
- that it is to function as a closed-end investment company with a capital of not less than Rs. 100 million:
Provided that an existing investment company shall raise its capital to Rs.100 million within a period of 3 years;
- that no director, officer or employee of such company has been convicted of fraud or breach of trust;
- that no director, officer or employee of such company has been adjudicated as insolvent or has suspended payment
or has compounded with his creditors;

Conceptual Approach to Taxes 35


Taxation System Chapter-01

that the promoters of such company are, in the opinion of the Authority, persons of means and integrity and have special
knowledge of matters which the company may have to deal with as an investment company.
KIBOR [U/s 2(30AA)] means Karachi Interbank Offered Rate applicable on first day of each quarter of the financial year-
Explanation: The introduction of KIBOR as basis for calculating additional tax and compensation rather than determining
these without prevalent market rates. U/s 171 at KIBOR concept has been introduced but for levy of additional tax u/s 205,
KIBOR plus 3% is discriminatory. For both purposes, the same basis should be applied.
"Leasing company" [U/s 2(30B)] means a leasing company as defined in the Non-Banking Finance Companies and
Notified Entities Regulation, 2007;
Explanation: The relevant provision of the said law reads as under:
"Leasing company" means a company engaged wholly in the business of leasing or which invests in such business at any
one time an amount equivalent to at least 75% of its assets.
Provided that cash and bank balances and investment in government securities shall be excluded to calculate investment in
leasing business for purposes of this definition.
"Liquidation" [U/s 2(31)] in relation to a company, includes the termination of a trust;

Explanation: This is an inclusive definition that in addition to the term 'liquidation' in relation to a company also covers the
termination of a trust.
"Local Government" [U/s 2(31A)] shall have the same meaning as in the Punjab Local Government Ordinance, 2001, the
Sindh Local Government Ordinance, 2001, the NWFP Local Government Ordinance, 2001 and the Baluchistan Local
Government Ordinance, 2001;
Explanation: The expression, "Local Government" has been inserted, replacing the term "local authority" wherever it
appears in the Ordinance. These two expressions are different in scope under their respective laws. The expression "local
authority" as denned in Section 2(28) of the General Clauses Act, 1897 has peculiar legal connotations to cover even "local
government" institutions. Definition of the term "Local Governments" which is a system of governance introduced in 2001
cannot be a substitute for "local authority".

"Member" [U/s 2(32)] in relation to an AOP, includes a partner in a firm;

Explanation: This is in relation to an AOP which also now includes partner in a firm as u/s 80 firms under the Partnership
Act are now included in the term 'AOP'-

"Minor child" [U/s 2(33)] means an individual who is under the age of 18 years at the end of a tax year;
Explanation: This is a new definition prescribing the age of 18 years for minority at the end of the tax year.
"Modaraba" [U/s 2(34)] means a modaraba as defined in the modaraba Companies and Modarabas (Floatation and
Control) Ordinance, 1980;
Explanation of u/s 2(34)"modaraba"
It means a Modaraba as defined in the Modaraba Companies and Modarabas Ordinance, 1980. The definition of the term in
the said law reads as under:
"Modaraba" means a business in which a person participates with his money and another with his efforts or skill or both his
efforts and skill and shall include Unit Trusts and Mutual Funds by whatever name called;
"Modaraba certificate" [U/s 2(35)] means a modaraba certificate as defined in the Modaraba Companies and Modarabas
(Floatation and Control) Ordinance, 1980;

Explanation: It has the same meaning as under the relevant law i.e. Modaraba Companies and Modarabas Ordinance of
1980., which says;

"Modaraba Certificate" means a certificate'' of definite denomination issued to the subscriber of the Modaraba
acknowledging receipt of money subscribed by him;
"modaraba company" means a company engaged in the business of floating and managing modaraba;
"Mutual Fund" [U/s 2(35A)] means a mutual fund registered or approved by the Securities and Exchange Commission of
Pakistan;

Explanation: The definition of "mutual fund" cover only such funds that are approved by the Securities and Exchange
Commission of Pakistan (SECP). Previously this condition was not imposed.

NCCPL [U/s 2(35AA)] means National Clearing Company of Pakistan Limited, which is a company
incorporated under the Companies Ordinance, 1984 and licensed as Clearing House by the SECP;

36 Conceptual Approach to Taxes


Preliminary Chapter-04

"Non-banking finance company" [U/s 2(35B)] means an Non-Banking Finance Company (NBFC) as defined in the
Non-Banking Finance Companies (Establishment and Regulation) Rules, 2003.

Explanation: There was a choice between a specific definition of every type of financial institution and one general definition
covering all of them. At present there simultaneously exist specific definitions in respect of various financial institutions as
well as general definition of the expression 'financial institutions'. Drafting assignments should be given to some professional
draftsmen to make the new Ordinance free of such legal mistakes and duplications.
Non Filer [U/s 2(35C)] means a person who is not a filer.
"Non-profit organization" [U/s 2(36)] means any person other than an individual, which is -
(a) established for religious, educational, charitable, welfare or development purposes, or for the promotion of an
amateur sport;
(b) formed and registered under any law as a non-profit organization;
(c) approved by the Commissioner Inland Revenue for specified period, on an application made by such person in the
prescribed form and manner, accompanied by the prescribed documents and, on requisition, such other documents
as may be required by the Commissioner Inland Revenue and none of the assets of such person confers, or may
confer, a private benefit to any other person;
Explanation: It covers organizations running for charitable, religious,-educational purposes and promotion of non-
professional sport. It covers such organizations that adhere to the above activities and get registered with the Commissioner
Inland Revenue and do not confer a private benefit on any other person. The concept is similar to organizations covered
under Rule 47(1) (d) of the repealed Income Tax Rules, 1982. For approval in terms of sub-clause (c) please see rule 211.
"Non-resident person" [U/s 2(37)] A person shall be a non-resident person for a tax year if the person is not a resident
person for that year.
Explanation: It means a person defined in section 81 as "A person shall be a non-resident person for a tax year if the
person is not a resident person for that year."
"Non-resident taxpayer" [U/s 2(38)] means a taxpayer who is a non-resident person;
Explanation: It means a person who is covered in clause (37) above.
"Officer of Inland Revenue" [U/s 2(38A)] means any Additional Commissioner Inland Revenue, Deputy Commissioner
Inland Revenue, Assistant Commissioner Inland Revenue, Inland Revenue Officer, Inland Revenue Audit Officer or any
other officer howsoever designated or appointed by the Board for the purposes of this Ordinance;
Explanation: The expression "taxation officer" defined in omitted clause (65) of section 2 became redundant after merger of
income tax, sales tax and federal excise in Inland Revenue Service.
"Originator" [U/s 2(39)] means Originator as defined in the Asset Backed Securitization Rules, 1999;
Explanation: It is as defined in Asset Backed Securitization Rules, 1989 as means a person who transfers to a Special
Purpose Vehicle any assets in the form of present or future receivables as a consequence of Securitization;
"Pakistan-source income" [U/s 2(40)]
(1) Salary shall be Pakistan-source income to the extent to which the salary -
(a) is received from any employment exercised in Pakistan, wherever paid; or
(b) is paid by, or on behalf of, the Federal Government, a Provincial Government, or a Local Government in
Pakistan, wherever the employment is exercised.
(2) Business income of a resident person shall be Pakistan-source income to the extent to which the income is
derived from any business carried on in Pakistan.
(3) Business income of a non-resident person shall be Pakistan-source income to the extent to which it is directly or
indirectly attributable to -
(a) a permanent establishment of the non-resident person in Pakistan;
(b) sales in Pakistan of goods merchandise of the same or similar kind as those sold by the person through a
permanent establishment in Pakistan;
(c) other business activities carried on in Pakistan of the same or similar kind as those effected by the non-
resident through a permanent establishment in Pakistan; or
(d) any business connection in Pakistan.

Conceptual Approach to Taxes 37


Taxation System Chapter-01

(4) Where the business of a non-resident person comprises the rendering of independent services (including
professional services and the services of entertainers and sports persons), the Pakistan-source business income of
the person shall include in addition to any amounts treated as Pakistan-source income under sub-section (3) any
remuneration derived by the person where the remuneration is paid by a resident person or borne by a permanent
establishment in Pakistan of a non-resident person.
(5) Any gain from the disposal of any asset or property used in deriving any business income referred to in sub-
section (2), (3) or (4) shall be Pakistan-source income.
(6) A dividend shall be Pakistan-source income if it is paid by a resident company.
(7) Profit on debt shall be Pakistan-source income if it is -
(a) paid by a resident person, except where the profit is payable in respect of any debt used for the purposes of a
business carried on by the resident outside Pakistan through a permanent establishment; or
(b) borne by a permanent establishment in Pakistan of a non-resident person.
(8) A royalty shall be Pakistan-source income if it is -
(a) paid by a resident person, except where the royalty is payable in respect of any right, property, or information
used, or services utilised for the purposes of a business carried on by the resident outside Pakistan through a
permanent establishment; or
(b) borne by a permanent establishment in Pakistan of a non-resident person.
(9) Rental income shall be Pakistan-source income if it is derived from the lease of immovable property in Pakistan
whether improved or not, or from any other interest in or over immovable property, including a right to explore for, or
exploit, natural resources in Pakistan.
(10) Any gain from the alienation of any property or right referred to in sub-section (9) or from the alienation of any
share in a company the assets of which consist wholly or principally, directly or indirectly, of property or rights referred
to in sub-section (9) shall be Pakistan-source income.
(11) A pension or annuity shall be Pakistan-source income if it is paid by a resident or borne by a permanent
establishment in Pakistan of a non-resident person.
(12) A technical fee shall be Pakistan-source income if it is -
(a) paid by a resident person, except where the fee is payable in respect of services utilised in a business carried
on by the resident outside Pakistan through a permanent establishment; or
(b) borne by a permanent establishment in Pakistan of a non-resident person.
(13) Any gain arising on the disposal of shares in a resident company shall be Pakistan-source income.
(13A) Any amount paid on account of insurance or re-insurance premium by an insurance company to an overseas
insurance or re-insurance company shall be deemed to be Pakistan source income.
(14) Any amount not mentioned in the preceding sub-sections shall be Pakistan-source income if it is paid by a resident
person or borne by a permanent establishment in Pakistan of a non-resident person.
(15) Where an amount may be dealt with under sub-section (3) and under another sub-section (other than sub-section
(14)), this section shall apply -
(a) by first determining whether the amount is Pakistan-source income under that other sub-section; and
(b) if the amount is not Pakistan-source income under that sub-section, then determining whether it is Pakistan-
source income under sub-section (3).
(16) An amount shall be foreign-source income to the extent to which it is not Pakistan-source income.
Explanation: It means income defined in section 101 covering both the Pakistan source and foreign source income.
"Pension Fund Manager" [U/s 2(40A)] means an asset management company registered under the Non-Banking Finance
Companies Rules, 2003, or a life insurance company registered under Insurance Ordinance, 2000, duly authorized by the
Securities and Exchange Commission of Pakistan and approved under the Voluntary Pension System Rules, 2005, to
manage the Approved Pension Fund;
"Permanent establishment" [U/s 2(41)] Means a fixed place of business through which the business of the person is
wholly or partly carried on, and includes:
(a) a place of management, branch, office, factory or workshop, premises for soliciting orders, warehouse, permanent
sales exhibition or sales outlet, other than a liaison office except where the office engages in the negotiation of
contracts (other than contracts of purchase);
(b) a mine, oil or gas well, quarry or any other place of extraction of natural resources;
(c) an agricultural, pastoral or forestry property;

38 Conceptual Approach to Taxes


Preliminary Chapter-04

(d) a building site, a construction, assembly or installation project or supervisory activities connected with such site or
project but only where such site, project and its connected supervisory activities continue for a period or periods
aggregating more than ninety days within any twelve-months period ;
(e) the furnishing of services, including consultancy services, by any person through employees or other personnel
engaged by the person for such purpose;
(f) a person acting in Pakistan on behalf of the person other than an agent of independent status acting in the ordinary
course of business as such, if the agent -
(i) has and habitually exercises an authority to conclude contracts on behalf of the other person;
(ii) has no such authority, but habitually maintains a stock-in-trade or other merchandise from which the agent
regularly delivers goods or merchandise on behalf of the other person; or
(g) any substantial equipment installed, or other asset or property capable of activity giving rise to income;
Explanation: The concept of PE was never a part of the domestic law and was always defined in various tax treaties. The
definition of PE incorporated in the new Ordinance is based on OECD Model Tax Treaty. It takes into account elaborate
situations and eventualities that constitute PE.
"Person" [U/s 2(42)] The following shall be treated as persons for the purposes of this Ordinance, namely:-
(a) An individual;
(b) a company or AOP incorporated, formed, organised or established in Pakistan or elsewhere;
(c) the Federal Government, a foreign government, a political sub-division of a foreign government, or public international
organisation
Explanation: This refers to the term defined in section 80.
PMEX [U/s 2(42A)] means Pakistan Mercantile Exchange Limited a futures commodity exchange company
incorporated under the Companies Ordinance,1984 and is licensed and regulated by the Securities and Exchange
Commission of Pakistan;
"Pre-commencement expenditure" [U/s 2(43)]
(1) A person shall be allowed a deduction for any pre-commencement expenditure in accordance with this section.
(2) Pre-commencement expenditure shall be amortized @ 20% on straight-line basis.
(3) The total deductions allowed in the current tax year and all previous tax years in respect of pre-commencement
expenditure shall not exceed the amount of the expenditure.
(4) No deduction shall be allowed where a deduction has already been allowed under any other section for the entire
amount of the pre-commencement expenditure in the tax year in which it is incurred.
(5) In this section, "pre-commencement expenditure" means any expenditure incurred before the commencement of a
business wholly and exclusively to derive income chargeable to tax, including the cost of feasibility studies,
construction of prototypes, and trial production activities, but shall not include any expenditure which is incurred in
acquiring land, or which is depreciable or amortized.
Explanation: This refers to expenditure referred to in section 25.
Example: A company incurred following expenses before commencement of its commercial activity.
Cost of feasibility study Rs. 40,000
Cost of trial production activities Rs. 20,000
Purchase of fixed assets Rs. 2,500,000
Required: Calculate the amount of pre-commencement expenses.
Solution:
Pre-commencement expenses Rs.
Cost of feasibility study 40,000
Cost of trial production 20,000
Total 60,000
"Prescribed" [U/s 2(44)] means prescribed by rules made under this Ordinance;

Conceptual Approach to Taxes 39


Taxation System Chapter-01

Explanation: It means as prescribed by the rules made under this Ordinance. The Board enjoys powers to make rules u/s
237. The rule-making power of the Board is subject to the limitation that any rule made should not be violative of the statute.
If a rule is repugnant to law, it will be ultra vires having no legal effect. The statements, forms, returns etc. prescribed by
rules have legal force and both the taxpayers and taxation officers are bound to make compliances, wherever required, in
the prescribed manner.
"Principal officer" [U/s 2(44A)] used with reference to a company or AOP includes -
(a) a director, a manager, secretary, agent, accountant or any similar officer; and
(b) any person connected with the management or administration of the company or AOP upon whom the Commissioner
Inland Revenue has served a notice of treating him as the principal officer thereof;

Explanation: The concept of Principal Officer is very important with reference to a company or an AOP against whom any
proceeding under the law has to be initiated. The definition covers a director, a manager, a secretary, an agent, any similar
officer as Principal Officer or any person connected with the management and administration of a company or association of
persons as Principal Officer on which the Commissioner Inland Revenue serves a notice.
"Private company" [U/s 2(45)] means a company that is not a public company;

Explanation: Private company under the Ordinance is defined exhaustively to mean a company that is not a public
company. The term "public company" is denned in section 2(47). So all the companies which are not covered in clause (47)
will automatically fall in the category of private companies. For the purpose of this Ordinance even the non-listed public
companies, which under the Companies Ordinance, 1984 are not private companies, are to be considered as private.
"Profit on a debt" [U/s 2(46)] whether payable or receivable, means -
(a) any profit, yield, interest, discount, premium or other amount, owing under a debt, other than a return of capital; or
(b) any service fee or other charge in respect of a debt, including any fee or charge incurred in respect of a credit facility
which has not been utilized;

Explanation: It is an exhaustive definition which means any profit, yield, interest, discount, premium or other amount
payable or receivable under a debt, other than a return on capital and any service fee or other charge in respect of a debt
including any such charge on a credit facility which has not been utilized.
"Public company" [U/s 2(47)] means -
(a) a company in which not less than 50% of the shares are held by the Federal Government or Provincial Government;
(ab) a company in which not less than 50% of the shares are held by a foreign Government, or a foreign company owned
by a foreign Government;
(b) a company whose shares were traded on a registered stock exchange in Pakistan at any time in the tax year and
which remained listed on that exchange at the end of that year; or
(c) a unit trust whose units are widely available to the public and any other trust as defined in the Trusts Act, 1882;

Explanation: It is defined to mean a company in which at least 50% of the shares are held by the Federal or Provincial
Governments, listed companies and unit trusts and any other public trust. This is a specific definition that has nothing to do
with the Companies Ordinance 1984, where the term is defined differently.
REIT Scheme [U/s 2(47A)] means a REIT Scheme as defined in the Real Estate Investment Trust Regulations
2015;
"Real Estate Investment Trust Management Company (RMC)" [U/s 2(47B)] means RMC as defined under the Real
Estate Investment Trust Regulations, 2015;
Rental REIT Scheme [U/s 2(47C)] means a Rental REIT Scheme as defined under the Real Estate Investment
Trust Regulations, 2015;
Explanation of u/s 2(47A) and (47Bl: "Real Estate Investment Trust (REIT) Scheme" and "Real Estate Investment
Trust Management Company (REITMC)"
These corporate bodies approved by the Securities and Exchange Commission Pakistan are meant for the purpose of
investment in real estate. These two entities have been given special benefits under the ITO, 2001 to promote a dose end
collective scheme constituted as a unit trust and managed by a real estate trust management company.
"Recognised provident fund" [U/s 2(48)] means a provident fund recognised by the Commissioner Inland Revenue in
accordance with Part I of the Sixth Schedule;

Explanation: It means a provident fund recognized in accordance with Part I of the Sixth Schedule.

40 Conceptual Approach to Taxes


Preliminary Chapter-04

"Rent" [U/s 2(49)] means rent as defined in section 15(2) and includes an amount treated as rent u/s 16; "rent" means any
amount received or receivable by the owner of land or a building as consideration for the use or occupation of, or the right to
use or occupy, the land or building, and includes any forfeited deposit paid under a contract for the sale of land or a building.
Rent also includes Non-adjustable amounts received in relation to buildings.
Explanation: The word "rent" wherever used in the Ordinance will have the above meanings unless the context otherwise
requires,
"Repealed Ordinance" [U/s 2(49A)] means Income Tax Ordinance, 1979;

Explanation: The Income Tax Ordinance, 1979 as a reference wherever appears in the new Ordinance will be referred to
as "repealed Ordinance". Repeal and savings should be seen in section 239.
"Resident company" [U/s 2(50)] A company shall be a resident company for a tax year if -
(a) it is incorporated or formed by or under any law in force in Pakistan;
(b) the control and management of the affairs of the company is situated wholly in Pakistan at any time in the year; or
(c) it is a Provincial Government or Local Government in Pakistan.
Explanation: A company shall be a resident company u/s 83 of the ITO, 2001 for a tax year if-
(a) it is incorporated or formed by or under any law in force in Pakistan;
(b) the control and management of affairs of company is situated wholly or almost wholly in Pakistan at any time in the
year; or
(c) it is a Provincial Government or local government in Pakistan.

"Resident individual" [U/s 2(51)] An individual shall be a resident individual for a tax year if the individual -
(a) is present in Pakistan for a period of, or periods amounting in aggregate to, 183 days or more in the tax year; or
(b) is an employee or official of the Federal Government or a Provincial Government posted abroad in the tax year.

The following method shall be used to determine residential status of an individual under Rule 14 of the Income tax
Rules, 2002
(a) Subject to clause (c), a part of a day that an individual is present in Pakistan (including the day of arrival in, and the
day of departure from, Pakistan) counts as a whole day of such presence;
(b) the following days in which an individual is wholly or partly present in Pakistan count as a whole day of such
presence, namely:-
(i) a public holiday;
(ii) a day of leave, including sick leave;
(iii) a day that the individual's activity in Pakistan is interrupted because of a strike, lock-out or delay in receipt of
supplies; or
(iv) a holiday spent by the individual in Pakistan before, during or after any activity in Pakistan; and
(c) a day or part of a day where an individual is in Pakistan solely by reason of being in transit between two different
places outside Pakistan does not count as a day present in Pakistan.
Explanation: It means a person denned u/s 82 an individual shall be a resident individual for a tax year if the individual-
(a) is present in Pakistan for a period of, or periods amounting in aggregate to,183 days or more in the tax year; or
(b) is an employee or official of the Federal Government or a Provincial Government posted abroad in the tax year.
"Resident person" [U/s 2(52)] A person shall be a resident person for a tax year if the person is -
(a) a resident individual, resident company or resident AOP for the year; or
(b) the Federal Government.

Explanation: It means a person denned u/s 81 person shall be a resident person for a tax year if the person is-
(a) a resident individual, resident company or resident association of persons for the year; or
(b) the Federal Government.
"Resident taxpayer" [U/s 2(53)] means a taxpayer who is a resident person;

Conceptual Approach to Taxes 41


Taxation System Chapter-01

Example: Differentiate between resident and non-resident persons.


(a) AOP in Pakistan while its control and management is not in Pakistan wholly or partly.
(b) An individual resident in Pakistan for a period of 185 days, however his stay in Pakistan was not continuous.
(c) A company incorporated in Pakistan.

Solution:
Non-resident (a) Resident (b) and (c)

"Royalty" [U/s 2(54)] means any amount paid or payable, as consideration for -
(a) the use of, or right to use any patent, invention, design or model, secret formula or process, trademark or other like
property or right;
(b) the use of, or right to use any copyright of a literary, artistic or scientific work, including films or video tapes for use in
connection with television or tapes in connection with radio broadcasting, but shall not include consideration for the
sale, distribution or exhibition of cinematograph films;
(c) the receipt of, or right to receive, any visual images or sounds, or both, transmitted by satellite, cable, optic fiber or
similar technology in connection with television, radio or internet broadcasting;
(d) the supply of any technical, industrial, commercial or scientific knowledge, experience or skill;
(e) the use of or right to use any industrial, commercial or scientific equipment;
(f) the supply of any assistance that is ancillary and subsidiary to, and is furnished as a means of enabling the
application or enjoyment of, any such property or right; and
(g) the disposal of any property or right.
"Salary" [U/s 2(55)] means salary as defined in section 12;

Explanation: This refers to salary as defined in section 12 which means any amount received by an employee from any
employment, whether of a revenue or capital nature, including from clauses (a) to (g) of the said section.
"Schedule" [U/s 2(56)] means a Schedule to this Ordinance;

Explanation: It means a schedule to this Ordinance, The schedules are as good as other parts of the statutes and it is
wrong to assume that in case of conflict between statute and schedules, the latter will prevail. In fact, in case of such a
conflict harmonious approach is to be adopted by the courts.
"Securitization" [U/s 2(57)] means securitization as defined in the Asset Backed Securitization Rules, 1999;
Explanation: "Securitization" means a process whereby any Special Purpose Vehicle raises funds by issue of Term
Finance Certificates or any other instruments with the approval of the Commission, for such purpose and uses such funds by
making; payment to the Originator and through such process acquires the title, property or right in the receivables or other
assets in the form of actionable claims;
"Share" [U/s 2(58)] in relation to a company, includes a modaraba certificate and the interest of a beneficiary in a trust
(including units in a trust);
Explanation: This is an inclusive definition which in addition to the ordinary meaning of the expression "share" includes
Modaraba Certificates and the interest of a beneficiary in a trust including unit trust. The aim is to bring these two categories
at par with shareholders of companies as far as taxation of distribution of profit by modarabas and unit trusts is concerned.
"Shareholder" [U/s 2(59)] in relation to a company, includes a modaraba certificate holder, a unit holder of a unit trust and
a beneficiary of a trust;
Explanation: It includes Modaraba Certificate holder and beneficiary of a trust. Since this is an inclusive definition all other
shareholders automatically fall under its meaning.
"Small Company" [U/s 2(59A)] means a company registered on or after 1.7.2005, under the Companies Ordinance, 1984,
which,-
(i) has paid up capital plus undistributed reserves not exceeding Rs.50 million;
(ia) has employees not exceeding 250 at any time during the year;
(ii) has annual turnover not exceeding Rs.250 million; and
(iii) is not formed by the splitting up or the reconstitution of company already in existence;

42 Conceptual Approach to Taxes


Preliminary Chapter-04

Explanation: 'Small Company" is denned to mean a company registered on or after 1st July 2005 under the Companies
Ordinance 1984 having paid-up capital plus undistributed reserves not exceeding Rs. 50 million and has an annual turnover
not exceeding Rs. 250 million and the number of employees during the year at any time should not exceed 250. To qualify
under this category the company should not be formed by the splitting up or the reconstitution of the business already in
existence. There has been a demand to promote corporate culture in the country to encourage formal and documented
businesses.
"Special Judge " [U/s 2(59B)] means the special judge appointed u/s 203.
"Special Purpose Vehicle" [U/s 2(60)] means a Special Purpose Vehicle as defined in the Asset Backed Securitization
Rules, 1999;
"Speculation business" [U/s 2(61)] "speculation business" means any business in which a contract for the purchase
and sale of any commodity (including stocks and shares) is periodically or ultimately settled otherwise than by the actual
delivery or transfer of the commodity, but does not include a business in which -
(a) a contract in respect of raw materials or merchandise is entered into by a person in the course of a manufacturing or
mercantile business to guard against loss through future price fluctuations for the purpose of fulfilling the person's
other contracts for the actual delivery of the goods to be manufactured or merchandise to be sold;
(b) a contract in respect of stocks and shares is entered into by a dealer or investor therein to guard against loss in the
person's holding of stocks and shares through price fluctuations; or
(c) a contract is entered into by a member of a forward market or stock exchange in the course of any transaction in the
nature of jobbing arbitrage to guard against any loss which may arise in the ordinary course of the person's business
as such member.
"Stock fund" [U/s 2(61A)] means a collective investment scheme or a mutual fund where the investible funds are
invested by way of equity shares in companies, to the extent of more than seventy five (75%) of the investment.
"Stock-in-trade" [U/s 2(62)] "stock-in-trade" means anything produced, manufactured, purchased, or otherwise acquired
for manufacture, sale or exchange, and any materials or supplies to be consumed in the production or manufacturing
process, but does not include stocks or shares;

Explanation: The persons who are dealing in stocks or shares will not be taxed u/s 18 and in their case any gain or loss
arising from the disposal of such commodities will be considered as capital gain.
"Tax" [U/s 2(63)] means any tax imposed under Chapter II, and includes any penalty, fee or other charge or any sum or
amount leviable or payable under this Ordinance;

Explanation: It means any tax imposed and includes any penalty, fee or other charge or any sum or amount leviable or
payable under this Ordinance.
"Taxable income" [U/s 2(64)] The taxable income of a person for a tax year shall be the total income of the person for
the year reduced (but not below zero) by the total of any deductible allowances under Part IX of this Chapter of the person
for the year.
Example: From following information, compute taxable income of the person.
Total income 400,000
Zakat paid to central zakat fund 10,000
Solution:
Total income 400,000
Less: Zakat 10,000
Taxable income 390,000

Explanation of u/s 2(64)"taxable Income" It refers to the definition given in section 9 which reads as under:

The taxable income of a person for a tax year shall be the total income of the person for the year reduced (but not below
zero) by the total of any deductible allowances.
"Taxpayer" [U/s 2(66)]
means any person who derives an amount chargeable to tax under this Ordinance, and includes-
(a) any representative of a person who derives an amount chargeable to tax;
(b) any person who is required to deduct or collect tax; or
(c) any person required to furnish a return of income or pay tax;

Conceptual Approach to Taxes 43


Taxation System Chapter-01

Explanation: It means any person who derives any amount chargeable under this Ordinance and includes.-
1. Any representative of a person, as defined u/s 172.
2. Any person who is required to collect or deduct tax under this Ordinance.
3. Any person required to furnish a return of income or pay tax under this Ordinance.
"Tax treaty" [U/s 2(67) and 107]
(1) The Federal Government may enter into an agreement with the foreign government for the avoidance of double
taxation and the prevention of fiscal evasion with respect to taxes on income imposed under this Ordinance and
under the corresponding laws in force in that country and may by notification make such provisions necessary for
implementing the agreement.
(2) Where any agreement is made as above the same shall be effective even anything contained in any law for the time
being in force with respect to the following for:
(a) relief from the tax payable under this Ordinance;
(b) the determination of the Pakistan-source income of non-resident persons;
(c) where all the operations of a business are not carried on within Pakistan, the determination of the income
within and outside Pakistan, or the income chargeable to tax in Pakistan in the hands of non-resident persons,
including their agents, branches, and permanent establishments in Pakistan;
(d) the determination of the income to be attributed to any resident person having a special relationship with a
non-resident person; and
(e) the exchange of information for the prevention of fiscal evasion or avoidance of taxes on income chargeable
under this Ordinance and under the corresponding laws in force in that other country.
(3) Any agreement referred above may include provisions for the relief from tax for any period before the commencement
of this Ordinance or before the making of the agreement.
"Tax year" [U/s 2(68)]
(1) Normal tax year shall be a period of twelve months ending on the 30th day of June and shall be denoted by the
calendar year in which the said date falls.
(2) Where a person's income year is different from the normal tax year such income year or period shall be that person's
special tax year.
(3) The Board in the case of a class of persons having a special tax year different from a normal tax year may permit to
use a normal tax year or vice versa.
(4) A person may apply, in writing, to the Commissioner Inland Revenue to allow him to use a twelve months' period,
other than normal tax year, as special tax year and the Commissioner Inland Revenue may by an order, allow him to
use such special tax year and vice versa.
(5) The Commissioner Inland Revenue shall grant permission only if the person has shown a compelling need to use
special tax year or normal tax year on such conditions as deem fit.
(6) An order shall be made after providing to the applicant an opportunity of being heard and where his application is
rejected the Commissioner Inland Revenue shall record in the order the reasons for rejection.
(7) The Commissioner Inland Revenue may, after providing to the person concerned an opportunity of being heard, by
an order, withdraw the permission granted.
(8) An order shall take effect from such date as may be specified in the order.
(9) Where the tax year of a person changes as a result of an order the period between the end of the last tax year prior to
change and the date on which the changed tax year commences shall be treated as "transitional tax year",
(10) A person dissatisfied with an order of Commissioner Inland Revenue may file a review application to the Board, and
the decision by the Board shall be final.
Explanation: It refers to the definition given in section 74.
Example: Differentiate between normal and special tax year.
(i) From 01-07-2015 to 30-06-2016
(ii) From 01-09-2015 to 31-08-2016
Solution: (i) is normal tax year while (ii) is special tax year.

44 Conceptual Approach to Taxes


Preliminary Chapter-04

"Total income" [U/s 2(69) and 10] The total income of a person for a tax year shall be the sum of the
(a) Person's income under each of the heads of income for the year; and
(b) Persons income exempt from tax under any of the provisions of this Ordinance;

Explanation: It means as defined in section 10 which reads as under:


"The total income of a person for tax year shall be the sum of the persons income under each of the heads of Income for the
year".
"Trust" [U/s 2(70) and 80] "trust" means an obligation annexed to the ownership of property and arising out of the
confidence reposed in and accepted by the owner, or declared and accepted by the owner for the benefit of another, or of
another and the owner, and includes a unit trust;

Explanation: It means as defined in section 80 reproduced below:


"trust" means an obligation annexed to ownership of property and arising out of the confidence reposed in and accepted by
the owner, or declared and accepted by the owner for the benefit of another, or of another and the owner, and includes a
unit trust;
"Turnover" [U/s 2(70A) and 113] Means,
(a) the gross receipts, exclusive of Sales Tax and Federal Excise duty or any trade discounts shown on invoices, or bills,
derived from the sale of goods, and also excluding any amount taken as deemed income and is assessed as final
discharge of the tax liability for which tax is already paid or payable;
(b) the gross fees for the rendering of services for giving benefits including commissions; except covered by final
discharge of tax liability for which tax is separately paid or payable;
(c) the gross receipts from the execution of contracts; except covered by final discharge of tax liability for which tax is
separately paid or payable; and
(d) the company's share of the amounts stated above of any association of persons of which the company is a member.
Example: Compute turnover for the purpose of section 113 from following information:
Sales (including sales tax @ 17%) 117,000
Half of the sales (including sales tax) are chargeable to tax under final tax regime.
Solution:
Turnover for section 113
Sales (117,000 x 100 / 117) 100,000
Less: Taxable under final tax regime (excluding sales tax) 50,000
Turnover for the purpose of section 113 50,000
"Underlying ownership" [Section 2(71)] in relation to an entity, means an ownership interest in the entity held,
directly or indirectly through an interposed entity or entities, by an individual or by a person not ultimately owned by
individuals.

Explanation: It means as defined in section 98 that says:


"underlying ownership" in relation to an entity, means an ownership interest in the entity held, directly or indirectly through an
interposed entity or entities, by an individual or by a person not ultimately owned by individuals,
"Units" [U/s 2(72)] means units in a unit trust;

Explanation: It means units in a unit trust.


"Unit trust" [U/s 2(73)] means any trust under which beneficial interests are divided into units such that the entitlements of
the beneficiaries to income or capital are determined by the number of units held.

Explanation: It means a trust as denned in section 80 and reads as under:


"unit trust" means any trust under which beneficial interests are divided into units such that the entitlements of the
beneficiaries to income or capital are determined by the number of units held.
"Venture Capital Company" [U/s 2(74)] and "Venture Capital Fund" shall have the same meanings as are assigned to
them under the Non-Banking Finance Companies (Establishment and Regulation) Rules, 2003;

Conceptual Approach to Taxes 45


Taxation System Chapter-01

Explanation: These shall have the same meanings as are assigned to them under Non-Banking Finance Companies Rules,
2003. Relevant provision of the said statute is as under;

"venture capital company" means a company licensed by the Commission to invest in venture projects through equity or
other instruments whether convertible into equity or not and provides managerial or technical expertise to venture projects,
or acts as a management company for management of venture capital fund;
"venture capital fund" means a. fund licensed under rule 26;
whistleblower [U/s 2(75)] means whistleblower as defined in section 227B;

46 Conceptual Approach to Taxes


Preliminary Chapter-04

MULTIPLE CHOICE QUESTIONS


Q.1 Accumulated profits include
(a) any reserve made up wholly or partly of any allowance, deduction, or exemption admissible under ITO, 2001.
(b) all profits of the company including income and gains of a trust up to the date of payment of dividend.
(c) includes all profits of the company including income and gains of a trust up to the date of its liquidation.
(d) All of the above.
Q.2 Approved gratuity fund means a gratuity fund approved by the Commissioner Inland Revenue in accordance with Part
III of:
(a) Sixth Schedule.
(b) Second Schedule.
(c) Third Schedule.
(d) First Schedule.
Q.3 Approved superannuation fund means a superannuation fund, or any part of a superannuation fund, approved by the
Commissioner Inland Revenue in accordance with Part II of:
(a) Sixth Schedule.
(b) First Schedule.
(c) Second Schedule.
(d) None of the above
Q.4 An asset management company means a company as defined:
(a) In Banking Companies Ordinance, 1962.
(b) In Companies Ordinance, 1984
(c) In Non-Banking Finance Companies and Notified Entities Regulations, 2007
(d) In Income Tax Ordinance, 2001
Q.5 AOP does not include:
(a) Firm.
(b) Company.
(c) Hindu undivided family.
(d) Artificial juridical person.
Q.6 Business includes:
(a) Trade.
(b) Commerce.
(c) Manufacture.
(d) All of the above.
Q.7 Which of the following is not a capital asset?
(a) Painting.
(b) Stock in trade.
(c) Shares of private company.
(d) Book.
Q.8 Charitable purpose includes:
(a) Relief of the poor.
(b) Education.
(c) Medical.
(d) All of the above

Conceptual Approach to Taxes 47


Taxation System Chapter-01

Q.9 Company includes:


(a) Firm.
(b) Hindu undivided family.
(c) Artificial juridical person.
(d) None of the above.
Q.10 Contribution to an Approved Pension Fund means contribution as defined in:
(a) Voluntary Pension System Rules, 2005.
(b) Income Tax Rules.
(c) Companies Ordinance, 1984.
(d) Sales Tax Act, 1990.
Q.11 Co-operative society means a co-operative society registered under
(a) the Co-operative Societies Act, 1925.
(b) any other law for the time being in force in Pakistan for the registration of co-operative societies.
(c) the Companies Ordinance, 1984.
(d) Both (a) and (b).
Q.12 Debt includes:
(a) Accounts payable.
(b) Promissory note.
(c) Bill of exchange.
(d) All of the above.
Q.13 Which of the following is not a deductible allowance:
(a) Donation to charitable institution.
(b) Zakat.
(c) WWF
(d) WPPF
Q.14 Depreciable asset has a useful life of:
(a) Less than 5 years.
(b) Exceeding one year.
(c) 12 months.
(d) One year.
Q.15 Depreciable asset is an asset which is used by a person:
(a) In deriving income from salary.
(b) For personal use.
(c) Income from business.
(d) Income from property.
Q.16 A person who holds an asset shall be treated as having made a disposal of the asset at the time when the asset is:
(a) Sold.
(b) Lost.
(c) Expired
(d) All of the above.
Q.17 Dividend includes:
(a) distribution by a company of accumulated profits to its shareholders, whether capitalised or not.

48 Conceptual Approach to Taxes


Preliminary Chapter-04

(b) a distribution made in respect of any share for full cash consideration, or redemption of debentures or
debenture stock, where the holder of the share or debenture is not entitled in the event of liquidation to
participate in the surplus assets.
(c) any advance or loan made to a shareholder by a company in the ordinary course of its business, where the
lending of money is a substantial part of the business of the company.
(d) any dividend paid by a company which is set off by the company against the whole or any part of any sum
previously paid by it and treated as a dividend to the extent to which it is so set off.
Q.18 Eligible Person for the purpose of Voluntary Pension System Rules, 2005, means an individual Pakistani who holds:
(a) Valid National Tax Number
(b) Computerised National Identity Card
(c) National Identity Card for Overseas Pakistanis issued by the National Database and Registration Authority.
(d) Any of the above
Q.19 "Employment does not include:
(a) a directorship or any other office involved in the management of a company.
(b) a position entitling the holder to a fixed or ascertainable remuneration.
(c) the holding or acting in any public office.
(d) Sole proprietorship.
Q.20 Which of the following are treated as employee for the purpose of Income Tax Ordinance, 2001?
(a) President.
(b) MNAs and MPAs
(c) Part time directors of the companies
(d) All of the above
Q.21 Fee for technical services means:
(a) Consideration for the rendering of any managerial, technical or consultancy services.
(b) Consideration for services rendered in relation to a construction, assembly or like project undertaken by the
recipient.
(c) Consideration which would be income of recipient chargeable under the head "Salary".
(d) All of above
Q.22 Finance society includes a co-operative society which accepts money on deposit or otherwise for the purposes of:
(a) Advancing loans or making investments in the ordinary course of business.
(b) Establishment of Finance society.
(c) Making charitable donation
(d) All of the above
Q.23 "Firm" means the relation between persons who have agreed to:
(a) Establish a non-profit organization
(b) Incorporate a company under Companies Ordinance, 1984.
(c) Share the profits of a business carried on by all or any of them acting for all
(d) None of the above.
Q.24 Income include:
(a) any amount chargeable to tax under this Ordinance.
(b) any amount subject to collection or deduction of tax at source covered under final tax regime.
(c) any loss of income
(d) amount of any bonus shares
(e) All of the above

Conceptual Approach to Taxes 49


Taxation System Chapter-01

Q.25 Industrial Undertaking means an undertaking which is:


(a) Set up in Pakistan
(b) Set up outside Pakistan
(c) Subsidiary of a foreign company
Q.26 Number of employees in an industrial undertaking which involves the use of electrical energy or any other form of
energy which is mechanically transmitted is:
(a) Less than 5
(b) 10 or more
(c) Less than 10
(d) 5 or more
Q.27 Number of employees in an industrial undertaking which does not involve the use of electrical energy or any other
form of energy which is mechanically transmitted is:
(a) 20 or more
(b) 10 or more
(c) Less than 20
(d) 40
Q.28 Which of the following is not an intangible asset under Income Tax Ordinance, 2001?
(a) Computer software
(b) License
(c) Expenditure that provides an advantage or benefit for a period of more than one year
(d) Expenditure incurred to acquire a depreciable asset
Q.29 KIBOR means Karachi Interbank Offered Rate applicable:
(a) On first day of each quarter
(b) On first day of each month
(c) On first day of each Year
(d) All of the above
Q.30 Member in relation to an AOP, includes:
(a) Partner in a firm
(b) Manager of the firm
(c) Director
(d) Employee of the firm
Q.31 Minor child means an individual who is under the age of 18 years at the:
(a) Beginning of a tax year
(b) End of a tax year
(c) Beginning of a Calendar year
(d) End of a Calendar year
Q.32 Mutual Fund means a mutual fund registered or approved by the:
(a) Appellate Tribunal Inland Revenue
(b) Commissioner Inland Revenue
(c) Registrar
(d) Securities and Exchange Commission of Pakistan
Q.33 Salary shall be Pakistan-source income to the extent to which the salary:
(a) is received from any employment exercised in Pakistan, wherever paid
(b) is paid by, or on behalf of, the Federal Government, a Provincial Government, or a Local Government in
Pakistan, wherever the employment is exercised
(c) Both (a) or (b)

50 Conceptual Approach to Taxes


Preliminary Chapter-04

(d) None of the above


Q.34 Business income of a resident person shall be Pakistan-source income to the extent to which the income is derived
from:
(a) Business carried on in Pakistan or outside Pakistan
(b) Business carried on outside Pakistan
(c) Business carried on in Pakistan
Q.35 Dividend shall be Pakistan-source income if it is paid by a:
(a) Company incorporated under Companies Ordinance, 1984.
(b) Non-resident company
(c) Resident company
(d) All of the above
Q.36 Profit on debt shall be Pakistan-source income if it is:
(a) Paid by a resident person
(b) Borne by a permanent establishment in Pakistan of a non-resident person.
(c) None of the above
(d) Both (a) and (b)
Q.37 Royalty shall be Pakistan-source income if it is:
(a) Paid by a resident person
(b) Borne by a permanent establishment in Pakistan of a non-resident person.
(c) None of the above
(d) Both (a) and (b)
Q.38 Rental income shall be Pakistan-source income if it is derived from:
(a) Lease of movable property in Pakistan
(b) Lease of immovable property in Pakistan
(c) Lease of movable property outside Pakistan
(d) Lease of immovable property outside Pakistan
Q.39 A pension or annuity shall be Pakistan-source income if it is:
(a) Paid by a resident person
(b) Borne by a PE in Pakistan of a non-resident person.
(c) None of the above
(d) Both (a) and (b)
Q.40 Which of the following is not a person for the purpose of Income Tax Ordinance, 2001?
(a) An individual;
(b) A company or AOP incorporated, formed, organised or established in Pakistan or elsewhere;
(c) The Federal Government, a foreign government, a political sub-division of a foreign government, or public
international organisation
(d) None of the above
Q.41 Pre-commencement expenditure shall be amortized @:
(a) 10%
(b) 15%
(c) 20%
(d) 5%
Q.42 Pre-commencement expenditure shall be amortized:
(a) On straight-line basis

Conceptual Approach to Taxes 51


Taxation System Chapter-01

(b) Using diminishing balance method


(c) Using sum of digits method
(d) According to the International Accounting Standards
Q.43 Principal Officer used with reference to a company or AOP includes:
(a) a director, a manager, secretary, agent, accountant or any similar officer
(b) any person connected with the management or administration of the company or AOP upon whom the
Commissioner Inland Revenue has served a notice of treating him as the principal officer thereof;
(c) Both (a) and (b)
(d) None of the above
Q.44 Private Company means a company that is:
(a) Listed company
(b) Subsidiary of a listed company
(c) Not a public company
(d) An AOP in which a private company has made investment
Q.45 Which of the following is not a public company?
(a) a company in which not less than 50% of the shares are held by the Federal Government or Provincial
Government
(b) a company in which not less than 50% of the shares are held by a foreign Government, or a foreign company
owned by a foreign Government
(c) a company whose shares were traded on a registered stock exchange in Pakistan at any time in the tax year
and which remained listed on that exchange at the end of that year
(d) a unit trust whose units are widely available to the public and any other trust as defined in the Trusts Act,
1882.
(e) None of the above
Q.46 Which of the following is not rent?
(a) Forfeited deposit paid under a contract for the sale of land
(b) Forfeited deposit paid under a contract for the sale of Building
(c) Non-adjustable amounts received in relation to buildings
(d) Non-adjustable amounts received in relation to Land
Q.47 A company shall be a resident company for a tax year if -
(a) It is incorporated or formed by or under any law in force in Pakistan
(b) The control and management of the affairs of the company is situated wholly in Pakistan at any time in the
year
(c) It is a Provincial Government or Local Government in Pakistan
(d) All of the above
Q.48 An individual shall be a resident individual for a tax year if the individual -
(a) Is present in Pakistan for a period of, or periods amounting in aggregate to, 183 days or more in the tax year
(b) Is an employee or official of the Federal Government or a Provincial Government posted abroad in the tax year
(c) Both (a) or (b)
(d) None of the above
Q.49 For the purpose of residential status of an individual, the day on which he arrives Pakistan shall be:
(a) Ignored
(b) Counted as whole day
(c) Counted as half day
(d) Counted as two days

52 Conceptual Approach to Taxes


Preliminary Chapter-04

Q.50 For the purpose of residential status of an individual, the day on which he leaves Pakistan shall be:
(a) Ignored
(b) Counted as one whole day
(c) Counted as half day
(d) Counted as 2 days
Q.51 For the purpose of residential status of an individual, the day on which he leaves Pakistan shall be:
(a) Ignored
(b) Counted as whole day
(c) Counted as half day
(d) Counted as two days
Q.52 An industrial undertaking is an undertaking that is engaged in:
(a) the manufacture of goods or materials or the subjection of goods or materials to any process which
substantially changes their original condition
(b) ship-building
(c) generation, conversion, transmission or distribution of electrical energy, or the supply of hydraulic power
(d) the working of any mine, oil-well or any other source of mineral deposits; and
(e) All of the above
Q.53 A small company has employees:
(a) Not exceeding 10
(b) Not exceeding 250 at beginning of the year
(c) Not exceeding 250 at any time during the year
(d) Not exceeding 250 at the beginning of the year
Q.54 Repealed Ordinance means:
(a) Income Tax Ordinance, 1979
(b) Income Tax Ordinance, 2001
(c) Income Tax Rules
(d) None of the above
Q.55 A day or part of a day where an individual is in Pakistan solely by reason of being in transit between two different
places outside Pakistan:
(a) Does not count as a day present in Pakistan
(b) Counted as whole day
(c) Counted as half day
(d) Counted as two days
Q.56 For the purpose of Income Tax Ordinance, 2001, Federal Government shall be considered as:
(a) Resident person
(b) Non-resident person
(c) None of the above
Q.57 Which of the following is not a Royalty income?
(a) the use of, or right to use any patent, invention, design or model, secret formula or process, trademark or other
like property or right;
(b) the use of, or right to use any copyright of a literary, artistic or scientific work, including films or video tapes for
use in connection with television or tapes in connection with radio broadcasting, but shall not include
consideration for the sale, distribution or exhibition of cinematograph films;
(c) the receipt of, or right to receive, any visual images or sounds, or both, transmitted by satellite, cable, optic
fiber or similar technology in connection with television, radio or internet broadcasting;

Conceptual Approach to Taxes 53


Taxation System Chapter-01

(d) Income from lease of moveable or immovable property.


Q.58 Which of the following is not a royalty income?
(a) the supply of any technical, industrial, commercial or scientific knowledge, experience or skill;
(b) the use of or right to use any industrial, commercial or scientific equipment;
(c) the supply of any assistance that is ancillary and subsidiary to, and is furnished as a means of enabling the
application or enjoyment of, any such property or right; and
(d) the disposal of any property or right.
(e) None of the above
Q.59 In relation to company, share includes:
(a) Modaraba certificate
(b) Interest of a beneficiary in a trust
(c) Units in a trust
(d) All of the above
Q.60 In relation to company, shareholder includes:
(a) Modaraba certificate holder
(b) Beneficiary of a trust
(c) Unit holder of a unit trust
(d) All of the above
Q.61 A small company has paid up capital plus undistributed reserves:
(a) Not exceeding Rs.50 million
(b) Exceeding Rs. 25 million
(c) Rs. 10 million
(d) Not exceeding Rs. 10 million
Q.62 The taxable income under NTR of a small company shall be charged to tax @ _______.
(a) 20%
(b) 25%
(c) 32%
(d) 35%
Q.63 A small company has turnover:
(a) Not exceeding Rs.250 million
(b) Exceeding Rs. 250 million
(c) Rs. 10 million
(d) Not exceeding Rs. 10 million
Q.64 Speculation business does not include a business in which:
(a) a contract in respect of raw materials or merchandise is entered into by a person in the course of a
manufacturing or mercantile business to guard against loss through future price fluctuations for the purpose of
fulfilling the person's other contracts for the actual delivery of the goods to be manufactured or merchandise to
be sold;
(b) a contract in respect of stocks and shares is entered into by a dealer or investor therein to guard against loss
in the person's holding of stocks and shares through price fluctuations; or
(c) a contract is entered into by a member of a forward market or stock exchange in the course of any transaction
in the nature of jobbing arbitrage to guard against any loss which may arise in the ordinary course of the
person's business as such member.
(d) All of the above
Q.65 Which of the following is not included in stock in trade?
(a) Raw material to be used in manufacturing
(b) Work in process

54 Conceptual Approach to Taxes


Preliminary Chapter-04

(c) Shares
(d) Finished goods manufactured or acquired
Q.66 Taxpayer means any person who derives an amount chargeable to tax under this Ordinance, and includes-
(a) any representative of a person who derives an amount chargeable to tax
(b) any person who is required to deduct or collect tax
(c) any person required to furnish a return of income or pay tax
(d) All of the above
Q.67 Normal tax year shall be a period of twelve months ending on:
(a) 1st day of July
(b) 30th day of June
(c) 31st day of December
(d) None of the above
Q.68 Tax year which is different from normal tax year is called:
(a) Special tax year
(b) Transitional tax year
(c) Assessment year
(d) Financial year
Q.69 Where a person changes his tax year from special to normal or normal to special tax year the period between the end
of the last tax year prior to change and the date on which the changed tax year commences shall be treated as:
(a) Special tax year
(b) Transitional tax year
(c) Assessment year
(d) Financial year
Q.70 "Turnover" means:
(a) the gross receipts from sales of goods less sales tax, Federal Excise duty, trade discounts shown on invoices,
or bills and income assessed under final tax regime;
(b) the gross fees for the rendering of services for giving benefits including commissions except covered under
final tax regime;
(c) the gross receipts from the execution of contracts except covered by final tax regime; and
(d) the company's share of the amounts stated above of any association of persons of which the company is a
member.
(e) All of the above
Q.71 "Permanent establishment" means a fixed place of business through which the business of the person is wholly or
partly carried on, and includes:
(a) a place of management, branch, office, factory or workshop, premises for soliciting orders, warehouse,
permanent sales exhibition or sales outlet, other than a liaison office except where the office engages in the
negotiation of contracts (other than contracts of purchase);
(b) a mine, oil or gas well, quarry or any other place of extraction of natural resources;
(c) an agricultural, pastoral or forestry property;
(d) All of the above
Q.72 "Permanent establishment" means a fixed place of business through which the business of the person is wholly or
partly carried on and does not include:
(a) any substantial equipment installed, or other asset or property capable of activity giving rise to income;
(b) a building site, a construction, assembly or installation project or supervisory activities connected with such
site or project continue for a period or periods more than 90 days within any 12 months period ;
(c) the furnishing of services, including consultancy services, by any person through employees or other
personnel engaged by the person for such purpose;
(d) None of the above

Conceptual Approach to Taxes 55


Taxation System Chapter-01

ANSWERS
Q 1 2 3 4 5 6 7 8 9
A (d) (a) (a) (c) (b) (d) (b) (d) (d)
Q 10 11 12 13 14 15 16 17 18
A (a) (d) (d) (a) (b) (c) (d) (a) (d)
Q 19 20 21 22 23 24 25 26 27
A (d) (d) (a) (a) (c) (e) (a) (b) (a)
Q 28 29 30 31 32 33 34 35 36
A (d) (a) (a) (b) (d) (c) (c) (c) (d)
Q 37 38 39 40 41 42 43 44 45
A (d) (b) (d) (d) (c) (a) (c) (c) (e)
Q 46 47 48 49 50 51 52 53 54
A (d) (d) (c) (b) (b) (b) (e) (c) (a)
Q 55 56 57 58 59 60 61 62 63
A (a) (a) (d) (e) (d) (d) (a) (b) (a)
Q 64 65 66 67 68 69 70 71 72
A (d) (c) (d) (b) (a) (b) (e) (d) (d)

56 Conceptual Approach to Taxes


Preliminary Chapter-04

ICMAP PAST PAPERS THEORETICAL QUESTIONS


Q.NO. 2(a) August 2014 Define the term public company as per the Income Tax Ordinance, 2001.
Q. 2(a) February 2014 Define the term Electronic Record as per the Income Tax Ordinance, 2001.
Q. NO. 2(b) February 2013 Explain the term ''Industrial undertaking'' as per section 2(29C) of the ITO, 2001.
Q. NO. 2(a) Summer 2011 Define the following terms under the Income Tax Ordinance, 2001:
(i) Business (ii) Charitable purpose (iii) Employment
Q. NO. 2(a) Winter 2010 Define the terms (i) Association of Persons (ii) Resident Company (iii) Profit on Debt
Q. NO. 2(a) Summer 2010 Define the terms (i) Fee for technical services (ii) Employment (iii) Small company
3 (c) Summer-2010 Define the terms (i) Fee for technical services (ii) Non-profit organization
Q. NO. 2(c) Winter 2008 What is the definition of Small Company u/s 2(59A) of the I.T.O. 2001?
Q. NO. 3(c) Summer 2008 Define the terms (i) Fee for technical services (ii) Non-profit organization
Q. NO. 2(c) Winter 2007 In the light of the ITO-2001 what do you understand from the term Trust?
Q. NO. 2(d) Winter 2007 Define the terms (i) Pre-commencement expenditure u/s 25 (ii) Rent, u/s 15.
Q. NO. 3(a) Summer 2007 Define the term amalgamation as per section 2(1A) of the ITO-2001.
Q. NO. 3(b) Summer 2007 Define the term industrial undertaking u/s 2(29C) of the ITO-2001.
Q. NO. 3(c) Summer 2007 Define the term profit on debt as per section 2(46) of the ITO-2001.
Q. NO. 4(b) Summer 2006 Explain (i) Resident (ii) Association of Persons (iii) Finance society (iv) Trust (v) Royalty
Q. NO. 7(a) Winter 2005 Explain (i) Association of Persons (ii) Debt (iii) Employer (iv) Non-profit organization
Q. NO. 2(a) Summer 2005 Who is treated as Persons u/s 80 of the ITO-2001?
Q. NO. 2(b) Summer 2005 Define the terms (i) Resident and non-person (ii) Resident individual (iii) Resident company
Q. NO. 6(a) Summer 2005 Define Fee for Technical Services under ITO-2001.
Q. NO. 3(b) Summer 2004 Explain the term Public Company under ITO-2001.
Q. NO. 7 Summer 2004 Define the terms (i) Banking company (ii) Debt (iii) Dividend (iv) Income
Q. NO. 2 Summer 2003 Define the terms (a) Amalgamation (b) Fee for technical services (c) Tax year (d) Non profit
organization

Conceptual Approach to Taxes 57


Taxation System Chapter-01

CA MOD C PAST PAPERS THEORETICAL QUESTIONS


Q. NO. 7 Spring 2014 Certain payments made by a private limited company to its shareholders can be treated as
dividend. Explain the above in the context of ITO, 2001. Also identify the exceptions to this rule.
Q. No. 2 (b) Spring 2013
Explain the term Associates. State the circumstances under which a shareholder in a company and the company
may be regarded as associates.
Q. No. 5 (a) Spring 2013 Differentiate between public company and private company within the meaning of Income Tax
Ordinance, 2001.
Q. No. 4(b) Spring 2013 What do you understand by the term Dividend as referred to in the Income Tax Ordinance, 2001?
Briefly discuss the provisions relating to imposition of tax on dividend.
Q.2 (a) Autumn 2012 What do you understand by the term Royalty as described in the Income Tax, 2001?
Q.2 (a) Spring 2012 Briefly discuss concepts of Public Company and Small Company as explained in ITO, 2001.
Q.6 (b) Spring 2012 Under the Income Tax Ordinance 2001, where a person is reasonably expected to act in accordance
with the intentions of another person, both persons are considered as associates.
Required: (i) Explain the term person in the above context. (ii) State the circumstances in which a company and its
shareholder shall be considered as associates.
Q.4 (b) Autumn 2011 Explain the term industrial undertaking as specified in the Income Tax Ordinance, 2001.
Q.1 Spring 2002 Define the following with reference to the Income Tax Ordinance 1979:
(a) Assessee (b) Capital asset (c) Dividend (d) Public Company
Q.1 Autumn 2002 (a) Under what circumstances Advance or Loan to a shareholder by a private company would be treated
as dividend with reference to the provision of the Income Tax Ordinance, 1979.
(b) Rose Company Ltd. has a paid up capital of Rs. 5,000,000 consisting 500,000 shares of Rs.10 each. On 30.6.2001 the
companys balance sheet shows accumulated profits of Rs. 1,500,000. Last year the company also created a reserve of Rs.
1,000,000 for issue of Bonus shares. The company has to be liquidated. The official liquidator realized Rs. 6,500,000 and
distribution among the shareholders was made at the rate of Rs.13 per share. Shewani Group owns 200,000 shares in the
company.
How much of the amount received by Shewani Group is dividend? Please explain your answer.
(c) Please define terms Co-operative Society and Finance Society with reference to ITO, 1979 and compare the same.
Autumn 2001 Q.1 Define and explain the following with reference to the Income Tax Ordinance, 1979:
(a) Assessment Year (b) Special Income Year (c) Resident (d) Tax (e) Average rate of tax

58 Conceptual Approach to Taxes


Introduction and Geographical Source of Income Chapter-05

Chapter

5 INTRODUCTION AND GEOGRAPHICAL


SOURCE OF INCOME

Section Rule TOPICS COVERED


What is income tax?
Taxability in Pakistan
2(52), 82, 83, 84 14 Resident person and determination of residential status
Total income
Tax regimes
Income tax return
101 Determination of geographical source of income
50, 51, 102 and 103 Special cases for taxation of foreign source of income
Multiple Choice questions with solutions
ICMAP and CA Mod C past papers theoretical questions

(FOR CAF-6 AND ICMAP STUDENTS)


Introduction:
This chapter deals with the basic concepts of Income Tax Ordinance, 2001. Being as direct tax it is levied on the income of
tax payers and the Federal Board of Revenue is responsible to monitor its levy, collection and other procedural issues.
What is income tax?
Income tax is a direct tax which a person is liable to pay on his income earned during the tax year.
Taxability in Pakistan
Persons are divided into two categories for the purpose of taxability:
1. Resident: Resident person is taxable for both Pakistan Source Income and Foreign Source Income (excluding
foreign source salary income if foreign tax paid or foreign salary is not taxable over there or when there is double
taxation treaty agreement).
2. Non Resident: Non-resident person is taxable for Pakistan Source Income only.
Resident person [U/s 2(52)]
A person shall be a resident person for a tax year if the person is-
(a) a resident individual, resident company or resident AOP for the year; or
(b) the Federal Government.
Determination of residential status
Residential Status of an individual [Section 82]
Residential status for tax purpose has nothing to do with the nationality or domicile for an individual because the residential
status is based on number of days of physically presence in Pakistan during a tax year. Therefore, a foreigner may be a
resident person and a Pakistani national may be a non-resident for tax purposes. A person may be resident in a tax year and
non resident in next tax year and vice versa.

Physical stay in a tax year in Pakistan Status Status


(a) 0-182 days Non-Resident
(b) 183 or more days (equal or more than 50% of days in a year) Resident
(c) A government employee posted abroad in the tax year is resident irrespective of his physical stay in Pakistan.

Conceptual Approach to Taxes 59


Introduction and Geographical Source of Income Chapter-05

Rule 14 of the Income Tax Rules 2002:


Rule to count days an individual present in Pakistan becomes vital when a person has frequent visits to or from
Pakistan. Rule 14 of the Income Tax Rules prescribes the procedure for counting of days as under:
Part of a day that an individual is present in Pakistan counts as a whole day including:
- A day of arrival in Pakistan
- A day of departure from Pakistan
- A public holiday
- A day of leave
- A day that the individual's activity in Pakistan is interrupted because of a strike, lock-out or delay in receipt of
supplies
- A holiday spent in Pakistan before, during or after any activity in Pakistan
- A day In Pakistan solely by reason of being in transit does not count as a day present in Pakistan
Residential Status of Company [Section 83]
(a) A company incorporated in Pakistan, provincial government and local governments are resident without any
condition.
(b) A company incorporated outside Pakistan is resident if control and management of the affairs is situated wholly in
Pakistan in the year.
Residential Status of Association of Persons (AOP) [Section 84]
AOP shall be considered as resident if control and management of affairs is situated wholly or partly in Pakistan in the
year.
Total income
Income tax is generally applicable on total income (including exempt income) that has been divided into following 5 heads
of income.
NTR = Normal tax regime SBI = Separate block of income FTR = Final tax regime

S. No. Head of income Tax regime


1 Salary income NTR and SBI
2 Property income NTR
3 Business income NTR and FTR
4 Capital gains NTR and Fixed tax as SBI
5 Other sources NTR and SBI under FTR

Tax regimes
There are three tax regimes:
Normal tax regime (NTR):
Incomes which are chargeable to tax under NTR are added together to obtain taxable income and then tax rate according to
the slab rate is applied to determine tax liability.
Separate block of income under NTR (SBI):
Incomes which are chargeable to tax under Separate Block of Income are taxable under Normal Tax Regime, however,
income is not added in taxable income and further, fix rates applicable for income covered under Separate Block of Income
to determine tax liability.
Minimum tax liability under NTR:
Applicable under sections 113, 148, 153 and 235.
Final tax regime (FTR):
In case of Incomes which are chargeable to tax under Final Tax Regime, tax is deductible at a fixed rate on income and
such tax deduction is treated as final discharge of tax liability.
Separate block of income (SBI) under FTR:
Incomes which are chargeable to tax under SBI are taxable under Final Tax Regime; however, income is not added in
taxable income and further, fixed rates applicable for income covered under Separate Block of Income to determine tax
liability.

60 Conceptual Approach to Taxes


Introduction and Geographical Source of Income Chapter-05

Total tax liability:


Total tax liability is calculated as follows:
Tax liability under NTR xxx
Tax liability under SBI xxx
Tax liability under FTR xxx
Total tax liability xxx

Income tax return


Under Income Tax Ordinance, 2001, certain persons are required to file income tax return.
What is income tax return?
Income tax return is a declaration of total income and tax liability which a person is required to file to the income tax
department. A return of income, if complete shall be treated as assessment order.
What is assessment order?
An order in which relevant income tax authority assess the total income and tax liability of a taxpayer, is called assessment
order. In Pakistan, return of income filed by the taxpayer is treated as assessment order if return filed is complete in all
respects.
Determination of geographical source of income [S 101]
Salary shall be Pakistan source income which:
(a) is received from any employment exercised (place of rendering of services) in Pakistan, wherever paid; or
(b) is paid by, or on behalf of, the Federal Government, a Provincial Government, or a Local Government in Pakistan,
wherever the employment is exercised (place of rendering of services).
Rental income shall be Pakistan source income :
If it is derived from the lease of immovable property in Pakistan including a right to explore for, or exploit, natural sources in
Pakistan (for owner of immoveable property).
Any gain from the alienation of any property or right
As stated above the rental income or from the alienation of any share in a company the assets of which consist wholly or
principally, directly or indirectly, of property or rights shall be Pakistan source income.
Business income of a resident person:
Shall be Pakistan source income to the extent to which the income is derived from any business carried on in Pakistan.
Business income of a non-resident person
Shall be Pakistan source income which is directly or indirectly attributable to-
(a) A Permanent Establishment of the non-resident person in Pakistan;
(b) Sales in Pakistan of goods merchandise of the same or similar kind as sold by the person through a Permanent
Establishment in Pakistan;
(c) Other business activities carried on in Pakistan of the same or similar kind as those affected by the non-resident
through a Permanent Establishment in Pakistan; or
(d) Any business connection in Pakistan.
A dividend shall be Pakistan-source income
If it is-
(a) Paid by a resident company; or
(b) Dividend u/s 2(19)(f) for remittance of after tax profit of a branch of a foreign company operating in Pakistan other
than branch of Petroleum Exploration and production.
Profit on debt shall be Pakistan-source income if it is -
(a) paid by a resident person, except where the profit is payable in respect of any debt used for the purposes of a
business carried on by the resident outside Pakistan through a permanent establishment; or
(b) borne by a permanent establishment in Pakistan of a non-resident person.

Conceptual Approach to Taxes 61


Introduction and Geographical Source of Income Chapter-05

A royalty shall be Pakistan-source income if it is -


(a) paid by a resident person, except where the royalty is payable in respect of any right, property, or information used, or
services utilised for purposes of a business carried on by resident outside Pakistan through a permanent
establishment; or
(b) borne by a permanent establishment in Pakistan of a non-resident person.
A pension or annuity shall be Pakistan-source income
if it is paid by a resident or borne by a permanent establishment in Pakistan of a non-resident person.
Any gain arising on the disposal of shares in a resident company shall be Pakistan source income.
A technical fee shall be Pakistan-source income if it is -
(a) paid by a resident person, except where the fee is payable in respect of services utilised in a business carried on by
the resident outside Pakistan through a permanent establishment; or
(b) borne by a permanent establishment in Pakistan of a non-resident person.
Any gain arising on the disposal of shares:
in a resident company shall be Pakistan source income.
Any amount paid on account of insurance or re-insurance premium
by an insurance company to an overseas insurance or re-insurance company shall be deemed to be Pakistan source
income.
Any amount not mentioned above:
Shall be Pakistan source income if it is paid by a resident person or borne by a permanent establishment in Pakistan of a
non-resident person.
Where amount may be dealt with Pakistan source income business income of non-resident and under any other above sub-
sections, this section shall apply -
(a) by first determining whether the amount is Pakistan source income ; and
(b) if amount is not Pakistan source income under that sub-section, then determining whether it is Pakistan source
income .
Foreign source income
An amount shall be foreign source income to the extent to which it is not Pakistan source income.
Example
Mr. Asif is marketing manager of ABC Pakistan (Pvt.) Ltd. since 2005. His responsibilities include looking after and
supervision of the marketing related matter of Asian region. In the said region, major set ups are situated in India,
Bangladesh and Singapore. The location wise bifurcation of his monthly salary is as under:
Particulars Pakistan India Bangladesh Singapore
Pay 600,000 530,000 500,000 350,000
House rent 250,000 210,000 200,000 140,000
Utilities 65,000 50,000 45,000 35,000
Medical reimbursements 150,000 10,000 55,000 30,000
Total 1,065,000 800,000 800,000 555,000
However, in the current year, his financial results are as under:

Particulars India Bangladesh Singapore Amount

Net foreign source income 2,205,000 620,000 245,000 3,070,000


You are required to compute the taxable income and tax liability of Mr. Asif for the tax year 2016.

62 Conceptual Approach to Taxes


Introduction and Geographical Source of Income Chapter-05

Solution
Mr. Asif
Tax year 2016
Computation of taxable income
Particulars Pakistan India Bangladesh Singapore
Pakistan source income
Gross salary 1,065,000 800,000 800,000 555,000

Less: Exempt medical reimbursement 150,000 10,000 55,000 30,000


Taxable 915,000 790,000 745,000 525,000
Total of all countries Pakistan source of income 2,975,000
Foreign source income
India Bangladesh Singapore Total gross
Net foreign source income 2,205,000 620,000 245,000 3,070,000
Total income (Pakistan + Foreign Source) 6,045,000
Taxpayer is a non-salaried person hence rates for non-salaried persons have been applied.
Tax on Rs. 6,045,000 [1,319,500 + 35% x (6,045,000 - 6,000,000)] 1,335,250
THREE CONSIDERATIONS FOR FOREIGN SOURCE OF INCOME
(a) Foreign source salary income shall be subject to fulfillment of payment and exempt source of income.
(b) Where incomes other than foreign source salary income are added in the foreign tax credit is allowed after
considering double taxation treaty agreements.
Example: Differentiate between Pakistan Source Income and Foreign Source Income.
(a) Salary paid by Provincial Govt. of Pakistan to an employee working abroad.
(b) Business income received by a non-resident not through his permanent establishment.
(c) Dividend income received by a non-resident from a resident company.
(d) Miss Amna is a resident person sold shares of a non-resident company at profit.
(e) Pension paid by a non-resident person to a resident person.

Solution: Pakistan source income: a, b, c Foreign source income: d, e,


Example:
Following is the details of income of Mr. Bhatti during the year 2016 are as under.
Rs.
1. Interest on Foreign Bonds (one-third is received in Pakistan). 150,000
2. Agricultural income from Bangladesh, received there but later on remitted to Pakistan 70,000
3. Property income from UK received outside Pakistan 500,000
4. Business income from Turkey which is controlled through a permanent establishment in Pakistan
(Rs. 45,000 is received in Pakistan) 135,000
5. Dividend paid by a resident company but received outside Pakistan 100,000
6. Remittance (not in the nature of income) brought to Pakistan 120,000
7. Profits from a business in Karachi and managed from outside Pakistan 70,000
8. Profit on sale of an asset in Pakistan but received in USA 250,000
9. Pension from Pakistan Government but received in USA 70,000
What is the gross total income of Mr. Bhatti, if he is: (i) resident in Pakistan and (ii) non-resident in Pakistan?

Conceptual Approach to Taxes 63


Introduction and Geographical Source of Income Chapter-05

Solution:
RESIDENT IN NON-
PAKISTAN RESIDENT
1. Interest on Foreign Bonds
(one-third is taxable on receipt basis) 50,000 -
(two-third is taxable on accrual basis) 100,000 -
2. Agriculture income in Bangladesh income accrued and received outside Pakistan 70,000 -
3. Income from property in UK received outside Pakistan:
Income accruing and arising outside Pakistan 500,000 -
4. Income earned from business in Turkey which is controlled through a permanent
establishment in Pakistan (Rs. 45,000 is received in Pakistan)
Rs. 45,000 is taxable on receipt basis 45,000 45,000
Balance is taxable on accrual basis 90,000 90,000
5. Dividend paid by a resident company but received outside Pakistan
income declared to be Pakistan source [Section 101(6)] 100,000 100,000
6. Remittance (not in the nature of income brought to Pakistan
No question of taxability arise
70,000 70,000
8. Profit on sale of an asset in Pakistan but received in London:
Income declared to be Pakistan source [Section 12(5)] 250,000 250,000
9. Pension from Pakistan Government but received in London:
Income declared to be Pakistan-source [Section 101(11)] 70,000 70,000
1,345,000 625,000

Special cases of taxability of foreign source income


Foreign source income of a short term resident [Section 50]
An individual shall be exempt in respect of his foreign-source income which is not brought / received in Pakistan if he is
resident only by reason of his employment and he is present in Pakistan for not exceeding 3 years.
This section shall not apply
(a) on business established in Pakistan by the person being an individual.
(b) Any foreign source income brought or received in Pakistan by the person being an individual.
Foreign source Income of a returning expatriate [Section 51]

If an individual citizen of Pakistan (returning expatriate) is resident in the current tax year but was non-resident in the 4
preceding tax years, his foreign-source income shall be exempt current tax year and in the following tax year.
Salary earned outside Pakistan shall be exempt if a citizen of Pakistan leaves Pakistan during a tax year and remains
abroad during that tax year.
Foreign source salary of resident individuals [Section 102]
Any foreign-source salary received by a resident individual shall be exempt from tax if the individual has paid foreign income
tax on such salary or his employer has withheld from the salary and paid to the revenue authority of the foreign country in
which the employment was exercised.
A credit or exemption shall be allowed only if the foreign income tax is paid within 2 years after the end of the tax year in
which the foreign income was derived by the resident taxpayer otherwise in the absence of double tax treaty agreement the
same shall be taxable in Pakistan. [Section 103]

64 Conceptual Approach to Taxes


Introduction and Geographical Source of Income Chapter-05

MULTIPLE CHOICE QUESTIONS


Q.1 Income tax is
(a) Direct tax
(b) Indirect tax
(c) Semi-direct tax
(d) None of the above
Q.2 A person is liable to pay income tax on taxable income earned during the
(a) Financial year
(b) Calendar year
(c) Tax year
(d) (a) and (c) above
Q.3 Resident persons are taxable for
(a) Pakistan Source Income
(b) Foreign Source Income
(c) Both (a) and (b)
(d) Business income
Q.4 Residential Status of an individual shall be determined on the basis of:
(a) Nationality
(b) Number of days stay in Pakistan
(c) Govt. employee
(d) Either (b) or (c)
Q.5 For income tax purposes, a foreigner:
(a) May be a resident person
(b) Cannot be a resident person
(c) None of the above
Q.6 For a resident individual, physical stay basis is necessary for
(a) Equal or less than 182
(b) Equal or more than 183 days
(c) No physical stay is necessary
Q.7 A government employee posted abroad is resident because:
(a) He stays in Pakistan for 182 days
(b) He stays in Pakistan for 183 or more days
(c) He does not stay in Pakistan
(d) He has no limitation on stay in Pakistan
Q.8 Part of a day an individual is present in Pakistan for a day of arrival, departure, public holiday or day of leave
(a) Counts as a whole day
(b) Counts as half day
(c) Is not counted
Q.9 A day that the individuals activity in Pakistan is interrupted because of a strike, lock-out or delay in receipt of supplies
(a) Is not counted
(b) Counts as a whole day
(c) Counts as half day
(d) None of the above

Conceptual Approach to Taxes 65


Introduction and Geographical Source of Income Chapter-05

Q.10 A day in Pakistan only by reason of being in transit between two different countries
(a) Counts as whole day
(b) Is not counted
(c) Counts as half day
(d) None of the above
Q.11 Company incorporated outside Pakistan is resident if control and management of the affairs is situated:
(a) Wholly or partly in Pakistan
(b) Wholly in Pakistan
(c) Partly in Pakistan
(d) Any where
Q.12 AOP shall be resident if control and management of the affairs is situated:
(a) Wholly in Pakistan
(b) Partly in Pakistan
(c) Wholly or partly in Pakistan
(d) Anywhere
Q.13 Total income has been divided into:
(a) 1 head of income
(b) 3 heads of income
(c) 5 heads of income
(d) No heads
Q.14 Salary is Pakistan source of income if it is in Pakistan
(a) Received from any employment exercised outside Pakistan
(b) Received from any employment exercised in Pakistan
(c) Received from any business exercised in Pakistan
Q.15 Profit on debt is not Pakistan source income if
(a) it is paid by resident person
(b) it is paid by permanent establishment of non-resident
(c) it is paid by non-resident person.
Q.16 Pension paid by a non-resident person to a resident person is
(a) Pakistan source
(b) foreign source
(c) local source
Q.17 Foreign source of short term resident is not taxable if it not
(a) brought / paid in Pakistan
(b) paid / received in Pakistan
(c) brought / received in Pakistan
Q.18 Total tax liability is equal to tax under
(a) NTR plus FTR
(b) NTR plus minimum tax liability
(c) FTR plus minimum tax liability
Q.19 If income tax return is complete then it is called
(a) a provincial assessment order
(b) an assessment order
(c) a re-assessment order
66 Conceptual Approach to Taxes
Introduction and Geographical Source of Income Chapter-05

Q.20 Foreign source income of a short term resident is not taxable if he is not present in Pakistan
(a) not less than 4 years
(b) more than 3 years
(c) more than 5 years
Q.21 A credit or exemption on foreign salary income shall be allowed only if the foreign tax is paid within
(a) 3 years after the end of tax year
(b) 2 years after the end of tax year
(c) 4 years after the end of tax year
Q.22 Income tax return is the declaration of
(a) total income & taxable income
(b) total taxable income & tax liability
(c) total income & tax liability
Q.23 Where the fee is payable in respect of services utilized in a business carried on resident outside Pakistan through
permanent establishment is
(a) Pakistan sources income
(b) foreign source income
(c) both a & b

ANSWERS

1 (a) 2 (c) 3 (c) 4 (d) 5 (a)

6 (b) 7 (d) 8 (a) 9 (b) 10 (b)

11 (b) 12 (c) 13 (c) 14 (b) 15 (c)

16 (b) 17 (c) 18 (a) 19 (b) 20 (b)

21 (b) 22 (c) 23 (b)

Conceptual Approach to Taxes 67


Introduction and Geographical Source of Income Chapter-05

ICMAP PAST PAPERS THEORETICAL QUESTIONS


Q.No. 2(b) August 2014 Mr. Aslam is a non-resident person working as a Senior Manager in one of the renowned
companies of the United States of America. In addition to his foreign salary income, he has some properties and
investments in shares in Pakistan from which he is earning handsome income. Mr. Aslam has little knowledge about the
Pakistani Tax Laws and he is worried about the payment of his tax liability for the tax year ended June 30, 2014.

Required: Suppose you are Tax Consultant and Mr. Aslam has sought your professional opinion in respect of the following
matters in the light of the Income Tax Ordinance, 2001:
(i) Being a non-resident whether foreign-source income and Pakistan-source income of Mr. Aslam are taxable
or exempt from tax? Discuss.
(ii) What types of Pakistan-source Income are taxable?
(iii) Under what regime the Pakistan-source income of Mr. Aslam will be treated?
(iv) What is the last date for submitting the statement in lieu of return in respect of his Pakistan-source income?

Q. No. 3(a) February 2014 Noorani Merchant and Co. is a tax consultancy firm. It has a list of clients who seek advices in
respect of various tax matters.

Required: Assume that you have been working as a Tax Advisor of Noorani Merchant and Co. and is given a task to
determine the residential status of the following clients for the tax year ended June 30, 2014 under the given three
scenarios. Also substantiate your answer with reasons in the light of the provision of the Income Tax Ordinance, 2001 and
the Income Tax Rules, 2002:

(i) Mr. Fahim resides in London and works as Chief Accountant in a British Company. Assume that he has come
to Pakistan for the first time on a special assignment from his company on March 1, 2014 and left Pakistan on
October 31, 2014.

(ii) Mr. Saleem is a Federal Government employee. Assume that he is posted to the United Arab Emirates for taking
special training on Petroleum Exploration Project from July1, 2013 to June 30, 2014.
Ms. Saima has got a job in St. Micheal Pharma, a reputable company of United State of America (USA). She went to USA
on December 28, 2013 to assume her responsibilities as a Managing Director of the company. Assume that in April, 2014
her company sent her to China on training. On May 31st 2014 on her way back to USA she stayed in Karachi for three days
due to cancellation of flights.

(b) There are various modes of charging Income Tax under the Income Tax Ordinance, 2001 commonly known as Tax
Regimes. Briefly describe each of the following modes of taxation:-

Normal Tax Regime (NTR)


Separate Tax Regime (STR)
Final Tax Regime or Presumptive Tax Regime (FTR)
Minimum Taxation Regime (MTR)
Q. NO. 2(c) SUMMER 2010 What does section 102 of the ITO, 2001 say about foreign source salary of resident individuals?
Q. NO. 2(c) SUMMER 2009 In each of the following cases, you are required to identify the income as Pakistan source
income or foreign, source income, and provide the relevant provision of the sections used as the basis for identification:
(i) Mr. Walter Vincent, is the employee of United Nations, who is employed in Pakistan. His salary is paid by United
Nations through direct credit to his foreign bank account.
(ii) Mr. Jonty Rhodes has been hired for one month as coach of national cricket team for training in Dubai. He has been
paid US $100,000 as fee for professional services by the Federal Government of Pakistan.
(iii) Mr. Hammad Ali is a resident person for tax purpose. He has earned an income of Rs. 500,000 from the disposal of
shares of a company incorporated outside Pakistan.
(iv) M. Clifford, a resident of United Kingdom, has made a gain of UK 1000 from the sale of the shares of a company
listed in London Stock Exchange. The principal activity of the company is to explore for the natural resources of
Pakistan.
(v) Mr. Kareem, a resident of Pakistan, has been paid an amount of US $100,000 on account of secret formula for
making a chemical, by a company incorporated outside Pakistan.
Q. NO. 2(c) SUMMER 2008 Describe the provisions of the ITO, 2001 for foreign source salary of resident individuals.
Q. NO. 2(c) SUMMER 2007 How residential status of an individual is determined under rule 14 of the Income Tax Rules,
2002?
Q. NO. 3(a) WINTER 2006 Explain the foreign source salary of resident individual u/s 102 of the Income tax Ordinance,
2001.

68 Conceptual Approach to Taxes


Introduction and Geographical Source of Income Chapter-05

CA MOD C PAST PAPERS THEORETICAL QUESTIONS


Q. NO. 6(a) Spring 2014 State the provisions of Income Tax Ordinance, 2001 relating to foreign-source salary of
resident individuals.
Q. No. 4 (b) Autumn 2013 In view of the provisions of Income Tax Ordinance, 2001 and Rules made there under, determine
the residential status of the following persons for the tax year 2013:
(i) Ramiz proceeded to Saudi Arabia on 24 December 2012 to assume responsibilities on his new job. He visited Karachi
from 20 June 2013 to 24 June 2013 for presenting a paper in a seminar but due to unavoidable circumstances, the
seminar was cancelled.
(ii) Khalil, an officer working at Ministry of Foreign Affairs, since last three years, was posted to the Pakistans mission in
Geneva from 1 August 2012 to 30 June 2013.
(iii) Ali Associates is a partnership firm and provides consultancy services in Pakistan as well as United Kingdom (UK). The
management and control of its affairs is situated partly in UK and partly in Pakistan.

(iv) Smith, a Nigerian football coach, came to Pakistan on 28 February 2013. He left the country on 31 August 2013.

Q.3 (a) Spring 2013 State the provisions of the Income Tax ordinance, 2001 for determining the residential status of
an Association of persons.
Q.3 (a) Autumn 2012 State the provisions of the Income Tax Ordinance, 2001 with regards to the residential status of
individuals and companies.
Q.3 (b) Autumn 2012 Margaret, a German national was employed as a Technical Manager of Faiza Chemicals Limited, a
resident company, on 1 October 2010 for a term of two years. Under the terms of employment, she was allowed to deliver
lectures at various professional organizations. During tax year 2012, she conducted three workshop sessions, the details of
which are as follows:
- Workshop Session in Lahore: A fee of US$ 15,000 in equivalent Pak Rupees was received from a local event manager.
The fee was credited to her bank account maintained in Karachi.
- Workshop Session in Munich: A fee of US$ 25,000 was received in Germany in her Munich bank account.
- Workshop Session in Dubai: A fee of US$ 20,000 was remitted to her bank account in Karachi.
Required: Discuss the taxability of the amounts received by Margaret for conducting the workshop sessions during the
tax year 2012.
Q.4 (a) Autumn 2011 Briefly discuss residential status of the following persons for the tax year 2011 under the ITO, 2001.
(i) Mr. Shah has been working as an Information Analyst in the Ministry of Foreign Affairs. On 1 November 2010, he was
posted to Pakistan Embassy in Canada for three years.
(ii) Asif Learning Center is a partnership concern, providing IT training to professionals in Pakistan, UAE and Saudi
Arabia. Up to 31 July 2010, the management and control of its affairs was situated partly in Pakistan. However, with
effect from 1 August 2010, the entire management and control of the affairs of the partnership was shifted to Dubai.
(iii) Mr. Liaquat was sent to Pakistan on a special assignment by his UK-based company on 1 March 2011. He left
Pakistan on 09 September 2011.
(iv) Farooq Trading LLC was incorporated as a limited liability company in UAE. The management and control of its affairs
are situated wholly in Pakistan.
Q.2(b) March 2009: Briefly discuss residential status of following persons under the ITO, 2001 for the tax year 2009.
(i) Asif is an employee of Baluchistan Government, who has been sent to United Kingdom for an official assignment on
1.12.2007 for two years.
(ii) Messrs Akhtar Abbas and Co. is a partnership firm, doing business of financial consultancy in Pakistan as well as
United Arab Emirates (UAE). The management and control of its affairs is situated partly in UAE and partly in Pakistan.
Q.4(b) Spring (March) 2006: Mr. A, a Pakistani Citizen, returned to Pakistan in November 2004 after completing his
employment contract In United Arab Emirates (UAE). He worked till October 2004in UAE where there was no tax on
salaries. Mr. A is in Pakistan since then and has been employed by a local company. Explain the tax implication on Mr. A's
income, earned in UAE and Pakistan, for the tax year 2005.
Q.6(a) Sept 2003: Under what circumstances a resident individual is entitled to tax on his foreign source salary, and when is
the foreign tax treated as having been paid?

Conceptual Approach to Taxes 69


Introduction and Geographical Source of Income Chapter-05

Q.2(ii) March 2000: State the basis of taxation regarding residents and non-resident.
Q.6 Sept 2000: Briefly describe the provisions related to the scope of total Income.
Q.9 May 1994: Which of the following appear to be correct in the given choices?
The income of a non-resident individual is taxed in Pakistan which is:
(a) Earned and received in Pakistan
(b) Earned and received abroad
(c) Earned outside Pakistan but received in Pakistan
(d) Earned in Pakistan but received abroad

70 Conceptual Approach to Taxes


Computation of Taxable Income Chapter-06

Chapter

6 COMPUTATION OF TAXABLE INCOME

Topics covered Topics covered


Section Section
(Part - I for CAF-6 and ICMAP students) (Part - II for CA Mod F and ICMAP students)
Part - I Part - II
Taxation of companies, disposal of business by
4 Tax on taxable income 94, 95, 96
individual and AOP
Disposal of assets between wholly owned
companies, disposal of asset under a
5A Tax on un-distributed reserves 97,97A,98
arrangement and reconstruction and change in
control of an entity
Change in the constitution of AOP and
7A Tax on shipping of a resident 98A & 98B
Discontinuance of business or dissolution of AOP
7B Tax on Profit on Debt 98C Succession to business, otherwise than on death
General provisions relating to taxes Taxation of Permanent Establishment of non
8 105
imposed u/s 5, 5A, 6, 7, 7A & 7B resident
Taxable income, total income and heads
9, 10 & 11 107 Avoidance of double taxation
of income
86 Principle of taxation of individuals
87 Deceased individuals MCQs with solutions
ICMAP and CA Mod C past papers theoretical
92 Taxation of AOP
questions
Rates of tax of salaried individual and non
salaried individual
Tax or refund to be computed to the
219
nearest Rupee

PART I (For CAF-6 and ICMAP students)


1. Tax on taxable income [U/s 4]
Income tax shall be imposed each tax year (subject to this Ordinance), at the rate or rates specified the First
Schedule, as the case may be, on every person who has taxable income for the year. From the resultant income tax
computed subtraction shall be made for any tax credits allowed to the taxpayer for the year.
Where a taxpayer is allowed more than one tax credit for a tax year, the credits shall be applied in the following order-
(a) any foreign tax credit allowed u/s 103; then
(b) any tax credit allowed u/s 61 to 65; and then
(c) any tax credit allowed for taxes paid (including advance tax) and tax deducted at source,
Certain classes of income and income of certain classes of persons may be subject to separate taxation or collection
of tax or deduction of tax shall be treated as a final tax on the income of the person.
Income covered under FTR and SBI shall not be included in the computation of taxable income in accordance with
section 8 or 169, as the case may be.
2. Tax on un-distributed reserves [U/s 5A]
Subject to this Ordinance, a tax shall be imposed at the rate of 10%, on every public company other than a
scheduled bank or a Moradabad, that derives profits for a tax year but does not distribute cash dividends
within six months of the end of the said tax year or distributes dividends to such an extent that its reserves,

Conceptual Approach to Taxes 71


Computation of Taxable Income Chapter-06

after such distribution, are in excess of 100% of its paid up capital, so much of its reserves as exceed 100% of
its paid up capital shall be treated as income of the said company:
Provided that for tax year 2015, cash dividends may be distributed before the due date mentioned in section
118(2), for filing of return for tax year 2015.
The above provisions shall not apply to
a. a public company which distributes profit equal to either 40% of its after tax profits or 50% of its paid
up capital, whichever is less, within six months of end of tax year;
b. a company qualifying for exemption U/C (132) Part I of the Second Schedule.
c. a company in which not less than 50% shares are held by the government.
In this section, reserve includes amounts set-aside out of revenue or other surpluses excluding capital
reserves, share premium reserves and reserves required to be created under any law, rules or regulations.
3. Tax on shipping of a resident [U/s 7A]
In the case of any resident person engaged in the business of shipping, a presumptive income tax shall be
charged in the following manner, namely:-
(a) ships and all floating crafts including tugs, dredgers, survey vessels and other specialized
craft purchased or bare-boat chartered and flying Pakistan flag shall pay tonnage tax of an
amount equivalent to one US $ per gross registered tonnage per annum; and
(b) ships, vessels and all floating crafts including tugs, dredgers, survey vessels and other
specialized craft not registered in Pakistan and hired under any charter other than bare-
boat charter shall pay tonnage tax of an amount equivalent to fifteen US cents per ton of
gross registered tonnage per chartered voyage provided that such tax shall not exceed one
US $ per ton of gross registered tonnage per annum:
Explanation.- For the purpose of this section, the expression equivalent amount means the rupee
equivalent of a US dollar according to the exchange rate prevalent on the first day of December in the case of
a company and the first day of September in other cases in the relevant assessment year.
th
The provisions of this section shall not be applicable after 30 June, 2020.
4. Tax on Profit on Debt [U/s 7B]

Subject to this Ordinance, a tax shall be imposed, at the rate specified as under, on every person (other
than a Company) who receives a POD from any person mentioned in clause (a) to (d) of section 151(1).

1. Where profit on debt does not exceed 10%


Rs.25,000,000
2. Where profit on debt exceeds Rs 2,500,000 + 12.5% of the amount exceeding
25,000,000 but does not exceed Rs.
Rs.25,000,000
50,000,000
3. Where profit on debt exceeds Rs Rs 5,625,000 + 15% of the amount exceeding
50,000,000 Rs.50,000,000
The tax imposed above on a person who receives a POD shall be computed by applying the relevant rate
of tax to the gross amount of the profit on debt.
This section shall not apply to a POD that is exempt from tax under this Ordinance.

5. General provisions relating to taxes imposed under FTR / separate block of income [U/s 8]
Subject to this ordinance, where the tax imposed under Final Tax Regime / Separate Block of Income on the amount
in respect of which the tax is imposed and-
(a) Such amount shall not be chargeable to tax under any head of income in computing the taxable income of the
person who derives it for any tax year;
(b) No deduction shall be allowable for any expenditure incurred in deriving the amount;
(c) The amount shall not be reduced by any deductible allowance or the set off of any loss;
(d) The tax payable by a person under final tax regime / separate block of income shall not be reduced by any tax
credits allowed under this Ordinance; and

72 Conceptual Approach to Taxes


Computation of Taxable Income Chapter-06

(e) The liability of a person under final tax regime / separate block of income shall be discharged to the extent that
the tax has been paid in accordance with the relevant section or the tax payable has been deducted at source.
6. Taxable Income (U/s 9)
The taxable income of a person for a tax year shall be the total income (other than exempt income) of the person for
the year (but not below zero) less Zakat, Workers Profit Participation Fund (WPPF), Workers Welfare Fund (WWF)
and deductible allowance for profit on debt (u/s 64A).
Example: If the total income of Mr. Ali is Rs.500,000 then you are required to compute the taxable income of Mr. Ali,
if he paid the following amounts:
Zakat Rs. 18,000, WWF Rs. 5,000, WPPF Rs. 8,000 and Profit on debt (u/s 64A) Rs. 20,000

Solution:
Rs.
Total of income as per question 500,000
Less: deductible allowances: (18,000 + 5,000 + 8,000 +20,000) = (51,000)
Taxable income 449,000
7. Total Income (U/s 10)
The total income of a person for a tax year shall be the sum of the person's income under the heads of income
(including exempt income) for the year.
8. Heads of Income (U/s 11)
(1) For the computation of tax and total income, all income shall be classified under the following heads, namely-
(a) Salary
(b) Property income
(c) Business income
(d) Capital Gains and
(e) Income from Other Sources.
(2) Where the total deductions allowed to a person for a tax year under a head of income exceed the total of the
amounts derived by the person, he sustained a loss equal to the excess amount.
(3) The income of a resident person shall be computed by taking into account amounts that are Pakistan-source
income and foreign-source income.
(4) The income of a non-resident person shall be computed by taking into account only amounts that are
Pakistan-source income.
9. Principle of taxation of individuals [U/s 86] Subject to this Ordinance, the taxable income of each individual shall
be determined separately.
9.1 Deceased individuals [U/s 87] The legal representative of a deceased individual shall be liable for
(a) any tax that the individual would have become liable for if the individual had not died; and
(b) any tax payable in respect of the income of the deceased's estate.
The liability under this Ordinance shall be first charge on the deceased estate and shall be limited on the legal
representative under this section to the extent to which the deceased's estate is capable of meeting the liability.
In this section, "legal representative" means a person who in law represents the estate of a deceased person, and
includes any person who intermeddles with the estate of the deceased and where a party sues or is sued in
representative character the person on whom the estate devolves on the death of the party so suing or sued.
10. Tax liability of a Company and Association of Persons:
10.1 Principles of taxation of associations of persons [Section 92]
An AOP shall be liable to tax separately from the members of the association and where the AOP has paid tax the
amount received by a member of the association out of the income of the association shall be exempt from tax;
Provided that if at least one member of the AOP is a Company, the share of such company or companies shall be
excluded for the purpose of computing the total income of the AOPs & the Company or companies shall be taxed
separately at the rate applicable to the companies, according to their share.

Conceptual Approach to Taxes 73


Computation of Taxable Income Chapter-06

Example
Sultan (Pvt.) Limited and NA International (sole proprietor) of Mr. Asad have a joint venture in the name of Nimco
consultants. It is not a registered with registrar of firms. The share in interest of company and Mr. Asad in the joint
venture is 65:35 respectively. The JV is providing the consultancy services to its clients. Mr. Asad withdraws salary of
Rs. 60,000 per month. Total Income of the Joint Venture is Rs 1,700,000. Carry forward of losses of preceding years
is Rs. 600,000. Tax already deducted is Rs. 85,000. Mr. Asad also provided the loan to the joint venture and received
interest of Rs. 320,000 during the year:
You are required to calculate the taxable income and tax liability of the joint venture, Sultan (Pvt.) and Mr.
Asad for the tax year 2016.
Solution
Computation of tax liability of Joint Venture Rs.

Total income 1,700,000


Add: inadmissible expenses
Salary paid to Mr. Asad (60,000 x 12) 720,000
Profit on debt to Mr. Ahsan 320,000
1,040,000
Taxable income 2,740,000
Less: Carry forward losses (600,000)
Taxable income 2,140,000
Less Share as member in the income of AOP (65%) to be taxed in the hands of the Company 1,391,000
Balance taxable income in the hands of the AOP (35%) 749,000
Tax liability [Rs.7,000+ 10% (749,000 500,000)] 31,900

Computation of Share of members

Divisible income (749,000 31,900) 717,100


Salary (720,000)
Interest on loan (320,000)
Actual loss for Mr. Asad from AOP (322,900)

10.2 Practical example on determination of share from AOP:


IBN is an AOP with two partners Mr. Ghazali and Mr. Aftab with ratio of sharing 40:60.
Rs.
Net profit as per accounts 1,200,000
Depreciation as per accounts 600,000
Salary to partner (Mr. Aftab) 250,000
Brokerage to partner (Mr. Aftab) 100,000
2,150,000
Less: Tax depreciation 900,000
Taxable Income 1,250,000
Income Tax [32,000 + 15% x (1,250,000 - 750,000)] 107,000
Divisible Income 1,143,000
Share of profit from AOP shall be computed as under:
Partner Ghazali Partner Aftab Total
Salary Nil 250,000 250,000
Brokerage Nil 100,000 100,000
Balance 317,200 475,800 793,000
317,200 825,800 1,143,000
Although share of profit from AOP (after tax) is exempt in the hands of individual members but the same shall be
included for rate purpose in other taxable income except income taxed under Final tax regime / Separate block of
income
11. RATES OF INCOME TAX FOR TAX YEAR 2016 (For salaried, non-salaried individual and AOPs)
Tax rate for company is 32%, small company and modaraba company 25%
Tax rates on individual for the Tax Year 2016:
NON-SALARIED CASE AND AOP i.e. where taxable salary is nil or up to 50% of taxable income in case of
individual.

74 Conceptual Approach to Taxes


Computation of Taxable Income Chapter-06

S. No. Taxable income Rate of Tax


(1) (2) (3)

1. Where taxable income does not exceed Rs. 400,000 0%

2. Where the taxable income exceeds Rs. 400,000 but 7% of the amount exceeding Rs. 400,000
does not exceed Rs.500,000

3. Where the taxable income exceeds Rs. 500,000 but Rs.7,000 + 10% of the amount exceeding Rs.
does not exceed Rs.750,000 500,000

4. Where the taxable income exceeds Rs. 750,000 but Rs. 32,000 + 15% of the amount exceeding
does not exceed Rs.1,500,000 Rs. 750,000

5. Where the taxable income exceeds Rs. 1,500,000 Rs. 144,500 + 20% of the amount exceeding
but does not exceed Rs.2,500,000 Rs. 1,500,000

6. Where the taxable income exceeds Rs. 2,500,000 Rs. 344,500 + 25% of the amount exceeding
but does not exceed Rs.4,000,000 Rs. 2,500,000

7. Where the taxable income exceeds Rs. 4,000,000 Rs. 719,500 + 30% of the amount exceeding
but does not exceed Rs.6,000,000 Rs. 4,000,000

8. Where the taxable income exceeds Rs. 6,000,000 Rs. 1,319,500 + 35% of the amount exceeding
Rs. 6,000,000

Provided that in the case of an AOPs that is a professional firm prohibited from incorporating by any law or the rules
of the body regulating their profession, the 35% rate of tax mentioned against serial number 8 of the Table shall be
32% for tax year 2016 and onwards.
SALARIED CASE i.e. where taxable salary exceeds 50% of taxable income
S. No. Taxable income Rate of Tax
(1) (2) (3)
1. Where taxable income does not exceed Rs. 400,000 0%
2. Where the taxable income exceeds Rs. 400,000 but 2% of the amount exceeding Rs. 400,000
does not exceed Rs. 500,000
3.. Where the taxable income exceeds Rs. 500,000 but Rs.2,000+ 5% of the amount exceeding Rs.
does not exceed Rs. 750,000 500,000
4. Where the taxable income exceeds Rs. 750,000 but Rs. 14,500 + 10% of the amount exceeding
does not exceed Rs. 1,400,000 Rs. 750,000
5. Where the taxable income exceeds Rs. 1,400,000 Rs. 79,500 + 12.5% of the amount exceeding
but does not exceed Rs. 1,500,000 Rs.1,400,000
6. Where the taxable income exceeds Rs. 1,500,000 Rs. 92,000 + 15% of the amount exceeding
but does not exceed Rs. 1,800,000 Rs. 1,500,000
7. Where the taxable income exceeds Rs. 1,800,000 Rs. 137,000 + 17.5% of the amount exceeding
but does not exceed Rs. 2,500,000 Rs. 1,800,000
8. Where the taxable income exceeds Rs. 2,500,000 Rs. 259,500 + 20% of the amount exceeding
but does not exceed Rs. 3,000,000 Rs. 2,500,000
9. Where the taxable income exceeds Rs. 3,000,000 Rs. 359,500 + 22.5% of the amount exceeding
but does not exceed Rs. 3,500,000 Rs. 3,000,000
10. Where the taxable income exceeds Rs. 3,500,000 Rs. 472,000 + 25% of the amount exceeding
but does not exceed Rs. 4,000,000 Rs. 3,500,000
11. Where the taxable income exceeds Rs. 4,000,000 Rs. 597,000 + 27.5% of the amount exceeding
but does not exceed Rs. 7,000,000 Rs. 4,000,000
12. Where the taxable income exceeds Rs.7,000,000 Rs. 1,422,000 + 30% of the amount exceeding
Rs. 7,000,000

12. Tax or refund to be computed to the nearest Rupee [U/s 219]


In the determination of any amount of tax or refund payable under this Ordinance, fractions of a rupee less than fifty
paisa shall be disregarded and fractions of a rupee equal to or exceeding fifty paisa shall be treated as one rupee.

Conceptual Approach to Taxes 75


Computation of Taxable Income Chapter-06

PART II (For CA Mod F and ICMAP students)


13. Principles of taxation of Companies [U/s 94]
A company shall be liable to tax separately from its shareholders-
A dividend paid by a company shall be taxable under SBI except where the recipient is a Company.
A dividend paid by a non-resident company to a resident person shall be chargeable to tax under the head "Income
from Business" or "Income from Other Sources", unless the dividend is exempt from tax.
13.1 Tax rates applicable to Companies
Following tax rates are prescribed under Division II Part I First Schedule to the Income Tax Ordinance, 2001 for the
purpose of determination of liability of the Companies:
The rate of tax imposed on the taxable income of a company for the tax year 2016 is 32%.
The rate of tax for small company is defined in clause (ii) Division II Part I First Schedule to the Ordinance in the
following manner:
Where the taxpayer is a small company as defined in section 2, tax shall be payable at the rate of 25%.
The rates of tax in the case of modaraba is stipulated in clause 18, Part II, Second Schedule to the Ordinance in the
following manner:
In the case of a modaraba the rate of income tax shall be 25% of total income excluding such part of total income to
which Division III of Part I of the First Schedule or section l53 or section 154 applies.
The rates of tax are defined to the different categories of companies. The term company includes the following:
Banking company , public company, a unit trust, private company, co-operative society, a finance society, assets
management company, Financial institution, House Building Finance Corporation, investment company, leasing
company, Mutual Fund, non-banking finance company, Venture Capital Company, Real Estate Investment Trust
(REIT) and Real Estate Investment Trust Management Company.
14. Disposal of Business by individual / AOP to wholly owned Company [U/s 95 and 96]
(1) Where a resident individual or resident AOP disposes of all the assets of a business to a resident company,
no gain or loss shall be taken to arise on the disposal if the following conditions are satisfied:-
(a) The consideration received by the transferor / AOP for the disposal is a share of shares in the company
other than redeemable shares;
(b) the transferor / each member of AOP must beneficially own all the issued shares in the same proportion
as it was in the business assets immediately before the disposal to the company;
(c) the company must undertake to discharge any liability in respect of the assets disposed of to the
company;
(d) any liability in respect of the assets disposed of to the company must not exceed the transferor's / AOP
cost of the assets at the time of the disposal;
(e) the FMV of the share or shares received by the transferor / AOP for the disposal must be substantially
the same as the FMV of the assets disposed of to the company, less any liability that the company has
undertaken to discharge in respect of the assets; and
(f) the company must not be exempt from tax for the tax year in which the disposal takes place.
(2) Where aforesaid provisions applies -
(a) each of the assets acquired by the company shall be treated as having the same character as it had in
the hands of the transferor / AOP;
(b) the company's cost in respect of the acquisition of the assets shall be in the case of
(i) a depreciable asset or amortised intangible, the WDV of the asset or intangible immediately
before the disposal;
(ii) stock in trade lower of cost or NRV as per balance sheet value used by the transferor; or
(iii) in any other case, the transferor's / AOP cost at the time of the disposal;
(c) if, immediately before the disposal, the transferor / AOP has deductions allowed for depreciation, initial
allowance and amortization regarding transferred assets which have not been set off against the
transferor's income, the amount not set off shall be added to the deductions allowed to the company in
the tax year in which the transfer is made; and

76 Conceptual Approach to Taxes


Computation of Taxable Income Chapter-06

(d) the transferor's / AOP cost in respect of the share or shares received as consideration for the disposal
shall be
(i) in the case of a consideration of one share, the transferor's / AOPs cost of the assets
transferred as determined under 2(b) above, less the amount of any liability that the company
has undertaken to discharge in respect of the assets; or
(ii) in the case of a consideration of more than one share, the amount determined under sub-clause
(i) divided by the number of shares received.
In determining whether the transferor's / AOP deductions for depreciation, initial allowance and amortization have
been set off against income under sub section 2(c) above, those deductions shall be taken into account last.
15.1 Disposal of asset between wholly-owned companies [U/s 97]
Where a resident company disposes of an asset to another resident company, no gain or loss shall be taken to arise
on the disposal if the following conditions are satisfied:-
(a) Both companies belong to a wholly-owned group of resident companies at the time of the disposal;
(b) The remaining (b), (c) and (d) clauses of this section are same as stated above in clauses (c), (d) and (f) of
sub section 2 of combined sections 96 and 97 respectively.
Where the above provisions applies
The same conditions to be satisfied as stated above in combined sections 96 and 97 from (a) to (d) of sub section 2
shall apply except the following additional note.
The transferor and transferee companies belong to a wholly-owned group if
(a) one company beneficially holds all the issued shares of the other company; or
(b) a third company beneficially holds all the issued shares in both companies.
15.2 Disposal of asset under a scheme of arrangement and reconstruction [U/s 97A]
(1) No gain or loss shall be taken to arise on disposal of asset from one company to another company by virtue of
operation of a Scheme of Arrangement and Reconstruction u/s 282L and 284 to 287 of the Companies
Ordinance, 1984 or section 48 of the Banking Companies Ordinance, 1962 if the following conditions are
satisfied,:
The clauses (a) to (c) of this sub section are same as stated above in clauses (c), (d) and (f) of sub section 2 of
combined sections 96 and 97 respectively and scheme is approved by the High Court, SBP or SECP on or
after 1.7.2007.
(2) No gain or loss shall be taken to arise on issue, cancellation, exchange or receipt of shares as a result of
scheme of arrangement and reconstruction as stated above.
(3) Where the above provisions applies:
The same conditions as stated above in combined sections 96 and 97 from (a) to (d) of sub section 2 shall
apply except the following additional note.
Where sub-section (2) of this section applies and the shares issued by virtue of the Scheme of arrangement
and reconstruction disposed of the cost of shares shall be the cost prior to the operation of the said scheme.
16. Change in control of an entity [U/s 98]
Where there is a change of 50% or more in the underlying ownership of an entity, any loss incurred for a tax year
before the change shall not be allowed as a deduction in a tax year after the change, unless the entity -
(a) continues to conduct the same business after the change as it conducted before the change until the loss has
been fully set off; and
(b) does not, until the loss has been fully set off, engage in any new business or investment after the change
where the principal purpose of the entity or the beneficial owners of the entity is to utilise the loss so as to
reduce the income tax payable on the income arising from the new business or investment.;
In this section. "entity" means a company or AOP to which section 92(1) applies;
"ownership interest" means a share in a company or the interest of a member in an AOP; and
"underlying ownership" in relation to an entity, means an ownership interest in the entity held, directly or indirectly
through an interposed entity or entities, by an individual or by a person not ultimately owned by individuals.

Conceptual Approach to Taxes 77


Computation of Taxable Income Chapter-06

16.1 Change in the constitution of an AOP [U/s 98A]


Where, during the course of a tax year, a change occurs in the constitution of an AOP, liability of filing the return on
behalf of the AOP for the tax year shall be on the AOP as constituted at the time of filing of such return but the
income of the AOP shall be apportioned among the members who were entitled to receive it and, where the tax
assessed on a member cannot be recovered from him it shall be recovered from the AOP as constituted at the time of
filing the return.
16.2 Discontinuance of business or dissolution of an AOP [U/s 98B]
Subject to the provisions of section 117, where any business or profession carried on by an AOP has been
discontinued, or where an AOP is dissolved, all the provisions of this Ordinance, shall, so far as may be, apply as if
no such discontinuance or dissolution had taken place.
Every person, who was, at the time of such discontinuance or dissolution, a member of such AOP and the legal
representative of any such person who is deceased, shall be jointly and severally liable for the amount of tax payable
by the AOP.
17. Succession to business, otherwise than on death [U/s 98C]
(1) Where a person carrying on any business or profession has been succeeded in any tax year by any other
person, otherwise than on the death of the predecessor, and the successor continues to carry on that business
or profession,-
(a) the predecessor shall be liable to pay tax in respect of the income of the tax year in which the
succession took place up to the date of succession and of the tax year or years preceding that year;
and
(b) the successor shall be liable to pay tax in respect of the income of such tax year after the date of
succession.
(2) Where the predecessor cannot be found, the tax liability in respect of the tax year in which the succession took
place up to the date of succession and of the tax year or years preceding that year shall be that of the
successor in like manner and to the same extent as it would have been that of the predecessor, who shall be
entitled to recover it from the predecessor.
18. Taxation of a PE in Pakistan of a non-resident person [U/s 105]
The following principles shall apply in determining the taxable business income of a PE in Pakistan of a NRP namely:-
(a) The profit of the PE shall be computed on the basis that it is a distinct and separate person engaged in the
same or similar activities under the same or similar conditions and dealing wholly independently with the NRP
of which it is a PE;
(b) there shall be allowed as deductions any expenses incurred for the purposes of the business activities of the
PE including executive and administrative expenses so incurred whether in Pakistan or elsewhere;
(c) and (d) no deduction and income shall be considered for amounts paid / payable and received / receivable by
the PE to / from its head office or to another PE of the NRP other than towards reimbursement of actual
expenses incurred by the NRP to third parties by way of:
(i) royalties, fees or other similar payments for the use of any tangible or intangible asset by the PE;
(ii) compensation for any services including management services performed for the PE; or
(iii) profit on debt or insurance premium paid or payable on moneys lent to the PE, except in connection
with a banking business; and
No deduction shall be allowed in computing the taxable business income of a PE in Pakistan of a NRP for a tax year
for head office expenditure in excess of the amount as bears to the turnover of the PE in Pakistan the same
proportion as the non-residents total head office expenditure bears to its worldwide turnover.
In this u/s, "head office expenditure" means any executive or general administration expenditure incurred by the
NRP outside Pakistan for the purpose of the business of the Pakistan PE of the person, including any
(a) rent, local rates and taxes excluding any foreign income tax, current repairs, or insurance against risks of
damage or destruction outside Pakistan;
(b) salary and travelling paid to an employee employed by the head office outside Pakistan; and
(c) other expenditures which may be prescribed.

78 Conceptual Approach to Taxes


Computation of Taxable Income Chapter-06

19. Agreements for avoidance of double taxation and prevention of fiscal evasion [U/s 107]
1. The Federal Government may enter into an agreement, bilateral or multilateral with the government or governments
of a foreign countries or tax jurisdictions for the avoidance of double taxation and the prevention of fiscal evasion
and exchange of information including automatic exchange of fiscal evasion and exchange of taxes in income
imposed under the Ordinance or any other law for the time being in force and under the corresponding laws in force
in that country, and may, by notification in the official Gazette, make such provisions as may be necessary for
implementing the agreement,-
2. Notwithstanding anything contained in any other law to the contrary, the Board shall have the powers to obtain and
collect information when solicited by another country under a tax treaty, a tax information exchange agreement, a
multilateral convention, an inter governmental agreement, a similar arrangement or mechanism.
3. Notwithstanding the provisions of the Freedom of Information Ordinance, 2002, any information received or supplied
and any commitment communication or correspondence made, under a tax treaty, a tax information exchange
agreement, a multilateral convention, a similar arrangement or mechanism, shall be confidential subject to section
216(3).
(a) relief from the tax payable under this Ordinance; (b) the determination of the Pakistan-source income of non-
resident persons;
(c) where all the operations of a business are not carried on within Pakistan, the determination of the income
attributable to operations carried on within and outside Pakistan, or the income chargeable to tax in Pakistan in
the hands of non-resident persons, including their agents, branches and permanent establishments in
Pakistan;
(d) the determination of the income to be attributed to any resident person having a special relationship with a
non-resident person; and
(e) the exchange of information for the prevention of fiscal evasion or avoidance of taxes on income chargeable
under this Ordinance and under the corresponding laws in force in that other country.

Conceptual Approach to Taxes 79


Computation of Taxable Income Chapter-06

MULTIPLE CHOICE QUESTIONS


1. Business income of a minor child (other than income from inherited property) shall be clubbed with the income of his
parent with
(a) Lesser taxable income
(b) Greater taxable income
(c) Any of them or
(d) None of the above
2. The tax liability of a member of an AOP is determined by considering
(a) Share from AOP
(b) Other Income
(c) Both of income
(d) None of these
3. An AOP is taxed
(a) along with its members
(b) separate from its member
(c) Both a and b
(d) one of these
4. The foreign source salary income of a resident individual is
(a) exempt from tax
(b) Exempt if foreign tax is paid, where applicable
(c) Not exempt
5. A resident taxpayer deriving foreign source business income is allowed a tax credit of
(a) Foreign income tax paid
(b) Pakistan income tax payable
(c) Lesser of a and b
(d) none of above
6. Any foreign tax credit in excess of tax liability may be ____________.
(a) Refunded
(b) Carried back
(c) Carried forward
(d) None of these
7. __________received by a member of an AOP is taxable as share of income .
(a) Salary
(b) Commission
(c) Profit on debt
(d) all of (a) to (c)
8. The tax liability of a dissolved AOP shall be recovered from
(a) Majority holder members
(b) All the members
(c) Only from assets of AOP
(d) From the AOP and its all the members

80 Conceptual Approach to Taxes


Computation of Taxable Income Chapter-06

9. Which of the following incomes shall be included in the computation of taxable income under Income Tax Ordinance,
2001
(a) income covered under NTR
(b) income covered under FTR
(c) income covered under SBI under FTR
(d) all of above
10. Under Income Tax Ordinance, 2001, there are ____ heads of income.
(a) 2
(b) 3
(c) 5
(d) 10
11. A non-resident person shall pay the tax in respect of ______________.
(a) Pakistan source income
(b) foreign source income
(c) property income
(d) all of above
12. The total income for the tax year 2016 consists of _______.
(a) taxable income
(b) exempt income
(c) taxable and exempt income
(d) none of the above
13. Any income may be received by__________.
(a) taxpayer himself
(b) any person on his behalf
(c) his employer
(d) both a and b
14. Companies are chargeable to tax __________ its shareholders.
(a) separately from
(b) together with
(c) in respect of its
(d) none of above
15. The taxable income of a salaried taxpayer not exceeds Rs.______ is chargeable to tax @ 0%.
(a) 300,000
(b) 350,000
(c) 400,000
(d) 500,000
16. Where a full time teacher or researcher working in non-profit organization is having salary income as well as some
other taxable income, the benefit of reduction in tax liability shall be available on ___________ if his total taxable
income is less than Rs. 1,000,000.
(a) his total income
(b) his salary income only
(c) income other than salary
(d) all of above

Conceptual Approach to Taxes 81


Computation of Taxable Income Chapter-06

17. Rate of tax for modaraba and a small company is ____.


(a) 50%
(b) 35%
(c) 25%
(d) 20%
18. Share from AOP is ______ in the hands of members.
(a) exempt
(b) taxable
(c) safe
(d) all of above
19. The rate of tax for _____ are same for tax year 2016.
(a) salaried and non-salaried persons
(b) AOP and companies
(c) Non-salaried person and AOP
(d) none of the above
20. The Company share in income (under normal tax regime) of an AOP being as member is to be taxed _________
from the tax year 2016.
(a) exempt from tax
(b) in the hands of the AOP
(c) in the hands of Company without any tax credit
d) none of the above

ANSWERS
1 (b) 2 (c) 3 (b) 4 (b) 5 (c)
6 (d) 7 (d) 8 (d) 9 (a) 10 (c)
11 (a) 12 (c) 13 (d) 14 (a) 15 (c)
16 (b) 17 (c) 18 (a) 19 (d) 20 (c)

82 Conceptual Approach to Taxes


Computation of Taxable Income Chapter-06

ICMAP PAST PAPERS THEORETICAL QUESTIONS


Q. No. 3(a) February 2013 Write short answers of the following questions:
(ii) In case of a non-salaried taxpayer what would be the rate of income tax where the taxable income of an individual
exceeds Rs. 2,500,000?
Q. NO. 3(b) WINTER 2004 Describe main principles of taxation of companies as given in section 94 of Income tax
Ordinance, 2001?
Q. NO. 2(a) SUMMER 2004 What is the tax liability in case of succession to business, otherwise than on death, U/S 98(c) of
Income tax Ordinance, 2001?

Conceptual Approach to Taxes 83


Computation of Taxable Income Chapter-06

CA MOD C PAST PAPERS THEORETICAL QUESTIONS


Q. NO. 5(a) Spring 2015 Under the provisions of the Income Tax Ordinance, 2001 state the rules relating to
residential status of an Association of Person (AOP). Also explain the taxability of income of AOP, in the hands of the
firm and its members.

Q.NO. 2 (a) Autumn 2014 Briefly discuss the provisions of Income Tax Ordinance, 2001 in respect of the following
situations:
Farhan received Rs. 960,000 as his share of profit from AOP, during the tax year 2014. He also earns income from other
sources.

Q. No. 6 Autumn 2013 Ahmed is responsible for managing the property of his uncle who died on 5 February 2013. The
approximate worth of the property if Rs. 7 mission. In August 2013, a notice was received from income tax department in the
name of his uncle requiring details of his income for the tax year 2012 along with demand for payment of tax in respect of
previous year amounting to Rs. 8.5 million.
Required:
Advise Ahmed as regards the following:
(a) Extent of Ahmeds liability in respect of the income earned by his uncle before 5 February 2013.
(b) His obligations relating to the tax assessment proceedings pending/arising against his uncle.

Q. NO. 6(a) Autumn 2009 Mr. Zias father expired in March 2009. Being the only heir, he received all his fathers business
and assets. In August 2009, a notice was received from the income tax department in the name of his father to pay unpaid
tax liabilities along with penalty and additional tax. Mr. Zia is of the view that since his father expired, the notice is irrelevant.
Required: In the light of ITO, 2001, explain the correct legal position of Mr. Zia with regard to his fathers income tax
liabilities and the related income tax proceedings.
Q.3 Autumn 2001 In the light of provisions of Income tax Ordinance, 1979, who is liable to discharge the tax liability of a
deceased person, and to what extent?

84 Conceptual Approach to Taxes


Income From Salary Chapter-07

Chapter

7 INCOME FROM SALARY

Section Rule Topic covered Section Rule Topic covered

12 6 Salary 13 5 Valuation of conveyance


73 Taxability of salary 13 3 Salaries of servants paid by employer
110 Salary paid by private companies 13 4 Valuation of accommodation
12 Profit in lieu of salary 14 Employee share scheme
12 Golden handshake Retirement benefits
11, 50, 51, Geographical source of salary income
Marginal perquisites
101 & 102 and their taxability
13 Value of perquisites Exemptions
Loan provided by employer to
13
employee with no interest
MCQs with solutions
ICMAP and CA Mod C past papers
theoretical questions

For CAF-6 and ICMAP Students


1. Definition of salary [U/s 12]
Any salary received by an employee in a tax year, other than exempt salary under the Income Tax Ordinance 2001,
shall be chargeable to tax in that year under the head salary.
A person is salaried person where taxable salary exceeds 50% of taxable income from all heads of income.
Salary means any amount received by an employee from any employment whether of a revenue or capital nature:
"Employee" means any individual engaged in employment [including an employed director (Rule 6)]. [U/s 2(20)]
"Employer" means any person who engages and remunerates an employee. [U/s 2(21)]
"Employment" includes: [U/s 2(22)]
(1) A directorship or any other office involved in the management of a company;
(2) A position entitling the holder to a fixed or ascertainable remuneration; or
(3) The Holding or acting in any public office.

Example: Hafiz Bilal Rana is an employee of a company. He comes to office only 4 hours in a day. He received
Rs.18,000 salary from the company. Explain with reasons that he is considered as an employee of a company or
not.

Solution: Although he is a part time employee of a company, however the amount so received shall be treated as
salary income for the year.
Basic salary:
For the purposes of tax, basic salary means any consideration received or receivable as basic salary. However, for
the purposes of retirement benefits, basic salary includes dearness allowance.
2. Salary includes the following [U/s 12(2)(a), (b), (c) and (d)]
(a) Pay / wages or other remuneration, leave pay / leave encashment, overtime, bonus, commission and fee;
(b) Work condition supplements;

Conceptual Approach to Taxes 85


Income From Salary Chapter-07

(c) Allowances i.e. cost of living, subsistence, rent, utilities, education, entertainment or travel allowance excluding
any allowance solely expended for office purpose;
(d) Any expenditure incurred by an employee but paid or reimbursed by the employer other than for office
purpose.
(e) Profits in lieu of salary, perquisites, gratuity, pension or annuity, or any supplement to a pension or annuity,
benefit on account of employee share scheme, tax on salary paid by the employer and
(f) In the case of other assets given for use only, rental value or depreciation charged by the employer is the
taxable benefit for the employee e.g. TV is provided by the employer only for the use of employee.
3. Salary chargeable to tax:
Taxability of salary: [U/s 73]
Any income taxed on receipt basis shall not be taxable again on accrual basis and vice versa. Similarly if any
expenditure is deductible on due basis then the same shall not be deducted when it is paid and vice versa.
Salary paid by Private Companies [U/s 110]
The Commissioner Inland Revenue instead of charging tax on salary income, on cash basis may opt the accrual
basis in case of an employee of a Private Company. This treatment shall be applied where the employee in order to
avoid a higher tax rate, could have entered into agreement with his employer for payment of salary in a subsequent
year when his other income may be on lower side.
Suppose if there is income from other sources in the current year along-with income from salary and both as part of
total income may result into higher rate of tax in the current year therefore the employee with the consent of employer
may defer his full or part salary to be offered for tax when he will not have any income from other sources that will
result into lower rate of tax in the subsequent tax year.
Treatment of arrear salary received: [U/s 12(7) and (8)]
Employee may avail this option in writing to the Commissioner Inland Revenue to be taxed on accrual basis instead
of cash basis. It is allowed when the following conditions are met:
The salary is paid to an employee in arrears;
After including the arrears salary in the current years income the rate of tax is increased.
The declaration shall be made by the due date for furnishing of employees return of income for the tax year in
which amount was received or by such later date as the Commissioner Inland Revenue may allow.
Example:
In tax year 2015 Mr. Amir was paid salary for 8 months i.e. Rs. 800,000. In tax year 2016 he was paid salary for 16
months (including salary of 4 months of tax year 2015) i.e. Rs.1,600,000. Compute tax payable by him in tax year
2016.
Solution:
Mr. Amir
Computation of taxable income and tax liability:
Option A by including the arrear salary in current year income

TAX YEAR 2015


Rs.
Income from salary 800,000
Computation of tax liability
Tax on Rs. 800,000 [Rs.17,500 + 10% (800,000 750,000) 22,500

TAX YEAR 2016


Rs.
Income from salary 1,600,000
Computation of tax liability
Tax on Rs. 1,600,000 [92,000 + 15% x (1,600,000 - 1,500,000)] 107,000

Option B by offering the arrear salary in the respective year

TAX YEAR 2015


Rs.
Income from salary 1,200,000
Computation of tax liability
Tax on Rs. 1,200,000 [17,500 + 10% x (1,200,000 750,000) 62,500

86 Conceptual Approach to Taxes


Income From Salary Chapter-07

TAX YEAR 2016


Rs.
Income from salary 1,200,000
Computation of tax liability
Tax on Rs. 1,200,000 [14,500 + 10% x (1,200,000 750,000)] 59,500

Total tax payable as per option A (107,000 + 22,500) 129,500


Total tax payable as per option B (62,500 + 59,500) 122,000
Taxpayer shall pay tax as per option B with less tax payable.
Salary treated to have been paid and received: [U/s 12(5) and 69]
(A) A person (including an employer) shall be treated as having paid an amount, benefit or perquisite if it
is: [U/s 12(5)]
Paid by the person that may be a present, past or prospective, or an associate of the payer or a third party
under an arrangement with the payer.
(B) A person (including an employee) shall be treated as having received an amount, benefit or perquisite
if it is: [U/s 69]

Actually received by the person.


Applied on behalf of the person,
At the instruction of the person or under any law.
Made available to the person.
4. Profit in lieu of or in addition to salary [U/s 12(2)(e)]
The amount of any profit in lieu of salary or wages including any amount received:
(A) as compensation for the termination of employment paid voluntarily or under an agreement including
payments under golden handshake and loss of employment;
(B) payments from any fund other than related to the employees contribution;
(C) pension or annuity, or any supplement to a pension or annuity. [U/s 12(2)(f)] and
(D) any amount chargeable to tax under an employee share schemes u/s14.[U/s 12(2)(g)]
The amount of any profit in addition to salary or wages including any amount received
(A) as consideration at the time of entering into an employment agreement;
(B) as consideration on agreement regarding any conditions of employment or any changes in the employment
conditions;
(C) as consideration on a restrictive covenant agreement with any past, present or prospective employment;
4.1 Tax treatment on account of golden handshake payments or loss of employment on termination of
employment: [U/s 12(2)(e)(iii) and12(6)]
Where an employee received amounts on account of above then he may opt any one of the following options
available to him:
(A) To offer the same as income in the year of receipt; or
(B) The amount to be taxed at the rate in accordance with the following formula:
Preceding three years total tax on total taxable income under NTR
x 100
Total taxable income of preceding three tax years under NTR

Important note U/s 12(8):


The declaration shall be made by the due date for furnishing of employees return of income for the tax year in which
amount was received or by such later date as the Commissioner Inland Revenue may allow.
Example: Mr. Arif received taxable salary of Rs. 600,000 in tax year 2016. He retired on 1st day of May, 2016 and
received an amount of Golden Handshake of Rs. 900,000. If the following further information has been provided you
are required to compute the tax payable by him.
Last 3 years tax liability under normal tax regime 145,000
Last 3 years taxable income under normal tax regime 1,200,000

Conceptual Approach to Taxes 87


Income From Salary Chapter-07

Solution: A person who receives Golden handshake has two options available to him. He may opt to include this
amount in his taxable income or he may opt to tax this amount as a separate block of income.
Mr. Arif
Tax year 2016
Computation of taxable income and tax liability: Rs.
Option 1 (included the golden handshake in the current year income)
Income from salary:

Salary 600,000
Golden handshake 900,000
Taxable income 1,500,000
Computation of tax liability:
Tax on Rs. 1,500,000 [79,500 + 12.5% x (1,500,000 1,400,000)] 92,000
Option 2 (offered on the basis of last three years average rate)
Income from salary:
Salary 600,000
Taxable income 600,000
Computation of tax liability:
Tax on Rs. 600,000 [2,000 + 5% x (600,000 500,000)] 7,000
Tax on golden handshake
[Rs. 900,000 x 12.08% as per note attached) 108,720
Tax liability 115,720
Rate for golden handshake = 145,000/1,200,000 x 100 = 12.08%
Lower tax is payable in option 1, hence same shall be opted by the tax payer.
4.2 Computation of tax on tax [u/s 12(3) & (4)]
Any tax liability of employee paid by employer on behalf of employee shall be added in the income of the employee
by such amount.
The grossed up amount may be calculated under the following cases.
Where normal rates are feasible:
Example: An employee of a company has received total salary of Rs. 800,000. As per appointment letter the
employer shall pay Rs. 22,500 as tax free salary to employee. Compute the tax liability for the tax year 2016.
SOLUTION
Taxable salary excluding tax paid by the employer 800,000
Tax paid by employer 22,500
Total taxable income 822,500

Computation of taxable income and tax liability

[14,500 + 10% x (822,500 750,000)] 21,750


Less: paid by employer 22,500
Balance tax excess paid (750)

Important note: Tax on tax amount depends on the terms and conditions settled between employer and employee.
The salary income will be charged to tax on its gross value after deduction of exempt salary income or allowances
and perquisites. However, no expense or other deduction shall be allowed with respect to any expenditure incurred in
deriving salary income. Section 12(4) of the ordinance states that:
No deduction shall be allowed for any expenditure incurred by an employee in deriving amounts chargeable to
tax under the head Salary.
Marginal perquisites [Clause 53A of Part I of Second Schedule]
Some employers provide concessional benefits to their own facilities e.g. a Hospital may allow its employee free
medical treatment or concessional medical facility or a school allow its children of its employees to get free or
concessional fee and a railway, bus or air transport providing free or concessional transport facility to its employees.
Marginal perquisites are exempt except free or concessional passage provided by transporters including airlines to
its employees (including the members of their household and dependants). For this purpose, we have to take the
market value of facility provided to employees and add the same in the salary income of employee after deducting
any cost borne by the employee.

88 Conceptual Approach to Taxes


Income From Salary Chapter-07

5. Value of Perquisites [U/s 13]


The value of perquisites to the extent of provided by an employer to his employee shall include the following:
(1) Motor vehicle used wholly or partly for private purposes
(2) Salary of housekeeper, driver or other domestic assistant.
(3) Fair market value of utilities.
(4) Actual amount waived off or obligation discharged by employer on behalf of the employee.
(5) Transfer of property or services provided
(6) Facility of accommodation or housing
(7) Interest free loan or loan at concessional rate
(8) In addition to the above the value of any other benefit.
6. Non applicability of this section [U/s 13(2)]
This section shall not apply to any amount of allowances provided by employer and expenditure incurred by
employee for the performance of official duties of employment.
7. Valuation of perquisites, allowances and benefits
7.1 Valuation of conveyance Rule-5 and [U/s 13(3)]
In case of motor vehicle provided by an employer to his employee partly or wholly for personal use, then the amount
chargeable to tax under the head salary income shall be computed according to following rules:
It is worthwhile to mention here that where the motor vehicle provided by employer has only been used for the
business purposes then there will be no treatment of the same in the hands of the employee.
(A) Partly for personal and business use
5% of the cost of vehicle or the fair market value (excluding mark up) at the commencement of the lease where
vehicle is taken on lease by employer.
(B) For personal use only
10% of the cost of vehicle or the fair market value (excluding mark up) at the commencement of lease
where vehicle is taken on lease by employer.
Example
Following are the details of income of Hamid Sarfraz for the financial year ended June 30, 2016, who is employed
with a company as Senior Manager.
Salary income
Pay Rs. 60,000 per month
House rent allowance Rs. 27,000 per month
Utilities Rs. 8,000 per month
He was provided with a company maintained car of 800CC. Compute the taxable income of Hamid Sarfraz for the
year if.
1. The cost of the car to the company was Rs. 500,000 and car was provided for personal use only.
2. The cost of the car to the company was Rs. 500,000 and car was provided for business use only.
3. The cost of the car to the company was Rs. 500,000 and car was provided partly for personal and partly for
business use.
4. The car is acquired by company on lease of Rs.850,000 and the FMV of the car is Rs.500,000 and car was
provided for personal use only.
Solution
Case 1 Case 2 Case 3 Case 4
Rs. Rs. Rs. Rs.

Salary (60,000 x 12) 720,000 720,000 720,000 720,000


House Rent allowance (27,000 x 12) 324,000 324,000 324,000 324,000
Utilities (8,000 x 12) 96,000 96,000 96,000 96,000
Car provided for:
- Personal use only (500,000 x 10%) 50,000 - - -
- Business use only - - - -
- Business and personal use (500,000 x 5%) - - 25,000 -
- Personal use only (500,000 x 10%) - - - 50,000
Taxable income 1,190,000 1,140,000 1,165,000 1,190,000

Conceptual Approach to Taxes 89


Income From Salary Chapter-07

Notes:
1. Nothing is included in taxable income when conveyance is provided for business use only.
2. In cases of lease, FMV shall be used.
Conveyance allowance: Amount received as conveyance allowance (other than car) in cash shall be totally taxable
and shall be included in the salary income of the employee.
Important note: It is worthwhile to mention here that where the motor vehicle provided by employer has only been
used for the business purposes then there will be no treatment of the same in the hands of the employee.
Example: Following are the details of income of Hamid for the financial year ended June 30, 2016, who is employed
with a company as Senior Manager.
Salary income
Pay Rs. 60,000 per month
House rent allowance Rs. 27,000 per month
Utilities Rs. 8,000 per month

He was provided conveyance allowance of Rs. 50,000 for a year. No car is provided by the employer. Compute the
taxable income and tax liability of Hamid for the Tax year 2016.

Solution:
Salary 60,000 x 12 720,000
House Rent allowance 27,000 x 12 324,000
Utilities 8,000 x 12 96,000
Conveyance 50,000
Taxable Income 1,190,000

Tax Liability under normal rates:


[14,500 + 10% x (1,190,000 750,000)] 58,500

7.2 Salaries of servants provided by employer Rule 3 [U/s 13(5)]


Salaries of housekeeper, gardener, driver or other servants of employee paid by the employer in the tax year for
services rendered by such persons for the employee shall be included in the salary income of the employee.
7.3 Utilities provided by employer [U/s 13(6)]
The amount of utilities such as telephone facility provided by an employer then the fair market value of utilities shall
be included in the salary of the employee. Utilities benefit may be provided by employer free or at concessional rate
or utility allowance in cash.
7.4 Actual amount waived off or obligation discharged by employer on behalf of the employee [U/S 13(9) & (10)]
Actual amount waived off and obligation discharged by employer on behalf of employee shall be included in the
salary income of the employee.
7.5 Property is transferred or services are provided by employer [u/s 13(11)]
Where in a tax year, property is transferred or services are provided by an employer to an employee, the amount
included in the salary income of the employee shall be the fair market value of such benefit at the date of transfer or
providing of services:
7.6 Valuation of Accommodation - Rule-4 [u/s 13(12)]
(1) Facility of accommodation or Housing
Where in the tax year, accommodation or housing is provided by an employer to his employee, the value of
such accommodation shall be the amount that would have been paid by the employee if the accommodation
was not provided to him.
The value of accommodation under this rule shall not be less than from the 45% of minimum of time scale or
basic pay where minimum time scale not given.
Important note: In short the following meaning may be derived from this rule:
(a) Firstly compute 45% of minimum of time scale or basic pay (where MTS not given) and 30% in case of
Mufasal areas then
(b) Compute Fair market value of accommodation
First preference to calculation (a) however where calculation in (b) above is more than (a) then the same shall
be added in the salary income of the employee.
Example: Miss Hira, an employee of HBL as managing director has provided the following details of her
expected income and expenses for the year ending June 30, 2016.

90 Conceptual Approach to Taxes


Income From Salary Chapter-07

Basic Salary Rs. 1,800,000 per year


Dearness allowance 10% of basic salary
Bonus Rs. 50,000
Conveyance allowance Rs. 50,000
Leave fare assistance Rs. 60,000
The company disbursed on July 1, 2015 to her, Rs. 3 million interest free loan to be recovered from the final
dues on retirement.
Company has paid Rs. 850,000 as annual rent for the accommodation provided to Managing Director.
She has been provided with a company maintained car for business and personal use. The purchase price of
the car is Rs. 12 million. The company also pays salary to official driver @ Rs. 8,000 per month. You are
required to compute her taxable income and tax thereon and show all workings and assumptions.
Solution:
Hira (resident)
Computation of the income and tax liability thereon
For the tax year 2016 Rs.

Basic Salary 1,800,000


Dearness allowance 10% of basic salary 180,000
Bonus 50,000
Conveyance allowance 50,000
Leave fare assistance 60,000
Interest fee loan 3,000,000 x 10% 300,000
Accommodation for house:
Higher of 45% of basic salary or fair market value
45% of basic salary 810,000 or Rs.850,000 850,000
Car 12,000,000 x 5% 600,000
Taxable Income 3,890,000

Tax Liability under normal case:


[472,000 + 25% x (3,890,000 3,500,000)] 569,500

Note: There will be no treatment of salary of official driver @ Rs. 8,000 per month paid by the Company as it
was for the discharge of official performance.
(2) Treatment of House rent allowance:
Any amount provided as house rent allowance (other than accommodation provided by the employer) in cash,
the total amount shall be taxable and shall be included in the salary income of the employee.
Example: Following are the details of income of Mr. Afzal for the financial year ended June 30, 2016, who is
employed with a company as Senior Manager.
Salary income
Pay Rs. 60,000 per month
House rent allowance Rs. 27,000 per month
Utilities Rs. 8,000 per month
He was provided conveyance allowance of Rs. 50,000 for a year. No car is provided by the employer.
Compute the taxable income and tax liability of Mr. Afzal for the Tax year 2016.
Solution:
Salary 60,000 x 12 720,000
House Rent allowance 27,000 x 12 324,000
Utilities 8,000 x 12 96,000
Conveyance 50,000 50,000
Taxable Income 1,190,000
Tax Liability under normal rates:
[14,500 + 10% x (1,190,000 750,000)] 58,500

7.7 Treatment of any other perquisite [U/s [13(13)]


Where any other perquisite not covered in section 12 is provided by an employer to an employee, the value of such
perquisite at fair value (excluding employee cost to acquire such benefit) shall be included in the salary income of the
employee.

Conceptual Approach to Taxes 91


Income From Salary Chapter-07

7.8 Loan from employer to employee on or after 01-07-2002 [u/s 13(7)]


Where a loan is made, by an employer to an employee either interest free or at concessional rate, the amount to be
included in the salary shall be computed according to the table given below:

Loan provided at: Amount to be included in the salary of the employee:


Interest free loan, or no profit on interest is charged Interest computed at benchmark rate.

0%, less than the bench mark rate, equal to Interest computed at benchmark rate, less actual amount
benchmark rate. paid by employee on account of loan.
Higher than the benchmark rate OR Amount of loan is
Nothing shall be included.
Rs. 500,000 or less than Rs. 500,000.

The benchmark rate in tax year 2003 was 5% p.a. (with 1% increase in each following tax year and in 2012 it was
14% p.a. as defined in section 12(14). From tax year 2013 to onwards benchmark rate has been capped at 10%.
However, with effect from tax year 2011 such benefit shall not be taxable in cases where such benefit is extended
by the employer due to the waiver of interest by such employee on his accounts (e.g. provident fund etc) maintained
with the employer.
Example: Inaam is an employee in a group of companies. He derived following income during the income year July
1, 2015 to June 30, 2016.
Salary income per month Rs.
(i) Basic salary Rs. 20,000
(ii) House rent allowance Rs. 8,000
(iii) Utility allowance Rs. 1,000
He is provided with a 1,000CC car, which is partly used for companys business (Cost of car is Rs. 1,000,000). He
has also obtained loan from the employer. Calculate his taxable income in the following cases:
a) Amount of loan is Rs. 500,000 and interest charged by the employer is 7% p.a.
b) Amount of loan is Rs. 600,000 and interest charged by the employer is 0% p.a.
c) Amount of loan is Rs. 600,000 and interest charged by the employer is 6% p.a.
d) Amount of loan is Rs. 600,000 and interest charged by the employer is 11% p.a.
Solution:

Computation of taxable income Case (a) Case (b) Case (c) Case (d)
Rs. Rs. Rs. Rs.

Basic salary (20,000 x 12) 240,000 240,000 240,000 240,000


House rent allowance (8,000 x 12) 96,000 96,000 96,000 96,000
Utility allowance (1,000 x 12) 12,000 12,000 12,000 12,000
Company car benefit (Rs.1,000,000 x 5%) 50,000 50,000 50,000 50,000
Interest free loan (Note 1) - 60,000 24,000 -
Taxable income 398,000 458,000 422,000 398,000

Note 1: (Interest free loan)

Case a:
When the amount of loan is Rs. 500,000 or less, nothing shall be included in taxable income.

Case b:
Interest calculated at bench mark rate shall be included in taxable income i.e. 600,000 x 10%.

Case c:
Amount to be included in taxable income is calculated as follows:

Interest at bench mark rate (600,000 x 10%) 60,000


Amount to be included in taxable income 24,000

Case d:
When interest rate is higher than benchmark rate, nothing shall be included in taxable income

92 Conceptual Approach to Taxes


Income From Salary Chapter-07

7.9 Loan for the acquisition of asset or property [U/s 13(8)]


Where the employee uses a loan obtained from his employer for the acquisition of any asset or property producing
income chargeable to tax under any head of income, (it is considered that an amount of interest equal to benchmark
rate on this loan is paid). Deduction provided against this loan is provided as follows:
(1) Where the loan is interest free or interest is charged at lower than the benchmark rate, the amount equal to
benchmark rate shall be allowed as deduction to employee against income from such asset.
OR
(2) If the interest charged at rate higher than benchmark rate, total amount of interest charged shall be allowed as
deduction against income from such asset.
Example:
Mr. Zahid is an employee of ABC Textiles Limited. His taxable salary income for tax year 2016 is Rs. 800,000. On 1st
July, 2015 he acquired loan of Rs. 600,000 (at 8% interest rate) from his employer. Compute his taxable income for
tax year 2016 in following situations:
a. Loan used for purchase of plot against which he received ground rent of Rs. 400,000 by letting out the same
under 99 years lease agreement.
b. Loan used for purchase of house. He received rent of Rs. 400,000 by letting out this house.
c. Assume that the interest rate charged by employer is 20%. Loan used for purchase of plot against which he
received ground rent of Rs. 400,000 by letting out the same under 99 year lease agreement.
Solution:

Mr. Zahid
Tax year 2016
Computation of taxable income Case (a) Case (b) Case (c)
Rs. Rs. Rs.
Income from salary:
Taxable salary (given) 800,000 800,000 800,000
Add: 2% of loan of Rs.600,000 as income of the employee 12,000 12,000 -
Income from salary 812,000 812,000 800,000

Income from other sources:


Income from open plot as ground rent 400,000 - 400,000
Less: Interest on loan
Higher of actual interest paid or interest at benchmark rate (60,000) - (120,000)
340,000 - 280,000
Income from property:
Rent of house - 400,000 -
(Property income is chargeable to tax under normal tax regime (NTR)
and deductions are also allowable against income from property hence, (80,000)
in this case,1/5th repair allowance and interest shall be deducted.) (60,000)
Less 1/5th fixed repair allowance
Mark up on loan
Net property income - 260,000 -
Taxable income 1,152,000 1,072,000 1,080,000
8. Employee share schemes [U/s 14]
"Employee share scheme" means any agreement or arrangement under which a company may issue shares in the
company to an employee of the company or an employee of an associated company or the trustee of a trust or an
employee of an associated company.
Right or option to acquire shares [U/s 14(1)]:
The value of a right or option to acquire shares under an employee share scheme granted to an employee shall
not be chargeable to tax because no benefit in the form of salary shall be received by the employee unless disposal
of right or option.
Example: Explain the taxability of the following under the head salary with reference to employees share schemes.
(a) Grant of a an option
(b) Exercise of an option granted in prior year

Conceptual Approach to Taxes 93


Income From Salary Chapter-07

Solution:
(a) Nothing shall be included in his salary income until disposal off option.
(b) The exercise of an option granted in prior year is chargeable to tax under the employee share scheme as the
difference between the FMV of the shares less consideration given by the employee for purchase of shares
and the cost of acquisition of right.
Disposal of right or option to acquire shares [U/s 14(5)]:
Where an employee disposes off a right or option to acquire shares under an employee share scheme, the amount
chargeable to tax to the employee under the head "Salary" for that year shall include the amount of any gain made on
the disposal computed as under:
Consideration received for the disposal of the right or option xxx
Less: Cost of right or option xxx
If balance is positive then add the same in salary income however if the answer is negative then the same shall be
ignored because deduction of expenses against salary are not allowable.
Shares issued without limitation or restriction [U/s 14(2)]
Where an employee is issued shares under an employee share scheme, the amount chargeable to tax to the
employee under the head "Salary" for that year shall include the fair market value of the shares at the date of
issue, as reduced by any consideration given by the employee for the shares including amount if any given for the
grant of a right or option to acquire the shares.
Shares issued subject to limitation or restriction [U/s 14(3)]
If shares issued to an employee under an employee share scheme are with restriction on the transfer of shares
(a) no amount shall be chargeable to tax to the employee under the head "Salary" until the earlier of the time the
employee has a free right to transfer the shares or disposes off the shares; and
(b) the amount chargeable to tax to the employee shall be the fair market value of the shares at the time the
employee has a free right to transfer the shares or disposes off the shares, as reduced by any consideration
given by the employee for the shares including amount if any given for the grant of a right or option to acquire
the shares.
Cost of shares [U/s 14(4)]
The cost of the shares to the employee shall be the sum of -
(a) the consideration, if any, given by the employee for the shares;
(b) the consideration, if any, given by the employee for the grant of any right or option to acquire the shares; and
(c) the amount chargeable to tax under the head "Salary".
Example: Mr. Kamran was issued shares under employee share scheme on July 01, 2015. Information regarding this
is given below:
- FMV of shares on date of issue Rs. 50,000
- FMV of shares on June 30, 2016 Rs. 60,000
- FMV of shares on January 01, 2016 Rs. 55,000
- Amount paid by him for shares Rs. 30,000
- Amount paid by him for right Rs. 1,000
He disposed of all the shares on June 30, 2016.
Required: Calculate the incomes chargeable to tax under the heads income from salary and capital gain in the
following cases:
(a) Shares were issued without any restriction on transfer
(b) Shares were issued with restriction that shares cannot be transferred before January 01, 2016
Solution: Case (a) Case (b)
Rs. Rs.
Income from salary
FMV of shares (Note 1) 50,000 55,000
Less: Amount paid for:
Shares 30,000 30,000
Right 1,000 1,000
31,000 31,000
Amount chargeable to tax under the head "Income from salary" 19,000 24,000

94 Conceptual Approach to Taxes


Income From Salary Chapter-07

Capital gain
Higher of FMV or actual amount received against shares at disposal time (Note 2) 60,000 60,000
Less: cost of shares:
Amount paid for shares and right (calculated above) 31,000 31,000
Amount taxable under head salary (calculated above) 19,000 24,000
50,000 55,000
Capital gain 10,000 5,000

Note 1:
Case (a) As shares were issued without any restriction on transfer, FMV on issue date shall be taken into account.

Case (b) As shares were issued with restriction on transfer, FMV on the date the employee has free right to transfer
the shares shall be taken into account.

Note 2: In case of employee share scheme FMV at the time of disposal shall be taken into account and the same
shall not be compared with actual consideration received.
Retirement benefits
Retirement benefits available to taxpayer are as follows:
Gratuity (treated as pay)
Super-annuation Fund (treated as Profit in lieu of or in addition to salary)
Provident fund (treated as Profit in lieu of or in addition to salary)
Gratuity
Gratuity is a sum which is paid at the discontinuation of the employment to the employee computed by applying the
total number of years of service of the employee with the determined/agreed amount of salary. Gratuity conditional
exemptions are as under:
Clause 13 Part I, 2nd Schedule to the Income Tax Ordinance, 2001:
Any income representing any payment received by way of gratuity or commutation of pension by an employee on his
retirement or, in the event of his death, by his heirs as does not exceed-
(i) in the case of an employee of the Government, a Local Government, a statutory body or corporation
established by any law for the time being in force, the amount receivable in accordance with the rules and
conditions of the employee's services;
(ii) any amount receivable from any gratuity fund approved by the Commissioner in accordance with the rules in
Part III of the Sixth Schedule;
(iii) in the case of any other employee, the amount not exceeding Rs.200,000 receivable under any scheme
applicable to all employees of the employer and approved by the Board for the purposes of this sub-clause;
and
(iv) in the case of any employee to whom sub-clause (i), (ii) and (iii) do not apply, 50% of the amount receivable or
Rs.75,000, whichever is the less:
Provided that nothing in this sub-clause shall apply -
(a) to any payment which is not received in Pakistan;
(b) to any payment received from a company by a director of such company who is not a regular employee of
such company;
(c) to any payment received by an employee who is not a resident individual and to any gratuity received by an
employee who has already received any gratuity from the same or any other employer.
CBR vide its Circular No. 17 of 1959 and Circular No. 16 of 1967 provided the option to a taxpayer to be assessed
at the rates applicable for the current year or at the average rate of tax of his last three years income.
The following further instructions are also attached with this option:
1. Where average rate for last 3 years worked out at Nil, then no tax would be payable by the said taxpayer.
2. Where the employee was taxable in 3 preceding years but due to his absence from Pakistan, he was not
charged to tax, gratuity received by him will be taxed at the average rate of tax at which he would have been
liable if he was a resident person in 3 preceding years.
3. Gratuity income will be ignored at the time of computation of taxable income of a deceased person, however
the same may be added to the income of his legal heirs as income from other sources.

Conceptual Approach to Taxes 95


Income From Salary Chapter-07

Example:
ABC Ltd. has paid gratuity amounting to Rs. 1,075,000 to Mr. A in addition to the taxable salary of Rs.2,500,000 in
the tax year 2016. The past 3 years assessed tax results of his assessment are as under:
Tax year Taxable income Tax Liability
2015 1,800,000 177,800
2014 2,400,000 435,600
2013 1,400,000 155,628
The ABC Ltd. maintains an unapproved gratuity Fund. Mr. A is interested to know the different options available to
him for taxation of gratuity.
Solution:
Option No. 1 Taxation under Normal Manner:
Particulars Rs.
Taxable Salary 2,500,000
Gratuity (Rs.1,075,000 75,000) (Lower of Rs.75,000 or 50% of Rs.1,075,000) 1,000,000
Taxable income 3,500,000

Tax on Rs. 3,500,000 [359,500 + 22.5% x (3,500,000 - 3,000,000)] 472,000


Option No. 2 Taxation under Average rate of last 3 years income
Particulars
Taxable income under option 2 as computed above (without gratuity received) 2,500,000

Tax on salary
Tax on Rs. 2,500,000 [137,000 + 17.5% x (2,500,000 - 1,800,000)] 259,500

Tax on (1,075,000 less Rs. 75,000) Rs. 1,000,000 x 769,028 / 5,600,000 N-1 137,326
Total tax 396,826
N-1 Determination of average rates of tax
Tax year Taxable income Tax liability (say)
2015 1,800,000 177,800
2014 2,400,000 435,600
2013 1,400,000 155,628
Total 5,600,000 769,028
Note: Tax under option 2 is lower than the option 1 hence it is more beneficial to opt for the said arrangements.
Pension
Pension received is exempt under clause 8, 9, 12, 13, 16 & 17 Part I Second Schedule to the Ordinance subject to
certain exceptions. Gratuity & commutation of pension both enjoys exemption under clause13 of Part I Second
Schedule to the Ordinance as discussed it the preceding Para. However, a taxpayer receiving pension has not been
allowed any option for taxation at average rate of last three years income. The most important feature of these
payments is that the most of pensions are exempt from levy of tax.
The various exemptions in addition to clause 13 are as under.

Under clause Particulars Exemption


8 Pension received by citizen of Citizen of Pakistan from a former employer, other
Pakistan. than where the person continues to work for the
employer (or an associate of the employer).
Provided that where the person receives more than
one such pension, the exemption applies only to
the higher of the pensions received.
9 Pension of Govt. & Armed forces Received in respect of services rendered by a
employees. member of the Armed Forces of Pakistan or Federal
Government or a Provincial Government;
Granted under the relevant rules to the families and
dependents of public servants or members of the
Armed Forces of Pakistan who die during service.

96 Conceptual Approach to Taxes


Income From Salary Chapter-07

12 Commutation of pension from Govt. or Payment in the nature of commutation of pension


under any pension scheme approved received from Government or under any pension
by FBR. scheme approved by the Board for the purpose of
this clause
13 Gratuity & commutation of pension
from
recognised fund As discuss above in gratuity
un recognised fund
scheme approved by FBR
other than above
16 & 17 Pension & income of dependent of Families and dependents of the "Shaheeds"
Armed forces & civil forces. belonging to Pakistan Armed Forces & Civil Armed
(Exemptions u/c 16 and 17 are not Forces of Pakistan from the (a) Special family
directly related to salaried persons) pension (b) Dependents pension & (c) Childrens
allowance

Annuities
Annuities or any supplement to annuity are taxable as salary even it is paid voluntarily without any contractual
obligation of the present or ex-employer. All the annuities are taxable in the normal manner as clause (20) of Part I of
2nd Schedule has been omitted from tax year 2016.
However the tax credit aspect on the annuities is discussed under section 63 of the Ordinance where the same has
been allowed on the contributed or premium paid on a contract of annuities. The said tax credit is discussed in detail
in the chapter of tax credits.
Superannuation fund
The income from the superannuation fund is taxable to the extent of employers contribution and the interest credited
thereon under the following Clause:
Clause 25 Part I, 2nd Schedule to the Income Tax Ordinance, 2001:
Any payment from an approved superannuation fund made on the death of a beneficiary or in lieu of or in
commutation of any annuity, or by way of refund of conditions on the death of a beneficiary.
The plain reading of the aforesaid provision of the ordinance clearly shows that taxation of superannuation Fund is
totally in the same manner as laid down for the gratuity under clause 13 Part I Second Schedule to the Ordinance.
Benevolent fund
Sum paid out of employee Benevolent Fund is taxable in the normal manner, however, clause 24 Part I Second
Schedule to the Ordinance deals with the exception to the aforesaid law in the following manner:
Any benevolent grant paid from the Benevolent Fund to the employees or members of their families in accordance
with the provisions of the Central Employee Benevolent Fund and Group Insurance Act, 1969.

Provident fund:
9. Introduction:
From salary of employee, a certain amount is deducted every month on account of provident fund. Employer also
contributes the same amount in the provident fund. At the time of retirement of employee, accumulated balance in the
provident fund consisting of employees contribution, employers contribution and interest on fund is paid to employee
for his benefit.
Types of Provident Funds:
There are three types of provident funds
Provident Fund formed under the Provident Fund Act, 1925.
Recognized Provident Fund under the Income Tax Ordinance, 2001.
Unrecognized Provident Fund under the Income Tax Ordinance, 2001.
9.1 Provident Fund Formed under Provident Fund Act, 1925
A fund formed under the provisions o the Provident Act, 1925 is also named as Statutory Fund. Such type of fund is
generally formed by the Federal Government, Semi Government Institutions and Local Authorities etc.
In this type of fund subscription or deposits of any class or classes of employees are received and held on their
individual accounts, and includes any contributions and any interest or increment accruing on such subscriptions,
deposits or contributions under the rules of the Fund. [The Provident Funds Act, 1925 Section 2(d)]

Conceptual Approach to Taxes 97


Income From Salary Chapter-07

9.2 Recognized Provident Fund and Unrecognized Provident Fund:


Any provident fund recognized by CIR is called recognized provident fund and if it is not recognized by the CIR, then
it is called unrecognized provident fund.
10. Taxation of Provident Fund:
Provident Fund Formed under Provident Fund Act, 1925
Any amount received from Provident fund to which the Provident Funds Act, 1925 applies is fully exempt
from tax. [Clause 22 of Part I 2nd Schedule of Income Tax Ordinance, 2001].
Recognized Provident Fund under the Income Tax Ordinance, 2001
In case of recognized provident fund, following shall be included in the income of employee in every tax year
[Clause 3 of Part I of sixth schedule to Income Tax Ordinance, 2001].
(a) Contributions made by the employer less lower of one-tenth of the salary or Rs.100,000; and
(b) Interest credited on the balance of provident fund less higher of:
1. 1/3 of salary (basic salary + dearness allowance); OR
2. Interest calculated @ 16% p.a. (Reference note attached)
Although under the aforesaid clause there is no rate of interest has been given however the Federal
Government vide its SRO No. 1097(I) 84 dated 27-12-1984 fixed the rate of interest at 16% for the purposes of
interest calculation.
Accumulated balance due and becoming payable to an employee from a recognized provident fund is
exempt. [U/c 23 of Part I 2nd Schedule to the Income Tax Ordinance, 2001]
Unrecognized Provident Fund under the Income Tax Ordinance, 2001
An unrecognised provident fund is the fund which is neither recognised nor statutory. Income of the said
provident fund is taxable to the extent defined in section 12(2)(e) which states that:
from a provident fund or other fund, to the extent to which the amount is not a repayment of contributions made
by the employee to the fund in respect of which the employee was not entitled to a deduction;
The following facts are evident from the above:
(a) The employee contribution (already taxed by including in the employee gross salary income) however
interest thereon is totally taxable in the hands of employee.
(b) No tax shall be charged on amount contributed by the employer and interest thereon because the same
are taxable on receipt of accumulated balance due to aforesaid section and non coverage of the same
under Clause 23 of Part I 2nd Schedule to the Income tax Ordinance, 2001.
The following table shows the above position of various Provident Funds:

PF formed under Recognized Provident Unrecognized Provident


Particulars
Provident Fund Act, 1925 Fund Fund
Employees Already included in taxable Already included in taxable Already included in taxable
contribution salary income of salary income of salary income of employees
employees employees
Employers contribution Exempt under clause 22 of Employers contribution Not taxable at the time of
Part I of 2nd Schedule to the less lower of 1/10th of the contribution
Income tax Ordinance, salary (Basic pay +
2001 dearness allowance) OR
Rs. 100,000
Interest credited for the Exempt under clause 22 of Interest credited for the Not taxable at the time of
year Part I of 2nd Schedule to the year less higher of 1/3rd of contribution
Income tax Ordinance, the salary (Basic pay +
2001 Dearness allowance) OR
the interest amount
computed at 16% p.a.
Payment of Exempt under clause 22 of Exempt under clause 23 of Employers contribution &
accumulated balance Part I of 2nd Schedule to the Part I of 2nd Schedule to the interest on accumulated
Income tax Ordinance, Income tax Ordinance, balance (including interest
2001 2001 on employees contribution)
is taxable on receipt.

98 Conceptual Approach to Taxes


Income From Salary Chapter-07

Important note: There is no treatment of employees contribution to any provident fund, as tax on the same has
already been paid by offering gross salary in his income tax return by the taxpayer.
Example: Aman Ali is an employee of Mano Limited and the amounts provided to her were as follows:
Basic salary 250,000 Dearness allowance 25,000
House rent allowance 80,000 Conveyance allowance 50,000
Employees contribution 30,000 Employers contribution 35,000
Accumulated balance 650,000 Interest credited @ 18%
Required: Compute taxable income of Aman Ali according to income tax rules under the following situations.
(a) If all contributions of Provident Fund are in Recognized Provident Fund.
(b) If all contributions of Provident Fund formed under Provident Fund Act, 1925.
(c) If all the contributions of Provident Fund are in Unrecognized Fund.

Solution for case (a): Rupees Rupees

Basic salary 250,000


Dearness allowance 25,000
House rent allowance 80,000
Conveyance allowance 50,000

Employees contribution (already included in salary) 30,000 -

Employers contribution 35,000


Less: 10% of salary (note attached) (275,000 x 10%) 27,500 7,500
Or 100,000 whichever is lower shall be deducted

Interest credited (Rs. 650,000 x 18%) 117,000


Less: interest credited @ 16% (Rs. 650,000 x 16%) (104,000)
Or 1/3rd of salary Rs. 275,000 / 3, hence higher is 16%
Amount included in salary income 13,000
Taxable income 425,500

NOTE: Salary for the computation of provident fund is equal to:

Salary = Basic salary 250,000 + dearness allowance 25,000= 275,000


Solution for case (b) and (c):
The answer in both (b) and (c) shall be same as nothing shall be added where the provident fund is formed under
Provident Fund Act, 1925 and in case of (c) the amount to be added in salary income shall be at the time when
accumulated balance received related to unrecognized provident fund. Hence taxable income shall be Rs. 405,000 in
both (b) and (c) cases that are without provident fund adjustment under the given cases.

Conceptual Approach to Taxes 99


Income From Salary Chapter-07

Pakistan & Foreign source salary income

Geographical source of salary


Received by Taxability
income
1. Pakistan source salary income
[Section 101]:
a. Received from any employment
exercised in Pakistan, wherever
paid, or Taxable [Section 11(5) and (6)]
Resident / Non-resident
b. Paid by, or on behalf of, the individual In case of non resident the terms of
Federal Government, a double taxation treaty agreement, if any
Provincial Government, or a shall also be relevant for PSI
Local Government in Pakistan,
wherever the employment is
exercised

[Section 102]
Any foreign-source salary received by a
resident individual shall be exempt from tax
if the individual has paid foreign income tax
on such salary or his employer has withheld
from the salary and paid to the revenue
authority of the foreign country in which the
employment was exercised.
a. Resident Individual
[Section 103]

A credit or exemption shall be allowed only


if the foreign income tax is paid within 2
years after the end of the tax year in which
the foreign income was derived by the
resident taxpayer otherwise in the absence
of double tax treaty agreement the same
shall be taxable in Pakistan.
b. Non-resident individual Not taxable [Section 11(6)]

[Section 50]
2. Foreign source salary income:
c. Short term resident An individual shall be exempt in respect of
Salary income other than above. his foreign-source income which is not
brought / received in Pakistan if he is
[For all foreign source
resident only by reason of his employment
income]
and he is present in Pakistan for not
exceeding 3 years.
d. Returning expatriate
[Section 51]
[Citizen of Pakistan If an individual citizen of Pakistan (returning
coming back in expatriate) is resident in the current tax year
Pakistan] but was non-resident in the 4 preceding tax
years, his foreign-source income shall be
exempt in current tax year and in the
[For all foreign source
following tax year.
income]

e. Individual leaving
Pakistan during the year
and remains abroad during
that tax year Exemption for salary income [Section 51]

[Citizen of Pakistan
leaves Pakistan]

100 Conceptual Approach to Taxes


Income From Salary Chapter-07

EXEMPTIONS UNDER VARIOUS SECTONS OF THE INCOME TAX ORDINANCE, 2001

SECTION PARTICULARS OF EXEMPTION


42 Diplomatic and United Nations exemptions
43 Foreign government officials
44 Exemptions under international agreements
45 Presidents honours
47 Scholarships
50 Foreign source income of short term resident individuals
1. Diplomatic and United Nations exemptions [42(1) and (2)]
(1) The income of an individual entitled to privileges under the Diplomatic and Consular Privileges Act, 1972 shall
be exempt from tax under this Ordinance to the extent provided for in that Act.
(2) The income of an individual entitled to privileges under the United Nations (Privileges and Immunities) Act,
1948 shall be exempt from tax under this Ordinance to the extent provided for in that Act.
Pension of United Nations' employees [42(3)]
Any pension received by a person, being a citizen of Pakistan, by virtue of the person's former employment in the
United Nations or its specialised agencies (including the International Court of Justice) provided the person's salary
from such employment was exempt under this Ordinance.
2. Employee of a foreign government [43]
Any salary received by an employee of a foreign government as remuneration for services rendered to such
government shall be exempt from tax under this Ordinance provided -
(a) the employee is a citizen of the foreign country and not a citizen of Pakistan;
(b) the services performed by the employee are of a character similar to those performed by employees of the
Federal Government in foreign countries; and
(c) the foreign government grants a similar exemption to employees of the Federal Government performing similar
services in such foreign country.
3. Exemption under a tax treaty [44(1)]
Any Pakistan-source income which Pakistan is not permitted to tax under a tax treaty shall be exempt from tax under
this Ordinance.
Salary income under an aid agreement [44(2)]
Any salary received by an individual (not being a citizen of Pakistan) shall be exempt from tax under this Ordinance to
the extent provided for in an Aid Agreement between the Federal Government and a foreign government or public
international organization, where -
(a) the individual is either not a resident individual or a resident individual solely by reason of the performance of
services under the Aid Agreement;
(b) if the Aid Agreement is with a foreign country, the individual is a citizen of that country; and
(c) the salary is paid by the foreign government or public international organization out of funds or grants released
as aid to Pakistan in pursuance of such Agreement,
4. Allowances attached to honours, awards, etc. [45]
Any allowance attached to any Honour, Award, or Medal awarded to a person by the President of Pakistan shall be
exempt from tax under this Ordinance.
Any monetary award granted to a person by the President of Pakistan shall be exempt from tax under this Ordinance.
5. Scholarships [47]
Any scholarship granted to a person to meet the cost of the persons education shall be exempt from tax under this
Ordinance, other than where the scholarship is paid directly or indirectly by an associate.
6. Income of short-term residents [50]
(1) Subject to sub-section (2), the foreign-source income of an individual -
(a) who is a resident individual solely by reason of the individual's employment; and
(b) who is present in Pakistan for a period or periods not exceeding three years, shall be exempt from tax
under this Ordinance.
(2) This section shall not apply to -
(a) any income derived from a business of the person established in Pakistan; or
(b) any foreign-source income brought into or received in Pakistan by the person.

Conceptual Approach to Taxes 101


Income From Salary Chapter-07

EXEMPTIONS OF TOTAL INCOME UNDER PART I OF SECOND SCHEDULE TO THE INCOME TAX ORDINANCE,
2001

CLAUSE PARTICULARS OF EXEMPTION


3 Salary of employees of Institutions of Agha Khan Development Network, (Pakistan)
4 Salary of employee of Pakistani seafarer working on a foreign vessel
5 Allowances and perquisites of Govt. employees posted abroad
8 Pension
9 Pension of Govt. and Armed forces employees
12 Commutation of pension
13 Gratuity
16 Pension to dependents of Armed Forces Shaheeds
17 Income of dependents of Civil Armed Forces Shaheeds
19 Encashment of leave preparatory to retirement
20 Exemption omitted from tax year 2016
22 TO 26 Funds
39 & 40 Special allowance
51 TO 56 Different perquisites
53A Perquisites by virtue of employment
139 Medical expenditure by individuals
1. Exemptions under 2nd Schedule Part I:
1.1 Salary of employees of institutions of Agha Khan Development Network, (Pakistan) clause 3
Any income chargeable under the head "Salary" received by a person who is not a citizen of Pakistan, and
engaged as an expert or technical, professional, scientific advisor or consultant or senior management staff by
institutions of the Agha Khan Development Network, (Pakistan).
1.2 Income of a Pakistani Seafarer clause 4
Any income chargeable under head "Salary" received by a Pakistani seafarer working on a foreign vessel or
on Pakistan flag vessels for 183 days or more during a tax year provided that such income is remitted to
Pakistan, not later than two months of the relevant income year, through normal banking channels.
1.3 Allowances or perquisites to citizen of Pakistan by Government clause 5
Any allowance or perquisite paid or allowed as such outside Pakistan by the Government to a citizen of
Pakistan for rendering service outside Pakistan.
1.4 Pension Received by a citizen of Pakistani clause 8 & Pension of Govt. and Armed forces employees
clause 9
Already covered above in retirement benefits portion.
1.5 Computation of pension clause 12 & Gratuity or computation of pension on retirement of employee
clause 13
Already covered above in retirement benefits portion.
1.6 Pension to dependents of Armed Forces Shaheeds clause 16 &Pension / income of dependents
of Civil Armed Forces Shaheeds clause 17
Already covered above in retirement benefits portion.
1.7 Leave encashment clause 19
Any sum representing encashment of leave preparatory to retirement of a member of the Armed Forces of
Pakistan or an employee of the Federal Government or a Provincial Government.
1.8 Annuity clause 20
Exemption omitted from tax year 2016.
1.9 Amount received from voluntary pension system clause 23A
The accumulated balance up to 50% received from the voluntary pension system offered by a pension fund
manager under the Voluntary Pension System Rules, 2005 at the time of eligible person's -
(a) retirement; or (b) disability rendering him unable to work; or (c) death by his nominated survivors.
1.10 Funds clause 22 to 26
Already covered in above retirement benefits portion.

102 Conceptual Approach to Taxes


Income From Salary Chapter-07

1.11 Special allowance clause 39 and 40


Any special allowance or benefit or other perquisite to employee under salary income specially granted to meet
expenses wholly and necessarily incurred in the performance of the duties of an office or employment of profit
other than entertainment or conveyance allowance.
1.12 Perquisites exemptions to President, Chief of Staff, Provincial Gonverners and Ministers etc.
[Clause (51 to 53)]
(1) The perquisite represented by the right of the President of Pakistan, the Provincial Governors and the
Chiefs of Staff, Pakistan Armed Forces to occupy free of rent as a place of residence any premises
provided by the Government. (51)
(2) The perquisite represented by free conveyance provided and the sumptuary (entertainment) allowance
granted by Government to Provincial Governors, the Chiefs of Staff, Pakistan Armed Forces and the
Corps Commanders. (52)
(3) The following perquisites and allowances provided or granted by Government to the Ministers of the
Federal Government, namely :- (53)
(a) rent-free accommodation exceeds 10% of the basic salary of the Ministers concerned;
(b) house-rent allowance paid by Government in lieu of rent-free accommodation in so far as it
exceeds Rs. 550 per month:
(c) free conveyance; and
(d) sumptuary allowance.
1.13 Residency and perquisites exemptions to Honourable Judges [Clause (55 and 56)]
Under clause (55) The perquisites represented by the right of a judge of the Supreme Court of Pakistan or
of a judge of High Court to occupy free of rent as a place of residence any premises provided by Federal or
Provincial Government, as the case may be, or in case a judge chooses to resident in a house not provided by
Government, so much of income which represents the sum paid to him as house rent allowance.
Under clause (56) The following perquisites, benefits and allowances received by a Judge of Supreme
Court of Pakistan and Judge of High Court, shall be exempt from tax.
(a) Perquisites and benefits derived from use of official car maintained at Government expenses-
(b) Superior judicial allowance payable to a Judge of Supreme Court of Pakistan and Judge of a
High Court.
(c) Transfer allowance payable to a Judge of High Court.
The following perquisites of the Judge of Supreme Court of Pakistan or High Court shall also be
exempt from tax during service, and on or after retirement.
(a) The services of a driver and an orderly.
(b) 1000 (one thousand) free local telephone calls per month.
(c) 1000 units of electricity as well as (25 hm3 of gas) per month and free supply of water; and
(d) 200 liters of petrol per month.
(e) If during service, a judge dies, exemption from tax in respect of benefits and perquisites provided
to widow as mentioned in sub-clause (2) shall also be available.
1.14 Perquisites by virtue of employment - clause 53A
The following perquisites received by an employee by virtue of his employment, namely;-
(i) free or subsidized food provided by hotels & restaurants to its employees during duty hours;
(ii) free or subsidized education provided by an educational institution to the children of its employees;
(iii) free or subsidized medical treatment provided by a hospital or a clinic to its employees; and
(iv) any other perquisite or benefit for which the employer does not have to bear any marginal cost, as
notified by the Board.
1.15 Medical treatment or reimbursement received and Medical allowance clause 139
Full amount is exempt for benefit given as free medical treatment or hospitalization or both by an employer to
his employee or reimbursement received by the employee of the medical charges or hospital charges or both,
where such provision or reimbursement is in accordance with the terms of employment:
Provided that NTN of the hospital or clinic is given and the employer also certifies and attests the medical or
hospital bills: or
Any medical allowance received by an employee, (if free medical treatment or hospitalization or
reimbursement of medical or hospitalization charges is not provided) then 10% of the basic salary of the
employee is exempt.

Conceptual Approach to Taxes 103


Income From Salary Chapter-07

MULTIPLE CHOICE QUESTIONS


Q.1 The income received by an employee from --------- employer is treated as salary income
(a) Present
(b) Past
(c) Future or
(d) All of the above
Q.2 Benchmark rate of interest on loan provided by the employee to his employer in tax year 2015 is: ________
(a) 1%
(b) 5%
(c) 8%
(d) 10% or
(e) none of the above
Q.3 The value of any right or option to acquire shares is ________.
(a) Chargeable to tax
(b) Chargeable at a concessional rate
(c) Not chargeable to tax or
(d) None of the above
Q.4 An amount required as compensation on termination of employment is a ________.
(a) Perquisite
(b) Profit in addition to salary
(c) Work conditions supplements or
(d) None of the above
Q.5 Salary income is chargeable to tax on ________
(a) Accrual basis
(b) Actual receipt basis or
(c) Both (a) and (b)
Q.6 Salary income is chargeable to tax at the rates specified in the ________schedule to the ITO, 2001.
(a) First
(b) Second
(c) Third or
(d) Fourth
Q.7 In case an employee receive shares from a company under the employee share scheme the cost of shares would
be________
(a) Face value
(b) Price paid by employee
(c) Fair market value or
(d) None of the above
Q.8 In case of rent free accommodation the amount to be included in the salary income of the employee is ________
(a) FM value of rent
(b) 45% of basic salary
(c) Lower of FMR or 45% of basic salary or
(d) Higher of FMR or 45% of basic salary
Q.9 In case of conveyance provided by the employer to his employee for business use, amount to be included in salary
is________
(a) 10% of FMV of car
(b) 5% of cost of car

104 Conceptual Approach to Taxes


Income From Salary Chapter-07

(c) Higher of cost or FMR or


(d) None of the above
Q.10 Where an employee has a salary income with bonus, the bonus received shall be ___________salary income:
(a) Included in
(b) Excluded from
(c) Ignored for
Q.11 In case of rent free accommodation, amount to be included in the salary income of the employee is ____________.
(a) 10% of basic salary
(b) 20% of basic salary
(c) 25% of basic salary
(d) 45% of basic salary
Q.12 The amount expended by the employer on leave fare assistance is included in the salary income of the employee
_______.
(a) Whole
(b) Half
(c) Proportionate
(d) None of these
Q.13 An employee is entitled to deduct ________ incurred in deriving salary income.
(a) travelling expenses
(b) personal expenses
(c) entertainment expenses
(d) None of above
Q.14 House rent allowance provided by employer to his employees is _______.
(a) taxable
(b) not taxable
(c) exempt up to 45% of basic salary
(d) none of the above
Q.15 Tax free salary paid to an employee will be included in the _________of the employee.
(a) business income
(b) tax payable
(c) salary
(d) income from other sources
Q.16 An amount of perquisite is treated as received by an employee if it is paid _____.
(a) to him
(b) applied on his behalf
(c) to any person on his behalf
(d) all of above
Q.17 Entertainment allowance for official performance received by the employee is _______.
(a) exempt
(b) taxable
(c) 50% exempt
(d) 80% taxable
Q.18 The option available to an employee in respect of salary ________ shall be exercised in writing.
(a) received in arrears
(b) for the current year
(c) to be received in next year

Conceptual Approach to Taxes 105


Income From Salary Chapter-07

(d) none of the above


Q.19 The utilities provided by the employer to his employee are chargeable to tax under the head _____________ of
employee.
(a) Income from salary
(b) Other sources
(c) Income from business
(d) Income from utilities
Q.20 Gross salary consists of _______.
(a) basic salary
(b) perquisites
(c) profit in lieu of or addition to salary
(d) all of the above
Q.21 When the employer charges a _____ interest rate than the benchmark rate, then nothing shall be included in the
salary income of the employee.
(a) higher
(b) lower
(c) equal
(d) both a or c
Q.22 Under employee share scheme shares can only be issued to the ______ of the company.
(a) employees
(b) directors
(c) shareholders
(d) chief executive
Q.23 The right or option given to an employee ________.
(a) is always free of cost
(b) is chargeable to tax as business income
(c) is chargeable to tax as property income
(d) none of above
Q.24 An employee may be a person who is engaged in __________.
(a) employment
(b) business
(c) trading
(d) all of above
Q.25 If the insurance premium of the employee paid by the employer then it should be included in the _____ income of the
employee.
(a) business income
(b) salary income
(c) property income
(d) insurance income
Q.26 In case of the self-hiring of the property, it has ____ effect(s) under the law on the income of the recipient.
(a) one
(b) two
(c) three
(d) no
Q.27 In case of golden handshake, employee has option to offer the amount of golden handshake as _________.
(a) SBI under NTR

106 Conceptual Approach to Taxes


Income From Salary Chapter-07

(b) FTR
(c) income from business
(d) all of the above
Q.28 The Commissioner Inland Revenue is empowered to charge tax on the salary income of the employee of a private
limited Company on ______ basis where he has reason to believe that the salary income has not been deliberately
been deferred.
(a) cash
(b) accrual
(c) tax
(d) accounting
Q.29 A salaried person income is taxable @ 0%, if his / her annual income is equal or less than _______.
(a) 1,000,000
(b) 350,000
(c) 400,000
(d) 500,000
Q.30 Commission paid to a part time employee director is chargeable to tax under the head ______ of such director.
(a) salary income
(b) commission income
(c) property income
(d) all of above
Q.31 Any amount payable by employee to employer that has been waived of by the employer is _______ in the hands of
the employee.
(a) not taxable
(b) taxable
(c) exempt
(d) all of above

ANSWERS
1 (d) 2 (e) 3 (c) 4 (d) 5 (b)
6 (a) 7 (b) 8 (d) 9 (d) 10 (a)
11 (d) 12 (a) 13 (d) 14 (a) 15 (c)
16 (d) 17 (a) 18 (a) 19 (a) 20 (d)
21 (d) 22 (a) 23 (d) 24 (a) 25 (b)
26 (b) 27 (a) 28 (a) 29 (c) 30 (a)
31 (b)

Conceptual Approach to Taxes 107


Income From Salary Chapter-07

ICMAP PAST PAPERS THEORECTICAL QUESTIONS


Q. No. 3(c) September 2013 An employer having established approved superannuation funds is required to
contribute annually to the funds on a reasonable basis. However, there is a certain limitations which have been
specified under rule 110 of the Income Tax Rules, 2002 on the initial and annual contribution to the aforesaid funds.
Required:
Identify those limits and the procedures for payment of contribution by the employer beyond the specified limits.
Q. No. 3(c) February 2013 Mr. Rafiq wants to calculate his tax liability of income from salary, Being his tax advisor,
explain perquisites and allowances regarding 'valuation of accommodation' as per rule 4 of the Income Tax Rules,
2002.
Q.2 (b) April 2012 Section 13 of the Income Tax Ordinance, 2001 deals with the valuation of perquisites provided by an
employer for the purposes of computing the income of an employee for a tax year chargeable to tax under the head salary.
How the following perquisites shall be treated?
(i) Services of a housekeeper, driver, gardener, or other domestic assistant.
(ii) Utilities provided by employer.
(iii) Obligation of an employee to repay an amount owing by employee to another person is paid by the employer.
(iv) Loan made to the employee by the employer with no profit payable or the profit payable is less than benchmark rate.
Q. NO. 2(b)(i) SUMMER 2011 To whom a company may issue shares of the company under employee share scheme?
Q. NO. 3(c)(i) WINTER 2010 State the Rule 4 of the Income tax Rules, 2002 for valuation of accommodation provided by an
employer to the employee for the purpose of computing the income chargeable to tax under the head salary.
Q. NO. 2(b) WINTER 2010 What are the various amounts received by an employee as profit in lieu of or in addition to salary
and wages, which would be considered as part of salary?
Q. NO. 2(a) WINTER 2007 An employer may provide medical facility to its employees in any of the following manners:
(i) Free medical treatment or hospitalization.
(ii) Re-imbursement of medical expenses incurred by an employee, and
(iii) Payment of medical allowance instead of providing medical facility
Discuss the legal provisions under the ITO, 2001 under each of the above stated situations.
NOW SOLVE NUMERICAL Q. NO. 2(b) IN PAST PAPER OF WINTER 2007

108 Conceptual Approach to Taxes


Income From Salary Chapter-07

CA MOD C PAST PAPERS THEORETICAL QUESTIONS


Q.NO.3 Spring 2015 Munir resigned from his employment with Ali Industries Limited (AIL) with effect from 31
December 2014. He received following amounts infinal settlement:
Rs. 150,000 as Leave Encashment.
Rs. 4,000,000 under a Golden Handshake Scheme.

Munir had received a salary of Rs. 350,000 per month for a period of six months upto December 2014. His
taxable income and tax liability during the preceding five tax years were as under:

Tax year 2010 2011 2012 2013 2014


Total taxable income (Rs) 2,000,000 2,450,000 2,700,000 3,100,000 3,650,000
Total tax paid (Rs) 300,000 392,000 472,500 542,500 650,000

Required: As a tax consultant, advise Munir about the amount of income tax payable by him for the tax year 2015,
under the Income Tax Ordinance, 2001.

Q.NO. 3(a) Autumn 2014 Zaman is working as the Chief Executive Officer in Yasir Limited (YL). Following are the details of
sale and purchase relating to his capital assets during the tax year 2014.
(a) Under an employee share scheme, 25,000 shares of YL were allotted to Zaman, on 1 December 2011 for Rs. 25 each.
According to the scheme, he was not allowed to sell/transfer the shares before completion of two years from the date of
transfer. The face value of each share is Rs. 10 per share. Fair market value of the shares was as follows:
Rs. 40 per share on 1 December 2011
Rs. 48 per share on 30 June 2012
Rs. 55 per share on 30 November 2013
Rs. 61 per share on 30 June 2014
Required: Compute the amount to be included in the taxable income of Zaman for the tax year 2014.

Q.NO. 3(b) Spring 2008 A company intends to launch an Employee Share Scheme for its employees and for the purpose of
educating its employees in this regard, the management wants to prepare a summary containing the taxability of the
following:
(i) Option granted to an employee.
(ii) Disposal of the option to acquire shares under the employee share scheme.
(iii) Shares issued to an employee under the option that are subject to restriction on transfer.
Explain the timing and valuation aspects in respect of the above, with reference to the ITO, 2001.
Q.NO. 1(a) Spring 2007 (a) Briefly explain the taxability or exemption of the following allowances or perquisites:
(i) Free passage provided by a transporter to its employees;
(ii) Leased motor vehicle provided to an employee, exclusively for his personal use. Running and maintenance cost and
drivers salary is also borne by the employer.
(iii) Medical allowance paid at10 percent of basic salary.
Q. NO. 7(a) Spring 2006 Mr. Ahmed is a senior executive of a company and has opted for an Employee Share Scheme,
announced by the company. As per the scheme, the shares are compulsorily retained in a Trust and time of free right to
transfer has not arrived. However, the shares have been issued and he enjoys all rights of ownership. During the retention
period, he has received dividends and bonus shares.
Comment on the chargeability of income tax on dividends and bonus shares received by him.
Q.NO.8 Autumn 2005 A nationalized bank after privatization has announced a Golden Hand Shake Scheme for its
employees under which lump sum payments are proposed to be made to employees who opt for the scheme.
Discuss the chargeability of above amounts in the hands of employees.
Q.9 April 1995 Indicate which of the following income is exempt whether fully or partly:
i. Casual receipts of non-recurring nature;
ii. Not chargeable to income from business, profession,
iii. Capital gains and salary

Conceptual Approach to Taxes 109


Income From Salary Chapter-07

iv. Salary income of a foreign diplomat


v. Balance payable to an employee from Provident Fund
vi. Special Allowance to meet certain expenses in the performance of official duties
vii. Sum paid to an employee for meeting gas, water and electricity charges
viii. Salary income of a working woman
x. Compensation on termination of employee's services
Autumn 1994 Q.9 Answer the following statements considering the keys given therein:
Amount received from an approved Gratuity Fund is exempt to the extent of:
a) Rs. 200,000
b) Rs. 75,000
c) Whole amount.
NOW SOLVE FOLLOWING NUMERICAL QUESTIONS OF MODULE C / AFC PAST PAPER RELATED TO THIS TOPIC

Q. NO. 1 AUTUMN 2013


Q. NO. 1 AUTUMN 2007
Q. NO. 2(B) AUTUMN 2006
Q. NO. 2 AUTUMN 2005
Q. NO. 2 SPRING 2004
Q. NO. 2 AUTUMN 2003
Q. NO. 7 SPRING 2003
Q. NO. 7(B) AUTUMN 2002
Q. NO. 7 AUTUMN 2001

110 Conceptual Approach to Taxes


Income From Property Chapter-08

Chapter

8 INCOME FROM PROPERTY

Topic covered
Section
(For CAF-6 and ICMAP students)
15 & 16 Property income, rent, & rent chargeable to tax
15A Deductions allowed against income from Property
68 Fair market value of rent
15 & 39 Examples of property not taxable under this head of income
Rates of income tax on property income as per 1st schedule
16 Treatment of advances
66 Liability in case of co-owners
11, 50, 51 & 101 Geographical source of property income & their taxability
Property income no taxable / entitled to tax credit
MCQs with solutions
ICMAP & CA Mod C past papers theoretical questions

For CAF-6 and ICMAP Students


1. Income from property [u/s 15]
The rent received or receivable by a person for a tax year, other than rent exempt from tax shall be chargeable to
tax in that year under the head income from property. It means it shall be chargeable to tax on accrual basis.
Property or rental income from the lease of immovable property in Pakistan shall be Pakistan source income.
2. Rent [U/s 15(2)]
Rent means any amount received or receivable (inclusive of forfeited deposit for sale of land or building) by an
owner of land or a building as consideration for the use or occupation of, or the right to use or occupy the land or
building.
Signing amount paid by the tenant is also taxable under the head property income for the owner as the same paid at
the time of entering into rent agreement that is not refundable to tenant.
Owner:
The term owner has never been defined in the Ordinance that means the intention of the legislature is to keep the
meaning of the term owner far larger than the registered owner.
The circular No 14 of 1959 dated 10-08-1959 throw the light on this situation:
A person may be the owner of the property without being the owner of the land on which the property is built. It is
wrong to think that if the land on which the property has been built belongs to one person but the building on it has
been constructed by another person, the person owning the land will automatically be the owner of building. Owner of
the building is the person who has financed the construction of the building and it is he who will be liable to tax in
respect of bonafide annual letting value that building.
Therefore, the owner of land and building may happen to be two persons. The owner of building will be the person
who financed the construction of the building.

Land or building: These terms are also not defined in the ordinance; however, considering the generic meaning,
we can say that it may include followings:
Land Open Plots, playgrounds, gardens, Buses stand, cattle sheds etc.
Building The building includes open air theatres, swimming pools, stadiums, residential houses, building
let out for office use, or for storage or for use of a factory, music halls, theatre hall, inns, lecture
halls and other public auditorium used for cinema and stage shows.

Conceptual Approach to Taxes 111


Income From Property Chapter-08

Rent chargeable to tax includes:


(i) Actual rent received or receivable (including forfeited deposit) by a person or fair market rent whichever is
higher. ADD
(ii) 1/10TH of non adjustable advance (if any) [U/s 16(1)].
Fair market value (rent) [U/s 68]
Fair market value (rent) means the value of any property, rent, asset, service, benefit or perquisite at a particular time
shall ordinarily fetch on sale or supply in the open market. FMV shall be determined by CIR even where it is not
determinable for income tax purposes.
Example On the first day of July 2014, Mr. Kaleem rented out his house at a monthly rent of Rs.15,000 and also
received Rs.100,000 as advance not adjustable against rent. Compute rent chargeable to tax for the year.
Solution:
Rs.
Annual rent (15,000 x 12) (Higher of actual rent or FMR) 180,000
Add: 1/10th of advance not adjustable against rent (100,000/10) 10,000
Rent chargeable to tax 190,000
Example: On July 1st, 2015 Mr. Kaleem received Rs. 60,000 as advance from Mr. Faheem under a contract of sale
of his property for Rs. 600,000. After 2 months Mr Faheem failed to pay the remaining amount and advance of
Rs.60,000 was forfeited by Mr. Kaleem. Compute income of Mr Kaleem chargeable to tax under the head income
from property for the tax year 2016.
Solution:
Rs.
Advance deposit forfeited treated as property income 60,000
Rent Chargeable to Tax 60,000
Self Hiring of property: Where property owned by employee and after taking on rent from employee by employer
given the same for accommodation to employee [U/s 15(5)]
Where the employee is the owner of the property and FMV of accommodation has been included in the salary income
of the employee then actual rent shall be included under the head property income without comparing it with the FMV
of rented premises.
Example: Mr. Asif has received Rs. 30,000 as monthly basic salary from his employer. The accommodation facility
has also been provided by the employer. The accommodation is actually been in the ownership of the employee.
You are required to compute the income from salary and property income under the following situation.
Assuming that the actual rent of accommodation is Rs. 200,000 and FMV of the rented premises is Rs. 250,000.
Solution:
Income from salary Rs.
Annual salary income (Rs. 30,000 x 12 months) 360,000
Value of accommodation Higher of 45% of basic pay 162,000
and FMV of rented premises i.e. Rs. 250,000, hence 250,000
Taxable salary income 610,000
Income from Property
Actual rent (without comparing it with the FMV) 200,000
Total taxable income 810,000

3. Deductions in computing income chargeable under the head Income from Property [U/s 15A]
The following deductions or allowances shall be allowed in computing the income of a person chargeable to tax
under the head Income from Property for a tax year [U/s 15A (1)]:
(a) 1/5th repair allowance of rent chargeable to tax for the year, computed before any deduction allowed;
(b) any premium paid or payable in the year to insure the building against the risk of damage or destruction;
(c) any local rate, tax, charge, or cess in respect of the property or the rent from the property paid or payable by
the person to any local authority or government in the year, not being any tax payable under this Ordinance;
(d) any ground rent paid or payable in the year in respect of the property;
(e) any profit paid or payable in the year on any money borrowed including by way of mortgage, to acquire,
construct, renovate, extend, or reconstruct the property;

112 Conceptual Approach to Taxes


Income From Property Chapter-08

(f) share in rent and share towards appreciation in the value of property (excluding the return of capital, if any)
from the property paid or payable to the House Building Finance Corporation or a scheduled bank under a
scheme of investment in property in the year under that scheme;
(g) where the property is subject to mortgage or other capital charge, the amount of profit or interest paid on such
mortgage or charge;
(h) any administration and collection charges (wholly and exclusively to earn rent chargeable to tax) paid or
payable in the year not exceeding 6% of the rent chargeable to tax computed before any deduction;
(i) any expenditure paid or payable in the tax year for legal services acquired to defend the persons title to the
property or any suit connected with the property in a Court; and
(j) Treatment of unpaid rent:
Where there are reasonable grounds for believing that any unpaid rent in respect of the property is
irrecoverable, an allowance equal to the unpaid rent where
(i) the tenancy was bona fide, the defaulting tenant has vacated the property or steps have been taken to
compel the tenant to vacate the property and the defaulting tenant is not in occupation of any other
property of the person;
(ii) the person has taken all reasonable steps to institute legal proceedings for the recovery of the unpaid
rent or has reasonable grounds to believe that legal proceedings would be useless; and
(iii) the unpaid rent has been included in the income of the person chargeable to tax under income from
property for the tax year in which the rent was due and tax has been duly paid on such income.
Partly of fully recovery against already allowed unpaid rent [U/s 15A (2)]
Where any unpaid rent allowed above as a deduction is wholly or partly recovered, the amount recovered shall be
chargeable to tax in the tax year in which it is recovered.
Non payment of already allowed expenditure within three years [U/s 15A (3)]
Where a person has been allowed a deduction for any expenditure incurred in deriving rent chargeable to tax and the
person has not paid the liability or a part of the liability to which the deduction relates within three years of the end of
the tax year in which the deduction was allowed, the unpaid amount of the liability shall be chargeable to tax under
income from property in the first tax year following the end of the three years.
Partial of full payment after disallowance of expenditure [U/s 15A (4)]
Where an unpaid liability is chargeable to tax as a result of the application of sub-section (3) and the person
subsequently pays the liability or a part of the liability, the person shall be allowed a deduction for the amount paid in
the tax year in which the payment is made.
Limitations on allowable expenditures [U/s 15A (5) & (6)]
Any expenditure allowed to a person under this section as a deduction shall not be allowed as a deduction in
computing the income of the person chargeable to tax under any other head of income. [U/s 15A (5)]
The provisions of section 21 shall apply in determining the deductions allowed to a person under this section in the
same manner as they apply in determining the deductions allowed in computing the income of a person chargeable to
tax under the head Income from Business. [U/s 15A (6)]
4. Examples of Income from Property not taxable under this head
(1) Lease income of a building that is let out together with plant and machinery e. g. a flour mill. [U/s 15(3) and
39(1)(f)]
(2) Rent from the sub lease (by tenant) of land or a building. [U/s 39(1)(e)]
Explanation: Any amount received or receivable as consideration for the use or occupation, or right to use or
occupy, of any land or building not owned by the taxpayer falls under the head income from Other Sources.
(3) Provisions of amenities, utilities or any other service connected with the renting of the building. (Quote
practical case) [U/s 15(3A) and 39(1)(fa)]
(4) Ground rent of land (Quote practical case) [U/s [39(1)(d)]; and
(5) Amount received as consideration for the provision, use or exploitation of property, including from the grant of
a right to explore for, or exploit natural resources. [U/s 39(1)(i)]
(6) Income from a building used as store house in the surrounding area of agri land. [U/s 41]
(7) 1/10th of excess from already deposited amount received by tenant on vacating the possession of a property.
[U/s 39(1)(k) & (2)]

Conceptual Approach to Taxes 113


Income From Property Chapter-08

5. Treatment of utilities included in rent: [U/s 15(3A)]


Where any amount is included in rent received or receivable by any person for utilities or any other services
connected with the renting of the building, such amount shall be chargeable to tax under the head income from other
sources.
Example: Mr. Naeem received Rs. 20,000 as monthly rent from his tenant. As per rent agreement, Rs. 2,000 is
included in the amount of rent on account of electricity bill. Actual electricity bill paid by Mr. Naeem for the year is
Rs.8,000. Calculate his taxable income for the year.
Solution:
Income from Property
Property income (assumed after expenses) (20,000 - 2,000) x 12 216,000
Income from Other Sources
Amount received against electricity bills (Rs. 2,000 x 12) 24,000

Less: Expenses
Actual electricity bills paid 8,000
Taxable income from other sources 16,000
Total income covered under NTR 232,000

Example From the following information calculate the tax liability of Mr. Tayyab Zahid.
Annual rent 180,000
Property tax paid 20,000
Repair of property given on rent 50,000

Solution:

Mr. Tayyab Zahid


Computation of taxable income and tax liability:

Income from Property:


Annual rent 180,000

Less property tax paid (20,000)


1/5th fixed repair allowance (36,000)
124,000
Computation of tax liability:
Tax on property income:
(Tax @ 0% on Rs. 124,000) - -
6. Treatment of advances [u/s 16]:
(a) Adjustable advances:
Where the tenant paid advance which is adjustable against rent then no treatment is required while computing
the rent chargeable to tax because the same will automatically be included in the rent chargeable to tax.
(b) Non adjustable amounts (including pugree) received in relation to buildings: [U/s 16(1), (2) and (3)]
Amounts received from a tenant not adjustable against rent chargeable to tax under this section in the tax
year in which it was received and the following nine tax years in equal proportion.
Where the above amount received by the owner refunded to the tenant on termination of the tenancy before
the expiry of ten years, no portion of the amount shall be allocated in the tax year in which it is refunded or
to any subsequent tax year except provided as under.
Where the property is vacated and the owner let out the building or part to the succeeding tenant and
receives any amount not adjustable against rent from the succeeding tenant, the succeeding amount as
reduced by the earlier amount already charged to tax shall be treated as rent chargeable to tax in 10 years
in equal proportions. However, where the property is vacated by the earlier tenant after the expiry of 10 years
then succeeding amount shall not be reduced by the earlier amount already charged to tax and whole
amount received from succeeding tenant shall be charged to tax in 10 years in equal proportion.
Example: Mr. Sarfraz let out his house at a rent of Rs.10,000 p.m. He received a deposit of Rs.200,000 not
adjustable against rent, out of which he refunded Rs.100,000 to previous tenant, who vacated the house after 3
years tenancy. Calculate rent chargeable to tax

114 Conceptual Approach to Taxes


Income From Property Chapter-08

Solution: Rs.
Calculation of rent chargeable to tax:
Rent receivable (10,000 x 12) 120,000
Not adjustable ( 200,000 amount already charged to tax [Rs.100,000 x 3 / 10)] /10 17,000
Rent chargeable to tax 137,000

Important note: Practically when the tenant paid amount which is adjustable against rent then there is no treatment
required while computing the rent income because the same shall be automatically (transferred by an adjusting
accounting entry) to rent chargeable to tax on expiry of the period for which the adjustable advance was received.
7. Liability in case of co-owners [U/s 66]
In case of co-ownership in property and the share of each co-owner is determinable then share received by each co-
owner from property shall be included in his total income but this principle will not apply to business income i.e.
business of renting out of land or building.
Signing amount from the tenant is taxable under the head "income from property". Signing means the amount paid by
the tenant to the owner to enter in the tenancy agreement which is neither refundable nor it can be termed as deposit.
Example Mr. A and Mr. B are co-owners of a property. Their share in the property is equal. They received net income
from property of Rs. 400,000. Calculate the tax liability of both co-owners.
Solution: Mr. A Mr. B
Income from property:
Share of Mr. A (400,000 x 50% assumed after expenses) 200,000
Share of Mr. B (400,000 x 50% assumed after expenses) 200,000
200,000 200,000
Computation of Tax liability:
Tax payable by Mr. A and Mr. B:
Tax on Rs. 200,000 @ 0% - -
i.e. 0% upto Rs. 400,000
Example: Mr. X and Mr. Y are partners in XY Associates. Principal business of XY Associates is running a hotel
including renting out of rooms. Net income after deductions received (including room rents) during the tax year was
Rs. 500,000. Calculate the tax liability of firm and each partner assuming that share of each partner in profits of the
firm is equal. Mr. X and Mr. Y has no other source of income.
Solution:
In this case income is chargeable to tax under the head Income from business not under the head income from
property.
(Rupees)

Income of the AOP 500,000

Tax liability [7% x (500,000 400,000)] 7,000


As partners have no other source of income hence nothing shall be included in the income of partners for rate
purpose, hence, no tax is payable by the partners. Tax liability of the firm is Rs. 7,000.

Conceptual Approach to Taxes 115


Income From Property Chapter-08

8. Pakistan source & Foreign source property income


Geographical source of property
Received by Taxability
income
Pakistan source property income
[Section 101]: Taxable [Section 11(5) and (6)]
Resident / Non-resident Term and conditions of double
If it is derive from the lease of person taxation treaty agreements shall also
immovable property in Pakistan be considered for non residents
including a right to explore for, or having Pakistan source income.
exploit, natural resources in Pakistan.
a. Resident person Taxable [Section 11(6)]

[Section 50]
b. Short term resident
An individual shall be exempt in respect
of his foreign-source income which is not
[For all foreign source brought / received in Pakistan if he is
income] resident only by reason of his
employment and he is present in
Foreign source property income Pakistan for not exceeding 3 years.
Property income other than above
c. Returning expatriate [Section 51]

[Citizen of Pakistan If an individual citizen of Pakistan


coming back in Pakistan] (returning expatriate) is resident in the
current tax year but was non-resident in
[For all foreign source the 4 preceding tax years, his foreign-
income] source income shall be exempt in current
tax year and in the following tax year.
Non-resident individual Not taxable [Section 11(6)]

9. Property income not taxable / entitled to tax credit:


(1) Property used by owner for his own business or personal use are not taxable;
(2) Property owned by a Trust, welfare institution or and NPO etc. are now entitled to tax credit under section
100C of the Income Tax Ordinance, 2001 on fulfilment of specified conditions.

116 Conceptual Approach to Taxes


Income From Property Chapter-08

MULTIPLE CHOICE QUESTIONS


Q.1 _______________ is taken as rent chargeable to tax.
(a) Actual rent
(b) Fair market rent
(c) Higher of FMR and actual rent plus 1/10th of non-adjustable advance, if any
(d) Lower of FMR and actual rent
Q.2 ____________ part of advance not adjustable against rent is charged to tax every year.
(a) 10%
(b) 5%
(c) 12%
(d) 50%
Q.3 Advances adjustable against rent_____________ the future rent chargeable to tax.
(a) Increase
(b) Decrease
(c) has no effect
Q.4 The amount received by employee to vacate the possession less amount for taking the possession is charged to tax
under__________ in ten equal proportions in the succeeding 10 years.
(a) Income from other sources
(b) Property income.
(c) Business income
(d) none of these
Q.5 The income of the persons having definite share in a joint property shall be taxed_________.
(a) As association of person
(b) the respective share in the hands of the members
(c) none of the above
Q.6 Where the rent includes charges for utilities, then __________should be charged to tax under income from property.
(a) Whole of the rent
(b) Rent less amount for utilities
(c) Rent less expenditure for utilities.
(d) (b) or (c)
Q.7 The property income is taxable __________
(a) Under FTR
(b) Under NTR
(c) SBI under FTR
(d) None of above
Q.8 An individual will not pay tax if his income under the head property is_____________.
(a) Zero
(b) Rs.400,000
(c) Rs.100,000
(d) Rs.150,000
(e) None of these
Q.9 Income tax is charged @ __________ of rent chargeable to tax till Rs. 400,000 received by a Company (other than a
small company).

Conceptual Approach to Taxes 117


Income From Property Chapter-08

(a) 0%
(b) 5%
(c) 7%
(d) 33%
(e) none of the above
Q.10 Deduction for__________is allowed while computing income under the head property.
(a) 1/5th repair allowance
(b) Insurance premium for property damage
(c) Property tax for property
(d) All of the above
Q.11 If tax at source has been deducted or not from rent received even then the rent received shall taxable
under_________ for a company.
(a) FTR
(b) NTR
(c) Separate block of income
(d) None of above
Q.12 ____________of the forfeited deposit against sale of property is included in the rent chargeable to tax.
(a) 25%
(b) 50%
(c) 75%
(d) 100%
Q.13 The rent of the building let out together with the plant and machinery installed therein is taxable under ____________.
(a) Property income
(b) Business income
(c) Income from other sources
Q.14 Legal charges paid by the owner of property are allowed as deduction against property income under____________.
(a) NTR
(b) FTR
(c) Not allowed
(d) all of these
Q.15 In case where the property is hired by the employee and rent is payable by the employer then _________ shall be
included in the salary income of the employee.
(a) Actual rent
(b) Fair market value of rent
(c) none of these
Q.16 Where the employee or his / her spouse is the owner of the building and that building has been provided by the
employer against the entitlement of the rent free accommodation then it has ________________.
(a) One effect
(b) two effects
(c) no effect
Q.17 Obligation of the owner paid by the tenant included in the property income of the ___________ in the respective tax
year, where the same is as per terms of rent deed.
(a) Owner of the property
(b) Tenant
(c) None of these

118 Conceptual Approach to Taxes


Income From Property Chapter-08

Q.18 Property income received by subletting of property by the tenant is taxable under______________.
(a) Property income
(b) Income from other sources
(c) FTR
Q.19 If portion of the rent and utilities cannot be segregated from each other then expenses incurred for provision of utilities
shall
(a) Be deducted from rent
(b) not be deducted from rent
Q.20 Where the property income of a small company is more than Rs.1,000,000 then rate of tax is__________.
(a) 35%
(b) 25%
(c) 33%
Q.21 The amount received for utilities is taxable under the head _________________, only if separable.
(a) Income from property
(b) Income from other sources
(c) Income from business
Q.22 The repair charges and insurance premium paid shall be____________________ against rent chargeable to tax.
(a) allowed as deduction
(b) not allowed
(c) 1/10th is allowed
(d) none of the above
Q.23 The amount received for utilities is taxable under the head income from __________.
(a) salary
(b) property
(c) other sources
(d) business
th
Q.24 1/10 of any amount of advance received by the owner as advance ________ against rent is included in rent
chargeable to tax.
(a) adjustable
(b) not adjustable
(c) payable
(d) receivable
Q.25 Security received as an advance is ________ against rent.
(a) adjustable
(b) not adjustable
(c) payable
(d) receivable
Q.26 Amount received by tenant for vacating the possession, less amount already paid is chargeable to tax under the head
Income from ____________.
(a) salary
(b) property
(c) other sources
(d) business

Conceptual Approach to Taxes 119


Income From Property Chapter-08

Q.27 If the employee owned property after renting out to employer has been given to the employee for accommodation
then in such case the actual rent shall be taxable in the hands of employee without comparing it with the ________.
(a) FMV
(b) market value
(c) cost of asset
(d) none of the above
Q.28 Rent chargeable to tax includes ______________.
(a) amount forfeited under an agreement for sale of property
(b) accommodation allowance received from employer
(c) ground rent
(d) none of above
Q.29 Rent ________ is taxable against property income.
(a) received
(b) receivable
(c) payable
(d) both a and b
Q.30 Ground rent is chargeable to tax under the head income from _________.
(a) other sources
(b) salary
(c) property
(d) business
Q.31 Amount received from vacating possession is charged to tax in ten years in _______ proportion.
(a) equal
(b) unequal
(c) none of above
Q.32 Share of each person is taxable as ___ in case of property in co ownership.
(a) FTR
(b) NTR
(c) SBI under NTR
(d) all of above
Q.33 Rent chargeable to tax is _________ actual rent or FMR plus 1/10th of non adjustable advance.
(a) equal to
(b) higher of
(c) lower of
(d) none of above
Q.34 Any forfeited deposit received under a contract of sale of property by the owner of such property is taxable under the
head income from _______.
(a) property
(b) other sources
(c) salary
(d) business
Q.35 Income from any rented agricultural building derived will be treated as income from ______.
(a) property
(b) other sources
(c) salary
(d) none of above

120 Conceptual Approach to Taxes


Income From Property Chapter-08

ANSWERS
1 (c) 2 (a) 3 (c) 4 (a) 5 (b)
6 (d) 7 (b) 8 (b) 9 (d) 10 (d)
11 (b) 12 (d) 13 (c) 14 (a) 15 (c)
16 (b) 17 (a) 18 (b) 19 (a) 20 (b)
21 (b) 22 (a) 23 (c) 24 (b) 25 (b)
26 (c) 27 (a) 28 (a) 29 (d) 30 (a)
31 (a) 32 (b) 33 (b) 34 (a) 35 (d)

Conceptual Approach to Taxes 121


Income From Property Chapter-08

ICMAP PAST PAPERS THEORECTICAL QUESTIONS


Q.2 (c) April 2012 Answer the following as per Section 16 of the Income Tax Ordinance, 2001:
(i) Mr. Abubakar rented out his flat to Mr. Munnaf. At the time of rent agreement Mr. Abu bakar received an amount
from the tenant, which is not adjustable against monthly rent. How this amount shall be treated?
(ii) If on termination of the tenancy, the above mentioned amount is refunded to the tenant before expiry of ten years and
the house is not let out to any other person, how this situation shall be treated?
(iii) Where the circumstances as mentioned in (ii) above occur and the property is let out to another person (succeeding
tenant) and from the succeeding tenant an amount is received which is not adjustable against rent, how the refund of
the deposit of old tenant and the receipt of the new deposit shall be treated?
Q.NO. 2 (c) (ii) Summer 2011 List down the conditions in respect of a person not liable to pay tax on "Income from
property".

122 Conceptual Approach to Taxes


Income From Property Chapter-08

CA MOD C PAST PAPERS THEORECTICAL QUESTIONS


Q.NO.4 (a) Spring 2015
(i) Explain the term Rent in context of Income from property.
(ii) Specify the head of income under which the following amounts would be chargeable to tax:
rent from sub lease of a building.
amount included in rent for the provision of amenities, utilities and any other service connected with
renting of the building.

Q. No. 4 (b) Spring 2012 Yaqoot and Loha are joint owners of a bungalow which has been rented out for Rs. 70,000 per
month.
Required: Discuss the taxability of Yaqoot and Loha in respect of above income, in the light of Income Tax Ordinance, 2001.
Q.NO. 4(a) and (b) Spring 2009 Mr. Bukhari is a resident person and owns a property abroad. During the year, he received
rent amounting to Rs. 3 million from that property. The tax on rental income has been duly paid abroad and there is no tax
treaty between Pakistan and the country in which the property is situated.
(a) Explain the tax treatment of rental income received by Mr. Bukhari in Pakistan.
(b) Discuss the provisions of the Income tax Ordinance, 2001 regarding non-adjustable amount received from a tenant
by the owner of a building.
Q.NO. 1(a) Autumn 2006 Describe the term rent in the context of income from property.
(b) Autumn 2006 Through Finance Act, 2006 income from property has been subjected to final tax regime. However, the
provisions relating to taxability of income from property shall not apply to taxpayers who meet certain conditions. State these
conditions.
Q.NO. 1(c) Autumn 2006 Specify under which head of income, following amounts of rent would be chargeable to tax:
(i) Rent in respect of lease of a building together with plant and machinery.
(ii) Amount included in the rent of a building for the provision of amenities, utilities or any other service connected with
the renting of such building.
Q.3 Autumn 2002
(a) What is chargeable to tax under the head income from house property?
(b) Elaborate the terms house property and annual value.
(c) What is the rationale for allowing vacancy allowance?
(d) What are the rules prescribed for the allow-ability of unrealized rent?

NOW SOLVE FOLLOWING NUMERICAL QUESTIONS OF MODULE C / AFC PAST PAPER RELATED TO THIS TOPIC

Q. NO. 4(B) SPRING 2015


Q. NO. 4 SPRING 2014
Q. NO. 5 SPRING 2003
Q. NO. 4 AUTUMN 2002

Conceptual Approach to Taxes 123


Income From Property Chapter-08

124 Conceptual Approach to Taxes


Income From Business Chapter-09

Chapter

9 INCOME FROM BUSINESS

Topic covered
Section Rule (Part - I for CAF-6 and ICMAP students)
Part - I
2(9) & 18 Business & income from business
18 Profit on debt, lease amounts, dividend income from mutual funds
19 Speculation business
20 Deductions in computing income from business
10 Entertainment expenditure
11 Agriculture produce used as raw materials
26 Scientific research expenditure
27 Employee training facilities
28 Profit on debt, financial cost & lease payments
Special purpose vehicle
29 Bad debts
21 Deductions not allowed
Deductible allowance
60 Zakat
60A Workers welfare fund
60B Workers participation fund
11, 50, 51 & 101 Geographical source of business income & their taxability
(Part - II for CA Mod F and ICMAP students)
29A Provisions regarding consumers loan
30 Profit on non performing debts of a banking company
31 Transfer to Participatory reserve
Exemptions
Practice questions with solutions
MCQs with solutions
ICMAP & CA Mod C past papers theoretical questions

PART I (For CAF-6 and ICMAP students)

1. Business [U/s 2(9)]


The term "business" includes any trade, commerce, manufacture, profession, vocation or adventure or concern in
such nature but does not include an employment.
2. Incomes from business [U/s 18(1), (2), (3) and (4)]
The following incomes of a person (other than exempt income), shall be chargeable to tax as "Income from
Business"-
PARTICULARS
(a) the profits and gains of any business carried on at any time in the year;
(b) Income derived from the members by any trade, professional or similar association from the sale of goods or rendering
of services;
(c) any income from the hire or lease of tangible movable property;

Conceptual Approach to Taxes 125


Income From Business Chapter-09

(d) the fair market value of any benefit or perquisite derived by a person due to any past, present, or prospective
business relationship; and
Explanation: The word benefit includes any benefit derived by way of waiver of profit on debt or the debt itself under
the State Bank of Pakistan, Banking Policy Department, Circular No.29 of 2002 or in any other scheme issued by
the State Bank of Pakistan.
(e) any management fee derived by a management company.
Profit on debt [U/s 18(2)]
Where the business of a person is to earn such profit then such profit shall be taxed under the head business and not
under the head "Income from Other Sources".
Lease amount [U/s 18(3)]
Where a lessor being as a scheduled bank or an investment bank or a development finance institution or a
modaraba or a leasing company has leased out any asset (in the ownership of lessor or not) to a lessee then
such amount received in connection to lease arrangement shall be treated as business income of the lessor.
Dividend income from Mutual Funds [U/s 18(3)]
Any amount received by a banking company or a non-banking finance company from distribution by a mutual
fund or a Private Equity and Venture Capital Fund out of its income from profit on debt shall be treated as business
income and not as income from other sources.
3. Speculation business [U/s 19]
Speculation Business is considered separate from Non Speculation Business. The profits and gains arising from the
speculation business for a tax year shall be included in the persons income chargeable to tax under the head
"income from business". However u/s 58 the loss arising from speculation business carried on by the person
shall be set off only against the income of the person from any other speculation business of the person
chargeable to tax for that year, hence the same is treated as separate and distinct from any other business carried on
by the person.
"Speculation business" means any business in which a contract for the purchase and sale of any commodity
(including stocks and shares) is made otherwise than by the actual delivery or transfer of the commodity but does
not include contracts to guard against future loss in price of goods manufactured for sale or actual delivery,
holding of stocks and shares or contract to guard against loss that may arise due to forward market or stock
exchange related to jobbing and arbitrage are not included in speculation business.
4. Deductions in computing income chargeable under the head income from business [U/s 20]
4.1. Expenditure incurred wholly and exclusively for business [U/s 20(1)]
A person shall be allowed deduction in respect of expenditures incurred wholly and exclusively for business that
are covered under the main profit and loss heads i.e. cost of sales, administrative and selling expenses.
4.2 Use of animals for business [U/s 20(1A)]
Where animals which have been used for the purposes of business or profession (other than as stock in trade) and
have died or become permanently useless, the difference between the actual cost of the animals and the
amount, if any, realized in respect of the carcasses of animals.
4.3 Special provisions regarding normal depreciation, initial allowance and amortization of intangible assets [U/s
20(2)]
Where the expenditure is incurred in acquiring a depreciable asset or an intangible with a useful life of more than one
year or is pre-commencement expenditure, the person must depreciate or amortize the expenditure in accordance
with sections 22, 23, 24 and 25.
4.4 Legal, financial and administrative expenditure on amalgamation [U/s 20(3)]
Expenditure incurred by an amalgamated company on legal and financial advisory services and other administrative
cost related to amalgamation shall be allowed as expenditure.
Example: Following information is related to Mr. Kaleem who carries on the business of Dairy farm.
Rs.
Sale of milk and other dairy products 740,000
Amount realized on sale of died cow 2,000
Cost of died cow 40,000
Other expenses 350,000
Note: 20% of other expenses are not related to the business.
Required: Compute taxable income of the taxpayer by taking tax depreciation at Rs.15,000.

126 Conceptual Approach to Taxes


Income From Business Chapter-09

Solution:
Mr Kaleem
Computation of taxable income: Rs. Rs.
Income from business:
Sales 740,000
Less: expenses
Cost of died cow 40,000
Less sale value of died cow (2,000)
38,000
Other expenses (350,000 x 80%) 280,000
Depreciation 15,000

333,000
Profit before tax 407,000
Note: 20% of the other expenses were not for the purpose of business, therefore, these expenses were not allowed.
Other deduction available in the Income tax Ordinance and Income Tax Rules
5. Entertainment expenditure [Rule 10]
Entertainment expenditure (including meals, refreshments and reasonable leisure facilities as per norms and customs
of the business in Pakistan) related directly to the business on satisfaction of the following conditions:
expenditure incurred outside Pakistan on entertainment or where such expenditure related to head office
expenditure;
expenditure incurred in Pakistan on entertainment of foreign customers and suppliers, customers and clients
at business premises;
expenditure incurred on entertainment at a meeting of shareholders, agents, directors or employees; or
expenditure incurred on entertainment at the opening of branches.
6. Agriculture produce used as raw materials [Rule 11]
This rule applies to a person who is a cultivator or receiver of agricultural produce as rent in kind and who
uses agricultural produce raised or received as raw material in a business the income from which is chargeable
to tax under the head "Income from Business" as under:
Taxable income = Total income LESS market value of the agricultural produce used in business as raw material
Only the market value of the agricultural produce is deducted and no further deduction shall be made in respect
of the any expenditure incurred by the taxpayer as cultivator or as a receiver of rent in kind. The market value shall
be computed as under:
6.1 Where the produce is ordinarily sold in the market
Where the product is ordinarily sold in the market either in the raw state or after application of any process required to
make to product fit to be taken to the market then the market value shall be determined on the date of use.
6.2 Where the produce is not ordinarily sold in the market
The aggregate of expenses on cultivation and the land revenue or rent paid for the area in which it is grown.
Example
Mr. B owns a floor mill. He also owned agricultural land from which wheat is being grown and used in the floor mill as
raw material. Profit and loss account of Mr. B for the year ended June 30, 2016 is as under
Rupees
Sale of floor 1,000,000
Less expenditure:
Wheat cultivation cost 600,000
Wheat conversion cost 100,000
Other expenses 50,000
250,000
Important note: Wheat used in floor mill, if purchased from open market, would have cost to Mr. B Rs. 725,000.

Conceptual Approach to Taxes 127


Income From Business Chapter-09

Required:
(a) Taxable income of Mr. B, if wheat is ordinarily sold in the market.
(b) Taxable income of Mr. B, if wheat is not ordinarily sold in the market.
Solution:
(a) Taxable income of Mr. B, if wheat is ordinarily sold in the market in raw state.

Rupees
Sale of floor 1,000,000
Less expenditure:
Market value of wheat 725,000
Wheat conversion cost for floor 100,000
Other expenses 50,000
Taxable income 125,000

(b) Taxable income of Mr. B, if wheat is not ordinarily sold in the market in raw state.

Rupees
Sale of floor 1,000,000
Less expenditure:
Wheat cultivation cost (including land rent) 600,000
Wheat conversion cost 100,000
Other expenses 50,000
Taxable income 250,000

7. Scientific research Expenditure [U/s 26]


A person shall be allowed deduction on account of scientific research expenditures incurred in Pakistan wholly and
exclusively for business is admissible.
Scientific research means any activity undertaken in Pakistan in the fields of natural or applied science for the
development of human knowledge.
"Scientific research expenditure" means any expenditure incurred by a person on scientific research within
Pakistan for the purposes of developing the persons business, including any contribution to a scientific research
institution to undertake scientific research for the purposes of persons business, other than expenditure incurred:
in the acquisition of any depreciable asset or intangible;
in the acquisition of immovable property or
for the purposes of ascertaining the existence, location, extent or quality for a natural deposit.
8. Employee training and facilities [U/s 27]
A person shall be allowed a deduction for any expenditure (other than capital expenditure) incurred in a tax year in
respect of-
educational institution or hospital in Pakistan established for the benefit of employees and their
dependents;
institute in Pakistan established for the training of industrial workers recognized, aided, or run by the
Federal Government or a Provincial Government or a Local Government; or
training of any person as a citizen of Pakistan in connection with a scheme approved by the Board.
9. Profit on debt, financial costs and lease payments [U/s 28(1)]
The following shall be allowed as admissible expenses under the head business income:
1. PROFIT ON DEBT:
(a) profit on debt in the tax year to the extent that the proceeds or benefit of the debt used for the business
purposes;

128 Conceptual Approach to Taxes


Income From Business Chapter-09

(b) any amount incurred in the tax year to a modaraba or a participation term certificate holder for funds
borrowed and used for the business purposes;
(c) any amount incurred by a scheduled bank in the tax year on profit or loss sharing account or a deposit with the
bank as a distribution of profits;
(d) any amount incurred by HBFC, NDLC, SME bank in the tax year on profit and loss basis the share in profits
to the SBP on its investment in property, leasing operations and credit line respectively.
(e) any amount incurred by a person in the tax year on profit and loss basis to a banking company under a scheme
of musharika;
(f) any amount incurred by a person in the tax year to a certificate holder under a musharika as share in the profits
of the musharika; or
2. LEASE RENT:

Lease rental expenses for the year to a scheduled bank, financial institution, an approved modaraba, an approved
leasing company or a Special Purpose Vehicle on behalf of the Originator for use of an asset for business purposes;
3. FINANCIAL COST:
The financial cost of the securitization of receivables incurred by an Originator in the tax year from a Special
Purpose Vehicle being the difference between the amount received by the Originator and the amount of receivable
securitized from a Special Purpose Vehicle.
Where assets are transferred by an Originator on securitisation to a Special Purpose Vehicle, it shall be treated as a
financing transaction irrespective of the method of accounting adopted by the Originator.

Example: Following information is related to Mr Bilal who carries on the business of trading clothes.
(a) Sales Rs. 740,000,
(b) Cost of sales Rs. 550,000,
(c) Administrative expenses Rs. 110,000
Notes:
Administrative and selling expenses include:
(a) Accounting depreciation Rs. 10,000,
(b) Interest on loan (for business purpose) Rs. 20,000,
(c) Interest on loan (for personal purpose) Rs. 5,000
Required: Compute taxable income of the taxpayer if the tax depreciation Rs.15,000.
Solution:
Mr Bilal
Computation of taxable income: Rs.
Income from business:
Sales 740,000
Cost of sales (550,000)
Gross profit 190,000
Less: Administrative expenses (110,000)
Add: accounting depreciation 10,000
Add: Interest on loan for personal use 5,000
Less: Tax depreciation (15,000)
Taxable income 80,000
Irrespective of the method of accounting adopted by the originator, where as a result of securitization any assets are
transferred by him to a SPV it shall be treated as a financing transaction. [S 28(2)]
10. Bad debts [U/s 29(1) and (2)]
A person shall be allowed a deduction for bad debts, in a tax year if the bad debts which were previously included
in incomes or in respect of money lent by a financial institution have now been written off and reasonable
grounds exist that they cannot be recovered.
The amount of allowed deduction for a tax year shall not exceed the amount of the debt written off in the accounts of
the person in the tax year.

Conceptual Approach to Taxes 129


Income From Business Chapter-09

Recovery of bad debts in subsequent period or periods [U/s 29(3)]


If there is any recovery after allowing bad debts out of the allowed portion of bad debts and the recovery has
been accounted for in the books of account for accounting purposes, then the same shall be ignored for
computation of taxable business income in the tax year in which the recovery shall be made.
If there is any recovery after allowing bad debts out of the disallowed portion of bad debts and the recovery
has been accounted for in the books of account for accounting purposes, then the same shall be reversed for
computation of taxable business income in the tax year in which the recovery shall be made.
11. Deductions not allowed [U/s 21]
No deduction shall be allowed in computing the income of a person under the head business income in respect of the
following expenses.
(a) any cess (charge on the profit), rate or tax paid or payable in Pakistan or a foreign country that is levied
or assessed as percentage on the business profits (exceptions separately given);
(b) any amount of tax deducted from an amount derived by the person;
(c) Payments (including for the year, last year or advance) against salary, rent, brokerage or commission, profit
on debt, payment to non-resident, payment for services or fee paid where the same are either without
deduction of tax or tax deducted but not deposited;
(d) Entertainment expenditure shall not be allowed unless related directly to the business and fulfilling the
following limits specified under Rule 10:
Expenditure incurred outside Pakistan on entertainment or where such expenditure related to head
office expenditure;
Expenditure incurred in Pakistan on entertainment of foreign customers and suppliers, customers and
clients at business premises;
Expenditure incurred on entertainment at a meeting of shareholders, agents, directors or
employees; or
Expenditure incurred on entertainment at the opening of branches.
Entertainment expenditure includes meals, refreshments and reasonable leisure facilities as per
norms and customs of the business in Pakistan.
(e) any contribution made by the person to a fund that is not a recognized provident fund, approved pension fund,
approved superannuation fund, or approved gratuity fund;
(f) any contribution made by the person to any provident or other fund established for the benefit of employees
without deduction of tax u/s 149 from any payments;
(g) any fine or penalty paid or payable for the violation of any law, rule or regulation (exceptions given separately);
(h) any personal or capital nature expenditures;
(i) any amount carried to a reserve fund or capitalised in any way;
(j) any payment by an AOP of profit on debt, brokerage, commission, salary or other remuneration to its members
(separate important note given);
(k) any expenditure paid or payable under a single account head in aggregate exceeds fifty thousand rupees,
made other than by a crossed cheque or by crossed bank draft or crossed pay order or any other crossed
banking instrument showing transfer of amount from the business bank account of the taxpayer:
The online transfer and payments through credit cards shall be treated as through banking channel if the
same are verifiable from the business account of the payer to the business account of payee:
Provided further that this clause shall not apply in the case of-
Expenditures not exceeding Rs. 10,000;
expenditures of utility bills, freight charges, travel fare, postage and payment of taxes, duties, fee, fines
or any other statutory obligation;
(l) any salary paid or payable exceeding Rs.15,000 per month other than by a crossed cheque or direct transfer
of funds to the employee's bank account;
Example: Profit before tax of Sigma (Pvt) Ltd is Rs. 1,000,000. Following items have been included in the
computation of profit.
(a) Income tax paid (Tax deducted at source by others) Rs. 40,000
(b) Taxable salaries paid to employees without deduction of tax Rs. 425,000
(c) Donation to unapproved institution Rs. 20,000
Required: Compute taxable profit according to the provisions of Income Tax Ordinance, 2001.

130 Conceptual Approach to Taxes


Income From Business Chapter-09

Solution:
Taxable profit: Rs.
Profit before tax 1,000,000
Add: deductions not admissible:
Tax paid or deducted at source 40,000
Payment of salaries without deduction of tax 425,000
Donation to unapproved institution 20,000 485,000
Taxable profit 1,485,000
Example: The following payments of expenses made otherwise through crossed Cheque.
Nature of payment Rs.
Rent of one month paid to an AOP for Karachi rented office 50,000
Air tickets purchased 100,000
Payment of monthly wages of Mr. B 14,000
Bill paid for repair of car 22,000
Electricity bill paid 115,000
Telephone bill paid 48,000
Paid professional tax 30,000
Paid audit fee 60,000
Paid tax consultant 19,000
Compute the addition u/s 21(1) of the Income Tax Ordinance, 2001
Solution:
Calculation of addition u/s 21(1) Rs.

Rent paid to an AOP for Karachi rented office (N-5) 50,000


Air tickets purchased (N-1) -
Payment of wages of Mr. B (N-3) -
Bill paid for repair of car (N-2) -
Electricity bill paid (N-1) -
Telephone bill paid (N-1) -
Paid professional tax (N-1) -
Paid audit fee (N-4) 60,000
Paid tax consultant (N-2) -
Total inadmissible expenses 110,000

N-1 Condition of payment through cross cheques is not applicable on the said transactions.
N-2 Although single transaction exceeding Rs.10,000 but as the total under single head of account is not more
than Rs.50,000 therefore on the said transactions the provisions of section 21 are not applicable.
N-3 Wages less than Rs.15,000 per month are not required to be paid through banking channel.
N-4 These expenses are required to be paid through banking channel.
N-5 It has been assumed that other months payment against rent was made through banking channels.
Important notes on section 21
11.1 Tax, Charge or levy paid to the government [U/s 21(a)]
Although any cess (charge on the profit), rate or tax paid or payable in Pakistan or a foreign country that is
levied or assessed as percentage on the business profits is not admissible. However Workers Welfare Fund
and Workers Profit Participation Fund are payable on profit but the same are specifically allowed as deductible
allowances u/s 60A and 60B.
11.2 Penalty or Fine [U/s 21(g)]
Any fine or penalty for the violation of any law, rule or regulation is not admissible. However certain payments are
allowable as tax expenses which are not for the violation of any law, rule or regulation e.g.
1. Compensation for late repayment of loan instalment
2. Demurrage to custom authorities
3. Compensation / interest for breach of a contract in the ordinary course of business

Example: Profit before tax of Beta (Pvt.) Ltd. is Rs.1,000,000. Following items have been included in the
computation of profit.
(a) Income tax paid Rs. 40,000

Conceptual Approach to Taxes 131


Income From Business Chapter-09

(b) WWF paid Rs. 25,000


(c) Penalty for late filing of sales tax returns Rs. 15,000
(d) Penalty for late repayment of loan instalment 2,000
Required: Compute taxable profit according to the provisions of Income Tax Ordinance, 2001.
Solution:
Taxable profit: Rs.
Profit before tax 1,000,000
Add: deductions not admissible:
Tax paid or deducted at source 40,000
Penalty for late filing of sales tax returns 15,000
55,000
Taxable profit 1,055,000
11.3 Certain Payments to a member by an Association of Persons [U/s 21(j)]
Profit on debt, brokerage, commission, salary or other remuneration by an AOP to its members is not admissible for
tax purposes.
However rent paid by a firm to partner for his residence is a part of salary expense of the firm and therefore not
admissible for tax purpose. However, if a partner's premises are used by the firm and rent is paid to the partner then
the same shall be treated as rent allowable tax expense and need not be added back in the income of the AOP.
Example on determination of share from AOP:
XYZ is an AOP with two partners Mr. Ajmal and Mr. Khan with ratio of sharing 40:60.
Rupees
Net profit as per accounts 1,200,000
Depreciation as per accounts 600,000
Salary to partner (Mr. Khan) 250,000
Brokerage to partner (Mr. Khan) 100,000
2,150,000
Less: Tax depreciation 900,000
Taxable Income 1,250,000
Income Tax [35,000 + 15% (1,250,000 750,000)] 110,000
Divisible Income 1,140,000
Share of profit from AOP shall be computed as under:

Partner Ajmal Partner Khan Total


Salary Nil 250,000 250,000
Brokerage Nil 100,000 100,000
Balance 316,000 474,000 790,000
316,000 824,000 1,140,000
Although share of profit from AOP (after tax) is exempt in the hands of individual members but the same shall be
included for rate purpose in other taxable income except income taxed under FTR / SBI. One should observe that
salary is included in the share of profit from AOP in the above calculation. In fact, if a member of an AOP receives
salary or other remuneration, interest, commission or brokerage from the same AOP it becomes part and
parcel of share of profit from AOP and not considered as a separate Income.
12. Deductible Allowances
The following allowances shall be deducted from total income for computation of taxable income for the tax year.
1. Zakat (u/s 60):
Where a person has paid any amount as Zakat (other than Zakat deducted on profit on debt) then he shall be
entitled to deduct this amount from his income chargeable to tax.
If the amount of Zakat is more than the income chargeable to tax then excess shall not be refunded, carried
forward or carried back.
2. Workers Welfare Fund (u/s 60A):
A person shall be entitled to a deductible allowance for the amount of any Workers Fund paid by the person in
tax year under Workers Welfare Fund Ordinance, 1971. WWF is chargeable and payable along-with income
tax return of a person.

132 Conceptual Approach to Taxes


Income From Business Chapter-09

Section 4 of the said Act covers the following:


Every industrial establishment, the total income of which in any year of account commencing on or after the date
specified by the FG in the official Gazette in this behalf is not less than Rs. 500,000 shall pay to the fund in respect of
that year a sum equal to 2% of so much of its total income as is assessable under the Ordinance.
Every industrial establishment which is liable as above shall pay the amount due from it to the taxation Officer having
jurisdiction over the industrial establishment for purposes of the Ordinance.
The industrial establishment shall on or before the date prescribed for filing of returns of income or a statement
under the Ordinance, pay the amount due from it as above and furnish the proof of payment to the Taxation Officer or the
Commissioner as the case may be:
The term total income is defined as under in section 2(h) of the Act; and
(a) where return of income is required to be filed under the Ordinance, the profit before tax as per accounts or the
declared income as per the return of income, whichever is higher; and
(b) where return of Income is not required to be filed, the profit before tax as per accounts or 4% of the receipt as
per the statement filed u/s 115 of the Ordinance, whichever is higher.
Considering the above the salient features of the levy of WWF are as under:
WWF is chargeable on every Industrial undertaking whose taxable income exceeds the monetary threshold of
Rs. 500,000 and not on service sector taxpayers. An industrial undertaking is an undertaking which is engaged
in the manufacturing and subjection of goods either manually, electronically or through any other mode
WWF is equally chargeable on the incomes liable to tax under NTR and FTR.
Supreme Court of Pakistan has already held that the WWF is not chargeable on the exempt income.
WWF is not chargeable on the Government corporations and Institutions.
Rate of WWF is 2% of total income and u/s 60A of the Ordinance the same is an admissible deduction against
the total income of the person.
Circular No 1 of 2004 dated 3 January 2004 clarifies that the WWF payable expense is also admissible deduction
allowance in the cases where accrual based accounting is employed as regular method of accounting.
Example: Taxable income of Monika industries is Rs 800,000. Accounting profit of the company is Rs.625,000.
Compute the WWF on the taxable income of the company.
Rs.
Taxable income 800,000
WWF @ 2% 16,000
Taxable income 784,000
Example: Following information is related to Abbas Industries (Pvt) Ltd.
Sales 740,000
Cost of sales 550,000
Administrative expenses 110,000
Zakat 20,000
WWF 10,000
Notes:
1. Administrative and selling expenses include:
(a) Accounting depreciation 10,000
(b) Penalty for violation of law 2,500
2. Tax depreciation 15,000
Required: Compute taxable income of the taxpayer.
Solution:
Abbas Industries (Pvt) Ltd
Computation of taxable income: Rs.
Income from business:
Sales 740,000
Cost of sales (550,000)
Gross profit 190,000
Less: Administrative expenses (110,000)
Add: accounting depreciation 10,000

Conceptual Approach to Taxes 133


Income From Business Chapter-09

Add: Penalty for violation of law 2,500


Less: Tax depreciation (15,000)
Total income 77,500
Less: Deductible allowance:
Zakat (20,000)
WWF (10,000)
Taxable income 47,500
3. Workers Participation Fund (u/s 60B):

A person shall be entitled to a deductible allowance for the amount of any Workers Participation Fund paid by the
person in tax year under Companies Profit (Workers Participation) Act, 1968.
In view of the above, any sum paid for Workers Profit Participation Fund is admissible deductible allowance. It is
important to place on record that under clause (4)(d) of the Schedule to the Companies Profit (Workers Participation)
Act, 1968, any amount of undistributed workers profit participation fund will be paid as Workers Welfare Fund to the
Tax Authorities.
Pakistan source & Foreign source Income from business

Geographical source of Income from business Received by Taxability


Pakistan source business income [Section 101]:
1. A permanent establishment (PE) of the non- Taxable [Section 11(5)
resident person in Pakistan; and (6)]
2. Sales in Pakistan of goods merchandise of the Term and conditions of
same or similar kind as sold by the person Resident / non-resident double taxation treaty
through a PE in Pakistan; individual agreements shall also be
3. Other business activities carried on in Pakistan of considered for non
the same or similar kind as those affected by the residents having Pakistan
non-resident through a PE in Pakistan; or source income.
4. Any business connection in Pakistan
a. Resident Individual Taxable [Section 11(6)]

[Section 50]
An individual shall be
exempt in respect of his
Foreign source business income:
b. Short term resident foreign-source income which
Income from business other than above is not brought / received in
[For all foreign source Pakistan if he is resident
income] only by reason of his
employment and he is
present in Pakistan for not
exceeding 3 years.
[Section 51]
c. Returning expatriate If an individual citizen of
Pakistan (returning
expatriate) is resident in the
[Citizen of Pakistan current tax year but was
coming back in non-resident in the 4
Pakistan] preceding tax years, his
foreign-source income shall
[For all foreign source
be exempt in current tax
income]
year and in the following tax
year.
Non-resident individual Not taxable [Section 11(6)]

134 Conceptual Approach to Taxes


Income From Business Chapter-09

PART II (For CA Mod F and ICMAP students)


13. Provision regarding consumer loans [U/s 29A]
A non-banking finance company or the HBFC shall be allowed a deduction, not exceeding 3% of the income for
the tax year, arising out of consumer loans for creation of a reserve to off-set bad debts arising out of such loans.
Where bad debt cannot be wholly set off against reserve, any amount of bad debt, exceeding the reserves shall be
carried forward for adjustment against the reserve for the following years.
"Consumer loan" means a loan of money or its equivalent made by a non-banking finance company or the HBFC to
a debtor (consumer) and the loan is entered primarily for personal, family or household purposes and includes
debts created by the use of a lender credit card or similar arrangement as well as insurance premium
financing.
14. Profit on non-performing debts of a banking company or development finance institution [U/s 30]
A banking company or development finance institution or Non-Banking Finance Company (NBFC) or
modaraba shall be allowed a deduction for any profit accruing on a non-performing debt where the profit is credited
to a suspense account in accordance with the Prudential Regulations for the above named category, issued by
the SBP or the SECP.
Any profit deducted as above that is subsequently recovered shall be included in the income of the company or
institution or NBFC or modaraba chargeable under the head "Income from Business" for the tax year in which it is
recovered.
15. Transfer to participatory reserve [U/s 31]
A company shall be allowed a deduction for a tax year for any amount transferred by the company in the year to a
participatory reserve created u/s 120 of the CO, 1984 in accordance with an agreement relating to participatory
redeemable capital entered into between the company and a banking company as defined in the Financial
Institutions (Recovery of finances), Ordinance, 2001.
The deduction allowed as above for a tax year shall be limited to 5% of the value of the company's participatory
redeemable capital.
No deduction shall be allowed as above if the amount of the tax exempted accumulation in the participatory reserve
exceeds 10% of the amount of the participatory redeemable capital.
Where any amount accumulated in the participatory reserve has been allowed as a deduction as above is applied
by the company towards any purpose other than payment of share of profit on the participatory redeemable
capital the amount so applied shall be included in the income from business of the company in the tax year in
which it is so applied.

Example: A company has received amount as Participatory redeemable capital Rs.10,000,000 in the tax year 2016.
You are required to compute the amount that should be allowed as expense against business income of the
Company u/s 31 under the following situations
A. If the amount transferred during the tax year 2016 to Participatory reserve is 5% of the amount received.
B. If the amount transferred to Participatory reserve is 3% of the amount received.
C. If the amount transferred to Participatory reserve is 8% of the amount received.
D. If the accumulated balance in Participatory reserve is 10%.
F. If the accumulated balance in Participatory reserve is 14%.
G. If the amount transferred during the tax year 2016 to Participatory reserve is 5% of the amount received
however the amount was used other than to redeem capital.

Solution: Under case A, B, C and D the company shall be allowed expense equal to 5% of Participatory reserve of
the respective years. However in case of E and F the company shall not be allowed any expense as it exceeds from
maximum limit of 10% and used for any purpose other than for redemption of redeemable capital.

Conceptual Approach to Taxes 135


Income From Business Chapter-09

17. EXEMPTIONS FROM TOTAL INCOME [PART I OF SECOND SCHEDULE TO THE INCOME TAX ORDINANCE,
2001]

CLAUSE PARTICULARS OF EXEMPTION

9A Reduces tax to 2.5% in case of supply of locally produced motor cars

66 Income of certain charitable and other institutions


91 Income of Text Book Board
98 Income of Sports Boards (except Pakistan Cricket Board)
99 Income of Collective Investment or a REIT Scheme
99A Sale of immoveable property to Real Estate Investment Trusts
101 Profits of a Joint Venture Capital co. and Fund
107 Income of Subsidiary of Islamic Development Bank
107A Income derived by the Islamic development bank
117 Income from transport business in Azad Kashmir
126 Public Sector Universities
126A Income re: Gawadar services etc.
126B Profit and gains derived by coastal oil refinery at Khalifa point by IPIC for 20 years
126C Profits and gains derived by a taxpayer from an industrial undertaking set up in Larkano industrial estate
126D Profits and gains derived by a taxpayer from an industrial undertaking set up in Gawadar industrial estate
126E Profits and gains derived by corporate zone developers
126H Profits and gains derived by fruit processing, Malakand, Gilgit Baltistan and FATA
126I to 126K Profits and gains derived from industrial undertakings
131 Foreign income in respect of royalty, commission or fee etc.
132 Income of Private sector Power projects
132A Profits and gains derived by Bosicor oil Pakistan Limited
132B Coal mining project in Sindh
133 Income from export of computer software and IT services
136 Income of a special purpose vehicle for securitization of debts

REDUCTION IN TAX RATES [PART II OF SECOND SCHEDULE TO THE INCOME TAX ORDINANCE, 2001]

CLAUSE PARTICULARS OF EXEMPTION

2 Letting of petroleum pipelines


3 Services rendered outside Pakistan

REDUCTION IN TAX LIABILITY [PART III OF SECOND SCHEDULE TO THE ITO, 2001]

CLAUSE PARTICULARS OF EXEMPTION


The amount of tax payable, in a year in which the rupee is revalued or devalued, by a taxpayer
whose profits or gains are computed in accordance with the rules contained in the Fifth Schedule to the
2 Ordinance and who had entered with the Government into an agreement which provides for such reduction,
shall be reduced to the amount that would be payable in the absence of the revaluation or
devaluation of the rupee.
In respect of old and used automotive vehicles, tax under section 148 shall not exceed the amount specified
4 in Notification No. S.R.O. 577(I)/2005, dated the 6th June, 2005.

136 Conceptual Approach to Taxes


Income From Business Chapter-09

PRACTICE QUESTIONS WITH SOLUTIONS


Q. NO. 1 Net loss for the tax year 2016 of an AOP (BS and Co.) as per accounts before tax is Rs.(1,000,000). You are
required to compute the taxable income of the AOP from following information:
Administrative expenses include the following:
1. Tax deducted on cash withdrawal Rs. 37,000. 2. Salary paid to partners Rs.150,000. 3. Interest on loan from partner
Rs. 20,000. 4. Provision for bad debts 10,000. 5. Charity to beggar Rs.37,000. 6. Donation in kind to approved institutions
worth Rs. 24,000 7. Depreciation as per accounts Rs.195,000 8. Property tax on personal property of current year Rs.
55,000. 9. Statutory depreciation is Rs. 237,000
Solution: BS and Co.
Computation of taxable income
For the tax year 2016
Rs.
Loss as per accounts before tax (1,000,000)
Less:
Tax deducted on cash withdrawal 37,000
Salary paid to partners 150,000
Interest on loan from partner 20,000
Provision for bad debts 10,000
Charity to beggar 37,000
Donation to approved institution 24,000
Depreciation as per accounts 195,000
Property tax on personal property 55,000
528,000
Add:
Tax depreciation 237,000
Loss as per tax (709,000 )
Note: Donation to approved institution is not admissible expense and the firm cannot claim tax credit as there is loss during
the year.
Q NO.2 Explain whether the following are admissible as business expenditure under the Income tax Ordinance, 2001:
(a) Repayment of principal amount of lease rentals of plant and machinery.
(b) Sales Tax paid on the purchase of raw material to be used in the production of exempt supply
(c) Dividend paid by company
(d) Provision in respect of doubtful debts
(e) Penalty for late filing of sales tax return
Solution:
(a) Repayment of principal amount of lease rentals is admissible.
(b) This is also admissible expense as it cannot be claimed as input tax by the business.
(c) This is not admissible expense.
(d) Provision in respect of doubtful debts is not admissible.
(e) Penalty for the violation of any law or rule is not admissible expense.

Conceptual Approach to Taxes 137


Income From Business Chapter-09

MULTIPLE CHOICE QUESTIONS


Q.1 Any payment made on account of salary exceeding Rs.15,000 per month is not allowed as a deduction against
business income if made through
(a) cross cheque
(b) online transfer
(c) cash
(d) pay order
Q.2 Any donation to an approved institution is allowed as straight deduction against business income by
(a) Full amount
(b) Half amount
(c) Proportionate basis
(d) Amount calculated on average rate basis
Q.3 A deduction against business income is allowed for
(a) Provision for bad debts
(b) Actual bad debts
(c) Appropriation of profits
Q.4 Income tax is an
(a) Expense
(b) Provision against profits
(c) Appropriation of profits
Q.5 The expenditure or loss that was allowed as deduction if subsequently received is included in the income of the year
(a) In which it was allowed as expense
(b) In which it is received
(c) Following the year of receipt
(d) None of these
Q.6 A company is allowed a deduction on account of depreciation for
(a) Depreciable assets
(b) Intangibles
(c) Pre-commencement expenditures
(d) All of these
Q.7 Cash discount received is taxable under the head
(a) Income from Business
(b) Income from Capital Gains
(c) Income from other sources
(d) Not taxable
Q.8 Income from Business may be taxable under
(a) FTR
(b) NTR
(c) Separate block
(d) All of the above
(e) both (a) and (b)
Q.9 An income received by the lessor in connection with the lease is taxable under
(a) Income from property
(b) Income from Business
(c) Income from other sources
(d) None of these

138 Conceptual Approach to Taxes


Income From Business Chapter-09

Q.10 Income from business comprises of


(a) Income from speculation business
(b) Income from non-speculation business
(c) Both a and b
(d) None of these
Q.11 A person is required to deduct tax at source, where applicable in respect of payments against
(a) Salary
(b) Rent
(c) Dividend
(d) All of these
(e) only (a) and (c)
Q.12 _______ allowed as deduction against business income
(a) Accounting depreciation
(b) Tax depreciation
(c) Both of these
(d) None of these
Q.13 Any payment made to the employees as _______ will be allowed as deduction to employer.
(a) Brokerage
(b) Commission
(c) Salary
(d) all of the above
Q.14 Single transaction not exceeding _____ are allowed as deduction against business income
(a) 5,000
(b) 1,000
(c) 10,000
(d) 20,000
Q.15 Income tax is allowed as deduction against business income in the year
(a) In which income is earned
(b) In the following year
(c) Not allowed
(d) None of these
Q.16 Apportionment of common expenditure is necessary when income is derived from _______ class of income
(a) One
(b) Two
(c) Three
(d) More than one
Q.17 If a liability in respect of an expense allowed as deduction is not paid within ______, it shall be treated as income in
the year on expiry of the said ____________.
(a) One year
(b) Two years
(c) Three years
(d) Four years
Q.18 In case the asset is not owned by the lessor, amount received in connection with the lease is taxable under
(a) Income from property
(b) Income from business
(c) Income from other sources

Conceptual Approach to Taxes 139


Income From Business Chapter-09

(d) Capital gains


Q.19 The amount of deduction allowed on account of bad debts may be _______ to trade debtors
(a) Higher
(b) Less
(c) Equal
(d) Both b and c
Q.20 An amalgamation company is allowed a deduction on account of __________
(a) Legal expenses
(b) Selling expenses
(c) Management expenses
(d) None of these
(e) only (a) and (c)
Q.21 Where a person has been allowed a deduction in respect of a trading liability then the benefits from such liability shall
be chargeable to tax under____________.
(a) Business Income
(b) Income from other sources
(c) none of these.
Q.22 Any expenditure incurred on training of any Pakistani in connection with a scheme specially approved by Board for
the purpose of business is __________
(a) Deductible
(b) not deductible
Q.23 Expenditure incurred on the rental of a building for the scientific research, outside the Pakistan are __________
(a) allowable expenditure under IFB
(b) Allowable Expenditure under Income from other Sources
(c) not allowable
Q.24 If the lease rentals have been paid to the ____________then it shall be an allowable expenditure to the taxpayer.
(a) Schedule Banks
(b) modaraba
(c) Both of them
Q.25 __________ value of any benefit or perquisite arising from the past, present or prospective business relationship is
chargeable under the head
(a) Business Income
(b) Income from other sources
(c) none of these
Q.26 The function of buying and selling of financial securities by the dealer is called __________
(a) Jobbing
(b) arbitrage
(c) both
Q.27 Repair Charges and insurance premium of any asset used for the business and private purposes are _______under
the head of business income
(a) Allowable
(b) not allowable
(c) Proportionately allowed
Q.28 Provisions and P and L appropriations items are ________ expense.
(a) allowable
(b) not allowable
(d) manufacturing

140 Conceptual Approach to Taxes


Income From Business Chapter-09

(d) factory
Q.29 Which of the following expenses is not admissible?
(a) provision for doubtful debts
(b) bad debts
(c) penalty
(d) both a and c
Q.30 Depreciation is allowed as per ___________.
(a) accounts
(b) tax
(c) estimates
(d) not allowed
Q.31 Which of the following donations are allowed as straight deduction?
(a) donation to unapproved institution
(b) donation to institution under section 61
(c) donation to institution specified in clause 61
(d) none of above
Q.32 In an AOP the members are allowed only to receive ___________.
(a) salary
(b) commission
(c) profit on debt
(d) none of above
Q.33 Depreciation is allowed as per _____ schedule of the Income Tax Ordinance, 2001
(a) 1st
(b) 2nd
(c) 3rd
(d) 4th
Q.34 Expenses other than utility, postage and freight charges must be paid through cross cheques where any of them
exceeds Rs.______.
(a) 8,000
(b) 10,000
(c) 5,000
(d) 6,000
Q.35 Any expenditure is admissible if the same is wholly and exclusively __________.
(a) for business
(b) for personal use
(c) for personal expenses of members in case of AOP or company
(d) all of above
Q.36 Where the bad debts are recovered from the amount not allowed earlier then the same would be ___________.
(a) included in taxable income
(b) ignored
(c) reversed from accounting profit
(d) all of above
Q.37 Any amount received by a banking company or a non-Banking finance company out of profit on debt is taxable under
income from _________.
(a) salary
(b) property

Conceptual Approach to Taxes 141


Income From Business Chapter-09

(c) business
(d) other sources
Q.38 Expenses on entertainment of foreign customers and suppliers are ________.
(a) admissible
(b) not admissible
(c) both a and b
(d) none of above
Q.39 Any expenditure (unless otherwise stated) under a single account head exceeding Rs.50,000 is admissible if the
payment is made only through ___________.
(a) cash
(b) cross cheque
(c) bearer cheque
(d) all of above
Q.40 For tax purposes, business does not include ___.
(a) employment
(b) trading
(c) manufacturing
(d) service providing
Q.41 Income from speculation business is kept separate from _____ business incomes.
(a) non-speculation business
(b) other speculation business
(c) trading business
(d) business of service providing
Q.42 Arbitrage is a _________________.
(a) speculation business
(b) non-speculation business
(c) illegal business
(d) none of above
Q.43 Salary paid to sleeping partner of AOP is __________.
(a) admissible
(b) not admissible
(c) none of above
Q.44 Any amalgamated company is allowed a deduction against ________ in respect of all expenses incurred by it.
(a) business income
(b) salary income
(c) property income
(d) all of above
Q.45 The financial cost of securitization incurred by the ___ is allowed as an admissible deduction.
(a) APV
(b) SPV
(c) bank
(d) financial institution
Q.46 Which of the following is allowed as deductible allowance against business income?
(a) WWF
(b) WPPF
(c) Zakat under Zakat and Usher Ordinance
(d) all of above
142 Conceptual Approach to Taxes
Income From Business Chapter-09

Q.47 Tax credit is allowed on donation to ___________.


(a) Beggars
(b) financial institution
(c) banks
(d) approved institution u/s 61
Q.48 Any expenditure in nature of fine or penalties for violation of any law is _________ against business income.
(a) Admissible
(b) inadmissible
(b) both a or b
(c) none of above
Q.49 _______ receipts mean all receipts less deduction of expenditures.
(a) Gross
(b) net
(c) final
(d) all of above
Q.50 If any expenditure is allowed as business expense however the liability against the same has not been paid within
____ years including the year of its allow-ability then the same will be treated as income of the taxpayer after expiry of
the ____ years.
(a) 5
(b) 3
(c) 10
(d) 15
Q.51 If a deduction is allowed as expense and subsequently any benefit against the same, such as cash discount received
then the same will be taxable under the head income from _____.
(a) Business
(b) other sources
(c) salary
(d) property

ANSWERS
1 (c) 2 (d) 3 (b) 4 (b) 5 (b)
6 (a) 7 (a) 8 (d) 9 (b) 10 (c)
11 (a) 12 (b) 13 (d) 14 (c) 15 (c)
16 (d) 17 (c) 18 (b) 19 (d) 20 (e)
21 (a) 22 (a) 23 (c) 24 (c) 25 (a)
26 (b) 27 (c) 28 (b) 29 (d) 30 (b)
31 (c) 32 (d) 33 (c) 34 (b) 35 (a)
36 (c) 37 (c) 38 (a) 39 (b) 40 (a)
41 (a) 42 (b) 43 (b) 44 (a) 45 (b)
46 (d) 47 (d) 48 (b) 49 (b) 50 (b)
51 (a)

Conceptual Approach to Taxes 143


Income From Business Chapter-09

ICMAP PAST PAPERS THEORECTICAL QUESTIONS


Q. No. 2 (c) Spring 2013 M/s. National Corporation is a non-banking finance company. The main business of the company
is to provide a loan to consumers for small businesses, car financing, personal and household purposes.
Required: The corporation is seeking your advice in respect of the following under the provision of the Income Tax
Ordinance, 2001:
(i) M/s National Corporation is willing to segregate consumer loan and other loans, therefore, want to know the clear
definition of consumer loan. Define.
(ii) Explain how the provision against consumer loan is allowed as deduction to the corporation.
(iii) Where bad debt in respect of consumer loan cannot be wholly set off against reserve created for this purpose,
then what would be the treatment for setting off the bad debt?
Q. No. 3(a) February 2013 Write short answers of the following questions:
(iii) Whether the income of a university or an educational institution established for educational purpose and making profit is
exempt from income tax or not ?
(iv) Upto what period the income from export of computer software of IT Services or IT enabled services are exempt from
income tax?
Q. No. 3(b) February 2013 M/s. Kotton Limited, a holding company of five companies is engaged in the business of cotton
farming, ginning, spinning, weaving and trading. Approximately 60% of the output of each company is consumed by the
next follower, The remaining 40% is being sold in the open market through the trading company out of which 60% is
exported and 40% is sold in the market. Mr. Zaidi, Director Finance of the group of companies has received a notice from tax
department regarding applicability of section 108 of the Income Tax Ordinance, 2001' transactions between associates' Mr.
Zaidi approached you to seek advice about the following:
(i) What are the methods which may be applied to determine arm's length results under the Income Tax Rules, 2002?
(ii) Which method will be used where the arm's length results cannot be reliably determined under any of the methods as
mentioned at (i) above?
Q. No. 3 (a) August 2012 As per section 20 of the Income Tax Ordinance, 2001 a deduction is allowed in computing the
income of a person chargeable to tax under the head Income from Business for a tax year for any expenditure incurred
wholly and exclusively for the purpose of business. Rule 10 of the Income Tax Rules, 2001 laid down some conditions on a
deduction for entertainment expenditure incurred by a person.
Required:
(i) What are the conditions laid down in the rule 10 of the Income Tax Rules, 2002 for the deduction of entertainment
expenditures?
(ii) What is the benchmark for the allow-ability of the entertainment expenditures under the above rule?
(iii) Define the term entertainment under the rule 10 of the Income Tax Rules, 2002.
Q.3 (a) SUMMER 2012 As per Section 29 of the Income Tax Ordinance, 2001 relating to Bad Debt reply the following:
(i) Under what conditions a person shall be allowed deduction for bad debts in a tax year?
(ii) What will be the maximum amount of deduction?
(iii) Where a person was allowed a deduction in a tax year for bad debt and in a subsequent tax year the person receives
in cash or kind any amount in respect of that debt, how this amount shall be treated?
Q. NO. 3 (a) SUMMER 2010 As per Rule 10 of the Income Tax Rules, 2002:
(i) Define the term entertainment expenditure.
(ii) What is the condition of admissibility of entertainment expenditure?
(iii) What are the limitations on the deduction of entertainment expenditure?
Q. NO. 3 (a) WINTER 2009 Explain the provisions of section 29 of the ITO, 2001 with regard to recovery of bad debts in
subsequent years.
Q. NO. 2 (a) SUMMER 2007 Discuss the provisions u/s 29 of the ITO, 2001, regarding deduction of bad debts.
Q. NO. 2 (b) SUMMER 2006 State the expenses, which are not allowed as deduction while calculating the income under the
head Income from business under ITO, 2001.
Q. NO. 2 WINTER 2005 What are the incomes of a person for a tax year other than income exempt from tax & chargeable
to tax under head Income from Business
Q. NO. 3 SUMMER 2003 Discuss the deductions not allowed in computing income chargeable under the head Income from
Business.

144 Conceptual Approach to Taxes


Income From Business Chapter-09

CA MOD C PAST PAPERS THEORECTICAL QUESTIONS


Q.No.2 (b) Autumn 2014 Briefly discuss the provisions of Income Tax Ordinance, 2001 in respect of the following situations:
ABC (Private) Limited has decided to provide a loan of Rs. 5 million to one of its shareholders, for the purchase of a house.

Q. No. 5 (a) Spring 2012 Tamba Pakistan (Pvt.) Limited is engaged in the manufacture of pharmaceutical products. Its
board of directors has approved a 3-year loan to one of its major shareholders.
Required: Explain the tax implications of the above transaction on the company as well as the shareholder.
Q.3 Spring 2011 Carrot Ltd (CL) is engaged in the manufacture, import and sale of electronic appliances for the past twenty
years. When reviewing the companys tax provisions, you noticed the following amounts appearing in the tax calculation for
the year ended June 30, 20X2.
(i) Profit on debt of Rs. 500,000 paid on a working capital loan obtained from a foreign bank. CL did not deduct
withholding tax while paying profit on debt considering the bank does not have a Permanent Establishment in
Pakistan.
(ii) Expenditure of Rs. 450,000 on promotion of a product which is expected to generate revenue for twelve years.
(iii) Bad debt in respect of a staff loan, Rs. 25,000.
(iv) Reimbursement of expenses of Rs. 300,000 to CL by the parent company. This amount was incurred by CL in 20X1
on marketing a new product imported from Dubai.
(v) Initial allowance of Rs. 4,000,000 on a used equipment acquired locally from MSD Limited.
(vi) Financial charges amounting to Rs. 100,000 and depreciation amounting to Rs. 300,000 on a vehicle acquired on
finance lease from Radish Leasing. Lease rentals paid during the year amounted to Rs. 400,000.
Required: Under the provisions of ITO, 2001 discuss the admissibility of the above amounts for tax purposes.
Q.NO. 4(a) Spring 2008 Discuss the taxability of the following under the Income Tax Ordinance, 2001:
(i) Bad debts
(ii) Non-adjustable rent
(iii) Speculation business
Q.NO. 2 (a) Autumn 2007 What do you understand by the concept apportionment of expenditures as explained in the
Income Tax Ordinance, 2001?
Q.NO. 3 (a) Spring 2006 Explain with reasons, as to whether or not the following expenses are admissible business
expenditures:
(i) Penalty paid by a banking company on contravention of State Bank of Pakistans regulations.
(ii) Freight charges to forwarding agent amounting to Rs 60,000 paid in cash.
(iii) Payment of salary to employee from which tax was not deducted by the employer. However, the employee paid the
tax himself.
(iv) Tax deducted u/s 153 from payments received by a resident person on account of services rendered.
Q. NO. 3 (b) Autumn 2006 One of your clients, Japan and Company, a partnership having three partners, has sent you its
financial statements for the year ended June 30, 2006. Following items are appearing under the head Other income:
(i) Accounting profit on disposal of fixed assets.
(ii) Reversal of provision for doubtful debts related to year ended 30-06-2004
(iii) Dividend received from a listed company.
(iv) Profit on debt.
You are required to explain with reasons as to how the above items will be treated in the computation of taxable income.
Q.NO. 3 (c) Spring 2006 Explain the provisions of section 29 with regard to the recovery of bad debts in subsequent years.
Q.NO. 4 (b) Autumn 2006 Describe conditions mentioned in ITO, 2001 under which a loan will be classified as a consumer
loan.
Q.NO. 5 (a) Autumn 2006 Any income arising from any asset transferred by a person to his spouse is to be treated as
income of the transferor. Describe the circumstances under which this rule shall not be applicable.
Q.NO. 3 Spring 2005 Describe any five types of expenses that are not allowed to deducted under the head income from
business.
Q.NO. 7 Spring 2005 Discuss under what circumstances an expenditure incurred by a person are required to be
apportioned for the purpose of claiming a deduction under the Income Tax Ordinance, 2001?

Conceptual Approach to Taxes 145


Income From Business Chapter-09

Q.NO. 4 Autumn 2004 Describe the expenses which are allowable as a deduction on account of employees training &
facilities?
Q.NO 5 Autumn 2004 Discuss the conditions required to be fulfilled for claiming a deduction on account of bad-debts?
Q.5 Spring 2002 Explain whether the following are admissible as business expenditure under the Income Tax Ordinance
1979:
(a) Repayment of principal amount of lease rentals of plant & machinery.
(b) Sales tax paid on the purchase of raw material to be used in the production of exempt supply.
(c) Dividend
(d) Provision in respect of doubtful debts.
(e) Penalty levied u/s 108 of the Income Tax Ordinance, 1979 for failure to file statement u/s 139.
Q.5 Autumn 2002 Dreamland (Pvt) Ltd. has requested you to advice as regards the important aspects of law for
disallowance of expenses incurred in cash [u/s 24ff] and excess perquisites[u/s 24(i)]. Please write an advisory letter in
this regard explaining the law with suitable examples.
Q.2 Autumn 2001 Discuss briefly the legal position with respect to the admissibility or otherwise of the following as business
expenditure under the Income Tax Ordinance, 1979:
(a) Amount paid as income tax.
(b) Capital expenditure incurred on scientific research in Pakistan.
(c) Share of profit paid to a bank under a scheme of musharika.
(d) Interest paid by a firm to a partner of the firm.
(e) Salary paid otherwise through a crossed bank cheque etc. not exceeding Rs. 5,000..
Q. NO. 4 March 2000 Enumerate 12 in-admissible deductions fort computing income from business or profession contained
in section 21 of the Income Tax Ordinance.
Q. NO. 3 March 1999 Mr. B is MD of a public company. He is master of science in petroleum engineering with a little
background of accounting and tax. He has been informed that the assessing officer may question the method of accounting
of the company in the context of determination of business income. Whilst section 20 of the Income Tax Ordinance specifies
admissible deductions, section 21 of the Ordinance stipulate certain deductions which are not admissible under the law.
MD is not clear about certain things stated above and has asked you, as chief accountant of explain him the following in a
write up:
(a) What types of income are chargeable under the head business income?
(b) What are the six inadmissible deductions u/s 21 of the ordinance for computing business income.
How many types of method of accounting exist and under what circumstances assessing officer may not accept the tax
payers method of accounting.
Q. NO. 2 Nov 1996 Which income shall be chargeable under the head income from business and what deductions are
admissible while computing such income?
Q. NO. 4 Nov 1996 Answer the following statements, considering the keys given therein:
I) Income from business or profession is computed u/s: (a) 19 (b) 18 (c) 22
Q. NO. 9 May 1994 Which of the following appear to be correct in the given choices?
i) Rental income earned by a taxpayer engaged in the business of letting out shops in a large shopping plaza is taxed
as:
(a) Income from business or professions
(b) Property income
(c) Income from other sources
(d) Capital gains
ii) The profit on sale of building used for the purposes of taxpayers business is taxed as:
(a) Capital gains (b) Business income
(c) Exempt from tax (d) Taxed at reduced rate
Q. NO. 9 Nov 1994 Answer the following statement considering the keys given therein:
i) Income from business or profession is computed under: (a) Section 19 (b) Section 18 (c) Section 27

146 Conceptual Approach to Taxes


Assets and Depreciation Chapter-10

Chapter

10 ASSETS AND DEPRECIATION

Topic covered
Section Rule
(For CAF-6 and ICMAP Students)
PART I (FOR CAF-6 & ICMAP STUDENTS)
22 12 & 224 Depreciable asset
23 12 & 224 Initial allowance
23A 12 & 224 First year allowance
23B Accelerated depreciation to alternate energy projects [Part-II of Third Schedule]
24 Intangibles
25 Pre-commencement expenditure
Disposal of assets
Assets of leasing companies
75 Disposal & acquisition of assets
76 Cost of assets under various situations
77 Consideration received under various situations
PART II (FOR CA MOD F & ICMAP STUDENTS)
78 Non-arm's length transactions
79 Non-recognition rules
MCQs with solutions
ICMAP & CA Mod C past papers theoretical questions

PART - I (For CAF-6 and ICMAP Students)


DEPRECIATION
1. Depreciation
Where a person is using depreciable asset to derive income from business, then he shall be allowed to deduct
depreciation of the depreciable asset from his income from business.
2. Depreciable asset [U/s 22(15)]
Any tangible moveable or immovable property (other than unimproved land) or structural improvement to immoveable
property, owned by a person that;
has normal useful life of more than one year;
is likely to lose value as a result of normal wear and tear or obsolescence; and
is used by the person for his business purposes (wholly or partly)
but shall not include any tangible movable property, immovable property, or structural improvement to immovable
property in relation to which a deduction has been allowed under another section of this Ordinance for the entire cost of
the property or improvement in the tax year in which the property is acquired or improvement made by the person;
The depreciable asset includes building, plant and machinery, furniture and fixtures, computer hardware, technical
books, vehicles, air-craft, ships, below ground installation, offshore platforms and production installation in mineral oil
concerns.
Structural Improvement
Structural improvement in relation to immovable property includes any building, road, driveway, car park, railway line,
pipeline, bridge, tunnel, airport runway, canal, dock, wharf, retaining wall, fence, power lines, water or sewerage
pipes, drainage, landscaping or dam.

Conceptual Approach to Taxes 147


Assets and Depreciation Chapter-10

3. Eligible Depreciable asset [U/s 23(5)]


All depreciable assets are eligible depreciable assets except following:
(a) Any road transport vehicle not plying for hire. (Trains and busses are example of vehicles plying for hire.)
(b) Furniture and fittings
(c) Any plant and machinery that has been previously used in Pakistan
(d) Any asset whose total cost has already been allowed as deduction in the tax year in which it is acquired

Particulars to be furnished to claim depreciation / amortisation [Rule 12]


An allowance for depreciation and amortization shall be allowed u/s 22, 23 and 24 on furnishing to CIR the following
particulars and information with the return of income for the tax year:
Description of each depreciable asset and intangible.
The extent of part used for business
In case of acquisition during the tax year, the date of acquisition
The tax WDV of each asset at beginning of tax year
The amount of capital expenditure incurred on addition, alteration, improvement and extensions
Total value of each asset on which depreciation is allowable
WDV at beginning add: capital expenditure during tax year less: initial deprecation allowed, rates and amount
of initial and normal depreciation stating separately, normal useful life for each intangible, total depreciation or
amortization allowed for tax year
WDV of each depreciable asset and the cost of asset or its remaining useful life. and
On disposal of asset the sale proceed of the asset disposed of with WDV at beginning of tax year and the
excess or deficit on disposal.
Practically there is a separate annexure for furnishing of above information that is filed along-with the return of
income.
4. Conditions for leasing companies for claim of depreciation deduction [U/R 224]
The following conditions shall be fulfilled by a leasing company or a modaraba to claim deduction for depreciation on
lease of depreciable assets;-
(i) The leasing company is engaged principally in the business of leasing of assets and has been issued a licence
by the SECP to operate under the terms and conditions specified therein; and
(ii) the leasing company or a modaraba doing leasing business undertakes that where a motor vehicle is given on
lease, the purchase value thereof shall be restricted to the amount specified in the 3rd Schedule to the
Ordinance, for the purposes of claiming depreciation or the expenditure on such lease.

CALCULATION OF DEPRECIATION:
Depreciation shall be allowed only on depreciable assets. Rules regarding calculation of depreciation are as follows:
5. Where an asset not used for the whole of the year:
The depreciation on such asset shall be charged for the whole year e.g. assets used partly on seasonal basis in
sugar industry. [U/s 22(1)]
6. Where the useful life of an asset is one year:
No depreciation allowance shall be allowed however renewal or replacement cost shall be allowed as revenue
expenditure. [U/s 22(15)]
7. Rates of depreciation:
Depreciation shall be computed by applying the following rates against the written down value of the asset at the
beginning of the year [U/s 22(2)]:

SR. Type of Assets Rate


1. Building (all types) 10%
2. Furniture (Including fitting) and machinery and plant, Motor vehicles (all types), ships, technical 15%
or professional books.
3. Computer hardware including printer, monitor and allied items, Machinery and Equipment used 30%
in manufacture of IT products, aircrafts and aero engines, Aerial photographic apparatus.

148 Conceptual Approach to Taxes


Assets and Depreciation Chapter-10

4. In case of mineral oil concerns the income of which is liable to be computed with the rules in
Part I of the Fifth Schedule.
Below ground installations 100%
Offshore platforms and production installations. 20%
5. A ramp build to provide access to persons with disabilities not exceeding Rs.250,000 each. 100%

7.1 Written down value at the beginning of the year [U/s 22(5)]
The above mentioned rates are applied on the written down value (WDV) of the asset at the beginning of the year,
which is determined as below:
(a) In case of asset acquired during the year:
Cost xxx
Less: Initial allowance (if asset is eligible depreciable asset) (xxx)
WDV at the beginning of the year xxx
(b) In any other case:
Cost xxx
Less: Initial allowance allowed in previous years (xxx)
Less: Depreciation allowed in previous years (xxx)
WDV at the beginning of the year xx
7.2 Asset partly used for business:
Where an asset is partly used for business and partly for some other purpose then:
(a) Depreciation shall be allowed in proportion to the use of asset in the business [U/s 22(3),
(b) However, WDV shall be calculated in the normal way [U/s 22(6)].
Following example will demonstrate this situation:
Example - 1: A person acquired machinery in year 1 for Rs. 500,000 for business purpose. In year 2 he used that
machinery for business purpose for six months and for remaining six months he used that for some other purpose
while in year 3 he used that machinery wholly for business use.
Required: Calculate (a) depreciation allowed and (b) closing WDV for three years.
Solution: (a)
Year 3 Year 2 Year 1
Rs. Rs. Rs.
Cost 500,000
Less: initial allowance @ 25% 125,000
WDV at the beginning of the year (A) 270,937 318,750 375,000

Depreciation @ 15% (B) 40,641 47,813 56,250


Less: Depreciation not allowed because of non business use - 23,907 -
Depreciation allowed 40,641 23,906 56,250
WDV at the end of year (A - B) 230,296 270,937 318,750
8. Initial allowance [U/s 23]
A person who places an eligible depreciable asset into service in Pakistan for the first time in a tax year
shall be allowed a deduction of 25% for plant and machinery and 15% for building where the asset is used
by the person for the purposes of his business for the first time or the tax year in which commercial
production is commenced, whichever is later.
A deduction allowed to a leasing company or an investment bank or a modaraba or a scheduled bank or a
development finance institution in respect of assets owned and leased to another person shall be deducted
only against the leased rental income derived in respect of such assets.
"Eligible depreciable asset" means a depreciable asset other than -
any road transport vehicle unless the vehicle is plying for hire;
any furniture, including fittings;
any plant or machinery that has been used previously in Pakistan; or
any plant or machinery that has been allowed as deduction under another section for the entire cost of the
asset.
Important note: It is worthwhile to mention here that under this section there is no concept of number of days
because either the initial allowance shall be allowed or not allowed.

Conceptual Approach to Taxes 149


Assets and Depreciation Chapter-10

Example 2: In tax year 2015, Jazz Limited purchased a new plant and building for Rs.1,200,000 and Rs. 500,000
respectively. Calculate initial allowance if any, tax depreciation and closing WDV.
Solution:
Plant Building Total
(Rupees)

Cost 1,200,000 500,000 1,700,000


Initial allowance (25% for plant) and (15% for building) 300,000 75,000 375,000
900,000 425,000 1,325,000
Tax depreciation @ (15% and 10%) 135,000 42,500 177,500
Closing WDV 765,000 382,500 1,147,500
9. First year allowance [U/s 23A]
With effect from 01-07-2008 first year allowance @ 90% of the cost shall be allowed in respect of plant,
machinery and equipment installed by any industrial undertaking set up in specified rural and
underdeveloped areas or engaged in the manufacturing of cellular mobile phones and qualifying for
exemption under clause (126N) of Part I of the Second Schedule and owned and managed by a
company in lieu of initial allowance on "eligible depreciable assets". Except the above the provisions of the
Initial Allowance shall mutatis mutandis apply. The Federal Government may notify "specified areas" for the
purposes of this section.
Important note: It is worthwhile to mention here that under this section there is no concept of number of days
because either the first year allowance shall be allowed or not allowed.
Example 3: In tax year 2015, A new plant for Rs.1,200,000 was purchased by an industrial undertaking set up in
area specified by Federal Government for first year allowance. Calculate closing WDV of the plant.
Solution: In this case, first year allowance @ 90% shall be allowed instead of initial allowance.
Rs.
First year allowance (1,200,000 x 90%) 1,080,000
Tax depreciation ((1,200,000 1,080,000) x 15%) 18,000
Closing WDV (1,200,000 1,080,000 18,000) 102,000
10. Accelerated depreciation to alternate energy projects [23B and Part-II of Third Schedule]
Any plant, machinery and equipment installed for generation of alternate energy by an industrial undertaking set up
anywhere in Pakistan and owned and managed by a company shall be allowed first year allowance in lieu of initial
allowance u/s 23, at the rate of 90% against the cost of the eligible depreciable assets put to use after July 01, 2009.
A deduction allowed to a leasing company or an investment bank or a modaraba or a scheduled bank or a
development finance institution in respect of assets owned and leased to another person shall be deducted only
against the leased rental income derived in respect of such assets.
"eligible depreciable asset" means a depreciable asset other than -
(a) any road transport vehicle unless the vehicle is plying for hire;
(b) any furniture, including fittings;
(c) any plant or machinery that has been used previously in Pakistan; or
(d) any plant or machinery that has been allowed as deduction under another section for the entire cost of the
asset.
Important note: It is worthwhile to mention here that under this section there is no concept of number of days
because either the accelerated depreciation shall be allowed or not allowed.
Chart presentation of initial allowance, First year allowance & accelerated depreciation to alternate energy projects:
11. Intangibles [U/s 24]
Introduction:
The nomenclature of this term gives the impression that it only includes the cost of non-physical assets. However,
definition of this term under the tax law is far wider than this general impression. The definition of intangible in section
24 of the Ordinance is as under:

"intangible" means any patent, invention, design or model, secret formula or process, copyright, trade mark, scientific
or technical knowledge, computer software, motion picture film, export quotas, franchise, licence, intellectual property,
or other like property or right, contractual rights and any expenditure that provides an advantage or benefit for a period
of more than one year (other than expenditure incurred to acquire a depreciable asset or unimproved land).

150 Conceptual Approach to Taxes


Assets and Depreciation Chapter-10

The above definition reveals that it also includes any expenditure that provides an advantage or benefit for a period of
more than one year. Therefore, amortization of any cost which has useful life of a period exceeding on year is
allowed.
Intangible eligible for amortization
A person shall be allowed an amortisation deduction in a tax year for the cost of the person's intangibles;
that are wholly or partly used by the person in the tax year in deriving income from business
chargeable to tax and
that has a normal useful life exceeding one year.
The term cost of intangible in this section means any intangible, means any expenditure incurred
in acquiring or creating the intangible, including any expenditure incurred in improving or renewing
the intangible;
Limitation on amortization and method to compute amortization:
No deduction shall be allowed where a deduction has been allowed under another section of this
Ordinance for the entire cost of the intangible.
The total deductions allowed to a person under this section in the current tax year and all previous
tax years shall not exceed the cost of the intangible.
Formula to compute amortization deduction is as under:
Cost of intangible / Normal useful life of the intangible in whole years
Where an intangible asset useful life is more than ten years or not ascertainable:
An intangible with a normal useful life of more than ten years or that does not have an ascertainable
useful life shall be treated as if it had a normal useful life of ten years.
Where an asset not used for the whole of the year:
Where an intangible is used in a tax year partly in deriving income from business chargeable to tax
and partly for another use, the deduction allowed for that year shall be restricted to the fair
proportional part of the amount that would be allowed if the intangible were wholly used to derive
income from business chargeable to tax.
Where an intangible is not used partly only to derive income from business chargeable to tax, the
amortization shall be computed according to the following formula, namely-
Amount of amortization x Number of days in the tax year the intangible is used in deriving
income from business chargeable to tax / Number of days in the tax year
Amortization in case of disposal:
Where, in any tax year, a person disposes of an intangible, no amortization deduction shall be
allowed under this section for that year;
Gain / loss on disposal of intangibles:
if the consideration received by the person exceeds the WDV of the intangible at the time of
disposal, the excess shall be income of the person chargeable to tax in that year under the
head "Income from Business"; or
if the consideration received is less than the WDV of the intangible at the time of disposal,
the difference shall be allowed as a deduction in computing the person's income chargeable
under the head "Income from Business" in that year.
WDV in case of disposal of and intangible asset:
The WDV of an intangible at the time of disposal shall be the cost of the intangible reduced by
the total deductions allowed to the person in respect of the intangible or, where the intangible is
not wholly used to derive income chargeable to tax, the amount that would be allowed if the
intangible were wholly so used; and
The consideration received on disposal of an intangible shall be determined in accordance with
section 77. An intangible that is available for use on a day (including a non-working day) is
treated as used on that day.
Chart presentation of depreciation under section 22 and intangibles under section 24:

Conceptual Approach to Taxes 151


Assets and Depreciation Chapter-10

12. Pre-commencement expenditure [U/s 25]

Particulars Pre-commencement expenditure U/S 25


(1) (2)

1. To whom available A person shall be allowed deduction for the any above related expenditure.

2. Basis 20% straight line method

3. Limitation on pre- The total deductions allowed under this section in the current and all previous
commencement tax years shall not exceed the amount of the expenditure.
expenditure
No deduction shall be allowed for any pre-commencement expenditure where a
deduction has been allowed under another section of this Ordinance for the entire
amount of the pre-commencement expenditure.

4. Pre-commencement "Pre-commencement expenditure" means any expenditure incurred;


expenditure
before the commencement of a business wholly and exclusively to derive income
chargeable to tax,
cost of feasibility studies,
construction of prototypes
trial production activities, but
shall not include any expenditure which is incurred in acquiring land, or which is
depreciated or amortized.

EXCEPTION TO THE RULE COST INCURRED AND CONSIDERATION RECEIVED / SPECIAL POINTS ON DISPOSAL OF
DEPRECIABLE AND NON DEPRECIABLE ASSETS:
In case of disposal of the asset, following rules shall apply:
a. Total deduction (normal and initial allowance) allowed to a person during the period of ownership of a depreciable
asset shall not exceed the cost of the asset [U/s 22(7)].
b. No depreciation shall be allowed in the year of disposal [U/s 22(8)].
c. Gain / loss on disposal of depreciable asset;
If the consideration received against the disposal of depreciable asset is more than its WDV, then excess
shall be chargeable to tax under the head income from business [U/s 22(8a)].
If consideration received against the disposal of a depreciable asset is less than WDV, then the difference
shall be deducted from income chargeable to tax under the head income from business [U/s 22(8b)].
d. Where an asset was partly used for business and partly for some other purpose, then WDV at the time of
disposal shall be increased by depreciation not allowed on account of non business use [U/s 22(9)]. (See example
4)
e. If the cost of passenger transport vehicle not plying for hire is more than Rs. 2.5 million, then it shall be
considered equal to Rs.2.5 million and in this case disposal consideration shall also be reduced as per following
formula [U/s 22(10)]: (See example 5)
(Amount received on disposal of the vehicle x 2.5 million) / Actual cost of vehicle
Note: If the passenger transport vehicle in plying for hire then there is no limitation on cost of transport vehicle under
the aforesaid section.
f. The cost and consideration received in respect of a depreciable asset received as already discussed shall be
determined u/s 75 to 79 of the Income Tax Ordinance, 2001 [U/s 22(11)].
g. In case of disposal of immovable property, where consideration received exceeds the cost of asset then
consideration received shall be treated as the cost of asset [U/s 22(13)]. (See example 6)
h. Where a person exports an asset after using in Pakistan, then consideration received shall be treated as equal to
the cost of asset [U/s 22(14)]. (See example 7)
Example - 4: Consider the situation of example 1. What would be the treatment if asset is sold in year 4 for Rs. 500,000.

152 Conceptual Approach to Taxes


Assets and Depreciation Chapter-10

Solution: Rs.

Consideration received 500,000


Less: WDV
WDV at the beginning of year 4 230,296
Depreciation not allowed in year 2 23,906
254,203
Income chargeable to tax 245,797

Important Note: It is worthwhile to mention here that although section 22(8)(a) states that the profit on disposal of
depreciable asset is equal to the difference between sale price and written down value of depreciable asset, however
the same shall be read with the limitations imposed under this section by sub section 13(d) and 14 on various assets
disposal.
Example - 5: A person acquired a passenger transport vehicle for Rs. 3,000,000 for business purpose. This vehicle was
then sold for Rs.1,800,000 in year 2. Calculate gain on sale of vehicle in year 2.

Solution: Calculation of WDV at the end of year 1: Rs.

Cost for tax purpose 2,500,000


Depreciation @ 15% 375,000
WDV at the end of year 1 2,125,000

Calculation of allowed portion in disposal consideration: Allowed portion


(1,800,000 x 2,500,000 / 3,000,000) 1,500,000
Calculation of profit / (loss) on disposal:

Consideration received WDV as above (Rs.1,500,000 2,125,000)= (625,000)


Example - 6: From the following information compute gain on sale of immovable property (including building that is for
personal use or otherwise non depreciable):
Cost 400,000
Consideration received on disposal 500,000
Solution: As consideration received is more than cost of the immovable property, hence consideration received shall be
treated as cost of the property.
Consideration received 500,000
Cost of asset (treated as equal to consideration received) 500,000
Difference Nil

Example 7: From following information compute gain on sale of asset which has been exported after using in Pakistan:
Cost Rs.100,000, WDV Rs.40,000, Consideration received Rs.160,000.
Solution: In this case cost shall be equal to the consideration received:
Rs.
Consideration received (equal to cost of asset for tax) 100,000
Less: WDV 40,000
Profit on disposal (equal to accumulated depreciation) 60,000

ASSETS OF LEASING COMPANIES


A deduction allowed on account of normal depreciation to a leasing company or an investment bank or a modaraba or a
scheduled bank or a development finance institution in respect of assets owned and leased to another person shall be
deducted by the aforesaid respective persons. [U/s 22(13)(c)]
The total of deductions u/ss 22 to 23B allowable to a leasing company or an investment bank or a modaraba or a
scheduled bank or a development finance institution in respect of assets owned and leased to another person shall be
deducted only against the leased rental income derived in respect of such assets.
Example: From following information compute the depreciation admissible to the leasing company for two years;
- Lease rentals per year 1,200,000
- Cost of machinery 5,000,000
- Lease period 5 years
- Initial allowance 1,250,000
- Annual depreciation year 1 (15% of WDV) 562,500

Conceptual Approach to Taxes 153


Assets and Depreciation Chapter-10

- Annul depreciation year 2 (15% of WDV) 478,125


- Annul depreciation year 3 (15% of WDV) 270,938
- Annul depreciation year 4 (15% of WDV) 406,406
- Annul depreciation year 5 (15% of WDV) 345,445
Solution: The computation of depreciation for five years is as under:
Rs.
Year 1 Depreciation for the year 1,812,500
Less annual lease rentals 1,200,000
Balance depreciation C/F 612,500
Year 2 Add: depreciation for the year 478,125
Total depreciation 1,090,625
Less annual lease rentals 1,200,000
Balance depreciation C/F -

Important note: From the second year to onwards the lessor is entitled to claim full amount of depreciation as the lease
rentals exceeds from the total depreciation from third year to onwards.

DISPOSAL AND ACQUISITION OF ASSETS

Disposal and acquisition of assets [U/s 75]


(A) Disposal of asset arises under the following cases:

S. No. Disposal of assets U/S 75


(1) (2)
1. A person who holds an asset shall be treated as having made a disposal of the asset at the time the
person parts with the ownership of the asset or when an asset is sold, exchanged, transferred or
distributed or cancelled, redeemed, relinquished, destroyed, lost, expired or surrendered.
2. The transmission of an asset by succession or under a will shall be treated as a disposal of the asset
by the deceased at the time asset is transmitted.
3. The application of a business asset to personal use shall be treated as a disposal of the asset by the
owner of the asset at the time the asset is so applied.
4. Where a business asset is discarded or ceases to be used in business, it shall be treated to have
been disposed of.
5. A disposal shall include the disposal of a part of an asset.

(B) Acquisition of asset arises under the following cases:

S. No. Acquisition of assets U/S 75


(1) (2)
1. A person shall be treated as having acquired an asset at the time the person begins to own the asset,
including at the time the person is granted any right.
2. The application of a personal asset to business use shall be treated as an acquisition of the asset by the
owner at the time the asset is so applied.

Example: Distinguish between disposal and acquisition of asset under the following situations.
(a) Application of a business asset to personal use.
(b) Application of a personal asset to business use.
(c) Mr. Adnan sold a part of his business building for Rs. 1,000,000.
(d) There in no demand of Product-A produced by M/s Azeem and Co. Therefore proprietor discarded the
machinery which was used in production of Product-A.
(e) Factory building of AB and Co. was destroyed by earthquake.
(f) Mr. Amir exchanged his business vehicle for machinery.

154 Conceptual Approach to Taxes


Assets and Depreciation Chapter-10

Solution:
(a) Application of business asset to personal use is treated as disposal of asset.
(b) Application of a personal asset to business use is treated as acquisition of asset.
(c) Disposal of a part of asset is treated as disposal of asset.
(d) When an asset is discarded, it is treated as disposal of asset.
(e) When an asset is destroyed / lost, it is also treated as disposal of asset.
(f) In this case, there is a disposal of vehicle and acquisition of machinery.
COST OF ASSETS UNDER VARIOUS SITUATIONS [U/S 76]

The Board may prescribe rules for determination of cost for any asset, however except otherwise provided in the Income
Tax Ordinance, 2001 the cost of an asset shall be determined as under.
1. Cost of an asset purchased:
The cost of an asset purchased by a person shall be the sum of the following amounts:-
The total consideration given for the asset, including the FMV of any consideration in kind determined at the
time the asset is acquired;
any incidental expenditure incurred in acquiring and disposing of the asset; and
any expenditure incurred by the person to alter or improve the asset.
but shall not include any expenditure above that has been fully allowed as a deduction under this Ordinance.
Example: Determine the cost of asset from following information.
Rs.
Cash paid for purchase of asset 50,000
FMV of motorcycle given for purchase of asset 30,000
Legal expenses incurred on purchase of asset 10,000
Repair expenses (fully allowed as deduction) 5,000

Solution:
Rs.
Cash paid for purchase of asset 50,000
FMV of motorcycle given for purchase of asset 30,000
Legal expenses incurred on purchase of asset 10,000
Total 90,000
Note: As repair expenses have already allowed therefore the same shall not be added in the cost of asset.
2. Cost where personal asset applied for business use The cost of an asset treated as acquired shall be the FMV
of the personal asset determined at the date it is applied to business use.
Example: Rs.
Cost of personal asset purchased 100,000
Book value of asset as on June 30, 2016 85,000
You are required to compute the cost of asset if the same asset put to use for business purposes as on June 30,
2015 under the following situations:
A. If the fair market value of the personal asset as on June 30, 2016 is Rs.100,000.
B. If the fair market value of the personal asset as on June 30, 2016 is Rs.80,000.
C. If the fair market value of the personal asset as on June 30, 2016 is Rs.150,000.
Solution: In all the above cases the fair market value as on June 30, 2016 shall be taken as cost of asset irrespective
of its cost or book value for business purposes.
3. Cost of an asset produced or constructed
The cost of an asset produced or constructed by a person shall be the total cost incurred by the person in producing
or constructing the asset plus any expenditure in acquiring and disposing, alter or improving the asset incurred by the
person.
Example: Determine the cost of plant manufactured by an AOP for its own use from the following information.
Rs.
Salary of engineer (fully engaged in manufacture of plant) 50,000
Raw material purchased for manufacture of plant 550,000
Wages to labour (40% for manufacture of plant) 100,000

Conceptual Approach to Taxes 155


Assets and Depreciation Chapter-10

Solution:
Rs.
Salary of engineer (fully engaged in manufacture of plant) 50,000
Raw material purchased for manufacture of plant 550,000
Wages to labour (40% portion related to manufacture of plant) 40,000
Total 640,000
4. Cost of an asset acquired through foreign currency
Where an asset has been acquired by a person with a loan denominated in a foreign currency and before full and
final repayment of the loan, there is an increase or decrease in the liability of the person under the loan as expressed
in Rupees, the amount by which the liability is increased or reduced shall be added to or deducted from the cost of
the asset, as the case may be:
Explanation: Difference, if any, on account of foreign currency fluctuation, shall be taken into account in the year of
occurrence for the purposes of depreciation.

Example: On July 1, 2015, Mr. Zahid acquired a machine with loan in foreign currency ($50,000) equivalent to
Rs.4,000,000. On June 30, 2016, exchange rate was ($1 = RS. 85). Calculate the amount of tax depreciation and
initial allowance for tax year 2016.
Solution: In this case, change in value of loan shall not be considered for depreciation.
Cost of asset 4,000,000
Initial allowance [4,000,000 x 25%] 1,000,000
WDV for depreciation u/s 22 3,000,000
Depreciation for the year [3,000,000 x 15%] 450,000
Important note: Although the plain reading of the section states that any increase and decrease in exchange rate
before the final settlement of loan shall either be added or deducted from the cost of asset but the position is not so
as only that difference in exchange rate shall be recognized that will arise on actual repayment or availing further
foreign currency loan. The mere change in exchange rate without any repayment or acquiring loan will have no effect
on the cost of asset.
Explanation: Difference, if any on account of foreign currency fluctuation, shall be taken into account in the year of
occurrence for the purposes of depreciation.
The above position may be examined from the FBR Circular 3 of 2009 dated 17-07-2009 & CBRs Circular No.3 of
1991 dated March 09, 1991 that is also approved by the ATIR vide ITA No.1448/HQ 1989-90 dated 15Th November,
1990.
It is worthwhile to mention here that the above circular is still saved by virtue of provisions of section 239(10) of the
ITO, 2001.
5. Cost of an asset acquired under hedging agreement
In determining whether the liability of a person has increased or decreased as above the consideration shall be taken
of the person's position under any hedging agreement relating to the loan.
Example: On July 01, 2014, Mr. Zahid acquired a machine with loan in foreign currency ($50,000) equivalent to
Rs.4,000,000. This loan is covered under hedging agreement and he shall not be liable to pay any increase in the
amount of loan due to change in exchange rate. On June 30, 2015, exchange rate was ($1 = RS. 85). Calculate the
amount of tax depreciation and initial allowance for tax year 2015.
Solution: In this case, change in value of loan shall not be considered for depreciation purpose.
Initial allowance [4,000,000) x 25%] 1,000,000
Depreciation for the year [(4,000,000 1,000,000) x 15%] 450,000
6. Cost of an asset sold in parts
Where a part of an asset is disposed of by a person, the cost of the asset shall be apportioned between the part of
the asset retained and the part disposed of in accordance with their respective FMVs determined at the time the
person acquired the asset.
Example: On July 01, 2015, Mr. Zahid acquired a building for Rs. 500,000. In May, 2015 he disposed of 1/4th of
building for Rs. 300,000. On the date of acquisition, fair value of part sold was Rs. 200,000 and fair value of
remaining part was Rs.400,000. Determine gain / loss on disposal and cost of building retained.

156 Conceptual Approach to Taxes


Assets and Depreciation Chapter-10

Solution: Gain / loss on disposal: Rs.

Consideration received on disposal 300,000


Less: Cost apportioned on the basis fair value
(500,000 x 200,000 / 600,000) 166,667
Gain on disposal 133,333

Cost of asset retained: Rs.

Cost of total asset 500,000


Less: Cost of asset disposed of (as above) 166,667
Cost of asset retained 333,333

7. Cost of an asset acquired from the amount chargeable to tax


Where the acquisition of an asset by a person is the derivation of an amount chargeable to tax, the cost of the
asset shall be the amount so charged plus any amount paid by the person for the asset.
8. Cost of an asset acquired from the amount exempt from tax
Where the acquisition of an asset by a person is the derivation of an amount exempt from tax, the cost of the
asset shall be the exempt amount plus any amount paid by the person for the asset.
Example: Mr. Ahmed purchased vehicle to be used for his business purpose for Rs. 800,000. Rupees 200,000 was
paid from taxable income while remaining Rs. 600,000 was paid from an amount which is exempt from tax. What is
the cost of asset?
Solution: Cost of asset is determined by the amount paid and it is immaterial whether the amount paid as purchase
price of asset is taxable or exempt. Hence, the cost of asset is Rs. 800,000.
9. Cost of an asset acquired from the grant not chargeable and chargeable to tax
The cost of an asset does not include the amount of any grant, subsidy, rebate, commission or any other
assistance (other than a loan repayable with or without profit) received or receivable by a person in respect of the
acquisition of the asset, except to the extent to which the amount is chargeable to tax under this Ordinance.
Example: Mr. Naeem received grant of Rs. 500,000 from Government for purchase of an asset. Forty percent (40%)
of this grant is taxable and balance sixty percent 60% is exempt. Asset was purchased by Mr. Naeem for Rs.
700,000. You are required to determine the cost of asset.

Solution Rs.
Consideration paid for purchase of asset 700,000
Less: Exempt Government grant 420,000
Cost of asset 280,000

CONSIDERATION RECEIVED UNDER VARIOUS SITUATIONS [U/s 77]


The Board may prescribe rules for determination of consideration received for any asset, however except otherwise provided
in the Income Tax Ordinance, 2001 the disposal consideration shall be determined as under.
10. Disposal consideration on sale:
The consideration received by a person on disposal of an asset shall be the total amount received by the person
for the asset or the FMV thereof, whichever is the higher, including the FMV of any consideration received in kind
determined at the time of disposal.
Example: Mr. Jamshed sold his factory building to Mr. Amir for Rs. 1,000,000. However, the fair value of building was
Rs. 1,200,000. Compute gain / loss on disposal if WDV of building is Rs.600,000.
Solution:
Rs.
Consideration received
(Higher of actual amount or fair value) 1,200,000
Less: WDV 600,000
Gain on disposal 600,000

Conceptual Approach to Taxes 157


Assets and Depreciation Chapter-10

11. Disposal consideration for lost or destroyed asset


Where an asset has been lost or destroyed by a person, the consideration received for the asset shall include any
compensation, indemnity or damages received by the person under:-
an insurance policy, indemnity or other agreement;
a settlement; or
a judicial decision.
Example: Mr. Jamsheds factory building was destroyed because of earthquake during the tax year 2016. He
received Rs. 500,000 from insurance company in respect of this building. Compute gain / loss on disposal if WDV of
building is Rs. 1,600,000.
Solution:
Rs.
Consideration received
(Amount received from insurance company) 500,000
Less: WDV 1,600,000
Loss on disposal 1,100,000
12. Disposal consideration for business asset applied to personal use or discarded or ceased to be used
The consideration received for an asset treated as disposed shall be the FMV of the asset determined at the time it
is applied to personal use or discarded or ceased to be used in business, as the case may be.
Example: In tax year 2015 Mr. Khan discarded his business car from business and applied that car for his personal
use. Fair Value on the date of application to personal use was Rs. 500,000. Compute gain / loss on disposal if WDV
of car at the beginning of tax year is Rs. 400,000.

Solution: Consideration received Rs.

Fair value 500,000


Less: WDV 400,000
Gain on disposal 100,000
13. Disposal consideration in respect of leased assets
The consideration received by a scheduled bank, financial institution, modaraba, or leasing company approved by the
CIR in respect of an asset leased by the company to another person shall be the residual value received by the
leasing company on maturity of the lease agreement provided that the residual value plus the amount realized
during the term of the lease towards the cost of the asset is not less than the original cost of the asset,

Example: A leasing Company has lease out its plant and machinery on the following terms and conditions;
Rs.
Lease rentals for five years (Principal plus mark-up price) 6,000,000
Cost of plant and machinery to leasing Company 4,000,000
You are required to compute the disposal consideration of lease asset under the following situations:
(a) If the principal amount in total lease rental is Rs. 3,500,000 and its residual value is Rs. 200,000.
(b) If the principal amount in total lease rental is Rs. 3,800,000 and its residual value is Rs. 100,000.
(c) If the principal amount in total lease rental is Rs. 4,000,000 and its residual value is Rs. 200,000
Solution:
In case A and B as the principal amount plus residual amount is less than the cost of asset to the lessor therefore the
disposal consideration shall be taken as Rs. 4,000,000 that is not less than the cost of asset to the lessor. However in
case of C no adjustment shall be made in the disposal consideration as the same is more than the cost of asset to the
lessor.
14. Disposal consideration in respect combined sale of two or more assets
Where two or more assets are disposed of by a person in a single transaction and the consideration received for
each asset is not specified, the total consideration received by the person shall be apportioned among the assets
disposed of in proportion to their respective FMVs determined at the time of the transaction.
Example: In tax year 2016 Mr. Khan disposed of his two business cars for a sum of Rs. 1,200,000. WDV of car-1 is
Rs. 300,000 and car-2 is Rs.400,000. Fair Value on the date of this transaction was as follows:
Car-1 700,000, Car-2 300,000
Required: Compute gain on sale of these two cars.

158 Conceptual Approach to Taxes


Assets and Depreciation Chapter-10

Solution:
Rs. Rs.
Car-1 Car-2
Consideration received (apportioned on the basis of
fair values) i.e. 70% : 30% 840,000 360,000
Less: WDV 300,000 400,000
Gain / (loss) on disposal 540,000 (40,000)
Important note: The tax department shall accept the value that will be higher from FMV and disposal consideration
received of respective asset. As the in the said example the consideration received is higher the same has been
taken into account however where the fair value will be higher than the same shall be taken into account.
15. Determination of consideration received by board
Notwithstanding anything contained in this section, the Board may prescribe rules for determination of consideration
received for any asset.

PART - II (For CA Mod F and ICMAP Students)


NON ARMS LENGTH TRANSACTIONS AND NON RECOGNITION RULES

1. Non-arm's length transactions [U/s 78]


Where an asset is disposed of in a non-arm's length transaction
(a) the person disposing of the asset shall be treated as having received consideration equal to the FMV of the
asset determined at the time the asset is disposed; and
(b) the person acquiring the asset shall be treated as having a cost equal to the amount determined under (a)
above.

Example: Mr. Jamshed sold his factory building to his relative Mr. Amir for Rs. 500,000 i.e. transaction was non-
arms length transaction. However, the fair value of building was Rs. 1,200,000. Compute gain / loss on disposal if
WDV of building is Rs. 600,000.

Solution:
Rs.
Consideration received equal to FMV of building both for buyer and seller
irrespective of sale proceeds) 1,200,000
Less: WDV 600,000
Gain on disposal 600,000

2. Non-recognition rules for gain or loss on disposal [U/s 79]

No gain or loss shall be taken to arise on the disposal of an asset


(a) between spouses under an agreement to live apart;
(b) by reason of the transmission of the asset to an executor or beneficiary on the death of a person;
(c) by reason of a gift of the asset;
(d) by reason of the compulsory acquisition of the asset under any law where the consideration received for the
disposal is reinvested by the recipient in an asset of a like kind within one year of the disposal;
(e) by a company to its shareholders on liquidation of the company; or
(f) by an AOP to its members on dissolution of the association where the assets are distributed to members in
accordance with their interests in the capital of the association-
Provided the above provisions shall not apply where the person acquiring the asset is a non-resident person at
the time of the acquisition.
In the above all the cases the person acquiring the asset shall be treated as
(a) acquiring an asset of the same character as the person disposing of the asset; and
(b) acquiring the asset for a cost equal to the cost of the asset for the person disposing of the asset at the time of
the disposal.

Conceptual Approach to Taxes 159


Assets and Depreciation Chapter-10

Cost of replacement assets under compulsory acquisition:


The persons cost of a replacement asset referred to in clause (d) above shall be the cost of the asset disposed of
plus the amount by which any consideration given by the person for the replacement asset exceeds the
consideration received by the person for the asset disposed of. The formula for the above is as under:
Cost of asset disposed
Plus consideration given Less consideration received on disposed of asset
Example: Explain in which of the following cases gain or loss on disposal shall not be recognized:

a. Mr. Adnan gave his business car to his wife under an agreement to live apart.
b. Business car of Mr. Ikram was completely destroyed in an accident in current tax year and he received claim
from insurance company.
c. Mr. Adnan gave his business car to his wife under an agreement to live apart. His wife is a non-resident in tax
year 2015.
d. A company disposed of its assets to its shareholders on liquidation of the company.
e. An AOP disposed of its assets to its members on dissolution of the AOP in accordance with their interest in the
capital of the association, all the members are non-resident.
f. Mr. Amir gave his factory to his brother as gift. His brother is a non-resident.
Solution:
a. this case no loss or gain shall be recognized.
b. this case gain or loss shall be recognized which is equal to insurance claim received less WDV of the car.
c. this case gain or loss shall be recognized as car was given to non-resident person.
d. gain or loss shall be recognized.
e. n or loss shall be recognized as the members are non-resident.
f. n or loss shall be recognized as the brother is non-resident.

MASTER QUESTION
Briefly explain the tax treatment in respect of each of the following independent situations:
a) Aiza (Pvt.) Ltd has re-valued its Building in accordance with International Accounting Standards and
consequently charged depreciation on the re-valued amount.
b) Aiza (Pvt.) Ltd during the year has opened an overseas office in France and has claimed initial allowance and
depreciation on eligible depreciable assets purchased by the office.

c) Uzair Limited has charged impairment in respect of one of its depreciable assets. The Commissioner is of the
view that impairment expense will not be allowed as an expense.

d) Uzair Limited has discontinued a major product line of its business and envisages selling off the machinery
related to this product line over a period of one to two years to get the right price. Uzair Ltd wants to claim
depreciation on the idle machinery until disposed of.
e) Ms. Sana sells a number of personal vehicles in a tax year and makes a significant amount of profit in the
process. She is of the view that the said income is exempt from tax.

f) XYZ Ltd has recorded a gain on revaluation of its foreign currency balances at the year end. The gain
comprises of both realized and unrealized amount.
g) On July 2014, Ms. Sana purchased a vehicle not plying for hire amounting to Rs. 4,210,000 to be used solely
for the purpose of her business. While preparing the tax return she has claimed initial allowance and
depreciation as per the prescribed rates given in the Income Tax Ordinance, 2001 for the full year on
Rs.4,210,000.

h) In August 01, 2014 Mr. Azhar purchased accounting software amounting to Rs. 5 million for his business. The
software has a useful life of 13 years. Mr. Azhar has charged full year amortization on straight line basis over
the useful life of the software.

i) Entertainment expense payable amounting to Rs. 210,000 has been debited to profit & loss account of ABC
Ltd. The company has not deducted any tax on the said expense.

160 Conceptual Approach to Taxes


Assets and Depreciation Chapter-10

j) ABC (Pvt.) Ltd has charged depreciation according to the rates admissible under the tax law amounting to Rs.
125,000 on machinery taken on a finance lease from a scheduled bank in August 2009. Lease rentals paid
during the tax year 2015 amounted to Rs. 220,000. The leased machinery was transferred to owned assets
on maturity on 30 April 2015. On maturity the accounting WDV of the assets was Rs. 500,000, market value
was Rs. 800,000 whereas residual value of the asset was Rs. 50,000.

Solution:
a) Deduction for depreciation is associated with tax written down values of assets calculated with reference to
specific provisions. Accounting revaluation of assets has no bearing on tax written down value of assets.
Consequently, depreciation will be allowed on tax written down values of building without taking into account
the effect of revaluation. [Ref:S22(5)]
b) Initial allowance is only available on assets used in Pakistan. Accordingly, the company will not be entitled to
deduction on account of initial allowance on assets purchased by the branch for use in business outside
Pakistan. The company will however be allowed to claim normal depreciation on all depreciable assets. [Ref:
S 23 (1) and S 22]
c) The contention of the Commissioner is correct. Charge for impairment of fixed assets is not a tax deductible
expense. As the impairment charge will be ignored for tax purpose, the written down value of assets will not
be reduced by the charge and depreciation will be calculated as if no impairment has taken place.
d) One of the criteria for an asset to qualify as depreciable asset is that it should be used partly or wholly for
deriving business income. As the product line has been discontinued and the machinery is no more in use,
therefore, it ceases to qualify as a depreciable asset. Accordingly, no deduction will be allowed for
depreciation. [Ref: S 22 and S75(3A)]
e) Income from sale of personal motor vehicles is not taxable under the head Capital Gains. If the vehicles are
bought and sold with the motive of trade, the resultant gain will constitute business income. However, vehicle
intended for personal use are excluded from the definition of capital assets. [Ref: S 37(5) (d)]
f) Unrealized gain on revaluation of foreign currency balances is notional income in nature and is not liable to
tax. Foreign exchange gains will be included in the taxable income for the tax year in which realized.
g) Full year depreciation should be charged on restricted value of Rs. 2,500,000. As vehicle is not an eligible
depreciable asset, therefore, initial allowance cannot be claimed.
h) Amortization should be allowed for 91 days over the useful life of 10 years only. (S. 24(4), 24(6))

i) Tax is required to be deducted at the time of payment. Since the expense is still payable, therefore, company
has rightly claimed the said expense.

j) In case of assets taken on finance lease, lease rentals are an admissible deduction instead of depreciation.
Further, as the asset was transferred during the tax year 2015, therefore, full year depreciation will be allowed
on the residual value of the asset. No initial allowance will be allowed as the asset was already in use. [(S. 22,
S.28(1)(B),S23)].

Conceptual Approach to Taxes 161


Assets and Depreciation Chapter-10

MULTIPLE CHOICE QUESTIONS


Q.1. Pre commencement expenditure is a_______________.
(a) Fictitious asset
(b) Depreciable asset
(c) Eligible depreciable asset
(d) None of these
Q.2. Disposal consideration on sale of an asset is
(a) Amount received on sale
(b) FMV
(c) Lower of FMV and amount received
(d) Higher of FMV and amount received
Q.3. Disposal consideration in case of lost asset comprises of______________.
(a) Salvage value
(b) Insurance claim
(c) both aand b
(d) none of these
Q.4. An approved leasing company is the one approved by _________________.
(a) FBR
(b) SECP
(c) CIR
(d) Provincial government
Q.5. Depreciation calculated under _________ is allowed as deduction against business income.
(a) First schedule
(b) Second schedule
(c) Third schedule
(d) Fourth schedule
Q.6. FMV is determined in relation to _______________.
(a) Property
(b) Services
(c) Perquisites
(d) All of these
Q.7. In case of an asset partly used only for business then depreciation will be allowed as deduction on _________basis.
(a) Full year
(b) Half year
(c) Proportionate
(d) Not allowed
Q.8. The depreciation charged in case of non-depreciable asset used wholly for business would be _______.
(a) Zero
(b) Half
(c) Full
(d) Proportionate
Q.9. The question of depreciation allowance does not arise if the income from business is ____________.
(a) Permanently exempt
(b) Exempt for specific period
(c) not exempt
(d) all of the above

162 Conceptual Approach to Taxes


Assets and Depreciation Chapter-10

Q.10. Initial allowance on eligible depreciable assets (plant and machinery) is allowed at the rate of _________of asset.
(a) 40% of cost
(b) 40% of FMV
(c) 50% of WDV
(d) 25% of Cost
Q.11. In case of assets with a useful life of one year, depreciation is ___________allowed:
(a) Not
(b) on full year basis
(c) Proportionate basis
(d) All of these
Q.12. The initial allowance for depreciation is allowable for ___________.
(a) Depreciable assets
(b) Eligible depreciable assets
(c) Intangibles
(d) All of these
Q.13. Depreciation u/s 22 on assets partly for personal and partly for business use is allowable on basis
(a) Monthly
(b) Half yearly
(c) Quarterly
(d) Annual
(e) proportionate
Q.14. Where an intangible is not used for the business for the whole year, then amortization deduction would be on
______basis.
(a) Half of the charge
(b) Full year charge
(c) Proportionate
(d) None of these
Q.15. A person shall be allowed a deduction of amortization of pre-commencement expenditure on straight line basis
at_______.
(a) 10%
(b) 20%
(c) 25%
(d) 50%
Q.16. Amortization in any case cannot be___________ to / from total cost of intangible.
(a) Equal
(b) Less
(c) Higher
(d) None of these
Q.17. A deduction for amortization is allowed only when the intangible has a useful life of _________.
(a) Less than one year
(b) Equal to one year
(c) Higher than one year
(d) Indefinite period
Q.18. In case of export of an asset, disposal consideration would be treated as the __________________.
(a) Cost of asset
(b) FMV at the time of export
(c) Consideration received
(d) None of these

Conceptual Approach to Taxes 163


Assets and Depreciation Chapter-10

Q.19. The cost of passenger transport vehicles plying for hire for Tax year 2011 for depreciation purposes would be equal
to ____.
(a) Actual cost
(b) Rs. 2,500,000
(c) Higher of actual cost and 2,500,000
(d) Lower of 2,500,000
Q.20. Initial allowance for depreciation on passenger transport vehicles not plying for hire is _______________.
(a) Allowed in the first year
(b) Allowed in last year
(c) Not allowed
(d) none of these
Q.21. In case of intangibles not used for the whole year, amortization allowed would be ___________.
(a) or the full year
(b) Not to charged
(c) Charged on number of days basis
(d) none of the above
Q.22. Any gain or loss on disposal of intangible shall be treated as income or deduction under income from ___________.
(a) Business income
(b) Other sources
(c) Capital gains
(d) None of these
Q.23. When an asset is sold, exchanged, transferred or distributed or cancelled, redeemed, relinquished, destroyed, lost,
expired or surrendered, it shall be treated as ___________________.
(a) Acquisition of asset
(b) disposal of an asset
(c) none of the above
Q.24. The application of a business asset to personal use shall be treated as _______________________.
(a) Acquisition of asset
(b) disposal of an asset
(c) none of the above
Q.25. The cost of an asset purchased by a person shall be_____________________.
(a) Total consideration given
(b) incidental expenditure in acquiring and disposing
(c) any expenditure to alter or improve the asset
(d) all of these
(e) none of the above
Q.26. Where an asset has been acquired with a foreign currency loan the cost of an asset shall be fluctuated due to
__________________.
(a) increase in exchange rate without payment of loan
(b) decrease in exchange rate without payment of loan
(c) No effect unless loan is repaid
Q.27. Where the person disposing of the asset under non-arms length transaction, the consideration shall be treated as
having received equal to _______________________.
(a) Actual consideration received
(b) Fair Market Value
(c) none of the above.
Q.28. Depreciation shall be computed on Building (all types) against the written down value at the beginning of the year at
the rate of _______________.
(a) 10%

164 Conceptual Approach to Taxes


Assets and Depreciation Chapter-10

(b) 15%
(c) 30%
(d) 100%
(e) 20%
Q.29. Depreciation shall be computed on Furniture (Including fitting) and machinery and plant, Motor vehicles (all types),
ships, technical or professional books against the written down value at the beginning of the year at the rate of
_______________.
(a) 10%
(b) 15%
(c) 30%
(d) 100%
(e) 20%
Q.30. Depreciation shall be computed on computer hardware including printer, monitor and allied items, machinery and
equipments used in manufacture of IT products, aircrafts and aero engines, Aerial photographic apparatus against the
written down value at the beginning of the year at the rate of _______________.
(a) 10%
(b) 15%
(c) 30%
(d) 100%
(e) 20%
Q.31 Initial allowance on building is allowed at ____% of cost of building.
(a) 50
(b) 40
(c) 15
(d) 25
Q.32 Expenditures for acquiring land is included in _____.
(a) cost of land
(b) WDV of land
(c) depreciation of land
(d) all of above
Q.33 Depreciation in respect of asset acquired on lease is _________.
(a) admissible
(b) inadmissible
(c) equal to tax depreciation
(d) none of above
Q.34 Full year depreciation is charged in the year of _____.
(a) disposal
(b) destruction
(c) acquisition
(d) all of above
Q.35 Amount of depreciation allowed to ________ is restricted to the lease rental income derived during the year in
respect of leased assets.
(a) leasing companies
(b) individuals
(c) AOP
(d) all of above
Q.36 The cost of asset is allowed as expense where the asset with a useful life is ___ one year.
(a) less than or equal to
(b) more than
Conceptual Approach to Taxes 165
Assets and Depreciation Chapter-10

(c) none of above


Q.37 The amount of any grant receivable in respect of acquisition of asset shall be included in the cost of such asset
where such grant is _______.
(a) exempt
(b) taxable
(c) none of above
Q.38 The cost of land and the cost of immovable property on such land are shown ________.
(a) separately
(b) together
(c) in profit and loss account
(d) all of above
Q.39 The consideration received on disposal of an immovable properly shall be treated as its ___ if it exceeds the original
cost of asset.
(a) cost
(b) WDV
(c) depreciation
(d) all of above
Q.40 The ___ of an asset is treated as disposal consideration in case of export of a depreciable asset.
(a) cost
(b) WDV
(c) depreciation
(d) all of above
Q.41 Value of motor vehicle not plying for hire is restricted to Rs. ____ for income tax purposes.
(a) 1,500,000
(b) 2,000,000
(d) 2,500,000
(d) 3,000,000
Q.42 _____ of a discarded asset at the date when it is discarded is treated as its disposal consideration of the discarded
asset.
(a) cost
(b) WDV
(c) FMV
(d) accumulated depreciation
Q.43 When an asset is acquired with a loan in foreign currency and exchange rate fluctuates before any repayment then
the difference on account of foreign currency fluctuation should be ___ for depreciation purposes.
(a) ignored
(b) taken into account
(c) exempt
(d) none of above
Q.44 When an asset is wholly used in the tax year and sold at the end of year, then _____ shall be charged.
(a) full year depreciation
(b) no depreciation
(d) 50% depreciation
(e) none of above
Q.45 Rate of depreciation for building is 10% of __________.
(a) cost
(b) accumulated depreciation
(c) WDV
(d) cost of land

166 Conceptual Approach to Taxes


Assets and Depreciation Chapter-10

Q.46 The rate of depreciation for a ramp build to provide access to persons with disabilities not exceeding Rs.250,000
each is _____%.
(a) 50
(b) 70
(c) 80
(d) 100
Q.47 The normal useful life of an intangible asset is restricted upto ___ years from the date of purchase, where the life of
the intangible is either more than 10 years or not determinable.
(a) 20
(b) 10
(c) 5
(d) 2
Q.48 While considering non arms length transaction it ____ is considered as consideration received.
(a) WDV
(b) FMV
(c) cost
(d) all of above
Q.49 Where the consideration received is against assets sold in bulk, it would be apportioned on the basis of _________of
respective assets.
(a) WDV
(b) FMV at the date of disposal
(c) cost
(d) all of above
Q.50 _____ of an asset includes transfer of an asset between spouses under an agreement to live apart.
(a) acquisition
(b) destruction
(c) disposal
(d) none of above
Q.51 Tax depreciation is also known as _________ depreciation.
(a) statutory
(b) accounting
(c) actual
(d) all of above

ANSWERS
1 (a) 2 (d) 3 (b) 4 (c) 5 (c)
6 (d) 7 (a) 8 (a) 9 (a) 10 (d)
11 (a) 12 (b) 13 (e) 14 (c) 15 (b)
16 (c) 17 (c) 18 (a) 19 (a) 20 (c)
21 (c) 22 (a) 23 (b) 24 (b) 25 (d)
26 (c) 27 (b) 28 (a) 29 (b) 30 (c)
31 (c) 32 (a) 33 (b) 34 (c) 35 (a)
36 (a) 37 (b) 38 (a) 39 (a) 40 (a)
41 (d) 42 (c) 43 (a) 44 (b) 45 (c)
46 (d) 47 (b) 48 (b) 49 (b) 50 (c)
51 (a)

Conceptual Approach to Taxes 167


Assets and Depreciation Chapter-10

ICMAP PAST PAPES THEORECTICAL QUESTIONS


Q. No 2(a) March 2015 In the light of section 24 of the Income Tax Ordinance, 2001:
(i) Define the term, intangibles.
(ii) Briefly explain that how the allowable deduction will be computed, if and intangible asset is not used for the whole
tax year in deriving income from business chargeable to tax?
Q. No 2(b) March 2015 Ms. Sara is a resident individual of Pakistan. She is moving to Canada and planning to sell all
assets of her business to Sigma (private) Limited, a resident company of Pakistan and a wholly owned company. Before
leaving Pakistan, Sara intends to file her income tax return and seeking your advice in respect of the conditions where no
gain or loss will be accounted for on disposal of her baseness to Sigma.

Required: Being a Tax consultant briefly state Ms. Sara regarding conditions where no gain or loss will be accounted for on
disposal of all the assets to Sigma as per section 95 of the income tax ordinance, 2001

Q. No. 2 (a) (i) Spring 2013 Define the term depreciable asset in accordance with the provisions of section
22(15) of the Income Tax Ordinance, 2001.

Q. No. 2 (a) (ii) Spring 2013 Considering the depreciable asset is used in a tax year partly for deriving
income from business chargeable to tax and partly for another use, describe the extent to which the deduction
may be admissible on account of depreciation.
Q. NO. 3(a) SUMMER-2008 What is treatment of a depreciable asset under section 22 of the Income Tax Ordinance, 2001 if
it is disposed of in a tax year?
Q. NO. 3 (d) SUMMER 2007 Describe in detail:
(i) Disposal and acquisition of assets u/s 75 of the Income Tax Ordinance, 2001.
(ii) Business and personal assets u/s 75(7) of Income Tax Ordinance, 2001.
Q. NO. 6(a) WINTER-2006 Briefly state assets eligible for initial depreciation allowance u/s 23(5) of the Income Tax
Ordinance, 2001.
Q. NO. 4(b) WINTER 2005 Explain the terms b. Non-arms length transactions
Q. NO. 2 WINTER-2004 What do you understand by the following terms as described in Income Tax Ordinance, 2001?
1- Disposal of Assets 2- Initial allowance u/s 23 3- Eligible depreciable assets 4- Business assets and personal assets
Q. NO. 2 WINTER-2003 Describe amortization of intangibles as an allowable expense under section 24 of the Income Tax
Ordinance, 2001.

168 Conceptual Approach to Taxes


Assets and Depreciation Chapter-10

CA MOD C PAST PAPERS THEORECTICAL QUESTIONS


Q.NO.4 Autumn 2014 In Income Tax Ordinance, 2001 the term disposal has a wider connotation than sale
because it includes exchange, relinquishment, and extinguishment.
List the situations under which an asset owned by a person shall be treated to have been disposed of.

Q.6 Autumn 2012 In the context of Income Tax Ordinance, 2001,


(a) state the meaning of Intangible.
(b) discuss the rules relating to claiming of amortization deduction on intangibles.
Q. NO. 3(a) Autumn 2009 State the conditions which a tangible asset should meet to qualify as a depreciable asset.
Q. NO.6(b) Autumn 2007 A person who places an eligible depreciable asset into service in Pakistan for the first time in a
tax year shall be allowed initial depreciation allowance. List down the assets which do not come under the purview of
eligible depreciable assets for the purposes of initial allowance.
Q. NO.4(a) Spring 2005 Describe the assets that are not eligible for the purpose of claiming initial depreciation allowance.
Q. NO.3(a) Autumn 2004 List down the assets on which Initial allowance can not be claimed?
Q. NO.3(b) Autumn 2004 What are the prescribed rates of normal depreciation on the following assets as per the Third
Schedule to the Income Tax Ordinance, 2001?
(i) Factory building
(ii) Residential quarter for labour
(iii) Furniture
(iv) Plant and machinery
(v) Computer and hardware
(vi) Technical books
(vii) New ships
(viii) Motor vehicle
Q.4 Nov 1995 For each of the following questions / select the correct answer from the choices given:
Where a fixed asset (on which tax depreciation is allowed) is actually sold, sales proceed is determined to be:
(a) Sale price
(b) Fair Market Value (FMV)
(c) Sale price or Fair Market Value (FMV) whichever is higher
(d) Amount deemed to be the sale proceeds by the tax authorities.

NOW SOLVE FOLLOWING NUMERICAL QUESTIONS OF MODULE C / AFC PAST PAPER RELATED TO THIS TOPIC

Q. NO. 3(I) & (II) AUTUMN 2013


Q. NO. 3(B) AUTUMN 2009

Conceptual Approach to Taxes 169


Assets and Depreciation Chapter-10

170 Conceptual Approach to Taxes


Method of Accounting and Records Chapter-11

Chapter

11 METHOD OF ACCOUNTING AND


RECORDS

Topic covered
Section Rule (For CAF-6 and ICMAP students)

32 Method of accounting
Change in method of accounting
33 Cash basis of accounting
34 Accrual basis of accounting
Trading liability not paid & its recovery
35 Stock in trade
Computation of cost
36 Long term contracts
174 Records
28 to 31 Books of accounts prescribed
32 General form of Books of accounts, documents & records
33 Books of account, documents & record to be kept at specific place
MCQs with solutions
ICMAP & CA Mod C past papers theoretical questions

For CAF-6 and ICMAP students


1. Method of accounting [u/s 32(1), (2) and (3)]
A person income chargeable to tax shall be computed according to the method of accounting regularly
employed (that may be cash or accrual basis) by such person.
A company shall account for its income chargeable to tax under the head Income from business on accrual
basis (if company is deriving income chargeable to tax under any other head other than business
income, company can adopt cash basis method of accounting).
Any other person: on accrual or cash basis.
The Board may prescribe that any class of persons shall account for income chargeable to tax under the head
income from business on a cash or accrual basis.
2. Approval from Commissioner Inland Revenue (CIR) for change in method of accounting [u/s 32(4) and (5)]
A person may apply to the CIR in writing for change in method of accounting and such approval by the CIR in
writing shall be allowed only on satisfaction that such change is necessary to clearly reflect the business income
of such person.
After change in method of accounting the person shall make adjustments to all the items affected by the
change so that no item is omitted and no item is taken into account more than once.
Cash basis of accounting [Section 33]
"Income from Business" on a cash basis shall derive income when it is received and shall incur expenditure when it is
paid.
Accrual basis of accounting [u/s 34(1), (2) and (3)]
A person while accounting for "Income from Business" shall use accrual basis when it is due to the person and shall
incur expenditure when it is payable by the person.
(A) Under accrual when revenue is recognised:
An amount shall be due to a person when the person becomes entitled to receive it even if the time for
discharge of the entitlement is postponed or the amount is payable by instalments.

Conceptual Approach to Taxes 171


Method of Accounting & Records Chapter-11

(B) Under accrual when expenditure is recognised:


An amount shall be payable by a person when all the events that determine liability have occurred and the
amount of the liability can be determined with reasonable accuracy.
Example (covering both cash and accrual accounting methods): In tax year 2016 there was payment of
Rs.100,500 on account of electricity bills however the Rs.8,005 bill for the month of June-2016 was paid in July-2016.
Under cash accounting system:
The payment of Rs.100,500 shall be recorded under the cash accounting system whereas there will be no recording
of June bill paid in July-2016 in the books of account for the tax year 2016.
Under accrual accounting system:
The electricity bill of June paid in July-2016 shall be recorded in the books of account along-with payment of
Rs.100,500 related to first eleven months under the accrual accounting system.
3. Trading liability not paid and its subsequent full or part payment [u/s 34(5) and (6)]
Non-payment of an allowed expenditure (under the head business income) after a period of three years of the end
of the tax year in which the deduction was allowed will result into inclusion of the same in the business income in the
first tax year following the end of the 3 years. However where a person subsequently after the aforesaid
disallowance pays the liability or a part of liability the person shall be allowed a deduction for the amount paid in the
tax year in which payment is made.
Important Note: The aforesaid section is not applicable on where the debit side of a liability is not a tax expense e.g.
loan payable or advance from customers.
Example: Mr. A purchased goods on credit worth Rs.5,000 in tax year 2012. However, he paid this amount in tax
year 2016. Explain the treatment of this transaction.
Solution:
Tax year 2012 (the year in which expense claimed)
Amount of Rs. 5,000 shall be treated as admissible expense.
Tax year 2015 (on expiry of said three years)
Amount of Rs. 5,000 shall be added in taxable income as it has not been paid within three years from the end of the
tax year in which it was treated as admissible expense.
Tax year 2016 (on payment after disallowance of expense)
Rs.5,000 shall be deducted from taxable income.
4. Stock in trade [u/s 35(1), (2), (3) and (4)]
For "Income from Business" the cost of stock in trade disposed of by the person in the year shall be
computed as under, namely:
Stock consumed = Opening stock + Cost of stock acquired during the year - closing stock
The opening value of stock for a tax year shall be as under:
the closing value of stock-in-trade at the end of the previous year; or
Where business has started in the year, the FMV of any stock-in-trade at the date when such stock
ventured in the business.
Valuation of inventory shall be lower of cost and NRV at the end of the year.
where NRV stands for = Net Realizable Value
Example: From the following information provided by Zamir Ltd, compute the cost of stock in trade.
Opening stock (at fair value) Rs. 10,000, Purchases Rs.40,000 and Closing stock (cost) Rs. 5,000
Note: The NRV of closing stock is Rs. 4,000.
Solution:
Cost of stock in trade: Rs.
Opening stock 10,000
Add: purchases 40,000
Less: closing stock (at cost) (5,000)
Cost of stock in trade 45,000

172 Conceptual Approach to Taxes


Method of Accounting and Records Chapter-11

Important note: The effect of NRV less than the cost has not been considered as it is abnormal loss and the same
will be charged to profit and loss account.
5. Computation of cost [u/s 35(5) and (6)]
Cash basis of accounting (Has two options for cost of stock in trade):
A person accounting for Income from Business on a cash basis may compute cost of stock on either prime cost
method or absorption cost method, and
Accrual basis of accounting (has only one method for cost of stock in trade:
a person accounting for business income on accrual basis shall compute the cost of stock on absorption cost
method.
For stock in trade not readily identifiable:
Where particular items of stock in trade are not readily identifiable, a person may account for stock on
first in first out method or weighted average cost method
Once chosen a stock valuation method may be changed with the written permission of the CIR on such
conditions as imposed by the CIR.
Definitions
Absorption-cost method means the generally accepted accounting principle under which the cost of
an item of stock-in-trade is the sum of direct material costs, direct labour costs, and factory overhead
costs;
Average-cost method means the generally accepted accounting principle under which the valuation of
stock-in-trade is based on a weighted average cost of units on hand;
Direct labour costs means labour costs directly related to the manufacture or production of stock-in-
trade;
Direct material costs means the cost of materials that become an integral part of the stock-in-trade
manufactured or produced, or which are consumed in the manufacturing or production process;
Factory overhead costs means the total costs of manufacturing or producing stock-in-trade, other than
direct labour and direct material costs;
First-in-first-out method means the generally accepted accounting principle under which the valuation
of stock-in-trade is based on the assumption that stock is sold in the order of its acquisition;
Prime-cost method means the generally accepted accounting principle under which the cost of stock-
in-trade is the sum of direct material costs, direct labour costs, and variable factory overhead costs;
Stock-in-trade means anything produced, manufactured, purchased, or otherwise acquired for
manufacture, sale or exchange, and any materials or supplies to be consumed in the production or
manufacturing process, but does not include stocks or shares; and
Variable factory overhead costs means those factory overhead costs which vary directly with changes
in volume of stock-in-trade manufactured or produced.

6. Income from long term contracts [u/s 36]


A person income from business on an accrual basis shall compute such income for a tax year under a
long term contract on the basis of percentage of completion method.
The percentage of completion of a long term contract by using cost basis formula is as under:
Percentage of completion = Contract cost to date / Contract cost to date plus further expected contract cost x
100
A contract which is not completed within the tax year in which work is commenced is treated as long term
contract. However if it is estimated to be completed within 6 months from the date of its commencement
then it will not be treated as long term contract.
Example: Mr. Zahid entered into a contract for construction of building on July 01, 2014. Total contract price is
Rs.4,500,000 and it shall be completed in 4 years. It has been estimated that total cost to complete the contract is
Rs.3,375,000. In tax year 2016 cost of Rs.843,750 has been incurred on the contract.
Required: Compute the income chargeable to tax in respect of this contract for tax year 2016.

Conceptual Approach to Taxes 173


Method of Accounting & Records Chapter-11

Solution:
Income chargeable to tax in tax year 2016:
Percentage of completion:
(Cost incurred to date / Total contract cost x 100)
843,750 / 3,375,000 x 100 25%
Income chargeable to tax:
Total price of contract x percentage of completion (Rs. 4,500,000 x 25%) 1,125,000
Less total contract cost x percentage of completion (Rs. 3,375,000 x 25%) 843,750
Gross profit for the year 281,250
7. Records [u/s 174]
Except where allowed by the CIR, every taxpayer shall maintain in Pakistan such accounts, documents and records
as may be prescribed.
The CIR may disallow or reduce a taxpayer's claim for a deduction if the taxpayer is unable, without reasonable
cause, to provide a receipt, or other record or evidence of the transaction in support of the claim for
expenditure.
The accounts and documents required to be maintained shall be kept for six years after the end of the tax year to
which they relate;
Provided where any proceeding is pending before any authority or court the taxpayer shall maintain the
record till final decision of the proceedings.
The CIR may require any person to install and use an Electronic Tax Register of such type and description as may be
prescribed for the purpose of storing and accessing information regarding any transaction that has a bearing on the
tax liability of such person.
In this section deduction means any amount debited to trading account, manufacturing account, receipts and
expenses account or profit and loss account.
8. Prescribed books of account [RULE 28 to Rule 31]
Application through Rule 28
These rules apply for records to be kept by the taxpayer u/s 174.
The purpose of these rules is to prescribe the minimum level of books of accounts, documents and records to
be maintained by taxpayers;
A taxpayer accounting for income chargeable under the head "Income from Business" may maintain additional
records, add further columns or particulars in the forms or may keep such records in the manner that suits to the
taxpayer's business.
In this chapter:
"Legal practitioner" includes an advocate, pleader, tax practitioner and advisor or consultant on income tax,
sales tax, customs, central excise or salt tax laws.
"Medical practitioner" includes a doctor, surgeon, physician, dentist, psychiatrist, physiotherapist, tabib,
homeopath, vaid, veterinarian and any person practicing medicine under any other name.
Books of account, documents and records to be maintained [Rule 29]
Every taxpayer deriving income chargeable under the head "Income from business" shall maintain proper books of
account, documents and records with respect to -
all sums of money received and expended by the taxpayer and the matters in respect of which the receipt and
expenditure take place;
all sales and purchases of goods and all services provided and obtained by the taxpayer;
all assets and liabilities of the taxpayer; and
in case of a taxpayer engaged in assembly, production, processing, manufacturing, mining or like activities, all
items of cost relating to the utilization of materials, labour and other inputs.
If a taxpayer uses fiscal electronic cash register or computerized accounting software, it may issue cash-
memo/invoice/receipt generated by the electronic cash register or computer.
Duplicate copies and electronic or computer records of the cash-memo / invoice / receipt / patient-slip to be issued,
shall be retained by the taxpayer and form part of the records to be maintained.
The books of account, documents and records to be maintained for 5 years after the end of the tax year to which they
relate.

174 Conceptual Approach to Taxes


Method of Accounting and Records Chapter-11

The following are the various Rules regarding books of account to be maintained by various taxpayers.

S. No. Class of tax payers Rule


1 Taxpayer with business income up to Rs.200,000 or a new taxpayer 30(1)
2 Taxpayer with business income exceeding Rs.200,000 30(2)
3 Professionals 30(3)
4 Manufacturers having turnover more than Rs.2.5 million 30(4)
5 Electronic Tax Register (ETR) 30A
6 Non business taxpayers 31
1. Taxpayer with business income upto Rs.200,000 or a new taxpayer [Rule 30(1)]
Serially numbered and dated cash-memo / invoice / receipt for each transaction of sale or receipt containing the
following:
taxpayer's name or the name of his business, address, NTN and sales tax registration number, if any; and
the description, quantity and value of goods sold or services rendered;
Provided that where each transaction does not exceed Rs.100, one or more cash-memos per day for all such
transactions may be maintained;
Daily record of receipts, sales, payments, purchases and expenses, a single entry in respect of daily receipts, sales,
purchases and different heads of expenses will suffice; and
Vouchers of purchases and expenses.
2. Taxpayer with business income exceeding Rs.200,000 [Rule 30(2)]
Taxpayers with business income exceeding Rs. 200,000 and wholesalers, distributors, dealers and commission
agents:
Serially numbered and dated cash-memo / invoice / receipt for each transaction of sale or receipt containing the
following:
taxpayer's name or the name of his business, address, NTN and sales tax registration number, if any;
the description, quantity and value of goods sold or services rendered; and
in case of a wholesaler, distributor, dealer and commission agent, where a single transaction exceeds
Rs.10,000, the name and address of the customer;
Provided that where each transaction does not exceed Rs.100, one or more cash-memos per day for all such
transactions may be maintained;
Cash book and /or bank book or daily record of receipts, sales, payments, purchases and expenses, a single entry in
respect of daily receipts, sales, purchases and different heads of expenses will suffice;
General ledger or annual summary of receipts, sales, payments, purchases and expenses under distinctive heads;
Vouchers of purchases and expenses and where a single transaction exceeds Rs. 10,000 with the name and address
of the payee; and
Where the taxpayer deals in purchase and sale of goods, quarterly inventory of stock-in-trade showing description,
quantity and value.
3. Professionals [Rule 30(3)]
Professionals (like medical practitioners, legal practitioners, accountants, auditors, architects, engineers etc.):
Serially numbered and dated patient-slip / invoice / receipt for each transaction of sale or receipt containing the
following:
taxpayer's name or the name of his business or profession, address, NTN and sales tax registration number, if
any;
the description, quantity and value of medicines supplied or details of treatment/ case/ services rendered
(confidential details are not required) and amount charged; and
the name and address of the patient / client;
Provided that the condition of recording address of the patient on the patient slip under this clause shall not apply to
general medical practitioners;
Daily appointment and engagement diary in respect of clients and patients:
Provided that this clause shall not apply to general medical practitioners;

Conceptual Approach to Taxes 175


Method of Accounting & Records Chapter-11

Daily record of receipts, sales, payments, purchases and expenses; a single entry in respect of daily receipts, sales,
purchases and different heads of expenses will suffice; and
Vouchers of purchases and expenses.
4. Manufacturers having turnover more than Rs.2.5 million [Rule 30(4)]
Serially numbered and dated cash-memo / invoice / receipt for each transaction of sale or receipt containing the
following:
taxpayer's name or the name of his business, address, NTN and sales tax registration number, if any;
the description, quantity and, value of goods sold;
where a single transaction exceeds Rs. 10,000 with the name and address of the customer;
Cash book and / or bank book;
Sales day book and sales ledger (where applicable);
Purchases day book and purchase ledger (where applicable);
General ledger;
Vouchers of purchases and expenses and where a single transaction exceeds Rs. 10,000 with the name and address
of the payee; and
Stock register of stock-in-trade (major raw materials and finished goods) supported by gate in-ward and outward
records and quarterly inventory of all items of stock-in-trade including work-in-process showing description, quantity
and value.
5. Electronic tax register (ETR) [Rule 30A]
A person required to use an ETR shall -
Install the ETR within 7 days of its authentication by CIR holding jurisdiction over such case and obtain a register
identification number (RIN) for permanent affixture on the ETR;
Use the ETR to record only his own sales and ensure that each sale is made through it and print the receipt of each
safe containing the information in accordance with sub-rules(3) and (4) of rule 29 and rule 30, and to deliver the
original receipt to the purchaser;
In case of non-availability for use of the ETR, the safes may be recorded with the use of a substitute ETR, duly
authenticated by the CIR;
Prepare a daily and a monthly Accounting report containing the information as prescribed in these rules;
Ensure that the ETR operates correctly with particular regard to correct programming of the names of goods and
services and the correct allocation of their tax rates;
Promptly report any malfunctioning of the ETR to the person responsible for its servicing;
On demand by an authorized person, produce the ETR for inspection;
Ensure the inspection of the ETR before the authorized service management after 6 months;
Keep copies of ETR reports for a period of 5 years and produce the same for inspection by the CIR whenever
required to do so;
Safely keep the ETR ledger in the ETR casing and produce it whenever required by the CIR to do so; and
Ensure the inspection before further use of an ETR which has been or is suspected to have been interfered or
tempered with.
6. Non business taxpayers [Rule 31]

S. # Head of Income Record to be kept by the taxpayer


1. Income from salary Salary certificate indicating the amount of salary and tax
deducted there from.
2. Income from property Tenancy agreement, if executed;
Tenancy termination agreement, if executed;
Receipt for amount of rent received; and
Evidence of deductions claimed in respect of premium
paid to insure the building, local rate, tax, charge or cess,
ground rent, profit / interest or share in rent on money
borrowed, expenditure on collecting the rent, legal
services and unpaid rent.

176 Conceptual Approach to Taxes


Method of Accounting and Records Chapter-11

3. Capital gains Evidence of cost of acquiring the capital asset;


Evidence of deduction for any other costs claimed; and
Evidence in respect of consideration received on disposal
of the capital asset.

4. Income from other sources


(a) Dividend Dividend warrants

(b) Royalty Royalty agreement

(c) Profit on debt Evidence and detail of profit yielding debt;


Evidence of profit on debt and tax deducted thereon, like
certificate in the prescribed form or bank account
statement; and
Evidence of Zakat deducted, if any.

(d) Ground rent, rent from the sub-lease of Lease agreement and
land or building, income from the lease Lease termination agreement.
of any building together with plant or
machinery and consideration for
vacating the possession of a building or
part thereof:
Annuity or Pension Evidence of amount received.

(e) Prize money on bond, winning from a Evidence of income and tax deducted thereon, like
raffle, lottery or cross word puzzle certificate in the prescribed form.

(f) Provision, use or exploitation of Agreement


property
(g) Loan, advance, deposit or gift Evidence of mode of receipt of a loan, advance, deposit or
gift i.e., by a crossed cheque or through a banking
channel.
(h) General Evidence of deduction for any other expenditure claimed.

General form of books of accounts, documents and records [Rule 32]


The books of accounts, records and other documents required to be maintained by a taxpayer under these rules
may be kept on electronic media, provided sufficient steps have been taken to ensure the sanctity and safe keeping
of such accounts, documents and records.
The books of accounts, documents and records required to be maintained by a company under these rules shall
be maintained in accordance with international accounting standards and as required under the Companies
Ordinance, 1984.
Books of account, documents and records to be kept at the specified place [Rule 33]
(A) Where person derives business income:
The books of accounts, documents and records required to be maintained by a taxpayer under these rules shall be
kept at the place where the taxpayer is carrying on the business or, where the business is carried on in more
places than one, at the principal place of business or at each of such places if separate books of accounts are
maintained in respect of each place.
(B) Where person derives income other than business:
Where a person derives income from sources other than from business, books of accounts, documents and
records shall be kept at person's place of residence or such other place as may be so declared by such person.
The place or places where the books of accounts, documents and records are kept shall be clearly stated on the tax
return in the required column.

Conceptual Approach to Taxes 177


Method of Accounting & Records Chapter-11

MULTIPLE CHOICE QUESTIONS


Q.1. Companies account for their business transactions on
(a) Cash basis
(b) Accrual basis
(c) Both a and b
(d) None of these
Q.2. Individual and AOP may account for their all heads of income transactions on
(a) Cash basis
(b) Accrual basis
(c) optional from (a) or (b)
(d) None of the above
Q.3. The value of closing stock is computed on
(a) Cost basis
(b) NRV
(c) Lower of cost and NRV
(d) Higher of cost and NRV
Q.4. A person may change the method of accounting with the approval of
(a) No approval required
(b) CIR or
(c) Taxation Officer
Q.5. _______method may be used by the persons using cash basis of accounting.
(a) Marginal cost
(b) Absorption cost
(c) optional from (a) or (b)
(d) None of these
Q.6. Where particular items of stock in trade are not identifiable, _______cost method should be used.
(a) Prime
(b) Absorption
(c) FIFO
(d) LIFO
(e) none of (a) to (d)
Q.7. Under the ITO, 2001 long-term contracts take ________tax year to get completed
(a) More than one
(b) More than three
(c) More than five
(d) More than ten
Q.8. The income arising out of long term contracts is computed on basis.
(a) Cash
(b) Accrual
(c) Percentage of completion computed on revenue basis
(d) None of these
Q.9. The record maintained by a taxpayer required to be kept for a period of ________years after the end of year to which
they relate.
(a) one
(b) Two
(c) Three
(d) Six

178 Conceptual Approach to Taxes


Method of Accounting and Records Chapter-11

Q.10. The books of accounts of companies should be maintained in accordance with


(a) International Accounting Standard
(b) Companies Ordinance, 1984
(c) Both a and b
(d) None of the above
Q.11. An individual deriving income from business is required to maintain books at
(a) Residence
(b) Place of business
(c) any other place
(d) None of these
Q.12. A person deriving income from property is required to maintain _________ documents.
(a) Rent deed
(b) Tenancy termination agreement
(c) Receipt of rent
(d) All of the above
Q.13. In Prime cost method the cost of stock in trade consists of
(a) Direct material cost
(b) Direct labour cost
(c) Variable factory cost
(d) All of these
Q.14. A person deriving income from royalty is required to maintain
(a) Royalty agreement
(b) Rent deed
(c) Cost of acquisition
(d) All of these
Q.15. Where the person has commenced the business during the year, the opening value of stock shall be ---- at the time
when the stock is ventured in business.
(a) Fair market value
(b) Actual value
(c) none of these
Q.16. Person who is required to use the ETR shall keep the copies of ETR for the period of -------.
(a) 2 years
(b) 5 years
(c) 6 years
Q.17 Dividend income received by a company shall be taxed on_________.
(a) Accrual basis
(b) cash basis
(c) both a or b
(d) none of above
Q.18 Method of accounting for companies for business income is ____________.
(a) Accrual basis
(b) cash basis
(c) both a or b
(d) none of above
Q.19 A long term contract is a contract which is not completed in ______.
(a) one year

Conceptual Approach to Taxes 179


Method of Accounting & Records Chapter-11

(b) two years


(c) three years
(d) four years
Q.20 A long term contract may be for ____:
(a) Manufacture
(b) installation
(c) lease of assets
(d) all of above
Q.21 A person deriving income from other sources may keep his books of account at his ________.
(a) College
(b) university
(c) residence
(d) all of above
Q.22 A person may add ________ in the prescribed forms, according to his own requirements.
(a) Columns
(b) particulars
(c) both a and b
(d) none of above
Q.23 A person with a business income up to Rs.________ is required to maintain the vouchers of purchases and
expenses.
(a) 100,000
(b) 200,000
(c) 300,000
(d) 400,000
Q.24 __________ issued is the minimum document that is required to be maintained by a salaried taxpayer.
(a) Salary certificate
(b) salary slips
(c) detail of personal expenses
(d) all of above
Q.25 Certificate for the tax deducted at source is the evidence of _________.
(a) Filing of income tax return
(b) deduction of tax
(c) none of above
Q.26 Deduction means any amount _____ to trading account, manufacturing account, receipts and expenses account or
profit and loss account
(a) Debited
(b) credited
(c) added
(d) all of above

ANSWERS
1 (b) 2 (c) 3 (c) 4 (b) 5 (c)
6 (e) 7 (a) 8 (d) 9 (d) 10 (c)
11 (b) 12 (d) 13 (d) 14 (a) 15 (a)
16 (b) 17 (b) 18 (a) 19 (a) 20 (d)
21 (c) 22 (c) 23 (b) 24 (a) 25 (b)
26 (a)

180 Conceptual Approach to Taxes


Method of Accounting and Records Chapter-11

ICMAP PAST PAPERS THEORECTICAL QUESTIONS


Q. No. 3 (b) Spring As per rule 29 of the Income Tax Rules, 2002 every taxpayer deriving income chargeable under the
head Income from business shall maintain proper books of accounts, documents and records. List down such
documents and records and also state the period for which these records shall be maintained by the taxpayer.

Q. NO. 1(c) February 2013 (i) You have been appointed as Tax Adviser of Mr. Lodhi who has various residential and
commercial properties in the various parts of the city, He has rented out his properties to different tenants. Advise
Mr. Lodhi about the list of records which shall be issued and maintained by every taxpayer deriving income from
property.
Q. NO. 3 (b) SUMMER 2011 List down the minimum books of account, documents and records of taxpayers with business
income upto Rs. 200,000 under Rule 30 of the Income Tax Rules, 2002:
Q. NO. 3 (a) SUMMER 2009 What records are required to be maintained under Rule 29 of the Income Tax Rules, 2002 to
determine income from business?
Q. NO. 2 (b) SUMMER 2007 Define the following terms as per the provisions of Section 35 of the Income Tax Ordinance,
2001:
- Prime-cost-method
- Stock-in-trade
Q. NO. 2(a) SUMMER 2006 How are following defined under the Income Tax Ordinance, 2001?
(i) Cash-basis accounting
(ii) Accrual basis accounting
Q.NO. 3(b) Spring 2006 What is the basis of stock-in-trade computation under the Income Tax Ordinance, 2001 when the
taxpayer follows the cash basis of accounting?
Q.NO. 4(a) Autumn 2006 A company may account for income chargeable to tax under the head income from business on
cash basis or on accrual basis. Briefly discuss the rules relating to accrual of income and expenditure as explained in the
ITO, 2001.
Q. NO. 3 (a) WINTER 2005 Explain the following terms as defined in Income Tax Ordinance, 2001.
(i) Absorption cost method
(ii) Factory overhead costs
(iii) Prime cost method
(iv) Stock-in-trade
Q.NO. 3(a) Autumn 2003 Describe the method of accounting to be adopted by a person deriving business income from a
Long Term Contract?

Conceptual Approach to Taxes 181


Method of Accounting & Records Chapter-11

CA MOD C PAST PAPERS THEORECTICAL QUESTIONS


Q. No. 4 (a) Spring 2013 Inspired Pakistan Limited (IPL) wants to change its accounting year from 30 June to 31 December
as the income year of its parent company in USA ends on 31 December.
Required:
Advise IPL about the requirements of the Income Tax Ordinance, 2001 regarding the change for tax year from Normal to
special.
Q. No. 5 (a) Spring 2013 Describe the methods of accounting that may be adopted under the Income Tax Ordinance, 2001
by following persons deriving income chargeable to tax under the head ' Income from Business'.
(i) A company
(ii) Any person other than a company
(iii) An AOP deriving business income from a ' Long Term contract'
(b) State the provisions of the Income Tax Ordinance, 2001 relating to the change in method of accounting for income
chargeable to tax under the head ' Income from Business'.
Q.2 (b) March 2007 Discuss the provisions of the Income tax Ordinance, 2001 relating to the computation of opening and
closing stock.
Q. NO. 2(b) Spring 2007 Discuss the provisions of the Income tax Ordinance, 2001 relating to the computation of opening
and closing stock.
Q.3 (b) March 2006 What is the basis of stock-in-trade computation under the Income Tax Ordinance when the taxpayer
follows the cash basis of accounting?
Q. NO. 4(a) Spring 2004 Briefly explain when the expenditure is considered as incurred with reference to accrual basis of
accounting defined in the Income tax Ordinance, 2001.
Q. NO. 4(b) Spring 2004 Please mention the costing method(s) and stock valuation method(s) to be applied by a person
following accrual basis of accounting to account for income chargeable to tax under the head income from business under
the Ordinance.
Q. NO. 7(b) Spring 2004 For how many years the tax payer is required to maintain accounts and documents under the
relevant provision of the ITO, 2001.
Q. NO. 9 Autumn 2003 Briefly describe the minimum books of accounts, documents, and records that are required to be
maintained by the following taxpayers?
(a) Taxpayer (other than a company) deriving business income up to Rs.200,000.
(b) Taxpayer (other than a company) deriving business income exceeding Rs.200,000.
Q. NO. 6(b) Spring 2003 What method of accounting is required to be employed by a company deriving income from
business.
Q. NO. 6(c) Spring 2003 Briefly state the provisions relating to the change in the method of accounting of income from
business.

182 Conceptual Approach to Taxes


Capital Gains Chapter-12

Chapter

12 CAPITAL GAINS

Topic covered
Section
Rule (For CAF-6 AND ICMAP students)

37 Capital assets
Procedure to determine capital gain
37A 13F & 13H Gains on sale of securities
100B
Special provision relating to capital gain tax on securities [with Eight Schedule]
Loss u/s 37 & 37A
11, 50, 51 &
Geographical source of capital gains & their taxability
101
Capital gain exempt from tax
Treatment of bonus shares
Disposal
MCQs with solutions
Practice questions with solutions
ICMAP & CA past papers theoretical questions

For CAF-6 and ICMAP students


1. Definition
A gain arising on the disposal of a capital asset by a person in a tax year, (except exempt gain) shall be
chargeable to tax under the head capital gains.
Taxation of capital assets is split into two categories as:
A. Capital assets u/s 37.
B. Capital assets u/s 37A.
1.1 Capital assets: [Section 2(10) and 37(5)]
Capital asset means property of any kind held by a person, whether or not connected with a business, but
does not include the following:
Any stock-in-trade, consumable stores or raw materials held for the purpose of business;
Any depreciable and intangible property; or
Any movable property [(excluding the capital assets defined in section 38(5)] held for personal use by the
person or any member of the persons family dependent on the person. U/s 38(5) has stated the following are
capital assets:
A painting, sculpture, drawing or other work of art;
Jewellery;
A rare manuscript, folio or book;
A postage stamp or first day cover;
A coin or medallion; or
An antique.
Example: From the following information provided by Mr. Hamid, compute income chargeable to tax under the
head capital gains. (Ignore holding period of the assets)
(a) Profit on sale of finished goods Rs. 100,000.
(b) Gain on sale of jewellery Rs.50,000.

Conceptual Approach to Taxes 183


Capital Gains Chapter-12

(c) Gain on sale of personal car Rs. 350,000.


(d) Gain on sale of Antique Rs.10,000
Solution:
Mr. Hamid
Computation of taxable income and tax liability:
Capital gains: Rs.
Gain on sale of jewellery 50,000
Gain on sale of antique 10,000
Taxable income 60,000
Note: Item (a) and (c) have not been considered, as the same are not capital assets.
1.2 Capital assets u/s 37A:
Under this section capital assets are termed as securities. Securities include the following:
Shares of a public company,
Voucher of Pakistan Telecommunication Corporation,
Modarba Certificate,
An instrument of redeemable capital: defined in the Companies Ordinance, 1984 includes finance
obtained on the basis of participation term certificate (PTC), musharika certificate, term finance
certificate (TFC), or any other security or obligation not based on interest, other than ordinary share of a
Company, representing an instrument or a certificate specified denomination, called the face value or
nominal value, evidencing investment of the holder in the capital of the company on terms and
conditions of the agreement for the issue of such instrument or certificate or such other certificate or
instrument as the Federal Government may, by notification in the official Gazette, specify for the
purpose.
Debt securities means;
(a) Corporate debt securities such as TFCs, Sukuk Certificates (Sharia Compliant Bonds),
registered bonds, commercial paper, PTCs and all kinds of debt instruments issued by any
Pakistani or foreign Company or corporation registered in Pakistan; and
(b) Govt. debt securities such as treasury bills (T-bills), Federal Investment Bonds (FIBs), Pakistan
Investment Bonds (PIBs), Foreign Currency Bonds, Govt. papers, Municipal Bonds,
infrastructure Bonds and all kinds of debt instruments issued by foreign Govt., Provincial Govt,
local authorities and other statutory bodies.
Derivative products: Derivative products means a financial product which derives its value from the
underlying or other asset, may be traded on a stock exchange of Pakistan and includes deliverable
future contracts, cash settled future contracts, contracts of rights and options.
Public company u/s 2(47) means
A Company listed in Pakistan at the year end; or
A Company in which 50% or more shares are held by the Federal or Provincial Government or a foreign
Government or a foreign company wholly owned by a foreign Government or
A unit Trust.
Example: Which of the following capital assets are securities?
1. Jewellery
2. Shares of private company
3. Shares of public company
4. Painting
5. Musharika certificates
6. Vouchers of Pakistan Telecommunication corporation
7. First day cover
Solution: Item No. 3, 5 and 6 are securities, whereas the others fall in the definition of capital assets u/s 37.

184 Conceptual Approach to Taxes


Capital Gains Chapter-12

2. Procedure to determine capital gain: [U/s 37(2), (3) and (4)]


2.1 The gain arising on the disposal of a capital asset by a person shall be computed in accordance with the
following formula:
Consideration received by the person on disposal
(Higher of fair market value or actual amount received)
Less: cost of the asset [consist of the following u/s 76(2)]
Consideration given for the asset
Incidental expenditure incurred in acquiring and disposing off the asset
Any expenditure incurred to alter or improve the asset
Balance shall be the capital gain or loss
2.2. No amount shall be included in the cost of a capital asset for any expenditure incurred by a person that is
or may be deducted under another provisions of the Ordinance or that is referred as inadmissible u/s 21.
[U/s 38(4)]
2.3 Where a capital asset (other than immovable property) has been held by a person for more than one year
(other than capital assets defined in section 37A) the amount of gain arising on disposal of the asset shall
be taken as 3/4th while the balance 1/4th shall automatically be excluded from the taxable capital gain.
However if there is capital loss under this section the same shall be fully recognized without the
application of said exemption. [U/s 37(3)]
2.4 Capital gain on disposal of immovable property [U/s 37(3)
The exemption of 25% on disposal of capital assets is not available to immovable property.
Gain arising on the disposal of immovable property, held for a period up to 2 years, by a person in a tax
year, shall be chargeable to tax in that year under the head Capital Gains at the rates specified below:

S. No. Holding period Rate of tax


1. Where holding period of immovable property is up to one year. 10%
Where holding period of immovable property is more than one year but not
2. 5%
more than two years.

2.5 Where the capital asset becomes the property of the person under a gift, bequest, will, by succession,
inheritance, devolution, distribution of assets on dissolution of an AOP, or distribution of assets on
liquidation of a company the fair market value of the asset on the date of its transfer or acquisition shall
be treated to be the cost of the asset at the time of its disposal. [U/s 37(4A)] (Example B attached)
However no gain or loss shall be recognized at the original dates when the capital asset becomes the property
of the person under a gift, bequest and will etc. [U/s 79]
Example A Following information has been provided by Mr Ali:
Rs.
Consideration received on sale of share of a private company 96,000
Purchase price of the shares 20,000
Expenses incurred on purchase of shares 2,000
Expenses incurred on sale of shares 3000
Required: Compute income chargeable to tax under the head capital gains assuming:
(a) Holding period of the shares is 8 months.
(b) Holding period of the shares is 15 months.
Solution: (a)
Rs.
Consideration received 96,000
Less: Cost of shares:
Purchase price 20,000
Expenses on purchase 2,000
Expenses on disposal 3,000
25,000
Gain on disposal of shares (totally chargeable to tax as sold within one year) 71,000

Conceptual Approach to Taxes 185


Capital Gains Chapter-12

Solution: (b) Rs.


Actual capital gain as above = 71,000
Taxable capital gain (71,000 x 3/4th) = 53,250

Example B Mr Amir purchased 10,000 shares of a private limited company in tax year 2010 for Rs. 100,000.
In tax year 2016 he transferred 5,000 shares to his wife under an agreement to live apart. Further he has
gifted to his son 2,000 shares and sold remaining shares for Rs.60,000. Compute taxable income of Mr. Amir
under the head capital gain for tax year 2016.

Solution:
M R. AMIR
TAX YEAR 2016 Rs.

Capital gains:
3,000 shares sold for Rs. 60,000 60,000
Less: cost of 3,000 shares sold (100,000 x 3,000 / 10,000) (30,000)
Total capital gain 30,000
Taxable capital gain 3/4th of Rs.30,000 22,500
Note: No gain or loss has been recognised on disposal of 5,000 shares to his wife under an agreement to live
apart and gift of 2,000 shares to his son.
Example C Following information is related to Mr. K. (All amounts are in rupees)

Purchase Consideration received FMV at the time Holding


Capital asset
price on disposal of sale period
Shares of private company 50,000 40,000 60,000 6 months
House 1 1,000,000 1,500,000 1,500,000 1.5 years
House 2 1,500,000 2,000,000 2,000,000 3 years

Assuming he has no other taxable income, compute tax payable by Mr. K. for tax year 2016.
Solution:
Mr. K
Computation of taxable income and tax liability: Rs.
Shares of private company:
Consideration on disposal (higher of FMV or actual amount) 60,000
Purchase price (50,000)
10,000
House 1:
Consideration on disposal (higher of FMV or actual amount) 1,500,000
Purchase price (1,000,000)
500,000
House 2:
(Holding period is more than 2 years, hence nothing is taxable in case of house 2 ______-_____
Income taxable under NTR 510,000
Less: chargeable to tax under SBI 500,000
10,000
Tax liability (Income is below taxable limit), hence tax liability is zero -
Tax liability under SBI (500,000 x 5%) 25,000
Total tax liability 25,000

186 Conceptual Approach to Taxes


Capital Gains Chapter-12

Explanation of section 37 and 38

Is asset is a capital asset?

No: Yes:
Although gain shall not taxable under If chargeable to tax
capital gains but may be taxable as
business income or income from
other sources.

No: Yes:
The same may be exempt from tax If it is an asset mentioned under
under 2nd Schedule section 38(5)

Yes: No:
No loss will be recognized only gain will be Gain and loss both will be recognized
chargeable to tax by taking 25% exemption by taking 25% exemption on capital
on capital gain where applicable. gain where applicable.

3. Gain on sale of securities: [U/s (37A), Rule 13A 13N]


Capital gain on disposal of securities (other than exempt from tax) shall be treated under NTR as SBI from 01
July 2010. Tax shall be charged on capital gain on disposal of capital assets acquired for a period which is less than
24 months at following rates.
The gain arising on the disposal of a security by a person shall be computed in accordance with the following
formula:
Gain = (Consideration received by the person on disposal of the security) (Cost of acquisition of the security)
Gain from capital assets under this section shall not apply on gain on disposal of securities by a banking company or
an insurance company.
S. No. Period Tax Year Rate of tax
(1) (2) (3) (4)
1. Where holding period of a security is less than 12 months. 2016 15%
Where holding period of a security is equal or more than 12 months
2. 2016 12.5%
but less than 24 months.
Where holding period of a security is equal or more than 24 months 2016 7.5 %
3.
but less than four years.
3. Where holding period of a security is equal or more than four years. 2016 0%

Securities held for a period upto a maximum of one hundred eighty two days (182) and for a period upto a
maximum of three sixty five days (365) shall be taken as held for six months and one year respectively.
Capital gain arising on the disposal of any security shall be computed on the basis of First in First out
(FIFO) inventory accounting method. However, FIFO method shall not apply in respect of sale of shares
purchased on the same trading day. In that case gain or loss shall be computed by applying the average
method.
TAXATION OF GAIN ON DEBT SECURITIES FOR COMPANIES (Part I of first Schedule Proviso to Division
VII)
Capital gain arising from disposal of securities shall be treated as a separate block of income. For rate
purposes this income is split into two categories, namely, debt securities and all other securities.
Tax on gain on disposal of debt securities shall be 32% for companies and for small companies 25% for the tax
year 2016.

Conceptual Approach to Taxes 187


Capital Gains Chapter-12

COST OF ACQUISITION [Rule 13K(d)]


Cost of acquisition of any security means the market price of the security which the investor pays or would
have paid to purchase such security. Besides this general rule, the following principles shall apply while the cost
of securities acquired:
Sr. Particulars Cost of acquisition
1. Purchase of shares The market price of the security paid by the investor.
2. Right shares Discounted price at which the right shares are issued.
3. Acquisition through Market price of the security at which the deceased person and or would have
bequest or inheritance paid to purchase such security.
4. Bonus shares Market price of bonus shares immediately following the bonus shares which the
investor would have paid to purchases such shares.
5. Initial public offering (IPO) Actual price paid to the issuer.

Special provision relating to capital gain tax [U/S 100B read with Eight Schedule]
Capital gains on disposal of listed securities and tax thereon, subject to section 37A, shall be computed, determined,
collected and deposited in accordance with the rules laid down in the Eighth Schedule.
The above provisions shall not apply on the following:-
(a) mutual fund, a modaraba;
(b) Banking company, a non-banking finance company and an insurance company subject to tax under the Fourth
Schedule;
(c) a company, in respect to debt securities only; and
(d) Any other person or class of persons notified by the Board.

EIGHT SCHEDULE
RULES FOR THE COMPUTATION OF CAPITAL GAINS ON LISTED SECURITIES
1. Manner and basis of computation of capital gains and tax thereon
(1) Capital gains on disposal of listed securities, subject to tax under section 37A, and to which section 100B
apply, shall be computed and determined under this Schedule and tax thereon shall be collected and
deposited on behalf of taxpayers by NCCPL in the manner prescribed.
(2) For the purpose of sub-rule (1), NCCPL shall develop and automated system.
(3) Central Depository Company of Pakistan Limited shall furnish information as required by CCPL for discharging
obligations under this Schedule.
(4) NCCPL shall issue and annual certificate to the taxpayer on the prescribed form in respect of capital gains
subject to tax under this Schedule for a financial year:
(5) Provided that on the request of a taxpayer of if required by the commissioner, NCCPL shall issue a certificate
for a shorter period within a financial year.
(6) Every taxpayer shall file the certificated referred to in sub-rule (4) along with the return of income and such
certificate shall be conclusive evidence in respect of the income under this Schedule.
(6) NCCPL shall furnish to the Board within the Board within thirty days of the end of each quarter, a statement of
capital gains and tax computed thereon in that quarter in the prescribed manner and format.
(7) Capital gains computed under this Schedule shall be chargeable to tax at the rate applicable in Division VII of
part 1 of the First Schedule.
(8) The provisions of section 4B shall apply to the taxpayers under this schedule and taxed at the rates specified
in Division IIA of Part 1 of the First Schedule.
2. Sources of Investment
(1) Where a person has made any investment in the listed securities, enquires as to the nature and source of the
amount invested shall not be made for any investment made prior to the introduction of the Schedule, provided
that-
(a) a statement of investment s is filed with the Commissioner along with the return of income and wealth
statement for tax year 2012 within the due date as provided in section 118 of this Ordinance and in the
manners prescribed; and
(b) that the amount remains invested for a period of forty- five days upto 30th of June 2012, in the manner
as may be prescribed.

188 Conceptual Approach to Taxes


Capital Gains Chapter-12

(2) Where a person has made any investment in the shares of a public company traded at a registered stock
exchange in Pakistan from the date of coining into force of this Schedule till June 30, 2014, enquiries as to the
nature and sources of amount invested shall not be made provided that
(a) the amount remains invested for a period of one hundred and twenty days in the manner as may be
prescribed;
(b) tax on capital gains, if any, has duly been discharged in the manner laid down in this Schedule; and
(c) a statement of investments is filed with the Commissioner along with the return of income and wealth
statement for the relevant tax year within the due date as provided in section 118 of this Ordinance and
in the manner prescribed.
(3) For the purpose of this rule, amount of investment shall be calculated in the prescribed manner, excluding
market value of net open sale position in futures and derivatives, if such sale is in a security that constitutes
the said investment.
3. Certain provisions of this Ordinance not to apply
The respective provisions for collection and recovery of tax, advance tax and deduction of tax at source laid down in
the Parts IV and V of Chapter X shall not apply on the income from capital gains subject to tax under this Schedule
and these provisions shall apply in the manner as laid down in the rules made under this Ordinance, except where
the recovery of tax is referred by NCCPL to the Board in terms of rule 6(3).
4. Payment of tax collected by NCCPL to the Board
The amount collected by NCCPL on behalf of the Board as computed in the manner laid down under this Schedule
shall be deposited in a separate bank account with National Bank of Pakistan and the said amount shall be paid to
the Board along with interest accrued thereon on yearly basis by July 31st next following the financial year in which
the amount was collected.
5. Persons to whom this Schedule shall not apply
If a person intends not to opt for determination and payment of tax as laid down in this Schedule, he shall file an
irrevocable option to NCCPL after obtaining prior approval of the Commissioner in the manner prescribed. In such
case the provisions of rule 2 shall not apply.
6. Responsibility and obligation of NCCPL
(1) Pakistan Revenue Automation Limited (PRAL), a company incorporated under the of Companies Ordinance,
1984 or any other company or firm approved by the Board and any authority appointed under section 209 of
this Ordinance, not below the level of an Additional Commissioner Inland Revenue, shall conduct regular
system and procedural audits of NCCPL on quarterly basis to verify the implementation of this Schedule and
rules made under this Ordinance.
(2) NCCPL shall implement the recommendations, if any, of the audit report under sub-rule (1), as approved by
the Commissioner, and make adjustments for short or excessive deductions. However, no penal action shall
be taken against NCCPL on account of any error, omission or mistake that has occurred from application of
the system as audited under sub-rule (1).
(3) NCCPL shall be empowered to refer a particular case for recovery of tax to the Board in case NCCPL is
unable to recover the amount of tax.
7. Transitional Provisions
In respect of tax year 2012, for the period commencing from coming into force of this Schedule till June 30, 2012, the
certificate issued by NCCPL under rule 1(4) shall be the basis of capital gains and tax thereon for that period.
Example: Mr. Adnan sold some shares in tax year 2016. Detail of gain or loss on sale is given below:
Gain / (loss)
Rs.
(a) Shares of Alpha Chemicals (Pvt.) Ltd. 100,000
(Holding period is 15 months)
(b) Shares of Beta Industries Ltd. (Unlisted public Company) (60,000)
(Holding period is 6 months)
(c) Shares of Omega Limited (Listed Company) 10,000
(Holding period 9 months)
(d) Shares of Delta Limited (Listed Company) 20,000
(Holding period 18 months)
Required: Compute taxable income and tax liability for tax year 2016.

Conceptual Approach to Taxes 189


Capital Gains Chapter-12

Solution:
Mr. Adnan
Computation of taxable income and tax liability: Rs.
Capital gain:
Gain on shares of Alpha Chemicals (Pvt.) Ltd.
(Holding period is more than 1 year) (100,000 x 75%) 75,000
Loss on shares of Beta Industries Ltd. (Unlisted Public Company)
(In case of loss, holding period has no effect) (60,000)
Taxable income 15,000
Computation of tax liability:
Tax on taxable income (Below taxable limit) Nil
Tax on income under separate block of income:
Tax on gain on shares of Omega Limited (10,000 x 15%) 1,500
Tax on gain on shares of Delta Limited (20,000 x 12.5%) 2,500
Total tax liability 4,000
4. Loss U/S 37:
Capital losses shall be set off against the capital gains only during the same tax year and where such loss is not so
set off then the balance loss shall be carried forward for adjustment against capital gain up to six succeeding tax
years. If capital gain is exempt from tax then loss from such asset shall have no treatment under capital gain. [U/s38
(1) and (2)]
Example: From the following information, compute the amount of capital loss to be carried forward, if any
Taxable capital gain 40,000
Capital loss 80,000
Note: Out of given capital loss Rs. 20,000 relates to capital asset that is exempt from tax.
Solution:
Total capital loss 80,000
Less: capital loss of exempt capital asset 20,000
60,000
Less: Taxable capital gain 40,000
Capital loss to be carried forward 20,000
No loss shall be recognized on disposal of the assets mentioned below: [U/s 38(5)]
(i) A painting, sculpture, drawing or other work of art;
(ii) Jewellery;
(iii) A rare manuscript, folio or book;
(iv) A postage stamp or first day cover;
(v) A coin;
(vi) A Medallion; and
(vii) An antique.
Example: From following information compute taxable income and tax liability of Mr. A.
Rs.
Gain on sale of painting 40,000
Loss on sale of jewellery 20,000
Loss on sale of shares of an industrial undertaking in EPZ 10,000
Solution:
Capital gain chargeable to tax is Rs. 40,000.
Loss on sale of Jewellery is not recognised.
Loss on sale of shares of industrial undertaking in Export Processing Zone (EPZ) shall also not be recognized as gain
on such shares is exempt from tax.

190 Conceptual Approach to Taxes


Capital Gains Chapter-12

5. Loss U/S 37A:


Capital loss u/s 37A shall be set off against the gain from any other security chargeable to tax during the year; It shall
not be carried forward as it is treated as a separate block of income. [U/s 37A(5)]
Capital loss adjustment disallowed (Rule 13F)
Capital loss adjustment as mentioned above shall not be admissible in the following cases, namely
Wash Sale
Where capital loss realized on disposal of a specific security by an investor is preceded or followed in one months
period by purchase of the same security by the same investor, thus maintaining his portfolio.

Cross Trade
Where coordinated reshuffle of securities between two related accounts of the same investor or between two related
brokerage houses is undertaken and securities accumulating unrealized losses are sold to related accounts to
artificially realize capital losses in one account without actually selling the securities to an outsider.
Tax Swap sale
Where the investor having realized loss on a particular security does not repurchase the same security but
chooses another similar security in the same sector, thus, not only minimizing or eliminating altogether liability
on account of tax on capital gain, but also maintaining the portfolio broadly at the same risk return profile.
There shall be no tax if the securities are held for more than one year;
This section shall not apply to a banking company or an insurance company;
The holding period shall reckon from the date of acquisition to the date of disposal;
Gain under this section shall be treated as a separate block of income.

Payment of tax on capital gain [Rule 13H]


Every investor other than individual shall e-file statement of advance tax on capital gain on the capital gain
on the prescribed format within seven (7) days after the end of each quarter with the tax authority.
The liability to pay the due tax on capital gain shall lie on the investor who held the securities during the
period for which tax on capital gain is to be paid.
Example: Compute tax payable by Mr. Sabir for the tax year 2016 from following information:
Rs.
Gain on sale of shares of Omega Limited (Listed Company) 35,400
(Holding period is less than 6 months)
Loss on sale of shares of Delta Limited (Listed Company) (65,000)
(Holding period is equal to 12 months)
Solution:
Mr Sabir
Tax year 2016
Computation of taxable income and tax liability:
Rs.
Gain on sale of shares of Omega Limited 35,400
Less: Loss on sale of shares of Delta Limited 65,000
Unadjusted loss on sale of securities (29,600)
No tax is payable by Mr. Sabir as there is loss under separate block of income.
Note: The unadjusted loss on sale of security shall not be carry forward.

Conceptual Approach to Taxes 191


Capital Gains Chapter-12

Example: Compute tax payable by Mr. Jamil for the tax year 2016 from following information:
Rs.
1. Gain on sale of shares of Omega Limited (Listed Company) 20,000
(Holding period is less than 12 months)
2. Gain on sale of shares of Apex Limited (Listed Company) 20,000
(Holding period is equal to 12 months)
3. Gain on sale of shares of Zelda Limited (Listed Company) 20,000
(Holding period is equal to 48 months)
4. Loss on sale of shares of Delta Limited (Listed Company) 20,000
(Holding period is more than 12 months but less than 24 months)
Solution: Taxpayer has the option to adjust the loss on sale of security against gain on sale of any security
chargeable to tax during the year. The gain on sale of shares of Zelda Limited is not chargeable to tax because of its
holding period. However capital loss on sale of Delta Ltd. shall be firstly adjusted against the gain of Apex Ltd and
finally the gain of Omega Ltd. shall be charged tax at 15%.
Explanation of section 37A

Where asset is a capital asset is a


security as defined in section 37A(3).

No: Yes:
It may be chargeable to tax under It is chargeable to tax under separate
section 37 as given above block of income

Non applicability of section 37A: Treatment of losses:


a. Securities held for equal or more than 48 Loss shall be set off only against the
months. and gain of the person from any other
b. Banking and insurance companies etc. securities chargeable to tax under this
section and no loss shall be carried
forward to the subsequent tax year.

192 Conceptual Approach to Taxes


Capital Gains Chapter-12

Pakistan source & Foreign source capital gain

Geographical source of capital


Received by Taxability
gain

Pakistan source capital gain


Taxable [Section 11(5) and (6)]
[Section 101]:
In case of non resident the terms &
conditions of double taxation treaty
Gain arising on the disposal of Resident / Non-resident individual
agreement are also relevant for Pakistan
shares in a resident company
source income.
shall be Pakistan-source income.

a. Resident Individual Taxable [Section 11(6)]


[Section 50]
b. Short term resident An individual shall be exempt in respect of his
foreign-source income which is not brought /
received in Pakistan if he is resident only by
[For all foreign source income] reason of his employment and he is present in
Pakistan for not exceeding 3 years.
Foreign source capital gain:
Capital gain other than above. [Section 51]
c. Returning expatriate If an individual citizen of Pakistan (returning
expatriate) is resident in the current tax year
[Citizen of Pakistan coming back but was non-resident in the 4 preceding tax
in Pakistan] years, his foreign-source income shall be
exempt in current tax year and in the following
[For all foreign source income] tax year.
Non-resident individual Not taxable [Section 11(6)]

6. Capital Gains Exempt from tax:


The following capital gains are exempt from tax under Part-I of 2nd schedule:
1. Any distribution received by a taxpayer from a collective investment scheme registered by the Securities and
Exchange of Commission of Pakistan including NIT or a Mutual Fund out of the capital gains of the said
Schemes. Provided that this exemption shall only e available to only such mutual funds, collective investment
schemes that are debt or money market funds and these do not invest in shares U/c (103)
2. Any gain on transfer of a capital asset, being a membership right held by a member of an existing stock
exchange, for acquisition of shares and trading or clearing rights acquired by such member in new
computerized stock exchange in the course of corporatization of an existing stock exchange. U/c (110B)
3. Exemption on capital gains from units in Special Industrial Zones U/c 113 has been omitted from tax year
2016.
4. Any income from sale of shares of an industrial undertaking set up within the meaning of the Export
Processing Zone Authority Ordinance. U/c (114)
7. Treatment of bonus shares
Under the Companies Ordinance, 1984 a company may issue the following three ways to issue the share:
1. Shares issued for cash
2. Shares issued for other than cash
3. Issue of shares without any consideration (Bonus shares)
The taxation of bonus shares of listed companies are covered under the head income from other sources u/s 39 and
to be taxed @ 5% under section 236M on market price on the first day end price on closure of books. However the
Board has reserved the right to prescribe the taxation on bonus shares from unlisted and Private Ltd. Companies.
8. Disposal
The legislature intentionally used the word of disposal and it had not used the words of gain on sale. The reason to
use the term is that the term disposition is wider connotation than sale. It includes exchange, relinquishment and
extinguishments in addition to sale. This is evident from the explanation of disposal given in u/s 75 of the
Ordinance.

Conceptual Approach to Taxes 193


Capital Gains Chapter-12

Sales Ordinary sale involving transfer of assets in lieu of any consideration.


Disposition In relation to a property means disposition made by deed or will and also made by
or under a decree or under orders of court Tribunal or Authority.
Exchange Where transferee does not bring cash but something else in exchange.
Relinquishment of an asset Firm purchases right to manufacture a certain brand and of goods, on dissolution
one partner relinquishes the right to that license for a consideration.
Extinguishments of a right Say three partners of a firm hold right in goodwill. Two partners on dissolution
surrendered their right in favor of third Partner

194 Conceptual Approach to Taxes


Capital Gains Chapter-12

MULTIPLE CHOICE QUESTIONS


Q.1. Gain arising from the disposal of _________ is taxable under the head capital gains.
(a) Depreciable asset
(b) Eligible depreciable asset
(c) Securities
(d) All of the above
Q.2. ____________are capital assets.
(a) Stock in trade
(b) Sculpture
(c) Immovable property
(d) Both b and c
Q.3. A person who derives his income by dealing in shares of private, unlisted and public limited companies are covered
under the head.
(a) Income from business
(b) Income from other sources
(c) Capital gains or
(d) All of the above
Q.4. A loss on the disposal of _______ is not recognized under the Income tax Ordinance, 2001.
(a) Securities not chargeable to tax
(b) Medallion
(c) Both (a) and (b)
(d) None of (a) to (c)
Q.5. Bonus shares are issued by a company to its ______ without receiving any amount from them.
(a) Employees
(b) Customer
(c) Shareholders
(d) All of the above
Q.6. Where a capital asset (other than securities and immoveable property) is disposed off after one year of its acquisition,
then gain for tax purposes is ______ of actual gain.
(a)
(b)
(c)
(d) None of (a) to (c)
Q.7. Income from Modaraba certificates is now taxable under section _________.
(a) 37
(b) 37A
(c) 38(5)
(d) none of (a) to (c)
Q.8. Income from shares of a public company set up in any special Industrial zone is exempt up to-------- years from the
date of commencement of commercial production.
(a) Three
(b) Four
(c) Five
(d) None of the above
Q.9. Gain from sale of shares of Private Limited companies is taxable under section_________.
(a) 37
(b) 37A
(c) Not taxable
(d) None of (a) to (c)

Conceptual Approach to Taxes 195


Capital Gains Chapter-12

Q.10. Bonus shares are the shares issued by a company_____________.


(a) Free of cost
(b) Issued at concessional rate
(c) On credit
(d) None of the above
Q.11. A deduction of loss u/s 37A on disposal of security chargeable to tax may be adjusted against the capital gain of
____________.
(a) Any other security not chargeable to tax
(b) Any other security chargeable to tax
(c) Shares of (Pvt.) Ltd. company
(d) None of the above
Q.12. A company in which at least 50% of the shares are held by a foreign government is ___________.
(a) Private company
(b) Public company
(c) Foreign company
(d) All of the above
Q.13. Any incidental expenditure on disposal of capital assets shall form part of _____________.
(a) Cost of assets
(b) Disposal consideration
(c) Selling cost
(d) None of a to c
Q.14. Capital loss u/s 37 is allowed as deduction for those assets the gain of which is___________.
(a) Chargeable to tax
(b) Exempt from tax
(c) Both of these
(d) None of (a) to (c)
Q.15. At the time of devolution ___________ would be the cost of the asset.
(a) FMV
(b) Historical cost
(c) higher of a and b
Q.16. Medallion received as gift and disposed off later, the capital loss on the same shall________.
(a) be recognized
(b) not be recognized
Q.17. Expenditure that shall form part of the cost of the asset are __________.
(a) Any expenditure to earn such income
(b) All the expenditure excluding expenditure inadmissible u/s 21 of the ITO, 2001.
(c) None of the above
Q.18. Tax on capital gain shall be charged on capital gain arising from debt on securities u/s 37A w. e. f.
(a) 01-07-2014
(b) 01-07-2013
(c) 01-07-2012
Q.19. Capital gain tax shall not be chargeable on disposal of securities which are held for a period of __________.
(a) one year
(b) two years
(c) three years
(d) six months
(e) forty eight months
Q.20. Derivatives is a general term for financial assets that are derived from other_______________.
(a) fixed assets
(b) current assets

196 Conceptual Approach to Taxes


Capital Gains Chapter-12

(c) financial assets


(d) income
Q.21. Any distribution received from NIT or a Mutual Fund of ICP out of the Capital Gains on which tax has already been
paid are___________________.
(a) Permanently exempt
(b) Fully Taxable
(c) Chargeable to tax
Q.22. Which one of the following is not a security _______________.
(a) share of a public company
(b) voucher of PTC
(c) Modaraba Certificates
(d) Lease hold right
Q.23 Gain on disposal of immovable property is chargeable to at _____%, where holding period is up to one year.
(a) 10
(b) 2
(c) 6
(d) 12
Q.24 Where a capital asset under section 37 is sold after one year, ________ of gain on such assets is exempt from tax.
(a) 50%
(b) 25%
(c) 75%
(d) 100%
Q.25 A loss on the sale of jewellery is _______ under the head capital gain.
(a) recognized
(b) not recognized
(c) taxable
(d) none of above
Q.26 Redeemable capital instruments are chargeable to tax under section _____.
(a) 37
(b) 38
(c) 37A
(d) 39
Q.27 In case capital assets are acquired through inheritance, the FMV on the date of ________ is treated as cost of
assets.
(a) disposal
(b) acquisition
(c) destruction
(d) all of above
Q.28 Profit on sale of personal car is taxable under the head _______.
(a) income from salary
(b) income from property
(c) income from business
(d) none of above
Q.29 Where a security has been disposed off within twelve (12) months from the date of its acquisition the rate of tax shall
be higher as compared to the rate applicable after _________months.
(a) 12
(b) 5
(c) 1
(d) none of above

Conceptual Approach to Taxes 197


Capital Gains Chapter-12

Q.30 Holding period of security and other capital assets _______ on the taxability of capital gain on their disposal.
(a) have no effect
(b) have effect
(c) none of above
Q.31 Rates for taxability of capital gains under section 37A for AOP and individuals are____.
(a) different
(b) equal
(c) none of above
Q.32 Capital gain is _______ where transfer of assets between spouses under an agreement to live apart, under a gift,
bequest or will, by succession, inheritance or devolution and distribution of assets on dissolution of an AOP or on
liquidation of a company.
(a) taxable
(b) exempt
(c) not recognized
(d) none of above
Q.33 The unadjusted capital loss under section 37 can only be carried forward upto the period of ___ years immediately
after the year of occurrence for adjustment against income from capital gain only.
(a) seven
(b) three
(c) six
(d) ten
Q.34 Loss on securities chargeable to tax can be carried forward to subsequent ___ tax years.
(a) seven
(b) three
(c) six
(d) none of the above
Q.35 When a capital asset is disposed of within one year, _____ of gain is taxable.
(a) 100%
(b) 75%
(c) 25%
(d) 0%
Q.36 Provisions of section 37A are not applicable to _________.
(a) an insurance company
(b) a banking company
(c) both a and b
(d) none of above
Q.37 An option to buy a __________ is a derivative.
(a) treasury bond
(b) shares
(c) debentures
(d) all of above
Q.38 Capital loss under section 37 may not be adjusted against the capital gain under section ______.
(a) 37
(b) 38
(c) 37A
(d) all of above
Q.39 _______ received as gift shall be chargeable to tax.
(a) medallion
(b) old coin
(c) painting
(d) none of above

198 Conceptual Approach to Taxes


Capital Gains Chapter-12

ANSWERS
1 (c) 2 (d) 3 (c) 4 (c) 5 (c)
6 (c) 7 (b) 8 (d) 9 (a) 10 (a)
11 (b) 12 (b) 13 (a) 14 (a) 15 (a)
16 (b) 17 (b) 18 (a) 19 (a) 20 (c)
21 (a) 22 (d) 23 (a) 24 (b) 25 (b)

26 (c) 27 (b) 28 (d) 29 (a) 30 (b)

31 (b) 32 (c) 33 (c) 34 (d) 35 (a)

36 (c) 37 (a) 38 (c) 39 (d)

Conceptual Approach to Taxes 199


Capital Gains Chapter-12

PRACTICE QUESTIONS WITH SOLUTIONS


Q # 1 An individual has disposed off his shares holding in different companies as per following details:
Cost of Acquisition
Company No of Shares Disposal Consideration Rupees Per Shares
Rupees Per Share
ABC Limited 10,000 35 55 (sold within 12 months)
XYZ Limited 5,000 40 30 (sold after 12 months but before 24 months)
MN (Private) Limited 15,000 100 140
HN (Private) Limited 20,000 50 45
PN (Private) Limited 10,000 50 75
Required: Compute the amount of capital gain for the tax year after considering the following information:
1. ABC Limited and XYZ Limited are listed companies.
2. Shares of MN (Private) Limited were disposed after two years of purchases.
3. Shares of PN (Private) Limited were kept by the person for 6 months.
Solution
Capital gain on shares of listed companies (Separate Block of Income):
Rs.
Gain on shares of ABC Limited (10,000 x (55 35)) 200,000
Loss on shares of XYZ Limited (5,000 x (30 40)) (50,000)
Capital gain 150,000
Tax on above capital gain @ 15% (Separate block of Income) 22,500
Capital gain on shares of (Private) limited companies:
Rs.
Gain on shares of MN (Private) Limited (15,000 x (140 100)) x 75% 450,000
Loss on shares of HN (Private) Limited (20,000 x (45 50)) (100,000)
Gain on shares of PN (Private) Limited (10,000 x (75 50)) 250,000
Capital gain 600,000
Q # 2 Mr. Z owns different assets the detail of these assets along with mode and value of acquisition and nature of
transaction is as under:
During the year, Mr. Arshad brother of Mr. Z gifted 5,000 shares of M/s ABC (Pvt.) Limited to Mr. Z. Mr. Arshad purchased
these shares at Rs.100 per share. Mr. Z sold these shares for a sum Rs.625,000 as on 15 June, 2016. The fair market value
at the date when gift received was Rs.135.
Mr. Z has also paid a sum of Rs. 40,000 for purchase of a one Kanal plot in EME Society Lahore. Mr. Z has fortunately
succeeded in balloting and has was provided the opportunity to pay the instalments for the allotment of plot. Mr. Z paid first
instalment of Rs.100,000 on January 2016. However, Mr. Z felt that he would not be able to pay the further instalments,
therefore he sold that plot entitlement to Mr. S for a sum of Rs. 250,000 on June 27, 2016.
Mr. Z has also 10,000 shares of XYZ Limited, a listed company, which were transferred to him through inheritance from
father. The face value of these shares is Rs.10 per share and his father was originally allotted these shares. Mr. Z sold 2,000
shares out of them at Rs.30,000 on January 30, 2016. The price ruling in the market on the date of sale was Rs.20 per
share.
Mr. Z also has a habit of collection of postage stamps. His collection includes 2,000 stamps countries and occasion. He
collected these stamps in many years. The cost of these stamps aggregates to Rs.275,000. However, due to paucity of
space in the home, he is not able to continue this habit therefore he sold these stamps for sum of Rs.500,000 in a stamp
exhibition.
You are required to compute taxable income of Mr. Z for tax year 2016.
Solution
Rs. Rs.
Shares of ABC (Pvt.) Limited:
Consideration received 625,000
Less: Fair market value on date of gift deemed as cost (5,000 x 135) 675,000
(A) (50,000)
Sale of rights in the plot
Consideration received 250,000
Less payments made 140,000
Taxable capital gain (B) 110,000

200 Conceptual Approach to Taxes


Capital Gains Chapter-12

Stamps:
Consideration received 500,000
Less: Cost 275,000
225,000
75% taxable (C) 168,750
Taxable capital gain (A+B+C) 228,750
Capital gain on shares of XYZ Limited (Separate Block of Income):
Consideration received 40,000
Higher of: Actual or fair market value (30,000 or 40,000)
Less: cost (10 x 2,000) 20,000
(Assumed that fair market value on date of transfer is Rs.10 per share)
20,000
Q # 3 Mr. Shahbaz a resident individual earned Rs 650,000 from the sale of assets as shown below:
PURCHASE SALE GAIN / (LOSS)
DATE PRICE (Rs) DATE PRICE (Rs) Rs
Shares of Listed Company 10-12-14 350,000 31-7-15 200,000 (150,000)
Shares of Unlisted Company 15-7-14 500,000 30-11-15 900,000 400,000
Jewellery 15-5-14 750,000 20-12-15 1,400,000 650,000
Sculpture 01-7-13 400,000 31-01-16 300,000 (100,000)
Shares of private company 01-01-14 1,300,000 15-02-16 1,200,000 (100,000)
Shares of Listed Company 31-12-14 250,000 30-6-16 200,000 (50,000)
Required: Discuss the treatment and implications of each of above transaction under the ITO, 2001.
Solution
Capital assets u/s 37A Rs.
Loss on shares of listed companies (150,000 + 50,000) (200,000)
Loss (200,000)
Capital loss u/s 37A can neither be adjusted against gain realized u/s 37 nor can it be carried forward.
Capital assets u/s 37
Gain on shares of unlisted company (400,000 x 75%) 300,000
(Holding period is more than one year)
Gain on jewellery (650,000 x 75%) 487,500
(Holding period is more than one year)
Loss on sculpture (Loss of this capital asset is not recognized) -
Loss on shares of private company (100,000)
Total gain u/s 37 687,500
Q # 4 Explain the following with reference to Income Tax Ordinance, 2001.
(i) Capital assets (ii) Valuation of Capital assets (iii) Capital gains (iv) Adjustment of capital loss against capital gains.
Solution
(i) Capital assets
There are two categories of capital assets: 1. Capital assets other than specified in section 37A, 2. Capital assets
specified in section 37A.
(a) Capital assets other than specified in section 37A.
Capital asset has been defined as property of any kind, connected with business or not, but does not include:
(i) Stock in trade, consumable stores or raw materials held for business
(ii) Depreciable asset or amortizable asset (i.e. fixed assets and intangibles for business use)
(iii) Immovable property
(iv) Movable property held for personal use of the person or any dependent family member excluding
capital assets mentioned in section 38(5)
(b) Capital assets specified in section 37A.
A separate section 37A has been introduced by the Finance Act 2010 to cater the disposal transactions of the
following securities:
(i) Shares of a public company;

Conceptual Approach to Taxes 201


Capital Gains Chapter-12

(ii) Vouchers of PTCL;


(iii) Modaraba certificates
(iv) An instrument of redeemable capital, debt securities as defined in the Companies Ordinance 1984; and
(v) Derivative products
(ii) Valuation of Capital assets: If a person receives a capital asset under Gift, Bequest, Will, Succession, Inheritance,
Devolution, Dissolution of AOP or Liquidation of Company, then fair market value on date of transfer shall be treated
as cost of the capital asset while in other cases value of capital asset shall be equal to its purchase price.
(iii) Capital gain: Capital gain is computed as consideration received less cost of the capital asset + expenses incurred
exclusively for earning capital gains (including incidental expenses for acquiring and disposing the capital asset).
Consideration received is the higher of actual amount received or fair market value. In case, an asset is lost or
destroyed, consideration received shall be scrap value along with any compensation, indemnity or damages received
under an insurance policy, agreement, settlement or judicial decision.
(iv) Adjustment of capital loss against capital gains: Capital loss can be c/f only against future capital gains up to 6
years next following the tax year in which the loss occurred.
Loss on disposal of securities u/s 37A shall be set off only against the gain from any other securities u/s 37A and any
unadjusted loss shall not be carried forward to the subsequent tax year.
Q#5 Briefly explain the income tax implications in respect of each of the following independent situations for the tax year
2016
(a) Mr. Mobeen has also paid a sum of Rs. 60,000 for purchase of dining table set on 15 January 2002 for his personal
use. He sold the said set to Mr Gufran for a sum of Rs. 90,000 on 27 June, 2016.

(b) 15 February 2016 Bilal discarded a machine which he had imported from China for Rs. 1,000,000 on 1 January
2016 to start the business. However, the machine was badly damaged during the shipment, rendering it unfit for
use. The shipping company paid him Rs. 850,000 as damages. The scrap value of the machine on the date it was
discarded was estimated to be Rs. 200,000. The documentation charges incurred in connection with the claim for
damages were Rs. 25,000
(c) On June 15, 2016 Imran sold his personal car for Rs. 1,500,000. The car has been originally purchased for Rs.
1,200,000 on September 13, 2013.
Solution:
(a) Sale of dinning table set

Although there is gain on sale of dinning table of Rs. 30,000. However any movable property for personal use,
except for painting, sculpture, drawing, jewellery, rare manuscript, folio, book, postage stamps, first day cover, coin,
medallion or an antique, is not chargeable to tax.
(b) Disposal of machine

Since Bilal was not entitled to claim depreciation on this machine, the machine falls within the definition of a capital
asset. [S.37(5)(b)] Discarding an asset is also treated as a disposal of the asset. [S.75(3A)] The capital gain is
determined as:
Rs.
Consideration received 15 February 2016
Damages from the shipping company 850,000
Scrap value of the machine 200,000
1,050,000
Cost of the machine on 1 January 2016
Purchase price of the machine 1,000,000
Documentation charges incurred 25,000
(1,025,000)
Capital gain 25,000
Since the disposal was made within one year of acquiring the asset, the full amount of capital gain is taxable. [S.37
(3)]

(c) Sale of personal car


A moveable asset in the personal use of the taxpayer is not a capital asset. Therefore, the gain is not taxable [Ref.
Sec 37(5)(d)]

202 Conceptual Approach to Taxes


Capital Gains Chapter-12

ICMAP PAST PAPERS THEORECTICAL QUESTIONS


Q. NO. 7 (b) WINTER 2005 State the five capital assets for which loss on disposal is not allowed to be recognized /
claimed under Income tax Ordinance, 2001.
Q. NO. 8 (a) WINTER 2004 Explain the provisions of Carry forward of capital losses under the Income tax Ordinance, 2001.

Conceptual Approach to Taxes 203


Capital Gains Chapter-12

CA MOD C PAST PAPERS THEORECTICAL QUESTIONS


Q.NO.3 Autumn 2014 Zaman is working as the Chief Executive Officer in Yasir Limited (YL). Following are the details of
sale and purchase relating to his capital assets during the tax year 2014.
(b) He sold 24,000 shares of HQ (Pvt.) Limited on 30 June 2014 for Rs. 200 per share.
He had acquired these shares as follows:
18,000 shares were purchased at Rs. 55 per share on 25 June 2013.
6,000 shares were allotted as bonus shares on 28 February 2014.
(c) A gain of Rs. 300,000 was realized on the sale of shares of Zeeshan Industries Limited (ZIL), a public listed
company, in June 2014. The shares were acquired on 31 May 2013.
(d) Zaman sold a painting to his brother on 23 March 2014 for Rs. 1,800,000. Zaman had purchased this painting
for his residence, in an auction for Rs. 2,000,000 on 10 July 2011.
(e) He sold his old furniture to Furqan for Rs. 285,000 on 25 June 2014. The furniture was purchased in 2012 for Rs.
250,000.
Required: Compute the amount to be included in the taxable income of Zaman for the tax year 2014 and specify the head
of income under which the income would be classified.

Q.NO. 6(a) Spring 2010 Explain the term Capital Assets as referred to in the Income tax Ordinance, 2001.
Q.NO. 6(a) Spring 2007 Under the Income tax Ordinance, 2001, a deduction for capital loss is allowed when consideration
received on disposal of a capital asset is less than its cost. What are the exceptions to this rule?
Q.NO. 4(b) Spring 2005 Discuss which assets are not considered capital assets for the purpose of determining income
under the head Capital Gains.
Q. NO. 4(b) April 2005 Discuss which assets are not considered capital assets for the purpose of determining income under
the head capital gains.
Q. NO. 2(i) March 2000 state the basis of taxation regarding capital gains.
Q. NO. 4(a) March 1999 Explain the procedure for computation of capital gain.
Q. NO. 4(b) April 1998 Explain the basis of chargeability under the head capital gains.

NOW SOLVE FOLLOWING NUMERICAL QUESTIONS OF MODULE C / AFC PAST PAPER RELATED TO THIS TOPIC

Q. NO. 3(III) & (IV) AUTUMN 2013


Q. NO. 5(A) AUTUMN 2012
Q. NO. 6(B) SPRING 2010
Q. NO. 3(A) AUTUMN 2007

204 Conceptual Approach to Taxes


Income From Other Sources Chapter-13

Chapter

13 INCOME FROM OTHER SOURCES

Topic covered Topic covered


Section (Part - I for CAF-6 and ICMAP (Part - II for CA Mod F and ICMAP
Section Rule
students) students)
Part - I Part - II
39 Income from other sources 89 Authors
Loan, advance, share deposit or gift
111 Un-explained income or assets
as other source of income
Profit on debt received as arrears 228 Valuation of assets
Non application of section 39 228 Donation in kind
Deductions under income from
40
other sources
Deductions not allowable
46 Profit on debt exempt from tax MCQs with solutions
Geographical source of income from
11,50,51 &101 Practice questions with solutions
other sources & their taxability
Exemption under 2nd schedule to ICMAP & CA Mod C past papers theoretical
Income Tax Ordinance, 2001 questions

PART I (For CAF-6 and ICMAP students)


1. Income from other sources: [U/s 39(1)]
Income of every kind received by a person in a tax year, if it is not included in any other head, other than
income exempt from tax, shall be chargeable to tax in that year under the head income from other sources including
the following namely:
(a) Dividend (covered under separate block of income)
(b) Royalty
(c) Profit on debt.
(d) Additional payment on delayed refund under any tax law
(e) Ground rent.
(f) Rent from sub-lease of land or a building.
(g) Income from the lease of any building together with plant or machinery.
(h) Income from provision of amenities, utilities or any other service connected with renting of building.
(i) Any annuity or pension received by a person not being as an employee.
(j) Any prize on prize bond, or winning from a raffle, lottery, prize on winning a quiz, price offered by companies
for promotion of sale or cross-word puzzle.
(k) Any other amount received as consideration for the provision, use or exploitation of property, including from
the grant of a right to explore for, or exploit, natural resources.
(l) The fair market value of any benefit, whether convertible to money or not, received in connection with the
provision, use or exploitation of property; and
(m) Income arising to the shareholder of a company from the issuance of bonus shares.

Conceptual Approach to Taxes 205


Income From Other Sources Chapter-13

(n) Any amount received by the person as consideration for vacating the possession of a building or part thereof.
It shall be taxable as follows: [U/s 39(1) and (2)]
Amount received for vacating the possession
Less: any amount paid on acquiring the possession
Taxable under the head other sources = Additional amount received 10
(o) Any amount received by a person from Approved Income Payment Plan or Approved Annuity Plan under
Voluntary Pension System Rules, 2005.
2. Loan, advance, share deposit money or gift to be treated as income from other sources [U/s 39(3) and (4)]
Amount of loan, advance, share deposit money or gift received by a person in a tax year from another person
(excluding receipt from a company or financial institution) otherwise than by a crossed cheque, shall be treated
as income from other sources for the tax year in which it was received.
This section shall not apply on the following cases:
(i) Where amount is received through banking channel from a person who holds a NTN; or
(ii) Advance against sale of goods or supply of services.

Example: Which of the following incomes are covered under the head income from other sources
(a) Dividend received by an individual.
(b) Salary received from employer.
(c) Profit on debt received by banking company.
(d) Rent against plot received as ground rent.
(e) Rent of building received from tenant.
(f) Rent from sub-lease of a property by tenant.
(g) Income from hire of factory (inclusive of plant and machinery).
(h) Amount received against sale of shares of a private limited company.
(i) Income from utilities connected with the rented premises.
(j) Received prize on prize bond.
(k) Amount of any unexplained income, investment or expenditure treated u/s 111.
(l) Any amount of loan received from friend through cash.
(m) Additional payment for delayed refund of income tax
Solution: Income from other sources: (a), (d), (f), (g), (i), (k), (l) and (m) Income from salary: (b) Income from
property: (e) Income from business: (c) Capital gain: (h) Final Tax Regime: (j)
3. Profit on debt of previous year or years received in current year [U/s 39(4A and (4B)]
(a) Where any profit on debt derived from investment in National Saving Deposit Certificates and Defence
Savings Certificate, by a person related to previous year or years in the current year that has resulted to
taxable at a higher rate of tax as compared to the year to which the profit on debt relates,
(b) The person may elect, by notice in writing to the Commissioner Inland Revenue that such profit is to be taxed
at the rate applicable to the year or years to which the profit on debt relates.
(c) An option shall be filed by the person at the date of filing of return for the tax year in the year of receipt or by
such later date as allowed by the Commissioner Inland Revenue.
Example: Mr. Zia received Profit on debt Rs. 600,000 in 2016 including profit on debt of Rs.100,000 related to tax
year 2015.
Required: Calculate the amount of tax under the Income Tax Ordinance, 2001 by assuming that the tax rate in tax
year 2015 was 8% and in tax year 2016 it is 10%.
Solution:
Option A (Total income charged to tax at tax rate of 2016)
2016
Rs.
Income from other sources 600,000
Tax on Rs. 600,000 @ 10% 60,000
Tax liability = Rs. 60,000

206 Conceptual Approach to Taxes


Income From Other Sources Chapter-13

Option B
2015 2016
Rs. Rs.
Income from other sources 500,000 100,000
Tax on income
2016: tax rate is 10% 50,000
2015: tax rate is 8% 8,000
Tax liability (50,000 + 8,000) = Rs. 58,000
Note: As tax liability under option B is less hence, the same is opted by the taxpayer.
Non application of section 39 U/S 39(5):
This section shall not apply to any income received by a person in a tax year that is chargeable to tax under any other
head of income or subject to tax under section 5 as dividends, u/s 6 tax on certain payments to non-residents and
under section 7 tax on shipping and air transport income of a non-resident.
4. Income from other sources [U/s 40]
Where a persons income is chargeable to tax under the head income from other sources, he shall be allowed to
deduct expenditure incurred on deriving such income. These expenditures are as under:
A deduction shall be allowed for any expenditure (except capital expenditure) paid by the person to the
extent to which the expenditure is paid in deriving income from other sources.
Zakat paid under the Zakat and Ushr Ordinance on profit on debt.
Deduction for depreciation of plant, machinery or building in case of lease income from letting out such building
together with plant and machinery.
An initial allowance for any plant or machinery.
5. Deductions not allowable [U/s 40(4), (5) and (6)]
No deduction shall be allowed to a person under this section to the extent that the expenditure is deductible in
computing the income of the person under another head of income.
The inadmissible expenses u/s 21 shall also be used under this head in the same manner as they apply in
determining the deductions allowed in computing the income under the head income from business.
Expenditure is of a capital nature if it has a normal useful life of more than one year
Example: Mr. Imran let out his building together with plant and machinery to Mr. Amir. He provided you the following
information for the calculation of taxable income and tax liability:
a. Rent received Rs. 600,000
b. Repair and maintenance of building Rs. 35,000
c. Insurance (already claimed under the head business income) Rs. 60,000
d. Salary to collect the rent (one month salary to employee to collect the rent paid in cash) Rs. 50,000
He also received Rs. 15,000 as additional payment on delayed refund from income tax department.
Solution:
Mr. Imran (Resident)
Tax year 2016
Computation of taxable income and tax liability:
Income from other sources: Rs.
Rent of building with plant and machinery 600,000
Less: Admissible deductions:
Repair and maintenance of building 35,000
Insurance (see note 1 below) -
Salary to collect rent (see note 2 below) -
35,000
565,000
Additional payment on delayed refund 15,000
Taxable income 580,000
Computation of tax liability:
Tax [Rs. 7,000 + 10% on (Rs.580,000 500,000)] 15,000

Conceptual Approach to Taxes 207


Income From Other Sources Chapter-13

Note 1 The insurance has already been claimed under the head business income so the same now shall not be allowed
under the head income from other sources.
Note 2 Salary to collect rent is inadmissible as salary more than Rs.15,000 per month should be paid through crossed
cheque.
Example: Mr. Hasnat Nadeem, an individual, furnishes the following particulars of his income for the year ended
30.06.2016.
Rs.
(a) Dividends from a company listed on stock exchange (gross amount) 15,000
(b) Interest on fixed deposit account (net amount after tax deduction) 45,000
(c) Taxable salary 135,000
(d) Prize on prize bond (gross amount before tax deduction) 20,000
(e) Income as shareholder from bonus issue (assumed market value) 100,000
Calculate total income of Hasnat Nadeem for the tax year 2016 and tax thereon.

Solution:
Mr. Hasnat Nadeem (Resident)
Tax year 2016
Computation of taxable income and tax liability:
Income from salary: Rs.

Taxable salary 135,000


Taxable income 135,000
Computation of tax liability:
Tax on Rs. 135,000 (taxable income is below taxable limit) nil
Incomes covered under Separate block of income:
Tax on gross dividend Rs.15,000 @ 12.5% 1,875
Tax on gross interest on fixed deposit @ 10% (45,000 x 100 / 90) 5,000
Tax on gross amount of prize on prize bond @ 15% 3,000
Tax on income as bonus issue Rs.100,000 @ 5% 5,000
Total tax liability 13,875
Less: tax deducted at source:
On dividend 1,875
On profit on debt 5,000
On prize bond 3,000
On bonus issue 5,000
13,875
Balance tax nil
6. Pakistan source & Foreign source Income from other sources
Geographical source
of Income from other sources Received by Taxability
[section 101]
Taxable [Section 11(5) and (6)]
Pakistan source income from Resident / non-resident Term and conditions of double taxation treaty
other sources individual agreements shall be considered for non
residents having Pakistan source income.
a. Resident Individual Taxable [Section 11(6)]
[Section 50]
b. Short term resident An individual shall be exempt in respect of his
foreign-source income which is not brought /
received in Pakistan if he is resident only by
[For all foreign source reason of his employment and he is present in
income] Pakistan for not exceeding 3 years.
Foreign source income from
other sources c. Returning expatriate [Section 51]
If an individual citizen of Pakistan (returning
[Citizen of Pakistan coming expatriate) is resident in the current tax year but
back in Pakistan] was non-resident in the 4 preceding tax years, his
foreign-source income shall be exempt in current
[For all foreign source tax year and in the following tax year.
income]
Non-resident individual Not taxable [Section 11(6)]

208 Conceptual Approach to Taxes


Income From Other Sources Chapter-13

7. EXEMPTIONS UNDER VARIOUS SECTIONS OF THE INCOME TAX ORDINANCE, 2001


PROFIT ON DEBT [U/S 46]
Any profit received by a non-resident person on a security issued by a resident person shall be exempt from tax
where-
(a) the persons are not associates; and
(b) the security approved by board was widely issued by the resident person outside Pakistan for raising loan
outside Pakistan (profit on security also payable outside Pakistan) for use in a business in Pakistan.
EXEMPTIONS UNDER SECOND SCHEDULE PART I TO INCOME TAX ORDINANCE, 2001

CLAUSE PARTICULARS
72 Interest to non-residents for approved projects
74 Interest on income of HUBCO
75 Interest income of foreign agencies
78 Interest on foreign currency accounts
79 Interest on rupee account of non-resident Pakistanis
80 Income from private foreign currency accounts and COI
90 Interest on approved foreign loans

EXEMPTIONS UNDER SECOND SCHEDULE PART IV TO INCOME TAX ORDINANCE, 2001

CLAUSE PARTICULARS
11B Inter-corporate dividend within the group
11C Inter-corporate profit on debt within the group

PART II (For CA Mod F and ICMAP students)


8. Authors [U/s 89]
Where the time taken by an author of a literary or artistic work to complete the work exceeds 24 months, the
author may elect to treat any lump sum amount received by the author in a tax year on account of royalties in
respect of the work as having been received in that tax year and the preceding two tax years in equal proportions.
Explanation: The aforesaid section saves authors or artistic work from a higher rate of tax for any amount received
for such work where the work takes more than 24 months to complete.
Example: Mr. Hakeem introduced his new book in a seminar conduct in University of the Punjab. He has received
the consideration of Rs. 300,000 against this book after the seminar. You are required to compute his taxable income
if he completes his work:
1. Within one year
2. Within 30 month period
3. Within 18 months.
4. Within 25 months.
5. within 4 years.
Solution:
1. 300,000
2. 100,000 (300,000 / 3) in current year and preceding two years
3. 300,000
4. 100,000 (300,000 /3 ) in current year and preceding two years
5. 100,000 (300,000 / 3) in current year and preceding two years
9. Unexplained income or assets [U/s 111]
1. Where
(a) any amount is credited in a person's books of account;
(b) a person has made any investment or is the owner of any money or valuable article;

Conceptual Approach to Taxes 209


Income From Other Sources Chapter-13

(c) a person has incurred any expenditure, or


(d) any person has concealed income or furnished inaccurate particulars of income including
(i) the suppression of any production, sales or any amount chargeable to tax; or
(ii) the suppression of any item of receipt liable to tax in whole or in part,
and the person offers no explanation about such items or the explanation offered by the person is not, in
the Commissioner Inland Revenue's opinion, satisfactory, the said amount shall be included in the
person's income chargeable to tax in the tax year to which such amount relates under the head "Income from
Other Sources" to the extent it is not adequately explained:
Provided that where a taxpayer explains to agricultural income as nature and source, such explanation shall be
accepted to the extent of agricultural income worked back on the basis of agricultural income tax paid under
the relevant provincial law.;
2. Where

The declared cost of any investment or valuable article or the declared amount of expenditure of a
person is less than reasonable amount of such items, the Commissioner Inland Revenue may include
the difference in the person's income chargeable to tax under the head "Income from Other Sources" in the
tax year to which such amount relates.
3. Non application of provisions

The aforesaid provisions do not apply to any amount of foreign exchange remitted from outside Pakistan
through normal banking channels that is en-cashed into rupees by a scheduled bank and a certificate from
such bank is produced to that effect.
4. The Board may make rules u/s 237 for the purposes of this section.
Example
Following is the computation of taxable income provided by Mr. A.
Rs.
Sales 400,000
Cost of sales 200,000
Gross profit 200,000
Other profit and loss expenses 100,000
Taxable income from business 100,000
Following further information is available:
Sales include Rs. 10,000 received from a person. There was no invoice against this amount and no
explanation was provided by Mr. Ali about this amount.
Sales value is understated by Rs. 100,000 in computation of taxable income.
He purchased a car for personal use and declared the value of this car in his wealth statement to be
Rs.200,000. The reasonable value of car determined by the Commissioner is Rs. 500,000.
Required: Compute correct taxable income and tax liability of the taxpayer for the tax year 2016.
Solution:
Mr. Ali
Computation of taxable income tax liability: Rs.

Income from business


Incorrect taxable income (given above) 100,000
Add:
Un-declared sales 100,000
Less:
Unexplained income (10,000)
Correct taxable income from business 190,000
Income from other sources
Unexplained income 10,000
Difference between the reasonable and declared values of the car 300,000
310,000
Total taxable income 500,000
Computation of tax liability
Tax on Rs. 500,000 [0 + 7% x (500,000 - 400,000)] 7,000

210 Conceptual Approach to Taxes


Income From Other Sources Chapter-13

10. Valuation of assets [Rule 228(1), (2) & (3)]


The valuation of immovable property U/R 228(1) for the purposes of section 111 shall be taken to be-

( A) IN THE CASE OF OPEN the value determined by the development authority or government agency on the
PLOT basis of the auction price in respect of similar plots in the area where the plot in
question is situated or in case where such value is not determined, the value fixed
by the District Revenue Officer or provincial authority authorized in this behalf for
the purposes of stamp duty;

( B)
IN THE CASE OF the value equal to the average sale price of the sales recorded in the revenue record
AGRICULTUR AL LAND of the estate in which the land is situated for the relevant period or time; or

( C) IN THE CASE OF
CONSTRUCTED
value shall be determined at higher of FMV or the value fixed by the District Revenue
IMMOVABLE PROPERTY
Officer.

The valuation of motor cars & jeeps U/R 228(2) & (3) for the purposes of section 111 & 61 shall be taken to be-

M OTOR CARS & JEEPS (i) Imported new motor cars or jeeps: CIF value to donor (i.e. Cost, Insurance &
U/R 228(2) Freight) + Duties and charges till their registration.
(ii) Locally newly purchased motor cars or jeeps: Price paid by the donor +
Duties & charges till their registration.
(iii) Import of used motor cars or jeeps: Value at the import price adopted by the
Custom Authorities + Charges & duties till their registration.
(iv) Value adopted in the first year shall be reduced by 10% of the said value (i.e. on
straight line basis) for each successive year up to a maximum of 5 years.
(v) Used motor cars or jeeps locally purchased shall be valued as:
If motor cars or jeeps are up to 5 years old, value shall be original cost as
reduced by 10% for every year following the year in which it was imported or
purchased.
If motor cars or jeeps are more than 5 years old, value shall be higher of
purchase price paid by the donor for the used car or 50% of the original
value.

M OTOR CARS & JEEPS In no case shall the value be determined at an amount less than 50% of the value
U/R 228(3) determined in accordance with (i), (ii) & (iii) or the purchase price, whichever is higher.

11. Donation in kind [rule 228(4)]


The value of items donated in kind to non profit organizations for tax credit under section 61 summary is as under:

SR. H EADINGS E XPLANATION


1. I MPORTED ITEMS Value assessed for custom duty + all duties & charges paid by the donor

2. I TEMS MANUFACTURED IN P AKISTAN Purchase price + Duties & charges paid by the donor

3. U SED DEPRECIABLE ITEMS Tax WDV i.e. Cost tax depreciation

4. MOTOR CARS & JEEPS As given above U/R 228(2) & (3).

5. O THER ITEMS FMV as determined by the Commissioner Inland Revenue

Conceptual Approach to Taxes 211


Income From Other Sources Chapter-13

MULTIPLE CHOICE QUESTIONS


Q.1. Income from dividend received by a salaried person is taxed under _______.
(a) SBI under FTR
(b) FTR
(c) NTR
(d) both a and b
Q.2. Prize on winning a quiz is taxable under the _______.
(a) FTR
(b) NTR
(c) SBI under FTR
(d) all of above
Q.3. Examination fee and paper checking fee received by Government employees is taxable under _____.
(a) Income from salary
(b) income from other sources
(c) None of a and b
Q.4. __________ of the amount received by the tenant for vacating the possession of any building, less amount paid for
taking the possession is chargeable to tax in the year of receipt under the head income from property.
(a) 1/10th
(b) 1/5th
(c) 1/3rd
(d) None of above
Q.5. Amount received by tenant to vacate the possession of property is chargeable to tax in _________years in unequal
proportions.
(a) 5
(b) 10
(c) 20
(d) none of the above
Q.6. An amount equal to ______is allowed as deduction for expenditure incurred exclusively for deriving income under
income from other sources.
(a) Calculated on proportionate basis
(b) calculated at an average rate
(c) whole amount of expenditure
(d) None of the above
Q.7. Dividend income and profit on PLS account received by an individual or AOP although covered under the head
income from other sources but the same is chargeable to tax under ______.
(a) SBI under FTR
(b) NTR
(c) FTR
(d) SBI under NTR
Q.8. ______ is treated as full and final discharge of tax liability of the recipient receiving amount under FTR.
(a) Tax paid with the return
(b) tax deductible
(c) tax required to be collected
(d) both b and c
Q.9. Profit on special US Dollar Bonds on such deposited as were before 16-12-1999 is ________.
(a) Fully exempt
(b) Exempt on proportionate basis

212 Conceptual Approach to Taxes


Income From Other Sources Chapter-13

(c) Not exempt


(d) none of a to c
Q.10. Sales of irrigation water by an agriculturalist is _________.
(a) Exempted income
(b) taxable under income from property
(c) included in total income for rate purposes
(d) taxable under income from other sources
Q.11. Any expenditure incurred for earning of income from other sources ______.
(a) Admissible
(b) inadmissible
(c) admissible if according to the provisions of ITO, 2001.
Q.12. In case of profit on debt received in arrears and person wants to apply the tax rate that would have been applicable if
it is received in the relevant tax year then it should be received from the______ to exercise such option.
(a) National Saving Centre Scheme
(b) Defence saving certificates
(c) Both (a) and) (b)
Q.13 All the expenditure to earn any __________is allowed as deductions except capital expenditure.
(a) income from salary
(b) income from property
(c) dividend income
(d) income from other sources
Q.14 Ground rent is chargeable to tax under income from _____.
(a) property
(b) business
(c) salary
(d) other sources
Q.15 Gratuity received by the legal representatives after the death of employee will be included under head ________ of
deceased person.
(a) income from salary
(b) income from other sources
(c) income from business
(d) income from property
Q.16 Any income that is not covered under any other head of income is covered under the head ____________.
(a) Income from salary
(b) income from business
(c) Income from other sources
Q.17 Income from other sources earned by an AOP is ___________.
(a) taxable
(b) exempt
(c) allowed as deduction against other incomes
(d) all of above
Q.18 Profit on debt may be treated as _____________.
(a) Income from salary
(b) business income
(c) Income from other sources
(d) both b or c

Conceptual Approach to Taxes 213


Income From Other Sources Chapter-13

Q.19 Income from school is chargeable to tax under head _______________.


(a) Income from salary
(b) income from business
(c) Income from other sources
(d) both b or c
Q.20 Profit on debt received in ______ can be adjusted in the respective tax year by exercising option in writing.
(a) arrears
(b) advance
(c) none of above
Q.21 Unexplained income is chargeable to tax under ______.
(a) income from salary
(b) business income
(c) income from other sources
(d) income from property
Q.22 Additional payment on delayed refund is __________.
(a) exempt
(b) taxable as business income
(c) taxable under income from other sources
(d) taxable under income from salary
Q.23 Bonus issue received by a shareholder is taxable as income from other sources under ________.
(a) Normal tax regime
(b) Final tax regime
(c) Exempt from tax
Q.24 Income of an author is chargeable to tax under the head _____.
(a) income from salary
(b) business income
(c) income from other sources
(d) income from property
Q.25 The provisions of inadmissible expenses u/s 21 are ___ on the deductions claimed against income from other
sources.
(a) applicable
(b) not applicable
(c) none of a or b
Q.26 Where net amount received by the recipient for income covered under FTR the ____ amount received will be offered
for tax under the ITO, 2001.
(a) gross
(b) net
(c) none of above

ANSWERS
1 (a) 2 (c) 3 (b) 4 (d) 5 (d)
6 (c) 7 (a) 8 (d) 9 (c) 10 (d)
11 (c) 12 (c) 13 (d) 14 (d) 15 (b)
16 (c) 17 (a) 18 (d) 19 (b) 20 (a)
21 (c) 22 (c) 23 (a) 24 (c) 25 (a)
26 (a)

214 Conceptual Approach to Taxes


Income From Other Sources Chapter-13

PRACTICE QUESTIONS WITH SOLUTIONS


Q#1 Mr. C incurred following transactions during the year:
Mr. C has agricultural land in Lahore. Mr. C received Rs.100,000 as agricultural income under the tenancy agreement.
Mr. C is also an irregular writer in the Daily Jang. He received an aggregate sum of Rs. 65,000 on this literary work from the
said newspaper. Mr. C incurred expenses of Rs.15,000 on stationery and Rs.4,500 for the courier's service to send these
articles.
Mr. C received a loan of Rs.600,000 from his sister through bearer cheque. His sister is a housewife and she is not an
income tax payer.
You are required to compute the taxable income of Mr. C for the year 2016.
Solution:
Agricultural income exempt u/s 41 (Subject to payment of tax to
Provincial Govt. where applicable) -
Amount received from newspaper 65,000
Add: loan from sister received through bearer cheque 600,000
Less: expenses incurred on
Stationery (15,000)
Courier services (4,500)
Taxable income 645,500
Q#2 X, an individual, furnished the following particulars of his income for the year ended 30.06.2016.

(a) Dividends from a company listed on stock exchange Rs.20,000


(b) Interest on fixed deposit account 65,000
(c) Salary income 250,000
Calculate total income of X for the relevant tax year and tax thereon.
Solution:
(A). Dividend received by a taxpayer from a company shall be treated as a separate block of income and charged to tax at
the rate of 12.5%.
(B) Profit on fixed deposit received by a taxpayer from any person shall be treated as a separate block of income and
charged to tax at the rate of 10%.
(C) Salary income is taxable, but in this case as the income is below the threshold so no treatment is required for this.

Conceptual Approach to Taxes 215


Income From Other Sources Chapter-13

ICMAP PAST PAPERS THEORECTICAL QUESTIONS


Q. No. 3(a) February 2013 Write short answers of the following questions:
(v) Under what head of income, the income from rent received or receivable in respect of the lease of building with
plant and machinery is taxable?
Q. NO, 2 (b) (ii) SUMMER 2011 List down any six sources of income under the head "Income from other sources" as per
section 39 of the Income tax Ordinance, 2001.
Q. NO. 3 (a) WINTER 2007 What are the provisions for exemption from tax on profit on debt received by a non-resident
person u/s 46 of the Income tax Ordinance, 2001?
Q. NO. 6 (c) SUMMER 2005 Explain the term Profit on debt as described in Income tax Ordinance, 2001.

216 Conceptual Approach to Taxes


Income From Other Sources Chapter-13

CA MOD C PAST PAPERS THEORECTICAL QUESTIONS


Q.2 (a) March 2008 Mr. Anil is constructing his house and for the purpose of meeting construction expenses, he intends to
take a personal loan of Rs.500,000 from Mr. Kamran who is in the business of money lending. He has been advised by one
of his friends that such a loan may be included in his taxable income, under certain circumstances.
You are required to advise Mr. Anil about the circumstances under which the loan may be included in his taxable income.
Q.1 Sept 1999
(i) Which types of income are included under income from other sources?
(ii) What are allowable deductions from income from other sources?
Q.4 (ii) Sept 1999 Mr. Fawad got possession of a shop on 21.7.1997 by paying Rs. 100,000 as pugree to the outgoing
tenant. On 26.5.1998 he vacated the possession of the said shop and received Rs. 650,000 as consideration for vacating
the possession. What will be tax treatment of this amount in his income for the tax year 1998.
Q.2 Sept 1998 Give examples of income from other sources.
Q.4 Sept 1998 Mr. XYZ / MD of a public company attended a seminar on the subject of taxation. One of the speakers of the
seminar in his speech said that it is necessary to determine (a) tax status of the taxpayer to calculate the correct taxable
income and tax liability (b) he also said that expenditure incurred by a taxpayer may be of two types i.e. capital expenditure
or revenue expenditure and (c) he further said that the income tax Statute while taxing the income also provides relief from
taxes as well under the provisions section 53 of the Income Tax ordinance.
The MD has asked you as a Chief Accountant to explain to him in a write-up:
(i) How does tax liability differ according to the tax status of the person?
(ii) Does the tax treatment differ in respect of capital and revenue expenditure? If so explain with examples.
(iii) How and to what exemptions from tax are provided u/s 53 (now 147 under Income tax Ordinance, 2001) of the
Income Tax Ordinance, 1979.
Q.9 April 1995
i. Indicate which of the following income is exempt whether fully or partly:
ii. Interest on foreign currency accounts
iii. Income of professional sportsmen from benefit matches
ix. Dividend income from listed companies

Conceptual Approach to Taxes 217


Income From Other Sources Chapter-13

218 Conceptual Approach to Taxes


Losses Chapter-14

Chapter

14 LOSSES

Topic covered Topic covered


Section
Section Rule
Chart of set off and carry forward of
104 Foreign losses
losses
Chart of losses order of set off
56 Set off of losses 59AA 231D Group taxation
Set off of losses of companies operating
56A 59B Group relief
hotels
57 Carry forward of business loss 98 Change in control of an entity
Set off of business loss consequent to
57A Practice questions with solutions
amalgamation
Carry forward of speculation business &
58 & 59 MCQS with solutions
capital losses
Limitations on set off & carry forward of ICMAP & CA Mod C past papers theoretical
59A
losses questions
The present status of set off of and carry forward of losses under the five heads of income under the income tax
Ordinance, 2001 are as under:
Chart of set off and carry forward of losses:

SR. HEAD OF R ULES FOR SET OFF RULES FOR CARRY FORWARD
I NCOME
1. I NCOME FROM a. There can be no loss under this head as a As there can be no loss under this head
SAL ARY person deriving salary income is not allowed hence the question of carry forward is not
to deduct expenses incurred in deriving applicable.
salary income [u/s 12(4)].

b. Salary income cannot be used to set off the


loss under any other head of income.
2. I NCOME FROM a. There can be loss under this head of income Unadjusted loss under this head cannot be
PROPERTY as it is chargeable to tax under normal tax carried forward.
regime and expenses can be deducted
against property income u/s 15A.

b. Loss under this head can be set off against


any other head of income chargeable to
under NTR except salary income.

c. Income from property cannot be used to set


off the loss under any other head of income.
3. I NCOME FROM Loss can be set off against any other head of Unadjusted business loss can be carry
BUSINESS income chargeable to tax under NTR except forward for adjustment only against profit
NON salary and property incomes. However business under same head, upto six (6) succeeding
SPECUL ATION loss shall be set off last. tax years following the year in which loss
BUSINESS incurred.
4. I NCOME FROM Loss from speculation business can be set off Unadjusted speculation loss can be carry
BUSINESS only against the income of any other speculation forward for adjustment only against profit
SPECUL ATION business. under same head, upto six (6) succeeding
BUSINESS tax years following the year in which loss
incurred.

Conceptual Approach to Taxes 219


Losses Chapter-14

5( A) C APITAL GAINS Capital loss cannot be set off against any other Unadjusted capital loss can be carry forward
EXCEPT 5( B), head of income except against capital gain. for adjustment only against gain under same
5( C) & 5( D) AS head, upto six (6) succeeding tax years
UNDER following the year in which loss incurred.
5( B) C APITAL GAINS The loss on disposal of securities may be set off Unadjusted loss under this head cannot be
ON only against any other security chargeable to tax carried forward.
SECURITIES U / S under this section.
37A
5( C) C APITAL GAINS Loss on disposal of immoveable property cannot Loss under this head cannot be carried
IMMOVEABLE be set off. forward.
PROPERTY U / S
37(1A)
5( D) C APITAL GAINS Loss on disposal of moveable personal assets As loss under this section shall not be
ON shall not be recognised under the said section. recognized therefore the question of carry
PERSONAL forward does not arise.
MOVEABLE
ASSETS
SPECIFIED U/ S
38(5)
6. I NCOME FROM Loss can be set off against any other head of Loss under this head cannot be carried
OTHER income chargeable to tax under NTR except forward to succeeding tax years for
SOURCES salary and property incomes. adjustment.
7. FOREIGN Foreign loss can be set off only against the Unadjusted foreign loss can be carry forward
LOSSES income under the same head of income only. for adjustment only against profit under same
head, upto six (6) succeeding tax years
following the year in which loss incurred.

Losses order of set off:

1 Firstly compute income chargeable to tax under (normal tax regime) each head of income even where there is more
than one source under the same head of income.

2 Secondly current year losses under normal tax regime (as stated above in first chart) shall be set off against current
years income but the loss under the head business income under normal tax regime shall be set off last i.e. after set
off of other heads of income (as stated above in first chart).

3 Thirdly set off the previous years business losses (excluding tax depreciation brought forward) against current
years business income (excluding tax depreciation for the year) under normal tax regime, if any.

4 Finally set off the current year depreciation and previous years unabsorbed depreciation, amortization and initial
allowance against current years business income under normal tax regime, if any.

Important aspects:
1. The aforesaid Rules regarding set-off and carry forwarded of losses apply only to such incomes which are
taxable under Normal Tax Regime (NTR). Any income that falls under Final tax regime (FTR) or separate
block of income cannot be used for adjustment of losses under NTR.
2. The law does not permit any deduction while computing income under FTR or Separate block of income.
Hence, there cannot be loss under these streams of taxation.
3. Losses can be set-off only as provided above. Any loss that cannot be set off under the rules (e.g. loss on
disposal of personal assets, etc.) cannot be carried forward. [u/s 56(2)]
4. Where a person sustains losses under different heads of income, including Income from Business the
business loss shall be set off last, i.e. after setting off all other losses. [u/s 56(3)]
5. The depreciation allowance admissible under the Third Schedule shall be charged upto that portion only
which can be absorbed by the incomes. The general rules relating to set-off and carry forward of losses shall
not apply to unabsorbed depreciation. Amount of such depreciation shall not be taken as a normal business
loss rather, shall be treated separately.
6. If an income from a source is permanently exempt from tax, the loss, if any, from such sources cannot be set-
off or carried forward.

220 Conceptual Approach to Taxes


Losses Chapter-14

7. The Finance Act, 2003 has omitted the following sub section (2) of section 55;
Where a persons income from business is exempt from tax under this Ordinance as a
result of a tax concession, any loss sustained in the period of the exemption shall not
be set off against the persons income chargeable to tax after the exemption expires.

After the above omission the business loss sustained during the exempt period has, therefore, been made
adjustable against income of the postexemption period of industrial undertakings. The limitation period for
carry forward of loss for six (6) years as contained in section 57 would continue to be applicable.
Depreciation:
Where the business loss includes depreciation and amortization that has not been set off against income, the amount not set
off shall be allowed as deductions in the following tax years until completely set off.
1. Set off of losses (Section 56)
Except speculation loss and capital loss (including loss on securities u/s 37A) where a person sustains a loss for any
tax year under any other heads of income the person shall be entitled to set off the loss against the taxable income
under any other head of income except salary and property incomes for the year.
Where a person sustains a loss under a head of income for a tax year that cannot be set off the person shall not be
permitted to carry forward such loss to the next tax year.
Where, in a tax year, a person sustains a business loss and a loss under another head of income, the loss under the
head "Income from Business shall be set off last.
2. Set-off of losses of companies operating hotels [Section (56A)]
Where a company registered in Pakistan or Azad Jammu and Kashmir (A J and K), operating hotels in Pakistan or A
J and K, sustains a business loss in Pakistan or A J and K for any tax year shall be entitled to have the amount of the
loss to set off against the company's income in Pakistan or A J and K.
Example: A company is operating one hotel in Pakistan and two in Azad Jammu and Kashmir. The turnover and
profit and losses of respective hotels as per tax are as under:
Hotel (Pakistan) Hotel 1 (AJandK) Hotel 2(AJandK)
Turnover 2,000,000 800,000 500,000
Taxable income 400,000 - 100,000
Loss as per tax - 200,000 -
Required: Compute the tax liability of the company.
Solution:

Computation of income:

Taxable income of hotel in Pakistan 400,000


Add: Taxable income of hotel 2 in AJ and K 100,000
Less: Loss of hotel 1 in AJ and K (200,000)
Total taxable income 300,000

Computation of tax liability:


Higher of:
Alternative Corporate tax i. e. accounting profit (Rs. 400,000 x 17%) 68,000
OR
Corporate tax u/s 113C:

Tax on Rs. 300,000 @ 32% 96,000


OR
Tax on total turnover Rs. 3,300,000 @ 1% 33,000
As the tax payable under Corporate tax u/s 113C is higher hence than tax under
Alternative Corporate tax u/s 113C hence the same is to be paid by company 96,000
3. Carry forward of business losses (Section 57)
Where a person sustains a business loss for a tax year other than speculation loss and the loss cannot be wholly set
off, so much of the loss that has not been set off shall be carried forward to the following tax year and adjusted only
against the person's taxable business income for that year.

Conceptual Approach to Taxes 221


Losses Chapter-14

If a business loss sustained by a person for a tax year is not wholly set off then the amount of loss not set off shall be
carried forward to the following tax year and shall be carried forward for not more than six immediately succeeding tax
years.
Where a person has a business loss carried forward for more than one tax year, the loss of the earliest tax year shall
be set off first.
Where the business loss includes depreciation and amortization, that has not been set off against income, the
amount not set off shall be allowed as deductions in the following tax year until completely set off.
Example: Mr. Hanif has provided you the following information and asked you to compute his tax liability for tax year
2016.
Rs.
Income from non-speculation business 1 360,000
Loss from non-speculation business 2 (50,000)
Loss from speculation business 3 (50,000)
Solution:
Mr. Hanif
Computation of taxable income and tax liability: Rs.

Income from business:


Income from business 1 360,000
Loss from business 2 (50,000)
Taxable income 310,000
Computation of tax liability:
Tax on Rs. 310,000 x 0% Nil
Note: Loss from speculation business cannot be set off against any other head of income. Therefore, loss from
speculation business shall be carried forward for adjustment against speculation gains of next six succeeding tax
years.
Example: Following is the profit and loss account of Zia Enterprises for year 1. Compute losses to be carried forward.
Rs.
Sales 240,000
Cost of sales 140,000
Gross profit 100,000
Administrative and selling expenses 120,000
Loss (20,000)
Note: Administrative expenses include depreciation of Rs.10,000. Tax depreciation is Rs. 30,000.
Solution:
Zia Enterprises
Computation of taxable income and tax liability: Year 1
Rs.
Loss before tax (20,000)
Add: accounting depreciation 10,000
Business loss before tax depreciation (10,000)
(1) (As there is loss before tax depreciation, tax depreciation shall not be deducted from this loss. This loss of
Rs.10,000 shall be carried forward for next six years for adjustments against income under the head income
from business only. Whereas unabsorbed tax depreciation of Rs. 30,000 shall be carried forward for indefinite
period and shall also be set off at last.)
(2) Where a person has a loss carried forward for more than one tax year and depreciation allowance, the loss of
the earliest tax year shall be set off first and depreciation shall be taken into account last [Section 57(3)].
Carry forward of losses of export procession zone units [u/c 2 Part IV, second schedule]
In the case of losses referred to in section 57 in respect of an industrial undertaking set up in an area declared by the
Federal Government to be a "Zone" within the meaning of Export Processing Zones Authority Ordinance, 1980, the
period of 6 tax years specified in the said section shall not apply-

222 Conceptual Approach to Taxes


Losses Chapter-14

Example: P and L account for the year 1 of Ali Enterprises (industrial undertaking in Export Processing Zone) is
given below:
Rs.
Sales 300,000
Cost of sales 160,000
Gross profit 140,000
Administrative and selling expenses 160,000
Loss (20,000)
Note: Accounting depreciation 10,000
Tax depreciation 20,000
Solution:
Zia Enterprises
Computation of taxable income and tax liability: Year 1
Rs.
Accounting loss (20,000)
Add: accounting depreciation 10,000
Business loss (10,000)
Unabsorbed depreciation (20,000)
Both business loss assessed in the current year Rs. 10,000 and unabsorbed depreciation Rs. 20,000 shall be carried
forward for unlimited time for an industrial undertaking in Export Processing Zone.
4. Set-off of business loss consequent to amalgamation [Section (57A)]
Both the amalgamating and amalgamated companies can set off and carry forward their losses (excluding brought
forward and capital loss) against each other for a period of 6 years if, the amalgamated company continues the
business of amalgamating company for 5 years from the date of amalgamation and the scheme is approved by State
Bank of Pakistan, Securities and Exchange Commission of Pakistan or any court.
In case of amalgamation of banking company, non-banking company, non-banking finance company, modaraba or
insurance company, the accumulated loss (excluding speculation losses) of an amalgamating company or companies
shall be set off or carried forward against business income of the amalgamated company or vice versa for 6 years
provided the scheme is approved by State Bank of Pakistan, Securities and Exchange Commission of Pakistan or
any court.
The meaning of the amalgamation has been extended to include the companies engaged in providing services other
than trading companies.

Example: A Ltd. (amalgamating company) and B Ltd. (amalgamated company) amalgamated in tax year 2011. Detail
of losses of B Ltd. in tax year 2016 is given below:
Rs.
Brought forward losses before tax year 2011 10,000
Capital loss for tax year 2011 2,000
Un- adjusted business loss for tax year 2011 100,000
There was no loss in tax year 2012 to 2015. Following is the information relating amalgamated company for tax year
2015.
Income before setting of losses 400,000
Required: Compute the tax payable by the amalgamated company, rate of tax for companies is 32%.
Solution:
Rs.
Income before setting off of losses 400,000
Less:
Un-adjusted business loss of B Ltd. of tax year 2011 100,000
Taxable income 300,000

Tax payable @ 32% 96,000

Important Note: This example has been solved after taking clause 19 of Part-II of Second Schedule to the Income Tax
Ordinance, 2001. Hence if both the companies are small companies then benefit for the next two years may be availed
under clause 19 that is of 25% rate of tax.

Conceptual Approach to Taxes 223


Losses Chapter-14

5. Carry forward of speculation business and capital losses [Section 58 and 59]
Where a person sustains a speculation business loss for a tax year carried on by the person, the loss shall be set off
only against the income of the person from any other speculation business of the person chargeable to tax for that
year.
Where a person sustains a capital loss for a tax year, the loss shall be set off only against the income of the person
from any other capital gain of the person chargeable to tax for that year.
Example: In the light of the following facts for tax year 2016, work out total income of Mr. Tanveer Butt after making
inter-head adjustment of losses and incomes:
Rs.
1. Income from business 300,000
2. Loss from speculation business (200,000)
3. Loss from Capital Gains (70,000)
4. Income from Salary 100,000
5. Loss from 'Income from Other Sources' (50,000)
Solution: Rs.
Income from business 300,000
Loss from ' Income from other sources' (50,000)
Taxable Income for the year (without salary income) 250,000
Add salary income (separately added as no loss can
be adjusted against salary income) 100,000
Total taxable income 350,000
Notes:
N-1 Loss on 'Income from capital gains' cannot be set off other than Income from Capital Gains' so it will be carried
forward up to six succeeding tax years.
N-2 Loss on 'Income from Speculation Business' cannot be set off other than Income from Speculation Business
so it will also be carried forward up to six succeeding tax years .
6. Limitations on set off and carry forward of losses [Section 59(A)]
An AOP can set off and carry forward its losses according to the general provisions regarding set off and carry
forward of losses. As the share received by a member out of the incomes of AOP is exempt from tax, the member is
not allowed to set off and carry forward his respective share in losses of the association of person.
7. Foreign losses [Section 104]

Where a resident person sustains a foreign loss (i.e. loss from a foreign-source income) he may set it off
against his income under the same head of income. If the foreign loss is not fully set-off then the loss shall be
carried forward maximum upto six (6) tax years immediately succeeding the tax year during which the loss
occurred.
Important notes
(a) All foreign losses can be set-off and carried forward upto six (6) tax years.
(b) The foreign losses are adjusted against the same head of income. For this purpose, the speculation business
is treated as a separate head of income.
(c) Where more than one tax years losses are simultaneously carried forward, the loss of earliest year shall be
set-off first.
Example: Mr. Zahid suffered loss of Rs. 50,000 (converted in Pakistani currency) in business in Dubai in tax year
2015. His taxable income from his Dubai Business in tax year 2016 was Rs. 600,000.
Required: Compute tax payable by Mr. Zahid for tax year 2016.
Solution:
Mr. Zahid
Tax year 2016
Computation of taxable income and tax liability:
Rs.
Income from business:
Income from foreign business 600,000
Less: brought forward loss (50,000)

224 Conceptual Approach to Taxes


Losses Chapter-14

Taxable income 550,000


Computation of tax liability:
Tax on Rs. 550,000 [7,000 + 10% x (550,000 500,000)] 12,000

8. Group taxation [Section 59AA]


(1) Holding Companys and Subsidiary Companys of 100% owned group may opt to be taxed as one fiscal unit.
In such cases, besides consolidated group accounts as required under the Companies Ordinance, 1984
computation of income and tax payable shall be made for tax purposes.
(2) The companies in the group shall give irrevocable option for taxation under this section as one fiscal unit
(3) The group taxation shall be restricted to companies locally incorporated under the Companies Ordinance,
1984.
(4) The relief under group taxation would not be available to losses prior to the formation of the group.
(5) The option of group taxation shall be available to those group companies which comply with such corporate
governance requirements and group designation rules or regulations as may be specified by the Securities
and Exchange Commission of Pakistan from time and are designated as companies entitled to avail group
taxation.
(6) Procedure for group taxation RULE 231D
(A) A 100% owned company shall be a Subsidiary Company where another company, owns all equity
shares of such company except those held by nominees to meet the statutory requirements of the
Companies Ordinance, 1984.
(B) A Holding Company and each of its Subsidiary Companies of 100% owned group which fulfil the
conditions of section 59AA shall make separate application containing declaration of irrevocable option
for group taxation as one fiscal unit to the concerned Commissioner Inland Revenue in prescribed form
within the first quarter of the tax year for which group taxation is opted for.
(C) The application shall be signed by the respective Chief Executive Officer of the Holding Company and
Subsidiary Company, identifying the Commissioner Inland Revenue having jurisdiction over the Holding
Company or as the case may be, Subsidiary Company, NTN and Corporate Registration Number.
(D) The Holding Company as well as each Subsidiary Company shall furnish a certificate issued by the
Securities and Exchange Commission of Pakistan verifying that the company has been complying with
the Code of Corporate Governance as notified from time to time by the Securities and Exchange
Commission of Pakistan.
(E) The return for the tax year following the option for group taxation shall be prepared as one fiscal unit
under the name of the Holding Company and the tax liability shall be discharged or the refund shall be
claimed respectively as if the business of the Subsidiary Companies were the business of the Holding
Company. However, no effect shall be taken for losses and unabsorbed depreciation of Subsidiary
Companies for the tax year prior to the exercise of such option. Along with the group return, copies of
audited accounts of every company in the group shall be attached.
(F) On option for group taxation u/s 59AA(2), the Subsidiary Companys shall furnish their returns of income
in their respective tax jurisdiction along with a copy of application for group taxation for record and
future adjustments and intimating non-taxability of the returned income. The Subsidiary Companys
shall also intimate to the Commissioner Inland Revenue having jurisdiction over the Holding Company
regarding their option for group taxation.
(G) Tax matters relating to the period prior to adoption of one fiscal unit shall continue to be dealt by the
Commissioner Inland Revenue having jurisdiction over the Subsidiary Company.
(H) If there is divestment of a subsidiary company and the provisions of group taxation become
inapplicable, no effect shall be taken for group taxation during the year of disposal.
(I) All the provisions of the Ordinance as applicable on a Holding Company shall also apply to a Subsidiary
Company during the period when the group is taxed as one fiscal unit. Each company shall file
independent withholding statements as required under the provisions of the Ordinance.
(J) Relief u/s 59AA shall be limited only to the companies that are locally incorporated under the
Companies Ordinance, 1984 and all companies opting for group taxation shall have the similar
accounting period for computation of income.
(K) The transaction by any company within the group and with its associated companies shall be carried out
and recorded on arm's length basis.

Conceptual Approach to Taxes 225


Losses Chapter-14

Example: ABC holding company is having 100% of shares of its subsidiary XYZ Limited. The turnover and profit and
losses of holding and subsidiary as per tax are as under:
ABC Ltd XYZ Ltd
Turnover 5,000,000 2,000,000
Taxable income 1,000,000 -
Loss as per tax - 600,000
Required: Compute the tax liability of the company.
Solution:
Computation of income: Rs.
Taxable income of holding company 1,000,000
Less: Loss of subsidiary (600,000)
Total taxable income 400,000

Computation of tax liability:


Higher of:
Tax on Rs. 400,000 @ 32% 128,000
OR
Tax on total turnover Rs. 7,000,000 @ 1% 70,000
As the tax payable under Normal Tax Regime is higher than the minimum
tax payable u/s 113, hence the same is payable by the company 128,000
Note -1: This question has been solved on the assumption that all other conditions required for taxation under group
taxation have been met by the group.
Note -2: In the absence of information it has been assumed that the tax under Alternative Corporate tax u/s 113C of
the Income tax Ordinance, 2001 @ 17% on accounting profit is less than the corporate tax.
9. Group relief (Section 59B)
(1) Any company, being a subsidiary of a Holding Company, may surrender its assessed loss (excluding capital
loss) for the tax year (other than brought forward losses and capital losses), in favour of its Holding Company
or its subsidiary or between another subsidiary of the HC:
Provided that where one of the company in the group is a public company listed on a registered stock
exchange in Pakistan, the Holding Company shall directly hold 55% or more of the share capital of the
Subsidiary Company. Where none of the companies in the group is a listed company, the holding company
shall hold directly 75% or more of the share capital of the Subsidiary Company.
(2) The loss surrendered by the Subsidiary Company may be claimed by the Holding Company or a Subsidiary
Company for set off against its income under the head "income from Business" in the tax year and the
following two tax years subject to the following conditions, namely:
(a) there is continued ownership for 5 years, of share capital of the Subsidiary Company to the extent of
55% in the case of a listed company, or 75% or more, in the case of other companies;
(b) a company within the group engaged in the business of trading shall not be entitled to avail group relief;
(c) Holding Company, being a private limited company with 75% of ownership of share capital gets itself
listed within 3 years from the year in which loss is claimed;
(d) the group companies are locally incorporated companies under the Companies Ordinance, 1984;
(e) the loss surrendered and loss claimed under this section shall have approval of the Board of Directors
of the respective companies;
(f) the Subsidiary Company continues the same business during the said period of 3 years;
(g) all the companies in the group shall comply with such corporate governance requirements and group
designation rules or regulations as may be specified by the Securities and Exchange Commission of
Pakistan from time to time, and are designated as companies entitled to avail group relief; and
(h) any other condition as may be prescribed.
(3) The Subsidiary Company shall not be allowed to surrender its assessed losses for set off against income of
Holding Company for more than 3 tax years.
(4) Where the losses surrendered by a Subsidiary Company are not adjusted against income of the Holding
Company in the said 3 tax years, the Subsidiary Company shall carry forward the unadjusted losses in the
remaining 3 tax years.

226 Conceptual Approach to Taxes


Losses Chapter-14

(5) If there has been any disposal of shares by the Holding Company during the aforesaid period of 5 years to
bring the ownership of the Holding Company to less than 55% or 75%, as the case may be, the Holding
Company shall, in the year of disposal, offer the amount of profit on which taxes have not been paid due to set
off of losses surrendered by the subsidiary company.
(6) Loss claiming company shall, with the approval of the Board of Directors, transfer cash to the loss
surrendering company equal to the amount of tax payable on the profits to be set off against the acquired loss
at the applicable tax rate. The transfer of cash would not be taken as a taxable event in the case of either of
the two companies.
(7) The transfer of shares between companies and the shareholders, in one direction would not be taken as a
taxable event provided the transfer is to acquire share capital for formation of the group and approval of the
Securities and Exchange Commission of Pakistan or State Bank of Pakistan, as the case may be, has been
obtained in this effect. Sale and purchase from third party would be taken as taxable event.
Example: TS (Pvt.) Ltd., a fast growing IT solution provider, a wholly owned subsidiary of a listed company,
commenced its operations in 2011. The details of tax losses incurred by the subsidiary company are as follows:
Accounting year Tax year Losses as per return Assessed losses
Rs. Rs.
June 30, 2012 2012 5,750,000 4,500,000
June 30, 2013 2013 4,800,000 3,782,500
June 30, 2014 2014 4,200,000 3,500,100
June 30, 2015 2015 3,711,800 3,050,000
June 30, 2016 2016 2,750,800 2,200,000
One of the directors is of the view that the holding company can set off the losses of its subsidiary. As a Tax
Consultant, you are required to advise on the following:
(i) What are the pre-requisites for claiming the losses of the subsidiary?
(ii) How much amount can the holding company claim against the subsidiarys losses?
Solution:
(i) For solution of this part see the material given u/s 59B above.
(ii) After fulfillment of all the conditions as stated above the loss of the current year 2016 Rs. 2,200,00 may be
adjusted against the profit of the Holding Company for the tax year 2015 or against the profit of another
subsidiary of the holding company. The brought forward losses are not allowed to adjust against the Holding
Company profit however the Subsidiary Company shall carry forward the preceding years losses in normal
way.

CHANGE IN COMTROL OF AN ENTITY [98]


Where there is a change of fifty percent (50%) or more in the underlying ownership of an entity then any loss
incurred for a tax year before the change shall not be allowed as deduction (set-off) in the tax year after
change.
However, the above provision shall not apply If the following conditions are fulfilled:

1. The entity continues to conduct the same business until the loss has been fully set off; and

2. The entity does not engage in any new business or investment after the change until the loss has been
fully set off.

From the above it is evident that an entity may set-off it losses incurred prior to the change in underlying
ownership, if it continues to conduct the same business and does not engage in any new business or
investment.

Entity means a company or an association of persons [98(2)]

Underlying Ownership means ownership interest in the entity held, directly or indirectly through an
interposed entity or entities, by an individual or by a person not ultimately owned by individuals [u/s
98(2)].

Ownership Interest means a share in a company or the interest of a member in an Association of Persons
[u/s 98(2)].

Conceptual Approach to Taxes 227


Losses Chapter-14

PRACTICE QUESTIONS WITH SOLUTIONS


Q.NO.1 For the tax year 2016, Miss Summer provided the following particulars of income. Compute her total income.
Rs.
1. Profit from speculation business 150,000
2. Profit from' Capital Gains' 100,000
3. Loss from wholesale business 250,000
4. Income from dividend 40,000
Solution: Rs.
Profit from speculation business 150,000
Profit from capital gains (assumed u/s 37) 100,000
Loss from whole sale business (assumed covered under NTR) (250,000)
-
Dividend is taxable @ 12.5% or 7.5% fully under FTR.
Q.NO. 2 Miss Amna Hashmi, an individual, sustains a loss in speculation business of Rs. 20,000 during the tax year 2015.
The detail of other income and losses for the same period is an under. Rs.
i Income from salary 600,000
ii Capital Gain (exempt) 200,000
iii Dividend income 150,000
iv Loss from 'Income from Other Sources (250,000)
Required:
Compute total income of Miss Amna Hashmi after making inter-head adjustment of losses.
Solution: Rs.
Income from salary 600,000
Total taxable income 600,000

N-1 Loss on 'Income from Speculation Business' cannot be set off other than Income from Income from Speculation
Business so it will be carried forward up to six years
N-2 Exempt capital gain cannot be used to set off of losses.
N-3 Dividend is taxable @ 12.5% under FTR therefore any other loss cannot be set off against the same.
N-4 Loss under the head income from other sources cannot be set off against income from salary.
Q.NO.3 Fraz Attari, an individual, suffered a loss of Rs. 150,000 during the tax year 2016 from sale of a capital asset (shares
of a public company). For the same year his income from business is Rs. 195,000. He made inter-head adjustment and
computed total income at Rs. 45,000. Is he allowed to do so under the law?
Solution:
Any loss u/s 37A on a 'Security chargeable to tax' can only be set off against income on another security chargeable to tax
so the treatment done is not correct.
Q.NO. 4 Following are the particulars submitted by Mr. N for the tax year 2016. Calculate taxable income for the year.
Rs.
Income from Business 150,000
Loss from other sources (-)50,000
B/f assessed loss for assessment year 2008 (-)70,000
B/f assessed loss for assessment year 2010 (-)60,000
Solution: Rs.
Income from business 150,000
Loss from other sources (50,000)
100,000
Brought forward assessed loss for assessment year 2010 (60,000)
Taxable Income for the year 40,000
N-1 Loss of assessment year 2008 cannot be set off as it has passed more than six years.

228 Conceptual Approach to Taxes


Losses Chapter-14

Q.NO. 5 Mr. Afzaal, an individual, sustained a loss of Rs.250,000 from speculation business during the tax year 2016. This
could not be set-off against any profits of speculation business during the relevant year. The Commissioner Inland Revenue
accepted the loss while making an assessment. Can C carry forward this loss and what is the time limitation for carry
forward?
Solution:
1. Speculation loss can be carried forward for future speculation incomes.
2. It can be carried forward for next six succeeding tax years.
Q.NO. 6 Bilal Rana, an individual, furnishes the following information relevant for the year 2016.
Profit Loss
Rs. Rs.
Salary income (computed) 240,000
Income from property:
House A 150,000
House B 170,000
House C 210,000
Income from business or profession
Business A 80,000
Business B 100,000
Business C (speculation) 110,000
Business D (speculation) 230,000
Capital gain
Sale of shares of ABC (Pvt.) Limited 100,000
Sale of shares of XYZ (Pvt.) Limited 120,000
Loss from other sources 120,000
Required: Determine the net income of Bilal Rana for the tax year 2016.
Solution:
Rs. Rs.

Salary income (without set off of any loss against salary income) 240,000

Loss under the head Income from Property (150,000 170,000 210,000) (230,000)

Income from business A 80,000


Loss from business B (100,000)
Loss on Income from other sources (120,000)
(140,000)

Income from speculation business C 110,000


Loss from speculation business D (230,000)
Loss on 'Speculation business' c/f (120,000) -

Gain on Sale of shares of ABC (Pvt.) Ltd. 100,000


Loss on Sale of shares of XYZ (Pvt.) Ltd. (120,000)
Loss on 'Capital Gains' c/f (20,000) -
Total taxable income for the year 240,000
N-1 Loss under the head Income from Property cannot be set off against Income from Salary.
N-2 Out of total loss of Rs.140,000 the business loss of Rs.20,000 shall be carried forward whereas loss of Rs. 120,000
under the head of income from other sources shall not be carried forward.
N-3 Speculation loss cannot be adjusted with other than speculation income so unadjusted loss is carried forward.
N-4 Capital Loss cannot be adjusted with other than capital gains so unadjusted loss may be carry forward.
N-5 It is assume that capital assets are disposed of within one year.
Q.NO. 7 Calculate taxable Income of Mr. Z from the following:
Rs.
Salary income taxable 247,000
Income from business
Business A 93,000
Business B (100,000)

Conceptual Approach to Taxes 229


Losses Chapter-14

Speculative transaction 1 (125,000)


Speculative transaction 2 65,000
Capital gains
Sale of shares of AB (Pvt.) Ltd holding period less than 12 months 69,000
Sale of shares of CD (Pvt.) Ltd holding period 3 years 88,000
Sale of shares of EF (Pvt.) Ltd holding period 8 months (77,000)
Sale of shares of GH (Pvt.) Ltd holding period 19 months (68,000)
Sales of modaraba certificate holding period 2 months 108,000
Sales of listed shares holding period 11 months (99,000)
Sales of painting holding period 7 years 90,000
Sales of antique holding period 6 years (170,000)
Solution:

INCOME FROM SALARY Rs. Rs.


Salary income 247,000

INCOME FROM BUSINESS


Speculation loss on transaction 1 (125,000)
Speculation profit on transaction 2 65,000
Net speculation loss to be carried forward (60,000)

Income from business A 93,000


Business loss (100,000)
(7,000)
CAPITAL GAIN
Sale of shares of AB (Pvt.) Ltd holding period less than12 months 69,000
Sale of shares of CD (Pvt.) Ltd holding period 3 years
(88,000 x 75%) 66,000
Sale of shares of EF (Pvt.) Ltd holding period 8 months (77,000)
Sale of shares of GH (Pvt.) Ltd holding period 19 months (68,000)
Sale of painting
(90,000 x 75%) 67,500
Loss on Sale of antique holding period 6 years
(Not to be recognized) -
Gain on disposal of capital assets 57,500

Sales of modaraba certificate holding period 2 months 108,000


Sales of listed shares holding period 11 months (99,000)
Income from securities (SBI) 9,000

Taxable income 297,500


Note 1 Speculation loss shall be carried forward for six succeeding tax years for adjustment against the future speculation
income if any. Gain of sale of securities shall be treated as a separate block of income.
Note 2 Business loss has been adjusted against capital gain and not allowed to set off against salary income.

230 Conceptual Approach to Taxes


Losses Chapter-14

MULTIPLE CHOICE QUESTIONS


Q.1 Loss from the speculation business may be set off against___________.
(a) Income from any other head of income
(b) Income from speculation business only
(c) Both a and b
(d) None of the above
Q.2 Capital losses can_____________.
(a) Be set off against income from any head of income
(b) not be set off against any other head of income
(c) Be set off against capital gains during the same tax year
(d) None of the above
Q.3 The business loss during the specific exemption period may be set off against the business income___________.
(a) after the exemption period
(b) Before the exemption period
(c) Not to be set off and carried forward
(d) both a and b
Q.4 Loss incurred from speculation and non-speculation business can be carried forward in the immediately succeeding
___________tax years.
(a) 6 years
(b) 5 years
(c) 4 years
(d) 7 years
Q.5 There can be no loss under the head:
(a) Income from Salary
(b) Income from business
(c) Income from other sources
(d) None of the above
Q.6 While setting off the loss under the head income from other sources, business losses and unabsorbed depreciation
loss the adjustment shall be in the order of _____________.
(a) Loss from other sources, depreciation and business loss
(b) Depreciation, business loss and other income
(c) All shall be adjusted at the same time
(d) None of the above.
Q.7 An industrial undertaking set up in any export processing zone can carry forward its assessed business losses up
to______________.
(a) 6 years
(b) 5 years
(c) 10 years
(d) unlimited time
Q.8 The assessed brought forward losses and capital loss of the amalgamating company can be adjusted with the profits
and gains of the amalgamated company for up to:
(a) 6 years succeeding from the year of amalgamation
(b) 10 years from the succeeding year of amalgamation
(c) 5 years succeeding the year of amalgamation
(d) None of the above

Conceptual Approach to Taxes 231


Losses Chapter-14

Q.9 The business loss and unabsorbed depreciation can be set off or carry forward if the amalgamated company (other
than banking and non banking companies) continues to carry the business of the amalgamating company at least for
the period of_____________.
(a) 6 years
(b) 5 years
(c) 4 years
(d) None of above
Q.10 Any unabsorbed depreciation allowance under the Income Tax Ordinance, 2001 shall be carried forward up to the
period of_________.
(a) 6 years
(b) 7 years
(c) 10 years
(d) Indefinite period
Q.11 Foreign loss sustained by a resident person shall___________.
(a) Be set off against any head of income
(b) Be set off against business income only which is earned in Pakistan
(c) Be set off against his foreign source income and to be carried forward under the same head of income
(d) Not be set off
Q.12 Foreign losses can be carried forward up to the period of________.
(a) 10 years
(b) 6 years
(c) Cannot be carried forward
(d) None of the above
Q.13 Loss sustained by a permanently exempt business during the exemption period can_________.
(a) Not be set off or carried forward
(b) Be carried forward upto the period of 6 years after the end of exemption period
(c) Be carried forward upto the period of 6 years including the exemption period
(d) None of the above
Q.14 A subsidiary companys loss can_____________.
(a) Be set off and carry forward upto the period of 6 years
(b) Not be carry forward
(c) Be set off and carry forward for an indefinite period
(d) None of the above
Q.15 The holding company can claim the loss surrendered by a subsidiary if_____________.
(a) It holds at least 75% of share capital of a subsidiary when one company in the group is public company listed
on registered stock exchange
(b) It holds at least 55% of share capital of a subsidiary company when none of the companies in the group is a
listed company
(c) Both a and b
(d) None of the above
Q.16 The holding company or its subsidiary can set off the surrendered loss of a subsidiary if___________.
(a) There is a continued ownership for 5 years of share capital
(b) There is a continued ownership for 6 years of share capital
(c) There is a continued ownership of 3 years of the share capital
(d) None of the above

232 Conceptual Approach to Taxes


Losses Chapter-14

Q.17 Surrendered loss from a subsidiary shall be claimed by the holding company or its subsidiary if_____________.
(a) There is no change in the business of a subsidiary company
(b) It continues the same business during the period of 3 years from the year in which the loss is claimed
(c) Both a and b
(d) None of the above
Q.18 Where the loss is reverted back to the subsidiary company after the expiry of the prescribed time for set off by the
holding company the subsidiary may____________.
(a) Set off and carry forward the loss up to 3 years from the date it is reverted back
(b) Set off and carry forward the loss up to 6 years from the date it is reverted back
(c) Both a and b
(d) None of the above
Q.19 There can be no loss under the following heads of incomes______________.
(a) salary
(b) property
(c) business income under NTR
(d) income from other sources
Q.20 Loss on disposal of personal assets of a taxpayer ____ be set off and carried forward against any head of income.
(a) can
(b) cannot
(c) none of above
Q.21 Unabsorbed assessed business losses of preceding years shall be set off _____ the adjustment of brought forward
unabsorbed assessed depreciation allowance.
(a) after
(b) before
(c) at the same time of
(d) all of above
Q.22 Group relief is not available to company engaged in the business of _______.
(a) trading
(b) manufacturing
(c) providing of services
(d) none of above
Q.23 In case of adjustment of assessed business losses the loss of the earliest year shall be set off _________.
(a) first
(b) last
(c) none of above
Q.24 The losses of speculation business can only be carried forward against income from the _____ head.
(a) any
(b) same
(c) other
(d) all of above
Q.25 Capital losses cannot be carried forward against income of ____ head.
(a) any
(b) same
(c) other
(d) all of above

Conceptual Approach to Taxes 233


Losses Chapter-14

Q.26 The loss of the amalgamating company other than _____________ can be adjusted against the income of the
amalgamated company.
(a) brought forward losses
(b) capital losses
(c) business losses
(d) both a and b
Q.27 Where the current year loss of the amalgamating company cannot be set off against the income of the amalgamated
company then the same will be carried forward for ___ years.
(a) 5
(b) 6
(c) 7
(d) 8
Q.28 The members of an AOP are ________ to set off and carry forward their share of loss received by them from the
AOP.
(a) not allowed
(b) allowed
(c) both a or c
(d) none of above
Q.29 There is no time limit for carry forward of ________________.
(a) business loss
(b) unabsorbed depreciation
(c) unabsorbed amortization
(d) both b and c.
Q.30 The share of losses representing the retired or deceased partner ____ be set off by the AOP.
(a) can
(b) cannot
(c) none of above
Q.31 Losses of inherited business can be set off and carried forward by any taxpayer (excluding ______).
(a) minor
(b) deceased person
(c) salaried person
(d) all of above

ANSWERS
1 (b) 2 (c) 3 (a) 4 (a) 5 (a)
6 (d) 7 (d) 8 (d) 9 (b) 10 (d)
11 (c) 12 (b) 13 (a) 14 (a) 15 (d)
16 (a) 17 (c) 18 (a) 19 (a) 20 (b)
21 (b) 22 (a) 23 (a) 24 (b) 25 (c)
26 (d) 27 (b) 28 (a) 29 (d) 30 (b)
31 (a)

234 Conceptual Approach to Taxes


Losses Chapter-14

ICMAP PAST PAPERS THEORECTICAL QUESTIONS


Q.NO.3 (a) March 2015 Venus limited is a public listed copy having three (3) subsidiaries. One of its subsidiaries Mars
Limited has been suffering losses for last few years. The Board of Directors of Mars income tax ordinance, 2001.
Required:
In view of the provisions of section 59 B of the income tax ordinance, 2001 you are required to answer the following:
(i) For how long a continued ownership of the share capital would be required for Venus with Mars Limited for
adjustment of losses surrendered by the Mars against its incomes under the head income from business
and whose approval with regard to such surrendered losses would be required?
(ii) In case of Mars Limited what percentage of ownership of share capital is mandatory for Venus Limited?
(iii) For how man tax years Mars Limited can surrender its tax losses and how would its unadjusted losses be
treated after the specified period?
(iv) Which type of losses cannot be surrendered by a subsidiary in favor of holding company and other
subsidiary in the group?

Q. No. 2 (b) August 2012 M/s Ahsan Bilal and Company deals in speculation and non-speculation business. The
company is allowed to set off and carry forward its losses arising out of its speculation business under section
58 of the Income Tax Ordinance, 2001. The following information has been extracted from the books of accounts of the
company for the year ended June 30, 2012:
(Rupees)
Income from speculation business 550,000
Income from non-speculation business 820,000
Carried forward losses for the last three tax years from speculation business 800,000
In the light of section 58 of the Income Tax Ordinance, 2001 answer the following:
(i) Can the company set off its losses arising out of its speculation business? Briefly state the provision.
(ii) What would be the treatment of unadjusted losses arising out of speculation business under said section?
(iii) How long losses arising out of speculation business can be carried forward?
(iv) M/s Ahsan Bilal and Company has a loss carried forward for the last three years. Which year loss will be set
off first under the above section?
Q. NO.3 (a) WINTER 2008 What do you understand from the concept of loss carried forward under the Income tax
Ordinance, 2001 with regard to each of the following heads of income?
(i) Income from business. (ii) Speculation business losses. (iii) Capital losses.
Q. NO. 2 (d) SUMMER 2008 What a foreign loss is as described in the Income tax Ordinance, 2001? How these foreign
losses are treated under the Income Tax Ordinance, 2001?
Q.NO 4(a) Spring 2007 Explain speculation business and rules of set off and carry forward of losses out of speculation
business
(b) What do you understand by the term speculation business as referred to in the Income tax Ordinance, 2001? Briefly
discuss the rules relating to set off and carry forward of losses arising out of speculation business.
Q. NO 6 (b) WINTER 2006 (Briefly state) Set off losses other than the speculation business losses and capital losses u/s 56
of the Income tax Ordinance, 2001.
Q. NO. 4 (a) SUMMER 2006 Describe the speculation business and mention the businesses which are not included in
speculation business under the Income tax Ordinance, 2001.
Q. NO. 8 (a) WINTER 2004 Explain the provisions of Carry forward of capital losses under the Income tax Ordinance, 2001.
Q. NO. 4 (a) SUMMER 2004 Explain the provisions for Carry forward of speculation business losses under Income tax
Ordinance, 2001.

NOW SOLVE FOLLOWING NUMERICAL QUESTIONS OF ICMAP PAST PAPER RELATED TO THIS TOPIC

Q.NO. 3(B) WINTER 2008

Conceptual Approach to Taxes 235


Losses Chapter-14

CA MOD C PAST PAPERS THEORECTICAL QUESTIONS


Q. NO. 5(b) Spring 2015 State the rules relating to set-off and carry-forward of losses of AOP and its members.

Q. NO. 6(b) Spring 2014 Explain the term Foreign losses. State the provisions relating to set off and carry forward of
foreign losses, under the Income Tax Ordinance, 2001.
Q. 3 (b) Autumn 2011 Under the Income tax Ordinance, 2001 the loss surrendered by a subsidiary company may be
claimed by the holding company for set off against its business income in that tax year and following two tax years, subject
to certain conditions.
Required: List the conditions which are necessary for claiming the subsidiarys losses.
Q.NO. 6(b) Spring 2009 Discuss the provisions of the Income tax Ordinance, 2001 regarding set off and carry forward of
losses under the heads Income from Business and Capital Gains.
Q.NO. 1(b) Spring 2007 Explain the principles of taxation and filing of return relating to members of association of person
where the association is:
1. a professional firm
2. other than a professional firm.
Also discuss the rules relating to set off and carry forward of associations losses.
Q.NO. 6 Spring 2005 The time limit for claiming group relief by a holding company in respect of loss surrendered by a
subsidiary company.
Briefly discuss the time limit for claiming group relief by a holding company in respect of loss surrendered by a subsidiary
company? What conditions need to be fulfilled in this regard?
Q.NO. 8 Autumn 2004 Describe the provisions relating to set-off and carry forward of foreign losses under the Income tax
Ordinance, 2001?
Q.NO. 3(b) Autumn 2003 Briefly explain the law relating to set-off and carry forward of losses?
Q.6 Autumn 2002 Please write a brief note about the adjustment of loss incurred under any head of income in the current
year.
Q. NO. 6 May 1997 write short notes on set off and carry forward of business losses.

NOW SOLVE FOLLOWING NUMERICAL QUESTIONS OF MODULE C / AFC PAST PAPER RELATED TO THIS TOPIC
Q. NO. 3(B) SPRING 2013
Q. NO. 7(B) SPRING 2006

236 Conceptual Approach to Taxes


Tax Credits Chapter-15

Chapter

15 TAX CREDITS

Topic covered Topic covered


Section Rule (Part 1 For CAF-6 & ICMAP Section
students)
Miscellaneous provisions relating to tax
65 (Part II For CA mod F & ICMAP students)
credits
103 16 Foreign tax credit 65C Tax credit for enlistment
Tax credit for newly established industrial
61 Credit for charitable donations 65D
undertakings
Tax credit for industrial undertakings Established
62 Investment in shares & Insurance 65E
before July 01, 2011
63 Contribution to approved pension fund Table of tax credits under section 65B to 65E
Tax reductions for flying and submarine
Tax credit for employment generation by allowance, total allowance to pilot of Pakistani
64B
manufacturers airlines, senior citizen, full time teacher &
researcher & profit on Behbood certificates etc.
Tax credit for exempt share from Tax credit for tax already paid or deducted at
88 168
association of persons source
100C Tax credit for certain persons
Tax credit for a person registered under
65A MCQs with solutions
Sales Tax Act
ICMAP & CA Mod C past papers theoretical
65B Tax credit for investment
questions

(Part I For CAF-6 & ICMAP students)


1. Tax credits and rebates
There are different types of tax credit available under the Income Tax Ordinance, 2001. If a taxpayer is allowed more
than one tax credit then credits shall be applied in the following order: The following order may also be examined from
the FBR Income tax return format available on E-portal.
Tax credit Relevant clause / section
(1) Foreign tax credit Section 103
(2) Tax credits [Covered as under from (a) to (j)] Section 61 to 65E
a. Tax credit on charitable donations Section 61
b. Tax credit on investment in shares and insurance Section 62
c. Tax credit on Contribution to approved pension fund Section 63
d. Deductible allowance for profit on debt Section 64A
e. Tax credit for employment generation by manufacturers Section 64B
f. Tax credit for exempt share from AOP Section 88
g. Tax credit to a person registered under the Sales Tax Act, 1990 Section 65A
h. Tax credit for investment Section 65B
i. Tax credit for enlistment Section 65C
j. Tax credit for newly established industrial undertakings Section 65D
k. Tax credit for industrial undertakings established before the first day Section 65E
of July, 2011

Conceptual Approach to Taxes 237


Tax Credits Chapter-15

(3) Reduction in tax liability in case of flying and submarine allowance Clause (1) Part-III 2nd Schedule
(4) Reduction in tax liability in case of total allowances received by pilots of Clause (1AA) Part-III 2nd
Pakistani airlines Schedule
(5) Reduction in tax liability in case of senior citizens Clause 1A Part-III 2nd Schedule
(6) Reduction in tax liability in case of full time teacher or researcher Clause 2 Part-III 2nd Schedule
(7) Reduction in tax liability on Yield or profit on Behbood and Pensioners Clause (5) Part III 2nd Schedule
Certificates / Accounts
(8) Tax credit for tax already paid or deducted at source Under various sections

(See master example at the end of this chapter.)


2. Miscellaneous provisions relating to tax credits (Section 65)
(a) Where a person is member of an AOP and then Tax credit shall be calculated including share of profit from
AOP (See master example at the end of this chapter)
(b) Where a person is earning income from property then tax credit shall be calculated including the property
income with effect from tax year 2011 (See master example at the end of this chapter)
(c) Tax credit in excess of tax liability shall not be refunded, carried forward or carried back to a preceding tax
year. [Section 65(3)]
(d) If the person is member of AOP then tax credit in excess of tax liability can be claimed by the AOP by
agreement in writing and such agreement must be furnished with the return of AOP. [Section 65(4) and (5)]
3. Foreign tax credit (Section 103)
Tax credit on net foreign source income (after admissible deductions allowed under this Ordinance) other than the
salary income shall be lesser of foreign income tax paid or the Pakistani tax payable on average rate on total
income including foreign source income.
If foreign income is under more than one head including speculation business, the provisions of this section will apply
separately to each head.
The foreign tax credit will be allowed before any other credit but if the credit cannot be wholly allowed, the remainder
will neither be refunded, carried back nor carried forward to the following tax year.
A tax credit shall be allowed under this section only if the foreign income tax is paid within 2 years after the end of
the tax year in which the foreign income was derived by the resident taxpayer.
Foreign tax credit [Rule 16]
A resident taxpayer claiming a foreign tax credit for a tax year shall submit an application in the form as specified in
Part I of the First Schedule to IT Rules, 2002 for the credit with the taxpayer's return of income for that year.
An application for a foreign tax credit shall be accompanied by declaration from the payer along-with certified copy of
evidence of tax deducted at source from the foreign tax authority.
Where a resident taxpayer cannot obtain evidence of the deduction of tax from the payer of income as above, the
Commissioner Inland Revenue may accept such secondary evidence of the deduction as is determined by him.

Example: Miss Samina has provided you following information for computation of taxable income and tax liability:
Pakistan source income:
(a) Income from business Rs. 537,500,
(b) Income from other sources Rs. 100,000
Foreign source income:
Income from business 212,500
Note: Income tax paid on foreign source income is Rs. 10,000.
Solution:
Miss Samina
Computation of taxable income and tax thereon Rs. Rs.
Income from business (Pakistan source) 537,500
Income from other sources 100,000
Income from business (Foreign source) 212,500
Taxable income 850,000

238 Conceptual Approach to Taxes


Tax Credits Chapter-15

Computation of tax liability


Tax on Rs. 850,000 [32,000 + 15% x (850,000 750,000)] 47,000
Less: Foreign tax credit
Lower of:
- Pakistan tax in respect of foreign source income
(47,000 x 212,500 / 850,000) 11,750
OR
- Foreign income tax paid 10,000 10,000
Balance tax payable 37,000
4. Charitable donations (Section 61)
The Income tax Ordinance, 2001 lays down two classifications of donations made to the approved institutions.
(i) Tax credit u/s 61.
(ii) Straight deduction from total income u/c 61 of Part I of Second Schedule.
However, the amount of donation should not be more than 30% or 20% of taxable income for individual / AOP or
company respectively.
A tax credit shall be allowed on fulfilment of following conditions.
1. They are made in kind / cash to an educational institution, hospital or a relief fund established or run by
Federal Government, Provincial Government or Local Government or any non-profit organization during the
tax year and taxpayer possesses the evidences in respect of the same.
2. Donation must be made through banking channel, however, if donation is made to the institutions
specified in Clause 61 Part I Second Schedule, the condition of payment through banking channel is
not applicable.
3. The name of few institutions mentioned under clause 61 are as under:
1. Shaukat Khanum Memorial Trust, Lahore 2. Fatimid Foundation, Karachi
3. National Museums, National Libraries 4. Iqbal Memorial Fund

Example: Mr Qaisers business income for the tax year 2014 is Rs. 450,000 and he made donation of Rs.10,000.
Compute tax liability of Mr Qaiser if donation was made to:
(a) An unapproved institution
(b) Approved institution that falls in section 61
(c) Institution specified u/c 61 Part I Second Schedule.

Solution:
(a) Mr. Qaiser
Computation of taxable income and tax liability: Rs.
Income 450,000
Computation of tax liability:
Tax on Rs. 450,000 [7% x (450,000 400,000)] 3,500
No tax credit shall be allowed as donation was made to unapproved institution.
(b) Mr. Qaiser
Computation of taxable income and tax liability: Rs.
Income 450,000
Computation of tax liability:
Tax on Rs. 450,000 [7% x (450,000 400,000)] 3,500
Less: Tax credit on donation:
Tax credit shall be allowed on lower of:
- Actual amount of donation i.e. Rs. 10,000
- 30% of taxable income i.e. Rs. 135,000
Tax credit (10,000 x 3,500 / 450,000) 77
Tax liability 3,423

Conceptual Approach to Taxes 239


Tax Credits Chapter-15

(c) Mr. Qaiser


Computation of taxable income and tax liability: Rs.
Income 450,000
Less: Donation as straight deduction (see note 1 attached) 10,000
Taxable income 440,000
Computation of tax liability:
Tax on Rs. 440,000 [7% x (440,000 400,000)] 2,800
(Note 1): In this case the amount of donation shall be deducted from total income to compute taxable income
provided that donation should not exceed 30% of taxable income in case of individual.
5. Tax credit for investment in shares and insurance (Section 62)
A resident person (other than a Company) shall be entitled to a tax credit for a tax year either;
(A) Investment in shares
(a) new shares of listed company on a stock exchange in Pakistan provided the resident person is the
original allottee of the shares or the shares are acquired from the Privatization Commission of Pakistan
(b) The Ordinance is silent hence higher of the above two shall be considered for tax credit purposes.
Provided shares are required to be held for at least 24 months from the date of purchase but if a person
disposes of shares within 24 months then the amount of tax credit allowed shall be added in the tax
payable of the tax year in which shares were disposed of.
OR
(B) Life insurance premium
Any life insurance premium paid on a policy to a life insurance company registered by the SECP under the
Insurance Ordinance, 2000, provided the resident person is deriving income chargeable to tax under the
head salary or income from business.
Tax credit is allowed which is lesser of
(a) total cost of acquiring the shares or the total contribution or premium paid by the person (Higher from
three amounts as Ordinance is silent on this issue) or
(b) 20% of taxable income or
(c) Rs. 1,500,000.

Example: Following information is related to Mr. Iqbal.


a. Income from business Rs. 450,000,
b. Investment made in shares of a listed company Rs. 10,000
c. Life insurance premium paid Rs. 8,000
Required: Compute tax liability of Mr. Iqbal for tax year 2016.
Solution:
Note: Law is silent about a situation where a person has made investment in shares and also paid insurance
premium. Hence in this case, tax credit shall be calculated at higher of these two amounts.
Mr. Iqbal
Computation of taxable income and tax liability: Rs.
Income 450,000
Computation of tax liability:
Tax on Rs. 450,000 [7% x (450,000 400,000)] 3,500
Less: Tax credit on investment in shares and insurance:
Tax credit shall be allowed on lower of (A) or (B) as under:
(A)
Higher of investment in share or premium paid i.e. Rs. 10,000 or
(B)
- 20% of taxable income i.e. Rs. 90,000 or
- Rs. 1,500,000
Hence tax credit shall be on lower of (A) or (B) (10,000 x 3,500 / 450,000) 77
Tax liability 3,423

240 Conceptual Approach to Taxes


Tax Credits Chapter-15

6. Tax credit for Contribution to an approved pension fund (Section 63)


An eligible person deriving salary income or business income shall be allowed a tax credit in respect of
contribution to a pension fund approved by Voluntary Pension System Rules, 2005 shall be lesser of the following at
average rate of tax:
(a) Contribution or premium paid, or
(b) 20% of taxable income,
Provided if an eligible person joining the pension fund at the age of 41 years or above, during the first 10 years
starting from July 01, 2006 shall be allowed additional contribution of 2% p.a. for each year of age exceeding 40
years. Provided further that the total contribution allowed to such person shall not exceed 50% of the total taxable
income of the preceding year.
In this section "Eligible Person" , means an individual Pakistani who holds a valid NTN or CNIC or NIC for Overseas
Pakistanis issued by the National Database and Registration Authority:
Example: Mr Bilal Idrees business income for the tax year 2016 is Rs. 450,000 and he made contribution of
Rs.10,000 to a pension fund approved by Voluntary Pension System Rules, 2005. The taxable income of tax year
2014 was Rs. 430,000. Mr. Bilal Idrees age was 45 years when he joined the pension fund.
Required: Compute tax liability of Mr. Bilal Idrees for the tax year 2016.
Solution:
Mr. Bilal Idrees
Computation of taxable income and tax liability: Rs.
Income 450,000
Computation of tax liability:
Tax on Rs. 450,000 [7% x (450,000 400,000)] 3,500
Less: Tax credit on contribution to approved pension fund:
Tax credit shall be allowed on lower of:
(a) Actual amount of contribution i.e. Rs. 10,000 or
(b) 30% (instead of 20% with each year above 40 years there is 2%
increase in percentage) of taxable income i.e. Rs. 135,000 with cap of
50% of preceding year taxable income 215,000 (i.e. Rs. 430,000 x 50%)
Hence the lower is Rs.10,000 entitled for tax credit under this section:
Tax credit (10,000 x 3,500 / 450,000) 77
Tax liability 3,423
7. Deductible allowance for profit on debt (Section 64A)
Every individual shall be entitled to a deductible allowance for the amount of any profit or share in rent and
share in appreciation for value of house paid by the individual in a tax year on a loan by a scheduled bank or
non-banking finance institution regulated by the SECP or advanced by Government or the Local Government,
Provincial Government or a statutory body or a public company listed on a registered stock exchange in
Pakistan where the individual utilizes the loan for the construction of a new house or the acquisition of a
house.
The amount of an individuals deductible allowance allowed above for a tax year shall not exceed 50% of
taxable income or Rs. 1,000,000, whichever is lower.
Any allowance or part of an allowance under this section for a tax year that is not able to be deducted for the
year shall not be carried forward to a subsequent tax year.
8. Tax credit for employment generation by manufacturers (Section 64B)
For encouragement of establishing new manufacturing units, a tax credit for ten years has been provided through
Finance Act, 2015 by inserting a new section 64B. According to the provisions of this section, where a
company is formed for establishing and operating new manufacturing unit set up between tax year 2016 to tax
year 2018 (1st July 2015 to 30th June, 2018), it shall be given a tax credit of 1% of tax payable for every fifty (50)
employees from the date on which the manufacturing unit is ready to go into production (trial or commercial)
subject to the fulfillment of the following conditions;

Conceptual Approach to Taxes 241


Tax Credits Chapter-15

a) The employees shall be registered with the Employees Old Age Benefits Institution (EOBI) or Employees
Social Security Institutions (ESSI) of the Provincial Governments during the tax year.
b) The said tax credit is allowed up to maximum of 10% of the tax payable.
c) The manufacturing unit shall be managed by a company formed for operating the said manufacturing unit
and should be registered under the Companies Ordinance, 1984 and have registered office in Pakistan.
d) The manufacturing unit is not established by the splitting up or reconstruction or reconstitution of an
existing undertaking or transfer of plant and machinery of an existing undertaking before 1st July 2015.
Where at any subsequent stage it is discovered that the tax credit allowed under this section was availed
without fulfillment of any one of the above conditions, the Commissioner shall re-compute the tax payable by
the taxpayer on the basis of tax credit wrongly allowed under this section and shall be recovered under the relevant
provisions of the Ordinance.
Example: ABC (Pvt.) Ltd. a newly formed Company on July 01, 2015 has taxable business income for the tax year
2016 is Rs. 1,000,000. If the tax payable by the Company is Rs. 320,000 then compute tax credit under section 64B
by assuming that the Company has met all the preconditions as required under the said section and having 160
employees.
Solution:
ABC (Pvt.) Ltd.
Computation of taxable income and tax liability: Rs.
Tax payable 320,000
Less: Tax credit:
3% of tax payable (For 150 employees) 9,600
Balance tax payable 310,400
9. Tax credit for exempt share from association of persons:
For individuals (Section 88)
Share of profit from an AOP derived by an individual is exempt from tax and does not form part of total / taxable
income. However, where the individual has any income chargeable to tax as total / taxable income, other than the
share from an AOP, then such share of profit is included in the total / taxable income for rate purposes, i.e.
First, the income tax payable in calculated on taxable income inclusive of exempt share from AOP. Thereafter,
proportionate income tax payable in calculated on the income chargeable to tax, other than the share of profit from
AOP.
Technically this is not a tax credit (rebate in income tax payable) but for the sake of simplicity this is termed as a tax
credit. Accordingly exempt share of profits from the AOP is not treated as exempt income and included in the taxable
income; and
A tax credit is allowed on such exempt share of profits from the AOP calculated like other tax credits by applying the
average rate of income tax.
Example: Mr. Asim has income from other sources Rs. 300,000 and share from AOP Rs. 230,000 and paid zakat
Rs.8,000. Compute tax payable by him if tax credits and reductions other than AOP share are Rs. 9,000.
Solution:
Mr. Asim
Computation of taxable income and tax liability: Rs.

Income from other sources 300,000


Add: share of income from AOP (included for rate purposes) 230,000
Total income 530,000
Less Zakat paid 8,000
Taxable income 522,000

Gross tax [7000 +10% (522,000 500,000)] 9,200


Less tax reductions and credits as given in question 9,000
Balance income tax 200
Less income tax credit on share income from AOP (included for rate purposes)
(200 / 522,000 x 230,000) 88
Balance tax payable 112

242 Conceptual Approach to Taxes


Tax Credits Chapter-15

10. Tax credit to a person registered under the Sales Tax Act (Section 65A)
With effect from Tax year 2010 every manufacturer registered under the Sales Tax Act is entitled to tax credit of
2.5% of tax payable for Tax Year if 90% of the sales are made to persons who are registered under the Sales Tax
Act. for claiming credit, the person shall provide complete detail of the persons to whom the sales were made during
the tax year. The facility however is not allowed to a person whose income is covered under final tax regime or
minimum tax regime and further unadjusted balance carry forward facility is not available under this section.
Example: Mr. Amir is a registered manufacturer under the Sales Tax Act, 1990. His taxable income from business is
Rs.850,000. Compute tax payable by him if 95% of his sales are made to persons registered under the Sales Tax
Act, 1990.
Solution:
Mr. Amir
Computation of taxable income and tax liability: Rs.
Income from business 850,000
Computation of tax liability:
Tax on Rs. 850,000 [32,000 + 15% (850,000 750,000)] 47,000
Less: Tax credit (47,000 x 2.5%) 1,175
Tax liability 45,825
11. Tax credit for investment (Section 65B)
(a) Where a taxpayer being a Company invests any amount in the purchase of a plant and machinery for the
purposes of extension, expansion or balancing, modernization and replacement of plant and machinery already
installed there in, in an industrial undertaking set up in Pakistan and owned by it, credit equal to 10% of the
amount so invested shall be allowed against the tax payable (including on account of minimum tax and final
taxes payable) by it. Plant and Machinery should be purchased between 01-07-2010 and 30-06-2016,
In this case, tax credit in excess of tax liability shall be carried forward to adjust in following 2 tax years.
(b) A company setup in Pakistan before 01-07- 2011, which makes investment through 100% new equity during 01-
07-2011 and 30-06-2016, for the purposes of Balancing, Modernization or Replacement (BMR) of the plant and
machinery already installed in an industrial undertaking owned by the company. However, credit equal to 20% of
the amount so invested shall be allowed against the tax payable, including on account of minimum tax and
final taxes payable. The credit shall be allowed in the year in which the plant and machinery in the purchase of
which the investment as aforesaid is made, is installed therein.
In this case, tax credit in excess of tax liability shall be carried forward to adjust in following 5 tax years, however the
tax credit under this section shall not exceed from the aggregate limit defined a and b.
In this section the term new equity shall have the same meaning as defined in section 65E(7).
If it is subsequently discovered by the Commissioner Inland Revenue that any condition was not fulfilled, the credit
originally allowed shall be reversed.
An industrial undertaking shall be treated to have been setup on the date on which the industrial undertaking is ready
to go into production, whether trial production or commercial production.

Example: Following information is related to ABC (Pvt.) Ltd. for tax year 2015:
(a) Income from business Rs. 200,500
(b) Plant purchased for the purpose of balancing, modernisation and replacement Rs. 1,500,000
Required: Compute tax liability under section 65BA and 65B.
Solution under section 65A:
ABC (Pvt.) Ltd.
Computation of taxable income and tax liability: Rs.
Income from business 200,500
Computation of tax liability:
Tax on Rs. 200,500 @ 32% 64,160
Less: Tax credit for investment in fixed assets
(1,500,000 x 10%) 150,000
Tax liability Nil

Conceptual Approach to Taxes 243


Tax Credits Chapter-15

As the amount of tax credit is in excess of tax liability the taxpayer is not liable to pay any tax and the amount of
unadjusted tax credit in this case (only in this case) shall be carried forward for 2 succeeding tax years.
Solution under section 65B:
The amount of tax credit (Rs. 1,500,000 x 20%) Rs. 300,000 shall be allowed and no tax liability is to be paid by the
taxpayer and the unadjusted tax credit Rs. 235,840 shall be carried forward for 5 succeeding tax years.

(Part II For CA module F & ICMAP students)


12. Tax credit for enlistment (Section 65C)
With effect from tax year 2011 where a taxpayer being a Company opts for enlistment in any registered stock
exchange in Pakistan, a tax credit equal to 20% of the tax payable shall be allowed for the tax year in which the said
company is enlisted.
Example: Tax liability of Shalimar Ltd for tax year is Rs. 145,400 for tax year 2016 before tax credit for enlistment.
Compute the amount of tax credit if the company was enlisted in tax year 2016.
Solution:
Tax credit (145,400 x 20%) 29,080
This amount shall be deducted from 145,400 29,080 = 116,320 and this amount shall be paid by the company.
13. Tax credit for equity investment for newly established industrial undertakings [Section 65D]
Tax credit is admissible to a company formed for establishing and operating a new industrial undertaking for
manufacturing including corporate dairy farming in Pakistan, subject to the following conditions;
The company;
is incorporated under the Companies Ordinance, 1984 between 01-07-2011 and 30-06-2016 and has its registered
office in Pakistan;
The industrial undertaking:
- is set up between July 01, 2011 and June 30, 2016 and is managed by a company formed for operating the
said industrial undertaking;
- is not established by the splitting up or reconstruction or reconstitution of an undertaking already in existence
or by transfer of machinery or plant from an industrial undertaking established in Pakistan at any time before
01-07-2011; and
- The industrial undertaking is set up with 100% equity raised through issuance of new shares for cash
consideration
Provided that short term loans and finances obtained from banking companies or non-banking financial institutions for
the purposes of meeting working capital requirements shall not disqualify the taxpayer from claiming tax credit.
This tax credit is admissible in the tax year in which said industrial undertaking is set up or commercial production is
commenced, whichever is later, and the following 4 years.
The amount of tax credit is equal to 100% of the income tax payable including on account of minimum tax and final
taxes payable on the income arising from such industrial undertaking.
Where any credit is allowed and subsequently it is discovered, on the basis of documents or otherwise, by the
Commissioner Inland Revenue that any of the conditions specified above were not fulfilled, the credit originally
allowed shall be deemed to have been wrongly allowed and the Commissioner Inland Revenue may re-compute the
tax payable for the relevant year.
In this section and for sections 65B and 65E an industrial undertaking shall be treated to have been setup on the date
on which the industrial undertaking is ready to go into production, whether trial production or commercial production.
Example: Salman Limited was incorporated on July 10, 2011 for operating a new industrial undertaking established
by the company on August 1, 2011 with 100% equity owned by the Company. Taxable income of the company from
industrial undertaking during tax year 2015 was Rs.4,000,000. The Company is engaged in the manufacture of
motorcycles. Compute tax liability of the company for the tax year 2016.
Solution:
Salman Limited
Computation of taxable income and tax liability Rs.
Income from business
Income from manufacture of motorcycles 4,000,000

Taxable income 4,000,000

244 Conceptual Approach to Taxes


Tax Credits Chapter-15

Computation of tax liability:


Tax on Rs. 4,000,000 @ 32% 1,280,000
Less: Tax credit for newly established undertaking
Tax credit shall be allowed @ 100% of tax payable (1,280,000)
Balance tax - -
14. Tax credit for equity investment in the Balancing, Modernizations and Replacements (BMR) in and industrial
undertakings established before the first day of July, 2011 [Section 65E]
1. Where a taxpayer being a company, setup in Pakistan before the 01-07-2011, invests any amount, with 100%
new equity raised through issuance of new shares, in the purchase and installation of plant and machinery for
an industrial undertaking, including corporate dairy farming, for the purposes of-
(i) expansion of the plant and machinery already installed therein; or
(ii) undertaking a new project,
a tax credit shall be allowed against the tax payable in the manner (a) or (b) as under for a period of 5 years
beginning from the date of setting up or commencement of commercial production from the new plant or
expansion project, whichever is later.
(a) Where a taxpayer maintains separate accounts of an expansion project or a new project, as the case
may be, the taxpayer shall be allowed a tax credit equal to 100% of the tax payable, including minimum
tax and final taxes payable, attributable to such expansion project or new project.
(b) In all other cases, the credit shall be such proportion of the tax payable, including minimum tax and final
taxes payable, as is the proportion between the new equity and the total equity including new equity.
2. The plant and machinery should be installed at any time between the 01-07-2011 and 30-06-2016.
3. The amount of credit admissible shall be deducted from the tax payable, including minimum tax and final taxes
payable, by the taxpayer in respect of the tax year in which the plant or machinery is installed and for the
subsequent 5 years.
4. Where any credit is allowed and subsequently it is discovered, on the basis of documents or otherwise, by the
Commissioner Inland Revenue that any of the above condition specified was not fulfilled, the credit originally
allowed shall be deemed to have been wrongly allowed and the Commissioner Inland Revenue may re-
compute tax payable for the relevant year. and
5. In this section, new equity means equity raised through fresh issue of shares against cash by the company
and shall not include loans obtained from shareholders or directors:
Provided that short term loans and finances obtained from banking companies or non-banking financial institutions for
the purposes of meeting working capital requirements shall not disqualify the taxpayer from claiming tax credit.
An industrial undertaking shall be treated to have been setup on the date on which the industrial undertaking is ready
to go into production, whether trial production or commercial production.
Example: On July 01, 2015, AQ Limited made an investment of Rs. 500,000 in an industrial undertaking in Pakistan
for expansion of the plant and machinery already installed in the undertaking. Taxable income of the company during
tax year 2016 was Rs. 4,000,000. Compute tax liability of the company if total investment (equity and bank loan) in
industrial undertaking is Rs. 2,000,000.
Solution:
AQ Limited
Computation of taxable income and tax liability
Rs.
Income from business
Taxable business income 4,000,000
Taxable income 4,000,000
Computation of tax liability:
Tax on Rs. 4,000,000 @ 32% 1,280,000
Less: Tax credit for investment in industrial undertaking
Tax credit (1,280,000 x 500,000 / 2,000,000) (160,000)
Tax liability 1,120,000

15. Reduction in tax liability in case of flying and submarine allowances


Any amount received as
Flying allowance by flight engineers, navigators of Pakistan Armed Forces, Pakistani Airlines or Civil
aviation Authority, Junior Commissioned Officers or other ranks of Pakistan Armed Forces; and
Submarine allowance by the offices of the Pakistan Navy, shall be taxed @ 2.5% as a separate block of
income.

Conceptual Approach to Taxes 245


Tax Credits Chapter-15

Provided that the reduction under this clause shall be available to so much of the flying allowance or the submarine
allowance as does not exceed an amount equal to the basic salary.
16. Reduction in tax liability in case of total allowances received by pilots of Pakistani airlines
Total allowances received by pilots of any Pakistani airlines shall be taxed at a rate of 7.5%, provided that the
reduction under this clause shall be available to so much of the allowances as exceeds an amount equal to the basic
pay.
17. Reduction in tax liability in case of Senior citizens
Where the taxable income (excluding income covered under final tax regime) in tax year of a taxpayer (an
individual) aged 60 years or more on the first day of that tax year does not exceed Rs. 1,000,000, then his tax
liability shall be reduced by 50%.

Example: 62 years old Mr. Rizwan earned following incomes during the tax year 2016. Compute taxable income
and tax liability for tax year 2016.
(a) Taxable salary Rs. 150,000,
(b) Income from business 440,000,
(c) Examination fee 30,000
Solution:
Mr. Rizwan
Tax year 2016
Computation of taxable income and tax liability: Rs.
Income from salary:
Taxable salary 150,000

Income from business:


Income from business 440,000

Income from other sources:


Examination fee 30,000
Taxable income 620,000

As salary income is less than 50% of taxable income, hence taxpayer is non-salaried person.

Computation of tax liability:


Tax on Rs. 620,000 [7,000 + 10% (620,000 500,000)] 19,000
Less: Senior citizen allowance @ 50% of tax liability 9,500
Tax liability 9,500

18. Reduction in tax liability in case of full time teacher or a researcher


A full time teacher or a researcher of a recognized non-profit educational or research institution including government
training and research institutions shall be allowed a reduction of 40% of tax payable.
The institution, a university or a board of education must be recognized by Higher Education Commission.
This additional tax reduction would be allowed on tax liability on taxable salary. Other income, if any, would be
excluded for this purpose.
Example: Arif Ahmed is 69 years old and has earned following incomes during the tax year:
Taxable salary as a full time teacher from Punjab University Rs.340,000
Income from other sources Rs. 200,000
You are required to compute the tax liability of Mr. Arif for the tax year 2016.
Solution:
Rs.
Taxable salary 340,000
Income from other sources 200,000
Taxable income 540,000

246 Conceptual Approach to Taxes


Tax Credits Chapter-15

COMPUTATION OF TAX LIABILITY:

Tax on Rs. 540,000 [2,000 + 5%(540,000 500,000)] 4,000


Less: senior citizen allowance @ 50% 2,000
2,000
Less: Tax credit to Full time teacher
(2,000 x 340,000 / 540,000 ) x 40% 504
Balance tax payable 1,496
19. Yield or profit on Behbood and Pensioners Certificates / Accounts - Reduction in tax liability (Clause (6) of
Part III of 2nd Schedule to the Income tax Ordinance, 2001)
Profit on debt (yield or profit) on Behbood and Pensioners certificates or accounts under the National Savings
Scheme are not subject to deduction of tax at source and are chargeable to tax as total / taxable income. This is
contrary to all other profit on debts which are subject to deduction of tax at source at the rate of 10% and the tax so
deducted is the final tax, except for a company.
As a result thereof, in certain cases, where the taxable income falls into a higher tax brackets, the rate of tax on the
yield or profit on Behbood and Pensioners certificates or accounts is more than 10% as applicable on other profit on
debts.
Accordingly, if the proportionate income tax payable on profit on debt (yield or profit) from accounts exceeds 10%,
a reduction in income tax liability is allowed to the extent of such excess. As a result the tax on such profit on debt
(yield or profit) is restricted to 10%.

Example: Mr. Nasir has provided you following information for computation of taxable income and tax liability:
(a) Income from salary Rs. 300,000,
(b) Income from other sources Rs. 1,500,000
(c) Profit on Bahbood Saving certificates Rs.80,000
Solution:
Mr. Nasir
Computation of taxable income and tax thereon Rs.
Income from salary 300,000
Income from other sources 1,500,000
Profit on Bahbood Saving certificates 80,000
Taxable income 1,880,000
Taxpayer is a non-salaried person (salary income is less than 50% of taxable income).
Computation of tax liability
Tax on Rs. 1,880,000 [144,500 + 20% x (1,880,000 1,500,000)] 220,500
Less: tax credit (if any for senior citizen and full time teacher) 0
223,500
Reduction in respect of Behbood Saving Certificate (Note 1) 1,383
222,117
(Note 1) : Reduction in respect of Behbood Saving Certificate:
Proportionate tax on behbood saving certificate
Tax after tax reductions / taxable income x profit on Bahbood Saving certificates
(220,500 / 1,880,000 x 80,000) 9,383
Tax @ 10% [80,000 x 10%] 8,000
Excess tax (to be allowed as reduction in tax) 1,383
Important note: It in worthwhile to mention here that where the proportionate tax liability under NTR on Bahbood
Saving certificates is less than the amount computed on such profit at 10% then no tax reduction shall be computed
and the same reduced tax is to be paid by the taxpayer.

Conceptual Approach to Taxes 247


Tax Credits Chapter-15

20. Tax credit for tax already paid or deducted at source [Section 168(5)]
A person shall be allowed a tax credit for advance tax already paid or collected or deducted at source. If amount of
this tax credit is more than the tax liability, then excess shall be refunded to the taxpayer in accordance with section
170.
Master example covering all the tax credits:
Calculate tax liability of Mr. Hamid Sarfraz (66 years old, resident person) from following data available.

Rs.
Salary from Punjab University as full time teacher 600,000
Income from other sources (Taxable) 100,000
Taxable income from business in foreign country 100,000
Tax paid in foreign country in respect of above business income 20,000
Property income (after admissible deductions) 200,000
Zakat deducted at source 6,000
Zakat paid to relatives 40,000
Donation paid to approved charitable institution 12,000
Furniture donated to a Government Hospital with FMV 90,000
Shares acquired from privatization commission of Pakistan. 60,000
Share from AOP 150,000

Solution:
Pakistan source income: Rs. Rs.

Salary income 600,000


Income from other sources 100,000
Property income (after admissible deductions) 200,000

Income from business 100,000


Less: zakat deducted at source (6,000)
Taxable income 994,000
Share from AOP 150,000
Taxable income for rate purpose 1,144,000
As total salary income exceeds 50% of the total income, therefore the taxpayer is a salaried person for computation
of tax liability as under.

Computation of tax liability:

Tax on Rs. 1,144,000 [14,500 + 10% x (1,144,000 - 750,000)] 53,900


Total taxable income under NTR 1,144,000

Total tax liability under NTR as above 53,900

Less: foreign tax credit:


Lower of:
- Foreign tax paid 20,000
- Pakistan tax computed at average rate of tax
(53,900 / 1,144,000 x 100,000 ) 4,712
4,712
49,188
Less:
Rebate on donation
[(49,188 / 1,144,000 x (12,000 + 90,000)] 4,386
Rebate on shares and insurance
(49,188 / 1,144,000 x 60,000 ) 2,580

6,966
42,222
Tax payable on income excluding share from AOP (Rs.994,000 / 1,144,000 x 42,222) 36,686
Less: Senior Citizen Allowance @ 50% 18,343
Less: full time teacher allowance [(18,343 x 600,000 / 994,000) x 40%] 4,429
13,914

248 Conceptual Approach to Taxes


Tax Credits Chapter-15

21. Tax credit for certain persons [Section 100C]


(1) NPOs, trusts or welfare institutions as mentioned in sub section (2) shall be allowed a tax credit equal to
100% of the tax payable, including minimum tax and final taxes payable under any of the provisions of this
Ordinance, subject to the following conditions, namely;-
(a) Return has been filed
(b) Tax required to be deducted or collected has been deducted or collected and paid; and
(c) Withholding tax statements for the immediately preceding tax year have been filed.
(2) Persons eligible for tax credit under this section are same as were specified u/c (58), (58A), (59) and (60) of
Part I of 2nd Schedule to the Income tax Ordinance, 2001.

Conceptual Approach to Taxes 249


Tax Credits Chapter-15

MULTIPLE CHOICE QUESTIONS


Q.1. Foreign source income of a resident tax payer shall be chargeable to tax except.
(a) Salary income
(b) Property income
(c) Business income
(d) Capital gain
Q.2. Foreign tax credit is allowed for an amount equal to .
(a) Foreign income tax paid.
(b) Pakistan income tax payable.
(c) Lesser of a and b
(d) Higher of a and b.
Q.3. Foreign tax credit is allowed maximum up to _______ of income tax liability.
(a) 25%
(b) 50%
(c) 100%
(d) None of these
Q.4. Excess amount of foreign tax credit is only________________.
(a) Refunded.
(b) Carried Back.
(c) Carried forward.
(d) None of these.
Q.5. In order to avail the foreign tax credit the foreign income tax must be paid within _____ years after the year in which
income is earned.
(a) One year.
(b) Two years.
(c) Three years.
(d) Five years.
Q.6. Tax credit is allowed on__________ .
(a) Net foreign source income.
(b) Gross foreign source income.
(c) None of these.
Q.7. A person sustaining foreign source loss is allowed to ____ of such loss.
(a) set off against Pakistan source income
(b) set off against foreign source of income
(c) Both of these
Q.8. ________________tax credit is allowed before any other tax credit.
(a) Foreign tax credit.
(b) Tax credit for donations.
(c) Tax credit for tax already deposited.
Q.9. An individual making donation to a specified institution under clause 61is allowed a deduction on account of credit up
to ______ of taxable income.
(a) 15%
(b) 30%

250 Conceptual Approach to Taxes


Tax Credits Chapter-15

(c) 25%
(d) 0%
Q.10. Where any property is donated, then _______ of property is taken as value of donation.
(a) Cost
(b) Fair market value
(c) Any of these
(d) Higher of value recorded for capital value tax or Fair market value
Q.11. Donation to the Liaqat National Hospital Association, may be made up to______________.
(a) 50% of taxable income
(b) 95% of taxable income
(c) Total taxable income
(d) none of (a) to (c)
Q.12. A Tax Credit equal to _________ for donation made to unapproved institutions.
(a) Full amount of donations is allowed
(b) Half amount of donations is allowed
(c) Calculated at average rate of tax is allowed
(d) Not allowed
Q.13. A tax credit________ is allowed to a person making investment in the shares of a listed company (not being as first
allottee).
(a) Whole amount of donation
(b) At the average rate of tax
(c) Proportionate basis
(d) Not allowed
Q.14. A tax credit for investment in shares is made allowed for an investment which is,_____________.
(a) Total cost of shares
(b) 20% of taxable income
(c) Rs.1,000,000
(d) Lesser of a, b and c.
Q.15. Where a taxpayer disposes of the shares within ______ of the purchase, the tax liability of the person shall increase
by an amount equal to tax credit allowed to him in the year of disposal.
(a) 24 months
(b) 6 months
(c) 12 months
(d) None of these
Q.16. A tax credit at the average rate of tax shall be allowed to a person for contribution to approved pension fund if he
derives income from _________.
(a) Salary
(b) Business
(c) Both a and b
(d) None of these
Q.17. The total allowed contribution made to approved pension fund on the basis of age above 40 years on July 01, 2006
should not be more than _______ taxable income of the year.
(a) 10%
(b) 20%
(c) 30%
(d) 50%
Conceptual Approach to Taxes 251
Tax Credits Chapter-15

Q.18. Tax credit for enlistment of a company in any stock exchange in Pakistan shall be _____________of the tax payable
for the tax year in which the said Company is enlisted.
(a) 10%
(b) 15%
(c) 20%
(d) 25%
Q.19. Where tax liability is less than the tax credit of a person, who is also a member of an AOP and AOP having tax
payable then the excess amount would be_____________.
(a) Carried back
(b) Carried forward
(c) Claimed by the AOP as tax credit
(d) Claimed by the person as tax credit

ANSWERS
1 (a) 2 (c) 3 (c) 4 (d) 5 (b)
6 (a) 7 (b) 8 (a) 9 (b) 10 (b)
11 (d) 12 (d) 13 (d) 14 (d) 15 (a)
16 (c) 17 (d) 18 (c) 19 (c)

252 Conceptual Approach to Taxes


Tax Credits Chapter-15

ICMAP PAST PAPERS THEORECTICAL QUESTIONS


Q.NO.3 (a) August 2014 Discuss any two types of tax credits available under section 61, 62, 63 and 64 of the Income Tax
Ordinance, 2001.
Q.NO.3 (b) August 2014 Ms. Saleha has an investment of Rs. 600,000 in the Mutual Funds. Her employer assessed her
annual tax liability amounting Rs. 250,000 before allowance of any tax credit. However, her total taxable income is Rs.
2,500,000.
Required:
(i) What is the formula for calculation of tax credit for investment in shares?
(ii) Calculate the benefit of tax credit that can be availed by Ms. Saleha under the provision of the Income Tax
Ordinance, 2001.
(iii) Describe the condition where amount of tax payable, by the person for the tax year in which the
shares were disposed of, shall be increased by the amount of the credit allowed.

Q. No. 2 (b) Spring 2013 A company formed for establishing and operating a new industrial undertaking for
Manufacturing in Pakistan is allowed a tax credit equal to 100% of the tax payable on the taxable income arising from
such industrial undertaking for a period of five years from the date of setting up or commencement of commercial
production, whichever is later.
Required:
Specify the conditions which must be satisfied for availing the above tax credit.

Q. NO. 3 (d) SUMMER 2008 Define the types of tax credits available u/s 61 to Section 64 of the Income tax Ordinance,
2001.
Q. NO. 4 (b) SUMMER 2004 What are the requirements to avail Tax Credit on investment in shares by a person other than
a company u/s 62 of Income tax Ordinance, 2001? Also explain how tax credit is computed on acquisition and its treatment
on disposal of shares.

Conceptual Approach to Taxes 253


Tax Credits Chapter-15

CA MOD C PAST PAPERS THEORECTICAL QUESTIONS


Q. NO. 3 Spring 2014 Best Foods Private Limited (BFPL) is an industrial undertaking. It is considering to increase
its production capacity by installing a plant alongside its existing plant. The CFO of the company has informed the
Board that BFPL can claim tax credit equal to 100% of the tax payable on profits attributable to the new plant.
Required
Under the provisions of the Income Tax Ordinance, 2001 narrate the conditions that BFPL would need to comply with,
in order to be eligible to claim the above tax credit.
Q. 3 (c) Autumn 2012 Explain the provisions of the Income Tax Ordinance, 2001 pertaining to foreign tax credit available to
a resident taxpayer.
Q.3 (b) Spring 2012 A company formed for establishing and operating a new industrial undertaking for manufacturing in
Pakistan is allowed a tax credit equal to 100% of the tax payable on the taxable income arising from such industrial
undertaking for a period of five years.
Required: Narrate the conditions which must be satisfied for availing the above tax credit.
Q.NO. 2 Spring 2010 Mr. Qamar intends to donate an amount of Rs. 10 million to certain educational and welfare
institutions. In your capacity as his tax consultant, explain the tax relief which may be available in respect of such donation
and the conditions he must fulfill to avail such relief.
Q.NO. 5 (a) Spring 2008 Mr. Zulqarnain intends to make donations to certain charitable institutions. You are required to
advise him on the following:
(i) The types of institutions to whom the donation(s) would entitle him for tax credit.
(ii) The method of calculation of tax credit.
Q.NO. 4 (a) Spring 2007 Mr. Hamza intends to donate Rs. 5 million in cash to the following institutions:
An institution whose name is listed in the 2nd Schedule to the Income Tax Ordinance, 2001; and A non profit organization
working for the promotion of education in rural areas of Pakistan.
Explain the impact of the above donations on the tax liability of Mr. Hamza.
Q.NO. 6 Autumn 2005 Mr. Irfan intends to make a donation of Rs. 5 million in cash to certain institutions. Advise Mr. Irfan,
what tax benefits may be available to him and the conditions applicable thereon.
Q.NO.6 Autumn 2003 Under what circumstances a resident individual is entitled to claim exemption from tax on his foreign
source salary, and when is the foreign tax treated as having been paid?
Q.NO.6 (b)Autumn 2003 Explain the basis on which a foreign tax credit would be allowed to a resident taxpayer in respect
of foreign tax paid on foreign source income.
Q.NO.4 Summer 2002 Mr. Ashraf made the following donation during the income year:
a) Rs. 200,000 in cash to a relief fund sponsored by the Government.
b) personal car to an institution (approved). This car was purchased by Mr. Ashraf four year ago at the cost of the Rs.
80,000. The fair market value is Rs. 60,000.
c) Medicines to a private hospital (unapproved) purchased at the total cost of Rs. 10,000.
Advice Mr. Ashraf regarding the allowance for donation which may be claimed by him if his salary income is Rs. 800,000
after considering all admissible deductions.
Q.3 Spring 2002 Briefly explain the provisions of Income Tax Ordinance regarding the claim of:
a) Rebate for legal and educational expenditure incurred during the income year by an assessee.
b) Allowance for sum expended by an assessee on the purchase of books.
Q.4 Spring 2002 Mr. Ashraf made the following donations during the income year 2000-2001:
a) Rs. 200,000 in cash to a relief fund sponsored by the Government.
b) Personal car to an institution referred to in Clause (91) of the Second Schedule. This car was purchased by Mr.
Ashraf four years ago at the cost of Rs. 80,000. The fair market value is Rs.60,000
c) Medicines to a private hospital purchased at the total cost of Rs. 10,000.
Please advice Mr. Ashraf regarding the allowance for donation which may be claimed by him keeping in view the
requirement of Section 47 of the Income tax Ordinance, 1979 if his income for the relevant income year has been assessed
at Rs.800,000.

254 Conceptual Approach to Taxes


Common Rules Chapter-16

Chapter

16 COMMON RULES

Topic covered Topic covered


Section Rule Section
(For CAF-6 & ICMAP Students)
66 Liability in case of co-owners 72 Cessation of source of income
Rules to prevent double derivation & double
67 13 & 231 Apportionment of deductions 73
taxation
68 Fair market value 74 Tax year
69 Receipt of income
70 Recouped expenditure MCQs with solutions
71 Currency conversion CA Mod C past papers theoretical questions

(For CAF-6 AND ICMAP Students)


1. Liability in case of co-owners [Section 66]
(a) In case of co-ownership in property and the share of each co-owner is determinable then share received by each
co-owner from property shall be included in his total income but this principle will not apply to business income.
(b) If the share of co-owners is not defined even then the income shall be taxed on share of each co-owner in
the capital invested in the property.
(c) Signing amount from the tenant is taxable under the head "income from property". Signing means the amount paid
by the tenant to the owner to enter in the tenancy agreement which is neither refundable nor it can be termed as
deposit.
Example Mr. A and Mr. B are co-owners of a property. Their share in the property is equal. They received net income
from property of Rs. 400,000. Calculate the tax liability of both co-owners.
Solution:
Mr. A Mr. B
Income from property: Rs. Rs.
Share of Mr. A (400,000 x 50%) 200,000
Share of Mr. B (400,000 x 50%) 200,000
200,000 200,000
Computation of Tax liability:

Tax payable by Mr. A and Mr. B:


Tax on Rs. 200,000 nil nil
Example:
Mr. A and Mr. B are co-owners of a property. There share in the property is 60% and 40% respectively. They received
net income from property of Rs. 300,000. Calculate the tax liability of both co-owners.
Solution:
Mr. A Mr. B
Income from property: Rs. Rs.
Share of Mr. A (300,000 x 60%) 180,000
Share of Mr. B (300,000 x 40%) 120,000
180,000 120,000
Computation of Tax liability:
Income of Mr. A and B are below taxable limit hence not tax is
Payable by both the co-owners

Conceptual Approach to Taxes 255


Common Rules Chapter-16

Important notes:
It is important to mention here that if the share of each co-owner shall not be determinable then the same shall be
taxable in the ratio of capital invested in property.
Example
Mr. A and Mr. B carry on a business as partners. There share in the profit is 60% and 40% respectively. Taxable
income of business is Rs. 300,000.
Required: Calculate the tax liability of both Mr. A and Mr. B
Solution: As they are members of an AOP hence tax shall be paid by AOP.
Rs.
Taxable income of AOP under NTR 300,000
Tax liability of AOP
As income of AOP is below taxable limit, hence no tax is payable.
Note: This tax shall be paid by the AOP and members shall receive share of the divisible income i.e. Rs. 300,000.
This share received by the members shall be exempt from tax however the same shall be included in other income of
the members for rate purposes.
2. Apportionment of deductions (u/s 67): If an expenditure relates to
(a) derivation of incomes chargeable to tax under more than one head of income; or
(b) derivation of incomes chargeable to tax under Normal Tax Regime and Final Tax Regime or Separate Block of
Income; or
(c) derivation of income chargeable to tax and some other purpose,
then such an expenditure shall be apportioned on any reasonable basis taking account of the relevant nature and
size of the activities to which it relates. The board may make rules u/s 237 for the purposes of apportioning
deductions.
Apportionment of expenditures [Rule 13]
(a) Any expenditure that is incurred for a particular class or classes of income shall be allocated to that class or
classes.
(b) Any common expenditure excluding financial expenses relatable or attributable to non-business advances or
loans and the amount as stated above relatable to business including FTR and exempt income, shall be
allocated to each class of income according to the following formula, namely:-

Amount of common expenditure


X Gross receipts for the class of income
Gross receipts from all classes of income
(c) Where brokerage, commission and other income is to be taken into account on turnover of such transactions,
such income shall be compared with gross profit from business for apportionment of aforesaid common
expenditure.
(d) The basis used must be certified by the CAs or CMA that shall be accepted unless significant variations are
found and where books of accounts are not required to be audited, the reasonable basis based would be
accepted by the Commissioner Inland Revenue, unless variation is found. Significant variations would be
beyond the limits of 10% under any head of account.
(e) "Common expenditure" means expenditure that is not clearly allocable to any particular class or classes of
income, such as general administrative and other such allocable expenditures.
2.1 Computation of export profits attributable to export sales [U/R 231]
Where a taxpayer exports any goods manufactured in Pakistan, the taxpayer's profits attributable to export sales of
such goods shall be computed in the following manner namely:
(a) where a taxpayer maintains separate accounts of the business of exports of goods manufactured in Pakistan,
the profits of the export business shall be amount as may be determined by the Commissioner Inland Revenue
on the basis of such accounts; or
(b) in other cases, profits of such business shall be amount which bears to the total profits of the business of the
taxpayer from the sale of goods, the same proportion as the export sales of goods manufactured in Pakistan
bear to the total sales of goods.
In this rule:
(a) the expression "export sales" means the F.O.B. price of the goods exported; and

256 Conceptual Approach to Taxes


Common Rules Chapter-16

(b) the expression "total sales" means,-


i. the aggregate of exports sales as determined under clause (a);
ii. the ex-factory price of goods sold in Pakistan, where the goods exported out of Pakistan were
manufactured by the exporter; or
iii. the ex-godown price of goods sold in Pakistan, in other cases.
Example:
ABC Ltd is a listed company and provides you the following data to calculate the tax liability.
Rs.
Local sales 100,000
Exports 200,000
Cost of goods sold 50,000
Administrative expenses 40,000
Solution:
Local sales is chargeable to tax under NTR
Exports sales is chargeable to tax under FTR Rs. Rs. Rs.
NTR FTR Total

Sales 100,000 200,000 300,000


(percentage) 33.33 66.67 100.00

Cost of sales 16,667 33,333 50,000


Gross profit 83,333 166,667 250,000

Administrative expenses 13,333 26,666 40,000


Taxable income 70,000 140,000 210,000

Tax liability under NTR:


Tax on Rs. 70,000 @ 32% (A) 24,400

Turnover tax [100,000 x 1%] (B) 1,000


Higher of (A) or (B) 23,100

Tax liability under FTR:


Tax on export proceeds of Rs. 200,000 @ 1% 2,000
Total tax liability 25,100
3. Fair market value (Section 68):
(a) FMV of any property, rent, asset, service, benefit or perquisite at a particular time shall be the price which such
property, rent, asset, service, benefit or perquisite would ordinarily fetch on sale or supply in the open market
at that time.
(b) FMV shall be determined without regard to any restriction on transfer or to the fact that it is not convertible to
cash.
(c) FMV shall be determined by the Commissioner Inland Revenue where it is not ordinarily ascertainable.
Example:
Mr. Kaleem purchased a car from Bright Paint (AOP). The cost of the car is Rs. 750,000 in the books of Bright paint
whereas written down value of the car is Rs. 400,000. One of the car dealer told the company that the value of the
said car in the market is Rs. 375,000. Both parties are interested to know what would be the value of this car
according to tax rules.
Solution: Fair market value is Rs. 375,000.
4. Receipt of income (section 69):
A person shall be treated as having received an amount, benefit, or perquisite if it is
(a) actually received by him;
(b) applied on his behalf at his instruction or under any law; or
(c) made available to him.

Conceptual Approach to Taxes 257


Common Rules Chapter-16

Example: M/s. Nitro Chemicals has received the following information from his clients:
(a) Crook Ltd paid him a sum of Rs. 120,625 after deducting a tax of Rs. 4,375.
(b) IT international paid a sum of Rs.40,000 on behalf of Nitro to Cooker Man.
(c) Nitro Chemicals purchased medicines from Green pharmacy of Rs. 20,000 which are adjusted against the
balance of Green pharmacy and received the remaining amount of Rs.30,000.
You are required to record the receipts which are taxable for the tax year.
Client Taxable Receipt Comments
Crook Ltd. 125,000 Amount received and tax thereon.
IT International 40,000 Paid on behalf of company
Green pharmacy 50,000 Actual balance include Rs. 20,000
Total 215,000
5. Recouped expenditure (Section 70)
Where a person has been allowed a deduction for any expenditure or loss incurred in a tax year and subsequently he
receives in cash or in kind any amount in respect of such expenditure or loss then the amount so received shall be
included in the income chargeable under that head for the tax year in which it is received.
Example: Asif Health Business has incurred loss and unable to pay the financial charges of the bank and claimed
deduction of finance charges against the income of preceding years as follows:
Year 2015 Rs. 100,000 Year 2014 Rs. 85,000 Year 2013 Rs. 60,000
On 20 May 2016, the bank agreed to waive off mark up to the extent of Rs. 200,000 under a rescheduling agreement.
You are required to explain the tax exposure on such waiver of finance charges.
Solution: The waived amount of Rs 200,000 is taxable in the hands of Asif Health in the year of Waiver (2016) and
it will not be included in the preceding years.
6. Currency conversion (Section 71)
Every amount taken into account under this ordinance shall be in Rupees and if it is not in Pak rupees then this
amount shall be converted to the Rupee at the State Bank of Pakistan rate applying between the foreign currency and
the Rupee on the date the amount is taken into account for the ordinance.
Example: Alis Brother received an amount from his customer of US $ 750 on 01 April 2016, an expense incurred
in respect of this income on 02 February 2016 of 250 Euro. Exchange rate prescribed by the State Bank of Pakistan
are as under:

Date Currency Pak Rs conversion


01 April, 2016 Dollar RS. 83/Dollar
02 February, 2016 Euro RS. 122/Euro
Solution: Calculation of Income

Description Amount
$ 750 x Rs. 83 62,250
Euro 250 x Rs.122 (30,500)
Income 31, 750

7. Cessation of source of income (Section 72)


Where any income is derived by a person from any business, activity, investment or other source that has ceased
either before the commencement of the year or during the year then any income derived before the cessation of
source of such income shall be chargeable to tax on the basis that source had not ceased at the time the income was
derived.
8. Rules to prevent double derivation and double taxation (Section 73)
Where any amount is chargeable to tax on the basis that it is receivable then such an amount shall not be chargeable
again on the basis that it is received and vice versa.
Similarly where any expenditure is deductible on the basis that it is payable then such an expenditure shall not be
deductible again on the basis that it is paid and vice versa.
9. Tax year (section 74):
Tax year is of three types, Normal tax year, Special tax year and Transitional tax year.

258 Conceptual Approach to Taxes


Common Rules Chapter-16

9.1 Normal tax year:


A period of 12 months from 1 July to 30 June denoted by the calendar year in which the normal tax year ends. For the
year ending 30 June 2016 the tax year shall be 2016.
9.2 Special tax year:
Any income year ending other than 30th June is special tax year and denoted by the calendar year relevant to the
normal tax year in which the year-end falls.
The Board has authority to prescribe any special tax year in respect of any particular class of taxpayers.
If the tax year is not specified by the Board and a taxpayer wants to have any special tax year then he is required to
make an application to the Board specifying the reason for the purpose.
In respect of certain classes of income, following special income years are specified by the Board. The list of such
specified special income year is as under.

All companies except given below Tax year 1st July to 30th June 133 (R)/68, 31-07-1968

Notification No. (SRO) and


Classes of Taxpayer Special tax year
Date
Companies manufacturing sugar 1st Oct to 30th September 134(R)/68, 31-7-1968
st st
All persons exporting rice 1 January to 31 December 367(I) /74, 14-01-1974
st st
All persons carrying business of rice husking 1 September to 31 August 1153 (I) /79, 10-12-1979
All persons carrying on business of 1st April to 31st March 505 (I) /80, 13-05-1980
manufacturing and dealing in shawls
All insurance companies 1st January to 31st December 878 (I) /95, 30-08-1995
9.3 Transitional tax year:
If a normal tax year or special tax year changes then the period from the day next following the last full tax year to the
date of commencement of new tax year shall be treated as transitional tax year.
Example:
You are required to determine the tax year in the following cases:
1. XYZ Textiles Ltd. year end on 30 September of each year. Determine the tax year for the income year starts
on 1 October 2014 and ends on 30 September, 2015.
2. Ahmed Ltd. is a subsidiary of Don Limited. The company is granted special year end of 30 November. You are
required to determine the tax year of its income year 1 December, 2014 to 30 November, 2015.
Solution:
In both of cases, the tax year is 2016 as the next year end June falls within the calendar year of 2016.
Example:
The Normal income year of Azad (Pvt.) limited is 30 June, 2016. The company however sought permission to adopt
special income year of 30 September of each year. The concerned Commissioner Inland Revenue has granted
approval to the said company to adopt special year of 30 September. Determine the Normal, transitional and special
tax year?
Solution:
(a) Normal tax year will be 2016 for the income year relevant to the 1 July 2015 to 30 June, 2016.
(b) Transitional tax year will be 2016 for the period relevant to the period 1 July 2015 to 30 September, 2015.
(c) Special tax year will be 2016 for the income year 1 October, 2014 to 30 September, 2015.
9.4 Change in the tax year:
If a person wants to change the tax year then he shall apply to the Commissioner Inland Revenue. The
Commissioner Inland Revenue shall grant permission subject to condition, if any, only if the person has shown a
compelling need for the change;
If the application is rejected, the Commissioner Inland Revenue shall provide an opportunity of being heard to the
person and shall record in the order the reasons for such rejection. In this case the person may file a review
application to the Board and the decision of the Board shall be final.

Conceptual Approach to Taxes 259


Common Rules Chapter-16

MULTIPLE CHOICE QUESTIONS


Q.1. In case of co-ownership in property and the share of each partner is determinable then share received by each
partner from property shall be ________ his total income.
(a) excluded from
(b) included in
(c) divided by
(d) none of above
Q.2. Signing amount from the tenant is taxable under the head ________.
(a) property income
(b) income from other sources
(c) income from salary
(d) income from business
Q.3. If expenditure relates to derivation of incomes chargeable to tax under more than one head of income then such
expenditure shall be _______ on any reasonable basis taking account of the relevant nature and size of the activities
to which it relates.
(a) charged to tax
(b) excluded from taxable income
(c) apportioned
(d) none of above
Q.4. _______ means expenditure that is not clearly allocable to any particular class or classes of income, such as general
administrative and other such allocable expenditures.
(a) total expenditure
(b) capital expenditure
(c) overall expenditure
(d) common expenditure
Q.5. Export sales" means the _____ price of the goods exported
(a) C.I.F
(b) F.O.B
(c) Net
(d) Gross
Q.6. Total sales means ____________.
(a) export sales
(b) ex-factory price of goods sold in Pakistan, where the goods exported out of Pakistan were manufactured by
the exporter
(c) ex-godown price of goods sold in Pakistan, in other cases
(d) all of above
Q.7. FMV of any property, rent, asset, service, benefit or perquisite at a particular time shall be the price which such
property, rent, asset, service, benefit or perquisite would ordinarily fetch on sale or supply in the______ at that time.
(a) money market
(b) open market
(c) fair market
(d) share market
Q.8. Fair Market Value shall be determined without regard to any restriction on transfer or to the fact that it is not
convertible to __________.
(a) capital
(b) Net Realisable Value
(c) cost
(d) cash
Q.9. Fair Market Value shall be determined by the _________________where it is not ordinarily ascertainable.
(a) Appellant Tribunal Inland Revenue
(b) Commissioner Inland Revenue

260 Conceptual Approach to Taxes


Common Rules Chapter-16

(c) Board
(d) None of above
Q.10. A person shall be treated as having received an amount, benefit, or perquisite if it is __________.
(a) Actually received by him;
(b) applied on his behalf at his instruction or under any law;
(c) made available to him.
(d) all of above
Q.11. Where a person has been allowed a deduction for any expenditure or loss incurred in a tax year and subsequently he
receives in cash or in kind any amount in respect of such expenditure or loss then the amount so received shall
be___________.
(a) exempt
(b) ignored
(c) included in the income chargeable under that head for the tax year in which it is received
(d) none of above
Q.12. Every amount taken into account under this ordinance shall be in Rupees and if it is not in Pak rupees then this
amount shall be converted to the Rupee at the _________ rate applying between the foreign currency and the Rupee
on the date the amount is taken into account.
(a) State Bank of Pakistan
(b) KIBOR
(c) Exchange rate
(d) all of above
Q.13. Normal tax years means a period of 12 months from 1 July to 30 June denoted by the calendar year in which the
_______ ends.
(a) financial year
(b) tax year
(c) assessment year
(d) normal tax year
Q.14. The ________ has authority to prescribe any special tax year in respect of any particular class of taxpayers.
(a) Board
(b) Commissioner Inland Revenue
(c) Chief Commissioner Inland Revenue
(d) Appellate Tribunal Inland Revenue
Q.15. If a normal tax year or special tax year changes then the period from the day next following the last full tax year to the
date of commencement of new tax year shall be treated as _________.
(a) normal tax year
(b) special tax year
(c) transitional tax year
(d) none of above
Q.16. If a person wants to change the tax year then he shall apply to the _________.
(a) Board
(b) Commissioner Inland Revenue
(c) Chief Commissioner Inland Revenue
(d) Appellate Tribunal Inland Revenue

ANSWERS
1 (b) 2 (a) 3 (c) 4 (d) 5 (b)
6 (d) 7 (b) 8 (d) 9 (b) 10 (d)
11 (c) 12 (a) 13 (d) 14 (a) 15 (c)
16 (b)

Conceptual Approach to Taxes 261


Common Rules Chapter-16

ICMAP PAST PAPERS THEORETICAL QUESTIONS


Q. NO. 8(b) Summer 2005 Elaborate the procedure for applying and grant of approval for change of year from normal tax
year to special tax year.

262 Conceptual Approach to Taxes


Common Rules Chapter-16

CA MOD C PAST PAPERS THEORETICAL QUESTIONS


Q.5 (b) Autumn 2013 One of your clients Inqalab Limited wants to change its accounting year.

Required:
Write a brief note to the Finance Manager of the company explaining the requirements of Income Tax Ordinance, 2001 as
regards the following:
(i) Change in tax year
(ii) Determination of tax year and the date of filing of return in case the accounting year-end is changed from June to
December.
Q.4 (b) Autumn 2011 Explain the following as specified in the Income Tax Ordinance, 2001.
(ii) Fair Market Value (iii) Apportionment of Expenditures
Q.2(b) Sept 2007: One of your client which is a subsidiary of a foreign company wants to change Its accounting year from
June 30 to December 31 as the income year of its parent company ends on December 31, Advise the client about the
requirements of the ITO, 2001 regarding change in tax year from normal to special.
Q.2 (a) March 2005 Discuss the common rules with regard to the Fair Market Value
Q. 2 Spring 2005 Discuss the common rules with regard to the following under the Income Tax Ordinance, 2001:
(a) Fair Market Value (FMV)
(b) Income of Joint owners.
(c) Non-arms length transactions of disposal of assets.
Q. 2 Autumn 2004 Describe the common rules.
(a) Receipt of income
(b) Currency conversion

Conceptual Approach to Taxes 263


Common Rules Chapter-16

264 Conceptual Approach to Taxes


Returns and Assessments Chapter-17

Chapter

17 RETURNS AND ASSESSMENTS

Topic covered
Section Rule
(For CAF-6 AND ICMAP students)
114 34 Who should file the return of income
Requirements to file a return
Procedure to file the return
Special cases where less than twelve months return is required
Filing of return on demand
Conditions to revise return
115 35 & 39 Persons not required to furnish a return of income
Revision of statement in case of final taxation
116 36 Commissioner Inland Revenue power to require statement of final tax
Commissioner Inland Revenue power to require wealth statement
Wealth statement
Wealth statement for cases fall in final tax regime
117 Income tax requirements for discontinued business
118 73 Method to furnish the return
73 Persons required to file return / statements electronically
Due date for filing of return
119 Extension of time for furnishing returns and other documents
120 Status of complete return under the ordinance
Status of return not complete under the ordinance
121 Best judgment assessment
122 Amendment of assessments
122A Revision by commissioner inland revenue
122B Revision by Chief Commissioner
122C Provisional assessment
123 Provisional assessment in certain cases
124 Assessment giving effect to an order
124A Powers of tax authorities to modify orders
125 Assessment in relation to disputed property
126 Evidence of assessment
145 221, 222 & 223 Assessment of persons about to leave Pakistan
177 Audit by Commissioner Inland Revenue
181C Displaying of National Tax Number
221 Rectification of mistakes
Different tax strategies
MCQS with solutions
ICMAP and CA mod C past papers theoretical questions

Conceptual Approach to Taxes 265


Returns and Assessments Chapter-17

For CAF-6 & ICMAP students)


In this chapter
CIR stands for Commissioner Inland Revenue
Assessment procedure
After finalization of accounts the taxpayer prepares and files income tax return with the tax department by manual or on E.
portal along-with required documents. After filing of income tax return the tax department selects the cases for total audit or
for amendment in assessment. The said process is named as assessment process.
1. Who should file the return of income [Section 114(1)]
The following persons are required to furnish a return of income for a tax year, namely:-

(a) every company, owns immoveable property with a land area of 500 square
yards or more located in a rating area;
(b) every person (other than a company) whose owns a flat having covered area of 2,000 square feet or
taxable income for the year exceeds the more located in a rating area;
maximum amount not chargeable to tax,
(c) any non-profit organization, and owns a motor vehicle having engine capacity above
1000CC; and
(d) any welfare institution approved, has obtained National Tax Number.
(e) any person not covered in the above clauses is the holder of commercial or industrial connection of
who, electricity where the amount of annual bill exceeds
Rs.500,000.
has been charged to tax in respect of any of the is a resident person registered with any Chamber of
two preceding tax years; Commerce and Industry or any trade or business
association or any market committee or any professional
body including Pakistan Engineering Council, Pakistan
Medical and Dental Council, Institute of Chartered
Accountants of Pakistan or Institute of Cost and
Management Accountants of Pakistan.
claims a loss carried forward for a tax year; (1A) Every individual whose annual income under the head
Income from business exceeds Rs.300,000 and others
with annual income of Rs.400,000 in a tax year are also
required to furnish return of income for the tax year.
owns immovable property with a land area of
250 square yards or more or owns any flat
located in areas falling within the municipal
existing immediately before the commencement
of Local Government laws in the provinces; or
areas in a Cantonment or the Islamabad
Capital Territory;
Rating area: In this section rating area has not been defined but its means urban area where provinces tax applies.
Example: Briefly explain which of the following persons are required to file a return of income for the tax year 2016:
a) ABC (Pvt.) Ltd. was incorporated on July 02, 2012 and there was no activity during tax year 2016.
b) XYZ Ltd was incorporated in tax year 2015 and incurred loss during the tax year.
c) C Ltd was incorporated in tax year 2014 and taxable income of the company during the year was Rs. 454,000.
d) Mr. Ikram (NTN holder) is a salaried individual and his salary income during the tax year is below taxable limit.
e) Mr. Jamal is a non-salaried individual, his business income during the tax year is below taxable limit. He has not
obtained NTN certificate.
f) An individual owns an 800 CC motor vehicle.
g) Mr. Amir incurred loss in his business during the tax year. He wants to carry forward this loss.
h) Mr. Zahid (NTN holder) is 75 years old. He is living with his son and has no source of income.
i) Mr. Khan owns a property with the land area 550 square yards located in a non rating area.
j) Non-profit organisation or welfare institution.
k) Mr. Shahzad owns a flat having covered area 1900 square feet located in a rating area.

266 Conceptual Approach to Taxes


Returns and Assessments Chapter-17

Solution:
1. In all three cases from (a) to (c) company is required to file return of income irrespective of its commencement
of business of profit or loss.
2. In case of (d) as Mr. Ikram has obtained NTN certificate so he is required to file return of income irrespective of
his income.
3. In case of (e) Mr. Jamal is not required to file return of income as his income is below taxable limit and he has
not obtained NTN certificate.
4. In case of (f) individual is not required to file return as engine capacity of the vehicle owned by him is not
exceeding 1000 CC.
5. In case of (g) Mr. Amir is required to file return of income as he is claiming loss to be carried forward.
6. In case of (h) Mr. Zahid is required to file return of income as he is NTN holder.
7. In case of (i) although area of the land is more than 500 square yards however the same is not in the rating
area therefore Mr. Khan is not required to file return of income.
8. In case of (j) every non-profit organisation and welfare institution is required to file return of income.
9. In case of (k) Mr. Shahzad is not required to file return of income as covered area of flat owned by him is less
than 2000 square feet.
1.1 Requirements to file a return [Section 114(2)]
A return of income
shall be in the prescribed form and shall be accompanied by annexures, statements or documents,
shall fully state all the relevant particulars or information specified in return form and declaration of the records
kept by the taxpayer; and
shall be signed by the person or the person's representative, where applicable.
shall be accompanied with due payment of tax due as per return of income; and
shall be accompanied with a wealth statement.
1.2 Furnishing of return of income [Rule 34]
A verified return of income u/s 114 shall be furnished in the form as specified in Annexure-XIII of Part VI of the
Second Schedule to Income Tax Rules, 2002 and accompanied by applicable documents, statements,
certificates, annexes; and
In case of companies, the return of income shall be accompanied by audited accounts and reconciliation of
profits as per accounts and taxable income as per return.
1.3 Procedure to file the return [Section 114(2A)]
A return of income filed electronically or through any magnetic media or any other computer readable media
specified by the Board shall also be deemed to be a return.
The Board by notification in the official Gazette may make rules for determining eligibility of the data of such
returns and e-intermediaries that will digitize the data of such returns, statements or other documents etc. to
transmit the same electronically under their digital signatures.
1.4 Special cases where less than twelve months return is required [U/S 114(3)]
The CIR may by notice in writing require from a person to furnish a return of income for a period of less than twelve
months, where the person has died, become bankrupt or gone into liquidation, permanently to leave Pakistan or
where considers appropriate that the return should be furnished.
1.5 Filing of return on demand [U/S 114(4) and (5)]
The CIR may by notice in writing require any person to file the income tax return who in the CIR opinion is required to
file the return within 30 days from the date of service of such notice or such longer or shorter period as may be
specified in notice or allowed by the CIR for one or more of the last five completed tax years or assessment years.
1.6 Conditions to revise return [U/S 114(6) (6A)]
Any person after filing of return discovers any omission or wrong statement therein may revise return subject to the
following conditions:
it is accompanied by the revised accounts or revised audited accounts as the case may be; and
the reasons for revision of return, in writing, duly signed, by the taxpayers are filed with the return.

Conceptual Approach to Taxes 267


Returns and Assessments Chapter-17

it is accompanied by approval of the Commissioner in writing for revision of return; and


Taxable income declared is not less than and loss declared is not more than income or loss, as the case may
be, determined by an order issued u/s 121, 122, 122A, 122C, 129, 132, 133 or 221:
If any of the above conditions is not fulfilled, the return furnished shall be treated as an invalid return as if it had not
been furnished.
Provided further that the condition for approval of the Commissioner shall not apply, if revised return is filed
within 60 days of filing of return.
In cases where approval of Commissioner is required for revision and no such order has been made within sixty
days, approval shall be deemed to have been granted and the condition for approval of the Commissioner
under section 114(6)(ab) shall not apply.
Where a taxpayer files a revised return voluntarily and deposits the amount of tax evaded or short paid, along with the
default surcharge. The penalty status shall be as under:

Before receipt of notice u/s 177 or section 122(9) No penalty shall be recovered from him.
During the audit or before the issue of notice u/s 122(9) 25% of the penalties shall be recovered from him.
After the issue of show cause notice u/s 122(9) 50% of the penalties shall be recovered from him.

2. Persons not required to furnish a return of income [Section 115]


The following persons shall not be required to furnish a return of income for a tax year solely by reason of
ownership of land or flat within municipal limits or Islamabad capital territory: [Section 115(3)]
A widow;
an orphan below the age of 25 years
a disabled person or
a non-resident person.
Any person who is not obliged to furnish a return for a tax year where all the persons income is subject to final
taxation shall file a statement showing particulars of income for the tax year in such form and duly verified as
required. [Section 115(4A)]
2.1 Statement in lieu of return of income [Rule 39]
Where in lieu of Return of income statement is required to be filed namely incomes covered under Separate Block of
Income (treated as Final discharge) and final discharge of tax liability u/s 169 a statement in the prescribed form shall
be filed as prescribed in Part IV of the Second Schedule to the Income Tax Rules, 2002.
Where a taxpayer has income from a source which does not form part of total income and also income under any
head of income given in section 11 (except salary) is also required to file a prescribed statement.
Example: Briefly explain which of the following persons are required to file a return of income for the tax year 2016:
(a) A widow having property with land area of 600 square yards in rating area.
(b) An orphan of age 24 having salary income of Rs. 500,000.
(c) A disabled person receiving income from property of Rs.700,000.
(d) A non-resident having flat with covered area of 2500 square feet in rating area. He has no other source of
income.
Solution:
(a) Widow is not required to file return of income solely by the reason of owning immovable property.
(b) As the orphan is earning salary income more than the maximum non-taxable limit therefore he is required to
file the return of income.
(c) As the disabled person is earning income from property more than Rs.150,000 therefore he is required to file
the return of income.
(d) A non-resident person is not required to file return of income solely by the reason of owning immovable
property.
2.2 Revision of statement in case of final taxation [Section 115(4A)]
Any person after filing of statement of final taxation discovers any omission or wrong statement therein may furnish a
revised statement for that tax year at any time within five years from the end of the financial year in which the original
statement was furnished.

268 Conceptual Approach to Taxes


Returns and Assessments Chapter-17

Example: Which of the following statements of final taxation can be revised in tax year 2016?
1. Statement filed in tax year 2009 2. Statement filed in tax year 2013 3. Statement filed in tax year 2014
Solution: Statements filed in tax year 2013 and 2014 can be revised in tax year 2016.
Commissioner inland revenue's power to require statement of final tax [Section 115(5) and (6)]
The CIR may by notice in writing require any person to file the statement of final taxation who in the CIR opinion is
required to file the same within 30 days from the date of service of such notice or such longer or shorter period as
may be specified in notice or allowed by the Commissioner Inland Revenue for one or more of the last five
completed tax years or assessment years.
3. Commissioner Inland Revenue power to require wealth statement [Section 116(1)]
The CIR may require a wealth statement by notice in writing from any person being an individual on the compliance
date specified in notice and on such form and duly verified by stating therein particular's as on the date or dates
stated in the notice of:-
the person's total assets and liabilities;
the total assets and liabilities of the person's all the dependents;
any assets transferred by the person to any other person during the period or periods and the consideration for
the transfer;
Detail of total expenditures incurred by the person and all the dependents during the period or periods; and
the wealth reconciliation statement,
3.1 Wealth statement [Section 116]
Every resident individual taxpayer (including member of an AOP) liable to file a return of income for any tax year for
the tax year shall furnish a wealth statement and wealth reconciliation statement for that year along-with such return.
[Section 116(2)]
A person (including members of an AOP as stated above) on filing of return in response to a provisional assessment
u/s 122C shall furnish a wealth statement accompanied by wealth reconciliation statement and an explanation of
sources of acquisition of assets. [Section 116(2A)]
Where a person after filing of wealth statement discovers any omission or wrong statement therein may furnish a
revised wealth statement along with the revised wealth reconciliation and the reasons for filing revised wealth
statement, at any time before amendment in assessment for the tax year to which it relates is made. [Section
116(3)]
Example: Mr. Arif having declared income of Rs. 400,000 is required by the Commissioner Inland Revenue Inland
Revenue to file wealth statement for tax year 2016. Explain under the ITO, 2001 whether Mr. Arif is legally under
obligation to file the wealth statement or not.
Solution:
Now every individual is required to file the wealth statement along-with the income tax return however the CIR is
empowered to require any individual taxpayer to furnish a wealth statement. Hence, Mr. Arif is under obligation to file
the wealth statement on the demand of the Commissioner Inland Revenue.
3.2 Wealth statement for cases fall in final tax regime [Section 116(4)]
Every person (other than a company or an association of persons) filing statement covered under final tax regime
shall file a wealth statement along-with wealth reconciliation statement.
3.3 Furnishing of wealth statement [Rule 36]
A verified wealth statement shall be in the form specified in Part IV of the Second Schedule to the Income Tax Rules,
2002, accompanied by such documents, statements and certificates required under the Ordinance / Rules along-with
a wealth reconciliation statement.
4. Income tax requirements for discontinued business [Section 117]
Any person discontinuing a business shall give notice in writing to the CIR within 15 days of such effect. [Section
117(1)]
The person discontinuing the business shall by himself or so required by CIR by notice in writing furnish a return of
income for the period started from the first day of the tax year in which business is discontinued and ending on
the date of discontinuance and this period shall be treated as a separate tax year. [Section 117(2)]
Where no notice has been given by the taxpayer for discontinuance of business but the CIR has reasonable grounds
that a business has discontinued or is likely to discontinue, a notice may be served to such person to furnish within
specified time a return of income for such period that shall be treated as a return of income for all purposes. [Section
117(3) (4)]

Conceptual Approach to Taxes 269


Returns and Assessments Chapter-17

5. Method to furnish the return rule-73 and [Section 118]


A return of income, a statement for incomes covered under final tax regime or a wealth statement shall be furnished
by almost all the persons on E. portal except where allowed to file by post or courier service or delivery by hand.
[Section 118(1)]
5.1 Persons required to file return / statements electronically [Rule 73]
Following persons are required to file return / statements electronically.

Documents to be filed electronically


1 Companies and AOPs Return of income and withholding tax statements
2 A person registered under the Sales tax Act Return of income
3 An individual having taxable salary income Return of income, proof of tax deduction / payment and
wealth statement along with its reconciliation.
4 Federal government departments Withholding tax statements
5 Non-resident ship owner or non-resident aircraft Return of income and port clearance application
owner or charterer thereof
6 A non-corporate entity claiming refund Return of income
7 Any person claiming refund Application of refund
5.2 Due date for filing of return [Section 118, and 114(4) and (5)]
It is important to note that when to furnish the return of income, the return of income shall be furnished according to
the following dates:
The return of total income shall be filed by a company with tax year ended on or between first January and
30th day of June is up till 31st Dec next following the tax year and in any other case (other than the under
mentioned paragraph) of Company, individual or AOP up to 30th September next following the tax year.
[Section 118(2)]
All individuals earning salary income shall file return of income electronically in the prescribed form and it shall
be accompanied by the proof of deduction or payment of tax and wealth statement as required under section
116. Provided that the Board may amend the conditions specified in this sub section or direct that the
said condition shall not apply for a tax year. [Section 118(2A) read with SRO 791(I)/2015 dated 10-08-
2015]
The filing of return of income by salaried persons through E. portal and statement u/s 115(4) are to be filed
on or before August 31 next following the end of the Tax year to which it relates. [Section 118(3)]
The due date for filing of wealth statement shall be the same as for the filing of income tax return or
statement. [Section 118(4)]
Following is the chart of date of filing of return with wealth statement where applicable for different taxpayers having
different income years:
Year End Year End Year End Year End
STATUS TYPE OF INCOME 30-09-15 31-12-15 31-03-16 30-06-16
Last date of filing of
Only salary income 31-08-16 31-08-16 31-08-16 31-08-16
return
Individual
Last date of filing of
Non-salaried 30-09-16 30-09-16 30-09-16 30-09-16
return
Last date of filing of
Final Tax Regime (Only) 31-08-16 31-08-16 31-08-16 31-08-16
return
Final Tax Regime and Last date of filing of
AOP 30-09-16 30-09-16 30-09-16 30-09-16
Normal Tax regime return
Last date of filing of
Normal Tax Regime 30-09-16 30-09-16 30-09-16 30-09-16
return
Normal Tax Regime or Final Last date of filing of
Company 30-09-16 30-09-16 31-12-16 31-12-16
Tax Regime return

A return furnished after discontinuance of business shall be furnished by the due date specified in the notice.
[Section 118(5)]
The CIR may by notice in writing require any person to file the income tax return who in the CIR opinion is
required to file the return within 30 days from the date of service of such notice or such longer or shorter
period as may be specified in notice or allowed by the CIR for one or more of the last five completed tax
years or assessment years. [Section 114(4) and (5)]
Where a taxpayer is not on the National Tax Number Register and fails to file an application for National Tax
Number with the taxpayer's return of income, such return shall not be treated as a return for income tax
purposes. [Section 118(6)]

270 Conceptual Approach to Taxes


Returns and Assessments Chapter-17

6. Extension of time for furnishing returns and other documents [Section 119(3)]
If a person is required to furnish a return of income, a statement of final tax regime or a wealth statement then the
person may apply in writing to the CIR for an extension of time by the due date to furnish the same. [Section 119(1)
and (2)]
On receipt of application if CIR is satisfied that the applicant is unable to furnish the return of income and other allied
documents by the due date because of absence from Pakistan, sickness or other misadventure or any other
reasonable cause the CIR may allow in writing an extension of time for a period not exceed 15 days from the due
date of furnishing unless there are exceptional circumstances justifying a longer extension of time. [Section 119(3)
and (4)]
An extension of time granted to file the return and other documents shall not be deemed as allowed for tax
payable with the return and default surcharge shall be charged till the payment of the same. [Section 119(6)]
Example: A and B Co is an AOP having year end 30-06-2016. However on the last date of filing of income tax return
the information was not complete to file the income tax return. Now what option is available with the AOP to avoid
from the penalty of late filing of income tax return?
Solution: U/s 119 the AOP is having the option to file a request for extension regarding filing of income tax return on
the last date to file the income tax return.
7. Status of complete return under the ordinance [Section 120(1), (1A) and (2)]
Where a taxpayer has furnished a complete return of income other than a revised return for a tax year ending on
or after 1.7.2002 and the return shall be taken for all purposes to be an assessment order issued to the taxpayer
by the CIR on the day the return was furnished. However the Commissioner Inland Revenue may conduct audit of
the income tax affairs of a person u/s 177.
7.1 A return of income shall be taken to be complete on fulfilment of the following conditions:
A return of income
shall be in the prescribed form and shall be accompanied by annexures, statements or documents,
shall fully state all the relevant particulars or information specified in return form and declaration of the records
kept by the taxpayer; and
shall be signed by the person or the person's representative, where applicable.
Important note: A complete return filed by a taxpayer will be considered an assessment order as stated above and
the said assessment may be amended u/s 177 / 122 of the ITO, 2001. Even where the case of a taxpayer has been
selected for total audit u/s 177 or for amendment in assessment u/s 122 the loss already assessed u/s 120 may be
carried forward unless the same is not amended through an order under the aforesaid sections.
7.2 Status of return not complete under the ordinance [Section 120(3), (4), (5) and (6)]
Where the return of income furnished is not complete, the Commissioner Inland Revenue shall issue a notice
to the taxpayer informing him of the deficiencies (other than incorrect amount of tax payable computed or
short payment of tax payable) and directing him to provide such information by such date specified in the
notice.
Where a taxpayer fails to fully comply the notice by the due date, the return furnished shall be treated as an
invalid return.
Where in response to a notice the taxpayer has by the due date fully complied with the requirements of the
notice the return furnished shall be treated to be complete on the day it was furnished.
No notice of short documents shall be issued after the expiry of 180 days from the end of the financial
year in which return was furnished
8. Best judgment assessment [Section 121]
Where a person fails to:
furnish a return on receipt of notice or a return being as non-resident ship or aircraft owner or wealth statement to
produce before the CIR or a special audit penal appointed under section 177(11) or any person employed, accounts,
documents and records required to be maintained or required for the purpose of making assessment.
The CIR may, based on any available information or material and to the best of his judgement shall make
assessment of such person and the assessment, if any, treated to have been made on the basis of return or
revised return filed by the taxpayer shall be of no legal effect.
As soon as possible after making an assessment, the CIR shall issue the assessment order to the taxpayer
stating the taxable income, the amount of tax due, the amount of tax paid, if any and the time, place and
manner of appealing the assessment order.
An assessment order as stated above shall only be issued within five years after the end of the tax year or
the income year to which it relates.

Conceptual Approach to Taxes 271


Returns and Assessments Chapter-17

9. Amendment of assessments [Section 122]


The CIR may amend an assessment order treated as issued u/s 120 or issued u/s 121 or 122C by making
such alterations or additions as the CIR considers necessary. However no order shall be amended by the CIR
after the expiry of five years from the end of the financial year in which the CIR has issued the assessment
order to the taxpayer.
Where a taxpayer furnishes a revised return the Commissioner Inland Revenue shall treat as having made an
amended assessment of the revised return on the day on which the revised return was furnished.
Where an original assessment order has been amended the CIR may, after making, or causing to be made,
such enquiries as he deems necessary, further amend, as many times as may be necessary, the original
assessment within the later of -
(a) five years from the end of the financial year in which the CIR has issued the original assessment order;
or
(b) one year from the end of the financial year in which the CIR has issued the amended assessment order
to the taxpayer.
After the first amendment in the assessment order, the same shall only be further amended on the basis of
definite information, the CIR is satisfied that the person income has escaped assessment or under-assessed
or assessed at too low a rate that has resulted into prejudicial to the interest of revenue.
As soon as possible after making an amended assessment, the CIR shall issue an amended assessment
order by stating taxable income, amount of tax due, amount of tax paid and the time, place, and manner of
appealing the amended assessment.
No assessment shall be amended, or further amended, unless the taxpayer has been provided with an
opportunity of being heard.
10. Revision by the commissioner inland revenue [Section 122A]
The CIR may, suo moto, call for the record of any proceeding or under the repealed Ordinance in which an
order has been passed by any Officer of Inland Revenue other than the CIR (Appeals).
Where, after making such inquiry as is necessary, CIR considers that the order requires revision, the CIR may
make such revision to the order as the CIR deems fit.
The above order shall not be prejudicial to the person to whom the order relates.
The CIR shall not revise any order as above if:
(a) an appeal against the order lies to the CIR (Appeals) or to the Appellate Tribunal Inland Revenue, the
time within which such appeal may be made has not expired; or
(b) the order is pending in appeal before the Commissioner Inland Revenue (Appeals) or has been made
the subject of an appeal to the Appellate Tribunal Inland Revenue.
11. Revision by the Chief commissioner inland revenue [Section 122B]
The Chief Commissioner Inland Revenue may, either of his own motion or on an application made by the
taxpayer for revision, call for the record of any proceedings relating to issuance of an exemption or lower rate
certificate with regard to collection or deduction of tax at source, in which an order has been passed by any
authority subordinate to him.
Where, after making such inquiry as is necessary, Chief Commissioner Inland Revenue considers that the
order requires revision, the Chief Commissioner Inland Revenue may, after providing reasonable opportunity
of being heard to the taxpayer, make such order as he may deem fit in the circumstances of the case.
12. Provisional assessment [Section 122C]
Where in response to a notice a person fails to furnish return of income for any tax year, the CIR may, based on any
available information or material and to the best of his judgment, make a provisional assessment and issue a
provisional assessment order specifying the taxable income assessed and the tax due thereon.
The provisional assessment order shall be treated as the final assessment order after the expiry of forty five
days from the date of service of order of provisional assessment.
The provisional assessment order shall be vacated:
- In case of individual or AOP:
If return of income, wealth statement, wealth reconciliation statement and other required documents are filed
by individual or an AOP within 45 days.

272 Conceptual Approach to Taxes


Returns and Assessments Chapter-17

- In case of company:
If return of income along with audited accounts or final accounts, as the case may be, for the relevant tax year
are filed by the company electronically during the said period of 45 days
13. Provisional assessment in certain cases [Section 123]
Where a concealed asset of any person is impounded by any department or agency of the Federal Government or a
Provincial Government, the Commissioner Inland Revenue may, at any time before issuing any best judgment
assessment order or amended assessment order, issue to the person a provisional assessment order for the last
completed tax year regarding concealed asset of the person.
The Commissioner Inland Revenue shall finalise a provisional assessment order as soon as practicable.
14. Assessment giving effect to an order [Section 124]
The CIR or CIR (appeals) must give the appeal effect as per the following:

Type of order Time limitation for effect of appellate body order

Within two months from the end of the date the CIR is
For direct relief case u/s 124(4):
served with the appellate order.

Within two years from the end of the financial year in


For case of specific direction u/s 124(1):
which the order is received by the CIR

Within one year from the end of the financial year in which
For case set aside u/s 124(2):
the order is received by the CIR or authority

For case of assessment after a decision by any Within one year from the end of the financial year in which
Civil Court in Pakistan u/s 125: the decision in brought to the notice of CIR.

15. Powers of tax authorities to modify orders, etc. [Section 124A]


Where a question of law has been decided by a High Court or the Appellate Tribunal on or after 1.7-2002, the
CIR may, notwithstanding that he has preferred an appeal against the decision of the High Court or made an
application for reference against the order of the Appellate Tribunal, follow the said decision related to the said
question of law arising until the decision of the High Court or of the Appellate Tribunal is reversed or modified.
In case the decision of High Court or the Appellate Tribunal is reversed or modified, the CIR may,
notwithstanding the expiry of period of limitation prescribed for making any assessment or order, within a
period of one year from the date of receipt of decision, modify the assessment or order in which the said
decision was applied.
16. Evidence of assessment [Section 126]
The production of an assessment order or a certified copy of an assessment order shall be conclusive
evidence of the due making of the assessment and, except in proceedings of appeal, that the amount and all
particulars of the assessment are correct.
Any order of assessment or other document purporting to be made, issued, or executed may not be -
(a) quashed or deemed to be void or voidable for want of form; or
(b) affected by season of any mistake, defect, or omission therein,
if it is, in substance and effect, in conformity with this Ordinance.
17. Assessment of persons about to leave Pakistan [Section 145 and Rule 221, 222 and 223]
Where any person is likely to leave Pakistan during the tax year or shortly after its expiry with no intention of
returning to Pakistan, he shall give to the CIR a notice for this not less than 15 days before the date of his
departure.
The above notice shall be accompanied by a return or returns of taxable income in respect of the period
commencing from the end of the latest tax year and ending on the said date or where no assessment or return
has been made, the tax year or tax years comprising the period ending on the said date and the said period
shall be deemed to be a separate tax year.
The CIR may serve a notice on any person who, in his opinion, is likely to leave Pakistan as stated above, to
furnish within such time as may be specified in such notice, a return or returns of taxable income for the tax
year or tax years.
All the provisions of this Ordinance shall applicable as per relevant tax year.

Conceptual Approach to Taxes 273


Returns and Assessments Chapter-17

18. Tax clearance certificate [Rule 221]


A person leaving Pakistan permanently may apply to the CIR for a tax clearance certificate referred to in
section 145(3) where -
(a) the person has satisfied all income tax liabilities; or
(b) the person has made arrangements to the satisfaction of CIR for payment of income tax liabilities.
The above application shall be in the prescribed form under this rule and shall be accompanied by a certificate
of guarantee of the applicant's employer or business associate in the prescribed form under this rule.
18.1 Form of tax clearance certificate u/s 145 [Rule 222]
For section 145(3), a tax clearance certificate shall be in the form given under this rule.
19. Audit by Commissioner Inland Revenue [Section 177]
The CIR may call for any record or documents including books of accounts maintained for conducting audit of
the income tax affairs in respect of such person or any other person:
Provided that-
the CIR may, after recording reasons in writing call for record or documents including books of accounts of
the taxpayer; and
the reasons shall be communicated to the taxpayer while calling record or documents including books of
accounts of the taxpayer:
Provided further that the CIR shall not call for record or documents of the taxpayer after expiry of 6 years from
the end of the tax year to which they relate.
After completion of the audit, the CIR may after obtaining taxpayer's explanation on all the issues raised in the
audit, amend the assessment.
The fact that a person has been audited in a year shall not preclude the person from audit again in the next
and following years.
The Board may appoint a firm of Chartered Accountants or a firm of Cost and Management Accountants to
conduct an audit of the income tax affairs of any person or classes of persons and the scope of such audit
shall be as determined by the Board or the CIR on a case to case basis.
Any person employed by a firm may be authorized by the CIR, in writing, to exercise the powers for the
purposes of conducting an audit.
Where a person fails to produce before the assessment authorities any accounts, documents and records or
any other relevant document for the purpose of audit the CIR may proceed to make best judgment assessment
and the assessment treated to have been made on the basis of return or revised return filed by the taxpayer
shall be of no legal effect.
Explanation: For the removal of doubt, it is declared that the powers of the CIR under this section are not only
independent of the powers of the Board u/s 214C but also not restricts the powers of the CIR to call for the
record or documents including books of accounts of a taxpayer for audit and to conduct audit under this
section.
The Board may appoint as many special audit panels as may be necessary, comprising two or more
members from the following:-
(a) an officer or officers of Inland Revenue;
(b) a firm of Chartered Accountants as defined under the Chartered Accountants Ordinance, 1961;
(c) a firm of Cost and Management Accountants as defined under the Cost and Management
Accountants Act, 1966; or
(d) any other person as directed by the Board, to conduct an audit, including a forensic audit, of the
income tax affairs of any person or classes of persons and the scope of such audit shall be as
determined by the Board or the CIR on case to case basis.
Special audit panel shall be headed by a Chairman who shall be an OIR.
Powers under sections 175 and 176 for the purposes of conducting an audit under sub-section (11), shall
only be exercised by an officer or OIR, who are member or members of the special audit panel, and
authorized by the CIR.
Notwithstanding anything contained in sub-sections (2) and (6), where a person fails to produce before
the CIR or a special audit panel under sub-section (11) to conduct an audit, any accounts, documents
274 Conceptual Approach to Taxes
Returns and Assessments Chapter-17

and records, required to be maintained U/S 174 or any other relevant document, electronically kept
record, electronic machine or any other evidence that may be required by the CIR or the panel, the CIR
may proceed to make best judgment assessment U/S 121 of this Ordinance and the assessment treated
to have been made on the basis of return or revised return filed by the taxpayer shall be of no legal
effect.
If any one member of the special audit panel, other than the Chairman, is absent from conducting an
audit, the proceedings of the audit may continue, and the audit conducted by the special audit panel shall
not be invalid or be called in question merely on the ground of such absence.
Functions performed by an officer or officers of Inland Revenue as members of the special audit Panel,
for conducting audit, shall be treated to have been performed by special audit panel.
The Board may prescribe the mode and manner of constitution, procedure and working of the special
audit panel.
20. Displaying of National Tax Number (181C)
Every person deriving income from business chargeable to tax and who has been issued NTN shall display his NTN
at a conspicuous place at every place of his business.
21. Rectification of mistakes [Section 221]
The CIR, the CIR(Appeals) or the ATIR may, by an order in writing, amend any order passed by him to rectify
any mistake apparent from the record on his or its own motion or any mistake brought to his or its notice by a
taxpayer.
No order which has the effect of increasing an assessment, reducing a refund or otherwise applying adversely
to the taxpayer shall be made unless the taxpayer has been given a reasonable opportunity of being heard.
Where a mistake apparent on the record is brought to the notice of the CIR or CIR (Appeals) and no order has
been made before the expiration of the financial year next following the date on which the mistake was brought
to their notice, the mistake shall be treated as rectified. Further no rectification may be made after 5 years from
the date of the order sought to be rectified.
22. Different tax strategies
There are different approaches to whereby a person to mitigate or reduce the tax incidence over a person.
There are classified in the following three headings.
Tax evasion: Tax evasion is the approach whereby a person evades the tax due on its income. He did not
declare the true particulars of income or conceal the taxable activity from the concerned Authorities. The tax
evasion is a criminal Act and it is not only punishable but also liable to be prosecuted.
Tax avoidance: Tax avoidance scheme is a strategy whereby a person although pay the tax, however, he
understates his income or taxable activity. In order to achieve objective, the taxpayer may temper the record
and evidences. The open courts also deprecate this method and it also attracts penal provisions of law.
Tax planning: Tax planning is the strategy wherein a taxpayer plans its transaction in such a way that its tax
incidence may be reduced. The process is simply availment of the benefits, credits, exemptions available
under the law in favour of a taxpayer. The taxpayer may also take the advantage of any lacunas in the law.
However, such planning can only be exercised with the help of good legal experts. The accepted the fact and
acknowledge its applicability in the course of business transactions. Therefore, it is a permissible mode to
reduce the tax incidence of a person.

Conceptual Approach to Taxes 275


Returns and Assessments Chapter-17

MULTIPLE CHOICE QUESTIONS


Q.1. Following persons are not required u/s 114 to file the income tax return_____________.
(a) Company
(b) Individual having taxable income exceeds Rs. 400,000
(c) NPO
(d) NTN holder
(e) None of the above
Q.2. A person is required to file the return of income having immovable property of ___________at rating area.
(a) 250 square yards or more
(b) Less than 250 square yard or equal to 250 square yards
(c) 250 square meters or more
(d) None of the above
Q.3. If a salaried person has taxable salary income then he shall have to file_________.
(a) Evidence of tax deduction
(b) Income tax return
(c) Wealth statement and wealth reconciliation statement along with return of income
(d) All of the above
Q.4. The minimum time period granted by the Commissioner Inland Revenue to the taxpayer for furnishing the wealth
statement after serving the notice is__________.
(a) 30 days
(b) 40 days
(c) Not more than 30 days
(d) 20 days or such period as may be specified in such notice
Q.5. The Commissioner Inland Revenue may demand the furnishing of a wealth statement in respect of_______.
(a) The last tax year
(b) The last five completed tax years as a whole or any one
(c) Last three completed tax year as a whole or any one of them
(d) none of the above
Q.6. An AOP income under NTR with Tax Year ending 30-06-2016 may file its return uptill____________.
(a) 30-09-2016
(b) 31-12-2016
(c) 31-08-2016
(d) None of the above
Q.7. A company and AOP having year end August 31, 2015 will file their income tax returns uptill _________.
(a) 31-12-2015
(b) 30-09-2015
(c) 31-12-2016 and
(d) 30-09-2016
Q.8. A company having tax year ended on 30-06-2016 may file its return till_________.
(a) 30-09-2016
(b) 31-12-2016
(c) 31-08-2016
(d) None of the above
Q.9. The Commissioner Inland Revenue may grant the extension in the filing of income tax return for maximum
upto_______.

276 Conceptual Approach to Taxes


Returns and Assessments Chapter-17

(a) 15 days
(b) 30 days
(c) Less then 15 days
(d) Normally 15 days but longer extension can also be granted exceptional cases
Q.10. If a person fails to pay the income tax with the return of income, then he will___________.
(a) Revise the return of income
(b) Pay default surcharge for late payment
(c) be deemed as not filed the return
(d) both (b) and (c)
Q.11. The period within which the Commissioner Inland Revenue may demand to furnish an income tax return
is___________.
(a) 15 days
(b) 5 days
(c) 30 days or such longer or shorter period
(d) None of the above
Q.12. A notice by Commissioner Inland Revenue to furnish the income tax return on demand can be issued for the last
completed _____________.
(a) 5 years
(b) 6 years
(c) 10 years
(d) None of the above
Q.13. The Commissioner Inland Revenue may demand the return of income for the period less than 12 months in
case_______.
(a) The person has died
(b) The person has not paid the tax
(c) The person has gone into liquidation
(d) Both a and c
Q.14. The revised return may be furnished by the taxpayer within the period of ________from the end of the financial year
in which original return was furnished.
(a) 6 years
(b) 10 years
(c) 5 years
(d) 4 years
Q.15. The Commissioner Inland Revenue may made best judgment assessment on non-filing of return by the taxpayer
by_____.
(a) Without giving notice to the taxpayer
(b) after giving notice and using information / materials available to him
(c) by selecting the audit
(d) All of the above
Q.16. Where a person has to file the wealth statement, the Commissioner Inland Revenue may__________.
(a) give the notice to the respective person to file the wealth statement
(b) Can make a best judgment assessment
(c) Both a and b
(d) None of the above.
Q.17. Assessment order after making the best judgment assessment can be issued only within the period of ___ years after
the end of the Tax year or the income year to which it relates.
(a) 5 years

Conceptual Approach to Taxes 277


Returns and Assessments Chapter-17

(b) 6 years
(c) 10 years
(d) 3 years
Q.18. First amendment in an assessment can be made by the Commissioner Inland Revenue within the period
of____________.
(a) 5 years from the end of the financial year in which return was being furnished.
(b) 5 years from original assessment
(c) 6 years from original assessment
(d) 5 years after the year end
Q.19. The Commissioner Inland Revenue cannot revise the order of_______________.
(a) any taxation officer
(b) his own
(c) Commissioner (Appeals)
(d) All of the above
Q.20. Under the following cases revision cannot be made___________.
(a) Where the time for appeal has not expired
(b) Where the time for appeal has expired
(c) Where the order is pending in an appeal before any of the appellate authorities
(d) Both a and c
Q.21. The Commissioner Inland Revenue is required to pass an assessment order in case of direct relief case after the
appellate decision within _____________from the date of receipt of appellate order by the CIR.
(a) One year
(b) Two year
(c) Two months
(d) None of the above
Q.22. Where the appellate authority has ordered for the refund of excess tax deposited, the Commissioner Inland Revenue
shall issue necessary orders within_________.
(a) 3 months from the date the order was served to him
(b) 1 month
(c) 6 months
(d) 2 months from the date the order was served to him
Q.23. In case of disputed property, the Commissioner Inland Revenue shall issue the assessment order for income derived
from such property within__________.
(a) One year from the end of the financial year in which the court has decided the case
(b) Two years
(c) Three year after the end of the financial year in which the court has decided the case
Q.24. A person discontinuing his business shall give notice of discontinuance to the Commissioner Inland Revenue
within___________ from the date of such discontinuance.
(a) 30 days
(b) 60 days
(c) 15 days
(d) 20 days from the date of such discontinuance
Q.25. Every shareholder of the company is liable to pay the amount or tax of the company if he holds at least
___________of paid-up capital of the company.
(a) 10%
(b) 5%

278 Conceptual Approach to Taxes


Returns and Assessments Chapter-17

(c) 15%
(d) 20%

Q.26 A male salaried person having taxable income exceeds __________ is required to file the return of income.
(a) Rs.300,000
(b) Rs.200,000
(c) Rs.100,000
(d) Rs.400,000
Q.27 A person having annual salary income of Rs. 500,000 is required to file___________ with the tax department.
(a) Income tax return
(b) Wealth statement with wealth reconciliation
(c) both a and b
Q.28 A person is required to file a statement instead of return of income if his total income is derived from such sources
that are taxable under___________.
(a) FTR
(b) NTR
(c) as SBI under FTR
(d) Both a and c.
Q.29 National tax number is a must in order to file the income tax return ___________.
(a) On E- portal
(b) manually
(c) by post
(d) none of the above
Q.30 A taxpayer can revise the wealth statement as a result of any omission discovered_______.
(a) before amendment in assessment has been made
(b) after amendment in assessment has been made
(c) at any time after filing of wealth statement
(d) all of the above
Q.31 Extension may be granted by Commissioner Inland Revenue in respect of ____________.
(a) Annual statement of final tax regime
(b) Income tax return
(c) Wealth statement
(d) All of the above
Q.32 A taxpayer can apply for extension in __________ under section 119.
(a) Due date for payment of tax
(b) Due date for filing of income tax return
(c) Due date for filing of sales tax return
(d) none of the above
Q.33 A return of income by a company should be filed __________.
(a) Electronically
(b) manually
(c) by post
(d) All of the above
Q.34 __________ is empowered to select any person for an audit of his income tax affairs.

Conceptual Approach to Taxes 279


Returns and Assessments Chapter-17

(a) Commissioner Inland Revenue


(b) Board
(c) Commissioner Inland Revenue (Appeals)
(d) Both (a) and (b)
Q.35 Revision in an assessment order on the request of taxpayer may be made by the Commissioner Inland Revenue
where the order is pending in an appeal before __________.
(a) Appellate Tribunal Inland Revenue
(b) Commissioner Inland Revenue (Appeals)
(c) Both a and b
(d) None of the above
Q.36 The Chief Commissioner Inland Revenue is empowered to revise an order regarding issuance of exemption or lower
rate certificate passed by ____________.
(a) an authority subordinate to him
(b) Board
(c) himself
(d) Commissioner Inland Revenue (Appeals)
Q.37 Provisional assessment order is issued in respect of any ___________
(a) concealed asset
(b) tax payer who fails to file income tax return in response to notice u/s 114
(c) error in return filed by the taxpayer
(d) both a and b
Q.38 Concealed asset means any property or asset which is obtained____________.
(a) unlawfully
(b) from income which is taxable in the opinion of Commissioner Inland Revenue
(c) from exempt income
(d) from foreign remittance
Q.39 The Commissioner Inland Revenue is empowered to further appeal before the Supreme Court against the decision
made by the __________
(a) High court
(b) Commissioner Inland Revenue (Appeals)
(c) Appellate Tribunal Inland Revenue
(d) Chief Commissioner Inland Revenue
Q.40 While following the orders of the appellate authority, it is not necessary for the Commissioner Inland Revenue to ask
for re-filing or re-furnishing of any return, statement which___________.
(a) has already been filed
(b) has not been filed
(c) Both a or c
(d) None of the above
Q.41 The year in which the person has discontinued his business be treated as __________.
(a) normal tax year
(b) separate tax year
(c) special tax year
(d) transitional tax year.
Q.42 Where a private company is unable to pay its tax liability or goes into liquidation, the tax liability shall be recovered
from __________ of the company.

280 Conceptual Approach to Taxes


Returns and Assessments Chapter-17

(a) directors
(b) employed directors
(c) employees
(d) both a and b
Q.43 A director who has paid the tax of the company cannot recover it from the ________
(a) other directors
(b) company
(c) Commissioner Inland Revenue
(d) Both a or b.
Q.44 If the member of an AOP cannot pay the tax in respect of his share from AOP then it shall be recovered
from____________.
(a) AOP
(b) other member of the AOP
(c) both a and b
(d) None of the above
Q.45 A person who has been audited in a year can also be re-audited in the following years by__________.
(a) Commissioner Inland Revenue
(b) Commissioner Inland Revenue (Appeals)
(c) Appellate Tribunal Inland Revenue
(d) Chief Commissioner Inland Revenue
Q.46 The Commissioner may by notice in writing require from a person to furnish a return of income for a period of less
than twelve months, where ________.
(a) person has died
(b) become bankrupt or gone into liquidation
(c) permanently leaving Pakistan
(d) Commissioner Inland Revenue considers appropriate
(e) all of above

ANSWERS

1 (e) 2 (d) 3 (d) 4 (a) 5 (b)


6 (a) 7 (d) 8 (b) 9 (d) 10 (d)
11 (c) 12 (a) 13 (d) 14 (c) 15 (b)
16 (a) 17 (a) 18 (a) 19 (c) 20 (d)
21 (c) 22 (d) 23 (a) 24 (c) 25 (a)
26 (d) 27 (c) 28 (d) 29 (a) 30 (a)
31 (d) 32 (b) 33 (a) 34 (d) 35 (c)
36 (a) 37 (d) 38 (b) 39 (a) 40 (a)
41 (b) 42 (a) 43 (c) 44 (a) 45 (a)
46 (e)

Conceptual Approach to Taxes 281


Returns and Assessments Chapter-17

ICMAP PAST PAPERS THEORECTICAL QUESTIONS


Q.3 (a) Spring 2013 M/s. TCG is a Tax Consultancy firm. It has a list of clients whose income year ended on
different dates. Being a consultant inform your clients about the due dates for filing of income tax return in each of
the following cases:

(i) A company whose income year ended on 30 th September 2012.


(ii) A company whose income year ended on 31st December 2012.
(iii) A company whose income year would end on 31st March 2013.
(iv) A member of an association of persons (AOP) if the income year of the association of persons would end on
30th June 2013.
Q. No. 2 (a) August 2012 Every individual (whether salaried or non-salaried) whose taxable income for the year exceeds
the maximum amount that is not chargeable to tax under the Income Tax Ordinance, 2001 is required to furnish a
return of income for the tax year under section 114 of the Income Tax Ordinance, 2001. In the light of sub-section (2) of
section114, what credentials are required to be furnished with filing of income tax return?
Q. No. 3 (b) August 2012 Define the following terms under the Income Tax Rules, 2002:
(i) E-intermediary
(ii) Digital Certificate
(iii) Computerized Payment Receipt
Q. NO. 3 (b (i) SUMMER 2010 According to sub-section (1) of section 114 of the Income Tax Ordinance, 2001, name the
persons who are required to furnish a return of income.
Q. NO. 2 (a) WINTER 2009 M/s Z Attari Ltd., has a tax year ending on 30th June, is feeling difficulty in filing the return in
time. You are required to advise the management on the following issues:
(i) Who is responsible to file return on behalf of the company and what conditions shall be applicable to the return?
(ii) What procedure the company has to follow if extension in filing is required?
Q. NO. 2 (a) WINTER 2008 Define amendment of assessment as provided in section 122 of the Income Tax Ordinance,
2001.
Q. NO. 3 (b) WINTER 2006 Explain the provisions for extending time for furnishing returns and other documents u/s 119 of
the Income Tax Ordinance, 2001.
Q. NO. 6(c) WINTER 2006 Briefly state the power of tax authorities to modify the orders u/s 124-A of the Income Tax
Ordinance, 2001.
Q.3 Sept 2001 Who is liable to discharge the tax liability of a deceased person, and to what extent?
Q. NO. 4 (b, c) WINTER 2006 Explain the following under the Income Tax Ordinance, 2001.
(b) Best Judgment Assessment u/s 121 (c) Evidence of Assessment u/s 126
Q. NO. 6(b) SUMMER 2006 When assessment or amended assessment in relation to disputed property is made?
Q. NO. 7 WINTER 2004 Specify the persons required to file return of income and the persons exempted from filing return of
income U/S 114 and 115 of Income Tax Ordinance, 2001.

282 Conceptual Approach to Taxes


Returns and Assessments Chapter-17

CA MOD C PAST PAPERS THEORECTICAL QUESTIONS


Q.NO.2 Spring 2015
a) List the persons who are required to furnish a return of income for a tax year under the Income Tax Ordinance,
2001.
b) Specify the circumstances under which the Commissioner has powers to issue notice demanding a return of income
from certain person(s) for less than one year.
c) State the powers of the Commissioner if a taxpayer fails to furnish return as required under part (b) above, within
the specified time.

Q. No. 2 (c) Spring 2014


State the requirements which a person should comply with, on discontinuing a business.

Q. No. 2 (a) Spring 2013 Zia has discovered error in his annual income tax return submitted by him and intends to file a
revised return voluntarily.

Required:
Under the provisions of Income Tax Ordinance, 2001:
(i) Narrate the conditions which Zia has to comply with in order to submit a valid revised return.
(ii) State the benefits which Zia could derive by filing the revised return voluntarily,

Q. No. 2 (b) Spring 2013 Beena is engaged in the business of interior designing for the last many years. For the tax year
2013, her tax consultant had determined that income tax amounting to Rs. 345,000 was required to be paid with her annual
return which was due for filing on 31 August 2013.

On the date of filing of return, Beena was not present in Pakistan. Therefore, the consultant filed her return manually signing
it on her behalf. He also applied to the commissioner for extending the date of payment of tax by 15 days.

Required: Comment on the steps taken by Beenas consultant, in the light of Income Tax Ordinance, 2001.

Q. NO. 2 (a) Spring 2013 Discuss when a notice required to be served on a resident individual under the Income Tax
Ordinance, 2001 shall be treated as properly served.
(b) Mr. Dynamic has received a notice from the commissioner in which he identified certain errors and deficiencies in the
return filed for the last.

Required:

(i) State the deficiencies on account of which the return submitted by Mr. Dynamic may be regarded as incomplete for tax
purposes.

(ii) Narrate the circumstances under which the commissioner may amend Mr. Dynamics assessment order.
Q.4 (b) Autumn 2012 Zubaida is operating a business as a Wedding Event Planner since past 12 years. She had filed her
complete return for the tax year 2007 on 20 August 2007. On 1 September 2012, Commissioner Inland Revenue (CIR)
served a Show Cause Notice, requiring her to explain certain receipts which were credited to her account during the tax year
2007.
Zubaida is uncertain as to whether Commissioner Inland Revenue is empowered to issue such a notice after a lapse of so
many years.
Required:
Advise Zubaida about the validity of the Show Cause Notice issued by Commissioner Inland Revenue under the Income Tax
Ordinance, 2001.
Q.2 (b) Spring 2012 Identify the due dates for filing of income tax return in each of the following cases:
(i) A company whose income year ended on 30 September 2011.
(ii) A company whose income year ended on 31 December 2011.
(iii) A company whose income year would end on 31 March 2012.
(iv) A member of an association of persons (AOP) if the income year of the AOP would end on 30 June 2012.
Q.2 (a) Autumn 2011 Mrs. Hina has been living in USA for the last twenty years. She is the owner of two residential
properties in Karachi and Islamabad measuring 750 square yards and 1,000 square yards respectively.

Conceptual Approach to Taxes 283


Returns and Assessments Chapter-17

On 1 September 2011, the Commissioner Inland Revenue served a notice on her residential property located at Karachi, for
filing of return within 20 days from the date of receipt of notice for the tax year 2010. Mrs. Hinas servant informed her about
this notice over telephone.
Required: Discuss the validity of the notice in the light of provisions contained in the Income Tax Ordinance, 2001.
Q.2 (b) Autumn 2011 On 9 September 2011, Mr. Yaqoob received a notice from the Income Tax Department requiring him
to make payment of the outstanding tax demand in compliance of an assessment order issued by the CIR. Presently, Mr.
Yaqoob is experiencing cash flow difficulties and is therefore not able to pay the amount as required in the notice.
Required: As a tax consultant, advise Mr. Yaqoob about the due date for payment of tax and recourse available to him if he
is not in a position to make payment of the tax on time.
Q.NO. 3(b) Spring 2010 Mr. Sami has recently received a notice from the CIT to file return of income for the tax years 2003
and 2006 within 20 days of receiving the notice. In your capacity as a tax consultant, advise Mr. Sami on the following issues
along with appropriate explanations.
(i) Is the Commissioner Inland Revenue justified in issuing the above notice?
(ii) If Mr. Sami is not in a position to meet the deadline for filing the returns, can he get an extension?
Q.6 (a) Sept 2009 Mr. Zia's father expired in March 2009. Being the only heir, he received all his father's business and
assets. In August 2009, a notice was received from the income tax department in the name of his father to pay unpaid tax
liabilities along with penalty and default surcharge. Mr. Zia is of the view that since his father expired, the notice is irrelevant.
Required: In the light of Income Tax Ordinance, 2001 explain the correct legal position of Mr. Zia with regard to his father's
income tax liabilities and the related income tax proceedings.
Q.NO. 4(a) Autumn 2009 (a) State the provisions of the Income Tax Ordinance, 2001 with regards to rectification of
mistakes.
(b) Ayub Limited has been selected for the audit of its income tax affairs. The management is of the opinion that since their
tax affairs were audited last year also, they should not have been selected for audit this year.
Required: Discuss the managements point of view in the light of Income Tax Ordinance, 2001.
Q.NO. 5(a) Spring 2009 What do you understand by the term definite information as described in the Income Tax
Ordinance, 2001?
Q.NO. 5(b) Spring 2009 Nomani Industries (Pvt.) Limited filed their return of income for the year ended June 30, 2007 on
December 31, 2007. On January 15, 2009, the Taxation Officer issued a notice requiring the company to file the audited
financial statements. The Taxation Officer has also identified certain errors in the return of income filed for the year and has
shown his intention to amend the assessment of the Company for the year.
Required:
(i) When is a return of income regarded as incomplete for tax purposes?
(ii) Is the taxation officer justified in issuing the above notice? Explain.
(iii) Discuss the circumstances under which taxation authorities may amend the assessment order of a company.
(iv) Identify the specific date up to which the taxation authorities may amend the assessment of NI (Pvt.) Limited. Assume
that the return of income was complete on December 31, 2007 when it was filed.
Q.NO. 3(c) Spring 2008 Under what circumstances, the CIT can require a person to furnish a return of income for a period of
less than twelve months?
Q.NO. 2(a) Autumn 2008 List the persons who are required to file a return of income under the Income Tax Ordinance, 2001.
Q.NO. 5(b) Spring 2007 Under the ITO, 2001, the Commissioner Inland Revenue may serve upon the taxpayer, a notice
requiring him to pay any tax due within such time as may be specified in the notice.
Describe the modes of recovery available to the CIR, if the taxpayer fails to pay the amount of tax within the time specified in
the said notice.
Q.NO. 6(b) Spring 2007 Every resident taxpayer whose last declared or assessed income is Rs.500,000 or more is required
to furnish a wealth statement for that year along with the income tax return.
State the main particulars that are required to be included in the wealth statement.
Q.NO. 2(c) Autumn 2007 Mr. Rafiq, a salaried individual, whose taxable income for previous year was more than Rs.
500,000, has not filed the wealth statement. He is of the view that since he has no other source of income besides salary and
his employer has already filed the annual statement, he is not required to file a wealth statement.

284 Conceptual Approach to Taxes


Returns and Assessments Chapter-17

Evaluate Mr. Rafiqs point of view in the light of Income Tax Ordinance, 2001.
Q.6 (a) Sept 2007 A resident person is about to leave the country. Briefly explain the following in the light of Income Tax
Ordinance, 2001:
- responsibility of such person in relation to filing of tax return;
- powers of the Commissioner Inland Revenue in such situation;
- rules relating to assessment of income.
Q.NO. 6(a) Autumn 2007 A resident person is about to leave the country. Briefly explain the following in the light of Income
Tax Ordinance, 2001:
responsibility of such person in relation to filing of tax return;
powers of the Commissioner Inland Revenue in such situation;
rules relating to assessment of income.
Q.5 (a) March 2006 One of your clients, Mr. Nadir who is the legal representative of his deceased uncle Mr. Ather, has
approached you seeking your views with regard to his legal obligations u/s 87 on the following matters:
(i) Taxation of income earned by Mr. Ather prior to his death and the extent of tax liability of Mr. Nadir in respect of such
income.
(ii) Legality of the tax assessment proceedings pending against Mr. Ather at the time of his death.
Q.NO. 6(b) Autumn 2006 Under what circumstances, an assessment made can be amended or an amended assessment
can be further amended by the CIT?
Q. NQ. 6(c) Autumn 2006 The Income Tax Ordinance, 2001 empowers the Commissioner Inland Revenue to select a
person for audit of his tax records.
You are required to list down the criteria under which the CIT can select a person for tax audit.
Q.NO. 8(a) Spring 2006 What are the time limits prescribed by the ITO 2001, within which the Commissioner Inland
Revenue is required to pass an order to give effect to the decision of Income Tax Appellate Tribunal under the following
circumstances?
(a) The ITAT has set aside the assessment and order of the ITAT was received by the Commissioner Inland Revenue on
November 30, 2004.
(b) The ITAT has deleted the additions made by the assessing officer and the order of the ITAT was received by the
Commissioner Inland Revenue on December 15, 2004.
Q.NO. 4 Autumn 2005 How will tax be recovered if a private limited company fails to pay tax at the time of winding up.
Q.NO. 7 Autumn 2005 Under what conditions is it necessary for an individual to file a return of income.
Q.NO. 8 Spring 2005 What is the status of a complete return of income filed under the Income Tax Ordinance, 2001?
Q.8 March 2004 Describe the requirements of Income Tax Ordinance, 2001 for a person who is about to discontinue his
business
Q.NO. 7(a) Spring 2004 Please discuss the parameters of audit of income tax affairs of any person given in the Income Tax
Ordinance, 2001.
Q.NO. 8 Spring 2004 Describe the requirements of Income Tax Ordinance, 2001for persons who are about to discontinue
his business.
Q.NO. 7 Autumn 2003 Briefly state the time frame for filing the return of income by
(a) a company; (b) Persons other than a company.
Q.NO. 8 Spring 2003 What is meant by term amendment of assessments as laid down in Section 122 of the Income Tax
Ordinance, 2001.
Q.4 Autumn 2001 Your client A bank has received a notice u/s 92 of the Income Tax Ordinance, 1979 for the recovery of
tax from one of their account holder. Please advise your client on its obligations.
Q.5 Autumn 2001 Can the income tax be recovered from a director or shareholder of a a private company whose liability is
limited to the extent of amount paid on shares subscribed by him.

Conceptual Approach to Taxes 285


Returns and Assessments Chapter-17

Q.6 Autumn 2001 Describe briefly the provisions relating to re-opening of a completed assessment, including period of
limitation, if any?
Q.NO. 6 March 2000 can assessment once finalized be re-opened or modified? Discuss with basis and reasons.
Q.NO. 8 Sep 2000 an assessing officer has completed the assessment proceedings by making the following additions in the
income of a taxpayer:
- depreciation disallowed as claimed by the taxpayer;
- disallowances of printing and stationery expenses to the extent of 10% of the claim;
- non-acceptance of trading results by estimating sales from Rs. 1 million to Rs. 1.1 million and enhancing G.P rate
from 12% to 15%
The aggrieved taxpayer has approached you and requested to let him know his rights available under the law to protect his
interest. You are required to address him a reply including stating the deadlines within which such legal rights can be
availed.
Q.NO. 6 Sep 1999 what are the powers of the Commissioner Inland Revenue for revision of the assessment order. Also
explain the conditions for revision applications.
Q.NO. 8 Sep 1999 write short note on wealth reconciliation statement.
Q.NO. 2 April 1998 what are the provisions under the Income Tax Ordinance, 1979 for rectification of mistakes.
Q.NO. 2 April 1998 what are the provisions under the Income Tax Ordinance, 1979 for revision of an assessment order by
the CIR.
Q.NO. 6 May 1994 state different provisions under which assessment can be framed.
Q.NO. 7 Spring 1994 what are the various options available to a taxpayer who is not satisfied with the assessment order
framed by the income tax assessing officer.

286 Conceptual Approach to Taxes


Appeals and Revisions Chapter-18

Chapter

18 APPEALS & REVISIONS

Topic covered
Section Rule
(Part - I for CAF-6 & ICMAP students)
PART I
127 76 Appeal to the Commissioner Inland Revenue (Appeals)
128 Procedure in appeal
129 Decision in appeal
131 77 Appeal to the Appellate Tribunal
132 Disposal of appeals by the Appellate Tribunal
133 78 Reference to High Court
134A 231C Alternative Dispute Resolution
136 Burden of proof
223 Appearance by authorized representative

Part - II (For CA Mod F and ICMAP Students)


173 Liability and obligations of representatives
MCQs with solutions
ICMAP & CA Mod C past papers theoretical questions

PART I (For CAF-6 and ICMAP students)

1. Appeal to the Commissioner Inland Revenue (Appeals) [Section 127]


Who can file an appeal?
Any person dissatisfied with the order passed by the authorities specified in the ITO, 2001 may file an appeal before
the appellate authorities (except an assessment order under section 122C,).
No appeal shall be made by a taxpayer against the assessment order unless the taxpayer has paid tax due with the
return.
1.1 Conditions to be fulfilled for filing an appeal
An appeal shall-
(a) be in the prescribed form;
(b) be verified in the prescribed manner;
(c) state precisely the grounds upon which the appeal is made;
(d) be accompanied by the fee of Rs.1,000 in case of a company and Rs.200 in other cases; and
(e) be lodged with the Commissioner Inland Revenue (Appeals) within 30 days from the date of receipt of
assessment order. However the Commissioner Inland Revenue (Appeals) on an application in writing by the
appellant may admit an appeal after the expiration of 30 days on satisfaction that the appellant has reasonable
grounds for such delay in filing of appeal.
1.2 The following additional information/ documents are also required to be furnished under Rule-76 of the ITR,
2002
(i) The appeal should be filed in duplicate.
(ii) The appeal should be accompanied by the notice of demand (in original) and the copy of the assessment
appealed against.
(iii) A true copy of the form of appeal has been sent by registered post/acknowledgement due/courier service or
delivered to the concerned officer personally.

Conceptual Approach to Taxes 287


Appeals and Revisions Chapter-18

2. Procedure in appeal [Section 128]


The Commissioner Inland Revenue (Appeals):
(1) shall give notice of the day fixed for the hearing of the appeal to the appellant and to the Commissioner Inland
Revenue against whose order the appeal has been made. Where in a particular case, the Commissioner Inland
Revenue (Appeals) is of the opinion that the recovery of tax levied under this Ordinance, shall cause undue
hardship to the taxpayer, he, after affording opportunity of being heard to the Commissioner Inland Revenue
against whose order appeal has been made, may stay the recovery of such tax for a period not exceeding 30
days in aggregate.
(2) after affording opportunity of being heard to the Commissioner against whose order appeal has been made, may
stay the recovery of such tax for a further period of thirty days, provided that the order on appeal shall passed
within the said period of thirty days.
(3) may adjourn the hearing of the appeal from time to time.
(4) may, before the hearing of an appeal, allow an appellant to file any new ground of appeal not specified in the
grounds of appeal already filed by the appellant on satisfaction that the omission of the ground appeal was not
wilful or unreasonable.
(5) may, before disposing of an appeal, call for such particulars and enquiry with respect of the matters arising in the
appeal.
(6) shall not admit any documentary material or evidence which was not produced during the assessment unless he
is satisfied that the appellant has reasonable grounds for not producing such material or evidence.
3. Decision in appeal [Section 129]
(1) In disposing of an appeal the Commissioner Inland Revenue (Appeals) -
may make an order to confirm, modify or annul the assessment order after examining the evidence furnished
to him; or
(2) shall not increase the amount of any assessment order or decrease the amount of any refund unless the
appellant has been given a reasonable opportunity of being heard.
(3) as soon as practicable after deciding an appeal shall serve his order on the appellant and the Commissioner
Inland Revenue. However such order shall be passed not later than 120 days from the date of filing of appeal
or within an extended period of 60 days on reasons to be recorded in writing.
4. Appeal to the Appellate Tribunal [Section 131]
4.1 Who can file an appeal?
Where the taxpayer or Commissioner Inland Revenue objects to an order passed by the Commissioner
Inland Revenue (Appeals), the taxpayer or Commissioner Inland Revenue may appeal to the Appellate
Tribunal against such order.
4.2 Condition applicable to an appeal
An appeal shall be-
(a) in the prescribed form;
(b) verified in the prescribed manner;
(c) accompanied, except in case of an appeal preferred by the Commissioner Inland Revenue, by the
prescribed fee of Rs.2,000; and
(d) preferred to the Appellate Tribunal 60 days from the date of service of order of the Commissioner
Inland Revenue (Appeals) on the taxpayer or the Commissioner Inland Revenue, as the case may be.
However, the Appellate Tribunal may, on an application in writing by the appellant may admit an appeal
after the expiration of 60 days on satisfaction that the appellant has reasonable grounds for such delay
in filing of appeal.
Provided that if on filing of application in a particular case, the Appellate Tribunal Inland Revenue is of the
opinion that the recovery of tax levied under this Ordinance and upheld by the Commissioner Inland
Revenue(Appeals), shall cause undue hardship to the taxpayer, the Tribunal, after affording opportunity of
being heard to the Commissioner Inland Revenue, may stay the recovery of such tax for a period not
exceeding 180 days in aggregate:
Provided further that in computing the aforesaid period of 180 days, the period, if any, for which the recovery of
tax was stayed by a High Court, shall be excluded.

288 Conceptual Approach to Taxes


Appeals and Revisions Chapter-18

4.3 The following additional information/ documents are also required to be furnished under Rule-77 of the
ITR, 2002:
(i) The appeal should be filed in triplicate.
(ii) Appeal should be accompanied by a copy of the notice of demand and original copy of the assessment
appealed against.
(iii) A true copy of the form of appeal has been sent by registered post / acknowledgement due/courier
service or delivered to the concerned officer personally.
5. Disposal of appeals by the Appellate Tribunal [Section 132]
The Appellate Tribunal (ATIR):
(1) may before disposing of an appeal call for such particulars as it may require in respect of the matters arising
on the appeal.
(2) shall afford an opportunity of being heard to the parties to the appeal and, in case of default by any of the party
on the date of hearing, the Appellate Tribunal Inland Revenue may, if it deems fit, dismiss the appeal in
default, or may proceed ex parte to decide the appeal on the basis of the available record.
(3) shall decide the appeal within 6 months of its filing;
(4) Where the appeal relates to an assessment order, the Appellate Tribunal Inland Revenue may make an order
to -
(a) affirm, modify or annul the assessment order; or
(b) remand the case to the Commissioner Inland Revenue or the Commissioner Inland Revenue(Appeals).
(5) shall not increase the amount of any assessment or penalty or decrease the amount of any refund unless the
taxpayer has been given a reasonable opportunity of being heard.
(6) The Appellate Tribunal Inland Revenue shall communicate its order to the taxpayer and the Commissioner
Inland Revenue that shall be final on point of fact.
6. Reference to High Court [Section 133 and Rule 78]
(1) Within 90 days of the communication of the order of the Appellate Tribunal Inland Revenue, the aggrieved
person or the Commissioner Inland Revenue may prefer an application, in the prescribed form u/r 78 along
with a statement of the case, to the High Court, stating any question of law arising out of such order.
(2) The statement to the High Court shall set out the facts, the determination of the Appellate Tribunal Inland
Revenue and the question of law which arises out of its order.
(3) Where on an application made the High Court is satisfied that a question of law arises out of the order it may
proceed to hear the case.
(4) A reference to the High Court under this section shall be heard by a Bench of not less than two judges of the
High Court.
(5) The High Court upon hearing a reference shall decide the question of law raised by the reference and pass
judgment thereon specifying the grounds on which such judgment is based and the Appellate Tribunal Inland
Revenue order shall stand modified accordingly. The Court shall send a copy of the judgment under the seal
of the Court to the Appellate Tribunal Inland Revenue.
(6) Even where a reference has been made to the High Court, the tax shall be payable in accordance with the
order of the Appellate Tribunal Inland Revenue:
As a result of High Court judgement if the amount of tax is refundable to the taxpayer, the High Court may on
application by the Commissioner Inland Revenue within 30 days of the receipt of such judgment that he wants
to prefer appeal to the Supreme Court may postpone the refund until the disposal of appeal by the Supreme
Court.
(7) An application to High Court by a person other than the Commissioner Inland Revenue shall be
accompanied by a fee of Rs.100.
(8) Where recovery of tax has been stayed by the High Court by an order, such order shall cease to have effect
on the expiration of a period of 6 months following the day on which it was made unless the appeal is decided
or such order is withdrawn by the High Court earlier.

Conceptual Approach to Taxes 289


Appeals and Revisions Chapter-18

PRESENT STATUS OF APPEALS:

Name of Authority
Appellate whose order Limitation
Decision in Limitation period
Authority may be Filing fee period for Stay Power
appeal for decision
appealed filing appeal
against
1. CIR Rs.1,000 for Within 30 days (a) Direct Within 120 days CIR(A) may stay
Commissioner companies from the date relief from the date of the recovery of
Inland and Rs.200 of receipt of filing of appeal or tax not exceeding
Revenue in other Commissioner such extended time 30 days in
(Appeals) cases Inland (b) Specific as deem fit by the aggregate.
Revenue order direction
Commissioner
Inland after affording
Revenue(Appeals) opportunity of
being heard to
the
Commissioner
against whose
order appeal has
been made, may
stay the
recovery of such
tax for a further
period of thirty
days, provided
that the order on
appeal shall
passed within
the said period
of thirty days.
2. Appellate CIR (Appeals) Rs.2,000 but Within 60 days (a) Direct Within 6 months ATIR may stay the
Tribunal Inland not required from the date relief from the date of its recovery of tax not
Revenue from the of receipt of filing exceeding 180 days
(ATIR) income tax Commissioner in aggregate.
department Inland (b) Specific
direction
Revenue
(Appeals)
order (c) Set aside
3. High Court ATIR Rs.100 but Within 90 days (a) Direct HC on the expiry of
(HC) not required from the date relief 6 months following
from the of receipt of the day on which it
ATIR order Not applicable
income tax was made unless
department (b) Specific the appeal is
direction
decided or such
order is withdrawn
(c) Set aside by the High Court
earlier.
4. Supreme High Court
Court of
Pakistan
Note: Although
appeal may be
filed by the
taxpayer or by
the tax
department
however the
same is not a
part of ITO,
2001 as the
same is filed
under the
Constitution of
Pakistan.

290 Conceptual Approach to Taxes


Appeals and Revisions Chapter-18

7. Alternative dispute resolution [Section 134A and Rule 231C]


Appellate authority includes collector / Commissioner Inland Revenue (Appeals), Appellate Tribunal Inland Revenue,
High Court or Supreme Court.
7.1 Is alternative dispute resolution substitution of the existing conventional appellate system?
No, in fact, it works side by with the existing conventional appellate system. However at any stage of existing
appellate system an issue can be referred for Alternative Dispute Resolution and can also be referred back to
continue in the existing conventional appellate system.
7.2 What can be referred for alternative dispute resolution?
As a principle, any issue pertaining to tax liability, refund or rebate, waiver or fixation of penalty and relaxation
of time or any condition under the Federal Excise, customs, Income Tax or Sales Tax laws can be referred to
A. D. R. However it is expected that generally such cases would be referred for A. D. R. wherein contentious
issues or hardship arise as a result of any anomaly or lacuna in the law OR Arise due to some
misrepresentation of facts in the early stages of assessment and or subsequent appeals and are expected to
linger on in existing conventional appellate system for any reason.
It is also expected that while applying for Alternative Dispute Resolution the applicants will keep in mind the
amount of revenue involved. It should be understood that the interpretation of the law is the sole domain of the
existing conventional appellate system, hence outside the scope of Alternative Dispute Resolution. In other
words the scope of A. D. R. mechanism revolves mainly around the facts and circumstances of the cases. No
application may be made where prosecution proceedings have been initiated or where interpretation of
question of law, having effect on identical other cases, is involved.
7.3 At what stage a matter can be referred for alternative dispute resolution?
A matter can be referred for Alternative Dispute Resolution from either of the following stages:
(1) After assessment i.e. during pendency of appeal before the Collector / Commissioner Inland
Revenue(Appeals)
(2) During pendency of appeal before the Appellate Tribunal Inland Revenue, High Court or Supreme
Court.
Past and closed transactions cannot be referred for A. D. R. only those matters in dispute and cause hardship
can be referred for A. D. R. which are pending before any appellate authority. No application may be made
where interpretation of question of law is involved.
Prescribed fee Alternative Dispute Resolution does not involve any fees, charges or costs.
7.4 Can an application to alternative dispute resolution be submitted once the time limit for filing an
appeal or reference has expired?
It is well settled principle of law that once the time limit for filling an appeal or reference has expired it is a past
and closed transaction. Therefore, no application for Alternative Dispute Resolution is entertained. However,
the appellate authorities have inherent powers for condoning the delay in exceptional circumstances, once a
delay is condoned and the matter become an issue pending before any appellate authority it can be brought
for A. D. R. However, if the matter relates to time relaxation, it can be referred for Alternative Dispute
Resolution
7.5 Can alternative dispute resolution committee enhance the liability of tax or duty? No.
7.6 Can the matter be taken further if the appellate disagrees with the alternative dispute resolution
committee's recommendations?
The Alternative Dispute Resolution committee is an advisory body. The recommendations are neither
binding on the FBR nor the taxpayer. Thus, no occasion of taking up further arises. The recommendations
of the A. D. R. are only a guideline for the Board for resolution of the dispute of the hardship.
7.7 Can alternative dispute resolution committees rectify or review its recommendations?
The recommendation of the committee can be referred back by the Board on its own motion or on the request
of the taxpayer for rectification of any mistake apparent from the records or re-consideration of any fact or law,
which could not be considered earlier. However, the committee has no powers to review its
recommendations.
7.8 What if the decision of federal board of revenue is not acceptable to the taxpayer?
The matter reverts to the stage from where it was referred for Alternative Dispute Resolution. Referring a
matter to Alternative Dispute Resolution does not affect any of the rights already available to the taxpayer
under the relevant laws.

Conceptual Approach to Taxes 291


Appeals and Revisions Chapter-18

7.9 Does the additional duty or tax (interest) is payable?


Yes, even where under exceptional circumstances, the recovery of duty or tax is stayed or allowed to pay in
instalments the charge of additional duty or tax is mandatory. However, when the duty or tax liability stands
modified as a result of any decision including Board decision under Alternative Dispute Resolution the
additional duty or tax (interest) also stands automatically modified and re-calculated.
7.10 Can the decision of federal board of revenue under alternate dispute resolution be made precedent?
No. The resolution of a dispute or hardship arrived at between a taxpayer and Federal Board of Revenue is
only for the tax year covered by the agreement. Any such resolution cannot be quoted or used as precedence
in the same case or any other case.
8. Burden of proof [Section 136]
On an appeal by a taxpayer the burden to prove the extent to which the assessment order does not correctly reflect
his tax liability or the decision is erroneous lies with the taxpayer.
9. Appearance by authorised representative [Section 223]
Any taxpayer who is entitled or required to attend before the Commissioner Inland Revenue, the Commissioner
Inland Revenue (Appeals) or the Appellate Tribunal Inland Revenue in connection with any proceeding under the
Ordinance may, except when required under section 176 to attend personally, attend by an authorised representative.
An authorized representative of a taxpayer shall be a person who is a representative of the person may be a relative
of the taxpayer, a current full-time employee of the taxpayer, any officer of a scheduled bank with which the taxpayer
maintains a current account or has other regular dealings, any legal practitioner entitled to practice in any Civil Court
in Pakistan, any accountant; or any income tax practitioner.

PART II (For CA Mod F and ICMAP students)


10. Liability and obligations of representatives [Section 173]
Every representative of a person shall be responsible for performing any duties or obligations imposed on the person,
including the payment of tax.
Any above tax is payable by a representative of a taxpayer shall be recoverable from the representative only to the
extent of any assets of the taxpayer that are in the possession or under the control of the representative and in the
event of disagreement, such representative or person may obtain from the Commissioner Inland Revenue a
certificate stating the amount to be so retained pending final determination of the tax liability.
Every representative shall be personally liable for the payment of any tax due by the representative in a
representative capacity if, while the amount remains unpaid, the representative-
(a) Alienates, charges or disposes of any moneys received or accrued in respect of which the tax is payable; or
(b) disposes of or parts with any moneys or funds belonging to the taxpayer that is in the possession of the
representative or which comes to the representative after the tax is payable, if such tax could legally have
been paid from or out of such moneys or funds.
This section shall not relieve any person from performing any duties imposed by or under this Ordinance on the
person which the representative of the person has failed to perform.
10.1 None of the following shall act as authorised representative;
A person who has been dismissed or removed from service in the IT Department, a person having resigned
from service after having been employed in the IT Department for not less than 2 years for a period of 2 years
from the date of resignation, a person having retired from service in the IT Department for a period of 1 year
from the date of retirement in any case in which the person had made or approved any order of assessment,
refund or appeal within 1 year before the date of retirement; or a person who has become insolvent for so long
as the insolvency continues, or a person who has been convicted of an offence in relation to any income tax
proceedings for such period as the Commissioner Inland Revenue may, by order in writing, determine.
Where any legal practitioner or accountant is found guilty of misconduct in a professional capacity by any
authority entitled to take disciplinary action against the legal practitioner or accountant, an order passed by that
authority shall have effect in relation to any right to represent a taxpayer as it has in relation to the person's
right to practice as a legal practitioner or accountant.
Where any person other than above found guilty of misconduct in relation to any IT proceeding, the
Commissioner Inland Revenue may, by an order in writing, direct that the person cease to represent a
taxpayer before the Commissioner Inland Revenue, Commissioner Inland Revenue(Appeals) or Appellate
Tribunal but after the Commissioner Inland Revenue has given the person a reasonable opportunity being
heard.

292 Conceptual Approach to Taxes


Appeals and Revisions Chapter-18

Any person, against whom an order has been made may, within 30 days of service of notice of the order,
appeal to the Board to have the order cancelled. The Board may admit an appeal after the expiration of the
said period if satisfied that the appellant was prevented by sufficient cause from lodging the appeal within the
period.
No order made, shall take effect until thirty days after notice of the order is served on the person or, where an
appeal has been lodged until the disposal of the appeal.
"Accountant" means a chartered accountant, a cost and management accountant within the meaning of the
respective Acts or a member of any association of accountants recognised for this purposes by the Board; and
"income tax practitioner" means a person who is registered as such by the board, for this possesses having
such qualifications or who has retired after putting in satisfactory service in the Income Tax Department for a
period of not less than ten years in a post or posts not below that of Income Tax Officer.

Conceptual Approach to Taxes 293


Appeals and Revisions Chapter-18

MULTIPLE CHOICE QUESTIONS


Q.1 Commissioner (Appeals) is administratively subordinate to the:
(a) Commissioner Inland Revenue
(b) Chief Commissioner Inland Revenue
(c) Additional Commissioner Inland Revenue
(d) All of the above
Q.2 The appeal before the Commissioner Inland Revenue (Appeals) shall be filed against the order of the Commissioner
Inland Revenue within _______ from the date of receipt of such assessment order.
(a) 15 days
(b) 10 days
(c) 30 days
(d) 20 days
Q.3 The appeal fee to be paid by a Company for an appeal before the Commissioner Inland Revenue (Appeals) shall be
__________.
(a) Rs. 200
(b) Rs. 1,000
(c) Lower of Rs.1,000 or 10 % of tax levied
(d) Higher of Rs.1,000 or 10% of tax levied
Q.4 Income Tax Appellate Tribunal is appointed by the__________.
(a) Board
(b) Chief Commissioner Inland Revenue
(c) Supreme Court of Pakistan
(d) Federal Govt.
Q.5 The main function of the Appellate Tribunal is to hear the ___________.
(a) appeal against the order of Commissioner Inland Revenue
(b) appeal against the order of Commissioner Inland Revenue (Appeals)
(c) Both a and b
(d) appeal against Board
Q.6 The Board shall not accept any appeal filed against the order of Commissioner Inland Revenue in case of misconduct
if that appeal has been filed after the expiry of the ________________.
(a) 1 year
(b) 30 days
(c) prescribed time
(d) 2 years
Q.7 The appeal fee for an appeal before Commissioner Inland Revenue (Appeals) in case other than Companies
is_________.
(a) Rs. 200
(b) Rs.1,000
(c) Rs. 2,000
(d) Higher of Rs. 2,500 or 10 % of tax levied
Q.8 The appeal before Appellate Tribunal Inland Revenue against the order of Commissioner Inland Revenue (Appeals)
should be filed within _________from the date of receipt of order of Commissioner Inland Revenue (Appeals) by the
taxpayer.
(a) 30 days
(b) 15 days

294 Conceptual Approach to Taxes


Appeals and Revisions Chapter-18

(c) 40 days
(d) 60 days
Q.9 The memorandum of appeal to Appellate Tribunal Inland Revenue must be in triplicate and shall be accompanied
by________.
(a) Order against which the appeal is being made
(b) Order of the Commissioner Inland Revenue
(c) Both a and b
(d) Order of the Board, if any
Q.10 Income tax stay granted by Appellate Tribunal against the recovery of tax shall_____________.
(a) Not more than 6 months in aggregate
(b) Not more than 3 months
(c) Not less than 6 months in aggregate
(d) None of above
Q.11 On the basis of point of law the aggrieved party or the taxpayer has an option if not satisfied with the decision of the
tribunal to refer the case to______________.
(a) Supreme Court
(b) High Court
(c) Higher appellate tribunal
(d) None of the above
Q.12 The appeal before the High Court against the order of Appellate Tribunal may be filed by the taxpayer or the tax
department within________ of the communication of such orders of the Appellate Tribunal.
(a) 90 days
(b) 30 days
(c) 60 days
(d) 15 days
Q.13 The fee for referring the case to the High Court is_______________.
(a) Rs.500
(b) Rs.200
(c) Rs. 100
(d) Rs.1,000
Q.14 The Commissioner Inland Revenue can file an appeal against the order of High Court to Supreme Court under the
_________.
(a) Income tax Ordinance, 2001
(b) Companies Ordinance, 1984
(c) Wealth tax Act
(d) Constitution of Pakistan
Q.15 The following shall not be appointed as the authorized representative by the taxpayer to appear before any tax
authority:
(a) A relative of taxpayer
(b) A person who has retired from the service in the Income Tax Department, for the period of one year from the
date of retirement.
(c) Any legal practitioner who is entitled to practice in any Civil Court in Pakistan.
(d) Both a and b
Q.16 The appeal against the order of the Commissioner Inland Revenue in case of misconduct by the authorized
representative can be filed within______________ from the service of order.

Conceptual Approach to Taxes 295


Appeals and Revisions Chapter-18

(a) 30 days
(b) 15 days
(c) 60 days
(d) None of the above
Q.17. Revision is the reconsideration of the case by the_____________.
(a) higher administrative authority
(b) not by the same authority
(c) same authority
(d) both a or c
Q.18. Revision can be initiated by the authority on___________.
(a) its own motion
(b) application made by taxpayer
(c) both a or b
(d) Appellate Tribunal Inland Revenue
Q.19. Revision is different from_______________.
(a) provisional assessment
(b) best judgement assessment
(c) amendment of assessment
(d) both a and b
Q.20. The Board cannot interfere with the discretion of the ___________ in the exercise of his appellate function.
(a) Commissioner Inland Revenue (Appeals)
(b) Appellate Tribunal Inland Revenue
(c) Commissioner Inland Revenue
(d) both a and b
Q.21. The Commissioner Inland Revenue has the authority to grant extension in the filing period of ______, if necessary.
(a) income tax return
(b) wealth statement
(c) statement of deduction of tax
(d) all of above
Q.22. The Appellate Tribunal Inland Revenue has _______ types of members.
(a) five
(b) two
(c) four
(d) ten
Q.23. The taxpayer or the Commissioner Inland Revenue if not satisfied with the decision of the Commissioner Inland
Revenue (Appeals), can file an appeal to the ________________.
(a) Chief Commissioner Inland Revenue
(b) Appellate Tribunal Inland Revenue
(c) Supreme Court of Pakistan
(d) High court
Q.24. The term Accountant for authorised person means a _________.
(a) chartered accountant
(b) cost and management accountant
(c) member of any association of accountants recognized by board

296 Conceptual Approach to Taxes


Appeals and Revisions Chapter-18

(d) all of above


Q.25. Decision made by the Appellate Tribunal, on __________, shall be considered as final.
(a) point of law
(b) point of jurisdiction
(c) point of fact
(d) None of the above
Q.26. A Bench of ______ judges of the High Court shall hear the case presented there.
(a) 2
(b) 3
(c) 4
(d) 5
Q.27. The reference made to __________ shall not cease the payment of tax payable according to the orders of the
Tribunal.
(a) Supreme court
(b) High court
(c) Commissioner Inland Revenue (Appeals)
(d) Chief Commissioner Inland Revenue
Q.28. Matter or orders of Tribunal which involves the ___________ cannot be referred to the High Court.
(a) point of fact
(b) point of law
(c) both a and b
(d) None of the above
Q.29. The High Court may grant a stay to the Commissioner Inland Revenue for the __________, if Commissioner Inland
Revenue within 30 days of such decision file application that he shall file an appeal before the Supreme Court of
Pakistan.
(a) filing of income tax return
(b) payment of refund
(c) recovery of tax
(d) All of the above
Q.30. The stay of the High Court for the recovery of tax will cease to have any affect by the date which earlier of expiration
of ____ months from the date of stay or the date which the appeal is decided or the date on which stay order is
withdrawn by the High Court.
(a) 3
(b) 7
(c) 10
(d) 6
Q.31. Where any dispute is pending before the appellate authority, there is an alternative method for the resolution of
dispute by applying to board to solve the dispute by forming a committee which is known as ____________.
(a) dispute committee
(b) review committee
(c) alternative dispute resolution
(d) appeal committee
Q.32. All the members of the Alternative Dispute Resolution are generally from ______________.
(a) income tax group
(b) well known taxpayers

Conceptual Approach to Taxes 297


Appeals and Revisions Chapter-18

(c) Advocates of High Court


(d) all of above
Q.33. The recommendation of the committee is binding on the _________.
(a) taxpayer
(b) board
(c) Appellate Tribunal Inland Revenue
(d) none of the above
Q.34. No appeal can be made against the order of the Board appropriated by the recommendation of the ______________.
(a) supreme court
(b) High court
(c) Alternative Dispute Resolution
(d) Commissioner Inland Revenue (Appeals)
Q.35. The burden of proof in any appeal is on the party______________.
(a) who has filed the appeal
(b) against whom appeal is filed
(c) Both a and b
(d) None of the above
Q.36. The taxpayer may appoint his _________ to appear before the income tax authority on his behalf during any
proceedings under the Income Tax Ordinance, 2001.
(a) relative
(b) lawyer
(c) legal representative
(d) All of the above
Q.37. ___________ action can be taken against the authorized representative in case of misconduct by such person.
(a) legal
(b) disciplinary
(c) both a and b
(d) None of the above

ANSWERS
1 (b) 2 (c) 3 (b) 4 (d) 5 (b)
6 (b) 7 (a) 8 (d) 9 (c) 10 (a)
11 (b) 12 (a) 13 (c) 14 (d) 15 (b)
16 (a) 17 (d) 18 (c) 19 (d) 20 (d)
21 (d) 22 (b) 23 (b) 24 (d) 25 (c)
26 (a) 27 (b) 28 (a) 29 (b) 30 (d)
31 (c) 32 (d) 33 (d) 34 (c) 35 (a)
36 (d) 37 (b)

298 Conceptual Approach to Taxes


Appeals and Revisions Chapter-18

ICMAP PAST PAPERS THEORECTICAL QUESTIONS


Q. NO. 3(b) March 2015 As per section 172 of the income tax ordinance, 2001 there are certain persons who can act as an
authorized representative of resident/ non-resident taxpayers. Specify such persons who can act as authorized
representative of a non-resident taxpayer.
Q. NO. 6 WINTER 2005 What is the procedure given for Alternate Dispute Resolution in Section134A of Income tax
Ordinance, 2001?

Conceptual Approach to Taxes 299


Appeals and Revisions Chapter-18

CA MOD C PAST PAPERS THEORECTICAL QUESTIONS


Q.4 (a) Autumn 2012 In the light of Income Tax Ordinance, 2001, state how a matter would be decided in case of difference
in opinion on any point amongst the Members of a Bench constituted by the Chairperson of an Appellate Tribunal.
Q. No. 5 (b) Spring 2012 In a proceeding before the Income Tax Authority, a taxpayer can be represented by an Authorized
Representative.
Required: In the light of Income tax Ordinance, 2001 list down the persons who:
(i) can act as an Authorized Representative.
(ii) are not allowed to represent a tax payer in any proceedings before the Income Tax Authority.
Q.NO. 3(a) Spring 2010 Under the Income tax Ordinance, 2001 a taxpayer can be represented by an Authorized
Representative in a proceeding before the Income Tax Authority. You are required to list down the persons who:
(i) can act as an Authorized Representative.
(ii) are not allowed to represent a taxpayer in any proceedings before the Income Tax Authority.
Q.N. 6(a) Spring 2009 Briefly explain the term legal representative with reference to the Income tax Ordinance, 2001.
What are the obligations of a legal representative?
Q.NO. 6(a) Autumn 2006 One of your clients has received a notice from the Taxation Officer demanding payment of tax in
respect of an order issued by the Commissioner Inland Revenue against which your client intends to file an appeal before
the ITAT. You are required to explain the provisions contained in the Income tax Ordinance, 2001 regarding stay of demand
by the ITAT.
Q.NO. 3 Spring 2004 You being tax consultant of ABC Company, a partnership firm, has been informed that an order of
assessment has been served on the firm u/s 122 of the Ordinance on December 27, 2003 for the tax year 2003. The order is
accompanied by notice showing income tax payable of Rs. 10 million. The firms liability as per return of income was Rs. 15
million whereas tax assessed in the previous assessment was Rs. 18.7 million.
You are required to advise your client on the time by which the demand shown in the notice is payable, the time by which the
appellate authority may be approached and the mandatory payment required for filing appeal. You are also required to
advise on how the client could extend the date of payment of demand shown in the notice.
Q.NO. 10 Autumn 2004 Briefly state the time limit for filing an Appeal/Reference before the following forums?
(a) The Commissioner Inland Revenue of Income-tax (Appeals)
(b) Income tax Appellate Tribunal (ITAT)
(c) Reference to the High Court
Q.N.8 Autumn 2003 Briefly explain the requirements of payment of tax viz-a-viz filing an appeal before the Commissioner
Inland Revenue(Appeals)?
Q.9 Spring 2002 You have received a letter from Mr. Zubair Ansari who is seeking your advise regarding The mode and
procedure of filing an appeal under the ITO 1979. Please draft a suitable reply briefly describing the appellate procedure and
incorporate the following chart in your reply.

Name of Appellate Authority whose order may Limitation period Decision in Limitation period for
Filing fee
Authority be appealed against for filing appeal appeal decision

Q.NO. 2 March 1999 state remedies available to a taxpayer at different stages against the assessment order.
Q.NO. 6 May 1997 write short note on Appellate authorities.
Q.NO. 6 April 1995 what are the appeal forums available to an aggrieved taxpayer.
Q.NO. 6 April 1995 what are the conditions to be fulfilled before filing of income tax appeals.

300 Conceptual Approach to Taxes


Income Tax Authorities Chapter-19

Chapter

19 INCOME TAX AUTHORITIES

Topic covered
Section
(Part - I for CA & ICMAP students)
Part - I (For CAF-6 and ICMAP Students)
Federal Board of Revenue with table and chart (FBR)
Part - II (For CA Mod F and ICMAP Students)
Chief Commissioner Inland Revenue
Commissioner Inland Revenue
Officer of Inland Revenue
Appellate Tribunal Inland Revenue
206 & 206A Circulars and Advance ruling
209 Jurisdiction of income tax authorities
210 Delegation
211 Power of function exercised
212 Authority of approval
213 Guidance to income tax authorities
214 Income tax authorities to follow order of the Board
214A Condonation of time limit
214B Power of the Board to call for records
214C Selection for audit by the Board
214D Automatic selection for audit
215 Furnishing of returns, documents etc
216 Disclosure of information by a public servant
217 Forms and notices, authentication of documents
218 Service of notice and other documents
219 Tax or refund to be computed to nearest Rupee
220 Receipts for amounts paid
221 Rectification of mistakes
222 Appointment of expert
224 Proceedings under the Ordinance to be judicial proceedings
225 Proceedings against companies under liquidation
226 Computation of limitation period
227 Bar of suits in civil courts
227A Reward to Inland Revenue officers and officials
228, 229 & 230A The Directorate General of Internal Audit, Training and Research & withholding taxes
230 Directorate General (Intelligence & Investigation), Inland Revenue
230B Directorate-General of Law
230C Directorate-General of Research and Development
MCQS with solutions
CA MOD C past papers theoretical questions

Conceptual Approach to Taxes 301


Income Tax Authorities Chapter-19

Part I - For CAF-6 and ICMAP students


1. Organization of Federal Board of Revenue
The Federal Board of Revenue presently comprises Chairman and eleven members, appointed by the Federal
Government, as follows:

Chairman Federal Board of Revenue


Member Policy (Direct Taxes) Member Policy (Indirect Taxes)
Member Domestic Operation (South) Member (Facilitation and Taxpayer Education)
Member (Enforcement and Accounting) Member (Taxpayer Audit)
Member Customs (Operation) Member Domestic Operation (South)
Member (Strategic Planning, Research and Statistics) Member (Legal)

Director Generals
Director General (Information Management System) Director General (Human Resource)
Director General (Intelligence and Investigation) Director General (Training and research)
Directorate-General of Law Directorate-General of Research and Development

Part II - For CA module F and ICMAP students


In this chapter
FBR stands for Federal Board of Revenue
FG stands for Federal Government
CIR stands for Commissioner Inland Revenue
OIR stands for Officer of Inland Revenue
1.1 Federal Board of Revenue:
FBR means board established u/s 3 of the FBR Act, 2007 and it shall consist of not less than seven members
to be appointed by the FG.
The Board shall exercise all the powers and functions as were exercisable by the FBR under the FBR Act,
1924, and all other powers and functions conferred on it u/s 4.
The FG may appoint the Chairman on such terms and conditions as it may determine.
The Chairman may designate any member who shall act as an acting Chairman in his absence.

302 Conceptual Approach to Taxes


Income Tax Authorities Chapter-19

The Chairman shall appoint the Secretary of the Board who shall act under his direction and shall deal with all
matters connected with the meetings of the Board.
In case the appointment of the Chairman is delayed for any reason, the FG may appoint or designate most
senior member as acting Chairman.
The Board shall meet at least once in two months but a special meeting of the Board may be convened by the
Chairman at any time or on the request of any member.
The Board may constitute one or more committees to consist of members that are appointed by the Board and
they shall perform such functions as are entrusted to them by the Board.
The FG may delegate any of its powers under this FBR Act, 2007 to the Chairman on such terms and
conditions as the FG may determine.
All the income tax authorities except Board shall be subordinate to the Board
1.2 Powers and functions of the Board:
The Board shall exercise powers and perform all such functions that include the following, namely to:-
(a) implement the tax administration reforms;
(b) promote voluntary tax compliance and to make the Board a service oriented organization and to
implement comprehensive policies and programs for the education and facilitation of taxpayers,
stakeholders and employees, etc., in order to develop the Board into a modern efficient authority;
(c) adopt modern effective tax administration methods, information technology systems and policies in
order to consolidate assessments, improve processes, organize registration of tax payers, widen the tax
base, and make departmental remedies more efficient including enforcement of, or reduction or
remission in, duty, penalty or tax, in accordance with the relevant law for the time being in force;
(d) improve the productivity through a comprehensive and effective human resource strategy;
(e) identify and select through Internal Job Posting process the employees for designated jobs;
(f) grant additional allowances or any other incentives and rewards to the employees and members of the
Board;
(g) take appropriate measures including internal controls to combat corruption within the organizations
under the Board and provide checks to ensure the integrity of employees that is verified periodically
through applicable procedure which shall be made one of the criterion for promotion and incentives;
(h) re-designate existing posts within its jurisdiction, prepare job description of any post and create posts as
per rules;
(i) direct or advise, where necessary, investigation or inquiry into suspected duty tax evasion, tax and
commercial fraud, money-laundering, financial crimes cases and to coordinate with the relevant law
enforcement agencies;
(j) introduce and maintain a system of accountability of performance, competence and conduct of the
employees.
(k) implement the provisions of all the fiscal laws for the time being in force and to exercise all powers
provided under the provisions of the fiscal laws and to take any action, make policy, issue rules or
guidelines for the purpose to make the implementation of the fiscal laws clearer, transparent, effective
and convenient;
(l) implement international obligations pursuant to a treaty, resolution or any international commitment;
(m) establish a foundation for the welfare of the present and retired employees and their families, and to
create, establish, organize, assist in the social and cultural facilities;
(n) create a surplus pool of employees as and when required;
(o) make regulations, policies, programs, strategies in order to carry out the purposes of this Act;
(p) engage any person or entity on contract basis to carry out assignments or for the consultancy in
accordance with the rules of the Federal Government;
(q) regulate and enter into any agreement, contract, understanding, with any international organization or
institution or donor agency or counterpart entity with approval of the FG;
(r) create field formations of Board for greater efficiency in implementation of fiscal laws and refer to them
with appropriate titles;
(s) set up mechanism and processes that facilitate removal of grievances and complaints of the tax payers;

Conceptual Approach to Taxes 303


Income Tax Authorities Chapter-19

(t) carry out any other function, activity and acts, etc., as decided and determined by the Board;
(u) enable electronic communication in respect of all taxation matters such as e-filing, e-payments, e-
notice, e-notification, digital imaging, protocols or agreements as may be prescribed; and
(v) perform any other functions entrusted from time to time by the FG.
The Board may, where appropriate, issue statutory rules and orders (SROs), orders, circulars and instructions
for the enforcement of any of the provisions of fiscal law and the provision of this Act.
The Board shall perform all other functions assigned by the FG for the purpose of implementation of this Act.
2. Chief Commissioner Inland Revenue
The Chief CIR is appointed u/s 208 of the ITO, 2001. According to section 207(3A) CIR, Additional CIR, Deputy CIR,
Additional CIR, Revenue Officers, Inland Revenue Audit Officer, Superintendents Inland Revenue, Auditors Inland
Revenue and Inspectors Inland Revenue, shall be subordinate to the Chief CIR.
2.1 Powers and functions of Chief Commissioner Inland Revenue are discussed below:
The Chief CIR shall perform all or such functions and exercise all or such powers under this Ordinance as may
be assigned to him in respect of such persons or classes of persons or such areas as the Board may direct.
[U/s 209(1)]
Chief CIR may, by an order, confer upon or assign to any Officer Inland Revenue all or any of the powers and
functions conferred upon or assigned to the CIR, under this Ordinance, in respect of any person or persons or
classes of persons or areas as may be specified in the order. [U/s 209(2)]
Where a question arises as to whether a CIR has jurisdiction over a person, the question shall be decided by
the concerned Chief CIR. [U/s 209(6)]
2.2 Commissioner Inland Revenue Commissioner Inland Revenue is appointed u/s 208 of the ITO, 2001.
2.3 Powers and functions of CIR: Commissioner Inland Revenue has following powers and functions:
a. He has the power to approve the gratuity fund and superannuation fund. [U/s 2(3)]
b. He has the power to recognize the provident fund. [U/s 2(48]
c. He has the power to approve a non-profit organization. [U/s 2(36)]
d. He may allow a person to charge tax on income from salary on accrual basis and he may also charge income
under head salary on accrual basis if he is satisfied that payment of tax was deferred for tax avoidance. [U/s
12(7) and 110]
e. He may allow a person to change his method of accounting or method of valuation of stock. [U/s 32(4) and
35(6)]
f. He may allow a person to change his tax year. [U/s 74(3)]
g. He shall determine the fair market value of any property or rent, asset, service, benefit or perquisite if it is not
ordinarily ascertainable. [U/s 68(3)]
h. He may require a person to furnish a return of income for a period of less than 12 months. [U/s 114(3)]
i. He may require a person to file income tax return, statement of final taxation and wealth statement if the
person has failed to file these statements. [U/s 114, 115 and 116]
j. He may grant extension of time to furnish the return, certificate of statement, as the case may be. [U/s 119(3)]
k. He may conduct audit of the income tax affairs of a person u/s 177 and all the provisions of that section shall
apply accordingly. [U/s 120(1A)]
l. Where the return of income furnished is not complete, the CIR shall issue a notice to the taxpayer informing
him of the deficiencies and directing him to provide such information, particulars, statement or documents. [U/s
120(3)]
m. He may amend an assessment by making such alterations or additions as the CIR considers necessary. [U/s
122(1)]
n. The CIR may suo moto call for the record of any proceeding under this Ordinance or under the repealed
Ordinance in which an order has been passed by any Officer of Inland Revenue other than the CIR (Appeals).
[U/s 122A(1)]
o. CIR can make provisional assessment in certain cases [U/s 122C and 123]
p. He may grant the taxpayer an extension of time for payment of tax due under sub-section (2) or allow the
taxpayer to pay such tax in instalments. [U/s 137(4)]

304 Conceptual Approach to Taxes


Income Tax Authorities Chapter-19

q. CIR can recover the tax out of property and through arrest of taxpayer and the CIR shall have the same
powers as a Civil Court has under the Code of Civil Procedure, l908 for the purposes of the recovery [U/s 137]
r. He can recover tax from persons holding money on behalf of the taxpayer. [U/s 140]
s. He may allow any person to make payment without deduction of tax. [U/s 153(4)]
t. He shall, upon application in writing by the person, issue the person with an exemption or lower rate certificate.
[U/s 159(1)]
u. He can recover the tax from person from whom tax was not collected or deducted [U/s 162(1)]
v. In the event of the liquidation or bankruptcy of a person who has collected or deducted tax from a payment the
CIR shall have a first claim for that amount before any distribution of property is made. [U/s 166(2)]
w. The CIR may require any person to install and use an Electronic Tax Register. [U/s 174(5)]
x. The CIR has power to enter and search premises of any taxpayer in order to enforce any provision of the
ordinance. He may also authorize any valuer or expert to enter any premises and perform any task assigned to
him by the CIR. [U/s 175(1) and (2)]
y. The CIR may call for any record or documents including books of accounts maintained under this Ordinance or
any other law for the time being in force for conducting audit. [U/s 177(1)]
3. Officer of Inland Revenue
OIR means any Additional CIR, Deputy CIR, Assistant CIR, Inland Revenue Officer, Inland Revenue Audit Officer or
any other officer howsoever designated or appointed by the Board for the purposes of this Ordinance.
The CIR may, by an order in writing, delegate to any OIR, subordinate to the CIR all or any of the powers or functions
conferred upon or assigned to the CIR under this Ordinance, other than following powers:
(a) Power of delegation
(b) Powers of amendment of assessment contained in section 122(5A) to an OIR below the rank of Additional CIR
[U/s 210]
The exercise of a power or the performance of a function, of the CIR by an OIR shall not prevent the exercise of the
power, or the performance of the function, by the CIR. [U/s 211(2)]
4. Appellate tribunal inland revenue:
Appellate Tribunal Inland Revenue means the Appellate Tribunal Inland Revenue established u/s 130. The Appellate
Tribunal shall consist of a chairperson and such other judicial and accountant members as are appointed by the
Federal Government having regard to the needs of the Tribunal.
4.1 Appointment of Judicial Member:
A person may be appointed as a judicial member of the Appellate Tribunal if the person-
(a) has exercised the powers of a District Judge and is qualified to be a Judge of a High Court;
(b) is or has been an advocate of a High Court and is qualified to be a Judge of the High Court; or
(c) is an officer of Inland Revenue Service in BS-20 or above and is a law Graduate.
4.2 Appointment of Accountant Member:
A person may be appointed as an accountant member of an appellate tribunal if,-
(i) he is an officer of Inland Revenue Service equivalent to the rank of Regional Commissioner;
(ii) a CIR or CIR (Appeals) having at least three years experience as CIR or Collector; or
(iii) a person who has, for a period of not less than ten years, practiced professionally as a chartered accountant
within the meaning of Chartered Accountants Ordinance, 1961.
4.3 Appointment of chairperson:
The Federal Government shall appoint a member of the Appellate Tribunal as Chairperson of the Tribunal and except
in the special circumstances, the person appointed should be a judicial member.
Procedure of appellate tribunal (Functions)
i. The powers and functions of the Appellate Tribunal shall be exercised and discharged by Benches constituted
from members of the Tribunal by the Chairperson of the Tribunal.
ii. A bench shall consist of not less than two members of the Appellate Tribunal and shall be constituted so as to
contain an equal number of judicial and accountant members, or so that the number of members of one class
does not exceed the number of members of the other class by more than one. However, the FG may direct
that all or any of the powers of the Appellate Tribunal shall be exercised by any one member; or more
members than one, jointly or severally.

Conceptual Approach to Taxes 305


Income Tax Authorities Chapter-19

iii. The Chairman may constitute as many benches consisting of a single member as he may deem necessary to
hear such cases or class of cases as the FG may by order in writing, specify.
iv. The Chairman or any other member of the Appellate Tribunal authorized, in this behalf by the Chairman may,
sitting singly, dispose of any case where the amount of tax or penalty involved does not exceed Rs.5 million.
v. If the members of a Bench differ in opinion on any point, the point shall be decided according to the
opinion of the majority.
vi. If the members of a Bench are equally divided on a point, they shall state the point on which they differ and the
case shall be referred by the Chairperson for hearing on that point by one or more other members of the
Appellate Tribunal, and the point shall be decided according to the opinion of the majority of the members of
the Tribunal who have heard the case including those who first heard it.
vii. If there are an equal number of members of the Appellate Tribunal, the FG may appoint an additional member
for the purpose of deciding the case on which there is a difference of opinion.
viii. Subject to this Ordinance, the Appellate Tribunal shall have the power to regulate its own procedure, and the
procedure of Benches of the Tribunal in all matters arising out of the discharge of its functions including the
places at which the Benches shall hold their sittings.
5. Circulars [u/s 206]
(1) To achieve consistency in the administration of this Ordinance and to provide guidance to taxpayers and
officers of the Board, the Board may issue Circulars setting out the Board's interpretation of this Ordinance.
(2) A circular issued by the Board shall be binding on all Income Tax Authorities and other persons employed in
the execution of the Ordinance, under the control of the said Board other than CIR (Appeals).
(3) A Circular shall not be binding on a taxpayer.
6. Advance ruling [u/s 206A]
(1) The Board may, on application in writing by a non-resident taxpayer, issue to the taxpayer an advance ruling
setting out the Commissioner's position regarding the application of this Ordinance to a transaction proposed
or entered into by the taxpayer.
(2) Where the taxpayer has made a full and true disclosure of the nature of all aspects of the transaction relevant
to the ruling and the transaction has proceeded in all material respects as described in the taxpayer's
application for the ruling, the ruling is binding on the CIR with respect to the application to the transaction of
the law as it stood at the time the ruling was issued.
(3) Where there is any inconsistency between a circular and an advance ruling, priority shall be given to the terms
of the advance ruling.
Provided that this section shall not apply to a non-resident tax payer having a permanent establishment in Pakistan.
7. Jurisdiction of income tax authorities [U/s 209]
(1) The Chief Commissioners, the CIR and the CIR (Appeals) shall perform all or such functions and powers as
may assigned to them in respect of such persons or classes of persons or such areas as the Board may direct.
Provided that the Board or the Chief Commissioner, as the case may be, may transfer jurisdiction in respect of
cases or persons from one CIR to another.
(2) The Board or the Chief Commissioner may, delegate all or any of the powers by order in writing to any OIR
such powers and functions assigned to the CIR.
(3) An order as stated above shall be made only with the approval of Board by the Chief Commissioner and such
OIR shall be treated to be the CIR.
(4) The CIR shall have jurisdiction,-
(a) where the person's place of business is within such area, or where the business is carried on in more
than one place, the person's principal place of business is within such area; or
(b) in respect of any other person, if the person resides in such area:
Explanation.- The expression "place of business" as used above, means,-
(a) in the case of listed or unlisted public limited company, the place where the registered office is situated;
(b) in the case of other companies,-
(i) if the company is primarily engaged in manufacture or processing, the place where the factory is
situated;
(ii) if the company is primarily engaged in business other than manufacture or processing, the place where
main business activities are actually carried on.

306 Conceptual Approach to Taxes


Income Tax Authorities Chapter-19

(5) Where a question arises as to whether a CIR has jurisdiction over a person, the question shall be decided by
the Chief CIR Chief Commissioners concerned and, if they are not in agreement, by the Board.
(6) No person shall call into question the jurisdiction of a CIR after that person has furnished a return of income to
the CIR or, where the person has not furnished a return of income, after the time allowed by any notice served
on the person for furnishing such return has expired.
(7) The power shall include the power to transfer jurisdiction from one income tax authority to another.
(8) Where, in respect of any proceedings, an income tax authority is succeeded by another, the succeeding
authority may continue the proceedings from the stage it was left by that authority's predecessor.
8. Delegation [U/s 210]
(1) The CIR may by an order in writing, delegate to any OIR subordinate to him (falling in the jurisdiction of CIR)
all or any of the powers or functions assigned to the CIR, other than the power of delegation.
(2) The CIR shall not delegate the powers of amendment of assessment to an officer of Inland Revenue below the
rank of Additional CIR.
(3) The CIR may, by an order in writing, delegate to a special audit panel appointed under section 177(11),
or to a firm of chartered accountants or a firm of Cost and Management Accountants appointed by the
Board or the CIR to conduct an audit of person U/S 177, all or any of the powers or functions to conduct
an audit under this Ordinance.
(5) The CIR shall have the power to cancel, modify, alter or amend an order issued above.
9. Power or function exercised [U/s 211]
(1) Where, by virtue of an order an OIR or by a special audit panel appointed under section 177(11) exercises
a power or performs a function, such power or function shall be treated as having been exercised or performed
by the CIR.
(2) The exercise of a power, or the performance of a function, of the CIR by an OIR shall not prevent the exercise
of the power, or the performance of the function, by the CIR.
(3) The Board or with the approval of the Board an authority appointed under this Ordinance, shall be competent
to exercise all powers conferred upon any authority subordinate to it.
10. Authority of approval [U/s 212]
The Board may, by a general or special order, authorize the Regional Commissioner or the CIR to grant approval in
any case where such approval is required from the Board under any provision of this Ordinance.
11. Guidance to income tax authorities [U/s 213]
In the course of any proceedings under this Ordinance, the CIR or any taxation officer may be assisted, guided or
instructed by any income tax authority to whom he is subordinate or any other person authorized in this behalf by the
Board.
12. Income tax authorities to follow orders of the Board [U/s 214]
All income tax authorities and other persons employed in the execution of this Ordinance shall observe and follow the
orders, instructions and directions issued by the Board except that the Board that will interfere with the discretion of
the CIR (Appeals) in the exercise of his appellate function.
13. Condonation of time limit [U/s 214A]
Where any time or period has been specified within which any application is to be made or any act or thing is to be
done, the Board may itself or may empower any CIR or Chief Commissioner, in any case or class of cases, permit
such further time for filing of an application or the thing to be done.
Explanation,- For the purpose of this section, the expression any act or thing is to be done includes any act or
thing to be done by the taxpayer or by the authorities specified in section 207
14. Power of the Board to call for records [U/s 214B]
(1) The Board may, of its own motion, call for and examine the record of any departmental proceedings for the
purpose of satisfying itself as to the legality or propriety of any order passed therein and may pass such order
as it may think fit:
No order imposing or enhancing any tax or penalty than the originally levied shall be passed unless the person
affected by such order has been given an opportunity of being heard.
(2) No proceeding as above shall be initiated in a case where an appeal is pending or on the expiry of three years
from the date of original decision or order.

Conceptual Approach to Taxes 307


Income Tax Authorities Chapter-19

15. Selection for audit by the Board [U/s 214C]


(1) The Board may select persons or classes of persons for audit of Income Tax affairs through computer ballot
which may be random or parametric as the Board may deem fit.

(1A) Notwithstanding anything contained in this Ordinance or any other law, for the time being in force, the Board
shall keep the parameters confidential; and
(2) Audit of Income Tax affairs of persons selected as above shall be conducted as per procedure of total audit
u/s 177 except that the CIR may also call for record for total audit.
(3) For the removal of doubt it is hereby declared that Board shall be deemed always to have had the power to
select any persons or classes of persons for audit of Income Tax affairs.
Explanation. For the removal of doubt, it is declared that the powers of the CIR under section 177 are independent
of the powers of the Board under this section and nothing contained in this section restricts the powers of the CIR to
call for the record or documents including books of accounts of a taxpayer for audit and to conduct audit under
section 177.
16. Automatic selection for audit [U/s 214D]
(1) Every person shall be automatically selected for audit of income tax affairs for a tax year if.
a. the return is not filed within the date it is required to be filed as specified in section 118 or as the
case may be not filed within the time extended by the board u/s 214A or further extended for a
period not exceeding 30 days by the CIR U/S 119; or
b. the tax payable as per return has not been paid;
(2) Audit of income tax affairs of persons automatically selected under sub section (1) shall be conducted as per
procedure given in section 177 and all the provisions of this Ordinance shall apply accordingly.
Provided that audit proceedings shall only be initiated after the expiry of 90 days from the date as mentioned in
sub section (1);
(3) Subject to section 182, 205 and 214C, sub section (1) shall not apply if the person files the return within 90
days from the date as mentioned in sub section (1) and
(a) 25% higher tax than the previous tax year tax liability has been paid or
- 2% tax on turnover or tax payable under Part I of the First Schedule, whichever is higher has been paid
along-with a return by a person who in the preceding year has either not filed with the return or had declared
income below taxable limit. Where return was filed for immediately preceding tax year, turnover declared for
the tax year should not be less than the turnover declared in immediately preceding tax year.
(4) The provisions of sections 177 and 214C for a tax year shall not apply to a person registered as retailer
under rule (4) or rule (6) of the ST Special Procedure Rules, 2007 subject to the conditions that
(a) name of the person registered under rule (4) of the ST Special Procedure Rules, 2007
appears throughout the tax year in the sales tax ATL;
(5) Sub section (4) shall have effect from the date appointed by the Board through notification in official
gazette.
17. Furnishing of returns, documents etc [U/s 215]
Where, by virtue of an order of delegation of power by the CIR to any OIR the function and power to receive an
application, or to call for and receive, any returns of income, certificates, documents, accounts and statements from
any person or persons or class of persons, the filer on furnishing of aforesaid documents, shall be treated as having
been furnished to the CIR.
18. Disclosure of information by a public servant [U/s 216]
(1) All particulars contained in -
(a) any statement made, return furnished, or accounts or documents produced.
(b) any evidence given, or affidavit or deposition made, in the course of any proceedings under this
Ordinance, other than proceedings of offences and prosecutions; or
(c) any record of any assessment proceedings or any proceeding relating to the recovery of a demand,
shall be confidential and no public servant save as provided in this Ordinance may disclose any such
particulars.
(2) Notwithstanding anything contained in the Qanun-e-Shahadat, 1984, or any other law for the time being in
force, no' court or other authority shall be, save as provided in this Ordinance, entitled to require any public

308 Conceptual Approach to Taxes


Income Tax Authorities Chapter-19

servant to produce before it any return, accounts, or documents contained in, or forming a part of the records
relating to any proceedings under this Ordinance, or any records of the Income Tax Department generally, or
any part thereof, or to give evidence before it in respect thereof.
(3) Nothing contained in sub-section (1) shall preclude the disclosure of any such particulars-
(a) to any person acting in the execution of this Ordinance, where it is necessary to disclose the same to
him for the purposes of this Ordinance;
(b) to any person authorized by the CIR, in this behalf, where it is necessary to disclose the same to such
person for the purposes of processing of data and preparation of computer printouts relating to returns
of income or calculation of tax;
(c) where the disclosure is occasioned by the lawful employment under this Ordinance of any process for
the service of any notice or the recovery of any demand;
(d) to the Auditor-General of Pakistan for the purpose of enabling the Auditor-General to discharge his
functions under the Constitution;
(e) to any officer appointed by the Auditor-General of Pakistan or the CIR to audit income tax receipts or
refunds;
(f) to any officer of the FG or a Provincial Government authorized by such Government in this behalf as
may be necessary for the purpose of enabling that Government to levy or realize any tax imposed by it;
(g) to any authority exercising powers under the Federal Excise Act, 2005, the Sales Tax Act. 1990, the
Wealth Tax Act, 1963 or the Customs Act, 1969, as may be necessary for the purpose of enabling its
duty to exercise such powers;
(h) occasioned by the lawful exercise by a public servant of powers under the Stamp Act, 1899 to impound
an insufficiently stamped document;
(i) to the SBP to enable it to compile financial statistics of international investment and balance of
payment;
(j) as may be required by any order made u/s 19(2) of the Foreign Exchange Regulation Act. 1947 or for
the purposes of any prosecution for an offence u/s 23 of that Act;
(k) to the SECP or the Monopolies Control Authority for the purposes of the Securities and Exchange
Ordinance, 1969, the Monopolies and Restrictive Trade Practices Ordinance, 1970, the Companies
Ordinance, 1984 or the SECP Act, 1997, as the case may be;
(l) relevant to any inquiry into a charge of misconduct in connection with income tax proceedings against a
legal practitioner or an accountant;
(m) to a Civil Court in any suit or proceeding to which the FG or any income tax authority is a party which
relates to any matter arising out of any proceedings under this Ordinance;
(n) for the purposes of a prosecution for any offence under the Pakistan Penal Code, 1860 in respect of
any such statement, returns, accounts, documents, evidence, affidavit or deposition, or for the purposes
of a prosecution for any offence under this Ordinance;
(o) relevant to any inquiry into the conduct of an official of the Income Tax Department to any person or
officer appointed to hold such inquiry, or to a Public Service Commission, when exercising its functions
in relation to any matter arising out of such inquiry;
(p) as may be required by any officer or department of the FG or of a Provincial Government for the
purpose of investigation into the conduct and affairs of any public servant, or to a Court in connection
with any prosecution of the public servant arising out of any such investigation;
(q) to an authorized officer of the government of any country outside Pakistan with which the Government
has entered into an agreement for the avoidance of double taxation and the prevention of fiscal evasion;
or
(r) to the Federal Tax Ombudsman.
(4) Nothing in this section shall apply to the production by a public servant before a Court of any document,
declaration, or affidavit filed or the giving of evidence by a public servant in respect thereof.
(5) Nothing contained as above shall prevent the CIR from publishing, with the prior approval of the FG, any such
particulars as are referred therein.
(6) Nothing contained in sub-section (1) shall prevent the FG from publishing particulars and the amount of tax
paid by a holder of a public office as defined in the National Accountability Bureau Ordinance, 1999.

Conceptual Approach to Taxes 309


Income Tax Authorities Chapter-19

(7) Any person to whom any information is communicated and any person employee under the first-mentioned
person's control, shall be, in respect of that information, subject to the same rights, privileges, obligations, and
liabilities as if the person were a public servant.
(8) No prosecution may be instituted except with the previous sanction of the Board.
19. Forms and notices, authentication of documents [U/s 217]
Forms, notices, returns, statement, tables and other documents required under this Ordinance may be in such form
as determined by the Board for the efficient administration of this Ordinance and publication of such documents in the
official Gazette shall not be required-
The CIR shall make the documents available to the public in the manner prescribed.
A notice or other document issued, served or given by the CIR under this Ordinance shall be sufficiently
authenticated if the name or title of the CIR, or authorized OIR, is printed, stamped or written on the notice or
document or if it is computer generated and bears the authentication in the manner prescribed by the Board.
20. Service of notices and other documents [U/s 218]
(1) Any notice, order or requisition required to be served on a resident individual for the purposes of this
Ordinance shall be treated as properly served on the individual if -
(a) personally served on the individual or, in the case of an individual under a legal disability or a non-
resident individual, the representative of the individual,
(b) sent by registered post or courier service to the place specified in clause (b) of sub-section (2) or to the
individual's usual or last known address in Pakistan; or
(c) served on the individual in the manner prescribed for service of a summons under the Code of Civil
Procedure, 1908.
(2) The aforesaid provisions shall apply as it is in case of representative of a non resident individual in Pakistan.
(3) Where an AOP is dissolved, any notice, order or requisition required to be served under this Ordinance on the
association may be served on any person who was the principal officer or a member of the association
immediately before such dissolution.
(4) Where business has discontinued, any notice, order or requisition required to be served under this Ordinance
on the person discontinuing the business may be served on the person personally or on any individual who
was the person's representative at the time of discontinuance.
(5) The validity of any notice issued as above shall not be called into question after the return to which the notice
relates has been furnished or the notice has been otherwise complied with.
21. Tax or refund to be computed to the nearest Rupee [U/s 219]
In the determination of any amount of tax or refund payable, fractions of a rupee less than fifty paisa shall be
disregarded and fractions of a rupee equal to or exceeding fifty paisa shall be treated as one rupee.
22. Receipts for amounts paid [U/s 220]
The CIR shall give a receipt for any tax or other amount paid or recovered under this Ordinance.
23. Rectification of mistakes [U/s 221]
Already covered in chapter of Assessment.
24. Appointment of expert [U/s 222]
The CIR may appoint any expert as the CIR considers necessary for the purposes of this Ordinance, including for the
purposes of audit or valuation.
25. Proceedings under the Ordinance to be judicial proceedings [U/s 224]
Any proceedings under this Ordinance before the CIR, CIR (Appeals) or Appellate Tribunal shall be treated as judicial
proceedings.
26. Proceedings against companies under liquidation [U/s 225]
Notwithstanding anything contained in section 316 of the Companies Ordinance, 1984, leave of the Court shall not be
required for continuing with or commencing any proceeding under this Ordinance against a company in respect of
which a winding up order has been made or Provisional Liquidator appointed.
27. Computation of limitation period [U/s 226]
In computing the period of limitation, there shall be excluded -

310 Conceptual Approach to Taxes


Income Tax Authorities Chapter-19

(a) in the case of an appeal or an application under this Ordinance, the day on which the order complained of
was served; and
(b) in the case of an assessment or other proceeding.
(i) the period, if any, for which such proceedings were stayed by any Court, Appellate Tribunal or any other
authority; or
(ii) the period, if any, for which any proceeding for the tax year remained pending before any Court,
Appellate Tribunal or any other authority.
28. Bar of suits in Civil Courts [U/s 227]
No suit or other legal proceeding shall be brought in any Civil Court against any order made under this Ordinance,
and no prosecution, suit or other proceedings shall be made against any person for anything which is in good faith
done or intended to be done under this Ordinance or any rules or orders made there under.
Notwithstanding anything contained in any other law for the time being in force, no investigation or inquiry shall be
undertaken or initiated by any governmental agency against any officer or official for anything done in his official
capacity under this Ordinance, rules, instructions or direction made or issued there-under without the prior approval of
the Board-
29. Reward to Inland Revenue officers and officials [227A]
In cases involving concealment or evasion of income tax and other taxes, cash reward shall be sanctioned to the
officers and officials of Inland Revenue for their meritorious conduct in such cases and to the informer providing
credible information leading to such detection, as may be prescribed by the Board, only after realization of part or
whole of the taxes involved in such cases.
The Board may, by a notification in the official Gazette, prescribe the procedure in this behalf and specify the
apportionment of reward sanctioned under this section for individual performance or to collective welfare of the
officers and officials of Inland Revenue.
30. The Directorate General of Internal Audit, Training and Research and withholding taxes [U/s 228, 229 and
230A]
The Directorate General of internal Audit, training and research and withholding taxes shall consist of a Directors
General and as many Directors, Additional Directors, Deputy Directors and Assistant Directors and such other officers
as the Board, may by notification in the official Gazette, appoint.
The Board may, by notification in the official Gazette, specify the functions, jurisdiction and powers of the Directorate
General of Internal Audit, Training and Research and Withholding Taxes.
31. Directorate General (Intelligence and Investigation), Inland Revenue [U/s 230]
The Directorate General (Intelligence and Investigation) IR shall consist of a Director General and as many
Directors, Additional Directors, Deputy Directors and Assistant Directors and such other officers as the Board,
may by notification in the official Gazette, appoint.
The Board may, by notification in the official Gazette,-
(a) specify the functions and jurisdiction of the Directorate General and its officers; and
(b) confer the powers of authorities specified in section 207 upon the Directorate General and its officers.
32. Directorate-General of Law [U/s 230B]
The Directorate- General of Law shall consist of a Director General and as many Directors, Additional Directors,
Deputy Directors, Assistant Directors, Law Officers and such other officers as the Board may, by notification in the
official Gazette, appoint.
The Board may, by notification in the official Gazette, specify the functions, jurisdiction and powers of the Directorate-
General of Law.
33. Directorate-General of Research and Development [U/s 230C]
The Directorate-General of Research and Development shall consist of a Director General and as many Directors,
Additional Directors, Deputy Directors, Assistant Directors and such other officers as the Board may, by notification in
the official Gazette, appoint.
The Board may, by notification in the official Gazette, specify the functions, jurisdiction and powers of the Directorate-
General of Research and Development.

Conceptual Approach to Taxes 311


Income Tax Authorities Chapter-19

MULTIPLE CHOICE QUESTIONS


Q.1. _________ is the highest tax authority.
(a) Commissioner Inland Revenue
(b) Chief Commissioner Inland Revenue
(c) Board
(d) none of above
Q.2. Circulars are issued for the guidance of _____________
(a) taxpayer
(b) board
(c) both
(d) non of above
Q.3. For a Private Limited Company the jurisdiction place would be__________.
(a) where the person carrying on business
(b) where his residence is
(c) where the Commissioner Inland Revenue recommends
(d) none of above
Q.4. Exemption certificates are issued by_________.
(a) Commissioner Inland Revenue
(b) Chief Commissioner Inland Revenue
(c) Employer
(d) All of these
Q.5. Officers appointed by Board for the implementation of IT Ordinance, 2001 shall be__________.
(a) special officer
(b) Assistant Commissioner Inland Revenue
(c) taxation officer
(d) Chief Commissioner Inland Revenue
(e) all of the above
Q.6. CIR may assign his powers and functions to
(a) Chief Commissioner Inland Revenue
(b) Any taxation officer
(c) both of these
(d) none of these
Q.7. Delegation can be ___________.
(a) in writing
(b) oral
(c) both of these
Q.8. The provision for the confidentiality can be override
(a) where there is any fraud in return
(b) any additional information is required from third party
(c) both a and b
(d) none of the above

312 Conceptual Approach to Taxes


Income Tax Authorities Chapter-19

Q.9. Service of notice means in case of an individual___________.


(a) personally hand over of letter
(b) send to his AR
(c) send through registered post
(d) all of the above
Q.10. Service of notice is properly sent refers to, in case of an AOP______________.
(a) served at his registered office
(b) served to residence of principal officer
(c) served to any member of AOP
(d) all of these
Q.11. Expert can be _____________.
(a) AOP
(b) Company
(c) An individual
(d) all of these
Q.12 The __________ is the highest tax authority in Pakistan.
(a) Commissioner Inland Revenue
(b) Chief Commissioner Inland Revenue
(c) Board
(d) Appellate Tribunal Inland Revenue
Q.13 Valuer is appointed by _________.
(a) Chief Commissioner Inland Revenue
(b) Commissioner Inland Revenue
(c) Board
(d) Securities and Exchange Commission Pakistan
Q.14 The ______ has the authority to determine the jurisdiction of different tax authorities.
(a) Commissioner Inland Revenue
(b) Chief Commissioner Inland Revenue
(c) Board
(d) Appellate Tribunal Inland Revenue
Q.15 ______ has the authority to issue the tax exemption certificate.
(a) Commissioner Inland Revenue
(b) Chief Commissioner Inland Revenue
(c) Board
(d) Appellate Tribunal Inland Revenue
Q.16 All classes of taxpayers (other than a _________) can change their method of accounting for business income with
prior approval of CIR.
(a) individual
(b) company
(c) AOP
(d) None of above
Q.17 The board may implement a rule which may interfere with the discretion of _________.
(a) Commissioner Inland Revenue (Appeals)

Conceptual Approach to Taxes 313


Income Tax Authorities Chapter-19

(b) Chief Commissioner Inland Revenue


(c) High Court
(d) Appellate Tribunal Inland Revenue
Q.18 Taxation officer of Inland Revenue is a __________ to Commissioner Inland Revenue.
(a) Subordinate
(b) associate
(c) assistant
(d) both a and b
Q.19 _____ has the power to take necessary actions for the recovery of tax.
(a) Commissioner Inland Revenue (Appeals)
(b) Chief Commissioner Inland Revenue
(c) Commissioner Inland Revenue
(d) Appellate Tribunal Inland Revenue
Q.20 Income tax department may appoint any ______ for any work that required special advice.
(a) expert
(b) officer
(c) authority
(d) None of the above
Q.21 ________ have the authority to extend the time for the filing of income tax return.
(a) taxation officer
(b) Commissioner Inland Revenue
(c) Chief Commissioner Inland Revenue
(d) Board
Q.22 CIR can delegate his power whereas ______ cannot do so under the Income tax Ordinance, 2001.
(a) Commissioner Inland Revenue (Appeals)
(b) Chief Commissioner Inland Revenue
(c) Commissioner Inland Revenue
(d) all of the above
Q.23 The delegation of powers by any income tax authority is enforceable __________.
(a) in writing
(b) in oral
(c) both a and b
Q.24 Tax department should keep the particulars furnished by the taxpayer as________.
(a) safe custody
(b) for public information
(c) (c) both (a) and (b)
Q.25 Over ruling of confidentiality is allowed for the disclosure to any person authorized by _____.
(a) board
(b) Commissioner Inland Revenue
(c) Appellate Tribunal Inland Revenue
(d) None of the above
Q.26 Tax department can disclose any fact or particular to _________ by over ruling the confidentially.
(a) foreign government
(b) federal government
314 Conceptual Approach to Taxes
Income Tax Authorities Chapter-19

(c) provincial government


(d) both b and c
Q.27 Service of notice is properly served by delivering it to the authorized representative of a ____________.
(a) resident individual
(b) non-resident individual
(c) AOP
(d) companies
(e) all of above
Q.28 ________ in case of discontinuance of business is the only responsibility of the individual or by his representative.
(a) filing an application
(b) filing of return
(c) notice of discontinuous of business
(d) both b and c
Q.29 Legal advisor falls under the category of an ___________.
(a) income tax officer
(b) expert
(c) consultant
(d) both b and c

ANSWERS
1 (c) 2 (c) 3 (a) 4 (a) 5 (d)

6 (b) 7 (a) 8 (d) 9 (d) 10 (d)

11 (d) 12 (c) 13 (b) 14 (c) 15 (a)

16 (b) 17 (b) 18 (a) 19 (c) 20 (a)

21 (b) 22 (a) 23 (a) 24 (a) 25 (a)

26 (d) 27 (e) 28 (d) 29 (d)

Conceptual Approach to Taxes 315


Income Tax Authorities Chapter-19

ICMAP PAST PAPERS THEORETICAL QUESTIONS


Q.NO. 2(c) August 2014 Tax authorities are required to serve notices, orders or requisitions on certain persons
under the various provisions of the Income Tax Ordinance, 2001.
Required: Identify the manner when it should be treated that the notice has been properly served on following
persons:
(i) resident individuals.
(ii) association of persons (AOPs).

316 Conceptual Approach to Taxes


Income Tax Authorities Chapter-19

CA MOD C PAST PAPERS THEORETICAL QUESTIONS


Q.2 Spring 2002 State the classes of income tax authorities under the Income Tax Ordinance, 1979.
Q.2 (a) Autumn 2002 What is binding on all authorities to follow for the purpose of administration of law of Income Tax
under the Income Tax Ordinance, 1979?
What is specifically prohibited for CBR when issuing its orders, instructions for its subordinate officers?

Conceptual Approach to Taxes 317


Income Tax Authorities Chapter-19

318 Conceptual Approach to Taxes


Exemptions other than Covered in Respective Chapters Chapter-20

Chapter

20 EXEMPTIONS OTHER THAN COVERED


IN RESPECTIVE CHAPTERS

T OPIC COVERED
Section 41 to 55 Exemptions & tax concessions
Part I (2nd Schedule) Exemptions from total income
Part II (2nd Schedule) Reduction in tax rates
Part III (2nd Schedule) Reduction in tax liability
Part IV (2nd Schedule) Exemption from specific provisions

EXEMPTIONS & TAX CONCESSIONS (OTHER THAN SPECIFIED IN RESPECTIV CHAPTERS) UNDER VARIOUS
SECTIONS OF INCOME TAX ORDINANCE, 2001

SECTION PARTICULARS
48 Support payments under an agreement to live apart
49 Federal Government, Provincial Government and Local Government income
51 Foreign-source income of returning expatriates
53 Exemptions and tax concessions in the Second Schedule
54 Exemptions and tax provisions in other laws
55 Limitation of exemption

EXEMPTIONS (OTHER THAN SPECIFIED IN RESPECTIV CHAPTERS) FROM TOTAL INCOME [PART I OF 2ND
SCHEDULE]

CLAUSE PARTICULARS
23B Amounts received as monthly instalment from an income payment plan invested out of the accumulated
balance of an individual pension accounts
23C Any withdrawal of accumulated balance from approved pension fund
57 NIT, ICP, Mutual Funds, collective investment schemes, REIT, Venture Capital, Provident, Gratuity and
Superannuation Funds, Benevolent Funds, group insurance schemes, service funds, EOBI, certain Army
institutes and funds, pension fund, Voluntary pension balance. Sh. Sultan Trust and Sindh Province
pension fund
61 Donations to certain institutions and funds
64A Donations to victims of Terrorism
64B Punjab Relief Fund for internally Displaced Persons (IDPs) of NWFP
64C Flood Relief Fund for victims of flood 2010
65 Donations to research on Islamic history, art and culture, Istanbul
102A Govt. Subsidy
103A Inter-corporate dividend
104 Dividend from Pak Libya Holding
105 Dividend from Saudia Pak Industrial and Agricultural Inv. Co.
105A Dividend from Pak-Kuwait Investment Co.
105B Dividend from Agriculture Income.

Conceptual Approach to Taxes 319


Exemptions other than Covered in Respective Chapters Chapter-20

REDUCTION IN TAX RATES (OTHER THAN SPECIFIED IN RESPECTIV CHAPTERS) [PART II OF 2ND SCHEDULE]

CLAUSE PARTICULARS

5A Withholding tax on profit on debt payable to a non-resident person

24A 1% tax on Distributors of cigarette and pharmaceutical products and for large distribution houses

REDUCTION IN TAX LIABILITY (OTHER THAN SPECIFIED IN RESPECTIV CHAPTERS) [PART III OF 2ND SCHEDULE]

CLAUSE PARTICULARS
2 Currency devaluation/revaluation for petroleum and mineral business

4 Old and used automotive vehicles

EXEMPTION FROM SPECIFIC PROVISIONS (OTHER THAN SPECIFIED IN RESPECTIV CHAPTERS) [PART IV OF 2ND
SCHEDULE]

CLAUSE PARTICULARS
3 Donations to Agha Khan Hospital

5 Immunity to foreign exchange A/c

9A Steel melters, steel re-rollers etc.

9AA Ship breakers

11A NIT or collective investment

11B Non applicability of section 150 on inter-corporate dividends

11C Non applicability of section 151 on inter-corporate Profit on debt

11D LNG Terminal Operators

12 Agricultural Products

16 Non-application of minimum tax and other provisions on institutions of the Agha Khan

16A Non application of section 153(1) on payments against news print media services

19 Exemption to non-residents on securities

36A Bahbood Savings Certificate or Pensioners Benefit Account

38 No tax deduction on special purpose vehicle for securitisation

38A Non-application to Ventures Capital Companies


38C Non-application of tax deduction from dividend income by Islamic Development Bank

41 Exemption and option for presumptive tax for non-residents on contracts

42 Exemption from presumptive tax for certain services for sea-port and infrastructure projects

43A Exemption from deduction on supply of petroleum products imported by the same person.

43B Payments received on sale of air tickets by travelling agents

43C Payments received on supply of petroleum products

43D Payments for rendering or providing by carriage services

43E Exemptions to goods transport contractors paying 2.5% tax

45 Exemption from presumptive tax for purchases by manufacture-cum-exporter

45A 1% WHT on supplies etc. to textiles, carpets, leather, surgical goods, and sports good sector

46 Exemption from presumptive tax for oil distribution and refining

320 Conceptual Approach to Taxes


Exemptions other than Covered in Respective Chapters Chapter-20

46A Exemption from presumptive taxation for manufacturers for iron and steel, individuals or AOP

47A Relief for commercial importer-cum-supplier

47B Payments to NIT, REIT etc

47C Exporter of cooking oil or vegetable ghee to Afghanistan


47D Cotton ginners

56 Withholding tax on various imports

56A Person liable to withholding tax U/S 236E

56B Commercial importer U/S 148

56C Person file return of total income U/S 153 & 169

56D Person file return of total income of contract receipts

56E Person file return of total income of gross amount of services

56F Person file return of total income of gross amount of commission or discount

56G Person file return of total income of gross amount of commission

57 Companies operating large import houses

57A Large import houses

59 Withholding tax on profit on debt

60 Partly designed/assembled cypher devices

61 Cash withdrawal, by earthquake victims

62 Transfer of Assets on amalgamation of companies


63 Exemption to Dawat-ul-Hidiya

65 Profit of clean Development Mechanism Projects

66 Tax on electricity for exporters-cum-manufacturers

67 International Finance Corporation

67A Any stock exchange of Pakistan

68 Pakistan Domestic Sukuk Co. Ltd.

69 Asian Development Bank

70 Contractors of power projects

71 National Highway Authority

72 The ECO Trade and Development Bank

72A Hajj Group Operators

72B Exemption from tax U/S 148 for industrial Undertakings

73 Foreign experts

74 Civil Aviation (CCA)

75 Civil Aviation (CCA)


77 Solar PV panels / modules

77 Renewable sources of Energy

78 Special Economic Zone at Thar

Conceptual Approach to Taxes 321


Exemptions other than Covered in Respective Chapters Chapter-20

81 Section 165 not apply to manufacturer, distributor, dealer and wholesaler

82 Section 116 not apply to individual or a member or AOP

86 Application of provisions of section 111

91 Tillage and seed bed preparation equipment

92 Aircrafts

93 Halal meat production

322 Conceptual Approach to Taxes


Chapter 21 ___________Solved Past Papers Income Tax Numericals of CA Module C - (2001 to 2015)

Chapter

21
SOLVED PAST PAPERS INCOME TAX NUMERICALS OF MOD-
C (2001 TO 2015)
Note: All the following questions have been solved under the Income tax Ordinance, 2001 effective from July 01, 2015.

Q.NO.1 Spring 2015On 1 July 20X4, Tahir commenced business of manufacturing garments. Income statement of
the business for the year ended 30 June 20X5 is as follows:

Notes Rs. in 000


Sales 49,330
Less: Cost of sales (i) (39,150)
Gross profit 10,180
Less: Administrative and selling expenses (ii) (9,140)
Financial charges (iii) (2,500)
Other charges (iv) (1,358)
(2,818)
Add: Other income 3,875
Profit before taxation 1,057

Notes to the income statement:


i. On 15 July 20X4, used machinery was imported from China valuing Rs. 1,500,000. Depreciation @ 15% was
charged on machinery for the whole year and is included in cost of sales.
ii. Administrative and selling expenses include:
Rs. 975,000 paid for the purchase of computer software. The software is likely to be used for twelve years.

Cost of preparation of a feasibility study amounting to Rs. 250,000 which was issued prior to the commencement of business.
Salary of Rs. 50,000 per month was paid to Tahirs brother who handles the financial matters of the business.
iii. Financial charges include Rs. 80,000 pertaining to a vehicle obtained on lease from a leasing company. The cost of vehicle was Rs.
1,300,000. Depreciation of Rs. 260,000 has been included in administrative and selling expenses. Lease rentals paid during the year
amounted to Rs. 300,000.
iv. Other charges include:
running and maintenance expenses of vehicle amounting to Rs. 295,450. Use of vehicle for personal purposes was
approximately 20%.
provision for bad debts amounting to Rs. 25,000.

Other information:
i. Tahir was working in UAE for the past five years and had come back to Pakistan in April 20X4. He received an amount
equivalent to Rs.150,000 from his ex-employer as differential amount on his final settlement in August 20X4.
ii. He sold a plot for Rs. 3,500,000 which was inherited from his father in 20X1. Fair market value of the plot at the time of
inheritance was Rs. 1,500,000.
iii. 5,000 shares were purchased for Rs. 600,000 from initial public offering of a new listed company.

Conceptual Approach to Taxes _______ ___ _______________ __ 323


Chapter 21 _______ Solved Past Papers Income Tax Numericals of CA Module C - (2001 to 2015)

iv. Premium of Rs. 300,000 was paid on Tahirs life insurance policy.

Required:
Under the provisions of the Income Tax Ordinance, 2001 compute the taxable income and tax liability of Tahir for the tax
year 20X5. Provide comments in respect of items which do not appear in your computation.

Mr. Tahir
Resident : Individual
Tax Year : 2016
Computation of Taxable income and tax thereon
Income from Business (W-1) 1,939,840
Total Taxable Income 1,939,840

Tax on Rs. 1,939,840


[144,500 + (1,939,840 - 1,500,000) x 20%] 232,468
Less: Rebate on shares and life Insurance
( 232,468 / 1,939,840 x 387,968 ) (W-3) (46,494)
Tax Payable with Return 185,974

Working - 1
Income from Business

Profit before taxation 1,057,000


Add: Accounting Depreciation 225,000
Pre-commencement Expenses 250,000
Computer software 975,000
Depreciation on Lease Asset 260,000
Lease charges 80,000
Vehicle used for private purpose 59,090
Provision for bad debts 25,000 1,874,090
2,931,090
Less: Amortization of software (975,000 / 10 ) 97,500
Lease rentals paid during the year 300,000
20% Amortization of pre-commencement Expense 50,000
Tax Depreciation (W-2) 543,750 (991,250)
Taxable income from business 1,939,840

Working - 2
Depreciations WDV Depreciation
Cost of Machinery 1,500,000
Less: 25% Initial allowance (375,000) 375,000
1,125,000
Less: Depreciation (168,750) 168,750
Total 956,250 543,750
Working - 3
Higher of investment in shares OR Life Insurance

Shares of listed company A 600,000


Insurance Premium B 300,000
Or C 1,500,000
Or 20% of 1,939,840 D 387,968
Lower of A , B, C & D 387,968

324_________________ ____________ _______ _Conceptual Approach to Taxes


Chapter 21 ___________Solved Past Papers Income Tax Numericals of CA Module C - (2001 to 2015)

Note -1 As the taxpayer is a returning expatriate hence no treatment of foreign source salary has been made as the same
is exempt under section 51 of the Income tax ordinance, 2001.

Note -2 No treatment of gain on disposal of plot has been made as the same is not taxable due to its disposal after two
years from the date of its inheritence.

Q.NO.4(b) Spring 2015 On 1 July 2014, Fahim agreed to rent out a house to Mirza at a monthly rent of
Rs. 180,000 with effect from 1 August 2014 and received one years rent in advance. He also received Rs. 800,000 as a
security deposit which was partly used to repay the security deposit amounting to Rs. 400,000 received from the
previous tenant in July 2010 and partly used for renovation of the house.

Fahim also incurred the following expenses in respect of the above house:
(i) property tax of Rs. 15,000.
(ii) payment of interest amounting to Rs. 200,000 to his friend against amount borrowed for renovation of the
house.
(iii) insurance premium of Rs. 110,000.
(iv) Rs. 5,000 per month to Wasif for collection of rent.

Required:
Under the provisions of the Income Tax Ordinance, 2001 compute the taxable income of Fahim for tax year 2015
assuming he has no other income.

Mr. Fahim
Tax Year 2016
Computation of taxable income
Income from Property
Rent (180,000 x 11) 1,980,000
Unadjustable advance (W-1) 64,000
2,044,000
Less: Deductions u/s 15A
1/5th repair allowance 408,800
Property tax 15,000
Profit on debt for renovation of rented house 200,000
Insurance premium 110,000
Lower of actual collection & admin charges or 6% of RCT 55,000 (788,800)
Taxable Income 1,255,200

W-1
Working for Unadjustable advance
[ 800,000 - ( 400,000 / 10 x 4 ) ] / 10 64,000

Q.NO.6 Spring 2015 Aslam is a resident taxpayer who operates his business from Lahore (LHR) and Paris
(PAR). In August 2014, he established a new branch in Berlin (BER).

Following information is available in respect of his business operations for tax year 2015:

LHR PAR BER


----- Rs. in million -----
Income / (loss) from business 29 40 -15
Advance taxes paid in respective countries during the year 10 5 3
Income from capital gain (net of income tax of Rs. 3 million) - 27 -
Carried forward losses:
Loss from business - 55 -

Conceptual Approach to Taxes _______ ___ _______________ __ 325


Chapter 21 _______ Solved Past Papers Income Tax Numericals of CA Module C - (2001 to 2015)

Capital loss - 6 -

The following amounts paid by Aslam in respect of BER have been charged to LHR:
(i) salaries for the first three months amounting to Rs. 5 million.
(ii) rent expense for the year amounting to Rs. 7 million.

Required:
Under the provisions of the Income Tax Ordinance, 2001 calculate the tax payable by Aslam in the tax year 2015
and foreign tax losses to be carried forward to next year, if any.

Tax Year 2016


Computation of taxable income and tax thereon
LHR PAR BER
Rs. In million Rs. In million Rs. In million
Income / Loss 29 40 (15)
Add / (Less): salaries wrongly charged 5 - (5)
Rent wrongly charged 7 - (7)
41 40 (27)
Income From Business 41 40 (27)
Less: Adjustment against loss from Business b/f (55-40)= 15 c/f - (40)
(A) 41 - (27)

Income from Capital Gain - 30 -


Set-off Capital Loss - (6) -
(B) - 24 -

Total Income [A + B] 41 24 (27)


Total Taxable Income 41 24 (27)

The un-adjusted business loss of PAR of Rs. 15 million and Ber of 27 million shall be carried forward for adjustment
against succeeding years foreign source business income with limitation of maximum upto six tax years.

COMPUTATION OF TAX LIABILITY:


Tax on Rs. 65,000,000
[1,319,500 + (65,000,000 - 6,000,000) x 35%] = 21,969,500 22

Less tax credit for foreign source capital gain (Lower of Pakistan (3)
average tax OR Foreign tax paid)
19
Less advance tax paid (10)
Balance tax payable 9

No tax credit on foreign (PAR) source business income has been computed as the tax on such income in Pakistan is nil
as comared to Rs. 3 million in Ber.

Q.1CAF September 2014 Sultan is working as electronic engineer with Ansari Electrical Company Limited (AECL). He
has provided you with the following information for the tax year ended 30 June 2014:

(a)His monthly cash remuneration in AECL is as follows:


Rupees
Basic salary 480,000
Medical allowance 48,000
Utilities allowance 55,000
Market value of rent free accommodation 75,000

326_________________ ____________ _______ _Conceptual Approach to Taxes


Chapter 21 ___________Solved Past Papers Income Tax Numericals of CA Module C - (2001 to 2015)

(b)He was also provided the following benefits in accordance with the terms of his employment:
(i)Leave encashment amounting to Rs. 300,000.
(ii) Hospitalization cost is covered by an insurance policy up to the amount of Rs. 1.5 million. The insurance
premium relating to this benefit amounted to Rs. 55,000.
(iii) He is allowed to use his personal car for office use. Reimbursement of car running and maintenance expenses
amounted to Rs. 550,000. 15% of these expenses pertain to personal use.

(c)Rs. 200,000 were received from a private limited company for attending board meetings.
(d)A lump sum amount of Rs. 1.2 million was received as the author of a literary work.
(e)Sultan took three years to complete this literary work.
(f)Sultan is also a part time singer and owns a studio. He sold the premises in which the studio was situated for Rs. 10
million and shifted his musical instruments to new premises which he purchased for Rs. 15 million. He received Rs. 2.5
million from his father in cash as loan to pay for his new studio. The previous premises was purchased several years ago
for Rs. 1.4 million and had a tax written down value of Rs. 600,000 at the time of disposal.
(g) The net income from the studio for tax year 2014 was Rs. 990,000. The expenses include salaries of two
workers at Rs. 15,000 and Rs. 18,000 per month and utility bills amounting to Rs. 110,000. All expenses were paid in
cash.
(h) He won a car, in a competition held by Star Motor Limited for promotion of its sales. The fair market value of the car
was Rs. 850,000.
(i)He gifted 40 fans having a fair market value of Rs. 100,000 to an approved charitable organisation.
(j)An amount of Rs. 500,000 was paid by him as contribution to an approved pension fund.

Required:
Under the provisions of the Income Tax Ordinance, 2001 compute the taxable income and tax thereon for the tax year
2014.

Mr. Sultan
Resident : Individual
Tax Year 2016
Compitation of taxable income and tax thereon
Rs. Rs.
Income from Salary
Basic Salary 5,760,000
Medical allowance 576,000
Utilities allowance 660,000
Accommodation
Rent free accommodation 900,000
45 % of 5,760,000 2,592,000 2,592,000
Leave encashment 300,000
Conveyance allowance 82,500
9,970,500
Income from Business
Studio Income 990,000
Inadmissible expense 216,000
Gain on sale of studio 800,000 2,006,000

Income from other sources


Fee for attending BOD meetings 200,000
Loan received from father 2,500,000
Literary work as an author 1,200,000
Winning a car 850,000 4,750,000
Total Income 16,726,500
less: Winning a car covered under FTR (850,000)

Conceptual Approach to Taxes _______ ___ _______________ __ 327


Chapter 21 _______ Solved Past Papers Income Tax Numericals of CA Module C - (2001 to 2015)

Taxable Income 15,876,500

Computation of Tax Liability


Tax on Rs. 12,944,500
[1,425,000 + (15,876,500 - 7,000,000) x 30%] 4,087,950
Less: Rebate on donation
4,087,950 / 12,944,500 x 100,000 (25,748)
Less: Rebate on contribution to APF
4,087,950 / 12,944,500 x 500,000 (128,742)
Tax payable with return 3,933,459

Q. NO. 1 Spring 2014 Qamar is engaged in the business of manufacturing and repair of electric motors. His
accountant has prepared the following tax computation for the tax year 20X4:

Rs. in '000' Rs. in '000'

Sales of manufactured motors 45,000


Less: Cost of sales and administrative expenses
(excluding depreciation for the year) (33,000)
Income before depreciation 12,000
Less: Tax depreciation for the year (9,000)
3,000
Less: Brought forward business loss from tax year 20X3
(Total business loss was Rs. 4 million) (3,000)
Business income after adjusting business loss -
Add: Interest income received from a commercial bank 500
Income form other sources 850 1,350
Taxable income 1,350

Computation of tax
Tax liability 125
Less: Tax deducted by bank on interest income (500,000 x 10%) (50)
Tax payable 75

Following expenses are included in the cost of sales and administrative expenses:

Description Rs. In 000


Travelling expenses include travel and hotel expenses of Qamar's
visit to Malaysia for attending a trade fair 100

Electricity charges paid for Qamar's residence 150


Damages paid to a distributor for delayed supplies 200
Donations to a non-profit organization 300
Salary paid to Bari who is Qamar's brother. Advance tax has been 720
deducted from the salary
Fine paid to the Ministry of Environment for infringment of
200
environmental and safety laws
Unabsorbed depreciation brought forward from previous tax year 500

Qamar is not satisfied with the tax return prepared by his accountant and has requested you to review the return.

Required:
(a) Compute the revised taxable income of Qamar and tax payable by or refundable to him for the tax year 20X4.
(b) Briefly comment on treatment of the above items of expenses in your tax computation.

328_________________ ____________ _______ _Conceptual Approach to Taxes


Chapter 21 ___________Solved Past Papers Income Tax Numericals of CA Module C - (2001 to 2015)

Solution: (a)

MR. QAMAR
Revised taxable income & tax thereon
For the tax year 2016
Rs. in '000'
INCOME FROM BUSINESS U/S 18

Sales of manufactured motors 45,000


Less: Revised cost of sales and administrative expenses
(excluding depreciation for the year) (N - 2) (31,850)
13,150
Less: Brought forward business loss from tax year 20X3
(Total business loss was Rs. 4 million) (4,000)
9,150
Less: Tax depreciation for the current year (9,000)
Unadjustable tax depreciation - brought forward (b - 1) (500)
Total business loss for the year (350)

INCOME FROM OTHER SOURCES U/S 39

Income form other sources (assumed net of allowable expenses and fully
covered under normal tax regime) 850
Total income 500
Less straight deduciton U/C 61 of Part II of 2nd Schedule to the

Income tax Ordinance, 2001. [Rs. 300,000 or 30% of Rs. 500,000] (b - 2) 150
Taxable income 350

COMPUTATION OF TAX LIABILITY:

As the income is less than Rs. 400, 000 (maximum non taxable limit) hence no tax liability has been
computed under normal tax regime (read with N - 1).

Tax under NTR -


Tax under FTR on profit on PLS account 50
50
Less tax already deducted at source - on bank profit 50
Balance tax -

ASSUMPTIONS / BASIS:

(N - 1) In the absence of information no minimum tax under section 113 of the Income Tax Ordinance, 2001 has been
computed.

(N - 2) Revised cost of sales


Rs. Rs.
Cost of sales as per question 33,000
Add:

Less:
Electricity charges - residence 150
Donation to a non-profit Orgnization 300
Unabsorbed depreciation brought forward from previous tax year 500
Fine paid for infringement of environmental laws 200
(1,150)
Revised cost of sales 31,850

Conceptual Approach to Taxes _______ ___ _______________ __ 329


Chapter 21 _______ Solved Past Papers Income Tax Numericals of CA Module C - (2001 to 2015)

Solution: (b)

(1) Un-absorbed depreciation of (current year and brought forward) may be adjusted against any other head of income
chargeable to tax (except income under the head salary or income from property) under section 57(4) read with section
56(1) of the Income Tax Ordinance, 2001.

(2) It has been assumed that the limitations of clause 61 has been met by the AOP in order to claim the same as
straight deduction from total income.

(3) As the personal expenses are not allowable under section 21(h) of the Income tax Ordinance, 2001 hence the
electricity charges of residence has been accordingly deducted from the given cost of sales amount.

(4) As the fine or penalty for voilation of any law, rule or regulation is not allowable under section 21(g) of the Income tax
Ordinance, 2001 hence the fine paid for infirngment of enviornmental laws has been accordingly deducted from the given
cost of sales amount.

Q. NO. 4 Spring 2014 Basher and Jamil jointly own a house in Karachi. Basher has 75% share in the house. On 1
September 20X3, the house was let out at an annual rental of value of Rs. 6,500,000. This amount includes Rs. 186,000
per month for utilities, cleaning and security.

During the tax year 20X4, owners incurred the following expenditures in relation to the house:

Rupees
Utilities, cleaning and security 650,000
Repair and maintenance 810,000
Insurance premium 240,000
Collection charges 25,400
Mark-up on amount borrowed for extension of the house 840,000

Basher and Jameel have no other sources of income. All the above expenses were incurred by them jointly.

Required
Calculate taxable income of Basher and Jameel under appropriate heads of income for the tax year 20X4.

Solution:

COMPUTATION OF TAXABLE INCOME


FOR THE TAX YEAR 2015

INCOME FROM PROPERTY U/S 15: Rs. Rs. Rs.

Rent [(Rs. 6,500,000 / 12 x 10) - 1,860,000] 3,556,667

Less: allowable deductions u/s 15A

Repair allowance (Rs. 3,556,667 x 1/5) 711,333


Insurance premium 240,000
Actual collection & admin charges 25,400
6% of rent chargeable to tax 213,400
Lower of two amounts 25,400
Mark-up on amount borrowed 840,000 1,816,733
Net income from property of Co-owners 1,739,934

INCOME FROM OTHER SOURCES U/S 39:

Income from utilities, cleaning and securities (Rs. 186,000 x 10) 1,860,000

330_________________ ____________ _______ _Conceptual Approach to Taxes


Chapter 21 ___________Solved Past Papers Income Tax Numericals of CA Module C - (2001 to 2015)

Less expenses of utilities, cleaning and securities (650,000)


Total taxable income 1,210,000

COMPUTATION OF TAX LIABILITY AS AOP:

Tax on Rs. 1,210,000 [Rs. 32,000 + (Rs.1,210,000 - Rs. 750,000) x 15%] 101,000
Divisible income afer tax 1,109,000

Taxable income of Co-owners: Property income Income from Total


other sources
after tax (N - 1)
Rs. Rs. Rs.

Bashir 75% 1,304,950 831,750 2,136,700


Jameel 25% 434,983 277,250 712,233
Total taxable income 1,739,934 1,109,000 2,848,934

(N - 1) Share of both the members in income from other sources of the AOP has been included for rate purposes of
respective member of the AOP.

Q.1 Autumn 2013

Mrs. Aslam was employed with Sahal Limited (SL) as a Marketing Manager. On 30 June 2012 she resigned from her
employment with SL. On 1 July 2012, she joined Hassan Pakistan Limited (HPL), a quoted company, as a Marketing
Director. She has provided you the following information in respect of the tax year 2013:

(i) In July 2012, she received following amounts from SL in final settlement:
LeaveencashmentamountingtoRs.95,000.
GratuityofRs.500,000fromanunrecognizedgratuityfundmaintainedbySL.
ReimbursementofRs.100,000againstahealthinsurancepolicy.TheinsuranceclaimwaslodgedbySLon
behalf of Mrs. Aslam in January 2012.

(ii) In accordance with the terms of her employment, income tax related to her salary and benefits is to be borne by HPL.
Her emoluments / benefits during the tax year were as follows:

BasicsalaryofRs.200,000permonth.
MedicalallowanceofRs.60,000permonth.
RentfreeaccommodationwithannuallettingvalueofRs.480,000.
TravellingallowanceofRs.50,000permonth.60%oftheamountwasspentintheperformanceofofficial
duties.
Providentfund@10%ofbasicsalary.AnequalamountwascontributedbyHPL.

(iii) Under an employee share scheme, Mrs. Aslam was awarded 5,000 share in HPL on 1 January 2013. Under the
scheme she was not allowed to sell the shares up to 31 March 2013. she sold all the shares in HPL on 1 May 2013. Fair
value of the shares on the above dates was as follows:

Rs.20pershareon1January2013
Rs.28pershareon31March2013
Rs.32pershareon1May2013

(iv) On 31 December 2012, she received a loan of Rs. 400,000 from HPL. The loan carries a mark-up of 4% per
annum. The prescribed benchmark rate is 10%.

(v) She won the best executive employee award of HPL and received a laptop having a fair market value of Rs.
100,000
(vi) An amount of Rs. 355,000 was received from her spouse as support payment, under an agreement to live apart.

Conceptual Approach to Taxes _______ ___ _______________ __ 331


Chapter 21 _______ Solved Past Papers Income Tax Numericals of CA Module C - (2001 to 2015)

(vii) She paid Rs. 105,000 as zakat under the Zakat and Ushr Ordinance, 1980.
(viii) Donation of Rs. 70,000 was paid to an approved organization.

Required:
Compute the taxable income, tax liability and tax payable for the tax year 2013.
Note: Show all relevant exemptions, exclusion and disallowances.

Solution
MRS. ASLAM
SALARIED RESIDENT INDIVIDUAL
COMPUTATION OF INCOME & TAX THEREON
FOR THE TAX YEAR 2016
Rupees Rupees Rupees
Exempt /
Total income income not Taxable income
taxable
INCOME FROM SALARY U/S 12

From sahal limited (SL)


Leave encashment 95,000
Gratuity (N-1) 500,000 75,000 425,000

Reimbursement of insurance -

From hassan pakistan (HPl)


Basic salary 2,400,000
Medical allowance (N-2) 720,000 240,000 480,000

Rent free accommodation 480,000


Higher of Rs. 480,000 or Rs. 1,080,000 (45% of 2,400,000) 1,080,000 1,080,000
Travelling allownce (N-3) 600,000 360,000 240,000
Contribution to Provident fund (N-4) -
Employee share scheme 140,000 - 140,000
Loan from employer (N-5) 400,000 -
Award of best executive employee (N-6) 100,000
Total salary 4,960,000

CAPITAL GAINS U/S 37

Amount received to live apart (N-7) 355,000 -


Income before SBI 4,960,000
Less: Zakat (105,000)
Donation (assumed that organization was specified in clause 61 of 2nd schedule) (N-8) (70,000)
Taxable income 4,785,000

Separate Block of Income


capital gain on shares [Rs. (32-28) x 5,000 ] 20,000

COMPUTATION OF TAX LIABILITY:

Tax on Rs. 4,785,000 [597,000 + 27.5% x (4,785,000 - 4,000,000)] 812,875


Tax on capital gain on shares of listed company (Rs. 20,000 x 15% as sold within one year) 3,000
Total tax liability 815,875

ASSUMPTIONS / BASIS:

332_________________ ____________ _______ _Conceptual Approach to Taxes


Chapter 21 ___________Solved Past Papers Income Tax Numericals of CA Module C - (2001 to 2015)

N-1 Gratuity
Contribution to un-recognised gratuity fund U/C 13 is exempt upto Rs. 75,000 or 50% of the amount whichever is lower.

N-2 Medical allowance


It is exempt upto 10% of the basic salary remaining is taxable.
N-3 Travelling
It is assumed that the travelling allowance is given for both office and personal use so 60% of it is exempt for office use.

N-4 Provident fund


It is not mentioned in question what is the type of provident fund it is so it is assumed that the provident fund is un-
recognised provident fund so it is not taxable.
N-5 Loan from employer
Amount of loan is below Rs. 500,000 so it is exempt.
N-6 Award
Any benefit given by employer to employee is taxable under the head income from salary like commission paid to
employees.
N-7 Agreement to live apart
It is exempt under non-recognition rule.

N-8 Donation
No comparison with amount worked out at 30% of taxable income has been made as the actual donation amount is
already less than the same.

Q.3 Autumn 2013

Gulzar is a Pakistani resident and operates various businesses. He disposed off the following assets during the tax
year 2013:

(i) An immovable property was sold for Rs. 50 million. The cost of the immovable property was Rs. 25 million. Tax
depreciation of Rs.4 million had been allowed on the immovable property up to the tax year 2012.

(ii) A car was disposed of for Rs. 1.2 million. The car was acquired on 1 July 2011. The tax written down value of the car
at the beginning of tax year 2013 was Rs.0.9 million. The car was being used partly (70%) for business purposes. The
rate of depreciation for tax purposes is 20%.

(iii) An antique sculpture was purchased for Rs. 350,000 on 30 August 2000. It was sold for Rs. 1,500,000 an 28
February 2013 through auction. The auctioneer was paid a commission of Rs. 15,000 . Tax was deducted and paid by
Gulzar from the amount of commission within due date.

(iv) Listed securities were sold as follows :

purchase cost Sale proceeds


securities Date of purchase Date of sale
(Rs.) (Rs.)
A 20-Nov-12 500,000 17-Mar-13 400,000
B 05-Aug-12 320,000 08-Jun-13 600,000
C 01-Jun-12 650,000 17-Jun-13 700,000

Required:
Compute the amount of capital gain / loss arising on the above transactions under the provisions of the Income Tax
Ordinance, 2001.

Solution

MR. GULZAR

Conceptual Approach to Taxes _______ ___ _______________ __ 333


Chapter 21 _______ Solved Past Papers Income Tax Numericals of CA Module C - (2001 to 2015)

NON SALARIED RESIDENT INDIVIDUAL


COMPUTATION OF INCOME & TAX THEREON
FOR THE TAX YEAR 2016

Rs. Rs.
Immoveable property:
Consideration received from the sale of immovable property 50,000,000
Less: WDV of immovable property
Cost of asset U/S 22(13)(d) 50,000,000
Less: Depreciation charged upto 2013 (4,000,000) 46,000,000
Gain on sale of immovable property taxable under the head 4,000,000
Business income U/S 18

Antique sculpture:

Sale price U/S 76(10) 1,500,000


Cost of antique (including disposal expenses) (Rs. 350,000 + 15,000) 365,000
1,135,000
As holding period is more than 1 year, therefore 25% is exempt (283,750)
Capital gain on sale of antique 851,250

Depreciable motor vehicle:

Consideration received from the sale of depreciable motor vehicle 1,200,000

Less: WDV of motor vehicle


Cost of motor vehicle (Rs. 900,000 x 100 / 85) 1,058,824
Less tax depreciation allowed upto tax year 2012 (Rs.1,058,824 x Note attached (111,177) 947,647
15% x 70%)

Gain on sale of depreciable motor vehicle taxable under the 252,353


head Business income

Note: The cost of motor vehicle not plying for hire is well within the
maximum limit of Rs.2.5 million for pessanger transport vehicle &
further no initial allowance has been claimed on the assumption
that the motor vehicles are not for hiring purposes. Although the
normal depreciation rate of 20% on motor vehicles has been given
however 15% applicable rate as per 3rd Schedule has been used
for the solution.

Listed Companies shares: Company C

Sale price 700,000


Less: cost of shares at the date of disposal 650,000
50,000 50,000

Listed Companies shares: Company A

Sale price 400,000


Less: cost of shares at the date of disposal 500,000
(100,000) (100,000)
As holding period is less than one year therefore its loss may be
adjusted against capital gain from other taxable securities

Listed Companies shares: Company B

334_________________ ____________ _______ _Conceptual Approach to Taxes


Chapter 21 ___________Solved Past Papers Income Tax Numericals of CA Module C - (2001 to 2015)

Sale price 600,000


Less: cost of shares at the date of disposal 320,000
As holding period is less than 1 year, therefore its gain may be 280,000 280,000
adjusted against capital loss of Company A
Taxable capital gain under section 37A 230,000

Q.1 Spring 2013


Mr. Creative is working as Director Human Resources at Artistic Technologies Limited (ATL). Following are the details of
his income / receipts during the latest tax year.

(a) Monthly cash remuneration from ATL:

Basic salary Rs.300,000


Utility allowance 15% of basic salary
Medical allowance 12% of basic salary

(b) In addition to above, he was also provided the following benefits in accordance with his terms of employment:
(i) Rent-free furnished accommodation in a bungalow situated on a 500 square yard plot of land. Rent for comparable
accomodation facility in the vicinity is Rs 150,000 per month.

(ii)An 1800cc company maintained car. The car was purchased two years ago at a cost of Rs. 1,600,000 and is used
both for official and personal purposes.

(c) A house owned by Mr. creative had been leased out by him at a monthly rent of Rs 50,000.
The Lease expired on 31 December. Mr. Creative refused to renew the lease in spite of the tenant's offer to renew the
lease offer after increasing the rent by 10%.
He returned the non-adjustable deposit of Rs.300,000 to the tenant, which was received two years ago. The house was
immediately leased to his cousin without any security deposit on a monthly rent of Rs.48,000.

(d) Five years ago, Mr. Creative had purchased 20,000 shares of Rs.10 each, of an unlisted public company at the rate of
Rs.140 per share. After one year of acquisition, he received 8,000 bonus shares from the company. During the latest tax
year, he sold 75% of the bonus shares at a price of Rs. 145 per share.

e) During the latest tax year, following investments were made:


Rs.
Approved voluntary pension fund 600,000
Open end mutual fund 1,100,000

(f) During the latest tax year, he redeemed 4,000 units of an open-end mutual fund at Rs. 50 per unit and Mr. creative had
claimed a tax credit of Rs.40,000 on this investment.

(g) Donations of Rs.50,000 were paid to charitable institutions listed in the second schedule to the Income Tax
Ordinance, 2001.

(h) Tax deducted at source from his salary was Rs.737,000.

Required:
Compute the taxable income, tax liability and tax payable for the latest tax year.

Solution

Mr. Creative (Resident)


Computation of taxable income and tax liability
For the tax year 2016

INCOME FROM SALARY U/S 12 Note Rupees Rupees

Conceptual Approach to Taxes _______ ___ _______________ __ 335


Chapter 21 _______ Solved Past Papers Income Tax Numericals of CA Module C - (2001 to 2015)

Basic salary (Rs. 300,000 12) 3,600,000


Utility allowance (300,000 12 15%) 540,000
Medical allowance (300,000 12 x 12%) 432,000
Less: Exempt up to 10% of basic salary U/C 139 (360,000) 72,000
Rent free furnished accommodation (Higher of fair market rent Rs. 1,800,000
1,800,000 or 45% of basic pay (1,620,000) shall be added in the
taxable salary income)
Company maintained car (1,600,000 5%) U/R 5 80,000
6,092,000
INCOME FROM PROPERTY U/S 15

Rent (Rs. 50,000 6 months) + (Fair market rent Rs.55,000 x 6 N-1 630,000
months) (Assumed after allowable deductions)

INCOME FROM CAPITAL GAINS U/S 37

Gain on sale of bonus shares of unquoted Company (8,000 x 75% 870,000


x Rs.145 each)
Less cost of bonus shares (Rs.6,000 x 2,800,000 / 28,000) (600,000)
(In the absence of information it has been assumed that the cost of
bonus shares computed as above is equal to fair market value at
which 5% tax U/S 236N has already been paid under final tax
regime, otherwise the fair market value on which 5% tax has been
paid shall be considered as cost of bonus issue)
Capital gain 270,000
75% capital gain taxable 202,500

Total income 6,924,500


Less: Donation to charitable institutions (Lower of 30% of taxable (50,000)
income or actual amount paid hence acutal amount considered
under 2nd Schedule)
Taxable income 6,874,500

COMPUTATION OF TAX LIABILITY:

Tax on Rs. 6,874,500 [597,000 + 27.5% x (6,874,500 - 4,000,000)] 1,387,488

Less tax credit on


Approved voluntary pension fund (lower of actual amount 600,000
Rs.600,000 or 20% of taxable income i.e. Rs. 1,374,900)

Open end mutual fund (No tax credit computed on this N-2 -
investment as the same was made in the latest tax year and further
due to partial encashment within 24 months period the tax credit of
preceeding tax year shall be adjusted in the latest tax year and not
in the current tax year).
600,000
Less tax credit Rs. 1,387,488 / 6,874,500 x 600,000 (121,099)
1,266,389
Less tax deducted at source against salary income (737,000)
Balance tax payable 529,389

Notes
N-1: No treatment of security refunded after two years has been made in the current year as no further security
received by the landlord.

336_________________ ____________ _______ _Conceptual Approach to Taxes


Chapter 21 ___________Solved Past Papers Income Tax Numericals of CA Module C - (2001 to 2015)

N-2: In the absence of information it has been assumed that all the open end fund units sold during the current tax
year.

Q.No. 3 (b) Spring 2013

Imaginative Enterprise (IE) is an Association of persons and was formed two years ago. During the latest tax year, IE's
Pakistan source income amounted to Rs.2,500,000 and tax payable thereon amounted to Rs. 722,500.

Following are the details of its foreign source incomes, tax paid thereon and foreign losses brought forward for the latest
tax year:
Heads of income Foreign income Foreign tax paid Foreign losses
brought forward
RUPEES RUPEES RUPEES
Speculation business 500,000 125,000 (250,000)
Non- Speculation business (1,000,000) - -
Capital gains 750,000 75,000 (1,500,000)
Other sources 1,250,000 187,500 -
The foreign tax credit relating to Income from other sources which remained unadjusted during the last tax year
amounted to Rs.50,000.

Required:
Calculate total tax payable and foreign tax losses to be carried forward to next year (if any).

Solution

Imagivative Enterprise (Resident AOP)


Computation of taxable income and tax liability
For the tax year 2016
Rs. Rs.
Foreign source of incomes :
Non speculation:
Current year loss (N - 1) (1,000,000)

Capital gain U/S 37


Current year income 750,000
Brought forward losses 1,500,000
(750,000)
The unadjusted capital loss to c/f for adjustment against succeeding years capital 750,000 -
gains.

Other sources U/S 39


Current year income 1,250,000
Balance income from other sources 250,000

Speculation U/S 18
Current year income 500,000
Brought forward losses (250,000) 250,000
Net foreign source of income 500,000
Add Pakistan source of income 2,500,000
Total taxable income 3,000,000

COMPUTATION OF TAX LIABILITY:

Tax on Rs. 3,000,000 [344,500 + 25% x (3,000,000 - 2,500,000)] 469,500

Conceptual Approach to Taxes _______ ___ _______________ __ 337


Chapter 21 _______ Solved Past Papers Income Tax Numericals of CA Module C - (2001 to 2015)

Less: tax credit on foreign source of income under respective


heads:
Tax credit on income from other sources shall be allowed on lower
- Foreign tax paid on income from other sources or 187,500
- Pakistan tax on net foreign source of income
(Rs.469,500 / 3,000,000 x 250,000) 39,125
Hence lower allowed as tax credit (39,125)

Less: tax credit on foreign source of income under respective


heads:
Tax credit on speculation income shall be allowed on lower of:

- Foreign tax paid on speculation income or 125,000


- Pakistan tax on net speculation income
(Rs.469,500 / 3,000,000 x 250,000) 39,125
Hence lower allowed as tax credit (39,125)
Balance tax payable 391,250

Notes:
1. Non speculation losses adjusted against firstly income form other sources and balance against specualtion
income.
2. Last year unadjusted foreign tax credit on income from other sources is not allowed to be refunded, carried
forward or carried back.
3. The income tax given in question only on Pakistan source of income has been ignored as credit shall be on total
tax liability basis that is Pakistan source income plus foreign source income.

Q.1 Autumn 2012

Beena Sikandar is a lawyer and owns a law firm under the name Beena and Co. She is also Director Legal Affairs at
Ayesha Foods Limited. Details of her income for the tax year 2012 are as follows:
(A) INCOME FROM BEENA and CO.
Income Statement Note Rupees
Revenue (i) 8,500,000
Less: Expenses
Salaries (ii) 2,000,000
Gifts and donations (iii) 400,000
Lease charges (iv) 900,000
Professional fee (v) 400,000
Property expenses (vi) 350,000
Travel expenses 150,000
Other expenses (vii) 600,000
Tax withheld by clients 200,000
5,000,000
Net profit 3,500,000

Notes to the Income Statement


(i) Revenue includes Rs. 750,000 recovered from Rafia in respect of bad debts that had been written off while calculating
the taxable income for the tax year 2010. The amount was receivable against professional services rendered to Rafia.

(ii) Salary expenses include amount of Rs. 50,000 and Rs. 75,000 per month paid to Beena and her brother respectively.
Her brother looks after administration and financial matters of the firm.
(iii) Gifts and donations include gifts to clients, gift to her son and donation to Edhi Foundation amounting to Rs. 100,000,
Rs. 50,000 and Rs. 250,000 respectively.
(iv) A vehicle was obtained solely for official purposes on operating lease, from a bank. The lease commenced on 1 March

338_________________ ____________ _______ _Conceptual Approach to Taxes


Chapter 21 ___________Solved Past Papers Income Tax Numericals of CA Module C - (2001 to 2015)
(iv) A vehicle was obtained solely for official purposes on operating lease, from a bank. The lease commenced on 1 March
2012. Lease charges include Rs. 500,000 paid as security deposit to the bank.
(v) The professional fee includes an amount of Rs. 150,000 paid to a legal firm for defending a law suit filed against
Beena, in a family court.
(vi) Beena lives in an apartment situated above her office, and two-fifths of the total property expenses relates to this
apartment.
(vii) Other expenses include an amount of Rs. 150,000 paid for Beena's Golf Club membership which she exclusively
used to promote her business interests. The payment to the club was made in cash.
(B) DIRECTOR'S REMUNERATION FROM AYESHA FOOD LIMITED
(i) Beena received monthly remuneration of Rs. 100,000 from AFL.
(ii) During the year, she also received two bonus payments of Rs. 100,000 each. One of the bonus pertains to tax year
2011. It was announced last year but disbursed to her in the current year.
(iii) Beena has also been provided a vehicle, by AFL, for her personal as well as business use. The car was acquired by
AFL in May 2007 at a cost of Rs. 2,000,000. The fair market value of the car as at 30 June 2012 was Rs. 1,500,000.

(iv) She received a fee of Rs. 150,000 from AFL for attending the meetings of the Board of Directors (BOD).
(v) Details of tax deducted by AFL are as follows:
Rupees
From salaries 390,000
From fee received for attending the meetings of BOD 9,000
Required:
Compute the taxable income, tax liability and tax payable by Beena Sikandar for the tax year 2012. Provide appropriate
comments on the items appearing in the notes which are not considered by you in your computation.

Solution
Beena Sikandar (Resident)
Computation of taxable income and tax liability
For the tax year 2016
Notes Rs.
INCOME FROM BUSINESS U/S 18
Profit before tax as per accounts 3,500,000
Add:
Tax withheld by clients 200,000
Salary paid to Beena 600,000
Gifts to clients and son 1 150,000
Donation to Edhi Foundation 1 250,000
Security deposit on operating lease 2 500,000
Legal charges paid for personal law suit 150,000
Rent related to personal apartment (2/5 x 350,000) 140,000
Club membership fee paid in cash 150,000
2,140,000
5,640,000
INCOME FROM SALARY U/S 12
Monthly remuneration from AFL (Rs. 100,000 x 12) 1,200,000
Bonus received in tax year 2012 3 200,000
Conveyance (2,000,000 x 5%) U/R 5 100,000
Fee as employee for attending meeting of the BOD (N - 4) 150,000
1,650,000
Total taxable income 7,290,000

COMPUTATION OF TAX LIABILITY:

Conceptual Approach to Taxes _______ ___ _______________ __ 339


Chapter 21 _______ Solved Past Papers Income Tax Numericals of CA Module C - (2001 to 2015)

Salary is less than 50% of taxable income, hence rates of non-salaried individual applied.
Tax on Rs. 7,290,000 [1,319,500 + 35% x (7,290,000 - 6,000,000)] 1,771,000
Less: Proportionate tax on income chargeable to tax under
minimum tax liability (1,771,000 / 7,290,000 x 5,640,000) 1,370,156
400,844
Proportionate tax (as above) (A) 1,370,156
Minimum tax (deductible) (8,500,000 x 10%) (B) 850,000

Higher of (A) or (B) 1,370,156


1,771,000
Less: tax credit on donation
Tax credit shall be allowed on lower of:
- Actual amount of donation i.e. Rs. 250,000
- 30% of taxable income i.e. Rs. 2,187,000
Tax credit = [250,000 x 1,771,000 / 7,290,000] 60,734
Tax liability 1,710,266
Less: tax already paid or deducted at source
Tax withheld by clients 200,000
Tax deducted on salaries 390,000
Tax deducted on fee received as employee 9,000
599,000
Balance tax payable 1,111,266

Comments on items not considered in computations


a. Bad debts recovered
As bad debts of Rs. 750,000 was allowed as expense in tax year 2010 and now this amount has been recovered,
therefore, this amount is chargeable to tax. No treatment is required as this amount has already been included in revenue.

Notes
1. Gifts and donation
It is assumed that gifts are given in personal capacity, therefore not allowed for tax purposes. Tax credit shall be allowed
on donation to Edhi Foundation.
2. Security deposit on lease
It is assumed that security deposit does not form part of the lease rentals and shall be refunded by bank on repayment of
all lease rentals.
3. Bonus
Salary including bonus is chargeable to tax on receipt basis.

4. Meeting fee to director


As the meeting fee has been recived by the taxpayer as an employee hence the same is taxed as income under the head
salary income instead of taxation under the head income from other sources.

Q.5 (a) Autumn 2012

In May 2012, Hameeda sold certain personal assets at the following prices:
Rupees
Plot in DHA Karachi 10,000,000
Paintings 2,000,000
Jewellery 5,000,000

Additional information
(i) Plot in DHA Karachi was inherited by her from her father in May 2006. It was purchased by her father for Rs. 4,000,000
and market value at the time of inheritance was Rs. 5,000,000.

340_________________ ____________ _______ _Conceptual Approach to Taxes


Chapter 21 ___________Solved Past Papers Income Tax Numericals of CA Module C - (2001 to 2015)

(ii) Paintings were inherited from her mother in July 2011. these paintings were purchased by her mother for Rs.
1,000,000 and market value at the time of inheritance was Rs. 2,350,000.

(iii) Jewellery costing Rs. 3,000,000 was purchased and gifted to her by her husband in March 2009.

Required: Discuss the taxability of Hameeda in respect of the above gains/ losses on sale of assets in the context of
Income Tax Ordinance, 2001.

Solution

Hameeda (Resident)
Computation of taxable income and tax liability
For the tax year 2016

CAPITAL GAINS U/S 37 Rs. Rs.

Paintings:
Sale price 2,000,000
FMV at the date of inheritance 2,350,000 (350,000)
As the loss on painting is covered under section 38(5) hence the same
has not been considered for computation of taxable gain.

Jewellery:
Sale price 5,000,000
FMV at the date of inheritance (assumed equal to cost) 3,000,000
2,000,000
As holding period is more than 1 year, therefore 25% is exempt (500,000) 1,500,000

Plot in DHA: (Separate block of income)


Sale price 10,000,000
FMV at the date of inheritance 5,000,000
5,000,000
As holding period is more than 2 year, therefore gain is exempt (5,000,000) -
Taxable income under normal law 1,500,000

COMPUTATION OF TAX LIABILITY:

Tax on Rs. 1,500,000 [32,000 + 15% x (1,500,000 - 750,000)] 144,500

Q.5 (b) Autumn 2012

On 1 July 2010, Kashmala and Shumaila formed an Association of Persons (AOP) with the objective of providing
information technology support services to corporate clients. They contributed Rs. 1.2 million and Rs. 0.8 million
respectively in their capital accounts and agreed to share profits and losses in the ratio of their capitals.
For the year ended 30 June 2011, business loss and unabsorbed depreciation of Rs. 0.4 million and Rs. 0.3 million
respectively were assessed and carried forward. The total turnover of the AOP in 2011 was Rs. 40 million.
During the year ended 30 June 2012, the AOP incurred a net loss of Rs. 0.8 million on a turnover of Rs. 50 million. The
cost for the year was arrived after adjustment of the following:

(i) Salaries amounting to Rs. 0.5 million and Rs. 0.3 million were paid to Kashmala and Shumaila respectively.
(ii) Accounting depreciation on office assets amounted to Rs. 0.3 million.
The taxes withheld by the clients, for the year ended 30 June 2012 amounted to Rs. 0.55 million. AOP is entitled to claim
tax depreciation of Rs. 0.25 million in respect of the office assets.
Required: Calculate the taxable income, net tax payable and unabsorbed losses (including unabsorbed depreciation), if
any , to be carried forward by the AOP for the year ended 30 June 2012.

Conceptual Approach to Taxes _______ ___ _______________ __ 341


Chapter 21 _______ Solved Past Papers Income Tax Numericals of CA Module C - (2001 to 2015)

Solution

AOP
Computation of taxable income and tax liability
For the tax year 2016
INCOME FROM BUSINESS U/S 18 Rs.
Accounting loss 800,000
Add:
Salaries paid to partners (500,000 + 300,000) 800,000
Accounting depreciation 300,000
1,100,000

Business income as per tax 300,000


Less: B/f business loss (Rs.400,000 - 300,000 = 100,000 balance business loss shall be c/f) 300,000
Income for the year -
Less:
Tax depreciation for the year 250,000
Total loss related to unabsorbed depreciation for the year (250,000)
Less: brought forward unabsorbed depreciation 300,000
Total unabsorbed depreciation to be carried forward (550,000)

COMPUTATION OF TAX LIABILITY:


As there is nil income under normal law hence minimum tax under NTR is computed as under.
Minimum tax liability:
In the absence of information it has been assumed that the AOP has realized all the services income on which the
corporate sectors have partially deducted tax on the same, however the AOP is under obligation to pay the 10% minimum
tax on the entire gross receipts even when the corporate sectors clients have not fully deducted tax on the same.

10% minimum tax on gross receipts of Rs.50 million against services 5,000,000
Less tax deduted under section 153 550,000
Balance tax payable 4,450,000

As there is nil income during the tax year after adjustment of brought forward losses, hence business loss of Rs. 100,000
for current year shall be carried forward for the succeeding six tax years whereas the unabsorbed brought forward
depreciation loss shall be carried forward to the extent of Rs. 550,000 without any time limitation.

Q.1 Spring 2012


Dr. Sona is a leading Eye Specialist and is listed on the panels of two hospitals. He also manages a private clinic. A
summary of his receipts and payments for the latest tax year is as follows:

Receipts Note Rupees Payments Note Rupees


Consultation fees Rent of clinic 300,000
Household
- Hospitals (i) 1,880,000 1,960,000
expenses
Purchase of
- Clinic 2,400,000 640,000
motor car
Surgical
Income from surgery 500,000
equipment
Salary to
- Hospitals (i) 1,504,000 180,000
assistant
Clinic running
- Clinic 2,350,000 240,000
expenses

342_________________ ____________ _______ _Conceptual Approach to Taxes


Chapter 21 ___________Solved Past Papers Income Tax Numericals of CA Module C - (2001 to 2015)

Property income (ii) 1,012,000 Car expenses (iv) 200,000


Other income (iii) 75,000 Donation (v) 300,000
Notes to the receipts and payments are presented below:
(i) The amount received from hospitals is net of withholding tax.
(ii) Dr. Sona owns a commercial building which he has rented out. Details of net receipts are as follows:

Rupees
Rent for the year 870,000
Non-adjustable security deposit:
- received from a new tenant 700,000
- paid to old tenant (received three years ago) -500,000
Tax withheld -50,000
Property tax on building -8,000
Net receipts 1,012,000

(iii) The amount was received for writing an article in an international magazine on World Health Day.
(iv) 60% of the motor car expenses were incurred in connection with his personal use.
(v) Donation was given to a Government medical college for upgrading its library.
(vi) Depreciation on motor car and surgical equipment, under the 3rd Schedule of the ITO, 2001 is Rs. 96,000
and Rs. 75,000 respectively.
Required: Compute the taxable income, tax liability and tax payable by Dr. Sona for the latest tax year. Provide
appropriate comments on the items which are not relevant for your computations,

Solution of Q. NO. 1 Spring 2012


Dr. Sona
Tax year 2016 Clinic Hospital Total
Rs. Rs. Rs.
INCOME FROM BUSINESS U/S 18
Consultation fee from clinic 2,400,000
Surgical fee from clinic 2,350,000
4,750,000
Less: Admissible expenses
Clinic rent 300,000
Surgical equipments depreciation 75,000
Depreciation on motor vehicle 38,400 / 8,388,710 x 4,750,000 21,744
Car expenses (Rs.200,000 x 40%) = 80,000 / 8,388,710 x 45,299
4,750,000
Salary to assistant 180,000
Clinic running expenses 240,000
Business income 862,042
3,887,958 3,887,958
INCOME FROM PROPERTY U/S 15 & 16
Rental income 870,000
Advance against property (700,000 - 150,000) / 10 55,000
500,000/10 = 50,000 x 3 = 150,000
925,000
Less: Property tax paid (8,000)
1/5th repair allowance (185,000) 732,000

INCOME FROM OTHER SOURCES U/S 39


Other income 75,000 75,000
Fee from hospital (assumed net of tax hence grossed up 1,880,000 2,088,889
x 100 /90)

Conceptual Approach to Taxes _______ ___ _______________ __ 343


Chapter 21 _______ Solved Past Papers Income Tax Numericals of CA Module C - (2001 to 2015)

Surgery fee from hospital (assumed net of tax hence grossed up 1,671,111
1,504,000 x 100 / 90)
3,760,000
Less: proportionate expenses:
Depreciation on motor vehicle 38,400 / 8,510,000 x 3,638,710 16,419
Car expenses (Rs.200,000 x 40%) = 80,000 / 8,510,000 x 34,206
3,638,710
Other income
3,709,374 3,709,374
Total taxable income 8,404,332

COMPUTATION OF TAX LIABILITY:


Tax on Rs. 8,404,332 [1,319,500 + 35% x (8,404,332 - 6,000,000)] 2,161,016

Less: Tax credit on donation made of Rs. 300,000


(2,161,016 / 8,404,332 x 300,000 (77,139)
Tax payable 2,083,877
Less: proportionate tax on hospital services [(2,083,877 / 8,404,332 (A) 919,749
x 3,709,374)
Balance tax under NTR 1,164,127

Minimum tax deducted on hospital services:


(3,760,000 x 10%) (B) 376,000
Tax under NTR as above 1,164,127
Add: higher of A and B 919,749
Total tax liability 2,083,877

Less: already deducted on hospital services (376,000)


Less: tax withheld on property income (50,000)
Balance tax payable 1,657,877
Notes
1. Rental income
The rent received or receivable by a person for a tax year shall be chargeable to tax in that year under the head Income
from property. Therefore, only three months rent is included in the taxable income of Mr. Khursheed which is below the
minimum threshold limit for the taxability of rent.
2. House hold expenses
House hold are expenses not allowable against any head of income, so there is no treatment made in solution.
3. Purchase of motor car
Motor car purchased is not a business expense.

Q. NO. 1 Autumn 2011

Mr. Khursheed, a Pakistani national, was employed as the chief financial officer in Zulfiqar Gas Company (ZGC), since
1990. Following information pertains to his income for the tax year 2011:
(1)IncomefromZGC
Khursheed was employed with ZGC up to 31 December 2010. During this period he received the
following emoluments:
- Basic salary of Rs. 400,000 per month, medical allowance of Rs. 75,000 per month and utility allowance equivalent
to 10% of basic salary.

- A company-maintained car for official and private use. The car was purchased two years ago at a cost of Rs. 5
million. According to the companys policy, ZGC deducted Rs. 10,000 per month from his salary, for private use of the car.

344_________________ ____________ _______ _Conceptual Approach to Taxes


Chapter 21 ___________Solved Past Papers Income Tax Numericals of CA Module C - (2001 to 2015)

On 31 July 2010, Khursheed had undergone a major surgery and incurred an expenditure of Rs. 1,500,000. ZGC
reimbursed the entire amount as a special case as it was not covered under the terms of employment.
Due to poor health, Khursheed opted for early retirement on 31 December 2010 under the companys voluntary retirement
scheme. He received the following benefits on his retirement:
-Rs.7,500,000asagoldenhandshakeunderthevoluntaryretirementscheme.
-Rs.9,100,000fromanunapprovedgratuityfundmaintainedbyZGC.
- Transfer of companys car for Rs. 2,600,000. The amount was deducted from his final settlement. The fair market
value of the car as of 31 December 2010 was Rs. 2,800,000.
The tax deducted at source for the tax year 2011 amounted to Rs. 3,750,000.
(2)OtherInformation
- On 1 January 2011, Khursheed commenced business of marketing of horticultural plants and related items.
However, due to intense competition, he had to wind-up this venture on 31 May 2011. During this period, he had incurred
a loss of Rs. 750,000.
- He purchased 5000 shares for Rs. 500,000 from initial public offering of a new listed company on 1 June 2010. He
claimed a tax credit of Rs. 60,000 on such investment, against the tax payable for the tax year 2010. On 15 June 2011, he
sold these shares for Rs. 700,000.
-HeincurredalossofRs.500,000onthesaleofhisshareholdingsinaprivatelimitedcompany.
-HesoldhispersonalcarataprofitofRs.300,000.
- On 1 March 2011, he purchased an apartment for Rs. 5,000,000. 60% of this amount was financed by a scheduled
bank. During the tax year 2011, he paid markup amounting to Rs. 127,500. On 1 April 2011, he rented out the flat to Mr.
Abdul Sattar at a monthly rent of Rs. 25,000 and received advance rent for eight months.
-Hisaveragetaxratefortheprecedingthreeyearsis18%.
Required:
(a) Compute the amount of taxable income, tax liability and tax payable / (refundable), if any, for the tax year 2011.
(b) Briefly comment on the items which are not considered by you in the above computation.

Solution of Q. No. 1 Autumn 2011


Mr. Khursheed
Tax year 2016

INCOME FROM SALARY U/S 12 Notes Rupees Rupees

Basic salary (Rs. 400,000 6) 2,400,000


Utility allowance (400,000 6 10%) 240,000
Medical allowance (75,000 6) 450,000
Less: Exempt up to 10% of basic salary U/C 139 (240,000) 210,000
Medical reimbursement (totally taxable as not covered by the terms 1,500,000
of employment)
Company maintained car (5,000,000 5% 6/12) U/R 5 125,000
Less: Amount deducted from salary (10,000 6) (60,000) 65,000
Golden handshake payment 7,500,000
Gratuity (Rs. 9,100,000 - Exempt upto Rs. 75,000) 9,025,000
Difference in purchase value and FMV of vehicle as benefit for 200,000
employee
21,140,000
INCOME FROM PROPERTY U/S 15

Rent (Rs. 25,000 3 months) 1 75,000

INCOME FROM BUSINESS U/S 18


Loss from business 2 (750,000)

INCOME FROM CAPITAL GAINS U/S 37

Gain on sale of quoted shares 3 200,000 -

Conceptual Approach to Taxes _______ ___ _______________ __ 345


Chapter 21 _______ Solved Past Papers Income Tax Numericals of CA Module C - (2001 to 2015)

Capital loss on sale of private company shares 4 (500,000) -


Sale of personal car 5 300,000 -
Taxable income 21,215,000

Taxable income 21,215,000


Less: Income to be taxed separately-Golden handshake (7,500,000)
13,715,000
COMPUTATION OF TAX LIABILITY:

Tax on Rs. 13,715,000 [1,422,000 + 30% x (13,715,000 - 3,436,500


7,000,000)]
Tax on Rs. 7,500,000 (golden handshake) at 18% 6 1,350,000
4,786,500
Less: Tax credit relating to markup on housing loan
(Rs. 127,500 x 4,786,500 / 21,215,000) (28,766)
4,757,734
Add tax on capital gain on quoted Company shares (Rs. 200,000 x 25,000
12.5%) covered under Fixed tax regime
4,782,734
Add recouped tax credit U/S 62 3 60,000
Tax deducted at source by bank from salary income (3,750,000)
Balance tax payable 1,092,734
Notes

1. Rental income
The rent received or receivable by a person for a tax year shall be chargeable to tax in that year under the head Income
from property. Therefore, only three months rent is included in the taxable income of Mr. Khursheed. Further it has been
assumed that the same is after allowable deducitons under section 15A of the Ordinance.

2. Business loss
No loss (including Loss under the head business income) shall not be set off against income from salary and income from
property. However the loss shall be carried forward to adjust the same against the business income of six subsequent tax
years.

3. Gain on sale of quoted shares


Since, the sale of listed company's shares is made after holding the shares for more than twelve months but less than
twenty four months, the gain on sale of shares is taxable @ 12.5%.
Furthermore, the tax credit of Rs. 60,000 allowed in the tax year 2013 will be recouped as the same has been disposed off
within twenty four months from the date of purchase has been elapsed.

4. Capital loss
Where a person sustains a loss on sale of shares of a private company it is construed as a capital loss and it cannot be
set-off against any other head of income, but shall be carried forward to the next year and set off against the capital gain,
if any.

5. Sale of personal car


Income under the head Capital gains can arise only on the disposal of a capital asset. Since movable assets held for
personal use are excluded from the definition of capital asset, gain on sale of car is not a taxable income in the hands of
the Mr. Khursheed.

6. Golden handshake
It is assumed that last three years' average rate of tax for golden handshake is 18% and further it is more beneficial
for the taxpayer to opt for taxation of golden handshake at last three year's average rate as compared to this to
include the same in the current year's income.

346_________________ ____________ _______ _Conceptual Approach to Taxes


Chapter 21 ___________Solved Past Papers Income Tax Numericals of CA Module C - (2001 to 2015)

Q.5 Autumn 2011


Mr. Feroz has been the CEO of Aziz Foods Pakistan Limited (AFPL) for several years. He was given 2000 shares on 1
June 2009 by Aziz AG, Germany (the parent company of AFPL) at a price of 2.5 per share. The market price on that
date was 8.2 per share. The shares were transferable on completion of one year of service, from the date of issue of
shares.

The market price of the shares as on 1 June 2010 was 12.5 per share. On 10 April 2011, Mr. Feroz sold all shares at 13
per share. He paid a commission of 50 to the brokerage house.

The relevant exchange rates are as follows:


1 June 2009 1 = Rs. 118.10
1 June 2010 1 = Rs. 121.40
10 April 2011 1 = Rs. 123.90

Required: Compute the amount to be included in the taxable income of Mr. Feroz for tax years 2009, 2010 and 2011 and
specify the head of income under which the income would be classified.

Solution of Q. NO. 5 Autumn 2011

Mr. Feroz
Computation of taxable income

Tax year 2014 Rs.

Shares were issued in tax year 2014, however, transferability of the shares is restricted in tax year 2014 hence
nothing is taxable in tax year 2009.

Tax year 2015

Income from salary U/S 12


Market value of shares on 1 June, 2010 (12.5 x 2,000 x 121.40) 3,035,000
Less: cost of shares (2.5 x 2,000 x 118.10) 590,500
2,444,500
Tax year 2016

Capital gain U/S 37A

Consideration received (13 x 2,000 x 123.90) 3,221,400


Less: cost
Cost of shares after acquisition of the same (including salary income) 3,035,000
Commission expense (50 x 123.90) 6,195
3,041,195
Capital gain 180,205

Q. NO. 3 Spring 2011

Carrot Ltd (CL) is engaged in the manufacture, import and sale of electronic appliances for the past twenty years. When
reviewing the company's tax provisions, you noticed the following amounts appearing in the tax calculation for the year
ended June 30, 20X2.
1. Profit on debt of Rs. 500,000 paid on a working capital loan obtained from a foreign bank.CL did not deduct
withholding tax while paying profit on debt considering the bank does not have a Permanent Establishment in
Pakistan.
2. Expenditure of Rs. 450,000 on promotion of a product which is expected to generate revenue for twelve years.
3. Bad debt in respect of a staff loan, Rs. 25,000.
4. Reimbursement of expenses of Rs. 300.000 to CL by the parent company- This amount was incurred by CL in
20X1 on marketing a new product imported from Dubai.

Conceptual Approach to Taxes _______ ___ _______________ __ 347


Chapter 21 _______ Solved Past Papers Income Tax Numericals of CA Module C - (2001 to 2015)

5. Initial allowance of Rs. 4,000,000 on a used equipment acquired locally from MSD Limited.
6. Financial charges amounting to Rs. 100,000 and depreciation amounting to Rs. 300,000 on a vehicle acquired
on finance lease from Radish Leasing. Lease rentals paid during the year amounted to Rs. 400,000.
Required:
Under the provisions of CIR, 2001 discuss the admissibility of the above amounts for tax purpose.

Solution of Q. NO. 3 Spring 2011

1. It is not admissible as the withholding tax @ 20% under section 152(2) of the Income tax Ordinance, 2001 was not
deducted by CL limited.
2. It is an admissible expense as it is incurred for the purpose of business of company.
3. Bad debts in respect of loan to employees shall not be admissible under the Income Tax Ordinance, 2001 because the
same is not against a trading liability already allowed.
4. As the amount is the reimbursement of expenses incurred on behalf of parent company so the same shall be adjusted
against the receivable balance from parent and has no impact on taxable income.
5. Initial allowance on locally purchased equipment shall not be allowed as the asset is already used in Pakistan.
6. Amount of rental payment shall be allowed as deduction, however the amount of depreciation and financial charges
shall not be allowed as deduction.

Q.1 Autumn 2010

Mr. Zameer Ansari is working as a Chief Executive Officer in Wimpy (Private) Limited (WPL). Following are the details of
his income / receipts during the tax year 2010:

(a) His monthly cash remuneration in WPL is as follows:


Rupees
Basic salary 200,000
Medical allowance 30,000
Utilities allowance 10,000

(b) In addition to the above, he was also provided the following benefits in accordance with his terms of
employment:

(i) Medical insurance for hospitalization and surgery, limited to Rs. 1,500,000 per annum.
(ii) Payment of his children's school fees of Rs. 15,000 per month. The fee is deposited directly into the
school's bank account.
(iii) Rent free furnished accommodation on 1000 square yards. The accommodation is located within the
municipal limits of Karachi.
(iv) Two company-maintained cars. One of the cars was purchased by WPL for Rs. 3,000,000 and is
exclusively for his business use. The second car was obtained on lease on February 1, 2009 and is used
partly for official and partly for personal purposes. The fair market value of the leased vehicle at the time of
lease was Rs. 1,800,000.
(v) Leave encashment amounting to Rs. 100,000 was paid to Mr. Zameer on July 5, 2010.
(vi) An amount equal to one basic salary was paid by WPL to an approved pension fund.

(c) Mr. Zameer had received 15,000 shares of WPL on December 1, 2006 under an employee share scheme. He
had the option to transfer the shares on or after January 1, 2009. However, he sold all the shares on April 1,
2010

Fair value of the shares were as follows:


Rs. 35 per share on December 1, 2006
Rs. 42 per share on January 1, 2009
Rs. 48 per share on April 1, 2010
(d) An apartment owned by Mr. Zameer was rented on July 1, 2009 to Mr. Abdul Ghaffar at a monthly rent of Rs.
22,000. He received a non-adjustable security deposit of Rs. 150,000 which was partly used to repay the
non-adjustable security deposit amounting to Rs. 90,000 received from the previous tenant in July 2007.
He also incurred Rs. 20,000 on account of repair charges.

348_________________ ____________ _______ _Conceptual Approach to Taxes


Chapter 21 ___________Solved Past Papers Income Tax Numericals of CA Module C - (2001 to 2015)

(e) He earned profit amounting to Rs. 750,000 on fixed deposit account maintained with a bank. The bank
withheld income tax amounting to Rs. 75,000 and Zakat amounting to Rs. 250,000.
(f) Tax deducted at source from his salary, amounted to Rs. 650,000.

Required: Compute the taxable income, tax liability and tax payable by Mr. Zameer Ansari for the tax year 2010.

Solution of Q.1 Autumn 2010

Mr. Zameer Ansari


Tax Year 2016
Individual
Resident

Computation of taxable income and tax liability


INCOME FROM SALARY U/S 12 Rs. Rs.
Basic salary (Rs. 200,000 x 12 months) 2,400,000
Medical allowance (Although exempted upto 10% of basic salary, 360,000
clause 139 of 2nd schedule, however as the medical facility has
also been given in the question therefore the same is fully
chargeable to tax)
One month basic salary employer contributed towards pension fund 200,000
(As benefit)
Utility allowance (Rs. 10,000 x 12 months) 120,000
Medical insurance and hospitalization fully exempt U/C 139 -
Payment of his children's school fees 180,000
Rent free furnished accommodation (45% of basic salary shall be 1,080,000
included in the salary income assumed equal to FMV)

Conveyance provided by employer:


Wholly for business use (Fully exempt)
Partly for personal and business use (5% of FMV of leased vehicle) (1,800,000 x 5%) 90,000

Leave encashment received on 5 July 2012 (N-1) -


FMV of shares at the date when employee has free right to transfer -
(already included as benefit in the salary income of the tax year
2011)
4,430,000
INCOME FROM PROPERTY U/S 15 & 16

Rental income (Rs. 22,000 x 12 months) 264,000


Non-adjustable advance ((150,000-(90,000 / 10 x 2)/10) 13,200
Income from property 277,200

Less: 1/5th of rent as fixed repair allowance (55,440) 221,760

CAPITAL GAINS U/S 37

On sale of (Pvt.) Ltd. company shares after retention of more than 67,500
12 months therefore gain shall be taxable as capital gain shares
{15,000 x (Rs.48-Rs.42) x 75%}

INCOME FROM OTHER SOURCES U/S 39


Profit on fixed deposit account maintained with a bank (SBI as 750,000
FTR)

Conceptual Approach to Taxes _______ ___ _______________ __ 349


Chapter 21 _______ Solved Past Papers Income Tax Numericals of CA Module C - (2001 to 2015)

Total income 4,719,260


Less: Allowable deductions
Zakat paid under the Zakat and Ushr Ordinance (250,000)
Total taxable Income 4,469,260

COMPUTATION OF TAX LIABILITY:

Tax liability under NTR


Tax on Rs. 4,469,260 [597,000 + 27.5% x (4,469,260 - 4,000,000)] 726,047

Less: Tax credit on contribution to approved pension fund (although paid by


employer but the same is on behalf of employee)
Tax credit shall be allowed on lower of:
- Actual amount of contribution i.e. Rs. 200,000
- 20% of taxable income i.e. Rs. 893,852
Tax credit = (200,000 x 726,047 / 4,469,260) 32,491
693,556
Tax liability under FTR
Tax on Income from other sources
Tax on 750,000 @ 10% 75,000
Total tax liability 768,556
Less: Tax deducted at source from his salary (650,000)
Tax deducted on PLS (75,000)
Balance tax payable 43,556

NOTES
N-1 Leave encashment has been ignored as received after the tax year. Salary and perquisites are charged on the receipt
basis.

Q.1 Spring 2010

Sohail, Khaled and Qazi are members of an AOP and share profit and loss in the ratio of 2:2:1. The principal activity of the
AOP is trading of rice and wheat. Following are the details of the annual income / (loss) of the AOP and its members:

(i) The AOP suffered loss before tax amounting to Rs. 1,500,000. The loss has been arrived at after adjusting
rental income earned by the AOP, the details of which are as follows:

Rupees
Rental income 2,000,000
Related expenses:
Property tax 40,000
Depreciation 457,500
Net rental income 1,502,500

No tax was withheld on the rental income.

(ii) The expenses debited to profit and loss account include the following amounts paid to the members of the
AOP:

Sohail Khaled Qazi


Rs. Rs. Rs.
Salary 900,000 600,000 -
Interest on capital 300,000 300,000 500,000

350_________________ ____________ _______ _Conceptual Approach to Taxes


Chapter 21 ___________Solved Past Papers Income Tax Numericals of CA Module C - (2001 to 2015)

(iii) Sohail earned Rs. 800,000 from another business, of which he is the sole proprietor.
(iv) Khaled received an amount of Rs. 255,000 as share of income after tax, from another AOP where he is
entitled to 40% of the total profit. Tax on annual income of that AOP amounted to Rs. 112,500. He also earned
income of Rs. 900,000 from a sole proprietorship concern owned by him.
(v) Qazi works as a Freelance IT Consultant and provides consultancy services to corporate clients. He received
Rs. 940,000 from his clients after deduction of tax amounting to Rs. 60,000. The total expenses incurred in
providing the consultancy services amounted to Rs. 150,000.

Required: Assuming that the above data pertains to the tax year 2010, compute the taxable income and tax liability of the
AOP and each of its members.

Solution of Q.1 Spring 2010

Name of Taxpayer : AOP


National Tax Number :
Tax Year Ednded on : 30th June, 2015
Tax Year : 2016
Personal Status : AOP

TAXABLE INCOME OF THE FIRM

INCOME FROM BUSINESS U/S 18 (Rs.) (Rs.)

Loss as per P and L A/c (1,500,000)


Less: net rental income (1,502,500)
(3,002,500)
Add: Inadmissible Expenses
Salaries to partners:
Sohail 900,000
Khalid 600,000 1,500,000
Interest on Capital
Sohail 300,000
Khalid 300,000
Qazi 500,000 1,100,000
Loss under the head "Income from Business" (402,500)

INCOME FROM PROPERTY U/S 15

Rental income 2,000,000


Less: 1/5th repair allowance (400,000)
Less: Property tax (40,000) 1,560,000

COMPUTATION OF TAX LIABILITY:

In the absence of information no minimum tax under section 113 has been computed.

Tax liability under NTR on business loss (A) -

Tax liability under NTR on Property income


Tax on first Rs.1,560,000 [144,500 + 20% x (1,560,000 - (B) 156,500
1,500,000)]
Total tax liability of firm (A + B) 156,500

Total income of the firm (Property income less business loss) 1,157,500

Conceptual Approach to Taxes _______ ___ _______________ __ 351


Chapter 21 _______ Solved Past Papers Income Tax Numericals of CA Module C - (2001 to 2015)

Less firm tax liability 156,500


Divisible income 1,001,000

COMPUTATION OF SHARE FROM AOP Total Sohail Khaled Qazi


Rs. Rs. Rs. Rs.

Salary 1,500,000 900,000 600,000 -


Interest 1,100,000 300,000 300,000 500,000
Balance loss divided in profit sharing ratio (1,599,000) (639,600) (639,600) (319,800)
Total income 1,001,000 560,400 260,400 180,200

COMPUTATION OF INCOME and TAX LIABILITY OF PARTNERS Sohail Khaled Qazi

Rs. Rs. Rs.


Net income from another business as proprietor covered under 800,000 900,000 850,000
NTR (net taxable)
Share from another AOP under NTR after tax (included for rate - 255,000 -
purposes)
Share from AOP under NTR after tax (included for rate purposes) 560,400 260,400 180,200

Total share from AOP included for rate purposes 560,400 515,400 180,200
Total income 1,360,400 1,415,400 1,030,200

COMPUTATION OF TAX LIABILITY OF EACH PARTNER:

Tax on Rs. 1,360,400 [32,000 + 15% x (1,360,400 - 750,000)] 123,560


Tax on Rs. 1,415,400 [32,000 + 15% x (1,415,400 - 750,000)] 131,810
Tax on Rs. 1,030,200 [32,000 + 15% x (1,030,200 - 750,000)] 74,030

Less tax on share from AOP 50,899 47,997 12,949


Tax liability under NTR (A) 72,661 83,813 61,081
Minimum tax under NTR (B) - - 60,000
Tax liability: higher of (A) or (B) 72,661 83,813 61,081

Q.NO. 6(b) Spring 2010

Mr. Shahbaz, a resident individual, earned Rs. 700,000 from the sale of assets as shown below:

Purchase Price in Sale Price in Gain /


(loss)
Date Rupees Date Rupees Rupees
Shares of a listed company 10-12-08 350,000 7/31/2009 200,000 (150,000)
Shares of an unlisted company 15-07-08 500,000 11/30/2009 900,000 400,000
Jewellery 15-05-08 750,000 12/20/2009 1,400,000 650,000
Sculpture 01-07-05 400,000 1/31/2010 300,000 (100,000)
Shares of a (Pvt.) Ltd. Co. 01-01-09 1,300,000 2/15/2010 1,200,000 (100,000)

Discuss the treatment and the implications of each of the above transactions with brief reasons under the Income
Tax Ordinance, 2001.

Solution of Q.NO. 6(b) Spring 2010

Name of Taxpayer : Mr. Shahbaz


National Tax Number :
Tax Year Ednded on : 30th June, 2016
Tax Year : 2015

352_________________ ____________ _______ _Conceptual Approach to Taxes


Chapter 21 ___________Solved Past Papers Income Tax Numericals of CA Module C - (2001 to 2015)

Personal Status : Individual


Residential Status : Resident

INCOME FROM CAPITAL GAIN

Shares of a listed company U/S 37A


Loss on sale of shares of a listed company (securities) can only be adjusted against gain from any other securities
chargeable to tax under section 37A and any unadjusted loss shall not be carried forward.

Shares of unlisted company, private limited company and Jewellery U/S 37


- As the holding period of shares of unlisted company is more than one year, hence 25% of the capital gain shall be
exempt.

- Similarly the holding period of jewellery is more than one year, hence 25% of the capital gain shall also be exempt.

- Loss on sale of shares of private company shall be adjusted against gain on sale of unlisted company and
jewellery.

Loss on Sculpture U/S 38(5)


Loss on sale of Sculpture shall not be recognized.
Rs.
Gain on shares of unlisted company (400,000 x 75%) 300,000
Loss on shares of private company (100,000)
Gain on Jewellery (650,000 x 75%) 487,500
Capital gain chargeable to tax 687,500

Q.NO. 1 Autumn 2009

Mr. Zulfiqar, a senior executive of Mirza Petroleum Limited (MPL), retired on March 31, 2009 after completion of nineteen
years of dedicated service. The details of Mr. Zulfiqars income for the tax year 2009 are given below:

Income from MPL


(i) Monthly remuneration:
Rupees
Basic salary 280,000
Medical allowance 45,000
Utilities allowance 45,000
Cost of living allowance 25,000
Total monthly salary 395,000

Market value of rent free accommodation provided 120,000

(ii) As per terms of employment, tax liability of Mr. Zulfiqar to the extent of Rs. 200,000 is to be borne by MPL.

(iii) On his retirement, he received gratuity of Rs. 2,660,000 from an unrecognized gratuity fund maintained by
MPL.

(iv) He is receiving pension amounting to Rs. 50,000 per month from the date of his retirement.

Other Information
(v) He is also receiving pension of Rs. 12,000 p.m. from a multinational company where he worked from 1975 to
1990.

(vi) A plot inherited from his father was sold for Rs. 5,000,000. Fair market value of the plot at the time of
'inheritance was 'Rs. '1,000,000

(vii) On January 1, 2009, he rented out one of his residential bungalows to a private school for Rs. 100,000 per

Conceptual Approach to Taxes _______ ___ _______________ __ 353


Chapter 21 _______ Solved Past Papers Income Tax Numericals of CA Module C - (2001 to 2015)

month and received advance rent for two years.

(viii) Rs. 500,000 were invested in new shares offered by a listed company.

(ix) He paid mark up amounting to Rs. 250,000 on a house loan obtained from a scheduled bank.

(x) He incurred a loss of Rs. 20,000 on sale of a painting.

Required:
(a) Compute taxable income and tax liability of Mr. Zulfiqar for the tax year 2009.
(b) Briefly comment on the items which are not considered in the above computation.

Solution of Q.NO. 1 Autumn 2009

Name of Taxpayer : Mr Zulfiqar


National Tax Number :
Tax Year Ednded on : 30th June, 2016
Tax Year : 2016 Rs. Rs.
Personal Status : Individual

INCOME FROM SALARY U/S 12


Basic salary (Rs. 280,000 x 9 months) 2,520,000
Cost of living allowance (Rs. 25,000 x 9 months) 225,000
Salary for provident fund 2,745,000
Medical allowance (Rs. 45,000 x 9 months) 405,000
Less: exempt upto 10% of basic salary U/C 139 (Rs. 2,520,000 x (252,000) 153,000
10%)
Utilities allowance (45,000 x 9 months) 405,000
Rent free accommodation (N-2)
Higher of Rs. 120,000 or (45% x Rs. 2,520,000) will be taxable 1,134,000

Tax liability of Mr. Zulfiqar to the extent of Rs. 200,000 is to be 200,000


borne by MPL
Gratuity from an unrecognized gratuity fund exempt upto lesser of 2,660,000
75,000 or 50% of amount received
Less: Lesser of Rs. 75,000 or (50% x Rs. 2,660,000 = 1,330,000) 75,000 2,585,000

Pension amounting to Rs. 50,000 per month from the date of his 150,000
retirement (Rs. 50,000 x 3 months) (N-3)
Pension of Rs. 12,000 p.m. from a multinational company where he 144,000 144,000
worked from 1975 to 1990 (Rs. 12,000 x 12 months) (N-3)

Plot inherited from his father was sold for Rs. 5,000,000. N-1 -
Total salary income 7,366,000

INCOME FROM PROPERTY U/S 15

Rental income (6 months x Rs.100,000) assumed net amount after 600,000


expenses (N-4)
Total income income under NTR 7,966,000
Less deductible allowance for profit on debt (N - 6) (250,000)
Taxable income under NTR 7,716,000

COMPUTATION OF TAX LIABILITY:

Tax on Rs. 7,716,000 [1,422,000 + 30% x (7,716,000 - 7,000,000)] 1,636,800

354_________________ ____________ _______ _Conceptual Approach to Taxes


Chapter 21 ___________Solved Past Papers Income Tax Numericals of CA Module C - (2001 to 2015)

Less: Tax credits


Invested in new shares offered by a listed company U/S 62 (N - 7) 500,000
500,000
Tax liability / Taxable income x Investment in shares (1,636,800 / 106,065
7,716,000) x 500,000
Total tax payable 1,530,735
Less: Tax paid by employer 200,000
Balance tax payable 1,330,735

NOTES
N-1 Plot inherited from father was sold is not taxable on the assumption that the same is being held for more than two
years.
N-2 Higher of the FMV of rent or 45% of (MTS or Basic Salary)
N-3 Only one pension with the higher amount is exempted so Rs.150,000 is exempted.
N-4 Advance rent received is adjustable against rent therefore it is ignored. Just the rent for 6 months will be taxable.

N-5 Loss on sale of painting shall not be recognized.


N-6 Lower of actual amount Rs. 250,000, 50% of Taxable income (Rs.7,716,000 x 50%) = 3,858,000 or Rs. 1,000,000
hence Rs. 250,000 has been taken into consideration.

N-7 Lower of actual investment Rs. 500,000, 20% of Taxable income (Rs. 7,716,000 x 20%) = 1,543,200 or Rs.
1,500,000, hence lower amount of Rs. 500,000 has been taken into consideration.

Q. NO. 3(b) Autumn 2009

During the tax year 2009, Ishaq Enterprise disposed off the following assets:

(i) an immovable property was sold for Rs. 200 million. The cost of immovable property was Rs. 100 million. Upto tax
year 2008, tax depreciation of Rs. 10 million had been allowed on the immoveable property.

(ii) a plant was exported to Nepal. The export proceeds amounted to Rs. 28 million. The cost and written down value of
the plant was Rs. 25 million and Rs. 18 million respectively.

(iii) three trucks were disposed off for Rs. 2.5 million. They were acquired in tax year 2008. The tax written down value of
trucks at the begning of tax year 2009 was Rs. 2.4 million. The trucks were being used partly i. .e. 60% for business
purpsoes. The rate of deprciation for tax purposes is 20%.

Required:
Compute the tax gain or loss on disposal of each of the above assets.

Solution of Q. NO. 3(b) Autumn 2009


Rs. Rs.

Consideration received from the sale of immovable property 200,000,000


Less: WDV of immovable property
Cost of asset U/S 22(13)(d) 200,000,000
Less: Depreciation charged upto 2008 (10,000,000) 190,000,000
Gain on sale of immovable property 10,000,000

Consideration received from the export of plant 25,000,000


Less: WDV of plant
Cost of an asset U/S 22(14) 25,000,000
Less: Depreciation charged upto 2008 7,000,000 18,000,000
Gain on sale of plant 7,000,000

Conceptual Approach to Taxes _______ ___ _______________ __ 355


Chapter 21 _______ Solved Past Papers Income Tax Numericals of CA Module C - (2001 to 2015)

Consideration received from the sale of trucks 2,500,000


Less: WDV of trucks 2,400,000
Add: disallowed tax depreciation portion in the tax year 2008 Note attached 169,412 2,569,412
(2,400,000 x 100 / 85 x 15% x 40/100)

Loss on sale of motor trucks (69,412)

Note: There is no limitation on value of motor trucks as they are not passenger transport vehicle & further no initial
allowance has been claimed on the assumption that the motor trucks are not for hiring purposes.

Q. No. 5(a) and (b) Autumn 2009

Mr. Abdullah, an employee of a Malaysian based company, has been assigned to work in Karachi, in its subsidiary
company which is registered under the Companies Ordinance, 1984. The initial assignment of two years commenced on
March 1, 2009 and would be extended subject to mutual agreement. Mr. Abdullahs remuneration will be paid in Malaysia,
details of which are given below:

Nature of Income Amount in Equivalent:


Rupees
Pakistan source salary income for the tax year 2009 5,750,000
Pakistan source salary income for the tax year 2010 17,250,000
Foreign source salary income for the tax year 2009 12,000,000
Foreign source salary income for the tax year 2010 3,000,000

Required:
(a) Explain the residential status of Mr. Abdullah under the ITO, 2001 for the tax years 2009 and 2010.
(b) Compute taxable income of Mr. Abdullah for the tax years 2009 and 2010 with supporting comments.

Solution of Q. 5(a) and (b) Autumn 2009

a) RESIDENTIAL STATUS

YEAR 2015
Mr. Abdullah is a non-resident person because his stay in Pakistan was for 120 days that is less than 183 days in tax year
2009.

YEAR 2016
Mr. Abdullah is a resident person because he was in Pakistan for 183 days or more in tax year 2010.

b) TAXABLE INCOME Rs. Rs.


YEAR 2015 - As non resident individual Total
Pakistan source salary 5,750,000
Foreign source salary - 5,750,000

YEAR 2016 - As residnet individual


Pakistan source salary 17,250,000
Foreign source salary (See note below) - 17,250,000

Note: Foreign source salary of a resident shall be ignored for tax computation as tax on the same has already been
paid, however if foreign tax on the same is not paid within two years then the same shall be taxed in Pakistan.

Q.NO.1 Spring 2009

Mr. Manto worked as an employee in Berlin Hotel, Germany for a period of five years. During the said period he did not
visit Pakistan for a single day. He returned to Pakistan on July 1, 2008 and immediately joined as a General Manager in a
well-reputed hotel, based in Karachi.

356_________________ ____________ _______ _Conceptual Approach to Taxes


Chapter 21 ___________Solved Past Papers Income Tax Numericals of CA Module C - (2001 to 2015)

Assume that the details of his income for the tax year 2009 are as follows:

(i) Basic salary (per month) Rs. 100,000 House rent allowance (per month) Rs. 30,000 Medical 'allowance (per
month) 'Rs. '10,000

(ii) Besides medical allowance, he is also entitled to free medical treatment at approved hospitals.

(iii) He has been provided a company maintained 1600cc car which was used partly for official and partly for
personal purposes. The hotel has leased the car from a bank. The gross lease rentals payable over the
period of lease amount to Rs. 2,700,000. The fair market value of the car at the time of lease was
Rs. 1,600,000. The total lease rentals paid by the hotel during the year amounted to Rs. 800,000.

(iv) He is entitled to lunch at the hotels restaurants where the usual charges are Rs. 400 per person. He is
entitled to concessional rate of Rs. 40 per day which is deducted from his salary. Assume that there are 300
working days in the year.

(v) He went for a training course to Islamabad where boarding and lodging cost amounting to Rs. 150,000 was
borne by the hotel. He incurred a further expense of Rs. 125,000 which was reimbursed by the hotel.

(vi) Provident fund was deducted @10% of his basic salary. An equal amount was contributed by the hotel.
Interest credited to his provident fund account amounted to Rs. 48,000.

(vii) As per terms of employment agreed with Mr. Manto, tax of Rs. 249,200 on salary will be borne by the hotel.
(viii) During the year, he also received an amount of Rs. 94,000 (net of 6% withholding tax) from a local university
where he gave lectures on hotel management.
(ix) On July 15, 2008, he received a lump sum amount of Rs. 4,000,000 through a normal banking channel as final
settlement from Berlin Hotel.

(x) On August 1, 2008, he inherited 25,000 shares of a private limited company. The estimated fair market value
of the shares, on the date of inheritance, was Rs. 42 per share. He sold all the shares on February 28, 2009
at Rs.62 per share.
(xi) He paid zakat amounting to Rs. 200,000 to an approved organization, through cross cheque.

Required:
(a) Compute Mr. Mantos taxable income and tax payable for the tax year 2009.
(b) Briefly explain the treatment of items which are not considered in the above computation.

Solution of Q.NO.1(a) Spring 2009

Name of Taxpayer : Mr. Manto


National Tax Number :
Income year ended : 30th June, 2016
Tax Year : 2016
Personal Status : Individual
Residential Status : Resident
(Rs.) (Rs.)
INCOME FROM SALARY U/S 12

Basic salary (Rs.100,000 x 12 months) 1,200,000


House rent allowance (Rs.30,000 x 12months) 360,000
Medical allowance (Rs.10000 x 12 months) 120,000
(As the free medical facility seperately been provided hence
medical allowance shall be totally taxable)
Value of conveyance (5% of FMV (1,600,000 x 5%) of leased car) 80,000
Lunch at concessional rate (Exempt U/C (53A) as marginal cost to
employer) [(Rs.400 - 40) x 300 days] 108,000 -
Training and boarding and lodging cost (Exempt being for official purposes) 275,000 -

Employer's contribution for recognized provident fund 120,000

Conceptual Approach to Taxes _______ ___ _______________ __ 357


Chapter 21 _______ Solved Past Papers Income Tax Numericals of CA Module C - (2001 to 2015)

Less: Exempt upto lower of 10% of basic salary or 100,000 (100,000) 20,000
Lessor of (10% x Rs.1,200,000)120,000 or Rs.100,000
Employee contribution (not to be included as already included in salary) -
Interest on provident fund 48,000
Less: Exempt upto higher of 1/3rd of salary or calculated @ 16% (400,000) -
Tax liability paid by hotel 249,200
Amount received as final settlement from Berlin Hotel (not to include in income u/s 51 & 102) -
Total income from salary 2,029,200

CAPITAL GAINS U/S 37

Sale of inherited shares in (Pvt.) Ltd. Company Rs.(62-42) x 25,000 shares 500,000

INCOME FROM OTHER SOURCES U/S 39


Amount received on lecture given on hotel management (gross amount) 100,000
Total Income 2,629,200
Less: Deductible allowance
Zakat paid to an approved organization [Lower of 30% of taxable income or actual Zakat paid] (200,000)
Taxable income 2,429,200

COMPUTATION OF TAX LIABILITY:

Tax on Rs. 2,429,200 [137,000 + 17.5% x (2,429,200 - 1,800,000)] 247,110


Less: proportionate tax on service income
(100,000 x 247,110 / 2,429,200) (A) 10,172 236,938

Tax deducted on services (Minimum tax) (B) 6,000

Add: higher of (A) or (B) 10,172


247,110
Less: Tax deducted on services income (6,000)
Tax deducted on salary income (249,200)
Balance Tax refundable (8,090)

Solution of Q.NO.1(b) Spring 2009

As all the information has been considered while solving the Part (a) of this question hence there is not need any answer
for this part.

Q.NO. 3(b) Spring 2009

Mr. Qasm is in the business of manufacturing of leather products. The financial results of the business for the tax year
2009 are as follows:

Rupees

Sales 12,000,000
Cost of sales 10,000,000
Gross profit 2,000,000
Selling, administrative and other expenses 2,500,000
Net loss (500,000)

He had rented out the ground floor of his house and received Rs. 300,000 as rent thereof. No tax was deducted by his
tenant. Advance tax paid during the year includes the following:

Import of raw materials 200,000

358_________________ ____________ _______ _Conceptual Approach to Taxes


Chapter 21 ___________Solved Past Papers Income Tax Numericals of CA Module C - (2001 to 2015)

Electricity bills 70,000


Telephone bills 50,000

Required:
Compute the tax payable/refundable by Mr. Qasmi for the tax year 2009.

Solution of Q.NO. 3(b) Spring 2009

Name of Taxpayer : Mr. Qasim


National Tax Number :
Income year ended : 30th June, 2016
Tax Year : 2016
Personal Status : Individual
Residential Status : Resident

Computation of income and tax thereon Rs. Rs.

As there is loss under normal law hence 5% minimum tax liability on industrial (A) 18,000
electricity bill is to be paid by the tax payer upto monthly bill of Rs. 30,000 that comes
to Rs. 1,500 p.m. (Rs. 1,500 x 12 months)

Taxable income from property


Rent chargeable to tax (Assumed after allowable expenses) 300,000
Tax payable on income from property
Tax on 300,000 @ 0% (B) -
Higher of (A) & (B) is to be paid by the taxpayer 18,000

Less: tax paid / deducted at source


Import of raw materials 200,000
Electricity bills 70,000
Telephone bills 50,000 320,000
Balance tax refundable (302,000)

Note: Business loss is not allowed to set off against income from property, however the same shall be carried forward and
adjusted against business income only in succeeding six tax years.

Q.NO. 1: Autumn 2008

Mr. Ali Raza is working as a Senior Executive in DD Pakistan Ltd. The details of his income/receipts during the tax year
2008 are as follows:

(i) He received basic salary of Rs. 65,000 per month.


(ii) He was provided with furnished accommodation for which DD Pakistan Limited paid a rent of Rs. 25,000 per
month.
(iii) A company owned car was provided to him which was used partly for official and partly for private purposes.
The car was purchased at a cost of Rs. 500,000 but had a fair market value of Rs. 520,000.
(iv) Medical allowance of Rs. 150,000 was paid to him during the year. The actual medical expenses incurred by
him amounted to Rs. 40,000.
(v) He earned an income of Rs. 45,000 on the sale of jewellery but incurred a loss of Rs. 28,000 on sale of an
antique.
(vi) An apartment owned by him was rented on July 1, 2007 at a monthly rent of Rs. 10,000. He received a
non-adjustable security deposit of Rs. 100,000, which was partly used to repay the non-adjustable security
deposit received from previous tenant in July 2005, amounting to Rs. 70,000.

(vii) He incurred the following expenses on the apartment:


Rupees
Repairs 8,000

Conceptual Approach to Taxes _______ ___ _______________ __ 359


Chapter 21 _______ Solved Past Papers Income Tax Numericals of CA Module C - (2001 to 2015)

Share of rent to House Building Finance Corporation 15,000


(viii) Gross dividend of Rs. 12,000 was received from a listed company.
(ix) Provident fund was deducted @ 12% of his basic salary. An equal amount was contributed by the
company.
(x) He withdrew cash from the bank on which the bank deducted tax of Rs. 400.
(xi) Tax deducted by the company amounted to Rs. 170,000.

Compute his taxable income, total tax payable and tax payable with the return.

Solution of Q.NO. 1: Autumn 2008

Name of Taxpayer : Mr. Ali Raza


National Tax Number :
Income year ended : 30th June, 2016
Tax Year : 2016
Personal Status : Individual
Residential Status : Resident

INCOME FROM SALARY U/S 12 Rs. Rs.

Basic salary (Rs.65,000 x 12 months) 780,000


Rent-free accommodation higher of FMR or 45% of basic salary
higher of (Rs.25,000 x 12 months) 300,000 or (45% x 780,000) 351,000 351,000
conveyance facility partly for personal use (Rs.500,000 x 5%) 25,000
Medical Allowance 150,000
Less: 10% of basic pay (78,000) 72,000

Employer provident fund contributoin (12% of Basic pay) 93,600


Less: Exempt lower of 1/10th of salary (Rs.780,000 x 0.1) or Rs.100,000 (78,000) 15,600
Employee's contribution shall not be considered as already included in salary -
Total salary income 1,243,600

CAPITAL GAIN U/S 37

Gain on sale of Jewellery 45,000


Loss on sale of an antique shall not be recognized. - 45,000
Taxable income under NTR excluding SBI income 1,288,600

INCOME FROM PROPERTY U/S 15


Rental Income (Rs.10,000 x 12 months) 120,000
Add: Non-adjustable Advance [Rs.(100,000 - (70,000 / 10 x 2) /10] 8,600
128,600

Less 1/5th fixed repair allowance irrespective of actual repair expenses (25,720)
Share in rent to HBFC (15,000) 87,880
Taxable income 1,376,480

COMPUTATION OF TAX LIABILITY:


Tax on Rs. 1,376,480 [14,500 + 10% x (1,376,480 - 750,000)] 77,148

Tax on income covered under FTR


10% tax on gross dividend of Rs. 12,000 1,200
78,348
Less: Tax paid / deducted under sections
Cash withdrawal 231 A (400)
Salary income 149 (170,000) (170,400)

360_________________ ____________ _______ _Conceptual Approach to Taxes


Chapter 21 ___________Solved Past Papers Income Tax Numericals of CA Module C - (2001 to 2015)

Balance tax refundable (92,052)

NOTES:
N-1 Loss on sale of antique shall not be recognized.

Q.NO. 3(a): Autumn 2008

Mr. Henry is a UK national and provides independent consultancy services in his individual capacity, to United Autos
Limited, a Pakistani company. Mr. Henry has entered into a contract with the company. The companys accountant has
treated payment under this contract as being under an employment contract with the company.

Mr. Henry stayed in Pakistan for eight months during the tax year 2008. During the said period, he was only involved in
providing in-house independent consultancy services to different departments of the Company. Mr. Henry is of the view
that:

(i) Being a UK national, he will be a non-resident for Pakistan tax purpose;


(ii) His income from consultancy services provided by him under the contract of employment should be
classified as fees for technical services and shall be chargeable to tax at 15% of the gross amount of the
consideration received by him;
(iii) No tax was deducted from his remuneration. However, United Autos deposited an amount of Rs.275,000 in
the government treasury on his behalf. Mr. Henry believes that tax deposited on Mr. Henrys behalf does not
attract any additional tax incidence for him as he has not received the amount in cash, from the company.
(iv) Since his remuneration was agreed to be paid in Pound Sterling, the rate of conversion for tax purpose shall
be the rate applicable on the date of agreement. Any increase in value of Pound Sterling against Pakistan Rs.
should be non-taxable.
Briefly explain whether or not Mr. Henrys assumptions in (i) to (iv) above are in accordance with relevant provisions of the
CIR, 2001.

Solution of Q.NO. 3(a) Autumn 2008

i) Mr Henry assumption that being as UK National he is a non-reisdent for Pakistan tax purposes is not correct as the
residential status is being decided on number of days stay in Pakistan instead of Nationality base. As his stay in Pakistan
is for 183 days or more hence he is a reisdent for Pakistan tax purposes.

ii) As Mr. Henry is providing independent consultancy services to various departments of local company and the same
shall not be treated under the term employment hence a contract of services shall be treated as service contract and
higher of minimum tax at the rate of 10% or tax under normal tax regime on taxable profit, if any, is to be paid by Mr.
Henry.

iii) Tax paid by the company on behalf of Mr. Henry shall be treated as income and tax on the same is to be paid by Mr.
Henry on the same basis as given in Note 2 above.

iv) The presumption of Mr. Henry is again not correct as the conversion rate to the remuneration in pound sterling shall be
the date at which the amount received by Mr. Henry and not the date of agreement.

Q.NO.1 Spring 2008

Saleem, Rashid and Moin are partners in a partnership concern carrying on the business of manufacturing and sale of
consumer goods. They share profit and loss in the ratio 2:3:5 respectively. The results of operations of the firm are as
follows:

Rs. 000
Sales (including rental income) 76,000
Cost of sales 53,000
23,000
Selling, administrative and other expenses 16,250
Profit before tax 6,750

Conceptual Approach to Taxes _______ ___ _______________ __ 361


Chapter 21 _______ Solved Past Papers Income Tax Numericals of CA Module C - (2001 to 2015)

Tax deducted at source on import of raw material 900


Tax deducted at source on sale of goods 1,750

Other information:
(i) The firm has rented out a vacant portion of its factory to a company at an annual rental of Rs.1 million. Tax
was duly deducted by the lessee.
(ii) Mr. Saleem has earned income of Rs. 325,000 from another business as a sole proprietor. He also sold his
personal car for a loss of Rs. 50,000.
(iii) Mr. Rashid earned a gross income of Rs. 200,000 from another partnership firm where he is entitled to 25%
of the total profit of the firm. He also earned dividend of Rs. 50,000 from a listed company.
(v) Mr. Moin has no other source of income.

Required:
Assuming that the above data pertains to the tax year 2008, compute the tax liability of the firm and each of its partners
and the amount of tax payable by them alongwith the return of income.

Solution of Q.NO.1 Spring 2008

Name of Taxpayer : AOP


National Tax Number :
Income year ended : 30th June, 2016
Tax Year : 2016
Personal Status : AOP
Rs. Rs.
INCOME FROM BUSINESS U/S 18

COMPUTATION OF TAXABLE INCOME:

In the absence of information it has been assumed that the sale of the AOP is wholly to the persons that have
deducted withholding tax at source. Hence the income of the firm is fully covered under FTR.

Gross receipts of AOP on sale of goods manufactured Rs.1,750,000 x 100 / 4.5 38,888,889

Property Income 1,000,000


Less 1/5th repair allowance (200,000)
Rent chargeable to tax 800,000

COMPUTATION OF TAX LIABILITY:

4.5% tax on gross receipts as above 1,750,000


Add tax on income from property (32,000 + 15% (800,000 - Rs.750,000) ) 39,500
Total tax liability under the Ordinance 1,789,500

Less: Tax deducted at source

On import of raw materials 900,000


On property income (Rs. 1,000,000 - 150,0000) x 10% 85,000

On sale of goods 1,750,000 2,735,000


Balance tax refundable (945,500)

The income of the AOP coverved under FTR shall not be included in the income of the respective partner for rate
purposes.
TAX LIABILITY OF PARTNERS

Saleem

362_________________ ____________ _______ _Conceptual Approach to Taxes


Chapter 21 ___________Solved Past Papers Income Tax Numericals of CA Module C - (2001 to 2015)

Loss on sale of personal car shall not be recognized. -


Share in rental income after tax (Rs.800,000 - 39,500) x 2/10 152,100
Business income from sole proprietorship 325,000
Total Income 477,100
Tax liability 7% of [ Rs.476,500 - 400,000] 5,355

Rashid

Share from another AOP 200,000


Share in rental income after tax (Rs.800,000 - 39,500) x 3/10 228,150
Dividend Income (Rs. 50,000 not to be included in income) -
Total Income 428,150

Tax liability 7% of [ Rs.427,250 - 400,000] 1,908


Add tax on dividend income 5,000
Tax liability 6,908

Moin
Business Income from AOP -

No tax liability has been computed as there is no taxable income from any head of income hence the share from AOP in
retanl income shall also not included for rate purposes.

Q.NO.1 Autumn 2007

Mr. Ayub, after retirement from a multinational company as a senior executive, was rehired on contract for a period of
three years. However, due to certain reasons, the contract was prematurely terminated six months earlier i.e. on
December 31, 2006. The detail of emoluments received by him during the tax year 2007 are given below:

Rupees
Basic salary (per month) 70,500
Rent of furnished accommodation (per month) 30,000
Utilities allowance (per month) 12,000
Medical benefits reimbursed during the year 25,000

House rent was paid by the company directly to the landlord. Medical benefits were reimbursed against bills submitted by
Mr. Ayub.

On his retirement as a permanent employee, he had been paid gratuity from the approved fund. According to the rules of
the fund, he was also entitled to a special gratuity in lieu of his services rendered under the contract. Accordingly, an
amount of Rs. 120,000 was also paid out of the fund, on termination of the contract.
In lieu of premature termination, the following additional benefits were allowed to Mr. Ayub:
(i) A compensation for early termination of Rs. 150,000 was paid.
(ii) Mr. Ayub had obtained an interest free loan of Rs. 200,000 on July 1, 2006 which was payable in lumpsum on
March 31, 2007. 25% of the outstanding balance was waived and remaining amount of loan was deducted
from his final settlement. The benchmark rate according to the ITO, 2001 is 10%.
(iii) He was allowed to retain a 1600cc car which was in his use, at accounting book value of Rs. 650,000. The
fair market value of the car at the time of settlement was Rs. 700,000.
Required: Compute the taxable income and tax liability for the tax year 2007.

Solution of Q.NO.1 Autumn 2007

Name of Taxpayer : Mr. Ayub


National Tax Number :
Income year ended : 30th June, 2016
Tax Year : 2016

Conceptual Approach to Taxes _______ ___ _______________ __ 363


Chapter 21 _______ Solved Past Papers Income Tax Numericals of CA Module C - (2001 to 2015)

Personal Status : Individual


Residential Status : Resident

INCOME FROM SALALRY U/S 12 Rs. Rs.

Basic Salary (Rs. 70,500 x 6 months) 423,000


Rent of furnished accommodation (30,000 per month) higher of
Fair market rent or 180,000
45% of MTS or basic salary 190,350 190,350
Utilities allowance (Rs.12,000 x 6 month) 72,000
Medical benefits reimbursed during the year (fully exempted) -
Gratuity from the approved fund (N - 1)
A compensation for early termination (included in current year 150,000
income in the absence of last 3 year's income and tax thereon)
Interest on interest free loan (As the loan is less than Rs. 500,000 -
and rate is within benchmark rate hence no addition in the income
has been made)
Amount waived of loan (Rs. 200,000 x 25%) 50,000
Conveyance facility retained by employee at FMV 700,000
Total taxable income 1,585,350

COMPUTATION OF TAX LIABILITY:

Tax on Rs. 1,585,350 [92,000 + 15% x (1,585,350 - 1,500,000)] 104,803

NOTES
N-1 Assumed that Gratuity received is approved by Commissioner therefore it is totally exempted.

Q NO. 3(a) Autumn 2007

Mr. Zia inherited certain assets from his father in the year 2004. The fair market values of the assets on the date of
inheritance were as follows:

Fair Market Value


(Rs.)
25,000 shares of a private limited company 2,500,000
21,000 shares of a public listed company 462,000
Membership card of Karachi Stock Exchange 20,000,000
Jewellery 1,500,000
During the tax year 2007, Mr. Zia undertook the following transactions:
(1) He gifted some of the assets to his 20-year old son Mr. Ishaq. The detail and fair market values of the assets
are as follows:

Fair Market Value (Rs.)


10,000 shares of the private limited company 2,000,000
10,000 shares of the public listed company 1,700,000
Membership card of Karachi Stock Exchange 40,000,000
(2) The remaining shares were sold as follows:
sharesofprivatelimitedcompanyforRs.3,000,000,
sharesofpubliclimitedcompanyforRs.1,500,000.

Mr. Ishaq sold all the assets transferred through gift in the same year. The assets fetched the following amounts:

Sales Proceeds (Rs.)


10,000 shares of a private limited company 2,500,000
10,000 shares of a public listed company 1,500,000
Membership card of Karachi Stock Exchange 55,000,000

364_________________ ____________ _______ _Conceptual Approach to Taxes


Chapter 21 ___________Solved Past Papers Income Tax Numericals of CA Module C - (2001 to 2015)

Required:
(i) Based on the above information, compute the taxable income of Mr. Zia and Mr. Ishaq for the tax year 2007.
(ii) Give brief explanation for the items not included in the taxable income.

Solution of Q NO. 3(a) Autumn 2007

Mr. Zia
Gain on disposal of Assets Rs. Rs.
Gain from sale of shares of public listed company is taxable as SBI 1,258,000
@ 7.5% as it is held for more than 24 months (Rs. 1,500,000 -
242,000 FMV of sodl shares at the date of gift)

Gain on sale of shares of private company


Consideration received 3,000,000
Less: FMV at the time of inheritance Rs.(2,500,000 / 25,000 x (1,500,000)
15,000)
Gain on disposal 1,500,000
Capital gain for taxable purposes (Rs.1,500,000 x 0.75) 1,125,000

Gain on Acquisition of Jewellery


No gain or loss shall be recognized on the acquisition of any asset received by way of inheritance or gift.

Mr. Ishaq
No gain or loss shall be recognized on the acquisition of any asset received by way of inheritance or gift.

Gain on disposal of Assets


Loss from sale of shares of public listed company can neither be (200,000)
adjusted against gain realized under section 37 nor it can be carried
forward. Rs.(1,500,000 - 1,700,000)

Gain on sale of shares of private company


Consideration received 2,500,000
Less: FMV at the time of inheritance (2,000,000) 500,000

Gain on membership card of stock exchange


Consideration received 55,000,000
Less: FMV at the time of gift as deemed as cost (40,000,000) 15,000,000
Taxable Income 15,500,000

Solution of Q NO. 3(a) Autumn 2007

No gain or loss shall be recognized on the acquisition of any asset received by way of inheritance or gift.

Q.NO. 4(b) Autumn 2007

During the tax year 2007, Mr. Yahya, a resident person, derived an income of Rs. 1,500,000 from his business in
Pakistan. He has also earned an amount of US$ 30,000 from his business in a foreign country on which he paid income
tax to tax authorities of that country, amounting to US$ 10,500.

Compute the tax liability of Mr. Yahya for the tax year 2007.
Note: Applicable Tax Rate in Pakistan = 25%; US$ 1 = Pak Rupees 60.

Solution of Q.NO. 4(b) Autumn 2007

Conceptual Approach to Taxes _______ ___ _______________ __ 365


Chapter 21 _______ Solved Past Papers Income Tax Numericals of CA Module C - (2001 to 2015)

Name of Taxpayer : Mr. Yahya


National Tax Number :
Income year ended : 30th June, 2016
Tax Year : 2016
Personal Status : Individual
Residential Status : Resident

TOTAL TAXABLE BUSINESS INCOME U/S 18 Rs. Rs.

Pakistan source business income 1,500,000


Foreign source business income ($ 30,000 @ Rs. 60) 1,800,000
Total taxable business income 3,300,000

COMPUTATION OF TAX LIABILITY:

Tax on Rs. 3,300,000 [344,500 + 25% x (3,300,000 - 2,500,000)] 544,500


Less: Foreign tax credit i.e. Lower of
(A) Foreign income tax paid ($10,500 x Rs.60) 630,000
(B) Pakistan income tax (tax liability x Foreign source income /
Taxable income)
Rs.(544,500 x 1,800,000 / 3,300,000) 297,000 297,000
Balance tax payable 247,500

Q.NO. 2(a): Spring 2007

Explain the correct tax treatment in each of the following situations:

(i) In 1998, Mr. Hamid inherited a rare sculpture of Buddha which had a fair market value of Rs. 200,000 on the
date of inheritance. In February 2007, the sculpture was sold by him at Rs. 500,000.
(ii) In December 2006, Mr. Yahya entered into an agreement for sale of his residential plot to Mr. Moosa, who paid
an advance of Rs. 500,000. According to the agreement, Mr. Moosa was required to pay the balance by
February 28, 2007. However, instead of paying the balance amount, he terminated the sale agreement.
Mr. Yahya forfeited the advance of Rs. 500,000 in accordance with the terms of the agreement.
(iii) In September 2006, Mr. Saleem sold his personal car, Toyota Corolla, to one of his cousins at a price of Rs.
50,000 whereas the fair market value of the car was Rs. 200,000. The car was purchased by him in the year
2000 at a cost of Rs. '300,000
(iv) Mr. Ibrahim was working as a Chief Financial Officer in Dawood Pakistan (Pvt.) Limited, which is a wholly
owned subsidiary of Dawood AG, Germany. According to the Companys policy, Mr. Ibrahim was sent on
secondment to Germany on January 1, 2007 for a period of five years. During this period, half of his salary
will be credited to his bank account in Pakistan, whereas the remaining portion will be received by him in
Germany.

Mr. Zubair provided consultancy services to a listed company. In consideration for his services, he received a net amount
of Rs. 47,000 after tax deduction of Rs. 3,000.

Solution of Q.NO. 2(a): Spring 2007

i) Gain on sale of Sculpture Rs. Rs.


Consideration received 500,000
Less: FMV at the time of inheritance (200,000)
300,000
Taxable gain on sale of sculpture (as holding period is more than 1 225,000
year so 75% is chargeable to tax) (Rs.300,000*0.75)

366_________________ ____________ _______ _Conceptual Approach to Taxes


Chapter 21 ___________Solved Past Papers Income Tax Numericals of CA Module C - (2001 to 2015)

ii) Any forfeited money as advance against sale agreement of land 500,000
and building is included in the definition of "Rent". So, the amount
of forfeited money shall be chargeable to tax.

iii) As the sale of personal car is neither a business transaction nor a capital asset. There will be no tax treatment on the
disposal of personal asset. So, the cost, FMV or consideration are irrelevant.
iv) Mr. Ibrahim's salary income is taxable in Pakistan in 2007 and foreign source income in next 5 years as his
employment based in Pakistan according to section 101.

v) Any tax deducted at source on consultancy services is treated as minimum tax however the tax under normal law shall
be computed and higher from both is to be paid by Mr. Zubair.

Q.NO. 3(b) Spring 2007

The income of Mr. Yousuf during the tax year 2006 amounted to Rs. 120 million which included capital gains of Rs. 10
million and dividend income of Rs. 12 million. The tax liability for 2006 was Rs. 32 million out of which Rs. 4 million related
to tax on capital gains and dividend income. The following information is available for the quarter ended December 31,
2006:

Rs. in million
Tax deducted at source by the customers 3
Tax paid on import 2
Compute advance tax liability for the quarter ended December 31, 2006.

Solution of Q.NO. 3(b) Spring 2007

Name of Taxpayer : Mr. Yousaf


National Tax Number :
Income year ended : 30th June, 2016
Tax Year : 2016
Personal Status : Individual
Residential Status : Resident
Rs. (in millions) Rs. (in millions)
COMPUTATION OF ADVANCE TAX LIABILITY UNDER SECTION
147:

Total Income 120


Less: Capital Gain (See note - 1 below) -
Dividend Income (12) (12)
Net income (excluding income covered under SBI / FTR) 108

Computation of advance tax Liability


Total Tax 32
Less: Tax on dividend income 1.20
Balance tax under NTR (including capital gain covered U/S 37) 30.80

As the latest tax year income under NTR of Mr. Yousaf is more
than Rs. 500,000 therfore the quarterly advance tax liability on the
basis of latest tax year is as under.

Latest tax year tax liability under NTR 30.80


Quarterly advance tax liability shall be 1 / 4 th of the above amount 7.70

Conceptual Approach to Taxes _______ ___ _______________ __ 367


Chapter 21 _______ Solved Past Papers Income Tax Numericals of CA Module C - (2001 to 2015)

Note-1 In the absence of information it has been assumed that the capital gain is on capital assets covered under section
37 that is under NTR hence the same has also been considered while computing advance tax under section 147 of the
Income tax Ordinance, 2001.

Note-2 As for individual the dividend income is fully covered under SBI hence the same has been ignored for the
computation of advance tax under section 147.

Note-3 In the absence of information it has been assumed that the individual is engaged in trade business hence the tax
deducted at source on sale of goods and commercial import have not been deducted from the advance tax liability as the
same are fully covered under Final tax regime.

Q.NO. 2(b) Autumn 2006

Mr. Dollar has been working as a senior engineer in a local company. The detail of his monthly emoluments is as
under:

Basic salary Rs.100,000


Medical allowance Rs.15,000
Utilities allowance Rs.10,000

In addition to the above cash emoluments, he is entitled to the following perquisites:


(i) A car for his personal and official use, having cost of Rs.700,000 to the employer.
(ii) Rent free accommodation having monthly rent of Rs.20,000 or cash in lieu thereof. However he has opted to
take rent free accommodation.
(iii) Special allowance of Rs.15,000 to meet travelling, boarding and lodging expenses to be incurred by him in the
normal course of his employment duties.

You are required to compute the amount of tax to be deducted each month, from his salary for tax year 2007.

Solution of Q.NO. 2(b) Autumn 2006

Name of Taxpayer : Mr. Dollar


National Tax Number :
Income year ended : 30th June, 2016
Tax Year : 2016
Personal Status : Individual
Residential Status : Resident

INCOME FROM SALARY U/S 12 (Rs.) (Rs.)

Basic salary (Rs.100,000 x 12 months) 1,200,000


Medical allowance (Rs.15,000 x 12 months) 180,000
Less: 10% of basic salary exempt U/C 139 (Rs.1,200,000 x 10%) (120,000) 60,000

Utilities allowance (Rs.10,000 p.m) 120,000


Conveyance facility provided for personal and official use
Taxable at 5% of cost of conveyance (700,000 x 5%) 35,000
Rent free accommodation (Higher of FMR or 45% of MTS or B.S)
Higher of (Rs.240,000 (20,000 x 12) or Rs.540,000 (45% x 540,000
1,200,000))
Special allowance is not taxable as it is reimbursed for business purposes -
Total taxable income 1,955,000

COMPUTATION OF TAX LIABILITY:

Tax on Rs. 1,955,000 [137,000 + 17.5% x (1,955,000 - 1,800,000)] 164,125


Per month tax to be deducted by employer (Rs. 164,125 / 12 months) 13,677

368_________________ ____________ _______ _Conceptual Approach to Taxes


Chapter 21 ___________Solved Past Papers Income Tax Numericals of CA Module C - (2001 to 2015)

Q.NO. 1 Spring 2006

Ms. Fatima Hasan was working as a Marketing Head with Consumer Products Limited (CPL) at following emoluments:

(i) Basic salary Rs. 100,000 per month


(ii) House rent allowance Rs. 40,000 P.M.
(iii) Utilities allowance Rs. 15,000 per month
In addition to the above cash emoluments, she was provided with a Honda Civic car, exclusively for official use. The cost
of car to the Company was Rs. 1,000,000. As per Companys policy, the car was sold to Fatima in January 2005 at the
written down value of Rs. 100,000 whereas the fair market value of the same at the time of sale was Rs. 300,000.

In May 2005, Fatima was approached by Pharma Industries (Pvt.) Limited (PIL). They offered her employment at a higher
salary and some extra benefits, alongwith a one time payment of Rs. 200,000 as an inducement to accept their offer.
Fatima accepted PILs offer by resigning from CPL with effect from June 1, 2005. She joined PIL from July 1, 2005. The
amount of Rs 200,000 was, however, paid to her on June 29, 2005.
During the year, Fatima has also undertaken the following transactions:
(i) Shares in QP (Pvt.) Ltd. were sold for Rs. 500,000. These shares were acquired in the year 1999 at a cost of
200,000
(ii) A residential plot inherited in the year 2000 was sold for Rs. 1,000,000. The fair market value of the plot at the
time of inheritance was Rs. 200,000.
(iii) A painting purchased at a cost of Rs. 100,000 was sold for Rs. 75,000.
(iv) She had won a cash prize of Rs. 250,000 in a quiz show. Tax of Rs. 50,000 was deducted from the prize
money u/s 156.
(v) Dividend of Rs. 50,000 was received on account of shareholding in a listed Company. Tax of Rs. 5,000 was
deducted u/s 150.
(vi) She received a fee of Rs. 100,000 in consideration for preparing a research paper for a foreign University.
Fatima incurred Rs.10,000 on the printing of research paper and courier charges for sending the paper abroad.
(vii) An amount of Rs. 50,000 was donated to an approved charitable institution.
In the light of above information, compute the taxable income of Ms. Fatima for the tax year 2005 by giving brief
explanation for the items not included in the taxable Income.

Solution of Q.NO. 1 Spring 2006

Name of Taxpayer : Fatima Hassan


National Tax Number :
Income year ended : 30th June, 2016
Tax Year : 2016
Personal Status : Individual
Residential Status : Resident

INCOME FROM SALARY U/S 12 Rs. Rs.

Basic salary (Rs.100,000 x 11 months) 1,100,000


House rent allowance (Rs. 40,000 x 11 months) 440,000
Utilities allowance (Rs. 15,000 x 11 months) 165,000
Benefit on purchase of car from employer
FMV at the time of purcahase 300,000
Less: Consideration paid to employer (100,000) 200,000
Benefit from Pharma Industry (Pvt.) Limited 200,000
2,105,000
CAPITAL GAIN U/S 37

Gain on sale of shares in QP (Pvt.) Ltd. 500,000


Consideration received on disposal (200,000)

Conceptual Approach to Taxes _______ ___ _______________ __ 369


Chapter 21 _______ Solved Past Papers Income Tax Numericals of CA Module C - (2001 to 2015)

300,000
Taxable capital gain (holding period more than 1 year) 75% x Rs.300,000 225,000
Loss on sale of painting is not recognized (N - 1)

INCOME FROM OTHER SOURCES U/S 39

Consideration for preparing a research paper for a foreign University. 100,000


Less: Expenses paid on the printing of research paper and courier charges (10,000) 90,000
Total Income 2,420,000

Income taxable under FTR


Cash prize (gross amount) 250,000
Dividend received (gross amount) 50,000

NOTE: N-1 Gain on the sale of residential plot after two years of retention is not taxable under capital gains.

Q.NO. 7(b) Spring 2006

The records of Mr. A show the following results:

Particulars Rupees
Loss from income from other source after setting off dividend income of Rs. 30,000 (20,000)
Income from speculation business 10,000
Capital gains on disposal of shares of private limited companies 20,000
Loss from business of textiles after considering tax depreciation of Rs. 290,000 410,000

Required:
You are required to work out the following:
(i) taxable income;
(ii) tax liability; and
(iii) amount of loss that can be:
(a) adjusted against any other head of income;
(b) carried forward for maximum 6 years;
(c) carried forward for indefinite period.

Solution of Q.NO. 7(b) Spring 2006

Name of Taxpayer : Mr. A


National Tax Number :
Income year ended : 30th June, 2016
Tax Year : 2016
Personal Status : Individual
Residential Status : Resident

i) Taxable Income Rs. Rs.


Loss from business (excluding depreciation loss Rs. 290,000) (120,000)
Income from speculation business 10,000
Income from capital gain (assumed within one year under section 37) 20,000
Loss from other sources after dividend income (20,000)
Less: dividend income (30,000) (50,000)
Business loss (140,000)
Less tax depreciation (290,000)
Total loss as per tax (430,000)

ii) Computation of tax liability

370_________________ ____________ _______ _Conceptual Approach to Taxes


Chapter 21 ___________Solved Past Papers Income Tax Numericals of CA Module C - (2001 to 2015)

As there is loss under normal law and information regarding


turnover under normal law has not been given, hence tax only on
dividend income is to be computed as under.
Tax on dividend income (Rs. 30,000 x 10%) assumed at normal 3,000
rate

iii) Amount of loss that can be:


(a) adjusted against any other head of income
Loss under the head income from other sources and total business 460,000
loss (including depreciation) may be adjusted against profit from
any other head of income.

(b) Carried forward for maximum 6 years


Business loss (excluding depreciation loss) shall be carried forward 140,000
for 6 years.

(c) Carried forward for indefinite period.


Unabsorbed depreciation loss shall be carried forward for indefinite 290,000
period.

Q.NO. 2 Autumn 2005

Mr. Imran is a citizen of Pakistan. During the first nine months of the tax year 2005, he worked as financial controller of a
Pakistan based subsidiary of a multinational group. After that he was transferred and employed as Head of Finance of the
UAE based subsidiary of the Group. Mr. Imrans family stayed in Dubai throughout the year. The detail of income earned
by him during the tax year 2005 is given below:

From the UAE company:


Mr. Imran earned US $ 30,000 during the three-months employment in the UAE. No tax is deducted from salary earned
and paid in the UAE. To relocate Mr. Imran in UAE, the UAE Company incurred one time miscellaneous cost of Rs.
100,000 to move the household items of Mr. Imran from Pakistan to Dubai.

From Pakistan subsidiary:


(a) Basic salary Rs. 500,000 p.m.
(b) Medical allowance Rs. 45,000 p.m (no free medical or hospitalization facility is given to Mr. Imran under the
terms of employment).
(c) The company has provided Mr. Imran a TV and VCR costing Rs. 40,000 on which the company charges
depreciation at the rate of 20% in its books of accounts.
(d) Company has provided interest free loan to Mr. Imran amounting to Rs. 5 million which remained
outstanding throughout his employment with the company. Mr. Imran acquired a flat from the amount of loan
and rented it out at the rate of Rs. 50,0000 p.m. for a period of seven months. He also paid Rs. 35,000 as
property tax during the period.
(e) His familys housing cost in Dubai, borne by the company amounts to Rs. 30,000 p.m.
(f) Mr. Imrans travelling and related cost borne by the Pakistan subsidiary to meet his family, amounts to Rs.
30,000 p.m.
(g) During the employment with the Pakistan subsidiary, Mr. Imran had exercised option to acquire 300 shares of the
parent company at the rate of US $ 8 per share. At the time when the option was exercised, the value of the share was
US $ 10 (Rs.58) per share. Furthermore, during the year Mr. Imran sold 200 options previously received by him at a price
of US $ 3 per option (Rs. 171) after holding it for more than a year. Neither the Pakistan subsidiary nor Mr. Imran incurred
any cost in this regard.

Required:
Compute the taxable income of Mr. Imran for the tax year 2005 based on the data provided above.

Solution of Q.NO. 2 Autumn 2005

Name of Taxpayer : Mr. Imran

Conceptual Approach to Taxes _______ ___ _______________ __ 371


Chapter 21 _______ Solved Past Papers Income Tax Numericals of CA Module C - (2001 to 2015)

National Tax Number :


Income year ended : 30th June, 2016
Tax Year : 2016
Personal Status : Individual
Residential Status : Resident

SALARY FROM UAE COMPANY U/S 12 Rs. Rs.

Basic salary ($30,000 x Rs.90 assumed) 2,700,000


Expenses borne by company to move the household items 100,000 2,800,000

SALARY FROM PAKISTAN SUBSIDIARY U/S 12

Basic salary (Rs.500,000 x 9 months) 4,500,000


Medical allowance (Rs.45,000 x 9 months) 405,000
Less: 10% of basic salary (Rs.4,500,000 x 10%) 450,000 -
Cost of TV and VCR (Rs.40,000 x 20%) 8,000
Interest free loan (Rs.5,000,000 x 10% x 9 / 12) 375,000
Familys housing cost in Dubai (Rs.30,000 x 12 months) 360,000
Imrans travelling and related cost borne by the Pakistan subsidiary (Rs.30,000 x 9 months) 270,000
Share option scheme
Benefit on acquisition of shares (300 shares x 2 x 58) (Note 1 attached) 34,800
Benefit on sale of share options (200 options x 171) 34,200 69,000
Total Income 8,382,000

INCOME FROM PROPERTY U/S 15


Rental Income (Rs. 50,000 x 7 months) (assumed after allowable 350,000
expenses)
Taxable income 8,732,000

Note -1 Grant of an option or right is not taxable whereas exercise of an option to acquire shares is taxable where
the same is without restriction and limitation.

Q.NO. 15: Spring 2005

Mr B is the Chief Executive of a Multinational Company. Details of his emoluments are as follows:
Rs.
Basic Salary 8,800,000
Bonus 5,000,000
Utility allowance 880,000
Relocation allowance 200,000

Apart from the above he is provided with the following perquisites/benefits:


(i) A free unfurnished accommodation by the employer with land area of 2100 sq. yds.
(ii) Motor vehicle for both private and official use, cost of acquisition of which was Rs.2,000,000.
(iii) Children education fees for the year Rs.105,000.
(iv) House servant salaries for the year Rs.230,000.

According to the terms of employment the tax liability of Mr. B on the above benefits and perquisites from (i) to (iv) above
is borne by the employer. Tax liability on other remuneration is borne by himself.

Mr. B also owns a property which was let out on rent for a part of the year details of income and expenses incurred are
as follows:

(a) Rent Rs.50,000 per month.


(b) The property was let out on rent from December 2, 2003 to June, 2004
(c) Property tax paid Rs.35,000.

372_________________ ____________ _______ _Conceptual Approach to Taxes


Chapter 21 ___________Solved Past Papers Income Tax Numericals of CA Module C - (2001 to 2015)

The Bank account of Mr. B was credited with profit during the year amounting to Rs.6,300. During the year the
following amounts were withheld at source as Income Tax:

Rupees
From salary income 4,541,250
Tax paid by the employer 446,820
From profit on bank account 630
On receipt of rent 17,500

You are required to compute the taxable income and tax liability of Mr. B for the tax year 2004.

Solution of Q.No. 15 Spring 2005

Name of Taxpayer : Mr. B


National Tax Number :
Income year ended : 30th June, 2016
Tax Year : 2016
Personal Status : Individual
Residential Status : Resident
(Rs.) (Rs.)
INCOME FROM SALARY U/S 12

Basic Salary 8,800,000


Bonus 5,000,000
Utility allowance 880,000
Relocation allowance 200,000
Free unfurnished house higher of (FMV or 45% of MTS or Basic pay)
(45% of Rs. 8,800,000) is taxable 3,960,000
Conveyance facility partly for personal use (5% of Rs. 2,000,000) 100,000
Children education fees for the year 105,000
House servant salaries for the year 230,000
Tax paid by employer 446,820
19,721,820
INCOME FROM PROPERTY U/S 15

Rental income (Rs.50,000 x 7 months) 350,000


Less fixed 1/5 th repair allowance (70,000)
Less property tax (35,000)
Net income from property 245,000
Total taxable income 19,966,820

COMPUTATION OF TAX LIABILITY:

Tax on Rs. 19,966,820 [1,422,000 + 30% x (19,966,820 - 7,000,000)] 5,312,046


Add Tax under FTR
10% tax on PLS profit of Rs. 6,300 630
5,312,676
Less: Tax already paid
Tax paid by emplyer 446,820
Tax deducted from salary 4,541,250
Tax paid on PLS profit 630
On receipt of rent 17,500 5,006,200
Balance tax payable 306,476

Q.NO.15: Autumn 2004

Conceptual Approach to Taxes _______ ___ _______________ __ 373


Chapter 21 _______ Solved Past Papers Income Tax Numericals of CA Module C - (2001 to 2015)

Mr. A is the Chief Executive of a multinational company. Details of his emoluments are as follows:

Basic salary 4,004,520


Bonus 1,980,642
Utility allowance 400,452
Leave encashment 538,083
Other allowance 90,000
House rent allowance 1,802,040

Apart from the above he has received Directors fee amounting to Rs. 52,000. During the year he has sold shares that
were acquired through exercise of a Stock Option(being the a share, of a UK company) two years ago. The gain on sale
amounts to Rs.4,206,000.

He also owns a property which has been let out on rent. The details of rent received and expenses incurred are as
follows:

(a) Rent Rs.10,000 per month. The property was let out on rent for the whole year. The annual letting
value of the house is 'Rs.100,000.
(b) He has paid property tax amounting to Rs. 11,500.
(c) During the year he has paid Rs.6,000 for repairs and maintenance.

He has also received profit on PLS Account at Rs.6,500.

During the year the following amounts were withheld at source towards income tax.

(a) From salary income Rs. 3,600,000


(b) From profit on PLS Account Rs.650

You are required to compute the taxable income and tax liability of Mr. A for the tax year 2004

Solution of Q.No 15 Autumn 2004

Name of Taxpayer : Mr. A


National Tax Number :
Income year ended : 30th June, 2016
Tax Year : 2016
Personal Status : Individual
Residential Status : Resident
(Rs.) (Rs.)
INCOME FROM SALARY U/S 12

Basic salary 4,004,520


Bonus 1,980,642
Utility allowance 400,452
Leave encashment 538,083
Other allowance 90,000
House rent allowance 1,802,040
Directors fee (assumed as employee) 52,000
Total income 8,867,737

INCOME FROM PROPERTY U/S15

Rental Income (Rs.10,000 p.m x 12 months) 120,000


Less fixed 1/5th repair allowance (24,000)
Property tax (11,500)
Net income from property 84,500

374_________________ ____________ _______ _Conceptual Approach to Taxes


Chapter 21 ___________Solved Past Papers Income Tax Numericals of CA Module C - (2001 to 2015)

Taxable income 8,952,237

COMPUTATION OF TAX LIABILITY:

Tax on Rs. 8,952,237 [1,422,000 + 30% x (8,952,237 - 7,000,000)] 2,007,671

Tax on capital gain on disposal of Listed Company shares under 315,450


section 37A after two years (Rs. 4,206,000 x 7.5%)
Tax on profit on debt @ 10% 650
Total tax liability under NTR and FTR 2,008,321
Less: tax deducted at source from salary 3,600,000
Less: tax deducted at source on PLS profit on debt 650 3,600,650
Balance tax refundable (1,592,329)

NOTES

N-1 In the absence of information, it has been assumed that the shares of the company on which gain is given in
the question is in the definition of Public company hence tax has been levied accordingly u/s 37A of the Ordinance.
N-2 Profit on PLS is covered as SBI under final tax regime.

Q.NO. 2 Spring 2004

Mr. A is an employee of a multinational company incorporated in Pakistan. His remuneration during the year was as
follows -

(Rupees)
1 Basic Salary 1,117,245
2 Reward 22,062
3 Bonus 300,000
4 House Rent Allowance 643,514
5 Utility Allowance 111,724

The Company has provided him a car for personal and business use. The cost of the car was Rs.1,100,000. During the
year Mr. A has paid interest on loan borrowed for construction of a house amounting to Rs.115,000. In addition to the
above, Mr. A was granted Stock Option of 2500 shares by the Head Office of the Company at US$ 36 per shares. Out of
the above stock option, 1250 shares vested to him during the year were immediately exercised by him. The price of the
share at the time of exercise was US$ 41 per share. The exchange rate between US$ and Pak Rupee on the date on
which Mr. A exercised his option was US$ 1 = Rs.58.

During the year the company has withheld tax from his salary amounting to Rs. 695,000.
You are required to compute his taxable income and tax thereon for the Tax Year 2003.

Solution of Q.NO. 2 Spring 2004

Name of Taxpayer : Mr. A


Income year ended : 30th June, 2016
Tax Year : 2016
Personal Status : Individual
Residential Status : Resident
(Rs.) (Rs.)
INCOME FROM SALARY U/S 12

Basic Salary 1,117,245


Reward 22,062
Bonus 300,000
House Rent Allowance 643,514
Utility allowance 111,724

Conceptual Approach to Taxes _______ ___ _______________ __ 375


Chapter 21 _______ Solved Past Papers Income Tax Numericals of CA Module C - (2001 to 2015)

Conveyance facility for both official and for business use (5% of 1,100,000) 55,000
Employee stock option (1,250 shares x 58 x (41 - 36)) 362,500
Taxable Income 2,612,045

COMPUTATION OF TAX LIABILITY:

Tax on Rs. 2,612,045 [259,500 + 20% x (2,612,045 - 2,500,000)] 281,909


Less: tax withheld by employer 695,000
Balance tax refundable (413,091)
Notes
Note 1: In the absence of information, it has assumed that the loan availed is not fulfilling the requirements of section 64A
hence no deductible allowance has been claimed for the same.

Q.NO. 6 Spring 2004

Mr. A and B are equal partners of a Registered Firm (RF). The profit and loss account of RF shows profit before tax of Rs.
10 million for the year ended June 30, 2003. Assuming no other tax adjustment, the profit shown in the accounts is liable
to tax. You are required to compute the tax, if any, payable by the RF, Mr. A and Mr. B, assuming Mr. A and B have no
other source of income. Also give brief explanation of the treatment made in the computation.

Solution of Q.NO. 6 Spring 2004

Name of Taxpayer : AOP


National Tax Number :
Income year ended : 30th June, 2016
Tax Year : 2016
Personal Status : AOP

Divisible Income of the Firm (Rs.)


Profit before tax 10,000,000
Tax on Rs. 10,000,000 [1,319,500 + 35% x (10,000,000 - 6,000,000)] 2,719,500
Profit after tax 7,280,500
Share of each Partner
Share of A (7,280,500 x 1/2) 3,640,250
Share of B (Rs. 7,280,500 x 1/2) 3,640,250
7,280,500
Note: The partners are not required to pay any tax liability as they have no other source of income. Income from
AOP shall be exempt from tax in the hands of partners as the tax paid by the AOP. If the partners have any other
source of income then the share in profit from AOP shall be added in taxable income for rate purposes.

Q.NO. 2 Autumn 2003

Mr. Bashir Ahmed is an employee who had joined his current employment during the tax year 2003. His details of salary,
allowance and perquisites received from company A his previous employer and company B his present employer are
as follows:

Description Company A Company B


Rs. Rs.
Basic Salary 714,158 572,572
Bonus 150,000 71,800
House Rent Allowance 258,663 222,746
Utility Allowance 71,415 57,257
Conveyance provided by employer partly used for business and private 1,100,000
use- Cost of the car purchased by the company
Leave encashment 77,783 NIL

376_________________ ____________ _______ _Conceptual Approach to Taxes


Chapter 21 ___________Solved Past Papers Income Tax Numericals of CA Module C - (2001 to 2015)

Medical reimbursement as per the terms of employment 35,000 25,000


Ex-gratia payment received under Golden Handshake Scheme 2,048,300 -

The details of assessed income and assessed tax in respect of past three years is as follows:
Assessment year Assessed Tax assessed
Income
Rs. Rs.
2000-2001 1,309,570 269,902
2001-2002 1,545,850 371,255
2002-2003 2,264,940 557,633

During the year Company A had deducted tax u/s 149 amounting to Rs.270,000 and Company B had deducted tax u/s
149 amounting to Rs.800,000 from payments made to Mr. Bashir.

Required:
Compute the taxable income and tax liability of Mr. Bashir based on the data provided above for the tax year 2003.

Solution of Q.NO. 2 Autumn 2003

Name of Taxpayer : Mr. Bashir Ahmed


National Tax Number :
Income year ended : 30th June, 2016
Tax Year : 2016
Personal Status : Individual
Residential Status : Resident

INCOME FROM SALARY U/S 12 Total (Rs.)


Co. A + Co. B
Basic Salary 1,286,730
Bonus 221,800
House Rent Allowance 481,409
Utility Allowance 128,672
Conveyance provided by employer partly for personal use (1,100,000 x 5%) 55,000
Leave encashment 77,783
Medical reimbursement as per the terms of employment (fully exempt) 0
Ex-gratia payment 2,048,300
Salary Income taxable under normal procedure 4,299,694

COMPUTATION OF TAX LIABILITY:

Tax under Option 1


Normal procedure:
Tax on Rs. 4,299,694 [Rs.597,000 + {(4,299,694 - 4,000,000) x 679,416
27.5%]

Tax under Option 2


Tax on income without golden handshake
Tax on Rs. 2,251,394 [Rs.137,000 + {(4,299,694 - 2,048,300-
1,800,000) x 17.5%]
215,994

Tax on golden hand shake: ( tax of last 3 years / Taxable Income 479,553
of last 3 years x 100) x Amount of golden hand shake Rs.
(1,198,790 / 5,120,360 x 2,048,300)
Total tax payable 695,547

Conceptual Approach to Taxes _______ ___ _______________ __ 377


Chapter 21 _______ Solved Past Papers Income Tax Numericals of CA Module C - (2001 to 2015)

Tax liability
As tax under option 1 is lower than from tax payable under option 2 679,416
hence the tax payer shall opt to pay tax under option 1.
Less: Tax deducted at source 1,070,000
Balance tax refundable (390,584)

Note 1: for golden handshake payments Income Tax


year 1 1,309,570 269,902
year 2 1,545,850 371,255
year 3 2,264,940 557,633
Total 5,120,360 1,198,790
Golden handshake payment is taxed as separate block of income, if we include golden handshake payment in
salary income the higher amount of tax shall be payable on it.

Q.NO. 4 Autumn 2003

Compute the projected advance tax liability and net advance tax payable in respect of ABC Limited a public company, for
the quarter ended September 30, 2003. The data of turnover and tax liability assessed in respect of the latest assessed
tax year is as follows:

Rs.
(i) Gross sales (including sale of imported goods and export sales) 20,000,000
Sales of imported goods 2,000,000
Export sales 3,000,000
Agency commission 1,000,000
Sale of fixed assets 200,000
Dividend income 1,000,000
Miscellaneous income 1,500,000
(ii) Gross Tax Liability 1,200,000
Tax on export sales 30,000
Tax on Import of goods 108,000
Tax on dividend income 50,000
The projected turnover and taxes expected to be withheld at source are as follows:-
All figures are for the quarter ended September 30, 2003
(i) Gross sales 5,000,000
Sale of imported goods 500,000
Export sales 1,000,000
Dividend income Nil
Miscellaneous income 500,000
(ii) Tax collection/deduction:
- U/s 148 on goods imported for sale 24,000
- U/s 153 on sale of imported goods 95,000
- U/s 154 on export sale 10,000

Solution of Q.NO. 4 Autumn 2003


Rs. Rs.
Advance Tax Payable
(Latest assessed tax under Normal tax regime (N-1) / Latest tax 59,033
year turnover under NTR (N-2) x Actual turnover for the September
quarter (N-3)) (Rs. 1,012,000 / 15,000,000 x 3,500,000) / 4

Less: tax deducted under section 153 on sale of imported goods 95,000

Balance tax excess paid during the quarter (35,967)

378_________________ ____________ _______ _Conceptual Approach to Taxes


Chapter 21 ___________Solved Past Papers Income Tax Numericals of CA Module C - (2001 to 2015)

NOTES
N-1 Latest assessed tax under NTR
Gross Tax Liability 1,200,000
Less: Tax on export 30,000
Tax on import of goods 108,000
Tax on dividend 50,000 188,000
Net tax 1,012,000
N-2 Latest tax year turnover under NTR
Gross sales 20,000,000
Less: Export sales 3,000,000
Sales of imported goods 2,000,000 5,000,000
Net sales 15,000,000
N-3 Actual turnover for the quarter
Gross sales 5,000,000
Less: Export sales 1,000,000
Sales of imported goods 500,000 1,500,000
3,500,000

Q.NO.4 (a) Spring 2003

Mercury and Co. has provided you the following data:


Fair value of leased asset Rs.225,000
Interest rate 20.50%
10% of fair
Security Deposit paid
value
Depreciation of leased asset 33% per annum
Term of lease 3 years
Yearly rental in arrears Rs.96,890

Required:
You are required to compute the amount available for deduction from the taxable income of Mercury and Co for each
year. Please show proper working.

Solution of Q.NO.4 (a) Spring 2003

Name of Taxpayer : Mercury and Co.


National Tax Number :
Income year ended : 30th June, 2016
Tax Year : 2016
Personal Status : Company

1st Year Rs. Rs.


Business Income -

Add: Inadmissible deductions


Depreciation (Rs. 225,000 x 33%) 74,250
Interest Rs.(225,000 - 22,500) x 20.5% 41,513
115,763
115,763
Less:Admissible deductions
Security deposit as lease rental to claim after the expiry of lease period -
Lease rental 96,890
(96,890)
Taxable income 18,873
In every year Lease rental only is allowable as deduction.

Conceptual Approach to Taxes _______ ___ _______________ __ 379


Chapter 21 _______ Solved Past Papers Income Tax Numericals of CA Module C - (2001 to 2015)

2nd Year
Business Income -
Add: Inadmissible deductions
Depreciation 74,250
Interest (225,000 - 22,500 - 96,890 + 41,513) x 20.5% 30,160 104,410
104,410

Less:Admissible deductions
Lease rental 96,890 96,890
7,520

3rd Year
Business Income -
Add: Inadmissible deductions
Depreciation 74,250
Interest (225,000 - 22,500 - 96,890 - 96,890 + 30,160 + 16,480 90,730
41,513) x 20.5%
90,730
Less:Admissible deductions
Lease rental 96,890 96,890
(6,160)

Q.NO.4 (b) Spring 2003

Sun and Moon have recently registered as partnership. They have incurred the following expenditure.

Fees paid to consultants for preparation of registration deed Rs.50,000


Preparation of feasibility report Rs.100,000
Purchase of office equipment Rs.150,000
Purchase of machinery Rs.1,000,000
Trial run cost Rs.200,000
Installation cost Rs.50,000

Required: You are required to explain the tax treatment by computing the amount allowable as deduction in
accordance with the provisions of Income Tax Ordinance 2001

Solution of Q.NO.4 (b) Spring 2003

Amount allowable as deduction


Business Income -
Less: Pre-commencement expenditure written off (N - 1) 70,000
Initial allowance on fixed Assets (N - 2) 262,500
Depreciation on fixed Assets (N - 3) 140,625
Taxable income 473,125

NOTES
N-1 Precommencement expenditure written off @ 20%

Fees paid to consultant for preparation of registeration deed 50,000


Preparation of feasibility report 100,000
Trial-run cost 200,000
350,000
Written off (350,000 @ 20%) 70,000

N-2 Initial allowance on Fixed Assets @ 25%

380_________________ ____________ _______ _Conceptual Approach to Taxes


Chapter 21 ___________Solved Past Papers Income Tax Numericals of CA Module C - (2001 to 2015)

Machinery 1,000,000
Installation cost 50,000
1,050,000
1,050,000 @ 25% 262,500

N-3 Depreciation on Fixed Asset @ 15%


Office Equipment 150,000
Machinery 1,050,000
Less: initial Allowance (262,500) 787,500
Depreciable amount 937,500
Depreciation on Rs. 937,500 @ 15% 140,625

Q.NO.5. Spring 2003

Mr Amir-ud-din has recently constructed an office complex for the purposes of letting out. The office complex is also
equipped with its own electric generators for which tenants are separately charged on a monthly basis. As per terms and
conditions, Mr Amir-ud-din is also entitled to signing amount, which is nonrefundable.

For the tax year 2003 following information has been provided to you for the computation of his income from property and
tax liability thereon:

Rupees
Rent for the year already received 1,150,000
Rent for the year though due but irrecoverable 50,000
Signing amount (non-adjustable non-refundable) 100,000
Fire and water tax paid to the local authority 20,000
Lawyers fee for suit to recover rent 50,000
Lawyers fee for drafting master rent agreement 10,000
Salary of the caretaker who also collects monthly rent 36,000
Insurance premium being one per cent of market value of the property 200,000
Repair maintenance expenditure 50,000

Solution of Q.NO.5. Spring 2003

Name of Taxpayer : Mr. Amir-ud-din


National Tax Number :
Tax Year : 2016
Personal Status : Individual
Residential Status : Resident

INCOME FROM PROPERTY U/S 15 Rs.

Rent for the year already received 1,150,000


Rent for the year though due but irrecoverable 50,000
Signing amount (non-adjustable / non-refundable) 100,000
Gross property income 1,300,000

Less admissible deductions u/s 15A

1/5the repair allowance as fixed allowance 260,000


Irrecoverable rent (Firstly added and then to deduct for the computation of correct amount
of rent chargeable to tax) 50,000
Fire and water tax paid to the local authority u/s 15A (1)(c) 20,000
Lawyers fee for suit to recover rent 50,000
Salary of caretaker who also collects monthly rent (not exceeding 6% of rent chargeable to tax) 36,000
Insurance premium being one per cent of market value of the property 200,000

Conceptual Approach to Taxes _______ ___ _______________ __ 381


Chapter 21 _______ Solved Past Papers Income Tax Numericals of CA Module C - (2001 to 2015)

Total deductions 616,000


Net income from property 684,000

COMPUTATION OF TAX LIABILITY:

Tax on first Rs. 500,000 7,000


Tax on balance (Rs. 684,000 - 500,000) x 10% 18,400
Balance tax payable 25,400

Q.NO. 7 Spring 2003

Mr. Mushtaq has provided you with the following data for the computation of his total income and tax thereon for the tax
year 2003.

Basic salary 225,000 Bonus 50,000 Conveyance allowance 50,000 House rent allowance 101,250 Leave fare assistance
60,000

Cash paid to a non profit organization by way of donation Rs.20,000. Motor vehicle provided by employer and used partly
for personal and partly for business purpose. Running cost borne by employee Rs.30,000. During the year Mr. Mushtaq
was issued 5,000 shares under an employee share option scheme whereby he was offered shares at 25% discount to the
market value. The market value of shares is Rs.11 per share. House loan taken by Mr. Mushtaq Rs.200,000. Interest paid
on such loan during the year amounted to Rs.6,000.

Required:
You are required to compute his taxable income and tax thereon. Show all computations and assumptions, as
necessary.

Solution of Q.NO. 7 Spring 2003

Name of Taxpayer : Mr. Mushtaq


National Tax Number :
Income year ended : 30th June, 2016
Tax Year : 2016
Personal Status : Individual
Residential Status : Resident

INCOME FROM SALARY U/S 12


Rs. Rs.

Basic salary 225,000


Bonus 50,000
Conveyance allowance (N - 1) 50,000
House rent allowance 101,250
Leave fare assistance 60,000
Benefit from share option (N - 2) 13,750
Taxable Income 500,000

COMPUTATION OF TAX LIABILITY:

Tax on Rs. 500,000 [0 + 2% x (500,000 - 400,000)] 2,000

NOTES
N-1 The value of conveyance facility is not provided so there will be
no treatment. conveyance allowance is fully taxable.
N-2 Benefit from share option
FMV of shares 55,000
Less: Cost of shares (5,000 shares x 11 x 0.75) 41,250 13,750

382_________________ ____________ _______ _Conceptual Approach to Taxes


Chapter 21 ___________Solved Past Papers Income Tax Numericals of CA Module C - (2001 to 2015)

N-3 Donation paid in cash is not entitled for tax credit

N-4 Deductible allowance on markup on house loan has not been claimed as the same has been assumed not fall within
section 64A.

Q. NO.4: Autumn 2002

ABC Associates owns a building which is 30 percent occupied for its business. The rest 70 percent is on rent.

The following information is available:


Rupees
Annuallettingvalueofthepropertyowned 2,000,000
Rentreceivedfromtenants 1,800,000
Depreciationonbuildingunderthe
ThirdScheduletotheOrdinance 400,000
PropertyTax 100,000
Municipal/localgovernmenttaxes(agreementswithtenantsprovidethattenants 100,000
shouldpaythetaxes,costtobeallocatedproportionately)
Generalandadministrationexpenses 200,000

Rent received includes Rs. 600,000 for three years commencing from July 01 of the current year. ABC Associates follow
accrual basis of accounting and its income year is July-June 2002.

Required:
Please compute the income of ABC Associates under the head income from house property.

Solution of Q. NO.4: Autumn 2002

Name of Taxpayer : ABC Associates


National Tax Number :
Income year ended : 30th June, 2016
Tax Year : 2016
Personal Status : Company

INCOME FROM PROPETY U/S 15 Rs. Rs.

Annual letting value of the property owned being as Fair market (A) 2,000,000
rent
Rentreceivedfromtenants 1,800,000
Less advance for next two years (Rs. 600,000 / 3 x 2) (400,000)
Add owners burden paid by tenant in the form of taxes 100,000
Actual rent for the tax year (B) 1,500,000

Higher of actual rent or Fair market rent i.e. higher of (B) & (A) 2,000,000

Less admissible deductions U/S 15A

1/5thofrentchargeabletotaxasrepairallowance 400,000
Taxdepreciation(notanallowableexpense) -
PropertyTax 100,000
Muncipal/localGovt.taxesagainstproperty(70%relatedtorentedportion) 70,000
570,000
Net taxable income from property 1,430,000

Note - 1 It has been assumed that the data given in the question is only related to the rented out portion, unless where
specified.

Conceptual Approach to Taxes _______ ___ _______________ __ 383


Chapter 21 _______ Solved Past Papers Income Tax Numericals of CA Module C - (2001 to 2015)

Note - 2 General and administration expenses are not admissible against property income under section 15A of the
Income Tax Ordinance, 2001.
Note - 3 Taxes paid on behalf of owner by tenant shall be treated as income in the hands of the owner by virtue of section
69 of the Income Tax Ordinance, 2001.

Q. NO.7(b): Autumn 2002

Unique Ltd. has following salary related data for the period July 1, 2001 to June 30, 2002 of its three employees.

S G O
Rupees
(Salary and allowances per month)
Basic salary 37,500 23,000 6,000
House rent allowance 16,875 10,350 2,700
Conveyance allowance - 2,300 300
Utility allowance 4,000 2,500 1,000
Recovery of Provident Fund Loan 2,000 1,500 500
Additional information is as follows:

(i) S is provided a fully maintained car of 1000cc which is used both for private and business purpose.
(ii) G owns his conveyance and also incurs its running and maintenance cost. The conveyance is used partly for
business and private purposes.
(iii) S, G and O all were entitled for annual bonus due in Sept. 2001, the term of bonus is one basic pay.

On the basis of above, compute tax withholding per month under the CIR read with the Income Tax Rules, 1982.

Solution of Q. NO.7(b): Autumn 2002:

Name of Taxpayer : S, G & O


National Tax Number :
Income year ended : 30th June, 2016
Tax Year : 2016
Personal Status : Individual
Residential Status : Resident

INCOME FROM SALARY U/S 12 Rs. Rs. Rs.


S G O
Basic salary 450,000 276,000 72,000
Bonus 37,500 23,000 6,000
House rent allowance 202,500 124,200 32,400
Conveyance allowance - 27,600 3,600
Utility allowance 48,000 30,000 12,000
Recovery of Provident Fund Loan 24,000 18,000 6,000
Conveyance facility (Assumed vehicle cost Rs.1,000,000 x 5%) 50,000 - -

Taxable income 812,000 498,800 132,000

Computation of tax liability (lower of A or B ):


under normal case:
Tax liability:
(A)
S [14,500 + 10% x (812,000 - 750,000)] 20,700
G [0 + 2% x (498,800 - 400,000)] 1,976
O (not taxable) 0

20,700 1,976 -

384_________________ ____________ _______ _Conceptual Approach to Taxes


Chapter 21 ___________Solved Past Papers Income Tax Numericals of CA Module C - (2001 to 2015)

Tax per month to be deducted (Annual tax / 12 months) 1,725 165

Q. NO.4: Spring 2002:

Mr. Ashraf made the following donations during the income year 2000-2001:
(a) Rs. 200,000 in cash to a relief fund sponsored by the Government.
(b) Personal car to an institution referred to in Clause (61) of the Second Schedule. This car was purchased by
Mr. Ashraf four years ago at the cost of Rs. 80,000.
(c) The fair market value is Rs.60,000

Medicines to a private hospital purchased at the total cost of Rs. 10,000.

Please advice Mr. Ashraf regarding the allowance for donation which may be claimed by him keeping in view the
requirement of Section 47 of the CIR 1979 if his income for the relevant income year has been assessed at Rs. 800,000.

Solution of Q. NO.4 Spring 2002:

Name of Taxpayer : Mr. Ashraf


National Tax Number :
Income year ended : 30th June, 2016
Tax Year : 2016
Personal Status : Individual
Residential Status : Resident
Rs. Rs.
Income assessed 800,000

(a) Less direct deduction is allowed of donation amounting Rs. (200,000)


200,000 in cash to a relief fund sponsored by the Government.

(b) Less direct deduction is allowed for donation of persoal car to


an institution referred to in Clause (61) of the Second Schedule.

Value of deduction will be cost less depreciation of 3 years


Cost of car 80,000
Less: depreciation 3 years straight line method (Rs. 80,000 x 10% 24,000 (56,000)
x 3)

c) Neither deduction nor tax credit is allowed for the donation of -


medicines to private hospital
Taxable income 544,000

Q. NO.5: Spring 2002

Explain whether the following are admissible as business expenditure under the ITO 1979:
(a) Repayment of principal amount of lease rentals of plant and machinery. (b)Sales tax paid on the purchase of raw
material to be used in the production of exempt supply. (c) Dividend (d) Provision in respect of doubtful debts.

(b) Penalty levied u/s 108 of the CIR, 1979 for failure to file statement u/s 139.

Solution of Q. NO.5: Spring 2002:

a) Repayment of principal amount of lease rentals: Lease rentals are admissible where leased asset is acquired for
business purposes. In this case the repayment is principal amount with markup although not given in question.
b) Sales tax paid on the purchase of raw material to be used in the production of exempt supply shall be allowed as
admissible expense.

Conceptual Approach to Taxes _______ ___ _______________ __ 385


Chapter 21 _______ Solved Past Papers Income Tax Numericals of CA Module C - (2001 to 2015)

c) Dividend is not an allowable deduction while calculating the business income because it is profit and loss approprriation
item.

d) Provision in respect of doubtful debt is not allowed. Only actual bad debts are allowed.

e)PenaltyleviedunderSection108oftheIncomeTaxOrdinance,1979forfailuretofilestatementunderSection139
shall not be admissible under the income tax ordinance.

Q. NO.6: Spring 2002

Mr. Javaid, Managing Director of a multi national Company has submitted the following data for the income year ending 30
Rs.
Basic Salary 130,500 p.m
Bonus 325,500 full year
House rent allowance 43,500 p.m
Utilities 13,050 p.m

- Mr. Javaid has been provided with free use of a Company maintained car of 1,600 C.C.
- In accordance with terms of his employment Mr. Javaid was paid Rs. 60,000 being the cost of air ticket in
connection with a foreign tour. He last undertook a foreign tour three years ago.
- During the year Mr. Javaid sold 180,000 shares of Rs. 10 each purchased at par three years ago of
M/s Azmat (Pvt) Ltd for Rs. 65 per share.
- Zakat paid Rs. 12,000

Required:
You are required to calculate the total income of Mr. Javaid and tax payable thereon.

Solution of Q. NO.6: Spring 2002

Name of Taxpayer : Mr. Javed


National Tax Number :
Income year ended : 30th June, 2016
Tax Year : 2016
Personal Status : Individual
Residential Status : Resident

INCOME FROM SALARY U/S 12 Rs. Rs.

Basic Salary (Rs. 130,500 p.m) 1,566,000


Bonus 325,500
House rent allowance (43,500 p.m) 522,000
Utilities (13,050 p.m) 156,600
Free use of a Company maintained car of 1,600cc (N-1) -
Cost of air ticket in connection with a foreign tour (N-2) -
Total salary income 2,570,100

CAPITAL GAINS U/S 37

Consideration received (180,000 shares x 65) 11,700,000


Less: cost of shares (180,000 shares x 10) 1,800,000
Capital Gain 9,900,000
Taxable Capital Gian (9,900,000 x 75%) 7,425,000
9,995,100
Less: Zakat paid (12,000)
Total Taxable Income 9,983,100

386_________________ ____________ _______ _Conceptual Approach to Taxes


Chapter 21 ___________Solved Past Papers Income Tax Numericals of CA Module C - (2001 to 2015)

COMPUTATION OF TAX LIABILITY:

Tax on Rs. 9,983,100 [1,319,500 + 35% x (9,983,100 - 6,000,000)] 2,713,585

Notes
N-1: The value of car not provided and further it is assumed that the car was being for office use only.
N-2: Air ticket for foreign trip treated as exempt as it is for the discharge of official duties.

Q. NO.7 Autumn 2001

Mr Amir Ali is manager finance in a multinational company. He has received the following salary and other perquisites
during the year ended on June 30, 2000:

Basic salary Rs. 35,000 p.m.


Bonus 180,000
House allowances 18,000 p.m
Utilities allowance 50,000 p.a.

The employer provided him a 1300 c.c. car for office/personal use and medical facility worth Rs.25,000 during the
year.

Compute the total income of Mr Amir Ali and tax payable thereon.

Solution of Q. NO.7 Autumn 2001:

Name of Taxpayer : Mr. Amir Ali


National Tax Number :
Income year ended : 30th June, 2016
Tax Year : 2016
Personal Status : Individual
Residential Status : Resident

INCOME FROM SALARY U/S 12 Rs. Rs.

Basic salary (35,000 p.m.) 420,000


Bonus 180,000
House allowances (18,000 p.m) 216,000
Utilities allowance (50,000 p.a) 50,000
1300 cc car for office / personal use ( Value assumed Rs.1,000,000 hence 5% of the 50,000
said cost shall be included in salary income) -
Medical facility ( Assumed it is in accordance with terms of employment.)
Taxable salary income 916,000

COMPUTATION OF TAX LIABILITY:


Tax on Rs. 916,000 [14,500 + 10% x (916,000 - 750,000)] 31,100

Q. NO.8 Autumn 2001

T and H Enterprises is a registered firm comprising of two equal partners named Tariq and Hamid. During the year ended
on 30th June 2001 the partners besides their shares in the firm enjoyed income and sustained losses from the sources
given below:

Tariq

(a) Income accrued abroad but not remitted to Pakistan. 72,000

Conceptual Approach to Taxes _______ ___ _______________ __ 387


Chapter 21 _______ Solved Past Papers Income Tax Numericals of CA Module C - (2001 to 2015)

(b) Shares of loss from an association of persons 5,000


(c) Zakat paid 26,500

Hamid
(a) Speculation loss 25,000
(b) Profit on sale of car 13,000
(c) Income tax refund 5,000
(d) Zakat paid 14,000

The profit and loss account of the registered firm for the year ended on 30th June, 2001, shows the following
position:

Rs. Rs.
Salaries 300,000 Gross profit b/d 480,000
Office maintenance 5,000 Dividend from
Public Co. 250,000
Repairs 38,000
Provision for bad debts 14,000
Super tax paid for last year 5,000
Legal expenses 15,000
Commission to Tariq 16,000
Premium of life policies of
Partners 5,000
Depreciation 34,000
Net profit:
Tariq 149,000
Hamid 149,000 298,000
730,000 730,000

Notes:
(i) Tariq and Hamid are paid Rs.45,000 and Rs.55,000 respectively as salary. This is included in total salary
expense.
ii) Repairs includes Rs.18,000 being cost of a typewriter to be depreciated by 10%.
(iii) Legal expenses include Rs.6,000, on which tax is deductible.
(iv) Tax Depreciation excluding typewriter Rs.14,000.

Compute:
(a) the total income of the firm and taxes payable by it. (b) the total income of each partner and tax thereon.

Solution of Q. NO.8 Autumn 2001:

Name of Taxpayer : T and H Company


National Tax Number :
Income year ended : 30th June, 2016
Tax Year : 2016
Personal Status : Company

INCOME FROM BUSINESS U/S 18 Rs. Rs.

Net profit as per profit and loss Account 298,000


Add: Inadmissible deductions
Salaries to partners (45,000+55,000) 100,000
Cost of typewritter 18,000
Provision for bad debts 14,000
Super tax paid for last year 5,000
Commission to Tariq 16,000
Premium on life policy of partners 5,000

388_________________ ____________ _______ _Conceptual Approach to Taxes


Chapter 21 ___________Solved Past Papers Income Tax Numericals of CA Module C - (2001 to 2015)

Accounting depreciation 34,000 192,000


490,000
Less: Admissible expenses
Dividend received (N - 1) 250,000
Tax depreciation (N - 2) 15,800 (265,800)
224,200
COMPUTATION OF TAX LIABILITY:
(a) Tax on taxable income of Rs.224,200 @ 0% -
Divisible income after tax 224,200

Partners share of profit from AOP T H Total


Salary 45,000 55,000 100,000
Commission 16,000 - 16,000
Life insurance premium 2,500 2,500 5,000
Share of profit on Equal proportion 51,600 51,600 103,200
115,100 109,100 224,200

Tariq
Total income from foreign source 72,000 -
Less: Zakat (26,500) -
Taxable income 45,500 -
Add: Share of profit from AOP 115,100 -
Taxable income for rate purposes 160,600 -

Computation of tax liability:


Income is below the taxable limit, so no tax shall be charged.

Hamid
He has a speculation loss which is set off only against speculation profit. So nothing shall be included from share
from AOP.

NOTES
N-1 Dividend covered under FTR, so not included in taxable income.

N-2 Tax depreciation excluding depreciation of typewritter 14,000


Add: depreciation on typewritter @ 10% 1,800
15,800

Conceptual Approach to Taxes _______ ___ _______________ __ 389


Final Tax Regime and Minimum Tax Chapter-22

Chapter

22 FINAL TAX REGIME & MINIMUM TAX

Topic covered
Section
Section Rule
For CA Mod F & ICMAP students

General provisions regarding income Minimum tax on services & goods


169 153
under final tax regime transport services
148,150,151 Imports, dividend & profit on debt 235 Advance tax on electricity bills
Royalty or fee for technical services
152
received by non-residents
Advance tax on sale or transfer of
153 Sale of goods, services & contracts 236C
immovable property
154 Export & export indenting commission 6 18 Income from royalty
156 & 156A Prizes & winnings & petroleum products 6 19 Fee for technical services
Tax on income of non-resident shipping &
7, 143 37 &
233 Local indenting commission air transport person, ship owner & air craft
& 144 38
owner
234 & 234A Goods transport vehicle & CNG stations MCQs with solutions
113, 113A,
Minimum tax on income of certain
113B & Practice question for CA Mod F & ICMAP
persons
113C
Tax on import of packing material & ICMAP & CA Mod C past papers
148
edible oil theoretical questions

For CA Mod F and ICMAP students


1. General provisions regarding income under Final Tax Regime (Section 169):
Where an income is chargeable to tax under final tax regime:

a. The income shall not be chargeable to tax under any other head of income in computing the taxable income of
the person.

b. No deduction shall be allowable for an expenditure incurred in deriving the income or any deductible
allowances.

c. The amount of income shall not be reduced by any tax credits or set off of any losses (except stated
otherwise).

d. In case where a person has no income other than final tax regime and files a statement for final tax regime an
assessment order shall be deemed as made u/s 120 on the date of filing such statement.

e. Return of income is not required to be filed where the only source of income is final tax regime. However,
statement for final tax regime u/s 115(4) is required to be filed on or before the due date for filing of return of
income.

f. There shall be no refund of the tax collected or deducted except where the tax so collected or deducted is in
excess of the amount for which the taxpayer is chargeable under the ordinance.

g. If tax deductible has not been deducted, or short deducted, the said non-deduction or short deduction may be
recovered under section 162, and all the provisions of this Ordinance shall apply accordingly.

Conceptual Approach to Taxes 383


Final Tax Regime and Minimum Tax Chapter-22

2. Following incomes are chargeable to tax under final tax regime:

Sr. Nature of payment Section Rate


1. Commercial imports 148(7) & (8A) General rate 5.5% (6% for individual and
AOPs)
2. Dividend 5 and 150 General rate 10%
3. Profit on debt 151(3) 10%
4. Payments to non-residents 6 and 152(1B) and (1BB) Detail given as under
5. Sale of goods, rendering of 153(3)(a) and (c) Detail given as under
services and contracts
6. Exporters 154(4) Detail given as under
7. Prizes and winnings 156(3) 15% & 20%
8. Petroleum products 156A(1) and (2) 12%
9. Brokerage and commission 233(3) 12%
10. Goods transport vehicles 234(5) Rs.2 per Kg of laden weight
11. CNG stations 234A 4% of gas consumption charges

3. Commercial Imports [Section 148(7) & (8A)]: (Covering NTR / FTR / MTL)

1. Tax rate The Collector of Customs shall collect advance tax from every importer of goods
(other than the goods or persons specified by the Board) on the value of the
goods at:
5.5% for companies & industrial undertakings.
6% for AOPs and individuals
12% of the value of film in case of foreign produced film imported for the
purpose of screening & viewing.
4.5% tax collected at the time of import of ships by ship breakers.

2. Reduced tax rate Although the general rate of tax is 5.5%, however, the Board has specified reduced
and more than general rate of 5.5% in Division II of Part V of Chapter X of first
Schedule as rates of advance tax.

3. Value of goods Means value as increased by sales tax, federal excise duty and custom duty.

4. Exceptions from Final tax Tax required to be collected on import of goods is final tax except the following:
Regime
(a) raw material, plant, machinery, equipment and parts by an industrial
undertaking for its own use;
(b) fertilizer by manufacturer of fertilizer; and
(c) motor vehicles in Completely Built Unit (CBU) condition by manufacturer of
motor vehicles.
(d) large import houses. (Definition given in payment chapter u/s 148)
(e) a foreign produced film imported for the purposes of screening and viewing
(e) The tax required to be collected from a person on the import of edible oil and
packing material for a tax year shall be minimum tax.
(f) 12% tax collected on the value of film in case of foreign produced film imported
for the purpose of screening & viewing is adjustable

5. Minimum tax liability The tax required to be collected from a person on the import of edible oil and
packing material for a tax year shall be minimum tax.

6. Opting out from Final Commercial importers paying taxes under the Final Tax Regime of the Ordinance
Tax Regime may opt for Normal tax regime [NTR], provided the tax liability under NTR does not
fall below 100% of the tax collected on imports.
[Clause 56B of Part IV of the Second Schedule]

384 Conceptual Approach to Taxes


Final Tax Regime and Minimum Tax Chapter-22

Example:
Following information relates to Mr. Kamran for tax year 2016. He imports garments goods from America and sells
goods in the same condition in which they were imported.
Rs.
Sales 500,000
Expenses 350,000
Net Profit 150,000
Note: Expenses include imports of Rs. 300,000. For certain reasons, tax on half of the imports were not collected by
collector of custom duty.
Solution:
In this case tax required to be collected shall be treated as final tax liability of the taxpayer.
Tax required to be collected (300,000 x 6%) 18,000
Tax collected at import stage (8,250)
Balance tax payable 9,750
4. Dividends (Section 5 and 150):
Tax shall be deducted from gross dividend which shall be considered full and final tax for a person. This section is
not applicable in case where dividend is exempt from tax e.g. dividend received from agricultural business u/c 105B
of Part I of second schedule.

Particulars Rate of tax


nd
1. Dividend income is available to companys specified u/c 17 & 20 of part II of 2 schedule 7.5%
to the ITO, 2001.
2. Facility of reduced tax rate has also been extended to companies supplying coal
exclusively to power generation projects.
3. Normal rate shall be applicable on dividend income other than specified herein. 10%
4. In case of stock fund if dividend receipts are less than capital gains. 12.5%
5. Dividend received by a company from a collective investment scheme or a mutual fund, 25%
other than a stock fund,
6.
Example: Which of the following incomes are chargeable to tax under Final Tax Regime or Normal Tax Regime?
(a) Dividend received by individual
(b) Dividend received by company
Solution:
(a) Final tax regime
(b) Final tax regime
5. Profit on debt [Section 151(3)]
The following are the various profits on debts that are chargeable to tax under this section where:
(a) Deposit or a certificate under the National Saving Scheme or Post Office Savings Account.
(b) Profit is being on a saving account or Profit and loss sharing account or deposit maintained with the banking
Company or financial institution.
(c) Profit on any bond, certificate, debenture etc. including term finance certificates, certificates of investment
issued by a banking company or a financial institution, company as defined in the Companies Ordinance,
1984, body corporate or a finance society.
(d) Tax shall also be deducted from profit on securities other than (a) above issued by Federal Government or
Local Government.
to any person other than a financial institution.
Zakat (where applicable under Zakat and Ushar Ordinance) shall be deducted from the amount of profit on debt on
payment to the recipient.
10% tax deductible on interest income from all of the above shall be final tax (whether tax has been deducted or not)
under Separate Block of Income except the following:
1. A company
2. Profit on debt in respect of a loan agreement between a borrower and a banking company or a development
finance institution.

Conceptual Approach to Taxes 385


Final Tax Regime and Minimum Tax Chapter-22

3. As stated in (d) above.


4. Where profit is paid by any person other than banking company, a financial institution, or a company or a
finance society on any bond, certificate, or under any instrument in writing.
5. The rate for filers is 10% and for non filers it is 15% provided the Profit on debt is in excess of Rs.500,000.
6. For a non filer, other than a company, the 10% withholding tax will be final tax and any excess will be
treated as advance tax adjustable against his tax liability.
Example: Which of the following incomes are chargeable to tax under Final Tax Regime or Normal Tax Regime?
(a) Profit on Profit and loss sharing account (PLS) account received by individual and tax deducted @ 10%
(b) Profit on regular income certificates received by an AOP
(c) Profit on PLS account received by company
(d) Profit on PLS account received by individual, however, tax was not deducted
Solution:
Case a, b, d fall under Final tax regime, c. Normal tax regime
6. Payments to non-residents [Section 6 and 152]:
Status
Deduction/
payment
Section Nature of payment/transaction Tax rate Exemption collecting
for non
authority
resident
Royalty & fee for technical 15% of
Final
152(1) services: gross Nil Any person
discharge
Income from above in Pakistan amount
Contracts:
(a) a contract or sub-contract
under a construction, assembly
or installation project in
Pakistan, including a contract
for the supply of supervisory
6% of the
activities in relation to such Final
152(1A)
project; or
gross Nil Any person
discharge
amount
(b) any other contract for
construction or services
rendered relating thereto; or
(c) a contract for advertisement
services rendered by T.V.
Satellite Channels,
Insurance Premium or Re-
insurance premium:
Insurance premium or re-insurance
premium paid to non-resident person
5% of the
This shall not apply to an amount, Final
152(1AA)
with the written approval of the
gross Nil Any person
discharge
amount
Commissioner Inland Revenue that
is taxable to a permanent
establishment [PE] in Pakistan of
the non-resident person
Advertisement services: 10% of the
Final
152(1AAA) Payment for advertisement services gross Nil Any person
discharge
to a non-resident person amount
Final
152(2A)(i) Payment for sales of goods 3.5% Nil Any person
discharge
- For
transport
services 2% Final
152(2A)(ii) Payment for rendering of services Nil Any person
discharge
- For other
services 6%
Final
152(2A)(iii) Payment on execution of contract 6% Nil Any person
discharge

386 Conceptual Approach to Taxes


Final Tax Regime and Minimum Tax Chapter-22

2. Sub-section (1AA) shall not apply to an amount, with the written approval of the Commissioner Inland
Revenue that is taxable to a PE in Pakistan of the non-resident person.
3. The tax deductible as above shall be a final tax on the income of a non-resident person except the following:
Royalty and fee for technical services connected or through Permanent Establishment (PE) of non-
resident in Pakistan.
Exempt royalty and fee for technical services.
In respect of non-resident contractors u/c 41 of Part IV of the second schedule unless he opts for Final
Tax Regime for 3 years.
4. In this section prescribed person means a prescribed person as defined in sub-section (7) of section 153.
7. Sale of goods, rendering of services and contracts [Section 153(a) and (c)]:
Every prescribed person making a payment to a resident person shall deduct tax at the time of payment (including
sales tax) as per following schedule:

Payment for: Rule for tax deduction


Sale of goods 1. Tax shall be deducted on gross amount payable including sales tax.
2. In case of sale of rice, cotton seed or edible oils, rate of tax is 1.5%.
3. In case of sale of any other goods, rate of tax is 4.5% (for industrial undertakings
and Companies it is 4%).
4. Reduced rates under second schedule where applicable.

Rendering of services 1. Tax shall be deducted on gross amount payable.


2. In case of transport services, rate of tax is 2%.
3. In any other case, rate of tax is 10% (For companies it is 8%).
4. Where payment is made by an exporter or an export house to a resident person or
PE in Pakistan of a non-resident person for services of stitching, dying, printing,
embroidery, washing, sizing and weaving, rate of tax deduction is 1% - section
153(2).

Execution of contract 1. Tax shall be deducted on gross amount payable.


2. Rate of tax is 7.5%. For companies rate of tax is 7%.
3. Sportspersons contracts 10%.

Tax deductible on sale of good and execution of contract shall be final tax and tax deductible on rendering of services
shall be minimum tax except in the following cases:
- Tax deductible on sale of goods shall not be final tax if sale is made by a company being manufacturer or
public listed company.
- Tax deductible on execution of contracts shall not be final tax if payments are received by public listed
company.
Opting out from Final Tax Regime [Clause (56C), (56D) & (56E) of Part IV of the Second Schedule]
In case of supply of goods, contracts & services of stitching, dying & printing etc. provided to exporter & export
house, the said persons paying taxes under the Final Tax Regime of the Ordinance may opt for Normal tax regime
[NTR], provided the tax liability under NTR does not fall below 100% of the taxes deducted under respective sub
sections.
Example:
Following information relates of Mr. A. He is doing the business of manufacturing of garments for tax year 2015.
Rs.
Sales 6,500,000
Expenses 6,050,000
Net Profit 450,000
Note: All the sales amount has been received after tax deduction @ 4% being as industrial undertaking.

Conceptual Approach to Taxes 387


Final Tax Regime and Minimum Tax Chapter-22

Solution:
Computation of tax liability: Rs.
Tax deducted u/s 153 (6,500,000 x 4%) 260,000
Total tax liability 260,000
Less: tax deducted at source (u/s 153) 260,000
Balance tax - -
Example:
Following information relates of Mr. B. He is doing the business of manufacturing of garments for tax year 2015.
Rs.
Sales 6,500,000
Expenses 6,050,000
Net Profit 450,000
Note: There was no tax deduction under section 153.
Solution:
Computation of tax liability: Rs.
Tax deductible u/s 153 (6,500,000 x 4%) 260,000
Total tax liability 260,000
Less: tax deducted at source - _
Balance tax payable 260,000
8. Exports [Section 154(4)]:
1. Tax deductible under this section shall be final tax as per the following rates:
Nature of Status Deduction/collecting
Section
payment/transaction
Tax rate Exemption payment authority
(a) Direct export:

154(1) - Export proceeds 1% of export Nil Final Authorized dealer in


realization proceed received discharge foreign exchange
154(3A) - Export of the goods 1% at the time of
by an industrial export of goods Nil Final Export processing
undertaking without realization discharge Zones Authority
located in EPZ
154(3C) 1% at the time of Nil
- Clearing of the export of goods
goods exported Final
without realization discharge Collector of customs
154(3) (b) Indirect export: 1% of export Nil Final Banking Company
proceeds received discharge
- Realization of
proceeds of goods
sold to an exporter
under an inland
back -to-back letter
of credit or
standard purchase
order
154(3B) - Payment for a firm 1% of the export Nil Final Direct export and
contract to an proceeds realized discharge export house
indirect export registered under DTRE
Rules 2001
154(2) (c) Foreign indenting 5% of commission Nil Final Authorized dealer in
commission: received discharge foreign exchange
Realization of commission
due to indenting agent
2. Income from export of computer software, IT services or IT enabled services is exempt up to 30.06.2016.
3. Opting out from Final Tax Regime [Clause 41AA of Part IV of the Second Schedule]
Due to omission of the aforesaid clause the option to exporter or an export indenting agent paying taxes under the
Final Tax to opt for Normal tax regime [NTR] is no more available.

388 Conceptual Approach to Taxes


Final Tax Regime and Minimum Tax Chapter-22

Example: Compute taxable income and tax liability of Mr. Jamil for the Tax year 2015 from following data:
Local sales 600,000
Exports sales (total amount realized) 400,000
Cost of local sales 360,000
Cost of export sales 300,000
Other expenses related to local sales 100,000
Other expenses related to export sales 40,000
Solution:
Computation of taxable income Rs.

Local sales 600,000


Cost of local sales 360,000
Gross profit on local sales 240,000
Other expenses related to local sales 100,000
Taxable profit under NTR 140,000
Computation of tax liability:
Tax on taxable income under NTR (140,000 x 0%) -
Tax on export proceeds realized under FTR (400,000 x 1%) 4,000
Total tax liability 4,000
Note: Expenses related to income under FTR shall not be considered for computation of profit under NTR.
9. Prizes and winnings [Section 156(3)]:
Tax shall be deducted by every person on payment of prizes and winnings as per following schedule:

Gross amount of Rate of tax


Prize on prize bond and cross-word puzzle 15%
Any other prize (Raffle, Lottery, Prize on winning a quiz or Prize offered by companies for
20%
promotion of sales)
Where a prize or winning, is not in cash, the person while giving the prize shall collect tax on the fair market value of
the prize.
Tax deductible under this section shall be final tax.
Example: A person received prize of Rs. 5,000 (gross) on winning a cross word puzzle. Compute the amount of tax
liability on such income for the tax year 2015.
Solution:
Tax on prize on winning a cross word puzzle (Rs. 5,000 x 15%) Rs. 750
Note: It is to be noted that tax liability shall be equal to amount tax required to be deducted, whether tax has been
deducted or not.
10. Petroleum products [Section 156A(1) and (2)]:
Every person selling petroleum products to a petrol pump operators shall deduct tax from the amount of commission
or discount allowed to the operator at the rate of 12%.Amount deductible under this section shall be treated as final
tax.
Opting out from Final Tax Regime [Clause (56F) of Part IV of the Second Schedule]
In case of petroleum products, the said person paying tax under the Final Tax Regime of the Ordinance may opt for
Normal tax regime [NTR], provided the tax liability under NTR does not fall below 100% of the tax deducted under this
section.
Example: Ali is operating a petrol pump. Rs. 12,000 Tax deducted @ 12% on commission on sale of petrol. What will
be his tax liability?
Solution: In this case tax deductible on commission on sale of petrol is treated as full and final discharge of his tax
liability and no other tax is payable by him, however option for equal amount of tax is available under normal tax
regime.
11. Brokerage or commission [Section 233(3)]:
12% tax shall be deducted by the principal (being as Federal / Provincial / local Govt., a company or an AOP
constituted under law) at the time of payment of commission to agent. If the agent retains Commission or brokerage
from any amount remitted by him to the principal, he shall be deemed to have been paid the commission or
brokerage by the principal and the principal shall collect advance tax @ 12% from the agent.

Conceptual Approach to Taxes 389


Final Tax Regime and Minimum Tax Chapter-22

Tax rate is 7.5% for advertising agents, u/c 26 of Part II of second schedule.
Where any tax is required to be collected from a person as above, such tax shall be the final tax on the income of
such person.
Opting out from Final Tax Regime [Clause (56G) of Part IV of the Second Schedule]
In case of brokerage or commission, the said person paying tax under the Final Tax Regime of the Ordinance may
opt for Normal tax regime [NTR], provided the tax liability under NTR does not fall below 100% of the tax deducted
under this section.
Example: Mr. Ahsan earned commission on local business of Rs. 500,000 (gross) during the tax year and incurred
expenses of Rs. 100,000 in deriving this income. Compute his tax liability if tax was deductible on commission income
@ 12% but was not deducted.
Solution:
(FTR)
Tax liability: Tax required to be deducted on commission (500,000 x 12%) 60,000
In this case tax deductible on commission on sale of petrol is treated as full and final discharge of his tax liability and
no other tax is payable by him, however option for equal amount of tax is available under normal tax regime.
12. Goods Transport Vehicles [Section 234(5)] Any person at the time of collecting motor vehicle tax shall also collect
advance tax on it, if motor vehicle tax is collected in instalments the advance tax shall also be collected in
instalments.
Rs. 2 per Kg per annum shall be collected on registered laden weight from the owner of goods transport vehicle.
However where the registered laden weight of vehicle is less than 8,120 Kg (equal or more than 8,120 Kg @
Rs.1,200 per annum), advance tax shall not be collected after a period of 10 years from the date of first registration of
vehicle in Pakistan.
Where the tax so collected from any person from plying, or hiring out, of such vehicle being owner of goods transport
vehicle, the tax so collected shall be the final tax on the income of such person.

Example: Calculate the amount of tax on goods transport vehicle (for hiring) in tax year 2015, if the laden weight of
vehicle is as follows:
- Vehicle registered first time in Pakistan in 2013 with laden weight of 9,000 Kg.
- Vehicle registered first time in Pakistan in 2003 with laden weight of 8,200 Kg.
- Vehicle registered first time in Pakistan in 2003 with laden weight of 8,000 Kg.

Solution:
1. The tax shall be Rs. 18,000 i.e. Re. 2 per Kg.
2. The tax shall be Rs. 1,200 as the goods transport vehicle is more than 10 years old.
3. There is no tax liability as the vehicle after first registration is more than 10 years old.
13. CNG stations (Section 234A): The person preparing the gas consumption bill shall charge tax @ 4% of the amount
of gas consumption charges and such tax shall constitute final tax and further the taxpayer shall not be entitled to
claim any adjustment of withholding tax under any other head.
Example: Ali is operating a CNG station. Tax collected on gas consumption charges is Rs. 4,000. What will be his tax
liability?
Solution: The tax deducted on gas consumption charges is treated as full and final discharge of his tax liability.

390 Conceptual Approach to Taxes


Final Tax Regime and Minimum Tax Chapter-22

14. MINIMUM TAX


Sr. Nature of payment Section Rate

1. Minimum tax for 113 General rate 1% of turnover


companies (except
otherwise specified)
2. Minimum tax for 113 General rate 1% of turnover
individuals & AOPs
having annual turnover
Rs.50(M) or more in
respective tax years
3. 113 nd
Reduced rate of tax u/s Sr. Persons u/c Part III of 2 u/c Minimum tax as % of
113 are specified in Part Schedule the persons turnover
IX in the First schedule for the year
to the Ordinance. (1) (2) (3)
1. (a) Distributors of 7&8 0.2%
pharmaceutical products,
fertilizers, consumer goods
including fast moving
consumer goods and
cigarettes
(b) Flour Mills 10

(c) Petroleum agents and


distributors who are registered 13
under the STA, 1990; and
(d) Rice mills and dealers 13

2. Motor cycle dealers registered 15 0.25%


under the STA, 1990
3. (a) Oil marketing companies, 9 0.5%
Oil refineries, Sui Southern
Gas Company Ltd. And
SNGPL (for cases whose
annual turnover exceeds Rs.
1 billion).
12
(b) Pakistan International
Airlines Corporation; and
(c) Poultry industry including
breeding, boiler production,
egg production and poultry
feed production. 14
4. All other cases 1%
4. Minimum tax on builders 113A To be notify later by the Federal Govt.
5. Minimum tax on 113B 2% of the value of land notified by any authority for the purpose of
developers stamp duty
6. Alternative Corporate 113C Detail given in the explanation of this section
Tax
7. Imports of edible and 148(8) 5%
packing material
8. Services 153(1)(b) 10% (other than companies)
9. Goods transport services 153(1)(b) 2%
10. Electricity bills (other than 235(4)(a) up to bill Rs. 30,000
companies)

Conceptual Approach to Taxes 391


Final Tax Regime and Minimum Tax Chapter-22

Minimum tax on income of certain persons (section 113):

1. To whom (a) A resident company


applicable (b) An AOP having turnover of Rs.50 million or above in the tax year 2007 or in any
subsequent tax year, or
(c) An individual having turnover of Rs. 50 million or above in the tax year 2009 or in any
subsequent tax year.

2. When applicable If tax payable (other than tax payable or paid as final tax) is zero or less than minimum tax
on turnover because of:
(a) Loss for the current of earlier year,
(b) Exemption from tax, or
(c) Rebates of credits

3. When not This section shall not apply to a company which has declared gross loss before set off of
applicable depreciation and other inadmissible expenses. However if the loss is arrived at by setting of
depreciation and other inadmissible expenses of by changing accounting pattern, the
Commissioner Inland Revenue may ignore such claim and proceed to compute the tax as
per historical accounting pattern.

4. Rate of tax 1% or reduced tax rates as the case may be of the turnover.

5. Definition of Turnover means:


turnover (a) the gross sales or gross receipts, exclusive of Sales Tax and Federal Excise duty or any
trade discounts shown on invoices, or bills, derived from the sale of goods, and also
excluding any amount taken as deemed income and is assessed as final discharge of
the tax liability for which tax is already paid or payable;
(b) Gross fee for rendering of services, Gross receipts from contracts and commission
excluding chargeable to tax under final tax regime.
(c) If the company is member of an AOP then share of company in above amounts of the
AOP. (See Example A hereunder)

6. Facility of carry Minimum tax in excess of actual tax liability shall be carried forward and adjusted against the
forward tax liability of subsequent 5 tax years. Facility of carry forward has also been extended to
individuals and AOPs from the tax year 2014. (See Example B hereunder)

Example A: ABC Limited is member of an AOP with 50% share. Sale of AOP for the year is Rs. 100,000 while sale
of company for the year is Rs. 50,000 including sales tax of Rs.10,000. What would be the turnover of the company
for the purpose of section 113?
Solution:
Rs.
Sales of the company excluding sales tax 40,000
Add: Share in sales of AOP 50,000
Turnover of company for the purpose of section 113 90,000

Example B: Following data relates to ABC (Pvt.) Ltd:

Year Actual tax liability Minimum tax


At 1%
2014 10,000 (NTR) 50,000
2015 60,000 (NTR) 55,000
Solution:

Year 2014 Rs.


Tax payable 50,000
(Higher of actual or minimum tax)

Year 2015
Tax liability
(Higher of actual or minimum tax) 60,000
Less: b/f balance from minimum tax of tax year 2014 for adjustment (5,000)
* 55,000

392 Conceptual Approach to Taxes


Final Tax Regime and Minimum Tax Chapter-22

* Tax liability cannot be less than the minimum tax for the year i.e. Rs. 55,000.
Remaining minimum tax shall be carried forward i.e. 40,000 - 5,000 = Rs. 35,000.

15. Minimum tax on builders [U/s 113A]


The entirely newly substituted section provides th at minimum tax on builders. A person that derives income from the
business of construction and sale of residential, commercial or other buildings, shall pay minimum tax that shall be
notify later by the Federal Government. This section shall not have effect till the 30 th June, 2018.
16. Minimum tax on developers [U/s 113B]
The entirely newly substituted section provides t hat minimum tax on developers. A person who derives income
from the business of development and sale of residential, commercial or other plots, shall pay minimum tax 2% of
the value of land notified by any authority for the purpose of stamp duty.
17. Alternative Corporate Tax [U/s 113C]
(1) Notwithstanding anything contained in this Ordinance, for tax year 2014 & onwards, tax payable by a
company shall be higher of the corporate tax or alternative corporate tax.
(2) For the purposes of this section
(a) Accounting income means the accounting profit before tax for the tax year, as disclosed in the
financial statements or as adjusted under sub section (7) or sub section (11) excluding share from the
Associate recognized under equity method of accounting;
(b) Alternative corporate tax means the tax at a rate of 17% of a sum equal to accounting income less
the amount as specified in sub section (8) & determined in accordance with provisions of sub section (7)
hereinafter;
(c) Corporate tax means the total tax payable by the Company in respect of income which is subject to
tax under Division II of Part I of the including tax payable on account of minimum tax, final taxes
payable, under any of the provisions of this Ordinance but not including those mentioned in section 8,
161 and 162 and any amount charged or paid on account of default surcharge or penalty & the tax
payable under this section.
(3) The sum equal to accounting income, less any amount to be excluded there from under sub section (8)
shall be treated as taxable income for the purpose of this section;
(4) The excess of alternative corporate tax paid over the corporate tax payable for the tax year shall be carried
the tax payable under Schedule, for following;
(5) If the excess tax, as mentioned in sub section (4) is not wholly adjusted, the amount not adjusted shall be
carried forward to the following tax year & adjusted as specified in sub section (4) in that year and so on, but
the said excess cannot be carried forward to more than ten (10) tax years immediately succeeding the tax year
for which the excess was first computed;
Explanation: For the purposes of this sub section the mechanism for adjustment of excess of alternative
corporate tax over corporate tax, specified in this section, shall not prejudice or affect the entitlement of the
taxpayer regarding carrying forward & adjustment of minimum tax referred to in section 113 of this Ordinance.
(6) If corporate tax or alternative corporate tax is enhanced or reduced as a result of any amendment or as a
result of any order under the Ordinance, the excess amount to be carried forward shall be reduced or
enhances accordingly.
(7) For the purposes of determining the accounting income expenses shall be apportioned between the amount
to be excluded from accounting income under sub section (8) & the amount to be treated as taxable income
under sub section (3).
(8) The following amount shall be excluded from accounting income for the purposes of computing alternative
corporate tax:-
(i) Exempt income;
(ii) Income subject to tax under section 37A & final tax chargeable under sub section (7) of section 148,
section 150, sub section (3) of section 233
(iii) Income subject to tax credit u/s 65D & 65E.
(iv) Income subject to tax credit u/s 100C and
(v) Income of the Company subject to clause (18A) of Part II of the 2nd schedule.

Conceptual Approach to Taxes 393


Final Tax Regime and Minimum Tax Chapter-22

(9) The provisions of this section shall not apply to taxpayers chargeable to tax in accordance with the provisions
contained in the fourth, fifth & seventh schedules.
(10) Tax credit u/s 65B shall be allowed against alternative corporate tax.
(11) The commissioner may make adjustments & proceeds to compute accounting income as per historical
accounting pattern after providing opportunity of being heard.
18. Tax paid on import of edible oil and packing material (section 148):
The tax collected from a person on the import of edible oil and packing material for a tax year shall be minimum tax.
Edible oil:
Edible oils includes crude oil, imported as raw material for manufacture of ghee or cooking oil.
Example: A company is engaged in the business of sale of cooking oil by manufacturing. Tax deducted on import of
edible oil is Rs. 10,000. Compute the income tax liability of the company from following data assuming that all the
expenses are admissible:
Rs.
Sales 40,000
Cost of sales 20,000
Other operating expenses 10,000
Solution
Rs.
Sales 40,000
Cost of sales 20,000
Gross profit 20,000
Other operating expenses 10,000
Taxable income 10,000
Computation of tax liability:
Higher of:
Tax on Rs. 10,000 @ 33% 3,300
OR
Tax deducted on import of
Edible oils 10,000
Tax liability 10,000
19. Tax deducted on services (section 153):
a. For a person other than a company, tax deductible on gross amount of services @ 10% u/s 153 shall be
minimum tax.
b. If the person to whom the services are provided is not required to deduct tax u/s 153 then minimum tax shall
not be applicable.

Example: Compute the tax liability of Mr. A from following information:


1. Tax liability under NTR 15,000
2. Tax deducted on services u/s 153 10,000
Solution:
As the tax liability under NTR is higher than tax deducted on services, hence tax liability of the Tax payer is
Rs.15,000.
20. Advance tax on Electric bills (section 235(4)):
For a person other than a company tax collected on commercial or industrial electric bill up to Rs. 30,000 shall be
treated as minimum tax. In case of a company or where the amount of bill is more than Rs. 30,000 then the tax
collected shall not be treated as minimum tax and shall be adjustable / refundable against the tax liability of the
person.
Example: Compute the tax liability of Mr. B from following information:
1. Tax liability under NTR 10,000
2. Tax deducted on electric bills 15,000
Solution: Tax liability of the tax payer shall be higher of Rs. 10,000 or Rs. 15,000.

394 Conceptual Approach to Taxes


Final Tax Regime and Minimum Tax Chapter-22

21. Tax on certain payments to non-residents [Section 6]


(1) Subject to this Ordinance, a tax shall be imposed, at the rate specified in Division IV of Part I of the First
Schedule, on every non- resident person who receives any Pakistan-source royalty or fee for technical
services.
(2) The tax imposed under sub-section (1) on a non-resident person shall be computed by applying the relevant
rate of tax to the gross amount of the royalty or fee for technical services.
(3) This section shall not apply to -
(a) any royalty where the property or right giving rise to the royalty is effectively connected with a
permanent establishment in Pakistan of the non-resident person;
(b) any fee for technical services where the services giving rise to the fee are rendered through a
permanent establishment in Pakistan of the non-resident person; or
(c) any royalty or fee for technical services that is exempt from tax under this Ordinance.
(4) Any Pakistani-source royalty or fee for technical services received by a non-resident person to which this
section does not apply by virtue of clause (a) or (b) of sub-section (3) shall be treated as income from business
attributable to the permanent establishment in Pakistan of the person.

Income from royalty [Rule 18] Fee for technical services [Rule 19]

The income of a non-resident person by way of royalty Same as in column except that royalty shall be replaced
received from a resident person or a PE in Pakistan of a non- by the words fee for technical services
resident person shall be-
1. Same as in column 1 except that royalty shall
1. Royalty received as per agreement made before be replaced by the words fee received:
8.3.1980 covered under NTR:
2. Fee for technical services received as per
in the case a royalty received in pursuance of an agreement made on or after 8.3.1980 but before
agreement made before 8.3.1980, or an agreement 04-05-1981 covered under NTR:
made on or after the said date the proposal in respect
In the case of fee received in pursuance of an
of which was approved by the Government before the
agreement made on or after 8.3.1980 but before
said date, the gross amount of the royalty less the
deductions allowed u/s 40; or 4.5.1981, the gross amount of the fee less the
deductions allowed u/s 40 with a maximum total
2. Where final tax regime does not apply OR Royalty deduction equal to 20% of the gross amount of
received in pursuance to any other agreement: such fee; or
In any other case where FTR does not apply, the 3. Fee for technical services received covered
gross amount of the royalty less the following under NTR on which FTR is not applicable:
expenditure:
In any other case to which FTR does not apply, the
Any expenditure incurred in Pakistan in earning such gross amount of fee for technical services less the
income and in respect of any work done in pursuance following expenditures;
of such agreement; and
Any expenditure incurred in Pakistan to earn such
Any expenditure incurred outside Pakistan in fee for technical services, wherever paid.
pursuance of such agreement not exceeding 10%of
Any expenditure incurred outside Pakistan in
gross amount of royalty.
pursuance of such agreement not exceeding10%
No expenses shall be allowed where royalty is fully of gross amount of fee for technical services.
covered under FTR.
Provided that a non-resident may opt for taxation
under final tax regime by filing a written declaration/
option within 15 days of the commencement of
contract. Such option shall remain operative till
completion of the said contract.
No expenses shall be allowed where fee for
technical service is fully covered under FTR.

Conceptual Approach to Taxes 395


Final Tax Regime and Minimum Tax Chapter-22

Tax on shipping income of a non-resident person Tax on air transport income of a non-resident person
[Section 7] [Section 7]

A tax shall be imposed at 8% on the gross amount (except Same as given in first column except that rate of tax shall
exempt amount)on every non-resident person carrying on the be 3% and ships shall be replaced by air transport.
business of operating ships as the owner or charterer in
respect of the gross amount received or receivable (whether in
or out of Pakistan) for the carriage of passengers, livestock,
mail or goods embarked in Pakistan and outside Pakistan.
Non-resident ship owner or charterer [Section 143]
Non-resident aircraft owner or charterer [Section 144]
1. Before the departure of a ship owned or chartered
by a non-resident person from any port in Pakistan, 1. A non-resident owner his agent or charterer of an
the master of the ship shall furnish to the Commissioner aircraft liable for tax shall furnish to the
Inland Revenue a return showing the gross amount in Commissioner Inland Revenue within 45 days
respect of the ship. from the last day of each quarter of the financial
year quarterly returns showing the gross amount.
2. Where the master of a ship has furnished a return, the
Commissioner Inland Revenue shall, after calling for 2. Same as given in first column except that in place
such particulars, accounts or documents, determine the of master of the ship the non-resident agent or
amount of tax due in respect of the ship and shall notify charterer shall be replaced.
the master in writing the amount payable within 3. Where the tax is not paid within 3 months of
specified time. service of the notice, the Commissioner Inland
3. The master of a ship shall be liable for the tax as if it Revenue may issue a certificate to the authority by
were tax due under an assessment order. whom clearance may be granted to the aircraft
operated by the non-resident person and until the
4. Where the Commissioner Inland Revenue is satisfied tax has been paid such authority shall refuse
that the master of a ship or non-resident owner or clearance from any airport in Pakistan to any
charterer of the ship is unable to furnish the return aircraft owned or chartered by the non-resident.
required before the departure of the ship from a port in
Pakistan, the Commissioner Inland Revenue may allow
the return to be furnished within 30 days of departure of
the ship andon satisfactory arrangements for the
payment of the tax due in respect of the ship.
5. The Collector of Customs or other authorised officer
shall not grant a port clearance for a ship owned or
chartered by a non-resident person until tax due in
respect of the ship has been paid or arrangements
payment have been made to their satisfaction.

Return to be furnished by a non resident ship owner or Return to be furnished by a non resident aircraft
charterer [RULE 37] owner or charterer [RULE 38]

A return required to be furnished u/s 143 shall be Same as given in first column U/R 37 except that section
accompanied by such documents, statements and certificates 143 shall be replaced by section 144.
as specified in the form, and in the Ordinance, these rules and
circulars issued under the Ordinance and may be furnished by
any of the methods specified in rules 73 & 74.

396 Conceptual Approach to Taxes


Final Tax Regime and Minimum Tax Chapter-22

MULTIPLE CHOICE QUESTIONS


Q.1 Dividend received by a company is treated
(a) As a separate block of income
(b) Under NTR as SBI
(c) Under FTR
(d) Both a and c
Q.2 Gross dividend received is charged to tax at the rate of
(a) 5%
(b) 7.5%
(c) 10%
(d) 20%
Q.3 Fee for technical services received by a non resident within Pakistan is charged on the gross amount at the rate of
(a) 5%
(b) 10%
(c) 20%
(d) None of the above
Q.4 A non resident person or his authorized agent in case of shipping income shall furnish a return to the Commissioner
Inland Revenue within
(a) 60 days
(b) 30 days
(c) 15 days
(d) 40 days from the end of December and June in every financial year.
Q.5 Where a non-resident person derives income from royalty he shall pay tax on gross amount at the rate of
(a) 15%
(b) 10%
(c) 20%
(d) None of the above
Q.6 Maximum expenses incurred outside Pakistan in case of royalty which is chargeable to tax other than u/s 6(2) shall
be
(a) 15%
(b) 10%
(c) 20%
(d) 5% of the gross amount of royalty income
Q.7 Tax rate on import of edible oils including crude oil imported as raw material for the manufacture of ghee of cooking
oil is
(a) 2%
(b) 1%
(c) 5%
(d) 3%
Q.8 The tax under section 113A on builders shall be treated as _____________.
(a) Minimum tax
(b) Final tax
(c) Fixed tax
(d) None of the above

Conceptual Approach to Taxes 397


Final Tax Regime and Minimum Tax Chapter-22

Q.9 The tax under section 113B on developers shall be treated as _____________.
(a) Fixed tax
(b) Final tax
(c) Minimum tax
(d) None of above
Q.10 A person who pays profit on debt, on account or deposit maintained by a banking company or financial institution
shall deduct tax at source at rate of
(a) 10%
(b) 15%
(c) 5%
(d) None of above
Q.11 Tax rate applicable to the contract with a T.V channel for advertisement services is
(a) 2%
(b) 6%
(c) 3%
(d) 1% on the gross amount payable
Q.12 Tax deduction on payment to a resident person on account of sale of edible oil is at
(a) 1.5%
(b) 3.5%
(c) 6%
(d) None of the above
Q.13 Goods supplied to an exporter under an inland back-to-back letter of credit is taxable at the rate of
(a) 2%
(b) 5%
(c) 1%
(d) None of the above
Q.14 Tax deducted on imported plant & machinery for own use shall be treated under _____________.

(a) NTR
(b) FTR
(c) Minimum tax
(d) Both band c
Q.15 Prizes and winnings are covered under
(a) NTR
(b) FTR as SBI
(c) As a separate block
(d) Both b and c
Q.16 Prize on prize bond is taxable at the rate of
(a) 20%
(b) 15%
(c) 10 %
(d) 5% of the gross amount paid.
Q.17 Prize on winning a quiz is taxable at the rate of
(a) 20%

398 Conceptual Approach to Taxes


Final Tax Regime and Minimum Tax Chapter-22

(b) 15%
(c) 10%
(d) 5% of the gross amount paid.
Q.18 A person selling petroleum products to a petrol pump operator is required to deduct tax from the amount of
commission or discount allowed at the rate of____________.
(a) 15%
(b) 12%
(c) 20%
(d) None of the above
Q.19 The tax rate applicable to the indenting agent receiving commission or brokerage is____________.
(a) 12%
(b) 5%
(c) 5%
(d) None of the above
Q.20 Owner of goods transport vehicles shall pay tax on motor vehicles on the basis of _______.
(a) taxable income
(b) laden weight
(c) number of wheals
(d) none of above
Q.21 Where the income is taxable under ___ it shall not be chargeable to tax under any head of income while computing
the taxable income of the person.
(a) FTR
(b) SBI under NTR
(c) NTR
(d) all of above
Q.22 The person deriving income covered under ______ shall be allowed deduction for expenditure incurred in deriving
such income.
(a) FTR
(b) SBI under NTR
(c) NTR
(d) all of above
Q.23 Amount of tax paid by the owner of the goods transport vehicle having registered laden weight from 8,120 kg to
14,999 kg is taxable under_______________.
(a) FTR
(b) NTR
(c) SBI
(d) None of the above
Q.24 Amount of tax paid by the owner of the goods transport vehicle having registered laden weight above 59,999 kg is
taxable under________________.
(a) FTR
(b) NTR
(c) SBI
(d) None of the above
Q.25 The amount of tax per annum on the goods transport vehicle having registered laden weight more than 5,120 kg after
10 years of registration will be Rs.________.

Conceptual Approach to Taxes 399


Final Tax Regime and Minimum Tax Chapter-22

(a) Nil
(b) 1,200
(c) 7,200
(d) None of the above
Q.26 Turnover tax under section 113 may be levied at the rate of ________.
(a) 0.50%
(b) 1%
(c) 0.2%
(d) Any of the above
Q.27 The turnover tax in excess of NTR shall be adjusted against the normal tax liability for the immediately succeeding
the years for maximum period of ____ upto the extent that the NTR liability of the following years should not be less
than the turnover tax of the following years.
(a) 6 years
(b) 5 years
(c) 10 years
(d) 3 years
Q.28 Turnover tax is not applicable where there is ______ in a tax year before setting off of depreciation and inadmissible
expenses.
(a) gross loss
(b) net loss
(c) net profit
(d) gross profit
Q.29 The general tax rate of 10% is applicable on dividend received by ______________.
(a) individual
(b) company
(c) AOP
(d) All of the above
Q.30 Tax rate on dividend income is applied on the __________.
(a) net amount
(b) gross amount
(c) amount after zakat deduction
(d) none of above
Q.31 Dividend income by an individual from a power generation company shall be charged @ ___% covered under FTR.
(a) 10
(b) 7.5
(c) 6
(d) 0
Q.32 Where applicable tax at source is deducted by a company at the time of making payment to an AOP taxpayer for
______ the same shall deemed as final discharge of tax in respect of such income.
(a) dividend
(b) sale of goods
(c) commission
(d) all of the above
Q.33 Where the non-resident is likely to leave Pakistan, who has earned income under fee for technical services, within the
period of one year, he shall fulfil all those requirements as are applicable to the __________.

400 Conceptual Approach to Taxes


Final Tax Regime and Minimum Tax Chapter-22

(a) deceased person


(b) resident person
(c) discontinuance of business
(d) none of above
Q.34 The amount paid as tax by a non-resident person on his income as royalty shall fully discharge him from ________.
(a) tax liability
(b) business liability
(c) personal loan
(d) all of above
Q.35 Fee for technical services which are rendered through a Permanent Establishment of the non-resident will be charged
@ ___% of gross amount received.
(a) 6
(b) 15
(c) 20
(d) 30
Q.36 _____ means transfer of rights or granting of a licence in respect of a patent, invention, model, design, secret process
or formula, trade mark or similar property.
(a) dividend
(b) fee for technical services
(c) royalty
(d) all of above
Q.37 ______ includes the imparting of any information concerning technical, industrial, commercial or scientific knowledge,
experience or skill.
(a) dividend
(b) fee for technical services
(c) royalty
(d) all of above
Q.38 Tax on imports by the importer at the time of clearance from custom authorities in respect of fertilizer for
manufacturing purpose shall be treated as _______.
(a) final tax
(b) adjustable
(c) custom duty
(d) none of above
Q.39 Tax is deducted at ____% of the gross amount of profit on debt as reduced by Zakat under Zakat and Ushr
Ordinance.
(a) 10
(b) 15
(c) 20
(d) 30
Q.40 A person shall be not required to furnish the return of income when all the incomes derived by him in the tax year are
covered under __________.
(a) FTR
(b) SBI under NTR
(c) NTR
(d) all of above

Conceptual Approach to Taxes 401


Final Tax Regime and Minimum Tax Chapter-22

Q.41 Turnover tax is applicable on ___ on fulfilment of certain conditions.


(a) companies
(b) individuals
(c) AOP
(d) all of above
Q.42 While making payment in respect of brokerage or commission to the taxpayers, prescribed persons making payment
are required to _______.
(a) deduct tax at source
(b) deduct service charges
(c) inform to SECP
(d) all of above
Q.43 Tax deducted on brokerage or commission received by an agent is treated __________ of the agent for such income.
(a) normal tax liability
(b) final tax liability
(c) last tax liability
(d) all of above
Q.44 Where the taxpayer is an owner of goods transport vehicles, the tax deducted at source excluding motor vehicle tax
shall be treated as _________.
(a) final tax
(b) last tax
(c) minimum tax
(d) None of the above

MULTIPLE CHOICE ANSWERS


1 (c) 2 (c) 3 (d) 4 (b) 5 (a)
6 (b) 7 (c) 8 (a) 9 (c) 10 (a)
11 (b) 12 (a) 13 (c) 14 (a) 15 (b)
16 (b) 17 (a) 18 (b) 19 (b) 20 (b)
21 (a) 22 (c) 23 (a) 24 (a) 25 (a)
26 (d) 27 (b) 28 (a) 29 (d) 30 (b)
31 (b) 32 (d) 33 (c) 34 (a) 35 (b)
36 (c) 37 (b) 38 (b) 39 (a) 40 (a)
41 (a) 42 (d) 43 (b) 44 (d)

402 Conceptual Approach to Taxes


Final Tax Regime and Minimum Tax Chapter-22

PRACTICE QUESTIONS FOR CA MOD F AND ICMA STUDENTS


You are provided with the following data of financial results of some taxpayers relevant to the tax year 2016 in order to
compute tax liability:
1. Company 1 achieved the turnover of Rs. 300 million during the year. Taxable income was computed at Rs.2,000,000.
The taxable income of the company includes interest income amounting to Rs. 500,000.
2. Company 2 although achieved the gross sales volume of Rs. 400 million, however, this year the company could not
achieve profits and fetch losses to the tune of Rs. 20 million. It is worth mentioning that the products of the company
are subject to levy of sales tax and excise duty. During the year, the company charges sales tax amounting to Rs.45
million after allowing a discount of Rs. 30 million. Whereas the company has paid excise duty amounting to Rs. 20
million on the production of excisable goods.
3. Company 3 is a non-resident company and the company also suffered losses aggregating to Rs. 40 million as
compared to sales of 3 billion.
4. Company 4 is engaged in the supply of goods and execution of contracts. During the year, the company only executed
contract in FATA and due to non existence of banking system in FATA, all these sums are received in cash to the
company. Total contract receipts are Rs. 400 million. Total turnover of supplies made in Pakistan aggregates to Rs.200
million. Whereas aggregate overall profit of the company is Rs. 7 million. It is worth mentioning that the company
assessed carry forward losses for the last six years aggregates to Rs. 9 million. The company paid commission to its
dealers amounting to Rs. 7 million.
5. Company 5 is incorporated in July, 2015. Total paid up capital of the company is Rs. 20 million and during the year, the
company achieved a sales volume of Rs. 200 million. The company earned a profit of Rs. 1,800,000 during its first year
of operation.
6. Company 6 is incorporated in September, 2015. Total paid up capital of the company is Rs. 15 million and during the
year, the company achieved a sales volume of Rs. 200 million. The company has gross loss before depreciation and
inadmissible expenses Rs. 5 million during its first year of operation.
Compute the tax liability of the aforesaid taxpayer for the tax year 2016.
Solution:Computation of
tax liability:
1 2 3 4 5 6
A. Under minimum tax: Rs. Rs. Rs. Rs. Rs. Rs.
Gross turnover 300,000,000 400,000,000 3,000,000,000 600,000,000 200,000,000 200,000,000
Less: sales tax - 45,000,000 - - - -
Less: discount - 30,000,000 - - - -
Net turnover 300,000,000 325,000,000 3,000,000,000 600,000,000 200,000,000 200,000,000

Tax on turnover @ 1% 3,000,000 3,250,000 - 6,000,000 2,000,000 -

B. Under Normal tax:

Taxable income 2,000,000 (20,000,000) (40,000,000) 7,000,000 1,800,000 (5,000,000)


Carry forward of losses - - (9,000,000) - -
Income 2,000,000 (20,000,000) (40,000,000) (2,000,000) 1,800,000 (5,000,000)
Tax @ 32% 640,000 - - - 576,000 -
(25% for small company)
Tax liability
(Higher of A or B) 3,000,000 3,250,000 - 6,000,000 2,000,000 -
Definition of company includes the following:
Banking company, public company, private company, co-operative society, finance society, assets management company,
financial institutions, HBFC, investment company, leasing company, mutual fund, non-banking finance company, venture
capital company, real estate investment trust scheme and real estate investment trust management company.
Rate of tax:
1. The rate of tax imposed on the taxable income of a company for the tax year 2016 is 32%.
2. The rate of tax imposed on the taxable income of a small company shall be 25%.
3. The rate of tax imposed on the taxable income of a Modaraba shall be 25% excluding income covered under FTR.

Conceptual Approach to Taxes 403


Final Tax Regime and Minimum Tax Chapter-22

Tax credits for companies:


1. Charitable donations u/s 61
2. Profit on debt u/s 64
3. Tax credit for sales tax registered manufacturer u/s 65A
4. Tax credit for investment u/s 65B
5. Tax credit for enlistment u/s 65C
6. TC for newly established undertakings u/s 65D
7. Tax credit for industrial undertakings established before the first day of July 2011 u/s 65E

404 Conceptual Approach to Taxes


Final Tax Regime and Minimum Tax Chapter-22

ICMAP PAST PAPERS THEORECTICAL QUESTIONS


Q. 1(c) February 2013 (ii) State the provision regarding 'taxation of petroleum products' as per section 156A of the
Income Tax Ordinance, 2001.
Q. 2(a) April 2012 In the light of the Section 6 of the Income Tax Ordinance, 2001 relating to Pakistan-source royalty and
fee for technical services received by a non-resident person, answer
the following:
(i) At which rate shall the tax be imposed?
(ii) On which amount shall the tax be imposed?
(iii) On which types of royalty / fee for technical services this section shall not apply?

Q. NO. 2(b) SUMMER 2010 Explain minimum tax with reference to section 113 of the Income tax Ordinance, 2001.

Q. NO. 4 (a) WINTER 2006 Explain the term Turnover u/s 113(3) of the Income tax Ordinance, 2001.

Conceptual Approach to Taxes 405


Final Tax Regime and Minimum Tax Chapter-22

CA MOD C PAST PAPERS THEORECTICAL QUESTIONS


Q. NO. 5 Spring 2014 Under the provisions of Income Tax Ordinance, 2001, certain persons are required to pay
minimum tax at the rate of 1% of their turnover.
Required
(a) Explain the term Turnover for the purpose of minimum tax.
(b) List the persons who are required to pay minimum tax.
(c) Discuss the rules relating to carry forward of minimum tax paid to the subsequent years.
Q.2 (b) Autumn 2012 Tax imposed at the rate of 15% on every non-resident person who receives Pakistan source royalty is
considered to be a final tax on the amount in respect of which the tax is imposed.
Identify the exceptions to the above rule, according to the Income Tax Ordinance, 2001.
Q.6 (a) Spring 2012 After completion of the audit of Chandi Enterprise (CE) under the Income Tax Ordinance, 2001, the
Commissioner Inland Revenue has ordered the following amendments in the income tax return filed by CE.
(i) Payment of minimum tax at the rate of 0.5% on its total turnover of Rs. 45 million.
(ii) Disallowance of the following expenditures:
- Rs. 27,000 spent on Annual Eid-Milan Party arranged by the firm for its employees and their families.
- Penalty for late delivery amounting to Rs. 60,000 which had to be paid to a client on account of negligence on
the part of the Shipment Manager.
- Donation of Rs. 150,000 paid to the National Institute of Cardio Vascular Diseases(NICVD), established by the
Federal Government.
- Salary of Rs. 850,000 paid to the Managing Director, who is also a partner in the firm.
Required: Comment on the above amendments ordered by the Commissioner Inland Revenue, in the light of Income Tax
Ordinance, 2001.
Q.3 (a) Autumn 2011 Under the provisions of Income tax Ordinance, 2001 certain persons are required to pay minimum tax
at the rate of 1% of their turnover.
Required:
(i) Explain the term Turnover for the purpose of minimum tax.
(ii) List the persons who are required to pay minimum tax.
(iii) Discuss the rules relating to carry forward of minimum tax paid to the subsequent years.
Q. NO. 5(b) Spring 2010 Tax imposed at the rate of 15% on every non-resident person who receives Pakistan source
royalty or fee for technical services is considered to be a final taxed.
Required: Identify the exceptions to the above rule, as referred to in the Income tax Ordinance, 2001.
Q. NO. 5(b) Spring 2008 There are various situations in the ITO, 2001 when the rules applicable to Companies are different
from those applicable to other assessees. Briefly discuss the differences in the tax laws for an Individual and a Company in
the following cases:
(i) Dividend income (ii) Rental income
(iii) Method of accounting (iv) Payment against sale of goods or services
Q. NO. 4(a) Spring 2006 Under the Income tax Ordinance, 2001, tax imposed on non-residents in respect of their incomes
from Fees for Technical services and royalty shall be a final tax on the amount in respect of which the tax is imposed.
State under what circumstances, such incomes of a non-resident are not considered to be final tax liability.
Q. NO. 4(c) Autumn 2006 Explain the relevant provisions of Income tax Ordinance, 2001 regarding applicability of minimum
tax.
Q. NO. 11 Autumn 2003 Who are the persons liable to Turnover Tax and at what rate is the turnover tax payable? Are there
any exceptions.
Q.10 Spring 2002 For assessees enjoying resident status, presumptive tax regime is applicable only on those earning
business income. Comment
Q. NO. 5 September 2000 Explain as to how under FTR, tax liability arises for business class of taxpayer.
Q. NO. 6 May1996 What are the provisions regarding the taxability of income of an individual from dividends etc.
Q. NO. 7 April 1995 State your opinion with reasons about the following issues:
An importer or an exporter has incurred loss on imports and exports. Will there be any incidence of tax on such persons.

406 Conceptual Approach to Taxes


Deduction / Payment of Tax Chapter-23

Chapter

23 DEDUCTION / PAYMENT OF TAX

Topic covered Topic covered


Section Rule
Section Rule
39A to
(For CA Mod F & ICMAP students) 165A Furnishing of information by banks
39F
166 Priority of tax collected or deducted
137 Due date of payment of tax 167 Indemnity
123, 124 Recovery of tax out of property &
138 168 Credit for tax collected or deducted
& 125 through arrest of taxpayer
Recovery of tax by district officer
138A 175 Power to enter and search premises
(revenue)
Notice to obtain information or
138B Bankruptcy 176
evidence
Collection of tax in case of private
139 220 Receipts for amounts paid
companies and AOPs
Recovery of tax from persons
140 holding money on behalf of a 141 Liquidators
taxpayer
Recovery of tax due by non-resident
147 Advance payment of tax 142
member of an AOP
148, 148A Deduction or collection of tax at Recovery of tax from persons
146
&158 source assessed in Azad Jammu & Kashmir
Initiation, validity, etc., of recovery
158 Time of deduction of tax 146A
proceedings
Tax arrears settlement incentives
159 40 Exemption or lower rate certificate 146B
scheme
159 Rules for rats of tax deduction 170 71 Refunds
Payment of tax collected or Additional payment for delayed
160 43 171
deducted refunds
Reduction in default surcharge,
Failure to pay tax collected or
161 205A consequential to reduction in tax or
deducted
penalty
Rule 84 Registration of income tax
205 Default surcharge
to 90 Practitioners
Recovery of tax from whom the tax
162
was not collected or deducted
Recovery of advance payment or
163 MCQS with solutions
deduction of tax at source
Certificate of collection or deduction ICMAP & CA Mod C past papers
164 42
of tax theoretical questions
165 44 Statement of tax deduction

Conceptual Approach to Taxes 407


Deduction / Payment of Tax Chapter-23

For CA MOD F & ICMAP students

1. Due date of payment of tax [Section 137]

SR. Due date of payment of tax


1. The tax payable by a taxpayer on the taxable income of the taxpayer including minimum tax or tax payable
by a retailer for a tax year shall be due on the date for furnishing of return of income for that year.
2. Where any tax is payable under an assessment order or an amended assessment order a notice shall be
served upon the taxpayer specifying the amount payable within 30 days from the date of service of the
notice.
3. In case of provisional assessment, tax shall be payable after a period of 45 days from the date of service
of the notice. However, taxpayer may pay the tax payable prior to expiry of the period of 45 days. This facility
is not for tax payable with the income tax return.
4. Where a taxpayer is allowed to pay tax by instalments and the taxpayer defaults in payment of any
instalments, the whole balance of the tax outstanding shall become immediately payable. This facility is not
for tax payable with the income tax return.
5. Upon written application by a taxpayer, the Commissioner Inland Revenue on reasonable grounds shown
may grant the taxpayer an extension of time for payment of tax due or in instalments of equal or varying
amounts as the Commissioner Inland Revenue may deem fit.
6. The payment of tax after having an extension of time to pay tax due or tax to be paid in instalments shall be
made along-with default surcharge u/s 205 of the Ordinance.

2. Recovery of tax out of property and through arrest of taxpayer [Section 138]
For recovery of any tax due by a taxpayer, the Commissioner Inland Revenue may serve a notice requiring him to
pay the said amount within such time as specified in the notice.
If the amount of tax is not paid within specified time or within the further time as allowed by the Commissioner Inland
Revenue, then he may proceed to recover from the taxpayer the said amount by one or more of the following modes,
namely: -
(a) attachment and sale of any movable or immovable property of the taxpayer;
(b) appointment of a receiver for the management of the movable or immovable property of the taxpayer; and
(c) arrest of the taxpayer and his detention in prison for a period not exceeding six months.
Form of notice of recovery to be issued by commissioner inland revenue [Rule 123]
The notice required to be served upon the taxpayer u/s 138 shall be in the form as prescribed in part-Ill of First
Schedule to these rules.
Mode of service of notice [Rule 124]
The notice referred to in rule 123 and other notices under rules contained in this part shall be served as provided in
section 218 of the ITO, 2001.
Time limit for execution of the notice [Rule 125]
No step in execution of the notice as per rule 123 shall be taken until the period specified in the said notice has
elapsed.
If the Commissioner Inland Revenue is satisfied that the defaulter is to cancel, remove or dispose of the whole or any
part of such of his moveable property as would be liable to attachment in execution of a notice that the realization of
the amount of notice would in consequence be delayed or obstructed, he may at any time after the issue of the notice
under rule direct, for reasons to be recorded in writing, an attachment of the whole or part of such property:
If the defaulter whose property has been so attached furnishes security to the satisfaction of the Commissioner Inland
Revenue, such attachment shall be cancelled from the date on which such security is accepted by the Commissioner
Inland Revenue.
3. Recovery of tax by district officer (revenue) [Section 138A]
The Commissioner Inland Revenue may forward to the District Officer (Revenue) of the district in which the taxpayer
resides or carries on business or in which any property belonging to the taxpayer is situated, a certificate specifying
the amount of any tax due from the taxpayer and on receipt of such certificate, the recovery of tax from the taxpayer
shall be made as, it were an arrear of land revenue.

408 Conceptual Approach to Taxes


Deduction / Payment of Tax Chapter-23

4. Bankruptcy [Section 138B]


If a taxpayer is declared bankrupt, the tax liability shall pass on to the estate in bankruptcy. If tax liability is incurred by
an estate in bankruptcy, the tax shall be deemed to be a current expenditure in the operations of the estate in
bankruptcy and shall be paid before the claims preferred by other creditors are settled.
5. Collection of tax in the case of private companies and associations of persons [Section 139]
Where any tax payable by a private company in respect of any tax year cannot be recovered from the company,
every person who was, at any time in that tax year -
(a) a director of the company, other than an employed director, or
(b) a shareholder in the company owning not less than 10% of the paid-up capital of the company,
Shall be jointly and severally liable for payment of the tax due from the company,
Any director who pays tax as above shall be entitled to recover the tax paid from the company or a share of the tax
from any other director. Similarly a shareholder who pays tax shall be entitled to recover the tax paid from the
company or from any other shareholder that has 10% holding of the paid up of the company in that proportion.
Where any tax payable by a member of an AOP in respect of the member's share of the income of the AOP in
respect of any tax year cannot be recovered from the member, the AOP shall be liable for the tax due by the member.
6. Recovery of tax from persons holding money on behalf of a taxpayer [Section 140]
For the recovery of any tax due by a taxpayer the Commissioner Inland Revenue may by notice in writing require any
person owing to or holding money for taxpayer.
The amount mentioned in the notice-
(a) where the amount of the money is equal to or less than the amount of tax due by the taxpayer, shall not
exceed the amount of the money; or
(b) in any other case, shall be so much of the money as is sufficient to pay the amount of tax due by the taxpayer.
Where a person is liable to make a series of payments to a taxpayer the notice may specify an amount to be paid out
of each payment until the amount of tax due by the taxpayer has been paid. The date for payment specified in a
notice shall not be a date before the money becomes payable to the taxpayer or held on the taxpayer's behalf.
7. Advance payment of tax [Section 147]

Particulars Companies and AOPs Individuals


(1) (2) (3)
1. Basis Quarterly basis as under: Quarterly basis as under:
(Turnover for the quarter x Tax assessed for (Tax assessed for the latest tax year /
the latest tax year / Turnover for the latest tax 4)
year) Less Tax deduction or paid at source
Less Tax paid in the quarter (other than in the quarter (other than actual tax
actual tax on incomes covered at serial no 3 on incomes covered at serial no 3
below). below).
Minimum tax where applicable shall also be Same as in column 2
considered for the purpose of payment of
advance tax.
2. When applicable Without considering the quantum of income. If latest assessed taxable income is
equal to or greater than Rs. 500,000.
3. Incomes not to be Income covered under final tax regime Income from salary and income
considered for advance covered under final tax regime
tax
4. Last date of payment 25th September for September quarter 15th September for September
25th December for December quarter quarter
25th March for March quarter 15th December for December quarter
15th June for June quarter 15th March for March quarter
15th June for June quarter
5. Estimation on the a. Shall estimate the tax payable for the
basis of current year relevant tax year at any time before the
projections second installment is due. In case the tax Section is silent on this issue
payable is likely to be more than the
amount that the taxpayer is required to
pay under sub-section (4), the taxpayer
shall furnish to the CIR on or before the
due date of the second quarter an

Conceptual Approach to Taxes 409


Deduction / Payment of Tax Chapter-23

estimate of the amount of tax payable Same as in column 2.


by the taxpayer and thereafter pay 50%
of such amount by the due date of the
second quarter of the tax year after
making adjustment for the amount (if
any) already paid in terms of sub-
section (4). The remaining 50% of the
estimate shall be paid after the second
quarter in two equal instalments payable
by the due date of the third and fourth
quarter of the tax year.

b. If the said taxpayers of the view that his


income for the current tax year would
likely to be less than this latest tax year
then he may file an estimate of his income
to the Commissioner Inland Revenue and
shall pay advance tax accordingly. But if
advance tax paid according to this
estimate is less than 90% of tax liability of
the relevant tax year he shall be liable to
pay default surcharge.
6. Advance tax in the Shall be on the basis of quarterly estimated
absence of latest profit / turnover including the effect of Not applicable
assessed income or minimum tax.
declared turnover

Advance tax on capital gain from sale of securities [Section 147(5B)]


Adjustable advance tax on capital gain from sale of securities shall be chargeable as under, namely:-

S. No. Period Rate of Advance Tax


(1) (2) (3)
1. Where holding period of a security is less than six 2% of the capital gains derived during the quarter
months
2. Where holding period of a security is more than 1.5% of the capital gains derived during the quarter
six months but less than twelve moths
Provided that such advance tax shall be payable to the Commissioner Inland Revenue within a period of twenty one
days after the close of each quarter.
Credit / refund shall be allowed of advance tax paid under section 147 of the Ordinance.
8. Deduction or collection of tax at source (Section 148, 148A and 158):
Originally tax at source means the collection or deduction of tax at source from where an income is being generated
and it is considered as an advance payment of tax on behalf of the taxpayer. Later on withholding tax were
categorized into two segments
(i) Treating the tax collected as advance tax and
(ii) Considering it as full and final discharge of tax liability of the taxpayer in respect of the source against which
the deduction has been made.
Any notification issued under the said sub section and for the time being in force shall continue to remain in
force unless rescinded by the Board through notification in the official gazette.
Tax on local purchase of cooking oil or vegetable ghee [Section 148A]
The manufacturers of cooking oil or vegetable ghee, or both, shall be chargeable to tax at the rate of 2% on
purchase of locally produced edible oil.
The tax payable as above shall be final tax in respect of income accruing from locally produced edible oil.
9. Time of deduction of tax (Section 158)
A person required to deduct tax from an amount paid by the person shall deduct tax -
(a) in the case of deduction of tax on profit on debt, at the time the amount is paid or credited to the account of
recipient, whichever is earlier;
(b) in other cases, at the time the amount is actually paid; and
(c) Amount actually paid shall have the meaning as may be prescribed.

410 Conceptual Approach to Taxes


Deduction / Payment of Tax Chapter-23

10. Exemption or lower rate certificate (Section 159):


If the income of a person is:
(a) Exempt from tax;
(b) Chargeable to tax at lower rate, or
(c) is subject to 100% tax credit u/s 100C.
Then Commissioner Inland Revenue shall issue him an exemption or lower rate certificate upon application made by
him in writing.
A person who is required to deduct tax at source shall deduct tax according to the exemption or lower rate certificate
but if certificate is not presented to him then he shall deduct full amount of tax.
Exemption or lower rate certificate u/s 159 [Rule 40]
An application for a certificate u/s 159(1) read with clause (V) of paragraph 1 of Notification No. SRO
947(1)/2008, dated 5.9.2008, shall be in the form specified in Part VII (a) of the First Schedule to the Income
Tax Rules, 2002.
A system based exemption certificate issued by the Commissioner Inland Revenue for goods specified in the
aforesaid notification shall be in the form specified in Part VIII (a) of the First Schedule to Income Tax Rules,
2002.
11. Rules for Rates of Tax deduction (Section 159):
Any notification issued under the said sub sections and for the time being in force shall continue to remain in
force unless rescinded by the Board through notification in the official gazette.
12. Payment of tax collected or deducted (Section 160):
Any tax that has been collected or deducted shall be paid to the Commissioner Inland Revenue by the person making
the collection or deduction within the time and in the manner as may be prescribed.
Payment of tax collected or deducted [Rule 43]
As required u/s 160 and under the Sixth Schedule to the Ordinance, the tax collected or deducted under various
sections of the Ordinance or Sixth Schedule to the Ordinance shall be paid to the Commissioner Inland Revenue by
way of credit to the Federal Government,-
(a) where the tax has been collected or deducted by the Federal Government or a Provincial Government on the
day the tax was collected or deducted; or
(b) where the tax has been collected or deducted by a person other than the Federal Government or a Provincial
Government, by remittance to the Government Treasury or deposit in an authorized branch of the State Bank
of Pakistan or the National Bank of Pakistan, 7 days from the end of each week ending on every Sunday.
13. Failure to pay tax collected or deducted (Section 161):
Where a person fails to collect tax or deduct tax from a payment or having collected tax fails to pay the tax to the
Commissioner Inland Revenue as required, the person shall be personally liable to pay the amount of tax to the
Commissioner Inland Revenue who may pass an order to recover the same. No recovery shall be made unless the
person has been provided with an opportunity of being heard.
Where at the time of recovery of tax the tax that was to be deducted or collected has been paid by that person, no
recovery shall be made from the person who had failed to collect or deduct the tax but the said person shall be liable
to pay default surcharge @ 12% p. a. from the date he failed to collect or deduct the tax to the date the tax was
paid.
A person personally liable for an amount of tax as a result of failing to collect or deduct the tax shall be entitled to
recover the tax from the person from whom the tax should have been collected or deducted.
14. Default Surcharge (Section 205)
Where a person fails to pay any tax (excluding advance tax u/s 147), any penalty or any tax due from persons
holding money on behalf of a taxpayer or a liquidator, any tax u/s 160 on or before the due date, the said
person shall be liable to pay default surcharge @ 18% from the date the aforesaid amounts are due and
ending on the date on which it was paid.
Where a person fails to pay advance tax u/s 147, on or before the due date, the said person shall be liable to
pay default surcharge @ 18% from the date the amount was due and ending on the date on which it was paid.
Where a person fails to pay advance tax u/s 147(4A) and (6) or the tax so paid is less than 90% of the tax
chargeable for the relevant tax year, the said person shall be liable to pay default surcharge @ 18% from the
first day of April in that year to the date on which assessment is made or the thirtieth June of the
financial year next following, whichever is earlier.

Conceptual Approach to Taxes 411


Deduction / Payment of Tax Chapter-23

Provided that if the person opts to pay the tax due on the basis of an order under section 129 on or before the due
date given in the notice u/s 137(2) issued in consequence of the said order, and does not file an appeal u/s 131, he
shall not be liable to pay default surcharge for the period beginning from the due date of payment in consequence of
an order appealed against to the date of payment in consequence of notice u/s 137(2).
Provided that if the person opts to pay the tax due on the basis of an order u/s 129 on or before the due date given in
the notice under section 137(2) issued in consequence of the said order and does not file an appeal u/s 131, he shall
not be liable to pay default surcharge for the period beginning from the date of order u/s 161 to the date of payment.
15. Recovery of tax from the person from whom the tax was not collected or deducted (Section 162):
Where a person fails to collect or deduct tax from a payment, the Commissioner Inland Revenue may pass an order
to that effect and recover the amount not collected or deducted from the person from whom the tax should have been
collected or to whom the payment was made.
Such recovery of tax does not absolve the person who failed to deduct tax from any other legal action in relation to
the failure, or from a charge of default surcharge or the disallowance of a deduction for the expense to which the
failure relates.
16. Recovery of advance payment or deduction of tax at source (Section 163)
The provision of this Ordinance shall apply to any amount required to be paid to the Commissioner Inland Revenue
as advance tax or tax deducted at source as if it were tax due under an assessment order.
17. Certificate of collection or deduction of tax (Section 164)
Every person collecting tax or deducting tax from a payment shall furnish to the person from whom the tax has been
collected or deducted, copies of the challan of payment or any other equivalent document a certificate setting out the
amount of tax collected or deducted.
A person required to furnish a return of taxable income for a tax year shall attach to the return copies of the challan of
payment on the basis of which a certificate is provided to the person in respect of tax collected or deducted in that
year.
Certificate of collection or deduction of tax [Rule 42]
As required u/s 164(1), any person responsible for collecting tax under various sections of the Ordinance
(except in the case of salary) or deducting tax under the 6th Schedule to the Ordinance,
shall issue a certificate to the person from whom tax has been collected or deducted, in the form as set out in
Part VII of the Second Schedule to Income Tax Rules, 2002, within 15 days after the end of the financial year
or discontinuation of business etc.-
Where the person from whom tax has been collected or deducted requests for the issuance of the certificate
before the end of the financial year, the certificate shall be issued for the period in that year within 7 days of
such request.
If certificate issued has been lost, stolen or destroyed the recipient of the certificate may request, in writing, to
the issuer of the certificate to issue a clearly marked serially numbered certificate as duplicate.
18. Statement of tax deduction (Section 165)
Every person collecting or deducting tax from a payment furnish to the Commissioner Inland Revenue statement
even where no withholding tax is collected or deducted during the period.in the prescribed form setting out-
(a) The name, CNIC and NTN and address of each person from whom tax has been collected or deducted in
each month;
(b) the total amount of payments made to a person from which tax has been deducted or collected in each month;
and
(c) such other particulars as may be prescribed:
Every prescribed person collecting tax or deducting tax from payment shall furnish or e-file statements by the 20th
day of the month following the month to which the withholding tax pertains.
Explanation:- For the removal of doubt, it is clarified that this sub-section overrides all conflicting provisions
contained in the Protection of Economic Reforms Act, 1992, the Banking Companies Ordinance, 1962, the Foreign
Exchange Regulation Act, 1947 and regulations made under the State Bank of Pakistan Act, 1956, if any, on the
subject, in so far as divulgence of information u/s 165
A person may apply in writing, to the Commissioner Inland Revenue for an extension of time to furnish the statement
after the due date and the Commissioner Inland Revenue if satisfied that a reasonable cause exists for non-furnishing
of the statement by the due date may, by an order in writing, grant the applicant an extension of time to furnish the
statement.
Every person deducting tax from payment against salary shall furnish to the Commissioner Inland Revenue an
annual statement in the prescribed form.

412 Conceptual Approach to Taxes


Deduction / Payment of Tax Chapter-23

Annual statement of tax collected or deducted [Rule 44]


An annual statement required to be furnished u/s 165(1) for a financial year shall be in the form as set out in
Part VIII and Part IX of the Second Schedule to the Income Tax Rules, 2002.
A person responsible for collecting or deducting tax under various sections of this Ordinance shall furnish a
monthly statement within 20 days of the end of each month as set out in Part X of the Second Schedule to the
Income Tax Rules, 2002.
The above statements shall be accompanied by the evidence of deposit of tax collected or deducted to the
credit of the Federal Government.
Wherever required by the Commissioner Inland Revenue, a person shall furnish a reconciliation of the
amounts mentioned in annual and monthly statements with the amounts mentioned in the return of income and
allied documents submitted from time to time.
19. Furnishing of information by banks (Section 165A)
(1) Notwithstanding anything contained in any law for the time being in force including but not limited to the
Banking Companies Ordinance, 1962, the Protection of Economic Reforms Act, 1992, the Foreign Exchange
Regulation Act, 1947 and the regulations made under the State Bank of Pakistan Act,1956 if any, on the
subject, every banking company shall make arrangements to provide to the Board in the prescribed form
and manner,-
(a) Online access to its central database containing details of its account holders and all transactions made
in their accounts;
(b) a list containing particulars of deposits aggregating rupees one million or more made during the
preceding calendar month;
(c) a list of payments made by any person against bills raised in respect of a credit card issued to that
person, aggregating to rupees one hundred thousand or more during the preceding calendar
month;
(d) a consolidated list of loans written off exceeding rupees one million during a calendar year; and
(e) a copy of each Currency Transactions Report and Suspicious Transactions Report generated and
submitted by it to the Financial Monitoring Unit under the Anti-Money Laundering Act, 2010.
(2) Each banking company shall also make arrangements to nominate a senior officer at the head office
to coordinate with the Board for provision of any information and documents in addition to those listed in
sub-section (1), as may be required by the Board.
(3) The banking companies and their officers shall not be liable to any civil, criminal or disciplinary proceedings
against them for furnishing information required under this Ordinance.
(4) Subject to section 216, all information received under this section shall be used only for tax purposes
and kept confidential.
Banking Companies Reporting Requirements [Rule 39A to 39F]
For the purposes of section 165A of the Ordinance the above rules contain reporting requirements for banking
companies.
Definitions [U/R 39B]
(1) In this Chapter, unless there is anything repugnant in the subject or context,-
(a) Account Holders Deposits statement means Account Holders Deposits Statement as specified in Form
A,
(b) Banking Company Officer means a senior officer stationed at the head office and nominated by a
banking company to coordinate with the Board for provision of any information and documents required
by the Board;
(c) Credit Card Payments Statement means Credit Card Payments Statement as specified in Form B;
(d) Currency Transactions Report means currency transactions report generated and submitted by a
banking company to the Financial Monitoring Unit under the Anti-Money Laundering Act, 2010;
(e) information includes Account Holders Deposits Statement, Credit Card Payment Statement, Written
Off Loans Statement, currency transactions report, suspicious transactions report, details of any
information or data of account holders through online access to central database of the banking
company or any other information as required by the Board from the banking company;
(f) reporting banking company means a banking company required under section 165A of the Ordinance
to provide to the Board all the information and documents electronically or otherwise, mentioned in the
said section ;

Conceptual Approach to Taxes 413


Deduction / Payment of Tax Chapter-23

(g) Suspicious Transactions Report means suspicious transaction report generated and submitted by a
banking company to the Financial Monitoring Unit under the Anti-Money Laundering Act, 2010 (VII of
2010); and
(h) Written off Loans Statement means Written off Loans Statement as specified in Form C;

Furnishing of information [U/R 39C]


(1) The information, required to be furnished under section 165A if the Ordinance (other than information required
under clause (a) of sub-section (1) of section 165A) shall be provided, by the reporting banking company, in
the manner as specified in Account Holders Deposits Statement, Credit Card Payments and suspicious
transactions report.
(2) The information and online access required to be provided under clause (a) of sub-section (1) of section 165A,
shall be provided, by the reporting banking company, through online access to its central database containing
details of its account holders and all transactions made in their accounts.
(3) The information other than information provided I sub-rule (1) and (2), shall be provided by the reporting
banking company as specified in a notice issued in terms of section 165A(2) of the Ordinance,.
Authorized Persons [U/R 39D]
(1) Banking company officer, shall be nominated by the reporting banking company not later than thirty days of
coming into force of rules contained in this chapter.
(2) Where a banking company officer is not nominated within the time allowed as specified in sub-rule (1), the
President or any Principal Officer of the reporting banking company, stationed at the head officer, shall be
treated as banking company officer.
(3) The information required to be reported to the Board shall be provided by the banking company officer to the
Chairman, Federal Board of Revenue or any officer authorized by the Chairman in this behalf.
Provided that the officer authorized by the Chairman shall not be below the rank of Member of the Board.
Time of furnishing information [U/R 39E]
(1) Every banking company officer, shall furnish to the Board a monthly Account Holders Deposits Statement and
Credit Card Payments Statement as specified in Form A and Form B respectively, for immediately preceding
calendar month within seven days of the end of the preceding calendar month.
(2) Every banking company officer shall furnish to the Board an annual Written off Loans Statements as specified
in Form C for immediately preceding calendar year within three months of the end of the preceding calendar
year.
(3) Every banking company officer, shall furnish to the Board a copy of each currency transactions report and
suspicious transactions report generated by it at the time it is submitted to the Financial Monitoring Unit under
the Anti-Money Laundering Act, 2010.
(4) Every banking company officer, shall furnish to the Board any information and documents in addition to those
mentioned in sub-rules (1) or (3),within the time allowed by the Board.
Exclusions [U/R 39F]
(1) The information may not be provided by the banking company officer in respect of a person who holds
National Tax Number and has also filed return of income for the immediately preceding tax year.
(2) They information regarding the person mentioned in sub-rule (1) may be provided to the reporting banking
company by the Board, on 10th of every month.
20. Priority of tax collected or deducted [Section 166]
Tax collected by a person or deducted from a payment shall be-
(a) held by the person in trust for the Federal Government; and
(b) not subject to attachment in respect of any debt or liability of the person.
In the event of the liquidation or bankruptcy of a person who has collected or deducted tax from a payment the same
shall not form part of the estate of the person in liquidation or bankruptcy and the Commissioner Inland Revenue shall
have a first claim for that amount before any distribution of property is made.
Every amount that a person is required to deduct from a payment shall be -
(a) a first charge on the payment; and
(b) deducted prior to any other amount that the person may be required to deduct from the payment by virtue of
an order of any Court or under any other law.

414 Conceptual Approach to Taxes


Deduction / Payment of Tax Chapter-23

21. Indemnity [Section 167]


A person who has deducted tax from a payment and remitted the deducted amount to the Commissioner Inland
Revenue shall be treated as having paid the deducted amount to the recipient of the payment for the purposes of any
claim by the recipient for payment of the deducted tax.
22. Credit for tax collected or deducted [Section 168]
Where an amount of tax has been collected from a person or deducted from a payment made to a person, the person
shall be allowed a tax credit for that tax in computing the tax due by the person on the taxable income of the person
for the tax year in which the tax was collected or deducted.
No tax credit shall be allowed for any tax collected or deducted that is a final tax regime and a tax credit or part of a
tax credit allowed for a tax year that is not able to be credited for the year shall be refunded to the taxpayer.
No amount shall be deducted on account of service charges from the tax withheld or collected by any person,
however where the same are deducted the said person will be liable to pay the said amount to the Federal
Government and all the tax recovery provisions shall apply on the same.
23. Power to enter and search premises [Section 175]
(1) In order to enforce any provision of this Ordinance, the Commissioner or any officer authorised in writing by
the Commissioner for the purposes of this section
(a) shall, at all times and without prior notice, have full and free access to any premises, place, accounts,
documents or computer;
(b) may stamp, or make an extract or copy of any accounts, documents or computer-stored information to
which access is obtained under clause (a);
(c) may impound any accounts or documents and retain them for so long as may be necessary for
examination or for the purposes of prosecution;
(d) may, where a hard copy or computer disk of information stored on a computer is not made available,
impound and retain the computer for as long as is necessary to copy the information required; and
(e) may make an inventory of any articles found in any premises or place to which access is obtained under
clause (a).
(2) The Commissioner may authorize any valuer or expert to enter any premises and perform any task assigned
to him.
(3) The occupier of any premises or place to which access is sought above shall provide all reasonable facilities
and assistance for the effective exercise of the right of access.
(4) Any accounts, documents or computer impounded and retained as above shall be signed for by the
Commissioner or an authorised officer.
(5) A person whose accounts, documents or computer have been impounded and retained as above may
examine them and make extracts or copies from them during regular office hours under such supervision as
the Commissioner may determine.
(6) Where any accounts, documents or computer impounded and retained as above are lost or destroyed while in
the possession of the Commissioner, the Commissioner shall make reasonable compensation to the owner of
the accounts, documents or computer for the loss or destruction.
(7) This section shall have effect notwithstanding any rule of law relating to privilege or the public interest in
relation to access to premises or places, or the production of accounts, documents or computer-stored
information.
(8) In this section, occupier in relation to any premises or place, means the owner, manager or any other
responsible person on the premises or place.
24. Notice to obtain information or evidence [Section 176]

(1) The Commissioner may, by notice in writing, require any person, whether or not liable for tax under this
Ordinance
(a) to furnish to the CIR or an authorized officer, any information relevant to any tax leviable under this
Ordinance or to fulfill any obligation under any agreement with foreign government or governments
or tax jurisdiction, as specified in the notice;; or
(b) to attend at the time and place designated in the notice for the purpose of being examined on oath by
the Commissioner or an authorised officer concerning the tax affairs of that person or any other person
and, for that purpose, the Commissioner or authorised officer may require the person examined to
produce any accounts, documents, or computer-stored information in the control of the person ; or

Conceptual Approach to Taxes 415


Deduction / Payment of Tax Chapter-23

(c) the firm of chartered accountants, as appointed by the Board or the Commissioner, to conduct audit u/s
177, for any tax year, may with the prior approval of the Commissioner concerned, enter the business
premises of a taxpayer, to obtain any information, require production of any record, on which the
required information is stored and examine it within such premises; and such firm may if specifically
delegated by the Commissioner, also exercise the powers as provided in sub-section (4).
(1A) A special audit panel appointed under sub-section (11) of section 177, for any tax year, may, with the
prior approval of the CIR concerned, enter the business premises of a taxpayer, to obtain any
information, require production of any record, on which the required information is stored and examine it
within such premises and such panel may if specifically delegated by the CIR, also exercise the powers
as provided in sub-section (4).
(2) The Commissioner may impound any accounts or documents produced as above and retain them for so long
as may be necessary for examination or for the purposes of prosecution.
(3) The person from whom information is required, may at his option, furnish the same electronically in any
computer readable media.] Where a hard copy or computer disk of information stored on a computer is not
made available as required above, the Commissioner may require production of the computer on which the
information is stored, and impound and retain the computer for as long as is necessary to copy the information
required.
(4) For the purposes of this section, the Commissioner shall have the same powers as are vested in a Court, in
respect of the following matters, namely:
(a) enforcing the attendance of any person and examining the person on oath or affirmation;
(b) compelling the production of any accounts, records, computer-stored information, or computer;
(c) receiving evidence on affidavit; or
(d) issuing commissions for the examination of witnesses.
(5) This section shall have effect notwithstanding any law or rules relating to privilege or the public interest in
relation to the production of accounts, documents, or computer-stored information or the giving of information.
25. Receipts for amounts paid [Section 220]
The Commissioner Inland Revenue shall give a receipt for any tax or other amount paid or recovered under this
Ordinance.

26. Liquidators [Section 141]


Every person who is a liquidator of a company, a receiver appointed by a Court or appointed out of Court, a trustee
for a bankrupt; or a mortgagee in possession, shall, within 14 days of being appointed or taking possession of an
asset in Pakistan, whichever occurs first, give written notice thereof to the Commissioner Inland Revenue.
The Commissioner Inland Revenue shall, within 3 months of being notified, notify the liquidator in writing of the
amount which appears to the Commissioner Inland Revenue to be sufficient to provide for any tax which is or will
become payable by the person whose assets are in the possession of the liquidator.
A liquidator shall not without leave of the Commissioner Inland Revenue, part with any asset held as
liquidator until the liquidator has been notified as above.
A liquidator -
(a) shall set aside, out of the proceeds of sale of any asset by the liquidator, the amount notified by the
Commissioner Inland Revenue, or such lesser amount as is subsequently agreed to by the Commissioner
Inland Revenue;
(b) shall be liable to the extent of the amount set aside for the tax of the person who owned the asset; and
(c) may pay any debt that has priority over the tax as stated.
A liquidator shall be personally liable to the extent of any amount required to be set aside for the tax as above to
the extent that, the liquidator fails to comply with the said requirements.
27. Recovery of tax due by non-resident member of an AOP [Section 142]
The tax due by a non-resident member of an AOP in respect of the member's share of the profits of the association
shall be assessable and recovered out of the assets of the association or from the resident member personally.
A person making a payment shall be treated as acting under the authority of the non-resident member and shall be
indemnified in respect of such payments even there is anything in contrary in any written law, contract or
agreement.
The provisions of this Ordinance shall apply to any amount due under this section as if it were tax due under an
assessment order.

416 Conceptual Approach to Taxes


Deduction / Payment of Tax Chapter-23

28. Recovery of tax from persons assessed in Azad Jammu and Kashmir [Section 146]
Where any person assessed to tax for any tax year under the law relating to income tax in the Azad Jammu and
Kashmir has failed to pay the tax and the income tax authorities of the Azad Jammu and Kashmir cannot recover the
tax because -
(a) the person's residence is in Pakistan; or
(b) the person has no movable or immovable property in the Azad Jammu and Kashmir,
the Deputy Commissioner Inland Revenue in the Azad Jammu and Kashmir may forward a certificate of recovery to
the Commissioner Inland Revenue and, on receipt of such certificate, the Commissioner Inland Revenue shall
recover the tax.
A certificate of recovery under as above shall be in the prescribed form specifying -
(a) the place of residence of the person in Pakistan;
(b) the description and location of movable or immovable property of the person in Pakistan; and
(c) the amount of tax payable by the person.
29. Initiation, validity, etc., of recovery proceedings [Section 146A]
Any proceedings for the recovery of tax may be initiated at any time. The Commissioner Inland Revenue may, at any
time, amend the certificate issued, or recall such certificate and issue fresh certificate, as he thinks fit.
It shall not be open to a taxpayer to question before the District Officer (Revenue) the validity or correctness of any
certificate issued or any such certificate as amended or any fresh certificate.
The several modes of recovery shall be deemed to be neither mutually exclusive nor affect in any way any other law
for the time being in force relating to the recovery of debts due to the Government and the Commissioner Inland
Revenue may have recourse to any such mode of recovery notwithstanding that the tax due is being recovered from
a taxpayer by any other mode.
30. Tax arrears settlement incentives scheme [Section 146B]
The Board may make scheme in respect of recovery of tax arrears or withholding taxes and waiver of default
surcharge or penalty levied thereon. The Board may make rules u/s 237 for implementation of such scheme.
31. Refunds [Section 170 and Rule 71]
A taxpayer who has paid tax in excess of the amount which the taxpayer is liable to pay under ITO, 2001 may apply
to the Commissioner Inland Revenue for a refund of the excess.
Where any advance or loan u/s 2(19)(c) is repaid by a taxpayer, he shall be entitled to a refund of the tax, if any, paid
by him as a result of such advance or loan having been treated as dividend as stated above.
An application for a refund as above shall be -
(a) made in the prescribed form;
(b) verified in the prescribed manner; and
(c) made within two years of the later of -
(i) the date on which the Commissioner Inland Revenue has issued the assessment order to the taxpayer
for the tax year to which the refund application relates; or
(ii) the date on which the tax was paid
Where the Commissioner Inland Revenue is satisfied that tax has been overpaid, the Commissioner
Inland Revenue shall-
(a) apply the excess in reduction of any other tax due from the taxpayer under this Ordinance;
(b) apply the balance of the excess, if any, in reduction of any outstanding liability of the taxpayer to
pay other taxes; and
(c) refund the remainder, if any, to the taxpayer.
The Commissioner Inland Revenue shall, within 60 days of receipt of a refund application as stated above serve on
the person applying for the refund an order in writing of the decision after providing the taxpayer an opportunity of
being heard.
A person aggrieved by-
(a) an order passed as above; or
(b) the failure of the Commissioner Inland Revenue to pass an order within the time specified as above, may
prefer an appeal.

Conceptual Approach to Taxes 417


Deduction / Payment of Tax Chapter-23

Refund Application [Rule 71]


An verified application u/s 170 for a refund of tax shall be in the Performa specified in the Income Tax Rules, 2002
and shall be accompanied by such documents, statements and certificates as specified in the form, in the Ordinance
and Income Tax Rules, 2002.
Additional payment for delayed refunds [Section 171]
Where a refund due to a taxpayer is not paid within 3 months of the date on which it becomes due, the
Commissioner Inland Revenue shall pay to the taxpayer a further amount by way of compensation @ Kibor plus
0.5% per annum of the amount of the refund computed for the period commencing at the end of the 3 months period
and ending of the date on which it was paid.
Provided that where there is reason to believe that a person has claimed the refund which is not admissible to him,
the provision regarding the payment of such additional amount shall not apply till the investigation of the claim is
completed and the claim is either accepted or rejected.
A refund shall be treated as having become due-
(a) in case of a refund required to be made in consequence of an order on an appeal to the Commissioner Inland
Revenue (Appeals), an appeal to the Appellate Tribunal Inland Revenue, a reference to the High Court or an
appeal to the Supreme Court, on the date of receipt of such order by the Commissioner Inland Revenue; or
(b) in case of refund required to be made as consequence of revision order u/s 122A, on the date order is made
by the Commissioner Inland Revenue ; or
(c) in any other case, on the date the refund order is made.
Explanation:- For the removal of doubts, it is clarified where a refund order is made on an application under sub-
section (1) of section 170, for the purpose of compensation, the refund becomes due from the date refund order is
made and not from the date the assessment of income treated to have been made by the Commissioner under
section 120.
32. Reduction in default surcharge, consequential to reduction in tax or penalty [Section 205A]
Where, in consequence of any order made under this Ordinance, the amount of tax or penalty in respect of which
default surcharge chargeable is reduced, the default surcharge, if any, levied under the aforesaid section shall be
reduced accordingly.
REGISTRATION OF INCOME TAX PRACTITIONERS

33. Application of Chapter XIV [Rule 84] This chapter applies for the purposes of section 223, which provides for the
registration and regulation of income tax practitioners.
34. Application for registration as an income tax practitioner [Rule 85]
(1) A person satisfying the requirements in rule 86 and desiring to be registered as an income tax practitioner
shall make an application to the Director-General, Regional Tax Office in the form specified in Part X of the
First Schedule to these rules.
(2) Every application under this rule shall be accompanied by -
(a) a Treasury receipt for five hundred rupees required to be deposited as a non-refundable application fee
in any Government Treasury; and
(b) such documents, statements and certificates as specified in the form.
35. Prescribed qualification for registration as an income tax practitioner [Rule 86]
(1) For the purposes of the definition of "income tax practitioner" in section 223(11), a person applying for
registration as an income tax practitioner shall:-
(a) possess one of the following qualifications, namely:-
(i) a degree in Law at least in the second division, a degree in Commerce (with Income Tax Law and
Accounting or Higher Auditing as subjects or parts of subjects, whether compulsory or optional)
or a degree in Business Administration or Business Management (with Accounting and Income
Tax law as subject or parts of subjects, whether compulsory or optional) conferred by a
prescribed institution; or
(ii) a pass in a prescribed accounting examination.
(b) have worked for a continuous period of one year as an apprentice under the supervision of a chartered
accountant, cost and management accountant, legal practitioners entitled to practice in a civil court in
Pakistan or a registered income tax practitioner and having been registered as a chartered accountant,
cost and management accountant, legal practitioner and income tax practitioner for a period of not less
than 10 years.

418 Conceptual Approach to Taxes


Deduction / Payment of Tax Chapter-23

(2) For the purposes of sub-clause (i) of clause (a) of sub-rule (1), a degree conferred by a prescribed institution
that is a foreign university or institution shall only qualify if the degree is equivalent to a degree conferred by a
Pakistani university and is recognized as such by a Pakistani university.
(3) In this rule,
(a) "Institute of Chartered Accountants of Pakistan" means the Institute of Chartered Accountants of
Pakistan constituted under the Chartered Accountants Ordinance, 1961;
(b) "foreign institution" means any institution in a foreign country authorized to grant a degree under the
laws of the country;
(c) "foreign university" means any university in a foreign country incorporated by law, or accredited or
affiliated by any association of universities or college in the country or by any authority formed for that
purpose under the laws of that country;
(d) "prescribed accounting examination" means any of the following examinations, namely:-
(i) an examination equivalent to the intermediate examination conducted by the Institute of
Chartered Accountants of P
(ii) an examination equivalent to the intermediate examination conducted by any foreign institute of
chartered accountants and recognised by the Institute of Chartered Accountants of Pakistan as
equivalent to its intermediate certificate;
(iii) an examination equivalent to the final examination conducted by the Association of Chartered
Certified Accountants, United Kingdom; or
(iv) Part-III of examination for Cost and Management Accountants conducted by the Institute of Cost
and Management Accountants under the Cost and Management Accountants Act, 1966;and
(v) Certified public accountants of USA.
(e) "prescribed institution" means a university incorporated by any law in force in Pakistan or Azad
Kashmir, a foreign university or a foreign institution.
36. Registration of income tax practitioners [Rule 87]
On receipt of an application under rule 85, the Director General, Regional Tax Office may make such further
enquiries and call for such further information or evidence as may be considered necessary.
If the Director General, Regional Tax Office is satisfied that an applicant qualifies to be registered as an
income tax practitioner, the Director General, Regional Tax Office shall cause the applicant's name to be
entered in a register to be maintained for the purpose in the office.
The name of a person entered on the register of income tax practitioners shall be notified to the Commissioner
and the Appellate Tribunal.
The Director General, Regional Tax Office shall notify the applicant, in writing, of the decision on the
application.
Where the Director General, Regional Tax Office decides to refuse an application for registration, the notice
referred above shall include a statement of reasons for the refusal.
37. Duration of registration [Rule 88] Registration of a person as an Income Tax Practitioner shall remain in force until
any of the following occurs, namely;-
the person surrenders the registration by notice in writing to the Director General, Regional Tax Office.
the person dies; or
the person's registration is terminated by the Director General, Regional Tax Office.
38. Cancellation of registration [Rule 89]
(1) Any person (including an income tax authority) who considers that an income tax practitioner is guilty of
misconduct in a professional capacity may file a complaint in writing with the Director General, Regional Tax
Office.
(2) A complaint filed under sub-rule (1) shall be accompanied by affidavits and other documents as necessary to
sustain the complaint.
(3) On receipt of a complaint in writing under sub-rule (1), the Director General, Regional Tax Office shall fix a
date. hour and place which shall be no later than 21 days from the receipt of the complaint for enquiry into the
complaint.
(4) Within 7 days of receipt of the complaint, the Director General, Regional Tax Office shall serve a notice of the
complaint on the Income Tax Practitioner to whom the complaint relates and such notice shall -

Conceptual Approach to Taxes 419


Deduction / Payment of Tax Chapter-23

(a) inform the practitioner of the date, hour and place of the enquiry; and
(b) be accompanied by a copy of the complaint and any affidavits and other documents accompanying the
complaint.
(5) If, at the date fixed for enquiry, it appears that the notice and accompanying documents referred to in sub-rule
(4) have not been served as provided for in that sub-rule, the Director General, Regional Tax Office shall
adjourn the enquiry to a date then to be fixed and may direct that the notice and accompanying documents
shall be served by registered post or such other means as the Director General, Regional Tax Office sees fit.
(6) Not less than two days before the date or adjourned date fixed for the enquiry, the income tax practitioner
concerned shall file with the Director General, Regional Tax Office a signed explanation in writing and any
affidavit in reply intended to be used in the enquiry.
(7) On the date or adjourned date of the enquiry, the complainant shall file time to affidavits in reply intended to be
used at the enquiry.
(8) The Director General, Regional Tax Office may adjourn the enquiry from time to time to a date and place to be
fixed at the time of adjournment and may make such orders and give such directions in regard to the enquiry
and all matters relating thereto as the Director General, Regional Tax Office may think fit.
(9) On the date or adjourned date fixed for the enquiry, the Director General, Regional Tax Office may-
(a) hear and determine the complaint upon the affidavit and other documents, if any, filed and may allow
the complainant and income tax practitioner to be cross-examined on their affidavits; or
(b) hear and determine the complaint upon oral evidence.
(10) If the Director General, Regional Tax Office decides to hear oral evident, the procedure generally and as far as
practicable shall be that which is followed at the hearing of suits by Civil Courts, provided that the record of
oral evidence shall be kept in such manner as the Director General, Regional Tax Office may direct and, if a
shorthand writer is employed to take down evidence, the transcript of the writer's notes shall be a record of
deposition of the witnesses,
(11) If the Director General, Regional Tax Office decides that the income tax practitioner to whom the complaint
relates is guilty of professional misconduct, the Director General, Regional Tax Office shall cancel the
practitioner's registration.
(12) The Director General, Regional Tax Office shall give the complainant and the income tax practitioner to whom
the complaint relates notice, in writing, of the Director General, Regional Tax Office's decision on the
complaint.
39. Appeal to Federal Board of Revenue [Rule 90]
(1) The appeal against the decision of the Director General, Regional Tax Office shall lie with the FBR. However,
the FBR on filing of an appeal may, pending decision of appeal, allow the ITP to represent cases pending,
before decision is made by the Director General.
(2) The FBR shall decide the case of the ITP within 60 days of the filing of the appeal.

420 Conceptual Approach to Taxes


Deduction / Payment of Tax Chapter-23

DEDUCTION OR COLLECTION OF TAX AT SOURCE FOR THE TAX YEAR 2013


(UNDER PART - V OF CHAPTER - X OF INCOME TAX ORDINANCE, 2001)

IMPORTANT NOTES:

1. The tax deducted or collected by the Federal Government or Provincial Government shall be deposited U/R 43(a) on the day the tax was collected or
deducted AND U/R 43(b) within 7 days in other cases from the end of each week ending on every Sunday.

2. The tax deducted or collected under section 236M and 236N other than by the companies quoted on stock exchange and not quoted on stock
exchange respectively shall be deposited within fifteen days of the 1st day of the closure of the books.

3. The advance tax under this chapter (covering sections 231A to 236T) shall not be collected in the case of withdrawals made by,-
(a) the Federal Government or a Provincial Government;
(b) a foreign diplomat or a diplomatic mission in Pakistan; or
(c) a person who produces a certificate from the Commissioner that his income during the tax year is exempt.

SECTIO
N NATURE OF PAYMENT/ EXEMPTION STATUS OF DEDUCTION / TIME OF
TRANSACTION TAX RATE LIMIT PAYMENT OR COLLECTING DEDUCTION /
DEDUCTION AUTHORITY COLLECTION
1 2 3 4 5 6 7

FILER NON-FILER
148 Collector of At the same time
A foreign produced film imported for the 12% of value of customs and manner as the
custom duty is
purposes of screening and viewing goods assessed Nil Adjustable
payable in respect
of the goods
imported

i. Industrial undertaking importing re-


Tax required to be
meltable steel and directly reduced iron 1% of the 1.5% of
collected at import
for its own use; import the import
stage shall be final
value value as
ii. Persons importing potassic fertilizers in tax on the income of
increased increased
pursuance of Economic coordination importer arising
by Custom by
Committee of the cabinets decision No. from import. It will
duty, customs-
ECC-155/12/2004 dated the 9th be adjustable in
sales tax duty, sales
December, 2004; Nil case of import of: do do

Conceptual Approach to Taxes 421


Deduction / Payment of Tax Chapter-23

iii. Persons importing urea; and and tax and (a) Raw material,
federal federal plant,
iv. Manufacturers covered under excise excise machinery,
Notification No. S.R.O. 1125(I)/2011 duty. duty. equipment and
dated 31-12-2011 parts by an do do
v. Persons importing Gold; and industrial
undertaking for
its own use; do do
iv. Persons importing Cotton (b) Fertilizer by
manufacturer of do do
iv. Designated buyers of LNG on behalf fertilizer;
of Govt of Pakistan to import LNG (c) Cars in CBU
condition by
manufacturer of do do
Persons importing pulses 2% of the 3% of the cars; and
import import (d) Large import
value as value as house;
increased increased Tax required to be
by by custom- collected on the
Custom- duty, sales import of edible oil
duty, sales tax and and packing
tax and federal
material shall be
federal excise
excise duty duty. Nil
minimum tax.
Commercial importers covered under 3% of 4.5% of the
Notification No. S.R.O. 1125(I)/2011 dated import import
31-12-2011. value as value as
increased increased
by custom- by custom-
duty sales duty , sales
tax and tax and
federal federal
excise excise duty
duty.
Nil do do
Ship breakers on import of ships 4.50% 6.5 Nil do do
Industrial undertakings not covered under S. 5.50% 8%
Nos. 1 to 4
Nil
Companies not covered under S. Nos. 1 to 5 5.50% 8%
Nil do do
Persons not covered under S. Nos. 1 to 6 6% 9% Nil

422 Conceptual Approach to Taxes


Deduction / Payment of Tax Chapter-23

Important Notes:

1. "Large import house" means who fulfill all the conditions (i) have paid up capital of exceeding Rs. 250 (M) (ii) have imports exceeding Rs. 500 million during
the Tax year (iii) own total assets exceeding Rs. 350(M). (iv) is single object company; (v) maintain computerized records of import and sale of goods (vi)
maintains a system for issuance of 100% cash receipts on sales. (vii) present accounts for tax audit every year. (viii) is registered under the Sales Tax Act,
1990. (ix) makes sales of industrial raw material of manufacturer registered under the Sales Tax Act, 1990.

2. "Value of goods" means the value of the goods as determined under the Customs Act, 1969, as if the goods were subject to ad valourem duty increased by
the custom duty, federal excise duty and sales tax, if any, payable in respect of the import of goods.

3. "Edible oil" includes crude oil, imported as raw material for manufacture of ghee or cooking oil.

4. The Board may specify any goods or class of goods or person or class of persons importing such goods on which tax shall not be collected at import
stage.[148(2)]
5. The Finance Act, 2015 withdrew" [by omitting section 148(2)] the FBR's power to specify any goods or class of goods or person or class of persons importing
such goods on which tax shall not be collected at import stage.
6. Notwithstanding omission of section 148(2), the notifications issued under that sub-section and for the time being in force, shall continue to remain in force,
unless specifically rescinded through a notification issued for this purpose.
7. Tax required to be collected on import of edible oil and packing material for a tax year shall be the minimum tax. [148(8)]
8. Tax collected at the time of import of ships by ship-breakers shall be final tax. [148(8A)]
9. The manufacturer of cooking oil or vegetable ghee shall be chargeable to tax @ 2% on purchase of locally produced edible oil. Tax so charged shall be final
tax in respect of income accruing from locally produced edible oil.

NON APPLICABILITY OF PROVISIONS REGARDING TAX AT SOURCE U/S 148 U/C 56 and 60 OF PART IV OF SECOND SCHEDULE TO THE ITO, 2001:

1. U/C 56(i) Goods classified under PCT falling under chapters 86 and 99 except PCT Heading 9918.
(ia) Petroleum oils and oils obtained / import made by various petroleum products distribution companies (including oil refineries).
(ii) Goods imported by direct and indirect exporters covered under SRO 450(I)/2001 dated June 18, 2001.
(iii) Goods temporarily imported into Pakistan for subsequent exportation and which are exempt from custom duty and sales tax under SRO
1065(I)/2005 dated October 20, 2005.
(iv) Manufacturing Bond as prescribed under SRO 450(I)/2001 dated June 18, 2001.
(v) mineral oil imported by a manufacturer or formulator of pesticides which is exempt from customs duties under customs SRO 857/(I)/2008 dated
August 16, 2008.

2. U/C 60 Fully as well part designed / assembled cypher devices, for use within the country as are verified by Cabinet Division (NIBS) with reference to design,
quality and quantity.

Conceptual Approach to Taxes 423


Deduction / Payment of Tax Chapter-23

Section NATURE OF PAYMENT/ EXEMPTION STATUS DEDUCTION / TIME OF


TRANSACTION TAX RATE LIMIT PAYMENT OR COLLECTING DEDUCTION /
DEDUCTION AUTHORITY COLLECTION
1 2 3 4 5 6 7
149 Salary Annual average rate Estimated Adjustable Person responsible for Payment
annual salary
of tax Rs. 400,000 payment

Directorship fee for attending board meetings etc. 20% nil Same as above Same as above Same as above

IMPORTANT NOTES:

The employer while deducting tax at source, authorized to make necessary adjustment for any excess deduction or deficiency arising out of any previous deduction or failure to
make a deduction during the year and tax credits available under section 61 to 64 and tax at source on motor vehicle u/s 234, telephone bills, cash withdrawal from banks and
registration of new car.
Tax Shall be deducted on the gross amount of
150 dividend paid. For filers is 12.5% Nil Final discharge Person paying dividend Payment
For non filers is 17.5%

Money
Market Fund,
Income Fund,
REIT Scheme
Collective investment, REIT Scheme or mutual Stock or any other
funds. Fund fund Nil do do do
Individual 10% 10%
Company 10% 25%
AOP 10% 10%

Declared by purchaser of a power 7.5% Nil do do do


project privatized by WAPDA

Declared by a company setup for 7.5% Nil do do do


power generation

Dividend on shares of a company supplying coal 7.5% Nil do do do


exclusively to power generation process

in case of stock fund if dividend receipts of fund


are less than capital gains 12.5% Nil do do do

424 Conceptual Approach to Taxes


Deduction / Payment of Tax Chapter-23

IMPORTANT NOTES:

Every person (including non- resident company) paying a dividend shall deduct tax from the gross amount of the dividend paid at the rate specified. Tax deducted @
10% under this section will be final discharge of liability, whereas in the case of companies, it will be adjustable.
Rate of tax on dividend received from such a Developmental REIT Scheme which is set up by 30-06-2018 with the object (development and construction of residential
buildings shall be reduced by 50% for 3 years from 30-06- 2018. (Part-III Division I of First Schedule).

NON APPLICABILITY OF PROVISIONS OF TAX AT SOURCE U/S 150 UNDER VARIOUS CLAUSES OF PART IV OF 2ND SCHEDULE TO THE ITO, 2001:
U/C 11C The provisions of this section are not applicable to inter-corporate dividend within the group companies entitled to Group Taxation.
U/C 38A A Venture Capital Company.
U/C 38B Islamic Development Bank.
U/C 47B Any person making payment to NIT, a collective Scheme, a modaraba, approved pension fund, approved income payment plan, a REIT Scheme, a private
equity and venture capital fund, a recognized provident fund, and approved superannuation fund or an approved gratuity fund.

151 Profit on debt


Final discharge
Profit on debt (Except received by Payment or
(a) Filers (non a company or credit of profit
filers upto profit taxable u/s 7B) Payer to the account
1) Yield or profit (profit on debt) on account, deposit or a of Rs. 500,000) Nil Payment or
certificate under the National saving schemes or Post 10% of gross Government (Federal, credit of profit
office savings account. yield paid Provincial or Local) to the account

(b) Other Non


2) Profit on debt paid by Banking company or financial filers 17.5% of Nil
institution on account or deposit maintained. gross yield paid. do Do Do

3) Profit on securities , other than those mentioned in s. Nil do


151(1)(a), issued by federal / provincial government or a Banking company or
local government Financial institution do
4) Profit on bonds , certificates, debentures, securities or
instruments of any kind (other than loan agreements
between borrowers and banking companies or
development financial institutions) Nil do Payer Do

IMPORTANT NOTES:

This section is not applicable to any profit on debt u/s 152(2).

Conceptual Approach to Taxes 425


Deduction / Payment of Tax Chapter-23

Tax deducted under this section shall be final tax on profit on debt arising to taxpayer other than a company or a profit on debt that is taxable u/s 7B.
The rate of tax for a non filer shall be 10% if the yield or profit paid is equal or less than Rs. 500,000.
Tax rate is applied on the gross amount of Yield as reduced by Zakat paid.

NON APPLICABILITY OF PROVISIONS OF TAX AT SOURCE U/S 151 UNDER VARIOUS CLAUSES OF PART IV OF SECOND SCHEDULE TO THE ITO, 2001:

U/C 36A Any amount paid as yield or profit on investment in Bahbood Saving Certificates or Pensioners Benefit Account.
U/C 38 Special purposes vehicle (SPV)for the purposes of securitization.
U/C 38A A Venture Capital Company.
U/C 38C Islamic Development Bank
U/C 47B Any person making payment to NIT, a collective Scheme, a modaraba, approved pension fund, approved income payment plan, a REIT Scheme, a private
equity and venture capital fund, a recognized provident fund, and approved superannuation fund or an approved gratuity fund.
U/C 59(i) Profit or interest paid on TFC's issued by Prime Minister's Housing Development Company (Pvt.) Ltd. (PHDCL).
U/C 59(ii) Any payment as profit or interest on Prime Minister's Housing Development Company which has been issued on or after 01-07-1999.
U/C 59(iii) Income of a resident individual from Saving Accounts scheme of Directorate of National Savings, if monthly installment is up to Rs. 1,000.
U/C 72 Payments made to The ECO Trade and Development Bank.

15% of the gross Final discharge u/s


152(1) Royalty and fee for technical services amount Nil 8 Payer Payment
to non-resident paid

6% of gross
152(1A) Payment to non-resident for amount paid Nil Final discharge do do
construction, services or
advertisement control.

5% of gross
152(1AA) Payment of insurance premium amount paid Nil Final discharge do do
or re-insurance premium

152(1AA 10% of the gross


A) Advertisement services relaying from outside Pakistan amount paid Nil Final discharge do do

10% of the gross


152(2) Payment to non resident having PE in amount Nil Final discharge do do
Pakistan, in respect of profit on debt u/c paid
5A of Part II of Second Schedule

152(2) All other payment to non-resident 20% of the gross Nil Adjustable do do

426 Conceptual Approach to Taxes


Deduction / Payment of Tax Chapter-23

amount
paid
Non-
Filer Filer
152(2A)(i) Payment by way of advance to a permanent establishment
in Pakistan of a non-resident person
i) For sale of goods At the time of
In case of company 4% 6% Prescribed person making making the
In any other case 4.5% 6.5% Rs. 25,000 FTR / Adjustable the payment payment

152(2A)(ii) Rendering of services:

ii) Transport services 2% of the gross Rs. 10,000 Minimum tax do do


amount p.a.
Non-
Filer Filer
iii) All other services
(i) In the case of Companies 8% 12% Same as
(ii) In all other than Company Taxpayers 10% 15% above Minimum tax do do

Non-
Filer Filer
iv) Execution of contracts:
i) In case of sport person 10% 10%
ii) In the case of Companies : 7% 10% Same as
152(2A)(iii) iii) In the case of other than companies Taxpayers 7.5% 10% above FTR do do

IMPORTANT NOTES

IMPORTANT NOTES:

1. Any payment made to a non resident by a person who is liable to tax as agent of such non resident shall not be liable to deduction under section 152(3).
2. Before making payment, the agent shall file a declaration that he is the agent of the non resident.
3. On payments that are subject to deduction of tax under section 149, 150, 153, 156 or 233
4. With the written approval of the CIR, that is taxable to PE in Pakistan of the non resident person.
5. Any payment made to a non resident by a person who is liable to tax as agent of such non resident shall not be liable to deduction u/s 152(3)
6. Where the non-resident person is not chargeable to tax in respect of the income.
7. Before making a payment, the agent shall file a declaration that he is the agent of the non -resident.

Conceptual Approach to Taxes 427


Deduction / Payment of Tax Chapter-23

8. Where a person intends to make a payment to a non resident person without deduction of tax under this section (other than payments liable to reduced rate
under relevant agreement for avoidance of double taxation/ the person shall before making the payment, furnish to the CIR a notice in writing by stating
therein the name and address of the non-resident person and the nature and amount of the payment.
9. The commissioner may, on application made by the recipient of the payment referred above and after making such an inquiry as he thinks fit, may allow in the
cases where the tax deductible as above is adjustable, by order in writing, any person to make the payment, without deduction of tax or deduction of tax at a
reduced rate. [152(4A)]
10. The information required in note 6 above shall not apply to a payment on account of an import of goods where title to the goods passes outside Pakistan
and is supported by import documents, except an import that is part of an overall arrangement for the supply of goods, their installation, and any commission
& guarantees in respect of the supply where

a. the supply is made by the head office outside Pakistan of a person to a PE of the person in Pakistan;
b. the supply is made by a PE of the person outside Pakistan to a PE of the person in Pakistan;
c. the supply is made between associates; or the supply is made by a resident person or a Pakistan PE or a non-resident person; or educational and
medical expenses remitted in accordance with the regulations of the SBP.
(U/C 5A of Part-II of Second Schedule) the rate of tax to be deducted u/s 152(2), in respect of payment from profit from debt payable to a non resident person having
no PE in Pakistan, shall be 10% of the gross amount paid.
Provided that the tax deducted on profit on debt on instrument, government securities including treasury bills and Pakistan Investment Bonds shall be final tax on profit
on debt payable to a non resident person having no permanent establishment in Pakistan and the investments are exclusively made through a special rupees
convertible account maintained with the bank in Pakistan.
NON APPLICABILITY OF PROVISIONS OF TAX AT SOURCE U/S 152 UNDER VARIOUS CLAUSES OF PART IV OF 2ND SCHEDULE TO THE ITO, 2001:

U/C 41 Non resident person unless he opts for FTR.


Provided that the declaration of option is furnished in writing within three months of the commencement of the tax year and such declaration shall be irrevocable and
shall remain in force for three years.

153 Sales of goods:

Rs. 25,000 See important


153(1)(a) a) sales of rice, cotton seed or 1.5% of the gross p.a. notes Federal Govt. Payment
edible oil. amount paid attached company, AOP, Foreign
Non-
Filer Filer contractor, consultants,
b) Sales of any other goods
In case of companies 4% 6% Same as
the case of other than companies Taxpayers 4.5% 6.5% above consortium or joint venture do
and a person registered
under Sales Tax Act, 1990

428 Conceptual Approach to Taxes


Deduction / Payment of Tax Chapter-23

Reduced tax rates under Part II


of second schedule

Same as
a) Sales of edible oils to manufacturers 2% of the gross above do do
of cooking oil or vegetable ghee U/C amount.
13C

b) Tax from distributors of cigarettes and pharmaceutical 1% of the gross Same as


products and large import house U/C 24A amount above do

153(1)(b) Rendering of services:

a) Transport services 2% of the gross Rs. 10,000 do do


amount p.a.
Non-
Filer Filer
b) All other services
(iii) In the case of Companies 8% 12%
(iv) In all other than Company Taxpayers 10% 15% Same as
(v) Electronic & Print Medial for advertising services 1% 12% above do do

Filer Non Same as


153(1)(c) Execution of contracts: Filer above do do do
i) In case of sportsperson 10% 10%
ii) In the case of Companies : 7% 10%
iii) In the case of other than companies Taxpayers 7.5% 10%

Same as
153(2) Payments made by every exporter or 1% of the gross above do Every exporter or do
an export house to a resident person amount export house
or PE in Pakistan on account of
rendering of services of stitching,
dying etc u/c 13 of Part III of Ist
Schedule.

Conceptual Approach to Taxes 429


Deduction / Payment of Tax Chapter-23

IMPORTANT NOTES:

1.
Prescribed person making a payment may be a part, full or advance payment.
2.
Supply of goods making payments on account of goods by the payer.
3.
All exporters of goods making payments on account of goods exported are exempted from deduction of tax.
4.
Tax at source shall be deducted from gross amount of sale of goods.
5.
Provision of this section is not applicable to the following:
(i) Sales made by importer of the goods if he has paid the tax u/s 148 at the time of the import and the goods are sold in the same condition.
(ii) Payments to traders of yarn specified in the zero-rated regime of sales tax
(iii) refund of any security deposit
(iv) Payment made by the government or a local government to a contractor for construction materials supplied to the contractor by the
government or local government
(v) A cotton ginner who deposits in the government treasury, an amount equal to the amount of the tax deductible on the payment being made to
him
(vi) Purchase (by a modaraba, leasing company, banking company or financial institution) of an asset under a lease and buy back agreement.
(vii) Any payment for securitization of receivables by a SPV to the originator.
(viii) PE of a non-resident person.
6. Tax deducted u/s 153 shall be final tax on the income of a resident person from the following transactions:
(i) Sale of goods
(ii) Rendering or providing of service
(iii) Execution of contracts and
(iv) Rendering of services of stitching, dying, printing, embroidery, washing, sizing and weaving to an exporter or export house
7. For companies rendering or providing of services, tax deducted shall not be treated as final tax.
8. Tax deducted u/s 153 shall not be treated as final tax in respect of payment received on account of:
(i) Advertisement services, by owner of newspaper and magazines.
(ii) Sale of goods and execution of contracts by a public company listed on registered stock exchange in Pakistan; and the rendering or providing
services.
(iii) Tax deducted while making payment for services u/s 153(1)(b) shall be the minimum tax.
EXEMPTIONS FROM SPECIFIC PROVISIONS U/S 153 UNDER VARIOUS CLAUSES OF PART IV OF SECOND SCHEDULE TO THE INCOME TAX ORDINANCE,
2001:

Sr. UNDER Exemption from specific provisions


CLAUSE

1. 9A Steel melters, steel re-rollers etc.


2. 9AA Ship breakers
3. 16 Non application of minimum tax and other provisions on institutions of the Agha Khan
4. 38 Special purpose vehicle (SPV) for the purposes of securitization.
5. 42 Payments received by a resident person for providing services by way of operation of container or chemical or oil terminal at a sea port in Pakistan.
6. 43A Payments received by a person against supply of petroleum products imported by him under the Govt. deregulation policy of POL products.
7. 43B Payments received on sale of air tickets by travelling agents, who have paid withholding tax on their commission income.
8. 43C Payments received on supply of petroleum products.

430 Conceptual Approach to Taxes


Deduction / Payment of Tax Chapter-23

9. 43D Payments for rendering or providing by carriage services.


10. 45 Manufacturer cum exporter if certain conditions are satisfied.
11. 45A 1% WHT on supplies etc. to textiles, carpet, leather, surgical goods, and sports goods sector.
12. 46 Payments received by an oil distribution company or an oil refinery or PE of a non resident Petroleum Exploration and Production (E and P)
companies for supply of its products.
13. 46A Payments received by a manufacturer of iron and steel products relating to sale of goods manufactured by him.
14. 47A Payments received by a resident for supply of such goods, which were imported by him and on which tax was paid under section 148.
15. 47D Cotton ginners.
16. 57 Company operating Trading Houses which fulfills all conditions (i) have paid up capital of exceeding Rs.250 (M) (ii) own total assets exceeding Rs.
300 (M). (iii) maintain computerized records of imports and sales (iv) maintain a system for issuance of 100% cash receipts on sales (v) present
accounts for tax audit every year; (vi) is registered with the sales tax department
17. 57A Large import House.
18. 60 Fully as well part designed / assembled cypher devices, for use within the country as are verified by Cabinet Division (NIBS) with reference to
design, quality and quantity.
19. 67 Payments received by International Finance Corporation.
20. 68 Payments received by Pakistan Domestic Sukuk Company Ltd.
21. 77 Solar PV panels / modules.

1% of the amount Authorize dealer in foreign


154(1) Export proceeds realization. received Nil Final discharge exchange Export proceed

Realization of commission due to non-export indenting agent /


export indenting agent / export buying house 5% of the Authorize dealer in foreign
154(2) commission due Nil Final discharge exchange realization or at

1% of the
Realization of the goods to an exporter under an inland back - proceeds of the export of
154(3) to-back letter of credit export Nil Final discharge Banking Company goods

1% of the
Export of the goods by an industrial undertaking located in proceeds of Export processing zones
154(3A) EPZ export Nil Final discharge authority if made by an
undertaking in
EPZ
1% of the Direct export and export
proceeds of the house registered under
154(3B) Making payment for a firm contract to an indirect export export Nil Final discharge DTRE Rules 2001

1% of the
proceeds of
154(3C) Commission on clearing of the goods export Nil Final discharge Collector of customs Do
exported

Conceptual Approach to Taxes 431


Deduction / Payment of Tax Chapter-23

IMPORTANT NOTES:

154(5) The tax deducted u/s 154 shall be treated as final tax in respect of such income. However, it shall not be final tax in case of a person who opts that such income
not be subject to final taxation. This option shall be available from the tax year 2015 and shall be exercised every year at the time of filing of return u/s 114. Under such
a case the tax deducted shall be treated as minimum tax.
NON APPLICABILITY OF PROVISIONS OF TAX AT SOURCE U/S 154 UNDER VARIOUS CLAUSES OF PART IV OF 2ND SCHEDULE TO THE ITO, 2001:

U/C 47C An exporter in respect of cooking oil or vegetable ghee exported to Afghanistan from whom tax u/s 148 has been collected on import of edible oil.

Given in slab
155 Rent (including rent of furniture and for rate of tax Adjustable Payment
fixture) and services relating to
property.
Federal Govt.
Provincial Govt.
(a) For individual & AOP
Local authority or
company, non-profit
Upto Rs 150,000 Nil organization or a charitable
institution, diplomatic
10% of amount mission
exceeding Rs. of a foreign state and any
Exceeding Rs. 150,000 to Rs. 1,000,000 150,000 other person notified
by FBR.
Rs. 85,000 + a private educational
15% of amount institution, a boutique, a
exceeding beauty parlor, a hospital, a
Exceeding Rs. 1,000,000 Rs.1,000,000 clinic or a maternity home;

(b) For a company as payer 15% of gross Individuals or association of


rent Nil persons paying gross rent
of Rs.1,500,000 & above in
a year.

15% of the gross


156 Prizes on prize bonds and cross word amount Nil Final discharge Person making the Payment
puzzle payment.

Winning from raffle, lottery, quiz, prize


offered by companies for sale 20% of the gross Nil Final discharge do do
promotion amount.

432 Conceptual Approach to Taxes


Deduction / Payment of Tax Chapter-23

IMPORTANT NOTES

Where the prize is not in cash, then the person giving the prize shall collect the amount of tax due on the prize FMV.

Quote to prize money as car.


Filer Non-
156A Sale of petroleum product to Filer Nil Final discharge Person selling the
Petroleu to petrol pump operators. 12% 15% petroleum products
m At the time the
Products commission is
actually paid

156B Making
Withdrawal of balance under Yes Adjustable Pension Fund payment
Withdraw pension fund see note Manager from individual
al of
balance Average rate of
under tax
pension based on three
fund proceeding tax pension
years account
IMPORTANT NOTES

IMPORTANT NOTES:

1. Pension Fund Manager will deduct tax only if the amount is withdrawn
(i) Before the retirement age; or
(ii) In excess of 50% of his accumulated balance, if withdrawn at or after the retirement age
2. Tax shall not be deducted under the following cases:
The balance in the eligible persons individual pension account is invested in an approved income payment plan of a pension fund manager; or
paid to a life insurance company for the purchase of an approved annuity plan; or
is transferred to another individual pension account of the eligible person; or
the survivor pension account in case of death of the eligible person maintained with any other pension fund manager as specified by in the voluntary pension
system rules, 2005.
231A At the time of
Cash Rs: 50,000 Adjustable Banking company withdrawal of
Withdraw
al from a Payment of cash withdrawal exceeding Rs, 50,000/- in a
Bank day Filer 0.3%
Non filer 0.6% per day cash

Conceptual Approach to Taxes 433


Deduction / Payment of Tax Chapter-23

IMPORTANT NOTES:

U/C (28B) of Part II of 2nd Schedule: An Exchange company duly licensed and authorized by the SBP is subject to a reduced rate of 0.15% only on such
transactions which are exclusively dedicated for its authorized business related transactions. This reduced rate is subject to the condition that CIR has issued a
certificate for a financial year mentioning therein the details and particulars of its Bank Account being used entirely for business transactions.

NON APPLICABILITY OF PROVISIONS REGARDING TAX AT SOURCE U/S 231A UNDER VARIOUS CLAUSES OF PART IV OF 2ND SCHEDULE TO THE ITO,
2001:

U/C 61 Any cash withdrawal from a bank made by an earthquake victim against compensation received from Govt. of Pakistan.

Filer Non-
1. Sale against cash of any instrument including Filer
demand draft, payment order, CDR, STDR, RTC, 0.3% 0.6% At the time of
any other instrument of bearer nature or on receipt Rs. 25,000 Adjustable Every banking company sale of
of cash on cancellation of any of these per day non banking financial instrument
231AA instruments institution , exchange
Advance company or any authorized
tax on 2. Transfer of any sum against cash through online dealer of foreign exchange
transactio transfer, telegraphic transfer mail transfer or any 0.6% 0.6%
n in bank other mode of electronic transfer

Excise & Taxation


231B a) On Private motor vehicles registration and Nil Adjustable department Sales of car
transfer of ownership and manufacturer
Advance Filer Non filer
Tax on
Private Rs. Rs.
Upto 850cc 10,000 10,000
motor
vehicles 851cc -1000cc 20,000 25,000
1001cc - 1300cc 30,000 40,000
1301cc - 1600cc 50,000 100,000
1601cc - 1800cc 75,000 150,000
1801cc - 2000cc 100,000 200,000
2001cc - 2500cc 150,000 300,000
2501cc - 3000cc 200,000 400,000
above 3000cc 250,000 450,000

b) the rate of tax under sub-section (2) of section


231B shall be as follows- Filer Non filer Adjustable do do

434 Conceptual Approach to Taxes


Deduction / Payment of Tax Chapter-23

Rs. Rs.
Upto 850cc - 5,000
851cc -1000cc 5,000 15,000
1001cc - 1300cc 7,500 25,000
1301cc - 1600cc 12,500 65,000
1601cc - 1800cc 18,750 100,000
1801cc - 2000cc 25,000 135,000
2001cc - 2500cc 37,500 200,000
2501cc - 3000cc 50,000 270,000
above 3000cc 62,500 300,000

Same as point
c) Sale of vehicle by manufacturer (a) Nil Adjustable do do
IMPORTANT NOTES
IMPORTANT NOTES:

1. Tax u/s 231B on private motor cars shall not be collected if purchase is made by the Federal Government, a Provincial government, a foreign diplomat or a
diplomatic mission in Pakistan

2. Tax on registration of motor vehicle shall not be payable if the person produces evidence that tax has been paid u/s 231 B(3) in case of locally manufactured
vehicle or u/s 148 in the case of imported vehicle.
3. Rate of tax to be collected shall be reduced by 10% each year from the date of first registration in Pakistan.
4. Provisions of section 231 B shall not apply to the following persons;
(i) The Federal Government;
(ii) A Provincial Government;
(iii) A local Government;
(iv) A foreign diplomat; or
(v) A diplomatic mission in Pakistan. .
5. 'Date of first registration' means -
(i) The date of issuance of broad arrow number in case a vehicle is acquired from the Armed Forces of Pakistan;
(ii) The date of registration by the Ministry of Foreign Affairs in case the vehicle is acquired from a foreign diplomat or a diplomat mission in Pakistan;
(iii) The last day of the year of manufacture in case of acquisition of an unregistered vehicle from the Federal or Provincial Government; and
(iv) In all other cases the date of first registration by the Excise and Taxation Department.
6. 'Motor vehicle' includes car, jeep, van, sports utility vehicle, pick-up trucks for private use, caravan automobile, limousine, wagon and any other automobile
used for private purpose.

Conceptual Approach to Taxes 435


Deduction / Payment of Tax Chapter-23

233 Brokerage and commission

Brokerag
Federal Government,
e&
Provincial Government,
Commissi Non- At the time of
Local authority, Company
on Filer Filer payment
AOP constituted by or
a) In case of advertising agent 10% 15% Nil Final discharge under any law.
d) any other agent 12% 15% Nil Final discharge

IMPORTANT NOTES

IMPORTANT NOTES:

If the agent retains commission or brokerage from any amount remitted by him to the principal, it shall be deemed that the commission has been paid to him by the
principal and the principal shall collect tax from the agent.

NON APPLICABILITY OF PROVISIONS REGARDING TAX AT SOURCE U/S 233 UNDER VARIOUS CLAUSES OF PART IV OF 2ND SCHEDULE TO THE ITO,
2001:

U/C 38A A venture Capital Company.

U/C 47B Any person making payment to NIT, a collective Scheme, a modaraba, approved pension fund, approved income payment plan, a REIT Scheme, a private
equity and venture capital fund, a recognized provident fund, an approved superannuation fund or an approved gratuity fund.

233A Collection of tax by stock exchange


from:

a) On purchase of shares in lieu of the commission


earned by its Member 0.01% of purchase
value Nil Adjustable Stock exchange Payment
0.01% of sale
b) Commission on sale of shares value Nil Adjustable Stock exchange Do

233AA Advance Tax on margin financing through Stock 10% of profit,


Exchanges registered in Pakistan. markup or interest
earned by the
member; margin At the time of
financier or Mark-up /
securities lender Nil Adjustable NCCPL Interest is paid

436 Conceptual Approach to Taxes


Deduction / Payment of Tax Chapter-23

IMPORTANT NOTES:

NCCPL shall collect advance tax form:


1. the members of Stock Exchange registered in Pakistan;
2. margin financiers;
3. trading financiers and lenders in respect of margin financing in share business; or
4. securities lending under securities (Leveraged Markets and Pledging) Rules, 2011.
This provision is not applicable to a Mutual Fund specified in clause 57(2) of Part-l of Second Schedule.

234 Motor vehicle:


Tax on
Motor From plying or hiring of
Vehicle

Goods transport vehicle Rupees Nil Adjustable Person responsible Collection of


with registered laden weight: (Per annum) for collection of motor motor vehicle
Non-
Filer Filer vehicle tax tax
For plying or hiring of Goods Rs. 2 Rs. 4
per Kg per Kg
of the of the
laden laden
transport vehicle with weight weight
registered laden weight

Vehicle with laden weight of 8120 kgs or 1,200 Nil do do do


more, tax after ten year from first
registration in Pakistan
Passenger transport vehicle with
(Per seat per
annum)
Non-
registered seating capacity of: Filer Filer Adjustable
a) 4 to 9 persons Rs. 50 Rs. 100
b) 10 to 19 persons Rs. 100 Rs. 200
c) 20 and above Rs. 300 Rs. 500
Private motor car with engine
capacity of (Per annum) Adjustable
Filer non filer
(a) Upto 1000cc 800 1,200

Conceptual Approach to Taxes 437


Deduction / Payment of Tax Chapter-23

(b) Upto 1001cc to 1199cc 1,500 4,000


(c) Upto 1200cc to 1299cc 1,750 5,000
(d) Upto 1300cc to 1499cc 2,500 7,500
(e) Upto 1500cc to 1599cc 3,750 12,000
(f) Upto 1600cc to 1599cc 4,500 15,000
(g) 2000cc and above 10,000 30,000
Where motor vehicle tax is collected
in lump sum Lump sum
Filer non filer
(a) Upto 1000cc 10,000 10,000
(b) Upto 1001cc to 1199cc 18,000 36,000
(c) Upto 1200cc to 1299cc 20,000 40,000
(d) Upto 1300cc to 1499cc 30,000 60,000
(e) Upto 1500cc to 1599cc 45,000 90,000
(f) Upto 1600cc to 1599cc 60,000 120,000
(g) 2000cc and above 120,000 240,000

IMPORTANT NOTES:

Any person at the time of collecting motor vehicle tax shall also collect
advance tax

1. If the motor vehicle tax is collected in installments, the advance tax may also be collected in installments.
2. In respect of motor cars used for more than 10 years in Pakistan, no advance tax shall be collected after a period of 10 years.
3. In respect of a passenger transport vehicle with registered seating capacity of 10 or more persons, advance tax shall not be collected after a period of 10
years from the first day July of the year of make of the vehicle.
4. For goods transport vehicle with registered laden weight less than 8120 kg, advance tax shall not be collected after a period of 10 years from the date of first
registration in Pakistan.
5. 'Motor vehicle' shall have the same meanings as are assigned to it in section 231B(7).

234A Payment of
CNG Sales of CNG to CNG stations 4% of the gas Nil Final discharge Person preparing gas gas
Station consumption
charges consumption bill bill
Adjustable only in
235 Electricity commercial and industrial If exemption case Person preparing Payment of
Electricit certificate
y consumer with amount of bill. from of companies. electricity electricity bill
CIR is For all other person consumption bill

438 Conceptual Approach to Taxes


Deduction / Payment of Tax Chapter-23

produced. it
shall be minimum
a) Upto Rs. 400 Rs. 0 tax on
income of the
b) Rs 401 to Rs. 600 80 person, if the
bill is upto
c) Rs 601 to Rs. 800 100 Rs.30,000 per
d) Rs 801 to Rs. 1,000 160 month.
e) Rs 1,001 to Rs. 1,500 300
f) Rs 1,501 to Rs. 3,000 350
g) Rs 3,001 to Rs. 4,500 450
h) Rs 4,501 to Rs. 6,000 500
i) Rs 6,001 to Rs. 10,000 650
j) Rs 10,001 to Rs. 15,000 1000
k) Rs 15,001 to Rs. 20,000 1500
10% commercial
Exceeding Rs.20000 consumer
5% for industrial
consumer
IMPORTANT NOTES
IMPORTANT NOTES:

U/C 66 The exporters cum manufacturers of carpets, leather and articles thereof including artificial leather footwear, surgical goods, sports goods and textile and
articles thereof.

7.5% of monthly
electricity bill
where such bill is
235A equal or more than Rs. 74,999 Person preparing electricity On preparing
Electricity Domestic electricity consumption Rs. 75,000 (monthly bill) Adjustable bill bill
235B
Tax on In the manner
Steel Tax from Every Steel Melters, Steel Re-rollers, Rs, 1 per Unit of electricity
Melters, Composite Steel Units registered for the purpose of Electricity The person preparing consumption
Re-rollers Chapter IX of Sales Tax Special procedure Rules, 2007. consumed electricity consumption bill charges or
etc. Nil Non - Adjustable charged.

10% of the Govt, Person preparing telephone


236 Telephone where the monthly bill exceeding Foreign Adjustable / Internet bill or issuing / Payment of the
selling prepared card for telephone bill
exceed Rs.1,000. amount of bill diplomats, mobile phones / Internet or

Conceptual Approach to Taxes 439


Deduction / Payment of Tax Chapter-23

sales of
Diplomatic prepaid
mission in
Pak, person cards.
produces
exemption
certificate
issued
14% of amount of
Mobile and prepaid cards bill or by CIT Adjustable do
price of internet
prepaid card or
price of telephone
prepaid card or
sale of unit through
CD or whatever
form
236A
Sale by Collection of advance tax at the time of sale by public 10% of the gross At the time of
auction auction or auction by a tender of any property or goods. sale price Nil Adjustable Person making sale sale

236B
Advance
tax on
purchase
of Air Collection of advance tax on purchase of domestic air 5% of the gross At the time of
Ticket ticket. amount of air ticket Nil Adjustable Person making sale sale
IMPORTANT NOTES
For the purpose of this section:
Advance tax on purchase of air tickets shall not apply to routes of Baluchistan coastal belt, Azad Jammu and Kashmir, FATA, Gilgit-Baltistan and Chitral.

236C At the time of


Immovea Non- Person responsible for registering or
ble Advance tax on gross amount of consideration received Filer Filer registering or attesting attesting the
property on sale or transfer of immovable property 0.5% 1% Nil Adjustable transfer transfer
236D At the time of
Advance raising of
Tax on invoice /
Function payment of bill
& Every prescribed person shall collect tax on the total 5% of the invoice Owner, Manager& operator
Gathering amount of bill paid by a person holding or arranging value / payment of Nil of the Marriage Hall, Club,
s functions bill Adjustable Hotel, Marquee etc.

440 Conceptual Approach to Taxes


Deduction / Payment of Tax Chapter-23

IMPORTANT NOTES:

1. "Function" includes any wedding related event, seminar, a workshop, a session, an exhibition, a concert, a show, a party or other gathering held for such purpose.
2. "Prescribed person" includes the owner, lease-holder, an operator or a manager of a marriage hall, marque, hotel, restaurant, commercial lawn, club, community
place or other place used for such purpose.
3. Where the foods, services or any other facility is provided by any other person, the prescribed person shall also collect advance tax on payment for such food,
service or facility at the prescribed rate.

236E (a) Drama serial


Advance Rs, 100,000/= per
Tax on episode
Foreign Any licensing authority shall collect tax while certifying Nil Adjustable
Produced any foreign Tv drama serial or a play dubbed in urdu or (b)Play Rs, At the time of
TV Plays any regional language for seeing and viewing on any 100,000 / (single certification
& Serials channel landing right Play) Nil Adjustable Every licensing Authority

IMPORTANT NOTES:

"cable television operator", "DTH", "Distribution Service", "electronic media", "IP-TV, "loop holder". "MMDS", "Mobile TV" shall have the same meanings as are
assigned in the Pakistan Electronic Media Regulatory Authority Ordinance, 2002. In the case of IPTV, FM Radio, MMDS, Mobile TV, Mobile Audio, Satellite TV
Channel and Landing Rights the rate of tax shall be 20% of the permission fee or renewal fee. as the case may be.
Advance tax on cable operators and other electronic
media
License category Tax on license Tax on Renewal
H 7,500 10,000
H-I 10,000 15,000
H-II 25,000 30,000
R 5,000 30,000
B 5,000 40,000
B-1 30,000 50,000
B-2 40,000 60,000
236F B-3 50,000 75,000
Advance B-4 75,000 100,000
Tax on B-5 87,500 150,000
Cable B-6 175,000 200,000
Operators B-7 262,500 300,000
& other B-8 437,500 500,000 At the time of
Electronic B-9 700,000 800,000 Varying Rs. 7,500 Pakistan Electronic Media issuance of
s B-10 875,000 900,000 to Rs. 900,000 Nil Adjustable Regulatory Authority license

Conceptual Approach to Taxes 441


Deduction / Payment of Tax Chapter-23

236G
Advance
Advance tax has to be collected from wholesaler , Nil
Tax on
distributor & dealers at the time of sales made to them. 0.7% 1.4% at the time of
sales to From the distributors,
sale to
distributor dealers and wholesales to
i) Fertilizers Adjustable distributors,
s, dealers whom such sales have
dealers and
& been made
ii) Other than Fertilizer 0.1% 0.2% Nil wholesalers
wholesale
r

Every manufacturer,
distributor, dealer,
wholesaler or commercial
236H
importer of electronics,
Advance 0.5% of the gross
Advance tax has to be collected from retailers at the time sugar, cement, iron, and at the time of
Tax on amount of goods Nil Adjustable
of sales made to them steel products, fertilizer, sale to retailers
sales to sold
motorcycles, persticides,
Retailers
cigarettes, glass textile,
beverages, paint or foam
sector,
236I
Collection
of
Advance 5% of the amount At the time of
Educational Institution Rs.200,000 Adjustable
Tax by of fee fee collection
Education
al Educational institution
Institution

IMPORTANT NOTES:

Provisions of section 236I shall not apply to a non-resident person if: [236I(6)]
i) His stay in Pakistan is less than 183 days on the basis of copy of passport furnished to the educational institution;
ii) He furnishes a certificate that he has no Pakistan-source income; and
iii) Fee is remitted directly from abroad through normal banking channels to the bank account of the educational institution.

236J Every Market committee shall collect tax from dealers,


Advance commission agents or Arhetis on issuance or renewal of
At the time of
Tax on license Every Market Committee or
issuance or
dealers, Group or Class A Rs. 10,000 Nil Adjustable any commodity regulatory
renewal of
commissi Group or Class B Rs, 7,500 authority
license
on agents Group or Class C Rs, 5,000
and arhtis Any other category Rs, 5,000

442 Conceptual Approach to Taxes


Deduction / Payment of Tax Chapter-23

IMPORTANT NOTES:

"Market Committee" includes any committee or body formed under any provincial or local law made for the purposes of establishing, regulating or organizing
agricultural, livestock and other commodity markets.
Advance tax on purchase or transfer of immovable
property
At the time of
i) Where value of Immovable property is Upto 3 0% Person responsible for
registering or
million Rs.3 million registering or attesting
Adjustable attesting the
236K transfer
transfer
ii) Where the value immovable property is more than
1% 2%
3 million

IMPORTANT NOTES:

1. Provisions of section 236K do not apply to a scheme introduced by Government (Federal, Provincial) or an Authority established by Government for expatriate
Pakistanis with a condition that mode of payment shall be in the foreign exchange remitted from outside Pakistan through normal banking channels.'
2. Provided that the mode of payment by the expatriate Pakistanis in the said scheme or schemes shall be in the foreign exchange remitted from outside Pakistan
through normal banking channels;
3. Rate of tax for Non-Filer shall be 1% up to the date appointed the FBR through a notification in the official Gazette.
Advance tax on purchase of international Air tickets Rs. 16,000 per person Every airline, issuing ticket At the time of
(i) First/ Executive class
236L Nil Adjustable for journey originating from issuance of
(ii) Others excluding Economy Rs. 12,000 per person
Pakistan ticket
(iii) Economy
Rs.0
On 1st day of
5% of the bonus closure of
Bonus shares issued by Companies quoted on stock shares price on 1st Every company quoted on books to be
236M Nil Final discharge
exchange day of closure of stock exchange deposited within
books 15 days of tax
deduction
IMPORTANT NOTES:

1. While issuing bonus shares the listed company shall withhold 5% of the bonus shares to be issued as a security for payment of tax due on bonus shares.
2. Tax to be collected from the shareholder shall be 5% of the value of the total bonus shares issued to the shareholder. After payment of the tax, bonus shares
withheld shall be issued to the shareholder.
3. The value shall be determined on the basis of day-end price on the first day of closure of books.
4. Tax shall be collected by the company within 15 days of the first day of closure of books.
5. The shares withheld shall be deposited with the Central Depository Company or any other prescribed entity if tax on bonus shares have not been paid or collected
6. The shares so deposited shall be disposed of in the prescribed mode and manner and the proceeds shall be deposited to the Commissioner by way of credit to
Federal Government.
7. Issuance of bonus shares shall be deemed as income of the shareholder and tax paid/collected/deposited to Commissioner shall be treated as payment of tax on
behalf of the shareholder.
8. Tax paid shall be treated as final tax on income of shareholder arising from issuance of bonus shares.

Conceptual Approach to Taxes 443


Deduction / Payment of Tax Chapter-23

On 1st day of
closure of
5% of the bonus
books to be
Bonus shares issued by Companies not quoted on stock shares price on 1st Every company not quoted
236N Nil Final discharge deposited
exchange day of closure of on stock exchange
within 15 days
books
of tax
deduction
IMPORTANT NOTES:

1. Irrespective of the fact that the company has or has not collected the tax from the shareholder, it shall deposit the tax on bonus shares within 15, days of closure of
books.
2. Company shall be entitled to collect and recover the tax deposited on behalf of the shareholder before issuance of bonus shares.
3. Where a shareholder has neither paid the tax deposited by the company nor collected the bonus shares within a period of three months from the date of such
issuance, the company may dispose of so much bonus shares of such shareholder as are sufficient to recover the amount of tax paid by the company.
4. Tax shall be computed by applying the rate to the value of the total bonus shares issued to the shareholders. The value of bonus shares shall be the value on the
first day of closure of books.
5. Issuance of bonus shares shall be deemed as income of the shareholder and tax paid for such shares shall be treated as payment of tax on behalf of the
shareholder.
0.6% (from
236P (i) Every Banking Company shall collect Advance tax 01/07/2015 to
Advance from non-filers on sale of instruments, including demand 10/07/2015 and
tax on draft , pay order , special deposit receipt, cash deposit from 01/10/2015 to
Transaction At the time of
banking receipt, short term deposit receipt, call deposit receipt & onwards)
upto Rs. Every Banking Company sale of such
transactio rupee travelers cheque. (0.3% from Adjustable
50,000 in a instruments
n other 11/07/2015 to
day.
than ii) Every Banking Company shall collect advance tax 31/10/2015)
through from non filers on transfer of any sum through cheaque
cash or clearing, interbank or interbank transfers through
cheaque, online / telegraphic / mail transfer.

IMPORTANT NOTES:

1. This advance tax is applicable on transactions with non-filers.


2. Instrument includes demand draft, pay order, special deposit receipt; cash deposit receipt, short term deposit receipt, call deposit receipt, rupee travellers'
cheque or any other instrument of such nature.
3. Transfer of funds may be through cheques, online transfer, telegraphic transfer, mail transfer, direct debit, payments through internet, payments through
mobile phones, account to account fund transfers, real time account to account funds transfer, real time third party account to account fund transfer,
automated teller machine. (ATM) transfers, or any other mode of electronic or paper based funds transfer.
4. This advance tax shall not be collected in case of:
i) Pakistan Real time Interbank Settlement Mechanism (PRISM) transactions; or
ii) ii) Payments made for Federal, Provincial or Local Government taxes.

444 Conceptual Approach to Taxes


Deduction / Payment of Tax Chapter-23

236Q
Payment
to
(i) Payments for the right to use Industrial, Commercial,
resident
and scientific , equipment 10% Every prescribed person
for use of At the time of
Nil Final Discharge (As per (7) of 153
machiner payment
(ii) payments on account of rent of Machinery, Industrial, 10%
y&
and Commercial and Scientific equipment
equipmen
t

IMPORTANT NOTES:

Tax under this section shall not be deducted on payments for:


1. Agricultural machinery; and
2. Machinery leased by a leasing company, an investment bank, a modaraba, a scheduled bank or a development finance institution in respect of assets owned by
such institution.
236R
Education Banks, Foreign Exchange
At the time of
related Advance Tax be collected on the amount of education Companies or any person
5% Nil Adjustable remitting the
expenses related expenses remitted abroad. responsible for remitting the
expenses
remitted amount
aboard

IMPORTANT NOTES:

Education related expenses includes tuition fees, boarding and lodging expenses, any payments for distant learning to any institution or university in a foreign
country and any other expense related or attributable to foreign education.
236S
Dividend Same provisions will be applicable here as are applicable above in this chart under section 150.
in Specie
236T
Collection
of Tax by PMEX shall collect advance tax on;-
At the time of
Pakistan a) On Purchase of future commodity contract. 0.05% Nil Adjustable
Pakistan Mercantile purchase
Mercantil
Exchange Limited (PMEX) At the time of
e b) On Sale of future commodity contract. 0.05% Nil Adjustable
sales
Exchange
Limited
(PMEX)

Conceptual Approach to Taxes 445


Deduction / Payment of Tax Chapter-23

MULTIPLE CHOICE QUESTIONS


Q.1. The person who is liable to deduct or collect tax at source is required to deposit the same generally within the
___________after each fortnight.
(a) 10 days
(b) 7 days
(c) 15 days
(d) 30 days
Q.2. Persons liable to deduct tax at source are also required to file _______________statements.
(a) Monthly and quarterly
(b) semi-annually and annually
(c) quarterly
(d) None of the above
Q.3. The person in default of deduction of tax at source shall be liable to pay default surcharge on such amount per annum
for the period of its non-payment at the rate of_________.
(a) 10%
(b) 15%
(c) 20%
(d) 12 %
Q.4. While computing the taxable income of a person such expenses where the payment of the same has been made
without deduction of tax at source ___________________.
(a) Be allowed as admissible expenses
(b) Not be allowed as admissible expense
(c) Added to the loss of such person
(d) Both b and c
Q.5. Advance tax on capital gain u/s 37A for individual investor where holding period of securities is less than six months
is ______of capital gains derived during the quarter.
(a) 2%
(b) 10%
(c) 1.5%
(d) none of the above
Q.6. Where holding period of securities u/s 37A is more than 6 months but up to 12 months, advance tax by an AOP on
capital gain in the said quarter shall be chargeable at the rate of _________.
(a) 2%
(b) 1.5%
(c) 2.5%
(d) 5%
Q.7. While computing the amount of advance tax the tax in respect of following heads shall not be considered_____:
(a) Capital gain
(b) Income covered under FTR
(c) Income under NTR
(d) Both a and b
Q.8. An AOP is required to pay advance tax equal to one forth (1/4th) of last assessed tax liability if the taxable income of
that year is equal to or more that Rs.__________.
(a) 100,000
(b) 200,000

446 Conceptual Approach to Taxes


Deduction / Payment of Tax Chapter-23

(c) 150,000
(d) None of the above
Q.9. The amount of tax demanded should be paid from the date of service of notice within_______.
(a) 30 days
(b) 15 days
(c) 10 days
(d) 60 days.
Q.10 The amount of tax deducted at source shall be so deducted when the amount ______.
(a) becomes payable
(b) becomes receivable
(c) actually paid
(d) none of above
Q.11 Tax shall not be deducted if ____________ is presented.
(a) exemption certificate
(b) refund is due
(c) Income tax return
(d) sales tax return
Q.12 The Board is authorized to amend the rates of withholding tax by issuing a notification in the _______.
(a) newspaper
(b) official gazette
(c) ITO, 2001
(d) all of above
Q.13 Generally amendments are effective ________ the date of such amendment.
(a) after
(b) before
(c) from
(d) all of above
Q.14 Where a person responsible to deduct or deposit the tax fails to deduct or collect the tax at source or fails to deposit
within the prescribed time shall be __________.
(a) personally liable
(b) required to file income tax return
(c) none of above
Q.15 The amount of tax shall not be recovered from the person failing to deduct or deposit the tax where the person from
whom it was to be deducted has ________.
(a) close down his business
(b) already paid tax
(c) left Pakistan
(d) all of above
Q.16 A person in default of tax deduction shall pay _______ where the person from whom it was to be deducted has
already paid the amount of tax.
(a) extra tax
(b) default surcharge
(c) minimum tax
(d) all of above

Conceptual Approach to Taxes 447


Deduction / Payment of Tax Chapter-23

Q.17 If the person has not deducted tax at source and has made the payment of tax then _______.
(a) he may recover from other person
(b) he cannot recover from other person
(c) he shall be punished with fine
(d) all of above
Q.18 The certificate giving evidence of the deduction and collection of tax at source is issued by the ____.
(a) tax authorities
(b) person who has deducted tax
(c) person whose tax was deducted
(d) all of above
Q.19 A company shall pay turnover tax as advance tax ________ the quantum of its income.
(a) irrespective of
(b) with respect to
(c) none of above
Q.20 The last date to pay the advance tax by a Company related to the period from January to 31st March is __________.
(a) 25th March
(b) 31st March
(c) 1st January
(d) all of above
Q.21 While computing advance tax, turnover tax payable by the company shall be ________.
(a) ignored
(b) taken into account
(c) none of above
Q.22 If the latest assessed taxable income of an individual is Rs. ______ or more he is required to pay advance tax.
(a) 200,000
(b) 300,000
(c) 400,000
(d) 500,000
Q.23 Taxpayer may make adjustment of tax paid / deducted at source under NTR during the ______ of which advance tax
is payable.
(a) year
(b) month
(c) century
(d) quarter
Q.24 Taxpayer shall furnish the estimate to Commissioner Inland Revenue if he is of the opinion that his tax liability for the
current year will be ________ the previous year.
(a) less than
(b) more than
(c) equal to
(d) Both (a) and (b)
Q.25 The amount of tax paid in advance shall be adjusted against the _______ as per return of the taxpayer.
(a) preceding year tax payable
(b) refund of preceding tax year
(c) tax payable

448 Conceptual Approach to Taxes


Deduction / Payment of Tax Chapter-23

(d) none of above


Q.26 Instalments for the payment of tax demanded by Commissioner Inland Revenue _____ be granted by Commissioner
Inland Revenue.
(a) may
(b) cannot
(c) should
(d) none of above
ANSWERS

1 (b) 2 (d) 3 (d) 4 (b) 5 (a)

6 (b) 7 (b) 8 (d) 9 (b) 10 (c)

11 (a) 12 (b) 13 (c) 14 (a) 15 (b)

16 (b) 17 (a) 18 (b) 19 (a) 20 (a)

21 (b) 22 (d) 23 (c) 24 (d) 25 (c)

26 (a)

Conceptual Approach to Taxes 449


Deduction / Payment of Tax Chapter-23

ICMAP PAST PAPERS THEORECTICAL QUESTIONS

Q.NO.3 August 2014 The Finance Bank Limited (FBL) has remitted Rs. 700,000 as a commission to Pakistan Branch of
M/s. Technocom Inc., a non-resident Singapore based company. Although in the agreement it is clearly mentioned that the
applicable income tax can be deducted from the payment, but FBL remitted full amount without any deduction of tax under
Section 152(2) of the Income Tax Ordinance, 2001.
Required:
Elaborate the provisions contained in section 152 (3) of the Income Tax Ordinance, 2001 which may allow the payment
made by FBL without any deduction of tax.

Q. NO.2(b) February 2014 Macro Trading (MT) is a sole proprietorship owned by Mr. Waheed. He is engaged in the
manufacturing and supplying of herbal products for last many years. His taxable income for the year ended June 30,
2013 was Rs.1,000,000. Mr. Waheeds Tax Advisor apprised him to pay advance tax for the tax year ended June 30, 2014.

Required Keeping in view his Tax Advisors advice, Mr. Waheed need explanation with regards to the following queries as
per the Income Tax Ordinance, 2001:

(i) How the amount of advance tax liability would be calculated for the tax year ended June 30, 2014?

(ii) What would be the period of quarter and last date for payment of advance tax under each of the
following quarter namely:

Serial No. Quartered Ended Period of Quarter Last Date for Payment of Advance Tax
1 September ? ?
2 December ? ?
3 March ? ?
4 June ? ?

(iii) As amount of advance tax is calculated on the basis of the assessed tax liability of the latest tax year, which
sources of income shall not be included while computing taxable income and tax liability of the latest tax year?
If Mr. Waheed is of the opinion that his tax liability for the current year is less than the previous year, then how will he adjust
the amount of his advance tax installments?
Q. No. 2(a) February 2013 As per section 170 of the Income Tax Ordinance, 2001 who is entitled to claim refunds and
when does an application for a refund become due?
Q. No. 3(a) February 2013 Write short answers of the following questions:
(i) What is the condition for an 'individual' and 'association of persons' to qualify as a 'prescribed person' for the purpose of
making deduction at source while making payment under the provisions of section 153 of the Income Tax Ordinance, 2001?
Q.3 (b) (i) APRIL 2012 In the light of Rule 87 of the Income Tax Rules, 2002 describe the procedure for the registration of
income tax practitioners.
(iii) What is the duration of the registration of a person as an Income Tax Practitioner under rule 88 of the Income Tax Rules,
2002?
Q. NO. 3 (c) (i) SUMMER 2011 List down any three prescribed qualifications for registration as an income tax practitioner
under Rule 86 of the Income Tax Rules, 2002.
Q. NO. 3 (c) (ii) SUMMER 2011 Discuss the situations causing an end of the duration of registration of an income tax
practitioner under Rule 88 of the Income Tax Rules, 2002.
Q. NO. 2 (a) SUMMER 2008 What is the prescribed time limit for payment to the Commissioner Inland Revenue on account
of tax collected or deducted by the withholding agent?
Q. NO. 3 (b) WINTER 2007 What are the provisions of section 162 of the Income Tax Ordinance, 2001 regarding recovery
of tax from the person from whom tax was not collected or deducted?
Q. NO. 2 (a) WINTER 2006 Discuss the following under Income Tax Ordinance, 2001;
(i) Provisions for additional payments for delayed refunds under section 171
Q. NO. 4 (a) SUMMER 2005 Under what circumstances a person is liable to pay additional tax under section 205 of the
Income Tax Ordinance, 2001? and at what rate?
Q. NO. 4 (b) SUMMER 2005 Under what circumstances additional tax levied under section 205 of Income Tax Ordinance,
2001 can be reduced?
Q. NO. 4 (c) SUMMER 2005 When does additional payment for delayed refund become payable by the department to the
assessee under ITO-2001

450 Conceptual Approach to Taxes


Deduction / Payment of Tax Chapter-23

Q. NO. 8 (a) SUMMER 2005 Discuss the provisions contained in Section 137 of the Income Tax Ordinance, 2001 related to
due date of payment.
Q. NO. 2 (b) SUMMER 2004 How Quarterly Advance tax is calculated in respect of
1- A company
2- An Association of persons, and
3- An individual?

Conceptual Approach to Taxes 451


Deduction / Payment of Tax Chapter-23

CA MOD C PAST PAPERS THEORETICAL QUESTIONS


Q. NO. 8 of Spring 2014 Beta & Co. is an AOP. A partner in the firm has approached you to calculate the advance tax
liability of the firm for the fourth quarter. The following information has been provided to you:

Information from tax Information


return for 2013 pertaining to Tax Year
20X4
.. Rs. In Million ...

Turnover - Normal tax regime 600 625


- Final tax regime 200 240
Tax liability - Normal tax regime 13 -
- Final tax regime 14 17

The firm's turnover under normal and final tax regime, up to third quarter of the tax year 20X4 was Rs. 450 million and Rs.
140 million respectively.
Required:
Compute the advance tax payable for the fourth quarter pertaining to tax year 20X4.
Q. No. 4 (a) Autumn 2013 State the procedure to be followed when a person intends to make payment to a non-resident
person without deduction of tax.
Q. No. 6 (a) Spring 2013 Explain the term ' Value of Goods' in the context of collection of advance tax from an importer of
goods under the Income Tax Ordinance, 2001.
(b) Under the Income Tax Ordinance, 2001 the amount of tax required to be collected by the collector of customs from an
importer of goods at the specified rate shall be the final tax. What are the exceptions to this provision of the Ordinance?
(c) Details of income chargeable to tax for the last tax year, relating to four taxpayers are tabulated below;

Other sources
Name of taxpayers Salary property Business Total income
(Dividend)
RUPEES
Mr. Brilliant - 60,000 375,000 80,000 515,000
Miss Educated 250,000 350,000 - 200,000 800,000
Motivated & co (An AOP) - - 150,000 - 150,000
Confident services Limited - 240,000 245,000 35,000 520,000
Required:
In respect of each of the above taxpayers, explain whether they are required to pay advance tax under the Income Tax
Ordinance, 2001.
Q.4 (a) Spring 2012 Under the Income Tax Ordinance, 2001 every prescribed person is liable to deduct tax while making
payments on account of sale of goods, rendering of services and execution of contracts.
Required: State six exceptions to the above rule.
Q.3 (a) Spring 2012 On 1 January 2012, Peetal Limited (PL) signed an annual contract with Mr. Heera for the maintenance
of IT equipment for Rs. 20,000, payable on the 7th day of each month. The payments for January and February were made
as per the agreement.
On 01-03- 2012, PL received a notice from the Commissioner Inland Revenue to pay income tax of Rs. 300,000 which is
due from Mr. Heera.
Required: Discuss PLs position in respect of the notice issued by the Commissioner Inland Revenue.
Q.4 (a) Spring 2011 The tax collected on imports by large import houses is considered as a final tax unless they fulfill
certain conditions specified under the Income Tax Ordinance, 2001. You are required to list those conditions.
Q.NO. 4(b) Spring 2011 Every taxpayer whose income was charged to tax for the latest tax year is liable to pay advance tax
in the manner prescribed under the Ordinance, Specify the incomes which are not considered for the purpose of
ascertaining advance tax, under the Income Tax Ordinance, 2001.

452 Conceptual Approach to Taxes


Deduction / Payment of Tax Chapter-23

Q.NO. 4(c) Spring 2011 Mr. Laiq is an accountant in an association of persons and wants to pay advance tax for the first
quarter of the year. Under the provisions of Income Tax Ordinance, 2001 advise him about the method of computing the
amount of advance tax.
Q.NO. 4(b) Spring 2010 Hanif Limited (HL) has commenced its commercial business operations with effect from January 1,
2010. Since no assessment of HL has yet been finalized, the management is of the view that HL is not required to pay any
quarterly advance tax.
Required:
(i) Discuss the managements point of view under the Income Tax Ordinance, 2001.
(ii) What are the consequences of non-payment of advance tax?
Q.NO. 4(c) Spring 2010 On February 15, 2010 Income Tax Department initiated proceedings against Zaman Enterprises
(Private) Limited (ZEL) for monitoring of withholding taxes. After examining the statements filed up to January 2010 and the
information submitted by ZEL, the Commissioner Inland Revenue has issued a show cause notice in respect of the
following:
(i) No tax was deducted on payments of Rs. 5.5 million made to Shahid and Co. who is one of the main suppliers of
packing materials to ZEL. Shahid and Co. imports and sells the imported products in local market in the same
condition in which they are imported.
(ii) ZEL deducted withholding tax from the payments made to Mansoor Sons against supplies of various accessories.
However, withholding tax was deducted on amount excluding sales tax of Rs. 192,000.
(iii) Rs. 50,000 was paid to Mujahid Engineering as advance against services but no tax was deducted at the time of
payment.
(iv) ZEL deducted tax at the rate of 10% from payment of commission to its sales staff.
Required: With respect to each of the above transactions, comment on ZELs position including consequences (if any) in
the light of ITO, 2001.
Q.NO. 6(b) Autumn 2009 List the prescribed persons as specified by the Income Tax Ordinance, 2001 who are required to
deduct tax while making payment for supply of goods.
Q.NO. 3(a) Spring 2009 Advance tax collected by the Collector of Customs is considered as final tax on the income of the
importer arising from imports except in certain situations. You are required to list such exceptions.
Q.NO. 3(a) Spring 2008 AAS (Pvt.) Ltd was incorporated on July 1, 2006 and commenced commercial operations in the
same month. It suffered losses in the first year of its operations. Briefly explain how the company should determine the
amount of advance tax to be paid (if any) in the tax year 2008.
Q.NO. 5(b) Autumn 2008 Quarterly advance tax payable under section 147 is computed on the basis of estimated taxable
income. Prepare a list of various types of income which are not taken into consideration while calculating the amount of
advance tax.
Q.NO. 3(b) Autumn 2007 Every prescribed person is required to deduct tax while making payments on account of sale of
goods, rendering of services and execution of contracts. Specify any seven exceptions to this rule.
Every tax payer, whose income was charged to tax for the latest tax year, is liable to pay advance tax in accordance with the
ITO, 2001. You are required to list down the incomes which are excluded for the purpose of calculating advance tax.
Q.NO. 3 Autumn 2005 What is understood by the term, own estimate in the context of quarterly advance tax payment by
an assessee. Explain the implications of an incorrect estimate by the assessee.
Q.NO. 4(c) Spring 2004 Please mention the period for which advance tax is payable and the dates by which such advance
tax is payable.
Q.NO.4(d) Spring 2004 What would be the withholding tax rate on the payment of Rs. 50 million by a company to a resident
person for the execution of a turnkey contract. Please also explain the taxability of income from such turnkey contract in the
hands of resident person.
Q.NO.5 Autumn 2003 List down the taxes deducted/collected at source which are treated as full and final discharge of tax
liability?
Q.NO. 2 Spring 2003 Explain the provisions relating to payment of advance tax by an individual.
Q. NO. 6(a) Spring 2003 Briefly explain the salient features of deduction/collection of income tax at source on the following:
1. Payment of dividend to a corporate shareholder
2. Payment of rent
3. Imports

Conceptual Approach to Taxes 453


Deduction / Payment of Tax Chapter-23

Q.7 Spring 2002 What are the requirements for advance payment of tax by an individual under the Income Tax Ordinance,
1979?
Q.8 Spring 2002
(a) What are the consequences for non-payment or short payment of advance tax?
(b) What powers are provided in the Income Tax Ordinance, 1979 for recovery of tax if an assessee fails to voluntarily
pay the assessed tax liability?
Q.8 Autumn 2002 Explain whether income tax authorities can rectify their orders, if so, under what circumstances and is
there any limitation of time for doing so?
Q.10 Autumn 2002 Explain the law applicable in special case for recovery of income tax from a defaulted partner of a firm.
Q.7 (a) Autumn 2002 Elaborate the provision of Section 50(1) relating to withholding of tax on Salary.
Q.9 Autumn 2002 How would you make deduction of tax at source on payment of dividend to A resident individual
a. Public company listed in Pakistan
b. A non-resident individual
c. A non-resident company
Q.NO. 1 March 2000 who is required to withhold or collect tax under the following cases? Are there any exception
available?
(i) commission and brokerage
(ii) at the stage of import
(iii) at the time of payment of dividend
Q.NO. 1 Sep 2000 Discuss the provisions regarding recovery of tax.
Q.NO. 1 Sep 2000
(a) who is required to pay advance tax;
(b) how advance tax is computed;
(c) what are the dates for payment of advance tax
Q.NO. 7 Sep 2000 describe briefly the provisions of withholding tax in respect of:
(a) Payment of interest of profit on an account or deposit with a bank.
(b) Payment for services rendered by a resident person
(c) commission or brokerage
(d) payment of rent of property
Q.NO. 6 March 1999 briefly develop a write up for you clients in corporate as well as non-corporate sector advising them
about the provisions of section 53 (now section147) of the Income Tax Ordinance relating to advance tax before
assessment.
Q.NO. 5 Sep 1999 what are the provisions of the Income Tax Ordinance relating to withholding tax in respect of the
following payments?
i. payment to residents in respect of supply of goods, services and execution of contracts
ii. fee for technical services to non-resident
iii. collection of tax on casual income ( prize bond, lottery etc)
Q.NO. 6 May 1997 write short notes on advance payment of tax.
Q.NO. 8 May 1997 what are the provisions relating to tax withholding in respect of the payment of rent, salary, brokerage/
commission and fee for technical services to non-resident.
May 1994 the monetary limit for non-deduction of tax at source in respect of payment on account of services is rupees:
(a) 5,000 (b) 10,000
(c) 15,000 (d) 25,000
Q.NO. 9 Nov 1994 answer the following statements considering the keys given therein:
Advance tax on quarterly basis is payable:
(a) Before income year (b) at the start of income year (c) during income year

454 Conceptual Approach to Taxes


Offences and Prosecutions Chapter-24

Chapter

24 OFFENCES AND PROSECUTIONS

Topic covered
Section
(For CA Mod F & ICMAP students)
114, 115, 116 & 165 Penalty for Failure to Income tax return, statements and wealth reconciliation
174 Penalty for Failure to Issue cash memo, invoice or receipt
181 Penalty for Failure to Apply for Registration and Failure to Notify a Change of Material Nature
182 Offences and penalties for various defaults
137 Failure to Deposit the amount of tax or any part thereof
174 Failure to Maintain Records
176 Failure to furnish information
177 Failure to non-compliance with the provisions of total audit
209 & 210 Penalty for Obstruction in Performance of Duties
183 Exemptions from penalty and default surcharge
191 Prosecution for non-compliance with certain Statutory Obligations
192 Prosecution for false statement in verification
192A Prosecution for concealment of income
193 Prosecution for failure to maintain records
194 Prosecution for improper use of National Tax Number Card
195 Prosecution for making false or misleading statements
196 Prosecution for obstructing an income tax authority
197 Prosecution for disposal of property to prevent attachment
198 Prosecution for unauthorized disclosure of information by a public servant
199 Prosecution for abetment
200 Offences by companies and association of persons
201 Institutions of prosecution proceedings without prejudice to other action
202 Power to compound offences
203 Trial by Special Judge
203A Appeal against the order of a Special Judge
204 Power to tender immunity from prosecution
MCQs with solutions
ICMAP past papers theoretical questions

(For CA Mod F & ICMAP students)


The Income Tax Ordinance, 2001, levied various punishments in the form of default surcharge, penalties, prosecutions and
imprisonment on certain actions, omissions, abstinences or defaults. The various defaults under the Income tax Ordinance,
2001 along with punishments applicable on the same is as under.

Conceptual Approach to Taxes 455


Offences and Prosecutions Chapter-24

1. PENALTIES
Default Penalty Section
1. Where any person fails to furnish a return 0.1% of the tax payable in respect of that tax 114 & 118
of income within the due date year for each day of default subject to a
maximum penalty of 50% of the tax payable
provided that if the penalty worked out as
aforesaid is less than Rs. 20,000 or no tax is
payable for that tax year such person shall
pay a penalty of Rs. 20,000
1A. Where any person fails to furnish a Rs. 2,500 for each day of default subject to a 115, 165 and 165A
statement as required within the due date minimum penalty of Rs. 10,000 (SRO no.
978(I)/2013 dated 13-11-2013)
1AA Where any person fails to furnish wealth Higher of 0.1% of the taxable income per 114, 115 and 116
statement or wealth reconciliation week or Rs. 20,000.
statement.
2. Penalty for Failure to Issue cash memo, Higher of Rs.5,000 or 3% of the amount of 174 and Chapter VII of
invoice or receipt the tax involved the Income Tax Rules
3. Penalty for Failure to Apply for Rs. 5,000 181
Registration and Failure to Notify a
Change of Material Nature
4. Failure to Deposit the amount of tax or any (a) For the first time default, 5% of the 137
part thereof amount of the tax in default.
(b) For the second default, an additional
penalty of 25% of the amount of the tax
default.
(c) For the third and subsequent default, an
additional penalty of 50% of the amount
of tax in default.
5. Repetition of erroneous calculation in the Higher of Rs. 5,000 or 3% of the tax 137
return for more than 1 year resulting in involved.
lesser amount of tax paid
6. Failure to Maintain Records under the Higher of Rs. 10,000 or 5% of the amount of 174
Ordinance and Rules tax on income.
7. Failure to Furnish Information Rs.5,000 for the first default and Rs.10,000 176
for each subsequent default.
8. Failure to non-compliance with the (a) Such person on first notice shall be liable 177
provisions of total audit to pay a penalty of Rs.25,000.
(b) If he fails after the second notice also,
the amount of penalty shall be Rs.
50,000.
(c) Failure to produce the record on the third
notice will result in a penalty of Rs.
100,000.
9. Failure to furnish the information required Rs.25,000 for the first default and Rs.50,000 176
or to comply with any other term of the for each subsequent default.
notice served
10. Penalty for Making a False Statement etc. Higher of Rs. 25,000 or 100% of the amount 114, 115, 116, 174,
of resulting tax shortfall. Provided where the 176, 177 and general
taxpayer can prove that his intention was not
to defraud the authorities the penalty may be
reduced or decreased accordingly.
11. Penalty for Obstructing Tax Officials to Higher of Rs. 25,000, or 100% of the amount 175 and 177
Access to Premises etc. of tax involved.
12. Penalty for Concealment of income, Higher of Rs. 25,000 or an amount equal to 182
furnishing of inaccurate particulars or the tax which person sought to evade.
claiming of any deduction not actually However, if a person proves that he made
incurred or any act referred to in u/s 20, claim of exemption or expenditure
111 and general unknowingly the penalty will not be imposed.
Provided that where the taxpayer admits his
default he may voluntarily pay the amount of
penalty due under this section.

456 Conceptual Approach to Taxes


Offences and Prosecutions Chapter-24

13. Penalty for Obstruction in Performance of Rs. 25,000 209, 210 and general
Duties
14. General Higher of Rs. 5,000 or 5% of the amount of
If a person contravenes any of the tax involved.
provision of ITO, 2001, for which no
penalty has, specifically, been provided
15. Failure to Collect or Deduct Tax u/s 148, Higher of Rs. 25,000 or 10% of the amount 182
149, 150, 151, 152, 153, 153A, 154, 155, of tax.
156, 156A, 156B, 158, 160, 231A, 231B,
233, 233A, 234, 234A, 235, 236 and 236A
16. Any person who fails to display NTN Such person shall pay a penalty of Rs. 5,000 181C
Certificate at the place of business as
required under this Ordinance or the rules
made there under.
All the above penalties shall be levied in a consistent manner after passing an order in writing by the Commissioner Inland
Revenue, Commissioner Inland Revenue (Appeals) or the Appellate Tribunal. Moreover, it is necessary that person on
whom the penalty is being imposed, is provided an opportunity of being heard.
If subsequently an order is revised by an income tax authority due to which the amount on which the penalty was based is
reduced, the penalty will also be reduced accordingly.
Exemption from penalty and default surcharge [U/s 183]
The Federal Government (by notification) and the Board (by order) through official Gazette in writing may exempt any
person or class of persons from payment of the whole or part of the penalty and default surcharge on such conditions
and limitations as may be specified therein.

2. PROSECUTION

Default Prosecution Section


1. Prosecution for non-compliance with certain Fine or imprisonment for a term not 191
Statutory Obligations exceeding 1 year or both.

Where a person, without reasonable excuse, Shall further punishable with fine of
fails to - Rs.50,000 or imprisonment for a term
(a) comply with a notice to file the income tax not exceeding 2 years, or both.
return or wealth statement,
(b) pay advance tax,
(c) comply to collect or deduct tax and pay the
tax,
(d) comply to a notice to regarding recovery of
tax on holding money on behalf of a
taxpayer or in providing information or
evidence required by tax department,
(e) comply with the requirements of u/s 141(3)
or (4),or
(f) provide reasonable facilities and assistance
to the CIR in effective exercise of the right
to access and search premises,

If a person continues the offence and not filed


the income tax return or wealth statement,
without reasonable excuse, within the period
specified by the Court,
2. Prosecution for false statement in verification Rs.100,000 or imprisonment for a term 192
not exceeding 3 years, or both.

3. Prosecution for concealment of income to the Imprisonment up to 2 years or with fine 192A
extent that has impact on revenue of or both.
Rs.500,000 or more

Conceptual Approach to Taxes 457


Offences and Prosecutions Chapter-24

4. Prosecution for failure to maintain records (a) where the failure was deliberate, a 193
fine not exceeding Rs.50,000 or
imprisonment for a term not
exceeding 2 years, or both; or
(b) in any other case, a fine not
exceeding Rs.50,000.
5. Prosecution for improper use of National Tax Fine not exceeding Rs.50,000 or 194
Number Card imprisonment for a term not exceeding
2 years, or both.
6. Prosecution for making false or misleading 195
statements
A person who -
(a) makes a statement to an income tax (i) where the statement or omission
authority that is false or misleading in a was made knowingly or recklessly,
material particular; or with a fine or imprisonment for a
(b) omits from a statement made to an term not exceeding 2 years, or both;
income tax authority any matter or thing (ii) in any other case, with a fine.
without which the statement is misleading
in a material particular.
7. Prosecution for obstructing an income tax Fine or imprisonment for a term not 196
authority exceeding1 year, or both.
8. Prosecution for disposal of property to prevent Fine up to Rs.100,000 or imprisonment 197
attachment for a tern not exceeding three years, or
both.
9. Prosecution for unauthorized disclosure of Fine not below Rs. 500,000 or 198 and 216
information by a public servant imprisonment for a term not exceeding
one year, or both.
10. Prosecution for abetment Fine or imprisonment for a term not 199
exceeding 3 years, or both.
3. Offences by companies and association of persons [U/s 200]
Where an offence as stated above is committed by a company, every person who, at the time the offence was
committed, was -
(a) the principal officer, a director, general manager, company secretary or other similar officer of the company; or
(b) acting or purporting to act in that capacity,
shall be guilty of the offence and all the provisions of this Ordinance shall apply accordingly.
Where an offence as stated above is committed by an AOP, every person who, at the time the offence was
committed, was a member of the association shall be, guilty of the offence and alt the provisions of this Ordinance
shall apply accordingly.
4. Institutions of prosecution proceedings without prejudice to other action [U/s 201]
Notwithstanding anything contained in any law for the time being in force, a prosecution for an offence against this
Ordinance may be instituted without prejudice to any other liability incurred by any person under the Income tax
Ordinance, 2001.
5. Power to compound offences [U/s 202]
If a person has committed any offence, the Chief Commissioner may, with the prior approval of the Board, either
before or after the institution of proceedings, compound such offence on payment of tax due along with default
surcharge and penalty under the Income tax Ordinance, 2001.
6. Trial by Special Judge [U/s 203]
(1) The Federal Government may, by notification in the official Gazette, appoint as many special judges as it may
consider necessary, and where it appoints more than one Special judge, it shall specify in the notification the
territorial limits within which each of them shall exercise jurisdiction;
Provided the Federal Government may, by notification in official Gazette, declare that a special Judge
appointed under section 185 of the customs Act, 1969 shall have jurisdiction to try offences under this
Ordinance.
(1A) A Special judge shall be a person who is or has been a Sessions Judge and shall, on appointment, have the
jurisdiction to try exclusively an offence punishable under this Part other than an offence referred to in u/s 198.

458 Conceptual Approach to Taxes


Offences and Prosecutions Chapter-24

(1B) The provisions of the Code of Criminal Procedure shall apply to the proceedings of the court of a Special judge
the court of Special judge shall be deemed to be a Court of Sessions trying cases, and a person conducting
prosecution before the court of a Special judge shall be deemed to be a Public Prosecutor.
(2) A Special judge shall take cognisance of, and have Jurisdiction to try, an offence try only upon a complaint in
writing made by the Commissioner Inland Revenue.
(3) The Federal Government may, by order in writing, direct the transfer, at any stage of the trial, of any case from
the court of one Special judge to the court of another Special Judge for disposal, whenever it appears to the
Federal Government that such transfer shall promote the ends of justice or tends to the general convenience
of parties or witnesses.
(4) In respect of a case transferred to a Special judge by virtue of sub-section (1) or under sub-section (3), such
Judge shall not, by reason of the said transfer, be bound to recall and rehear any witness who has given
evidence in the case before the transfer and may act on the evidence already recorded by or produced before
the court which tried the case before the transfer,
7. Appeal against the order of a Special Judge [U/s 203A]
An appeal against the order of a special Judge shall lie to the respective High Court of a Province within 30 days of
the passing of the order and it shall be heard as an appeal under the Code of Criminal Procedure 1898 by a single
Judge of the High Court.
8. Power to tender immunity from prosecution [U/s 204]
(1) The Federal Government may, for the purpose of obtaining the evidence of any person appearing to have
been directly or indirectly concerned in, or privy to the concealment of income or to the evasion of tax, tender
to such person immunity from prosecution for any offence under this Ordinance or under the Pakistan Penal
Code or under any other Federal Law on condition of the person making full and true disclosure of the whole
circumstances related to the concealment of income or evasion of tax.
(2) A tender of immunity made to, and accepted by, the person concerned shall render the person immune from
prosecution for any offence in respect of which the tender was made and to the extent specified in the
immunity.
(3) If it appears to the Federal Government that any person to whom immunity has been tendered under this
section has not complied with the conditions on which the tender was made or is concealing anything or giving
false evidence, the Federal Government may withdraw the immunity, and any-such person may be tried for the
offence in respect of which the tender of immunity was made or for any other offence of which the person
appears to have been guilty in connection with the same matter.

Conceptual Approach to Taxes 459


Offences and Prosecutions Chapter-24

MULTIPLE CHOICE QUESTIONS


Q.1. Penalty for failure to file income tax return under section 114 is ___% of the tax payable.
(a) 0.1
(b) 0.2
(c) 0.3
(d) 0.4
Q.2. Penalty for failure to file income tax return under section 114 is subject to a minimum penalty of Rs. _______.
(a) 6,000
(b) 20,000
(c) 7,000
(d) 8,000
Q.3. Penalty for failure to file income tax return under section 114 is subject to a maximum penalty of _______% of tax
payable.
(a) 50
(b) 10
(c) 25
(d) 30
Q.4. Penalty for failure to issue cash memo, invoice or receipt is Rs. 5,000 or 3% of the amount of tax involved whichever
is ______.
(a) higher
(b) lower
(c) higher than Rs. 100,000
(d) all of above
Q.5. Penalty for failure to apply for registration and failure to notify a change of material nature shall pay a penalty of
Rs.______________.
(a) 6,000
(b) 5,000
(c) 7,000
(d) 8,000
Q.6. For the first time default in depositing the amount of tax or any part thereof, penalty is ___% of the amount of the tax
in default.
(a) 8
(b) 9
(c) 10
(d) 11
Q.7. For the second time default in depositing the amount of tax or any part thereof, penalty is ___% of the amount of the
tax in default.
(a) 25
(b) 29
(c) 30
(d) 35
Q.8. For the third time and subsequent default in depositing the amount of tax or any part thereof, penalty is ___% of the
amount of the tax in default.
(a) 25
(b) 50

460 Conceptual Approach to Taxes


Offences and Prosecutions Chapter-24

(c) 75
(d) 100
Q.9. In case of repetition of erroneous calculation in the return for more than one year resulting lesser amount of tax paid,
penalty is higher of Rs. _________.
(a) 10,000 or 3%
(b) 1,000 or 1%
(c) 5,000 or 3%
(d) 20,000 or 5%
Q.10. Penalty for failure to maintain records under the Ordinance and rules is higher of Rs.______.
(a) 10,000 or 5%
(b) 1,000 or 5%
(c) 5,000 or 3%
(d) 20,000 or 5%
Q.11. Penalty for non-compliance with the provisions of total audit after first notice shall be Rs.____.
(a) 6,000
(b) 25,000
(c) 7,000
(d) 8,000
Q.12. Penalty for non-compliance with the provisions of total audit after second notice shall be Rs.____.
(a) 50,000
(b) 15,000
(c) 20,000
(d) 25,000
Q.13. Penalty for non-compliance with the provisions of total audit after third notice shall be Rs.____.
(a) 50,000
(b) 75,000
(c) 100,000
(d) 125,000
Q.14. Penalty for failure to furnish information for first default is Rs. ______.
(a) 1,000
(b) 5,000
(c) 2,000
(d) 3,000
Q.15. Penalty for failure to furnish information for subsequent default is Rs. ______.
(a) 10,000
(b) 15,000
(c) 12,000
(d) 13,000
Q.16. Penalty for obstructing tax officials to access to premises shall be higher of Rs. _________ of the amount of tax
involved
(a) 10,000 or 10%
(b) 20,000 or 20%
(c) 25,000 or 100%
(d) 30,000 or 100%

Conceptual Approach to Taxes 461


Offences and Prosecutions Chapter-24

Q.17. Penalty for concealment of income shall be higher of Rs. ______ or an amount equal to the tax which person sought
to evade.
(a) 10,000
(b) 15,000
(c) 20,000
(d) 25,000
Q.18. Penalty for obstruction in performance of duties shall be Rs. _________.
(a) 10,000
(b) 15,000
(c) 20,000
(d) 25,000
Q.19. If a person contravenes any of the provision of ITO, 2001, for which no penalty has, specifically, been provided he
shall pay a penalty higher of Rs. _______ of the amount of tax involved.
(a) 10,000 or 5%
(b) 1,000 or 5%
(c) 5,000 or 5%
(d) 20,000 or 5%
Q.20. Penalty for failure to deduct or collect tax is higher of Rs __________ of the amount of tax.
(a) 10,000 or 5%
(b) 1,000 or 5%
(c) 5,000 or 5%
(d) 25,000 or 10%
Q.21. The __________ (by notification) and the Board (by order) through official Gazette in writing may exempt any person
or class of persons from payment of the whole or part of the penalty and default surcharge on such conditions and
limitations as may be specified therein.
(a) Federal Government
(b) Provincial Government
(c) Appellate Tribunal Inland Revenue
(d) all of above
Q.22. Any person who makes a statement in any verification in any return or other document furnished under this
Ordinance which is false and which the person knows or believes to be false, the person shall be punishable with a
fine up to Rs._______ or imprisonment or both.
(a) 200,000
(b) 100,000
(c) 400,000
(d) 50,000
Q.23. Where any person, in the course of any income tax proceedings, concealed income or furnished inaccurate
particulars of such income to the extent that has impact on revenue of Rs.______or more shall punishable with
imprisonment up to two years or with fine or both.
(a) 500,000
(b) 100,000
(c) 400,000
(d) 50,000
Q.24. A person who knowingly or recklessly uses a false National Tax Number Certificate on a return or other document
prescribed shall punishable with a fine not exceeding Rs.__________or imprisonment for a term not exceeding two
years, or both.
(a) 500,000

462 Conceptual Approach to Taxes


Offences and Prosecutions Chapter-24

(b) 50,000
(c) 100,000
(d) 10,000
Q.25. Where the owner of any property sells the property after the receipt of a notice from the CIR to prevent the CIR from
attaching it, shall punishable with a fine up to Rs.__________or imprisonment for a tern not exceeding three years, or
both.
(a) 500,000
(b) 50,000
(c) 100,000
(d) 10,000
Q.26. Where a person knowingly and will-fully aids, abets, assists, incites or induces another person to commit an offence
under the Income Tax Ordinance, 2001 the first mentioned person shall punishable with a fine or imprisonment for a
term not exceeding ________years, or both.
(a) 3
(b) 2
(c) 4
(d) 5
Q.27. Where an offence is committed by a company, _______ shall be guilty of the offence.
(a) principal officer
(b) director
(c) general manager or secretary
(d) all of above
Q.28. Where an offence is committed by an AOP, _______ shall be guilty of the offence.
(a) employee
(b) members
(c) legal representative
(d) all of above

ANSWERS
1 (a) 2 (b) 3 (a) 4 (a) 5 (b)
6 (c) 7 (a) 8 (b) 9 (c) 10 (a)
11 (b) 12 (a) 13 (c) 14 (b) 15 (a)
16 (c) 17 (d) 18 (d) 19 (c) 20 (d)
21 (a) 22 (b) 23 (a) 24 (b) 25 (c)
26 (a) 27 (d) 28 (b)

Conceptual Approach to Taxes 463


Offences and Prosecutions Chapter-24

ICMAP PAST PAPERS THEORECTICAL QUESTIONS


Q. NO. 2 (a)(iii) WINTER 2009 M/s Z Attari Ltd., has a tax year ending on 30th June, is feeling difficulty in filing the return in
time. You are required to advice the management on the following issues:
What penalties shall be applicable in respect of the following contraventions of the law:
(a) Failure in furnishing a return;
(b) Non-payment of tax or failure in payment of tax
(c) Concealment of Income.
Q. NO. 2 (b) WINTER 2006 Discuss the following under Income tax Ordinance, 2001
(i) Penalty for failure to furnish a return or statement u/s 182
Q. NO. 3 (a) SUMMER 2006 What are the conditions for imposition of penalty as given in section 190 of Income tax
Ordinance, 2001?
Q. NO. 3 (b) SUMMER 2006 What are the penalties under ITO-2001 for failure to:-
Maintain records (ii) Give notice for discontinuance of business
Q. NO. 8 (b) WINTER 2004 Describe details of the following penalties, as given in Income tax Ordinance, 2001.
1- Penalty for failure to furnish a return or statement
2- Penalty for non-compliance with the notice

464 Conceptual Approach to Taxes


Insurance Business Chapter-25

Chapter

25 INSURANCE BUSINESS

Topic covered
S. No.
(For CA Mod F & ICMAP students)
PART I
1 Definition
2 Categories of insurance
3 Taxability of insurance business
4 Computation of the surplus
5 General insurance
6 Mutual insurance association
7 Practical example
8 MCQs with solutions

Definition
Insurance is defined in the Insurance Ordinance 2000 in the following words:
Insurance means the business of entering into and carrying out policies or contracts, by whatever, name called, whereby,
in consideration of a premium received, a person promises to make payment to another person contingent upon the
happening of an event, specified in the contract, on the happening of which the second name person suffers loss and
includes reassurance and retrocession. Provided that a contract of life insurance shall be deemed to be a contract of
Insurance notwithstanding that it may not comply with the definition set out in this clause.
Categories
The Insurance is divided into two broad categories i.e. Life and General Insurance, General Insurance includes Fire,
Accidental, Marine, medical Insurance etc. The reporting pattern of financial statements of the Insurance Companies is
different from normal Accounts. A life Insurance Business is required to prepare 9 reports as a part of financial statements.
Whereas General business related companies are required to prepare 10 reports.
Taxability
Although normal rates of tax are applicable to Insurance companies, however, method for determination of taxable income is
different and section 99 deals with this issue in the following way:
Special provisions relating to insurance business: The profits and gains of any insurance business shall be computed in
accordance with the rules in the Fourth Schedule.
Rule 1-3 of the Fourth Schedule are applicable to Life Insurance Business. The said rules stipulate the provisions regarding
computation of profit and gains on the life Insurance business in the following manner:
The profits and gains of a taxpayer carrying on life insurance business chargeable under the head "Income from Business
shall be computed separately from the taxpayer's income from other business.
The income from life insurance business is liable to tax under income from business and the said income will be worked out
in the manner laid down hereunder:
The profits and gains of a life insurance business shall be the current years surplus appropriated to profit and loss account
prepared under the Insurance Ordinance, 2000, as per advice of the Appointed Actuary, net of adjustments under sections
22(8), 23(8) and 23(11) of the Insurance Ordinance, 2000 so as to exclude from it any expenditure other than expenditure
which is, under the provisions of Part IV of Chapter III, allowed as a deduction in computing profits and gains of a business
to the extent of the proportion of surplus not distributed to policy holders.

Conceptual Approach to Taxes 465


Insurance Business Chapter-25

From the accounting perspective, it is important to place on record that a life insurance contract is invariably a long term
contract, indeed a long term investment contract. Therefore, at the date of balance sheet, the liability in respect of future
claims has to be given due consideration. This is so because in future years, the premium to be received against ongoing
policies would be much less than the claims which may become payable against those policies. This difference is known as
net liability. It then follows that unless a life insurance company has reserves equal to the net liability, the company would
be considered to have made no profit.
However, under the tax law, the formula to compute the Profit and gains of life insurance business is as under:
Profit and gains of a life Insurance Business = Annual Average of surplus
Less adjustment of surplus or deficit disclosed by actuarial valuation made for the last inter valuation period ending before
the tax period
Add surplus or deficit included therein which was made in any earlier inter valuation period and expenditure allowed as a
deduction
Computation of the surplus
The following provisions shall apply in computing the surplus:
(a) The amounts
paid to,
or reserved for,
or expended on behalf of policy-holders
Shall be allowed as a deduction;
However:
(a) in the first computation of the surplus, no account shall be taken of amounts to the extent to which they are
paid out, or in respect of any surplus brought forward from a previous inter-valuation period; and
(b) if any amount reserved for policy-holders ceases to be so reserved, and is not paid to, or expended on behalf
of policy-holders, the sums previously allowed as a deduction under this Ordinance shall be treated as part of
the respective statutory fund for the tax year in which the amount ceased to be so reserved.
The transfer of any amount to reserve although allowed as deduction in computing the surplus. However, in case the
said reserve is not utilized for the purpose intended, then, the whole of the sum set apart for reserve and allowed as
deduction previously will be added in the surplus of the prevailing tax year of that time.
(b) Any amount
either written off or
reserved in the accounts, or
through the actuarial valuation balance sheet to meet depreciation, or loss on the realization of investments
shall be included in the surplus
However, any sums taken credit for in the accounts or actuarial valuation balance sheet on account of appreciation or
gains on the realization of investments shall be allowed as deduction; and
if it appears to the CIR, after consultation with the Securities and Exchange Commission of Pakistan (SECP), that the
rate of profit on debt or other factors employed in determining the liability in respect of outstanding policies is
inconsistent with the valuation of investments so as artificially to reduce the surplus, the CIR may make such
adjustment to the allowance for depreciation, or in respect of appreciation, of such investment as the CIR thinks
reasonable
The term investment is defined in the following manner:
"investments " includes all forms of shares, debentures, bonds, deposits and other securities, derivative instruments, and
includes immovable property whether or not occupied by the insurer;
In view of the aforesaid provision of the schedule, it clearly transpires that gain / loss due to appreciation / diminution in the
value of shares is allowable deduction for the insurance business. The said rules further states that:
(b) profit on debt accrued in the inter-valuation period in respect of any securities of the FG which have been issued or
declared to be income tax-free shall not be excluded, but shall be exempt from tax.
Rule 6A states that the capital gains on the sale of shares etc. shall not be included in the taxable income of the an
insurance company

466 Conceptual Approach to Taxes


Insurance Business Chapter-25

In computing income under this Schedule, there shall not be included capital gains , being income from the sale of
modaraba certificates or any instrument of redeemable capital as defined in the Companies Ordinance, 1984, listed on any
stock exchange in Pakistan or shares of a public company (as defined in section2(47) and the PTC issued by the
Government of Pakistan, derived up to tax year ending on the thirtieth day of June, 2010.
General insurance
Rule 5 of the Fourth Schedule deals with the General Insurance Business. The said Rule states that:
The profits and gains of any business of insurance (other than life insurance) shall be taken to be the balance of the profits
disclosed by the annual accounts required under the Insurance Ordinance, 2000, to be furnished to the Securities and
Exchange Commission of Pakistan (SECP) subject to the following adjustments:
(a) any expenditure or allowance, or any reserve or provision for any expenditure, or the amount of any tax deducted at
source from dividends or profit on debt received which is not deductible in computing the income chargeable under
the head Income from Business shall be excluded;
(b) subject to the provisions of rule 6A, any amount of investment written off shall be allowed as a deduction, but any
amount taken to reserve to meet depreciation of investments shall not be allowed as a deduction, and any sums
taken credit for in the accounts on account of appreciation of investment shall not be treated as part of the profits and
gains, unless these have been crystallized as gains or losses on the realization of investments; and
(c) no Reduction shall be allowed for any expenditure, allowance, reserve, or provision in excess of the limits laid down
in the Insurance Ordinance, 2000, unless the excess is allowed by the Securities and Exchange Commission of
Pakistan (SECP) and is incurred in deriving income chargeable to tax; and
(d) no deduction shall be allowed for any expenditure incurred on account of insurance premium or re-insurance
premium paid to an overseas insurance or reinsurance company or a local agent of an overseas insurance company
until tax at the rate of 5% is withheld on the gross amount of insurance or re-insurance premium.
Although simple manner of computation of tax on income of Insurance companies is to apply applicable rate to the
accounting profit of the company irrespective of the fact what are the components of the income and what are their
respective classification. No estimation / additions u/s 21, 22, 23, 24 or otherwise can be made except for provisions and
reserves which are inadmissible under the law e.g. provision for bad debts, etc.
Mutual insurance association
Rule 6: These rules shall also apply to the assessment of the profits and gains of any business of insurance carried on by a
mutual insurance association and such profits and gains shall be chargeable to tax under head "Income from Business".
(6A) Exemption of Capital Gains from the sale of shares [Omitted through Finance Act, 2015]
(6B) Capital Gains on disposal of shares of listed companies, PTC vouchers, modaraba certificate or instruments of
redeemable capital and derivative products shall be taxed at the following rates;

S. No. Period Tax Year Rate of tax


(1) (2) (3) (4)
1. Where holding period of a security is less than 12 months. 2016 15%
Where holding period of a security is equal or more than 12 months
2. 2016 12.5%
but less than 24 months.
Where holding period of a security is equal or more than 24 months 2016 7.5 %
3.
but less than four years.
3. Where holding period of a security is equal or more than four years. 2016 0%
(6C) Notwithstanding anything contained in this Ordinance, where loss on disposal of securities is sustained in a tax year,
the loss shall be set off only against the gain from any other securities chargeable to tax under Rule 6B and no loss
shall be carried forward to the subsequent tax year.
(6D) The provisions of section 4B shall apply to the taxpayers under this schedule and taxed at the rates specified in
Division IIA of Part I of the First Schedule.
Example:
Arshad Insurance Company Ltd. deals in General Insurance business and the company earned an accounting profit
of Rs. 21 million during the tax year 2015. Perusal of record shows that this profit includes followings:
No Tax has been deducted from the Legal and professional-charges amounting to Rs. 750,000.
The company claimed provision for bad debts amounting to Rs. 500,000 on account of policies premium
receivable but not received from some recurring parties.

Conceptual Approach to Taxes 467


Insurance Business Chapter-25

Company paid lease rental amounting to Rs. 655,000 to the leasing companies. The Accounting depreciation
on leasehold assets is Rs. 235,000 and lease financial charges aggregates to Rs. 175,000.
The income includes loss due to decline in the value of investments on the date of balance sheet amounting to
Rs. 325,000.
The company claimed a loss due to impairment of fixed assets amounting to Rs. 195,000.
Dividend Income of the company was Rs 2,000,000 during the year.
You are provided with above information for computation of taxable income and tax liability for the tax year 2015.
Solution
Arshad Insurance Company Ltd
Computation of taxable income and tax liability
For the tax year 2016 Rs.
Accounting Profit for the Year 21,000,000
Add
Impairment loss on fixed assets 195,000
Inadmissible legal and professional charges 750,000
Loss due to depreciation in value of investment 325,000
Depreciation on leasehold assets 235,000
Lease finance charges 175,000
Provision for bad debts 500,000
2,180,000
Less
Lease rentals 655,000
Taxable Income 22,525,000
Computation of tax liability
Tax on Taxable income Rs 22,525,000 @ 32% 7,280,000
Total Tax 7,280,000
Example:
ABC and Company Ltd. deals in life and General Insurance business. Followings are brought from accounting
records.
Accounting profit before tax Rs. 42.390 m
Dividend income Rs. 1.5 m
Provision for claims Rs. 5 m
Accounting depreciation Rs. 8.586 m
Tax depreciation Rs. 9.380 m
Profit on debt from securities of Federal Government Rs. 0.312 m
Claims are paid out of provisions but not claimed as deduction Rs. 2 m
Amount of provision for claims ceases to be so reserved Rs. 0.998 m
Remuneration exp of chief executive Rs.0.840 m
Remuneration of chief executive disallowed by SECP Rs. 0.199 m
Payment of re-insurance premium to an overseas re-insurance company without
the deduction of with-holding Tax (@ 5%) Rs. 1.75 m
Capital gain from sale of PTC vouchers issued by the Govt. of Pakistan
(where the holding period of such vouchers is 7 months) Rs. 0.897 m
Loss on disposal of Modaraba certificates Rs. 0.543

468 Conceptual Approach to Taxes


Insurance Business Chapter-25

Solution:
ABC Insurance Company Ltd.
Computation of taxable income and tax liability
For the tax year 2016
Income from business Rs.
Profit before Taxation as per P and L Account 42,390,000

Add back: Depreciation for separate consideration 8,586,000


Amount of provision for claims ceases to be so reserved 998,000
Remuneration of chief executive disallowed by SECP 199,000
Payment of insurance premium to an overseas re-insurance company 1,750,000
53,923,000

Less: Depreciation admissible as per rules 9,380,000


Total Income from business and other sources 44,543,000

Add: income from capital gain


Gain from sale of PTC vouchers 897,000
Less: Loss on disposal of Modaraba certificates 543,000
Net Capital Gain 354,000
Total income 44,897,000

Tax liability
32% tax on business and other income (44,543,000 x 32%) 14,253,760
15% tax on Capital Gain (354,000 x 15%) 53,100
Total tax liability 14,306,860

NOTES:
N-1 Dividend Income falls under normal tax regime.
N-2 Provision for claims is allowed as a deduction u/c (a) of sub-rule (1) of rule 3 of 4th schedule.
N-3 Profit on debt from securities of Federal Government is exempt from tax.
N-4 Payment of claims, paid out of provisions, shall not be allowed as deduction u/c (a) of sub-rule (2) of rule 3 of 4th
schedule.
N-5 As with-holding tax @ 5% is not deducted so payment of re-insurance premium to an overseas re-insurance company
is inadmissible.
N-6 Loss on Modaraba certificates can be set off against Capital gain.

Conceptual Approach to Taxes 469


Insurance Business Chapter-25

MULTIPLE CHOICE QUESTIONS


Q.1. Profit and gains from life insurance business shall be computed ________ other businesses under the head income
from business.
(a) separately from
(b) together with
(c) none of above
Q.2. Neither dividend nor any payment to debenture holders shall be declared or paid other than from the ___________.
(a) cash and bank balance
(b) shareholders fund
(c) sinking fund
(d) all of above
Q.3. A deduction of the amounts paid to, or reserved for, or expended on behalf of policy-holder shall be ______.
(a) allowed
(b) not allowed
(c) ignored
(d) added to income
Q.4. Profit on debt accrued in the inter-valuation period in respect of any securities of the Federal Government shall be
_________.
(a) taxable
(b) exempt from tax
(c) treated as income from other sources
(d) none of above
Q.5. The CIR may make reasonable adjustments to the allowance for depreciation / appreciation if it appears to him, that
the outstanding policies are _____ with the valuation of investment so as to artificially reduce the surplus.
(a) not in accordance with Insurance Ordinance
(b) consistent
(c) inconsistent
(d) all of above
Q.6. Any expenditure which is not deductible in computing the income chargeable under the head ___________ shall be
excluded for computation of income from insurance.
(a) Income from other sources
(b) capital gains
(c) Income from business
(d) Income from property
Q.7. Any payment of insurance premium or reinsurance premium to an overseas insurance or re-insurance company shall
be allowed as a deduction if tax at the rate of _____% is withheld on the gross amount of insurance or re-insurance
premium.
(a) 4
(b) 5
(c) 6
(d) 7
Q.8. Capital Gain, derived up to tax year ending on the thirtieth day of June, ____, is exempt from tax arising from the sale
of modaraba certificate.

470 Conceptual Approach to Taxes


Insurance Business Chapter-25

(a) 2013
(b) 2012
(c) 2011
(d) 2010
Q.9. Loss on disposal of securities shall be set off only against the gain from any other securities chargeable to tax under
Rule 6B and unadjusted loss shall _______.
(a) carried forward for 10 years
(b) carried forward for 6 years
(c) not be carried forward
(d) carried forward subject to certain conditions
Q.10. Investments include all forms of ________:
(a) shares
(b) debentures
(c) bonds
(d) all of above

ANSWERS

1 (a) 2 (b) 3 (a) 4 (b) 5 (c)


6 (c) 7 (b) 8 (d) 9 (c) 10 (d)

Conceptual Approach to Taxes 471


Insurance Business Chapter-25

472 Conceptual Approach to Taxes


Oil, Natural Gas and Other Mineral Deposits Chapter-26

Chapter

26 OIL, NATURAL GAS AND OTHER


MINERAL DEPOSITS

Topic covered
Section
(For CA Mod F & ICMAP students)
Special provisions relating to the production of oil and natural gas & exploration & extraction of other
100
mineral deposits
th
5 Schedule Rules for the computation of the profits & gains from the exploration & production of petroleum PART I & II
Practical example
MCQs with answers
Special provisions relating to the production of oil and natural gas, and exploration and extraction of other mineral
deposits [Sec. 100]
(1) Rues in Part I of 5th schedule shall apply in calculating the profits and gains from the exploration and production of
petroleum including natural gas, from refineries set up at Dhodak and Bobi fields, the pipeline operations of
exploration and production companies, the manufacture and sale of liquefied petroleum gas or compressed natural
gas and in calculating tax payable thereon.
(2) Sub section (1) shall only apply after 24th day of September, 1954 and onward.
(3) Rues in Part II of 5th schedule shall apply in calculating the profits and gains from the exploration and extraction of
such mineral deposits of a wasting nature (not being petroleum or natural gas) as may be specified by the Federal
government, carried on by the person in Pakistan.
The Fifth Schedule
Part I
Rules for the computation of the profits and gains from the exploration and production of petroleum
1. Exploration and production of petroleum as a separate business
Where any person carries on, or is treated as carrying on, under an agreement with the Federal Government, any
business which consists of, or includes, the exploration or production of petroleum in Pakistan or setting up refineries
at Dhodak and Bobi fields, income of exploration and production companies from pipeline operations, and
manufacture and sale of liquefied petroleum gas or compressed natural gas, such business or part thereof, as the
case may be, shall be, for the purposes of this Ordinance, treated as a separate business undertaking and the profits
and gains of such undertaking shall be computed separately from the income, profits, or gains from any other
business, if any, carried on by the person.
2. Computation of profits
(1) The profits and gains shall be computed in the manner applicable to income, profits and gains chargeable
under the head Income from Business.
(2) Where such person incurs any expenditure on searching for or discovering and testing a petroleum deposit or
winning access thereto but the search exploration, enquiry upon which expenditure is incurred is given up
before the commencement of commercial production, the expenditure allocable to a surrendered area or to the
drilling of a dry-hole shall be treated as lost at the time of the surrender of the area or the completion of the
dry-hole, as the case may be.
(3) Where the agreement provides that any portion of expenditure treated as lost (referred to as the said loss)
shall be treated in any of the following ways:
(a) The said loss shall be set off against any income (other than income from dividend) chargeable under
any head of that year and where the said loss cannot be fully set off then the unadjusted loss can be
carried forward up to the maximum of 6 years.
(b) The said loss in any year shall be set off against the income of such undertaking of the tax year in which
commercial production has commenced and where the loss cannot be wholly set off against the income
of such undertaking of that year, the portion not set off against the income, if any, of such undertaking of
that year, and if it cannot be wholly so set off the amount of loss not so set off shall be carried forward
for more than 10 years.

Conceptual Approach to Taxes 473


Oil, Natural Gas and Other Mineral Deposits Chapter-26

(4) After the commencement of commercial production, all the expenditure incurred prior thereto and not [treated
as] lost under sub-rule (2) and not represented by physical assets in use at the time the commercial production
shall be allowed as a deduction, so, however, that the portion of such deduction to be so allowed in any year
shall be such amount not exceeding 10% of the aggregate amount deductible in respect of onshore areas, and
not exceeding 25% for offshore areas, as may be selected by taxpayer.
(4A) Notwithstanding anything contained in this Schedule, a person, for tax year 2012 and onward, may opt to pay
tax at the rate of 40% of the profits and gains, net of royalty, derived by a petroleum exploration and
production undertaking:
Provided that this option shall be available subject to withdrawal of appeals, references and petitions on the
issue of tax rate pending before any appellate forum:
Provided further that the outstanding tax liability created under this Ordinance up to tax year 2011 is paid by
the 30th June, 2012:
Provided also that this option is available only for one time and shall be irrevocable.
(5) Any expenditure, including a royalty paid to the Federal Government by an onshore petroleum exploration and
production undertaking on, or after, the 1st July 2001 (not being in the nature of capital expenditure or personal
expenses of the taxpayer) laid out or expended after the commencement of commercial production wholly and
exclusively for the purpose of the business of production and exploration of petroleum carried on by such
undertaking shall be allowed as a deduction, provided that
(a) No deduction shall be allowed in respect of such expenditure incurred in the acquisition of depreciable
assets to which section 22 applies or in the acquisition of an intangible to which section 24 applies;
(b) Deductions u/ss 22, 23 and 24 shall be admissible in respect a assets referred to in clause (a);
(c) A depreciation deduction shall also be allowed u/s 22 in respect of such expenditure incurred on the
acquisition of the physical assets acquired before the commencement of commercial production and
were being used such undertaking on and after that date, as if such assets had been acquired at the
time of the commencement of commercial production at their original cost, by the amount of
depreciation deduction, if any, previously allowed to be deducted under this Ordinance.
(6) If, in any year, the deduction allowed Part IV of Chapter III and sub-rules (3) and (4) exceed the gross receipts
from the sale of petroleum produced in Pakistan, such excess shall be set off against other income (not being
dividends) and carried forward in the manner subject to the limitations in section 57, so however that no
portion of such excess shall be carried forward for more than six years.
(6) The limitation of six years specified in [sub-rule] (6) shall not apply to deprecation allowed to a person carrying
on the business of offshore petroleum exploration and production, in respect of any machinery, plant or other
equipment used in such exploration or production.
(7) For the purpose of the section 22, where any asset used by a person in the exploration and production of
petroleum is exported or transferred out of Pakistan, the person shall be treated as having made a disposal of
the asset for a consideration received equal to the cost of the asset as reduced by any depreciations allowed
under this Ordinance (other than an initial allowance u/s 23).
3. Depletion allowance:
The depletion allowance shall be allowed up to the 15% of gross receipts, not exceeding 50% of profits or gains
before deduction of such allowance, for any year ending after the date on which commercial production has
commenced.
4. Limitation on payment to federal government and taxes
(1) The aggregate of the taxes on income and other payments excluding a royalty as specified in the Pakistan
Petroleum (Production) Rules, 1949 or the Pakistan Petroleum (Exploration and Production) Rules, 1986 and
paid by an onshore petroleum exploration and production undertaking on or after 1.7.2001 to the Government
in respect of the profits or gains derived from such undertaking for a tax year shall not exceed the limits
provided for in the agreement, provided the said aggregate shall not be less than 50% of the profits or gains
derived by an onshore petroleum exploration and production undertaking and 40% of the profits or gains
derived by an offshore petroleum exploration and production undertaking, before deduction of the payment to
the Federal Government.
(2) In respect of any tax year commencing on or after 1.7.2002, the aggregate referred to in sub-clause (1) shall
not be less than 40% of the profit or gains derived by an onshore petroleum exploration and production
undertaking before the deduction of payment excluding royalty paid by an onshore petroleum exploration and
production undertaking to the Federal Government.
(3) If, in respect of any tax year, the aggregate of the taxes on income and payments to the Federal Government
is greater or less than the amount provided for in the agreement, an additional amount of tax shall be payable
by the taxpayer, or an abatement of tax shall be allowed to the taxpayer, as the case may be, so as to make
the aggregate of the taxes on income and payments to the Federal Government equal to the amount provided
for in the agreement.

474 Conceptual Approach to Taxes


Oil, Natural Gas and Other Mineral Deposits Chapter-26

(4) If, in respect of any year, the payments to the Federal Government exceed the amount provided for in the
agreement, so much of the excess as consists of any tax or levy referred to in sub-clause (b) of clause (3) of
rule 6 shall be carried forward and treated, for the purposes of this rule, as payments to the Federal
Government for the succeeding year, provided that the whole of the payments to the Federal Government
exceeding the amount provided for in such agreement may be carried forward if so provided for in any
agreement with a taxpayer made before the first day of 1970.
5. Decommissioning cost:
With effect from the Tax Year 2010. "Decommissioning Cost" as certified by a Chartered Accountant or a Cost
Accountant, in the manner prescribed, shall be allowed over a period of ten years or the life of the development and
production or mining lease whichever is less, starting from the year of commencement of commercial production or
commenced prior to the 1 July, 2010, deduction for decommissioning cost as referred earlier shall be allowed from
the Tax Year 2010 over the period of ten years or the remaining lift of the development and production or mining
lease, whichever is less.
The provisions of section 4B shall apply to the taxpayers under this Part and taxed at the rates specified in Division
IIA of Part I of the First Schedule.
6. Provision relating to rules:
The Federal Board of Revenue may make rules for the purposes of any matter connected with, or incidental to the
operation of this Part.
7. Definitions
(1) Agreement means an agreement entered into between the Federal Government and a taxpayer for the
exploration and production of petroleum in Pakistan.
(2) Commercial Production means production as determined by Federal Government.
(3) Payment to Federal Government means amounts payable to the Federal Government or to any Federal
Government authority in Pakistan-
(a) in respect of royalties as specified in the Pakistan Petroleum (production) Rules, 1949 or the Pakistan
Petroleum (Exploration and Production) Rules, 1986.
(b) In respect of any tax or levy imposed in Pakistan peculiarly applicable to oil production or to extractive
industries or any of them and not generally imposed upon oil industrial and commercial activities.
(4) Petroleum means crude oil, natural gas and case-head petroleum spirits as defined in the Pakistan
Petroleum (Production) Rules 1949 or the Pakistan Petroleum (Exploration and Production) Rules 1986 but
does not include refined petroleum products.
(5) Surrender means the termination of rights with respect to an art including the expiration of rights according
to the terms of an agreement.
(6) Surrendered area means an area with respect to which the rights of the person have terminated by
surrender or by assignment or by termination of business.
(7) Taxes on Income and Tax includes income tax, but does not include payments to the Federal
Government.
(8) Well-head Value shall have the meaning assigned to it in the agreement between the Federal Government
and the taxpayer and in the absence of any such definition in the agreement, the meaning assigned to it in
Pakistan Petroleum (production) Rules, 1949 or the Pakistan Petroleum (Exploration and Production) Rules,
1986.
Part II
Rules for the computation of the profits and gains from the exploration and production of mineral deposits
(Other than petroleum)

1. Exploration and production of mineral deposits as a separate business


Where any person carries on, or is treated as carrying on, under an agreement with the Federal Government, any
business which consists of, or includes, the exploration or production of petroleum in Pakistan or setting up refineries
at Dhodak and Bobi fields, income of exploration and production companies from pipeline operations, and
manufacture and sale of liquefied petroleum gas or compressed natural gas, such business or part thereof, as the
case may be, shall be, for the purposes of this Ordinance, treated as a separate business undertaking (hereinafter
referred to as "such undertaking") and the profits and gains of such undertaking shall be computed separately from
the income, profits, or gains from any other business, if any, carried on by the person.
2. Computation of profits

(1) Subject to the provisions of this Part, the profits and gains of such undertaking shall be computed in the
manner applicable to income, profits and gains chargeable under the head "Income from Business".

Conceptual Approach to Taxes 475


Oil, Natural Gas and Other Mineral Deposits Chapter-26

(2) All expenditure on prospecting and exploration incurred by such undertaking up to the date of commercial
production shall be, to the extent to which it cannot be set off against any other income of such undertaking,
treated as a loss.
(3) The loss referred to in sub-rule (2) shall be carried forward and set off against the income of such undertaking
after the commencement of commercial production, so, however, that if it cannot be wholly set off against the
income of such undertaking of the tax year in which the commercial production had commenced, the portion
not so set off shall be carried forward to the following year and so on, but no such loss shall be carried forward
for more than ten years beginning with the year in which commercial production commenced.
(4) After the commencement of commercial production, depreciation in respect of machinery and plant for
extracting the ore shall be allowed as a deduction from the profits and gains of the tax year in which they are
used for the first time in an amount equal to the original cost of such asset and the provisions of section 22
shall apply accordingly.
Under Rule 2A
The provisions of section 4B shall apply to the taxpayers under this Part and taxed at the rates specified in
Division IIA of Part I of the First Schedule.
3. Depletion allowance

(1) In determining the profits and gains of such undertaking for any year an additional allowance (hereinafter
referred to as the "depletion allowance") shall be made equal to 20% of the taxable income of such
undertaking (before the deduction of such allowance).
(2) No deduction under sub-rule (1) shall be made unless an amount equal to the depletion allowance is set apart
and left as a reserve to be utilised for the development and expansion of such undertaking.
(3) Where a depletion allowance is made in any tax year and subsequently it is utilized for any purpose contrary to
the provisions of sub-rule (2), the amount originally allowed under this Ordinance shall be treated as having
been wrongly allowed and the Commissioner may, notwithstanding anything contained in the Ordinance, re-
compute the taxable income of the taxpayer for the relevant tax years and the provisions of section 122 shall
apply, so far as may be, thereto, the period of five years specified in the section being reckoned from the end
of the tax year in which the amount was so utilised.
Tax Exemption of Profits from Refining or Concentrating Mineral Deposits

(1) Where such undertaking is also engaged in the business of refining or concentrating in Pakistan the mineral
deposits extracted by it in Pakistan, so much of the profits and gains (hereinafter referred to as the "said
amount") derived from such business as does not exceed 10% of the capital employed in such business (such
capital being computed in accordance with such rules as may be made by the Board for the purposes of this
rule) shall be exempt from tax.
(2) Where the profits and gains of such business computed for any tax year cover a period which is less or more
than one year, the amount of profits and gains exempt under sub-rule (1) shall be the amount which bears the
same proportion to the said amount of profits as the said period bears to a period of one year.
(3) The profits and gains of the business to which this rule applies shall be computed in accordance with Part IV of
Chapter III.
(4) Nothing contained in this rule shall apply to an undertaking formed, by the splitting up or reconstruction or
reconstitution of business already in existence or by the transfer to a new business of any building, machinery,
or plant used in a business which was carried on before 1.7.1975.
(5) The provisions of this rule shall apply to the tax year in which commercial production is commenced or the loss
or allowance, if any, under sub-rules (3) or (4) of rule 2, as the case may be, has been set off or deducted in
full, whichever is the latter, and for the next following four years.
4. Provisions relating to rules
The Board may make rules for the purposes of any matter connected with, or incidental to the operation of this Part.
5. Definitions
(1) Commercial Production means production as determined by the CIR; and
(2) Petroleum means crude oil, natural gas and case-head petroleum spirits as defined in the Pakistan
Petroleum (Production) Rules 1949 or the Pakistan Petroleum (Exploration and Production) Rules 1986 but
does not include refined petroleum products.
Example:

ABC Ltd signed a concession agreement with Pakistan for oil exploration in District Dadu. You are provided with the
following data and are requested to compute tax:

476 Conceptual Approach to Taxes


Oil, Natural Gas and Other Mineral Deposits Chapter-26

(i) ABC Ltd. incurred expenditure Rs. 200,000 on oil exploration however no commercial production commenced during
year 1. Further no exploration was either abandoned or the area was surrendered by exploratory during the first year.
(ii) During year 2 expenditures on exploration were incurred to the tune of Rs. 400,000. The commercial production this
year again could not commence but the company decided to abandon one of the dry holes. The expenditure
attributable to dry hole was Rs. 200,000.
(iii) The company commenced commercial production during year 3. The expenditure up to commencement of
commercial production amounted to Rs. 1,000,000. This includes expenditure of Rs. 600,000 incurred upto year 2
and also includes Rs. 200,000 represented by physical asset in use at the time of commencement of commercial
production.
You are provided with following further information
(i) Expenditure amount to Rs. 400,000 incurred after the commercial production also includes Rs.100,000 which
represent physical assets in use.
(ii) The gross revenue from sale of crude oil was Rs.1,200,000.
(iii) Royalty @ 12.5% was payable.
Required: On the basis of the foregoing information please compute taxable income, tax payable and / or royalty refundable
or capable of being c/f.
Solution
Onshore area Offshore area
Rs. Rs. Rs. Rs.
Sale of crude oil 1,200,000 1,200,000
Royalty @ 12.5% (not admissible for offshore area) (150,000) -
Expenditure after CCP 400,000 400,000

Represents physical assets in use (100,000) (300,000) (100,000) (300,000)

Expenditure prior to CCP 1,000,000 1,000,000


Lost expenditure (200,000) (200,000)
Represents physical assets in use (200,000) (200,000)
600,000 600,000

10% of above (25% for offshore area) (60,000) (150,000)

Tax depreciation on physical assets

- Prior to CCP Rs. 200,000


- 15% normal depreciation (20% for offshore area) 30,000 40,000

- After CCP Rs. 100,000


- 25% initial allowance 25,000 25,000
- 15% normal depreciation (20% for offshore area) 11,250 (66,250) 15,000 (80,000)

Lost expenditure of year 2 (200,000) (200,000)


Gain prior to depletion allowance 423,750 470,000
Depletion allowance - 15% of Rs. 1.2 million or 50% of gain prior
to depletion allowance whichever is lower (180,000) (180,000)
Taxable income 243,750 290,000

Income tax @ 40% (for onshore area) 97,500


Income tax @ 32% (for offshore area) 92,800

Aggregate limit on income tax and other payment to the FG:


Income tax payable 92,800
Royalty payable 150,000
242,800
40% of profit of Rs. 290,000 116,000
Excess 126,800
Income tax payable Nil

1. There is no provision for refund or c/f of royalty.


2. Tax rate and aggregate limit are specified in the concession agreement but in the absence of specific information we
took 32% being normal tax and 40% aggregate limit being the minimum % mentioned in the 5th schedule.

Conceptual Approach to Taxes 477


Oil, Natural Gas and Other Mineral Deposits Chapter-26

MULTIPLE CHOICE QUESTIONS


Q.1. Profit and gains from exploration and production of petroleum including natural gas from refineries set up at Dhodak
shall be computed under________ Schedule.
(a) 5th
(b) 1st
(c) 3rd
(d) 2nd
Q.2. Profit and gains from exploration and production of petroleum shall be computed ________ other businesses.
(a) separately from
(b) together with
(c) none of above
Q.3. Profit and gains from exploration and production of petroleum shall be computed in the manner applicable to income
chargeable under the head ___________.
(a) income from other sources
(b) income from property
(c) capital gains
(d) income from business
Q.4. Where a person incurs any expenditure on searching for or discovering and testing a petroleum deposit or winning
access thereto but the search exploration, enquiry upon which expenditure incurred is given up before the
commencement of commercial production, the expenditure shall be __________.
(a) treated as lost
(b) deducted from other incomes of the taxpayer
(c) added to other incomes of the taxpayer
(d) none of above
Q.5. After commencement of commercial production deduction in respect of expenditures in the tax year is restricted upto
___% of aggregate amount deductible in respect of onshore areas.
(a) 25
(b) 10
(c) 50
(d) 75
Q.6. After commencement of commercial production deduction in respect of expenditures in the tax year is restricted upto
___% of aggregate amount deductible in respect of offshore areas.
(a) 25
(b) 10
(c) 50
(d) 75
Q.7. The depletion allowance shall be allowed up to the 15% of gross receipts, not exceeding ____% of profits or gains
before deduction of such allowance, for any year ending after the date on which commercial production has
commenced
(a) 25
(b) 10
(c) 50
(d) 75
Q.8. Agreement means an agreement entered into between the ____ and a taxpayer for the exploration and production
of petroleum in Pakistan.

478 Conceptual Approach to Taxes


Oil, Natural Gas and Other Mineral Deposits Chapter-26

(a) Provincial Government


(b) Federal Government
(c) Board
(d) None of above
Q.9. Commercial Production means production as determined by ________.
(a) Provincial Government
(b) Federal Government
(c) Board
(d) None of above
Q.10. Petroleum means _____ as defined in the Pakistan Petroleum (Production) Rules 1949
(a) crude oil
(b) natural gas
(c) case-head petroleum spirits
(d) all of above
Q.11. ______ means the termination of rights with respect to an art including the expiration of rights according to the terms
of an agreement.
(a) surrender
(b) termination
(c) breach of contract
(d) all of above
Q.12. Surrendered area means an area with respect to which the rights of the person have terminated by _________.
(a) surrender
(b) assignment
(c) termination of business
(d) all of above
Q.13. The profits and gains from exploration and production of mineral deposits shall be computed in the manner applicable
to income, profits and gains chargeable under the head ____
(a) Income from other sources
(b) Property income
(c) Business income
(d) Capital gains
Q.14. Depletion allowance shall be allowed up to the ________%.
(a) 10
(b) 20
(c) 30
(d) 40
Q.15. An amount equal to the ______ shall be set apart and left as a reserve to be utilized for the development and
expansion of such undertaking.
(a) depletion allowance
(b) total expenses incurred
(c) total income earned
(d) all of the above
Q.16. The aggregate of taxes on income paid on or after 1.7.2001 shall not be less than ___% of the profits or gains derived
by an onshore petroleum exploration and production undertaking

Conceptual Approach to Taxes 479


Oil, Natural Gas and Other Mineral Deposits Chapter-26

(a) 40
(b) 50
(c) 60
(d) 70
Q.17. The aggregate of taxes on income paid on or after 1.7.2001 shall not be less than ___% of the profits or gains derived
by an offshore petroleum exploration and production undertaking
(a) 40
(b) 50
(c) 60
(d) 70
Q.18. In respect of any tax year commencing on or after 1.7.2002 the aggregate of taxes on income paid on or after
1.7.2001 shall not be less than ___% of the profits or gains derived by an onshore petroleum exploration and
production undertaking
(a) 40
(b) 50
(c) 60
(d) 70
Q.19. With effect from the Tax Year 2010. "Decommissioning Cost" as certified by a Chartered Accountant or a Cost
Accountant shall be allowed over a period of ___ years or the life of the development and production or mining lease
whichever is less.
(a) 12
(b) 11
(c) 10
(d) 15
Q.20. Where an undertaking along-with Part I of 5th Schedule is also engaged in the business of refining or concentrating in
Pakistan the mineral deposits extracted by it in Pakistan, so much of the profits and gains derived from such business
as does not exceed 10% of the capital employed in such business shall be ____________.
(a) taxable at 10%
(b) taxable at 20%
(c) fully taxable
(d) exempt from tax
Q.21. Where the profits and gains of refining and concentrating business computed for any tax year cover a period which is
less or more than one year, the amount of profits and gains exempt shall be the amount which bears the same
proportion to the said amount of profits as the said period bears to a period of ____.
(a) one year
(b) two years
(c) last three years
(d) six months

ANSWERS
1 (a) 2 (a) 3 (d) 4 (a) 5 (b)
6 (a) 7 (c) 8 (b) 9 (b) 10 (d)
11 (a) 12 (d) 13 (c) 14 (b) 15 (a)
16 (b) 17 (a) 18 (a) 19 (c) 20 (d)
21 (a)

480 Conceptual Approach to Taxes


Banking Business Chapter-27

Chapter

27 BANKING BUSINESS

Topic covered
Section
(For CA Mod F & ICMAP students)
100A Special provisions relating to banking business
th
7 Schedule Rules for the computation of the profits & gains of banking company & tax payable thereon
Practical example
MCQs with solutions

Special provisions relating to banking business [section 100A]

1) Rules in 7th schedule shall apply in calculating the income, profit and gain of a banking company
2) Sub-section (1) shall apply form tax year 2009 and onward.

The seventh schedule


rules for the computation of the profits and gains of banking company and tax payable thereon
(1) The following adjustments shall be made, where applicable, in the total net income, profit and gains before tax from
all sources as per annual accounts reported to the State Bank of Pakistan.
(a) Accounting depreciation, initial allowance and amortization shall be added back to the income while tax
depreciation, initial allowance and amortization shall be allowed u/s 22, 23 and 24. In case of asset given on
finance lease, any deduction or allowance shall be inadmissible.
(b) Provisions of deductions not admissible u/s 21, depreciation allow-ability on disposal of depreciable asset u/s
22(8) and computation of cost and disposal consideration u/s 75 to 79 shall apply accordingly.
(c) If the actual provision of advances and off balance sheet items is greater than the 1% (5% of for consumers
and SMEs) of total advances then the provision shall be restricted to 1% or 5% as the case may be, of total
advances otherwise actual provision shall be allowed on having certificate from the external auditor that the
provision is in line with the prudential regulations. However the excessive provision over 1% or 5% as the case
may be, shall be carried over in subsequent years.
If provisioning is less than 1% of advances, for a banking company then actual provisioning for the year shall be
allowed. Further that if provisioning is less than 5% of advances for consumers and small and medium enterprises
(SMEs) then actual provisioning for the year shall be allowed and this provisioning shall be allowable from the first
day of July, 2010.
(a) Bad debts classified as sub-standard shall be inadmissible expense.
(b) Provision of sub-rule (d) shall apply as it is whether the taxpayer reclassifies any addition made under sub-rule
(d) as doubtful or loss.
(c) If a taxpayer reclassify any addition made under sub-rule (d) in subsequent year as recoverable then a
deduction shall be allowed in computing the taxable income.
(d) The adjustments made on account of Financial Instruments (IAS 39) and Investment Property (IAS 40) shall
be excluded in computing the taxable income.
(f) An adjustment shall be made for exclusions from income on account of paragraph (g) for determining the cost
of related item in the financial statement in the year of disposal of such item or asset or the discharge of
liability, as the case may be.
(2) 1. Where the deduction allowed for any expenditure other than bad debts or any liability, for which the deduction
was allowed, remains unpaid for 3 years from the tax year in which the deduction was allowed, shall be added
in taxable income in the 1st tax year following the end of 3 years.
2. A deduction shall be admissible only when the actual payment for the unpaid liability is made.

Conceptual Approach to Taxes 481


Banking Business Chapter-27

3. When the shares of listed company are disposed of within 1 year from the date of acquisition and results in
loss then the loss can be adjusted against the Business Income. Un-adjusted loss shall be carried forward to
the following tax year and set off against capital gain. Loss shall be carried forward for maximum of 6 years.
(3) Treatment for Shariah compliant banking
1. Any special provisions or rules by State Bank of Pakistan for Shariah Compliant Banking shall not be taken
into account for income tax purposes.
2. A statement, certified by the auditors of the Bank, shall be attached to the return of income to disclose the
comparative position of transaction as per Islamic mode of financing and as per normal accounting principles.
Adjustment to the income of the company on this account shall be made according to the accounting income
for purpose of this schedule.
(4) Head office expenditure
1. In case of foreign banks head office expenditure shall be allowed as deduction as per the following formula,
Gross receipt of permanent establishment in Pakistan / World gross receipts x Total Head Office expenditure.
2. The head office expenditure u/s 105(3) means any expenditure executive or general administration
expenditure incurred by non-resident person outside Pakistan for the permanent establishment of the person,
including
(a) Any rent, local rates and taxes excluding any foreign income tax, current repairs, or insurance against
risks of damage or destruction outside Pakistan;
(b) Any salary paid to an employee employed by the head office outside Pakistan;
(c) Any travelling expenditures of such employee; and
(d) Any other expenditure which may be prescribed.
Section 105(4) states that no deduction shall be allowed in computing the income of a permanent
establishment in Pakistan of a non-resident person chargeable under the head Income from Business for
(a) Any profit paid or payable by the non-resident person on debt to finance the operations of permanent
establishment; and
(b) Any insurance premium paid or payable by the non-resident person in respect of such debt.
3. The head office expenditure shall only be allowed if it is charged in the books of accounts of the permanent
establishment and a certificate from external auditors is provide to the effect that the claim of such
expenditure;
(i) has been made in accordance with the provision of this rule; and
(ii) is reasonable in relation to operation of the permanent establishment in Pakistan.
(5) Advance tax:
1. The banking company shall be required to pay advance tax for the year in 12 equal installments payable by
15th of every month.
2. Provisions of withholding tax shall not apply.
(6) Tax on income computed
income computed shall be chargeable to tax under the head Income from Business at the rate applicable in Division
II of Part I of the First Schedule.
(7A) The provisions of Minimum tax on the Income of certain person (section 113) shall apply to banking company.
(7B) From Tax year 2015 and onwards, Income from Dividend and income from Capital Gains shall be taxed at the rate
specified in Division II of Part II of the First Schedule.
(7C) The provisions of section 4B shall apply to the taxpayers under this schedule and taxed at the rates specified in
Division IIA of Part I of the First Schedule.
(8) Exemptions
(1) Exemptions and tax concessions under the 2nd schedule shall not apply to banking company.
(1A) The accumulated loss under the head "Income from Business" (not being speculation business losses) of an
amalgamating banking company or banking companies shall be set off or carried forward against the business
profits and gains of the amalgamated company and vice versa, up to a period of six tax years immediately
succeeding the tax year in which the loss was first computed in the case of amalgamated banking company or
amalgamating banking company or companies.

482 Conceptual Approach to Taxes


Banking Business Chapter-27

(2) If both the holding and subsidiary companies are banking companies then Group Relief (section 59B) shall
apply. Chartered Accountants firm on the panel of the State Bank of Pakistan shall approve the accounts of
group companies. Approval from State Bank of Pakistan is required for surrender and claim of loss.
(3) Group taxation (section 59AA) shall apply for the 100% owned group of banking companies with the approval
of State Bank of Pakistan.
(9) Transitional Provisions
1. The provisions of bad debts (sec. 29) and Provision regarding consumer loans (sec. 29A) shall apply.
Amounts provided before or for tax year 2008 as irrecoverable or doubtful advances, neither claimed not
allowed as tax deductible in a tax year, shall be allowed only when such advances are actually written off
against such provision.
2. Amounts provided before or for tax year 2008 as irrecoverable or doubtful advances, which were neither
claimed not allowed as tax deductible in a tax year but were written off in accounts, are written back in the tax
year 2009 and thereafter in any tax year in accounts, shall be excluded in computing the taxable income.
3. If a banking company give or acquire any asset at finance lease up to the tax year 2008 then the provisions of
this schedule shall not apply. The banking company shall recognize the income and deduction in respect of
asset leased on finance in accordance with the provisions of Ordinance as if this schedule has not come in to
force:
Provided that the un-absorbed depreciation in respect of such asset shall be allowed to be set-off against the said
lease rental income only.
(10) Provisions of Ordinance to apply: The provisions of the Ordinance not specifically dealt with in the aforesaid rules
shall apply as it is to the banking company.
(11) The federal government may, from time to time, by notification in official Gazette, amend the schedule so as to add,
modify or omit any entry therein.

Example: Finance manager of XYZ banking company has provided you the following information for the computation of
taxable income and tax liability of the company for the tax year 2016.
Rs. In 000
Profit before taxation as per P and L account 48,000
Accounting depreciation 9,000
Admissible depreciation by the taxation authority 10,500
Bad debts 750
Provision for bad debts 999
Total advances shown in balance sheet 66,600
Outstanding liability not paid over last 3 years ending on 2013 1,500
Amount paid for outstanding liability in 2015 which was become overdue
in 2012 and was added back to the taxable income in 2014 875
Un-adjusted loss on sale of shares of listed company is brought forward this year 150
Head Office expenditure charged to the branch (in accounts) 3,855
Total head office expenditure 12,850
Gross receipts of branch in Pakistan 320,000
Total world gross receipts 1,280,000
Total advance tax paid for the tax year 2016 16,500

Conceptual Approach to Taxes 483


Banking Business Chapter-27

Solution:
XYZ Banking Company
Computation of taxable income and tax thereon
for the tax year 2016
Income from business Rs. (in 000)
Profit before Taxation as per P and L Account 48,000
Add: Accounting depreciation 9,000
Bad debts (N-1) 750
Provision for bad debts (N-2) 999
Outstanding liability not paid over last 3 years ending on 2013 1,500
Head Office expenditure charged to the branch (in accounts) 3,855
Total 64,104
Less: Admissible depreciation by the taxation authority 10,500
Provision for bad debts (N-2) 666
Amount paid for outstanding liability 875
Admissible Head Office expenditure (N-3) 3,212.5
Total business income 48,850.5
Tax liability
35% tax on business and other income (48,850,500 x 35%) 17,097.675
Less: Advance tax 16,500
Total tax liability 597,675

NOTES:

N-1 Bad debts are not allowed as deduction


N-2 Provision for bad debts is allowed maximum to 1% of total advances, it shall be calculated as
Provision for bad debts = Total Advances x 1%
= Rs. 66,600 x 1%
= Rs. 666

N-3 Head Office shall be allowed as deduction according to the following formula
Head Office expenditure = (A / B) x C
= (Gross receipts of branch in Pakistan / World gross receipts) x Total Head Office expenditure
= Rs. (320,000 / 1,280,000) x 12,850
= Rs. 3,212.5

N-4 Loss on sale of shares of listed company shall be adjusted against business income in the 1st year the loss was
occurred. The un-adjusted loss on sale of shares of listed company brought forward from first year shall be adjusted
against Capital gain only.

484 Conceptual Approach to Taxes


Banking Business Chapter-27

MULTIPLE CHOICE QUESTIONS


Q.1. Rules of ___ schedule shall apply in calculating the income, profit and gain of a banking company.
(a) 3rd
(b) 4th
(c) 5th
(d) 7th
Q.2. Which of the following is admissible deduction while calculating income of a banking company?
(a) Tax depreciation
(b) Initial allowance
(c) Tax amortization
(d) all of above
Q.3. Provision of advances and off balance sheet items is restricted to ______% for consumers and SMEs.
(a) 2
(b) 4
(c) 5
(d) 1
Q.4. Provision of advances and off balance sheet items for consumers and SMEs in excess of 5% shall be ________.
(a) Ignored
(b) allowed as deduction
(c) carried forward
(d) none of above
Q.5. Bad debts classified as sub-standard shall be ________.
(a) Admissible
(b) inadmissible
(c) added into income
(d) none of above
Q.6. If a taxpayer reclassify any addition on account of sub-standard debts in subsequent year as recoverable then
__________.
(a) allowed as deduction
(b) it shall be added into income
(c) it shall be ignored
(d) none of above
Q.7. The adjustments made on account of Financial Instruments (IAS 39) and Investment Property (IAS 40) shall be
______ in computing the taxable income
(a) Included
(b) ignored
(c) excluded
(d) all of above

Conceptual Approach to Taxes 485


Banking Business Chapter-27

Q.8. Where the deduction allowed for any expenditure, remains unpaid for 3 years from the tax year in which the
deduction was allowed, shall be added in taxable income in the ___ tax year following the end of above said years
(a) 4th
(b) 3rd
(c) 2nd
(d) 1st
Q.9. When the shares of listed company are disposed of within ___ from the date of acquisition and results in loss then the
loss can be adjusted against the Business Income
(a) 1 year
(b) 2 years
(c) 3 years
(d) 4 years
Q.10. Un-adjusted loss on disposal of shares of listed company within 1 year from date of acquisition shall be carried
forward to the following tax year and set off against _______.
(a) Capital gains
(b) income from business
(c) income from other sources
(d) all of above
Q.11. Un-adjusted loss on disposal of shares of listed company within 1 year from date of acquisition shall be carried
forward for maximum of _______.
(a) 5 years
(b) 6 years
(c) 3 years
(d) 10 years
Q.12. In case of foreign banks head office expenditure shall be allowed as deduction as per the following formula:
Gross receipt of PE in Pakistan /_____ x Total Head Office expenditure.
(a) World gross payments
(b) World gross receipts
(c) World net payments
(d) Net receipts
Q.13. Banking company shall be required to pay advance tax for the year in ___ equal installments.
(a) 4
(b) 8
(c) 12
(d) 16
Q.14. Capital gain on the sale of shares of listed companies disposed of after one year shall be taxed at ___%.
(a) 5
(b) 6
(c) 35
(d) 10
Q.15. Provisions of section 113 shall _______ to a banking company.
(a) Apply
(b) not apply
(c) none of above

486 Conceptual Approach to Taxes


Banking Business Chapter-27
nd
Q.16. Exemptions and tax concessions under the 2 schedule shall ___ to banking company.
(a) Apply
(b) not apply
(c) none of above
Q.17. If both the holding and subsidiary companies are banking companies then Group Relief (section 59B) shall ___.
(a) Apply
(b) not apply
(c) none of above
Q.18. The ______ may, from time to time, by notification in official Gazette, amend the schedule so as to add, modify or
omit any entry therein
(a) State Bank of Pakistan
(b) FBR
(c) Federal Government
(d) all of above

ANSWERS

1 (d) 2 (d) 3 (c) 4 (c) 5 (b)


6 (a) 7 (c) 8 (d) 9 (a) 10 (a)
11 (b) 12 (b) 13 (c) 14 (c) 15 (a)
16 (b) 17 (a) 18 (c)

Conceptual Approach to Taxes 487


Banking Business Chapter-27

488 Conceptual Approach to Taxes


Chapter 25 Solved Past Papers Income Tax Numericals of ICMAP Stage IV - (2003 to 2015)

Chapter

28
SOLVED PAST PAPERS INCOME TAX NUMERICALS OF
ICMAP STAGE IV - (2003 TO 2015)
Note: All the following questions have been solved under the Income tax Ordinance, 2001 effective from July 01,
2015

Q.NO.4 March 2015 Mr. Hassan has been working as an Accounts Executive in Prime Limited which is a public limited
company. In addition to his salary, other perks and allowances are also provided to him by his employer. He has various
other sources of income as well. Assume you are income tax consultant and Mr. Hassan has submitted the following
information for the tax year ended on June 30, 2015 for calculation of is taxable income and tax liability:

Rupees
Basic salary per annum 500,000
Perquisites and allowances paid by the employer:
House rent allowance 110,000
Utilities 25,700
Entertainment allowance 13,000
Reimbursement of medical expenses 7,750

Two cars have been provided to Mr. Hassan and maintained by the company. One car is used wholly for the companys

business purposes having cost of Rs. 300,000 and the other one is used for his family exclusively costing to Rs. 350,000.

Income from other sources:


Mr. Hassan has been maintaining a profit and loss sharing account in ITC bank. The bank has credited a profit of Rs.
11,300 in his account during tax year.
He is also a non-professional writer and has received Rs. 20,000 on account of his literary work which was starter and
completed during the year.

Other information:
Mr. Hassan paid annual premium of Rs. 12,000 for life insurance policy and Rs. 4,000 for health insurance policy.

He also paid donation of Rs, 15,000 to an approved charity organization.


He also paid Zakat of Rs. 10,000 under the Zakat and Usher Ordinance, 1980.

Required:
Calculate the taxable income and tax liability of Mr. Hassan for the tax year 2015. Provide all necessary notes to support
your calculations.

Solution

Name of Tax Payer : Mr. Hassan

Conceptual Approach to Taxes _________________________________489


Chapter 25 Solved Past Papers Income Tax Numericals of ICMAP Stage IV - (2003 to 2015)

Income year ended : 30th June 2016


Tax year : 2016
Personal Status : Individual
Residential Status : Resident
Computation of taxable income and tax thereon
Rs. Rs.
Income from Salary
Basic Salary 500,000
House rent allowance 110,000
Utilities 25,700
Entertainment allowance 13,000
Reimbursement of medical expenses 7,750 -
Conveyance for personal use (350,000 x 10 % ) 35,000
683,700
Income from Other Sources
Non-professional writer fee 20,000
20,000
Total income 703,700
Less: Zakat (10,000)
Taxable Income 693,700

Computation of tax liability


Tax on Rs. 693,700
[ Rs.2,000 + (693,700 - 500,000) x 5% ] 11,685

Less: Rebate only on life insurance premium


( 11,685 / 693,700 x 12,000 ) 202
Less: Rebate on donation
( 11,685 / 693,700 x 15,000 ) 253 (455)
Tax payable under NTR 11,230
Add tax FTR on PLS account on Rs. 11,300 @ 10% 1,130
Tax payable with return 12,360

Notes - 1
Car provided by the employer which was wholly used for business purpose will not be added
in the income of Mr. Hassan as the same was used for the discharge of offical duties.

Q.NO. 4 August 2014 Prime Leather Works (Pvt.) Limited has provided following information for the tax year
2014 to calculate its taxable income and tax liability:

Description Note Amount (Rs. )


Sales 1 60,000,000
Less: Cost of sales 2 -40,000,000
Gross Profit 20,000,000
Less: General and admin expenses 3 -12,000,000
Less: Selling expenses 4 -3,000,000
Add: Other Income 5 1,000,000
Net Profit 6,000,000

1 Total sales include sales of Rs. 20 million which are subject to final taxation.

490_______________________________ Conceptual Approach to Taxes


Chapter 25 Solved Past Papers Income Tax Numericals of ICMAP Stage IV - (2003 to 2015)

2 Cost of sales includes accounting depreciation of Rs. 4 million.


3 The following expenses are also included in the general and administrative expenses:
a. Accounting depreciation of Rs. 2 million.
b. Bonus paid to the employees Rs. 3 million.
c. Provision for doubtful receivables Rs. 1.5 million.
d. Provision for the post employment benefits Rs. 2 million.
e. Fines and penalties paid to Federal Board of Revenue (FBR) Rs. 0.7 million.
4 Selling expenses include commission expense of Rs. 1.5 million. However, tax has not been withheld
while making the payment of the commission.
5 Other income includes the following items:
a. Accounting gain on disposal of fixed assets Rs. 0.2 million.
b. Interest income Rs. 0.3 million.
c. Dividend income from listed securities Rs. 0.5 million.
Additional Information:
6 Tax depreciation is Rs. 10 million.
7 Bad debts of Rs. 0.5 million.
8 Gratuity paid amounting Rs. 1.7 million.
9 Taxable gain on disposal of fixed asset is Rs. 0.15 million.
10 Breakup of advance tax is as follows:
a. On supplies u/s 153 Rs. 0.7 million.
b. Tax on utilities Rs. 0.3 million.
c. Tax on cash withdrawals Rs. 0.05 million.
d. Tax on dividend income Rs. 0.05 million.
e. Tax on interest income Rs. 0.03 million.
Required:
Calculate taxable income and net tax liability of Prime Leather Works (Pvt .) Limited for tax year 2014.

Solution

Prime Leather Works (Pvt.) Limited


Resident : Private Company
Tax Year: 2016
Computation of taxable income and tax thereon
Rupees Rupees
Income from Business
Net Profit before tax 6,000,000

Add: Inadmissible items


Accounting Depreciation in cost of sales 4,000,000
Accounting Depreciation in admin exp. 2,000,000
Provision for doubtful receivable 1,500,000
Provision for post employment benefits 2,000,000
Fines and penalties 700,000
Commission expense 1,500,000
Taxable gain on disposal 150,000 11,850,000
17,850,000
Less: Admissible items
Accounting gain on disposal 200,000
Tax depreciation 10,000,000

Conceptual Approach to Taxes _________________________________491


Chapter 25 Solved Past Papers Income Tax Numericals of ICMAP Stage IV - (2003 to 2015)

Bad Debts 500,000 (10,700,000)


7,150,000
Less: Dividend income cover under SBI (500,000)
6,650,000
Less: Apportionment of sales subject to final taxation
(6,650,000 x 20 / 60 ) (2,216,667)
Total Taxable Income 4,433,333

Computation of tax liability:

Tax on Rs. 4,433,333 @ 32 % A 1,418,667


Alternate Coporate tax u/s 113 C
Tax on Rs. 6,000,000 x 17% B 1,020,000
Minimum Tax u/s 113
Tax on Rs. 60,000,000 @ 1 % C 600,000
Tax liability higher of A , B or C 1,418,667
Tax on Rs. 20,000,000 @ 4% 800,000
2,218,667
Tax on dividend income Rs. 500,000 @ 10 % 50,000
2,268,667
Less: advance tax (800,000 + 300,000 + 50,000 + 50,000 + 30,000) (1,230,000)
Balance tax payable 1,038,667

Q. NO.4 Spring 2014

Mr. Samiullah has been working as Assistant Manager-Marketing for last 15 years in M/s. Moonlight Limited, a public limited
company. In addition to salary, perks and allowances given to him by the company, he has various other sources of income.
Assume that Mr. Samiullah is 62 years and has been retired from the company services on June 30, 2014. You are his Tax
Consultant. He has submitted the following information for the tax year ended June 30, 2014 in order to seek your advice in
respect of the calculation of his taxable income and tax liability.

Rupees

A. i) Basic salary per annum 500,000

ii) Perquisites and al allowances paid by the employer:

Bonus 80,000
Entertainment allowance 10,000
Dearness allowance 180,000
House rent allowance 225,000
Gratuity (scheme approved by FBR) 625,000
Encashment of leave preparatory to retirement 120,000

B. Property Income:

Mr. Samiullah rented his house @ Rs. 12,000 per month w.e.f 1st Jul y, 2013. He received a deposit of
Rs.150,000 not adjustable against rent, out of which he refunded Rs.75,000 to previous tenant, who vacated the
house after 3 years' occupancy. Tenant also paid property tax of Rs. 6,000 as per lease agreement. Assume that
Mr. Samiullah claimed the following expenditure for the year ended 30th June, 2014:-
Rupees Rupees

492_______________________________ Conceptual Approach to Taxes


Chapter 25 Solved Past Papers Income Tax Numericals of ICMAP Stage IV - (2003 to 2015)

Interest on borrowed capital 7,000


Insurance premium paid to cover the risk for property damage 10,000
Repair and maintenance expenses 25,000

C. Other information
Share from unregistered firm (AOP) 20,000

D. Zakat deducted at source 8,000

Required:

(a) Calculate the taxable income and tax liability of Mr. Samiullah for the year ended June 30, 2014.
(b) Mr. Samiullah is 62 years. Can he claim a tax rebate @ 50% available to senior citizens? Why or why not? State
the reasons in support of your answer.

Solution: (a)

Mr. Samiullah (Resident)


Computation of taxable income and tax liability
For the Tax year 2016

INCOME FROM SALARY U/S 12 (Rupees) (Rupees)

Basic salary 500,000


Bonus 80,000
Entertainment allowance 10,000
Dearness allowance 180,000
House rent allowance 225,000
Encashment of leave preparatory to retirement 120,000
Gratuity (scheme approved by FBR) 625,000
Exempt U/C 13(iii) of Part I of 2nd Schedule (200,000) 425,000
Total taxable salary (A) 1,540,000

INCOME FROM PROPERTY U/S 15

Rent received (Rs. 12,000 x 12 months) (Note 1) 144,000


Property tax paid by tanent 6,000
Advance not adjustable against rent [150,000 - (75,000 x 3/10)] / 10 = 12,750
162,750
Less admissible deductions U/S 15A

1/5th repair allowance (Rent chargeable to tax Rs. 162,750 x 1/5) = (32,550)
Interest on borrowed capital (7,000)
Insurance premium paid to cover the risk for property damage (10,000) 113,200
(B)

INCOME FROM OTHER SOURCES U/S 39

Share from unregistered firm (AOP) (Assumed under NTR and after tax) (C) 20,000
(Included for rate purposes)
Total income (A + B +C) 1,673,200

Less: Zakat (8,000)


Taxable income under NTR 1,665,200

As taxable salary is more than 50% of the total taxable income, hence the taxpayer is a salaried person and accordingly his
tax liability is computed as under.

Conceptual Approach to Taxes _________________________________493


Chapter 25 Solved Past Papers Income Tax Numericals of ICMAP Stage IV - (2003 to 2015)

COMPUTATION OF TAX LIABILITY:

Total income taxable under NTR (including share from AOP) 1,665,200

Tax on Rs. 1,665,200 [92,000 + 15% x (1,665,200 - 1,500,000)] 116,780


Less Tax on share from a unregistered firm (AOP) [Rs. 116,780 / 1,665,200 x 20,000] = 1,403
Balance tax on taxable income (excluding share from AOP) 115,377

Solution: (b)

Although Mr. Samiullah has met one condition of age of 60 years or more on the first day of the tax year however because
his taxable income exceeds Rs.1(M) therefore 50% reduction in tax liability under clause 1A of Part III of 2nd Schedule to the
Income Tax Ordinance, 2001 as senior citizen shall not be allowed.

Spring 2013 Q. 4

M/s. Golden Gate Limited (GGL) is a private limited company. The company manufactures and supplies consumer goods.
GGL sells its product through various distributors in Karachi, Lahore and Islamabad. The following is the profit and loss
account of GGL for the year ended on June 30, 2013:

Rs. '000' Rs. '000'


Sundry expenses 2,240 Gross Profit 235,200
Office salaries 29,120 Interest on bank deposit 300
Rent, rates & taxes 8,960 Recovered bad debts
Legal charges 2,016 (allowed in the past) 448
Finance charges on leased assets 350 Dividend 672
Advertisement 5,600
Auditor's fees 6,720
Cost of issue of debentures 5,600
Loss on sales of furniture 2,240
Provident fund contribution 7,840
Bad debts 4,480
Vehicle expenses 8,960
Fire insurance premium 7,840
Preliminary expenses 1,008
Provision for taxes 10,080
Provision for bad debts 4,480
Liquidated damages 3,360
Depreciation 44,800
Net Profit 80,926
236,620 236,620
Additional Information:

The following information is available:


(i) Sundry expenses include donation of Rs. 502,000 paid to an unrecognized charitable institution.
(ii) Office salaries include Rs.6,000,000 paid to one of the directors.
(iii) Provident Fund is recognized by the Income Tax Department.
(iv) Vehicle expenses are not vouched and verifiable to the extent of Rs.1,881,000.
(v) Actual depreciation works out to Rs.32,650,000 only.
(vi) Lease rental for the year are Rs.1,750,000.

Required:

494_______________________________ Conceptual Approach to Taxes


Chapter 25 Solved Past Papers Income Tax Numericals of ICMAP Stage IV - (2003 to 2015)

Calculate the taxable income and tax liability of the company for the tax year 2013 from the above data.

Solution

M/S GOLDEN GATE LIMITED (GGL)


COMPUTATION OF TAXABLE INCOME & TAX THEREON
FOR THE TAX YEAR 2016

INCOME FROM BUSINESS U/S 18 Note Rs. Rs.

Profit as per accounts 80,926,000

Add: Inadmissible items U/S 21 & 174

Provision for taxes 10,080,000


Provision for bad debts 4,480,000
Unvouched vehicle expenses 1,881,000
Donation to un-recognised institution 502,000
Accounting depreciation 44,800,000
Finance charges on lease asset 350,000 62,093,000
143,019,000
Less: Admissible items U/S 20
Lease rental 1,750,000
Tax depreciation 32,650,000
Dividend (To be taxed under FTR) 672,000 35,072,000
Taxble income under NTR 107,947,000

COMPUTATION OF TAX LIABILITY:

Alternative Corporate tax U/S 113C [Accounting profit under NTR x 17%] (A) 13,757,420
[Rs. 80,926,000 x 17%]

Tax liability under corporate tax under NTR @ 32% (B) 34,543,040

Actual tax payable under NTR: higher of (A) or (B) 34,543,040


Add: Tax liability under FTR on dividend income (Rs. 672,000 x 10%) 67,200
Total tax liabiltiy 34,610,240
Add: Tax on gross dividend receipts 67,200
Total tax liabiltiy 34,677,440
Less: Tax deducted on dividend 1 67,200
Balance tax payable 34,610,240

Assumptions and basis

1. It is assumed that tax on dividend income has duly been deducted and deposited, hence credit of the same has
been claimed against tax liability under final tax regime (FTR).

2. It is assumed that loss on sale of furniture and bad debts are in accordance with the provisions of sections 22
and 29 of the Income Tax Ordinance, 2001.

3. It is assumed that company is resident and although minimum tax under section 113 is applicable, however the
same has not been computed in the absence of turnover and accordingly no comparison with tax under normal
tax regime (NTR) has been made.

4. It is assumed that the preliminary expenses are in accordance with the provisions of section 25 of the Income
Tax Ordinance, 2001.

Conceptual Approach to Taxes _________________________________495


Chapter 25 Solved Past Papers Income Tax Numericals of ICMAP Stage IV - (2003 to 2015)

Q.5 August 2012

Global International Limited engaged in the manufacturing and trading of FMCG in the country . The shares of the
company are listed on all the stock exchanges of Pakistan. Following information has been extracted from the profit and
loss account of the company for the year ended 30th June, 2012.
Rupees

Sales 55,300,000
Cost of sales 22,120,000
Gross profit 33,180,000
Add:
Dividend received 300,000
Less
Director's sal aries 7,500,000
Staff salaries 12,150,000
Contribution to employees provident fund (a) 1,320,000
Administrat ive and selling expenses 2,550,000
Depreciation (b) 1,100,000
Entertainment expenses (c) 950,000
Insurance (d) 900,000
Fees (e) 650,000
Total expenses 27,120,000
Net income 6,360,000

Note:
(a) Employees' Provident Fund Trust is revocable at the option of Managing Director of the company and an application for
approval has been filed with the relevant tax authority.

(b) Depreciation includes Rs .300,000 for plant & machinery. Depreciation on all assets is charged on rates for normal
depreciation given in the Third Schedule to the Income Tax Ordinance, 2001. Written down value of plant & machinery for
the purpose of calculating tax depreciation is Rs.1,350,000 which includes addition during the year of new machinery of the
value Rs.650,000.

(c) Entertainment expenses include Rs.200,000 reimbursed to a director of the company for which no support is available.

(d) Insurance includes prepaid expenses amounting to Rs.320,000.

(e) The company has paid fees to the tax consultant for defending taxpayer's appeal in Income Tax Appellate Tribunal.

Required:
Compute the taxable income and the tax liability of the Global International Limited. Give proper comments where any given
information has not been utilized in the computation.

Solution Q.5 August 2012

Global International Limited


Computation of taxable income and tax thereon
For the tax year 2016
Rs. Rs.
INCOME FROM BUSINESS U/S 18

Profit before tax as per accounts 6,360,000

Add: Inadmissible items U/S 21

(a) Employee's provident fund (unapproved) 1,320,000

496_______________________________ Conceptual Approach to Taxes


Chapter 25 Solved Past Papers Income Tax Numericals of ICMAP Stage IV - (2003 to 2015)

(b) Accounting depreciation on plant and machinery 300,000


(c) Entertainment expenses without supporting evidence 200,000
(d) Prepaid insurance 320,000
2,140,000

Less: (b) Tax depreciation on plant and machinery (N-1) 340,625


Dividend (taxable under the head income from other sources) 300,000
640,625
7,859,375
INCOME FROM OTHER SOURCES U/S 39

Dividend income (covered under FTR) 300,000

Total taxable income 7,859,375

COMPUTATION OF TAX LIABILITY:

Alternative Corporate tax U/S 113C (A) 1,081,200


[[Accounting profit under NTR Rs. 6,360,000 x 17%]

Corporate Tax
Tax on Rs. 7,859,375 x 32% (B) 2,515,000

Minimum tax u/s 113 (55,300,000 x 1%) (C) 553,000

Tax liability under NTR: Higher of (A), (B) or (C) 2,515,000

Tax liability under FTR:


Tax on gross dividend @ 10% 30,000
2,545,000
Less: Tax deducted on gross dividend (N-2) (30,000)
Balance tax payable 2,515,000

(N-1) Tax depreciation on plant and machinery:


Normal Initial Total
W.D.V. depreciation allowance depreciation
15% 25%
Rs. Rs. Rs. Rs.
U/S 22 U/S 23
Opening WDV 1,350,000
Less addition during the year (650,000)
700,000 105,000 - 105,000

Addition during the year 650,000


Less 25% initial allowance 162,500
487,500
Less: Initial allowance (650,000 x 25%) - 162,500 162,500

Normal depreciation on addition during the year


(WDV Rs.487,500 x 15%) 73,125 - 73,125
178,125 162,500 340,625

N-2: In the abscence of information it has been assumed that tax on dividend has duly been deducted & deposited.

N-3: As the legal fee for defending taxpayer's appeal in Income tax Appellate Tribunal is admissble expense hence the
same has no effect on the taxable income.

Conceptual Approach to Taxes _________________________________497


Chapter 25 Solved Past Papers Income Tax Numericals of ICMAP Stage IV - (2003 to 2015)

February - 2013 Q. 4
Mr. Noor has been working as a senior Manager in Karachi Terminal Limited, Assume he has proivded following
information about his income pertainig to year ended June 30, 2013:

Salary Income Rupees

Basic salary 840,000


Bonus 100,000
Dearness allowance 84,000
House rent allowance 420,000
Medical allowance (actual expenses Rs.30,000) 80,000
Contribution by Mr, Noor towards recognized provident fund (including 168,000
equal conribution by the company)
Interest credited during the year (provident fund) 300,000
Other Income
Remuneration for literary work. (Mr. Noor is a non-professional writer) 120,000
Profit on profit and loss sharing bank account (net of 10 % tax withheld) 9,000
Capital loss on sale of shares of public listed company 35,000

Rent from house let out 192,000


Expenditure claimed against property income :
Property tax 15,000
Repair & maintenance 5% of rent

Other Information :
Zakat paid 10,000
Note: It is assumed that literary work was started and completed during the tax year.

Required:
Being a tax consultant you are required to calculate Mr. Noor's taxable income and his income tax liability
for the tax year 2013.

Solution

Name of Taxpayer Mr.Noor


Nation Tax Number xxx
Tax year 2016
Personal status Individual - salaried
Residential status Resident
Computation of income & tax thereon
Rs. Rs.
INCOME FROM SALARY U/S 12

Basic salary 840,000


Bonus 100,000
Dearness allowance 84,000
House rent allowance (N-1) 420,000
Medical allowance (N-2) -
Employer contribution towards recognized provident fund (N-3) -
Interest on provident fund (N-4) -

498_______________________________ Conceptual Approach to Taxes


Chapter 25 Solved Past Papers Income Tax Numericals of ICMAP Stage IV - (2003 to 2015)

1,444,000
CAPITAL (LOSS) U/S 37 (N-5) -

INCOME FROM OTHER SOURCES U/S 39

Remuneration for literary work 120,000

INCOME FROM PROPERTY U/S 15

Rental income 192,000

Less property tax (15,000)


1/5th repair allowance rent chargeable to tax (Rs. 192,000 x 1/5) (38,400) 138,600
Total Income 1,702,600
Less zakat paid (assumed under Zakat & Ushar Ordinance) (10,000)
Taxable Income 1,692,600

COMPUTATION OF TAX LIABILITY:

Tax on income under NTR:

Tax on Rs.1500,000 92,000


Tax @ 15% on [1,692,600 - 1,500,000] 28,890 120,890
Add tax on profit on PLS account (Rs.10,000 x 10%)= 1,000
Total tax liability 121,890

Less tax already paid / deducted at source


Tax deducted on profit on PLS account U/S 151 (1,000)
Balance tax payable 120,890

NOTES:

(N - 1) House rent allowance is totally taxable.


(N - 2) Medical Allowance: Rs.

Medical Allowance 80,000


Less exempt upto 10% of basic salary U/C 139 (840,000 x 10%) 84,000
-
(N - 3) Contribution towards recognized provident Fund:

Employer's contribution to provident fund (Rs. 168, 000/2) 84,000


(Equal contributiion by the employer company)
Less: Exempt upto lower of 10% of salary (924,000 x 10%) 92,400
OR Rs. 100,000. 100,000 92,400
-

Nothing will be included in total income of the tax payer as the contribution is within the exemption limit.
Salary for provident fund purposes mean basic salary plus dearness allowance (i.e. Rs. 840,000 + 84,000).
No treatment of employee contribution as the same is already included in taxable salary of the
employee.

Conceptual Approach to Taxes _________________________________499


Chapter 25 Solved Past Papers Income Tax Numericals of ICMAP Stage IV - (2003 to 2015)

(N - 4) Rs.

Amount of interest credited 300,000


Less exempt higher of 1/3rd of salary (Rs. 924, 000/3) or interest @ 16% p.a. 308,000
Taxable amount -

Interest credited to a provident fund is exempt upto the higher of one-third of salary or amount on
interest calculated @ 16% p. a. As the rate of interest is not given hence 1/3rd of salary has been taken
as exempt.

(N - 5) Capital gain on sale of shares of listed companies is taxable as a separate block of income. Loss on such
shares may be set- off by a person against gain on securities chargeable to tax. There is no treatment of
the said loss as there is no gain on sale of securities.

(N - 6) Profit on PLS bank account received are taxable under Final Tax Regime hence are not included in total
income. Tax deducted is treated as fina discharge of tax liability for such income.

APRIL - 2012 Q. 5
Mr. Abdul Rehman is Chief Accountant in a multinational company. He is assumed to have received the following
perks during the tax year 2012:
Income from Salary: Rs.

Basic salary 412,500


House rent allowance 182,800
Utilities 40,600
Employer's contribution to recognized provident fund 31,950
Medical allowance 37,300
Leave encashment 50,600
T.A / D.A for official duty 33,300
Bonus 82,500
During the year, tax has been deducted on salary at Rs.26,500. Further he has paid Zakat at Rs.35,000 under
the Zakat and Ushar Ordinance and Rs.56,000 as donation to an approved institution.

Mr. Abdul Rehman has been provided with a 1300cc car which cost Rs.1,250,000 to the company. The car is
used for both personal and official purposes.
Capital Gain: Rs.
Capital gain on buying and selling of shares of an unlisted company
(shares were held for eight months). 52,000
Income from Property:

Four years ago he rented out his house for a monthly rent of Rs.24,000. At that time he also received Rs.120,000
from the tenant as deposit which is not adjustable against monthly rent. During the current tax year he received
Rs.288,000 as rent.
Required:
Compute taxable income and tax liability of Mr. Abdul Rehman for the tax year 2012.

Solution:

Mr. Abdul Rehman (Resident)


Computation of taxable income and tax liability
For the Tax year 2016

500_______________________________ Conceptual Approach to Taxes


Chapter 25 Solved Past Papers Income Tax Numericals of ICMAP Stage IV - (2003 to 2015)

INCOME FROM SALARY U/S 12 (Rupees) (Rupees)

Basic salary 412,500


House rent allowance 182,800
Utilities 40,600
Employer's contribution to recognized provident fund 31,950
Exempt upto lower of:
- Rs. 100,000 OR
- 10% of Basic salary i.e 41,250 41,250 -
Medical allowance 37,300
Exempt upto lower of 10% of basic salary U/C 139 41,250 -
Leave encashment 50,600
T.A / D.A for official duty (Exempt) -
Conveyance (1,250,000 x 5%) U/R 5 62,500
Bonus 82,500
Total taxable salary (A) 831,500

CAPITAL GAIN U/S 37

Capital gain on buying and selling of shares of an unlisted


company (shares were held for eight months). (B) 52,000

INCOME FROM PROPERTY U/S 15

Rent received (Note 1) 288,000


Advance not adjustable against rent (120,000 / 10) 12,000
Rent chargeable to tax 300,000
Less 1/5th repair allowance of rent chargeable to tax (Rs. 300,000 x 1/5) (60,000)
Net rent chargeable to tax (C) 240,000
(A + B + C) 1,123,500
Less: Zakat 35,000
Taxable income under NTR 1,088,500
As taxable salary is more than 50% of the total taxable income, hence the taxpayer is salaried person

COMPUTATION OF TAX LIABILITY:

Total income taxable under NTR 1,088,500

Tax on Rs. 1,088,500 [14,500 + 10% x (1,088,500 - 750,000)] 48,350


Less: Tax credit on donation U/S 61:
Tax credit shall be allowed on lower of the following:
- Actual amount of donation i.e. 56,000
- 30% of taxable income i.e. 1,088,500 x 30% = 326,550
Tax credit = 56,000 x 48,350 / 1,088,500 2,487
Total tax liability 45,863
Less: Tax deducted at source 26,500
Balance tax payable 19,363

Note 1 In the absence of information only 1/5th repair allowance has been claimed against income from property.

Summer - 2011 Q. 2 (c) (i)

From the following compute the amount of "Income from Property" chargeable to tax for the year ending June 30, 2010
Rupees
Rent of the property 50,000 (per month)

Conceptual Approach to Taxes _________________________________501


Chapter 25 Solved Past Papers Income Tax Numericals of ICMAP Stage IV - (2003 to 2015)

Forfeited amount under a contract for the sale 100,000


Refundable security deposit 500,000
Annual maintenance charges paid by the owner of the property 25,000

Solution:

Name of taxpayer
Computation of taxable income and tax liability
For the Tax year 2016
Rs.
INCOME FROM PROPERTY U/S 15

Rent of property (Rs. 50,000 x 12 months) 600,000


Add: Forfeited amount under a contract for the sale 100,000
Rent chargeable to tax 700,000

Less 1/5th repair allowance of rent chargeable to tax before any deduction (140,000)
(Ignore maintenance expenses that may be less or more than 1/5th allowance)
Rent chargeable to tax for taxability purposes 560,000

(N - 1) No treatment of refundable security has been made as the same shall autocatically transferred to rental income
on expiry against which advance rent has been given.

Summer - 2011 Q. 5
Mr. Hussain Ahmad is an officer in a public listed company. He derived the following salary income during the tax
year ended June 30, 2010:
Rs.
Basic salary 50,000 (per month)
House rent allowance 22,500 (per month)
Utility allowance 5,000 (per month)
Medical allowance 6,000 (per month)
Leave encashment 50,000
(i) Company also provide him with a 800CC car valuing Rs. 600,000 as per books of accounts. The car is used for
official and personal purposes.
(ii) He has paid Zakat of Rs. 25,000 to an approved institution.
(iii) He paid a donation of Rs. 20,000 to a charitable institution for which a tax credit is allowed u/s 61.

(iv) He paid donation of Rs.10,000 to an approved institution specified u/c (61) of part 1 of second Schedule.

(v) He received a pension amounting to Rs. 7,000 p.m from his past employment in the government.

Required: Compute the amount of taxable income and tax liability of Mr. Hussain for the tax year 2010.
Solution:
Mr. Hussain Ahmad (Resident)
Computation of taxable income and tax liability
For the Tax year 2016
Rs. Rs.
INCOME FROM SALARY U/S 12

Basic salary (Rs. 50,000 x 12 months) (Note - 1) 600,000


House rent allowance (Rs. 22,500 x 12 months) (Note - 1) 270,000
Utility allowance (Rs. 5,000 x 12 months) (Note - 1) 60,000
Medical allowance (Rs. 6,000 x 12 months) 72,000
Less: Exempt upto 10% of basic salary U/C 139 60,000 12,000
Leave encashment (Note - 1) 50,000

502_______________________________ Conceptual Approach to Taxes


Chapter 25 Solved Past Papers Income Tax Numericals of ICMAP Stage IV - (2003 to 2015)

Conveyance (Rs. 600,000 x 5% of cost of asset to employer) U/R 5 30,000


Pension as Govt. employee (exempt u/c 9 of Part I of second schedule) -
Total income 1,022,000
Less: Zakat paid (maximum upto total income) (25,000)
Donation to institution specified in u/c 61 of Part 1 of 2nd Schedule:
Lower of following two amounts shall be deducted from income (as exempt income)
- 30% of taxable income (30 / 130 on Rs. 997,000) however here is 997,000
less 10,000 donation allowable = 987,000 x 30% = 296,100 or
- Actual amount of donation i.e. Rs. 10,000 (10,000)
Taxable income 987,000

COMPUTATION OF TAX LIABILITY:

As the person is having only salary income hence tax liability for a salaried person is computed as under:

Tax on Rs. 987,000 [14,500 + 10% x (987,000 - 750,000)] 38,200

Less: Tax credit on donation u/s 61


Tax credit shall be allowed on lower of following
- 30% of taxable income i.e Rs. 296,100
- Actual amount of donation i.e. Rs. 20,000
Tax credit = (20,000 x 38,200 / 987,000) (774)
Balance tax payable 37,426

(Note 1) Basic salary, utility allowance, house rent allowance and leave encashment are totally taxable.

Summer - 2010 Q. 5
The Trading & Profit and Loss Account of M/s. Arshad Limited for the year ended on 30th June, 2009 is as under;

Rs. "000" Rs. "000"


Sales 2,300,000
Less: Cost of sales 1,571,000
Gross profit 729,000
Less: expenses
Salaries and wages 170,000
Office rent 55,000
Telephone expenses 39,500
Travelling and conveyance 41,500
Forwarding 61,000
Entertainment 2,500
Miscellaneous 9,000
Office stationery 9,000
Depreciation 58,000
Income tax for last year 54,000
Bad debts 9,000
Doubtful debts 5,000
Donations 900
Liquidated damages 500
Insurance 6,000
Provision for taxation 79,000 (599,900)
Net Profit 129,100
Notes and additional information:
(i) A sum of Rs. 900,000 written off last year and allowed by the Income Tax Department, has been recovered and
credited to bad debt reserve.

Conceptual Approach to Taxes _________________________________503


Chapter 25 Solved Past Papers Income Tax Numericals of ICMAP Stage IV - (2003 to 2015)

(ii) Un-vouched and un-detailed expenses included in the entertainment amounted to Rs. 350,000.
(iii) Depreciation allowable as per the Income Tax Law is Rs. 66,000,000.
(iv) Salaries and wages include payment of Rs. 250,000 without deducting tax at source.
(v) Salary paid amounting to Rs. 240,000 in cash.
(vi) Donation Rs. 500,000 paid to an approved institution specified in Clause (61) of part I of second schedule.
(vii) Donation Rs. 400,000 paid to an approved institution but not specified u/c (61) of part I of second schedule.
(viii) Pre-paid insurance Rs. 250,000.
Required: Compute the taxable income and tax liability of the Company for the tax year 2009.
Solution:
M/s Arshad Limited
Computation of taxable income and tax liabiltiy:
For the Tax year 2016
Rs. Rs.
INCOME FROM BUSINESS U/S 18 (000) (000)
Profit as per accounts 129,100
Add: Inadmissible items

Accounting depreciation 58,000


Income tax for last year 54,000
Doubtful debts 5,000
Donation to approved institution (credit shall be allowed) 400
Donation to approved institution (under clause 61) 500
Provision for taxation 79,000
Bad debts recovered from allowed portion (assumed credited to liability 900
otherwises the same shall be ignored for taxable income comuptation as
already included in given accounting profit)
Unvouched and un-detailed expenses 350
Payment of salaries without tax deduction & cash payment (Rs.250,000 + 240,000) 490
Pre-paid insurance 250
198,890
Less: Admissible items 327,990

Tax depreciation (66,000)


Total income 261,990
Less: Donation to approved institution (under clause 61)
(Straight deduction shall be allowed up to lower of
actual amount of donation or 20% of taxable income)
- Actual amount of donation i.e. Rs. 500,000 (500)
- 20% of taxable income i.e. Rs. 43,665,000
Comparison shall be made in separately 20% x (Rs. 261,990,000 x 100 / 120)
Taxable income 261,490

COMPUTATION OF TAX LIABILITY:

Alternative Corporate tax U/S 113C (A) 21,947


[[Accounting profit under NTR Rs. 129,100 x 17%]

Tax on taxable income @ 32% (B) 83,677

Turnover tax u/s 113 @ 1% of Rs.2,300,000,000 (C) 23,000

Tax liability higher of (A), (B) or (C) 83,677

504_______________________________ Conceptual Approach to Taxes


Chapter 25 Solved Past Papers Income Tax Numericals of ICMAP Stage IV - (2003 to 2015)

Less: Tax credit on donation


Tax credit shall be allowed on lower of:

- 20% of taxable income i.e Rs. 52,298


- Rs.400
Hence tax credit (400 x 83,677 / 261,490) 128
Balance tax payable 83,549

Winter - 2009 Q. 4
Following is the Profit and Loss account of M/s Fast Track Company (Pvt.) Ltd for the year ended on 30-06-2009

Rs. Rs.
Sundry expenses 8,000 Gross Profit 840,000
Office salaries 104,000 Casual income 2,000
Rent, rates and taxes 32,000 Premium on issue of debentures 40,000
Income tax 10,400 Recovered bad debts
Legal charges 7,200 (allowed in the past) 1,600
Advertisement 20,000 Dividend 8,000
Auditor's fees 24,000
Cost of issue of debentures 20,000
Loss on sales of furniture 8,000
P. F. contribution 28,000
Bad debts 16,000
Vehicle expenses 32,000
Fire insurance premium 28,000
Communication 3,600
Provision for taxes 36,000
Provision for bad debts 16,000
Liquidated damages 12,000
Depreciation 160,000
Net Profit 326,400
891,600 891,600
Required:
Compute the net taxable income of the company for the tax year 2010 from the above data after keeping in view
the following notes:
(i) Sundry expenses include Rs.1,600 paid to an institution not recognized u/s 61.
(ii) Office salaries include Rs.20,000 paid to one of the directors.
(iii) Provident Fund is recognized by the Income Tax Department.
(iv) Vehicle expenses are not vouched and verifiable to the extent of Rs.6,000.
(v) Actual depreciation works out to Rs.136,000 only.
Solution:
M/s Fast Track Company (Pvt) Limited
Computation of taxable income
For the tax year 2016 Rs. Rs.

INCOME FROM BUSINESS U/S 18

Profit as per accounts 326,400

Add:
Income tax 10,400
Provision for taxes 36,000
Provision for bad debts 16,000
Unvouched vehicle expenses 6,000
Donation to unapproved institution 1,600
Accounting depreciation 160,000 230,000

Conceptual Approach to Taxes _________________________________505


Chapter 25 Solved Past Papers Income Tax Numericals of ICMAP Stage IV - (2003 to 2015)

Less:
Tax depreciation 136,000
Dividend (excluded to calculate income from business) 8,000 144,000
412,400
INCOME FROM OTHER SOURCES U/S 39

Dividend income (Not to include in taxable income as fully covered under final tax regime) -
Taxable income 412,400

Notes:

1. Cost on issue of debenture and premium on issue of debenture expenses alongwith casual income, preimum on
issue of debentures and recovery from already allowed bad debts have no impact on taxable income hence the
same has been ignored.

2. In the abscence of sales no turnover tax has been computed, however where the turnover tax is higher than the
tax as compared to the tax under normal tax regime and alternative corporate tax U/S 113C of the Ordinance
then the same is to be paid by the Company.

Summer - 2009 Q. 3(b)

Mr. Ali Hassan a professor and Irani citizen entered into an employment contract with a government university in Pakistan for
teaching and research work. The university is incorporated u/s 42 of the Companies Ordinance, 1984 as a non-profit
organization.

The employment contract was effective from November 01, 2007. However, Mr. Ali Hassan arrived in Pakistan on November
02, 2007. Since November 03, 2007 was Sunday therefore he could not join his office. On Monday November 04, 2007 he
became ill and had to be hospitalized for the next five days and joined office on November 09, 2007. Due to his continuous
illness he took sick leave and went back to Iran on November 10, 2007.
Mr. Ali Hassan came back to Pakistan on January 03, 2008 and remained in Pakistan for the purpose of his employment till
June 30, 2008.

Required:
(i) State the provisions applicable to .Resident Individual. under Rule 14 of the Income Tax Rules, 2002, to
determine the number of days an individual is present in Pakistan.

(ii) Determine the residential status of Mr. Ali Hassan with reasons in accordance with Rule 14 of the Income Tax
Rules, 2002.

Solution:

(i)
The following rules apply in computing the number of days an individual is present in Pakistan in a tax year:
(a) A part of a day that an individual is present in Pakistan (including the day of arrival in, and the day of departure
from, Pakistan) counts as a whole day of such presence;

(b) the following days in which an individual is wholly or partly present in Pakistan count as a whole day of such
presence:

(i) a public holiday;


(ii) a day of leave, including sick leave;
(iii) a day that the individual's activity in Pakistan is interrupted because of a strike, lock-out or
delay in receipt of supplies; or
(iv) a holiday spent by the individual in Pakistan before, during or after any activity in Pakistan;

(c) a day or part of a day where an individual is in Pakistan solely by reason of being in transit between two different
places outside Pakistan does not count as a day present in Pakistan.
(ii)
Days

506_______________________________ Conceptual Approach to Taxes


Chapter 25 Solved Past Papers Income Tax Numericals of ICMAP Stage IV - (2003 to 2015)

- Day of arrival in Pakistan on November 2, 1


- Public holiday (Sunday) on November 3, 1
- Sick leave from November 4, to November 9 6
- Day of departure from Pakistan on November 10, 1
- From January 3rd (day of arrival) to 30-06-2016 180
189

As his stay in Pakistan has met the condition of minimum 183 days, therefore he is resident individual under Rule 14 of the
Income Tax Rules, 2002.

Summer - 2009 Q. 4

Mr. Jamshaid is an executive in a group of companies. He derived following incomes during tax year 30-06-2008.

Particulars
(i) Salary Income (per month):
Rs.
Basic Salary 20,000
House rent allowance 8,000
Utility allowance 1,000
Medical allowance 1,000
Expenses on children books 400

He is also provided with a 1000cc car, which is partly used for company business. As per books of accounts, the cost of the
car is Rs.650,000. He has also been granted with a housing loan of Rs.500,000 on which no profit/ interest has been
charged.

In addition to above, he also received gratuity of Rs.70,000 from his previous employer during the year. The gratuity fund is
not approved by the Commissioner Income Tax or Federal Board of Revenue.

Tax deducted at source from salary amounted to Rs.15,000.

(ii) Property Income:


Rs.
Rent from a house let out per month 10,000
He incurred following expenses on this property during the year:
- Repairs 30,000
- Collection charges 7% of rent
- Ground rent 10,000
- Property tax 15,000
- Rent-sharing with housing finance company 3,000 per month
He received a deposit of Rs.200,000, not adjustable against rent, out of which he refunded Rs.100,000 to previous tenant
who vacated the house after 3 years tenancy.
Rs.
(iii) Other Income:

Profit on PLS bank account (net of 10% tax withheld) 9,000


Commission from insurance company and from sale of plots 18,000
(net of 10% taxes withheld)
Lecturing and examination services fees from professional institutes 18,800
(net of 6% tax withheld)
Birthday present cash bonds 50,000

(iv) Gain on sale of shares of KayToo Ltd., a public listed company 55,000

Required: As a tax consultant you are required to compute Mr. Jamshaids total income and his income tax liability for the tax
year 2008.

Conceptual Approach to Taxes _________________________________507


Chapter 25 Solved Past Papers Income Tax Numericals of ICMAP Stage IV - (2003 to 2015)

Solution
Mr. Jamshaid
Computation of taxable income and tax liability:
For the tax year 2016
Rs. Rs.
INCOME FROM SALARY U/S 12

Basic salary (Rs. 20,000 x 12 months) 240,000


House rent allowance (Rs. 8,000 x 12 months) 96,000
Utility allowance (Rs. 1,000 x 12 months) 12,000
Medical allowance (Rs. 1,000 x 12 months) 12,000
Less: Exempt upto 10% of basic salary U/C 139 of Part 1 of 2nd schedule 24,000 -
Expenses on children books (reimbursed by employer) 4,800
Conveyance (650,000 x 5%) U/R 5 of Income tax Rules, 2002 32,500
Interest free loan (not included in taxable income as loan does not
exceed Rs. 500,000) -
Gratuity received 70,000
Less: Exempt under clause 22 of Part I of 2nd Schedule lower of:
- Rs. 75,000
- 50% of amount receivable i.e. 35,000 35,000 35,000
420,300
INCOME FROM PROPERTY U/S 15

Rent from house (Rs. 10,000 x 12 months) 120,000


Add: unadjustable advance [(200,000 - (100,000 x 3) /10)] /10 17,000
Rent chargeable to tax 137,000
Less: allowable expenses U/S 15A
- Repairs (Actual restricted to 1/5th of rent chargeable to tax) (27,400)
- Collection & admin. Exp (Maximum 6% of rent chargeable to tax) (8,220)
- Ground rent (10,000)
- Property tax (15,000)
- Rent-sharing with housing finance company 3,000 per month
Less: Rent sharing with housing finance company (36,000) 40,380

INCOME FROM OTHER SOURCES U/S 39

Lecturing and examination services fee (18,000 / 90%) 20,000


Birthday present cash bonds (Not taxable as a sort of liability) -
Taxable income 480,680

COMPUTATION OF TAX LIABILITY:

As the taxable salary income of the indiviual constitute more than 50% of the total taxable income hence tax under salary
slab is computed as under.

Tax liability under NTR


Tax on Rs. 480,680 [0 + 2% x (480,680 - 400,000)] (A) 1,614
Less tax under NTR on services income (assumed same amount
as profit on services) Rs.20,000 x 1,614 / 480,680 (B) 67
Net tax under NTR (A - B) (C) 1,546
Minimum tax on services (Rs. 20,000 x 10%) = (D) 2,000 2,000
Tax liability (C) plus higher of (B) and (D) 3,546
Tax liability under FTR / SBI
Tax on profit on PLS account (9,000 x 10% / 90%) 1,000
Tax on commission (18,000 x 10% / 90%) 2,000

508_______________________________ Conceptual Approach to Taxes


Chapter 25 Solved Past Papers Income Tax Numericals of ICMAP Stage IV - (2003 to 2015)

Tax on gain on shares of public listed company


(assumed that holding period is more than 4 years) -
Total tax liability 6,546
Less: Tax deducted at source
On salary 15,000
Profit on PLS account 1,000
On commission income 2,000
On sevices 1,200
19,200
Balance tax refundable (12,654)

Winter - 2008 Q. NO. 3 (b)


Four partners firm comprising Mr. JS, KK, TT, RR are sharing profit and losses equally. The unadjusted loss of AOP stands
at Rs. 444,000. Mr. RR submits resignation and retires from business.
Required: 1- State the Set off and Carry forward of losses of AOPs.
2- Compute the amount of loss to be carried forward by the firm.
Solution
1- State the Set off and Carry forward of losses of AOPs.
An AOP, being taxable independent of its members, is entitled to set-off and carry forward its losses as other persons are
entitled. As the share received by a member out of the incomes of an AOP is exempt from tax, the member is not allowed to
set-off and carry forward his respective share in losses of the AOP. Only AOP can set-off and carry forward its losses in
accordance with the rules specified in sections 56 to 59 of the Income Tax Ordinance, 2001
2- Compute the amount of loss to be carried forward by the firm.
Rs.
Total unadjusted loss 440,000
Less: share of Mr. RR ( 440,000 / 4) 110,000
Loss to be carried forward 330,000
Winter - 2008 Q. 4
Mr. Shahbaz is an officer in a Pakistani Airline. He provided the following particulars of his sources of income pertaining to
the tax year 2009:
Rs.
Salary 210,000
Bonus 10,000
Reward on passing an examination required by the terms of his employment 35,000
House allowance (per month) 10,000
Conveyance allowance (per month) 1,500
Medical allowance (Actual expenses Rs. 17,000) 20,000
Entertainment allowance 12,000
Flying allowance 60,000
Property income (including Rs. 2,000 per month for rent of furniture and fittings) 120,000
Expenditures claimed:
Legal expenses 7,500
Property tax 5,000
Insurance premium 3,000
Water charges paid (current Rs.10,000 + arrears Rs. 2,000) 12,000
Dividend (Zakat deduction Rs. 250 and tax deduction Rs. 1,000) 10,000
Leave encashment 25,000
Birthday present 10,000
Insurance money received on maturity of policy 100,000
Capital gain on sale of shares of a private ltd., co.
(Shares were retained for 18 months) 25,000
Expenses on children education (receipts are available) 40,000
Professional books purchased (receipts are available) 5,000

Conceptual Approach to Taxes _________________________________509


Chapter 25 Solved Past Papers Income Tax Numericals of ICMAP Stage IV - (2003 to 2015)

Zakat paid 10,000


Tax deducted at source 10,000
Required:
Compute total income and tax liability of Mr. Shahbaz for the tax year 2009.

Solution
Mr. Shahbaz
Computation of taxable income tax liability
For the tax year 2016
Rs. Rs.
INCOME FROM SALARY U/S 12
Salary 210,000
Bonus 10,000
Reward on passing an examination required by the terms of his employment 35,000
House allowance (totally taxable) (Rs. 10,000 x 12 months) 120,000
Conveyance allowance (assumed not for discharge of official performance) (Rs. 1,500 x 12) 18,000
Medical allowance (Actual expenses shall be ignored) 20,000
Less: Exempt upto 10% of basic salary U/C 139 21,000 -
Entertainment allowance (assumed not for discharge of official performance) 12,000
Leave encashment 25,000
Flying allowance (Note 2) 60,000 -
430,000
INCOME FROM PROPERTY U/S 15
Income from property (including furniture & fixtures) 120,000
Less: rent of furniture and fittings (Rs. 2,000 x 12 months) 24,000
Rent chargeable to tax 96,000

Less: allowable expenses


1/5th repair allowance of rent chargeable to tax 19,200
Legal expenses 7,500
Property tax 5,000
Insurance premium 3,000
Water charges paid (current Rs.10,000 + arrears Rs. 2,000) 12,000
46,700 49,300
CAPITAL GAINS U/S 37
Gain on sale of shares of private company (25,000 x 75%) 18,750
(Holding period is more than 1 year hence 25% is exempt)
INCOME FROM OTHER SOURCES U/S 39
Rent of furniture and fittings (accrual basis 2,000 x 12) 24,000
Total income 522,050
Less: Zakat (including zakat paid on shares) 10,250
Taxable income including SBI income 511,800
COMPUTATION OF TAX LIABILITY;
As the salary income of the individual constitute more than 50% of the total income hence tax under salary slab
is computed as under.

Tax liability under NTR


Tax on Rs. 511,800 [2% x (511,800 - 400,000)] 2,236
Flying allowance (Rs. 60,000 x 2.5%) 1,500
(Taxable as separate Block of income @ 2.5% upto one annual Basic Pay)
Tax on dividend (SBI under FTR) 1,000
Total tax liability 3,236
Less: Tax deducted at source

510_______________________________ Conceptual Approach to Taxes


Chapter 25 Solved Past Papers Income Tax Numericals of ICMAP Stage IV - (2003 to 2015)

Tax on dividend 1,000


Other tax deducted at source 10,000
11,000
Balance tax refundable (7,764)

(Note 1) Birthday present, insurance policy on maturity, expenses on childern education and professional books
purchased have no impact on computation of taxable income and tax thereon.

(Note 2) Flying allowance received as Officers other than as Pilot shall not be included in taxable income as the same is
taxable as a SBI @ 2.5% upto basic pay u/c (1) of Part III of 2nd Schedule to the Income Tax Ordinance, 2001. If
the flying allowance is more than the basic pay then the balance shall be included in the taxable income under
NTR of the individual.

If the flying allowance and other allowances shall be received as Pilot of any Pakistani Airline the same shall be
included in the taxable income under NTR upto basic pay. If the flying and other allowances are more than the
basic pay then the balance shall be taxable as a SBI @ 7.5% u/c (1AA) of Part III of 2nd Schedule to the ITO,
2001.

Summer - 2008 Q. 2(b)


Mr. Naseer is an employee of M/s. ABC (Pvt) Limited, the terms of employment provide that the tax will be paid by the
company. The company paid a total of Rs. 1,535,000/= to Mr. Naseer and the Income Tax Department. The rate of tax is
14%.

Required: Calculate the amount of tax paid by ABC (Pvt) Limited u/s 149 of the Income Tax Ordinance, 2001 on account of
salary on account of salary and the amount of salary paid to Mr. Naseer.
Solution
Salary ?
Tax @ 14% ? x 14%
Total amount paid 1,535,000
? + (? x 14%) = 1,535,000
1.14? = 1,535,000
? = 1,535,000 / 1.14
? = 1,346,491
Salary paid (as calculated above) = 1,346,491
Tax paid (1,535,000-1,346,491) = 188,509
Summer - 2008 Q. 4
Company Zaighm Chemicals (Pvt) Ltd . was incorporated on 1st January, 2006 and started its production and services
activities from 15th January 2006. Company has total 150 employees. Its paid-up capital and reserves as on 30-06- 2006
were as under:-
Rs.
Paid-up capital 21,000,000
Losses carried forward (taxable loss) (155,000)
General reserves -
20,845,000
During the financial year ended 30-06-2007, its books of accounts show the following balances:
i) Raw Material
a Imported value (before custom duty and taxes) 55,000,000
a Local purchases 4,500,000
ii) Other manufacturing / trading expenses 6,500,000
iii) Selling and admin expenses 12,600,000
78,600,000
Additional information:
1) To sell its products, the company's value addition is 30% of the cost of goods sold.

Conceptual Approach to Taxes _________________________________511


Chapter 25 Solved Past Papers Income Tax Numericals of ICMAP Stage IV - (2003 to 2015)

2) Both the raw material and finished goods are subject to following levies:
i) Sales Tax @15 %.
ii) Additional Sales Tax @ 2 % on imports.
iii) Special Federal Excise Duty @ 1 % on imports and goods sold.
iv) Customs duty on import value @ 25%.
v) Income Tax @ 6% on import value plus customs duty and Sales Tax.
3) Manufacturing expense includes Rs. 5,000,000 on account of depreciation charged on plant and machinery and Rs.
80,000 charged on computers in selling and administration expenses. The rate of depreciation charged by company on both
types of assets is 25%.

4) A laptop purchased for Rs. 80,000 for use by the Chief Executive was charged to profit and loss expenses.

5) Utilites bills amounting to Rs.760,000 charged to profit and loss expense include Rs. 86,000 of withholding income tax and
Rs. 134,000 sales tax.

6) No opening and closing stocks.

7) Applicable tax depreciation: @15% on machinery and @ 30% on computer and laptop respectively (ignore initial tax
depreciation)
Required:
a) Under what category this company falls for Income Tax purposes?
b) Workout company's sales.
c) Workout its net profit chargeable to income tax for the tax year 2007 (ignore initial depreciation).
d) What are company's tax and duties liabilities under the:
i) Customs Act, 1969. ii) Sales Tax Act, 1990
iii) Federal Excise Act, 2005 iv) Income Tax Ordinance, 2001
Solution
a) Under what category this company falls for Income Tax purposes?
Being as manufacturer the income of the company is fully covered under Normal Tax Regime u/s 153 of the Income tax
Ordinance, 2001 and being as small company the rate of tax shall be 25%.

b) Workout company's sales


Sales = Cost of goods sold + (Cost of goods sold x 30%)
Sales = 79,750,000 + (79,750,000 x 30%)
Sales = 79,750,000 + 23,925,000
Sales = 103,675,000
Cost of goods sold
Imported value (before custom duty and taxes) 55,000,000
Custom duty @ 25% 13,750,000
Local purchases 4,500,000
Other manufacturing / trading expenses 6,500,000
79,750,000
c) Workout profit chargeable to income tax
Rs. Rs.
Sales 103,675,000
Cost of sales 79,750,000
Gross profit 23,925,000
Selling and admin expenses 12,600,000
Profit before tax as per accounts 11,325,000

Add: Accounting depreciation ( 5,000,000 + 80,000 ) 5,080,000


Purchase of Laptop charged to profit and loss account 80,000
Income tax and sales tax paid with electricity bills (86,000 + 134,000) 220,000
5,380,000
Less: Tax depreciation:
On plant and machinery [ (5,000,000 / 25%) x 15%] 3,000,000

512_______________________________ Conceptual Approach to Taxes


Chapter 25 Solved Past Papers Income Tax Numericals of ICMAP Stage IV - (2003 to 2015)

On computers [ (80,000 / 25%) x 30%] (Assumed for business use) 96,000


On laptop (assumed used) (Rs. 80,000 x 30%) 24,000
3,120,000
13,585,000
Less: Losses b/f 155,000
Taxable income 13,430,000

d) (iv)
COMPUTATION OF TAX LIABILITY:

Alternative Corporate tax U/S 113C (A) 1,925,250


[[Accounting profit under NTR Rs. 11,325,000 x 17%]

Tax on Rs. 13,430,000 @ 25% (B) 3,357,500

Minimum tax u/s 113


Tax on Rs. 103,675,000 x 1% (C) 1,036,750

Tax liability: Higher of (A), (B) or (C) 3,357,500

Less:
Tax required to be on imports
[ { (55,000,000 + 13,750,000) x 1.17 } x 5% ] 4,021,875
Advance tax paid with electricity bill 86,000
4,107,875
Balance tax refundable (750,375)
d) (i)
Duty payable under Custom Act, 1969.
Imported value (before custom duty and taxes) 55,000,000
Custom duty @ 25% 13,750,000
d) (ii)
Tax payable under Sales Tax Act, 1990.
3% additional sales tax is not applicable on imports to be used for own use.
Output tax:
Sales tax on sales (103,675,000 x 17%) 17,624,750
Input tax:
Sales tax on purchases [ (55,000,000 + 13,750,000) x 17%] 11,687,500
Sales tax payable 5,937,250
d) (iii)
Tax payable under Federal Excise Act.
No amount is payable under this Act in tax year 2016 as SED is no more applicable.

Winter - 2007 Q. 2(b)


X Limited has granted an option to all of its employees for purchase of its 1,000 shares at a price of Rs. 25/- per share.
Habib, one of the employees has exercised the option available to him. The market value of the shares at the time of
exercise of option was Rs. 45/- per share.

Required: Compute the income, if any, chargeable under the following heads of income:
(i) Salary income; and (ii) Capital gain

Solution

Mr. Habib
Computation of taxable income

Conceptual Approach to Taxes _________________________________513


Chapter 25 Solved Past Papers Income Tax Numericals of ICMAP Stage IV - (2003 to 2015)

INCOME FROM SALARY U/S 12

FMV of shares under employee shares scheme (1,000 x 45) Note 1 45,000
Less: cost of shares (1,000 x 25) 25,000
20,000

CAPITAL GAIN U/S 37 Note 2 -


Taxable income 20,000

Note 1: It is assumed that shares were issued to employees without any restriction on their sale or transfer.

Note 2: As shares have not been sold by Mr. Habib, hence nothing shall be taxable under head capital gain.

Winter - 2007 Q. 4
Mr. Arif, Baqar and Umer are member of an Association of Persons (AOP) "FRIENDSCO" and share the profit &
loss in the ratio of 1:2:3 respectively. They wanted to know their tax liability for the tax year, 2008. Accountant of
M/s FRIENDSCO., has prepared the following profit and loss account:

(Rupees)
Sales 6,400,000
Cost of sales 3,200,000
Gross profit 3,200,000
Selling and admin expenses 2,400,000
Net profit before tax 800,000

Additional Information:

1. It is a wholesale business and sales include supplies of Rs. 800,000 to government departments subject to
withholding tax.

2. Expenses include:

(i) Accounting depreciation of Rs. 75,000 on vehicle with W.D.V., of Rs. 500,000

(ii) Provision for bad debts of Rs. 50,000 has been made, whereas actual bad debts are Rs. 80,000.

(iii) Commission of Rs. 120,000 has been paid to Mr. Arif for promotion of sales
(iv) Utility bills amounting to Rs. 80,000 charged to expenses include Rs. 15,000 income tax withheld on
these bills.

3. Mr. Baqar is a sleeping partner. He is working as full time teacher in a university and receives monthly pay and
allowances as under:

(i) Pay Rs. 60,000

(ii) House rent allowance Rs. 30,000


4. Mr. Umer is in reciept of income from property of Rs. 50,000 per month.

5. The accounting depreciation on vehicle is also charged @ 15% of W.D.V., which coincides with the statutory
rate of depreciation.

Solution

M/s FRIENDSCO
AOP
Computation of taxable income and tax liability
For the tax year 2015
Rs. Rs. Rs.
Total NTR FTR

514_______________________________ Conceptual Approach to Taxes


Chapter 25 Solved Past Papers Income Tax Numericals of ICMAP Stage IV - (2003 to 2015)

Allocation percentage (on basis of sales) 100% 87.50 12.50


(Rs. 5,600,000 / 6,400,000 x 100) = 87.5%

Sales 6,400,000 5,600,000 800,000


Cost of sales 3,200,000 2,800,000 400,000
Gross profit 3,200,000 2,800,000 400,000
Selling and admin exp. (assumed including bad debts) 2,400,000 2,100,000 300,000
Net profit before tax 800,000 700,000 100,000

Add:
Accounting depreciation 75,000 65,625 9,375
Provision for bad debts 50,000 43,750 6,250
Commission paid to partner (Mr. Arif) 120,000 105,000 15,000
Income tax withheld on utility bills 15,000 13,125 1,875
260,000 227,500 32,500
Less:
Tax depreciation (Rs. 500,000 x 15%) = 75,000 65,625 9,375
Taxable income of AOP 985,000 861,875 123,125

Computation of tax liability:

Tax liability under NTR:


Tax on Rs. 861,875 [32,000 + 15% x (861,875 - 750,000)] (A) 48,781

Minimum tax u/s 235(4)


Tax paid with electricity bill (B) 15,000

Higher of (A) or (B) 48,781

Tax liability under FTR:


Tax on Government Supplies (Rs. 800,000 x 4.5%) = 36,000
Total tax liability 84,781
Less: Tax paid / deducted at source
Tax on supplies ( Rs.800,000 x 4.5%) 36,000
Tax on electricity bills 15,000
51,000
Balance tax payable 33,781

Divisible Income of AOP:

Taxable income under NTR 861,875


Tax on income under NTR 48,781
Divisible income 813,094

AOP income from FTR has been ignored for divisiable income among the partners.

Calculation of share of profit from AOP


TOTAL Mr. Arif Mr. Baqar Mr. Umer
Share 6 1 2 3

Commission 120,000 120,000 - -


Balance divided (in sharing ratio) 693,094 115,516 231,031 346,547
813,094 235,516 231,031 346,547

COMPUTATION OF TAXABLE INCOME and TAX LIABILITY OF MEMBERS


Mr. Arif Mr. Baqar Mr. Umer

INCOME FROM SALARY U/S 12

Conceptual Approach to Taxes _________________________________515


Chapter 25 Solved Past Papers Income Tax Numericals of ICMAP Stage IV - (2003 to 2015)

Pay (Rs. 60,000 x 12 months) 720,000


House rent allowance (Rs. 30,000 x 12 months) 360,000
Taxable income 1,080,000
Income from property - - 600,000
Share from AOP (for rate purpose) - 231,031 346,547
Taxable income for rate purpose 1,311,031 946,547

Note: Share from AOP has not been included in income of Arif and Umer as they have no other income
chargeable to tax under NTR.

Computation of tax liability: Mr. Arif Mr. Baqar Mr. Umer

As the salary income of Mr. Baqar constitute more than 50% of the total income hence tax under salary slab is
computed as under.

Tax liability under NTR:

Tax on Rs. 1,311,031 [14,500 + 10% x (1,311,031 - 750,000)] - 70,603 -


Tax on Rs. 946,547 [32,000 + 15% x (946,547 - 750,000)] - - 61,482

Less: 40% reduction to full time teacher - 28,241 -


- 42,362 61,482
Less: Tax on share profit from AOP
(42,362 / 1,311,031 x 231,031) - 7,465 -
(61,482 / 946,547 x 346,547) - - 22,510
Balance tax payable - 34,897 38,972

Note: In the absence of information it has been assumed that the property income is after allowable expenses.

Summer - 2007 Q. 4

Colonel (Retired) Ahmed Ali is the Managing Director of a listed public company and has provided the following details of his
expected income and expenses for the year ending June 30, 2007:

Salary income:

1. Basic salary Rs. 1,800,000 per year

2. Dearness allowance 10% of basic salary

3. Medical allowance Rs. 25,000 per month

Managing Director is not entitled for any reimbursement of actual expenses of hospitalization or medical
treatment.

4. The company disbursed on July 1, 2006 to Colonel Ahmed Ali, Rs. 3 million interest free loan to be recovered
from the final dues on retirement. The bench mark rate for the year 2007 is 7% as notified by Federal
Government

5. Company has paid Rs. 850,000 as annual rent for the accommodation provided to Managing Director.

6. He has been provided with a company maintained car for business and personal use. The purchase price of car
is 1.2 million. Company also pays salary to driver @ Rs. 8,000 per month

7. Colonel Ahmed Ali is receiving pension from Army @ Rs. 10,000 per month

Property income:

Colonel Ahmed Ali owns a flat, which has been let out @ Rs. 45,000 per month. He has incurred following
expenses to date on flat during the year.

a) Repair and maintenance Rs. 30,000 / -


b) Property tax Rs. 20,000 / -

516_______________________________ Conceptual Approach to Taxes


Chapter 25 Solved Past Papers Income Tax Numericals of ICMAP Stage IV - (2003 to 2015)

c) Insurance of flat Rs. 15,000 / -

Other Income:

1. Colonel Ahmed Ali has also received Rs. 500,000 as his share u/s 92(1) of Income Tax Ordinance, 2001 being
the member of an AOP, where the AOP has paid the tax.

2. Colonel Ahmed Ali has won a cash prize of Rs. 200,000 from a company which offered the prize for promotion
of sales u/s 156 of Income Tax Ordinance.

Required: Compute the expected total income, taxable income and income tax thereon for the tax year 2007.

Solution

Colonel (Retired) Ahmed Ali


Computation of taxable income and tax liabiltiy
For the tax year 2016
Rs. Rs.
INCOME FROM SALARY U/S 12

Basic salary 1,800,000


Dearness allowance (Rs. 1,800,000 x 10%) 180,000
Medical allowance (Rs. 25,000 x 12 months) 300,000
Less: Exempt upto 10% of basic salary U/C 139 180,000 120,000
Interest free loan (3,000,000 x 10% p.a. for the whole year) 300,000
Accommodation provided
Higher of:
Actual Rs. 850,000; or
45% of basic salary i.e. 810,000 850,000
Conveyance ( 1,200,000 x 5%) U/R 5 60,000
Salary to driver (Rs.96,000 x 50% for personal use) 48,000
Pension from Army (exempt) -
Taxable salary income 3,358,000

INCOME FROM PROPERTY U/S 15

Rent (Rs. 45,000 x 12 months) 540,000


1/5th fixed repair allowance irrespective of actual repair and maintenance (108,000)
Property tax (20,000)
Insurance of flat (15,000)
397,000
Taxable income 3,755,000
Share of profit from AOP included for rate purposes 500,000
Taxable income for rate purposes 4,255,000

Computation of tax liability:

As the salary income of the indiviual constitute more than 50% of the total income hence tax under salary slab is
computed as under.

Tax liability under NTR

Tax on Rs. 4,255,000 [597,000 + 27.5% x (4,255,000 - 4,000,000)] 667,125

Tax on income excluding share from AOP


(667,125 x 3,755,000 / 4,255,000) 588,732

Tax liability under SBI as FTR


Cash prize under section 156 (Rs. 200,000 x 20%) 40,000
Total tax liability 628,732

Note: Although no information has been provided however the aforesaid liability shall be reduced by Rs. 40,000 tax

Conceptual Approach to Taxes _________________________________517


Chapter 25 Solved Past Papers Income Tax Numericals of ICMAP Stage IV - (2003 to 2015)

deducted at source on cash prize.

Winter - 2006 Q. 5

You are engaged in the Income Tax Consultancy services. One of your clients Mr. A.B Malik, an employee of M/s. Excellent
Airlines, Peshawar, asks you to determine his total income, taxable inocme and income tax payable by him for tax year, 2007
on the basis of the following information:

Salary Income (per month)

Basic salary Rs. 22,500/- (in pay scale of Rs. 15,000 - 750 - 26,250)
Dearness allowance Rs. 1,500/-
Special relief allowance @ Rs. 15% of basic salary.
Flying allowance Rs. 15,000/-
Entertainment allowance Rs. 500/-
Medical allowance @ 5,000/- No free medical facility of hospitalization or re-imbursement of medical charges is provided by
the employer.

Accommodation is provided by the employer. Mr. A. B. Malik is entitled to house rent allowance @ 60% of the minimum of
pay scale, had the accommodation not been provided.

A 800cc vehicle is provided by the employer partly for official and partly for private use. The cost of the vehicle to the
employer is Rs. 450,000/-

Property Income

Mr. A. B. Malik owns a house which is on rent @ Rs. 16,000 per month.

Investment / contribution during the year.

Investment in shares Rs. 50,000.


Contribution to an approved pension fund Rs. 100,000.
Mr. A. B. Malik also obtained the concessional loan amounting to Rs. 1,500,000/- from the employer, mark-up rate of which is
4% per annum (Assume that the benchmark rate for the tax year, 2007 is 9% per annum).

Solution
Mr. A.B. Malik
Computation of taxable income and tax liability
For the tax year 2016

INCOME FROM SALARY U/S 12: Rs. Rs.

Basic salary (Rs. 22,500 x 12 months) 270,000


Dearness allowance (Rs. 1,500 x 12 months) 18,000
Special relief allowance (Rs. 270,000 x 15%) 40,500
Entertainment allowance 6,000
Medical allowance (Rs. 5,000 x 12 months) 60,000
Less: Exempt upto 10% of basic salary U/C 139 27,000 33,000
Accommodation:
Higher of 60% or 45% of MTS [(Rs. 15,000 x 12) x15%] 108,000
Conveyance (450,000 x 5%) U/R 5 22,500
Concessional loan from employer
Interest at benchmark rate of 10% 150,000
Less actual interest at 4% 60,000 90,000
588,000
Flying allowance (Note 1) 180,000 -

518_______________________________ Conceptual Approach to Taxes


Chapter 25 Solved Past Papers Income Tax Numericals of ICMAP Stage IV - (2003 to 2015)

INCOME FROM PROPERTY U/S 15:


Rent (16,000 x 12) (assumed after deductions) 192,000
Taxable income 780,000

COMPUTATION OF TAX LIABILITY:


As the salary income of the individual constitute more than 50% of the total income hence tax under salary slab
is computed as under.

Tax liability under NTR


Tax on Rs. 780,000 [14,500 + 10% x (780,000 - 750,000)] (A) 17,500
Less: Tax credit on investment in shares and insurance
Tax credit shall be allowed on lower of:
- Actual amount of investment i.e. Rs. 50,000
- 20% of taxable income i.e. Rs. 156,000
- Rs. 500,000
Tax credit = (50,000 x 17,500 / 780,000) 1,122
Less: Tax credit on contribution to approved pension fund
Tax credit shall be allowed on lower of:
- Actual amount of contribution i.e. Rs. 100,000
- 20% of taxable income under NTR including SBI income i.e. Rs.156,000

Tax credit = (100,000 x 17,500 / 780,000) 2,244


(B) 3,365
Add: Tax on Flying allowance (Rs.180,000 x 2.5% upt One annual basic pay) (C) 4,500
Balance tax payable D=A-B+C 18,635

(Note 1)

Flying allowance received as Officers other than as Pilot shall not be included in taxable income as the same is taxable as a

SBI @ 2.5% upto one basic pay u/c (1) of Part III of 2nd Schedule to the Income Tax Ordinance, 2001. If the flying allowance

is more than the basic pay then the balance shall be included in the taxable income under NTR of the individual.

If the flying allowance and other allowances will be received as Pilot of any Pakistani Airline the same shall be included in the

taxable income under NTR upto basic pay. If the flying & other allowances are more than basic pay then the balance shall be

taxable as a SBI @ 7.5% u/c (1AA) of Part III of 2nd Schedule to the Income Tax Ordiannce, 2001.

Summer - 2006 Q. 4
Mr. Asghar Abbas is assistant manager in an engineering organization. He has engaged you as his tax consultant and

provided you with the following information of his estimated incomes and expenses for the year ending 30th June, 2006:-

Salary Income (per month) Rs.


Basic salary 18,000
House rent allowance 10,000
Utility allowance 6,000
Medical allowance 3,000
Conveyance allowance 5,000

Mr. Asghar owns and maintains a car, which he uses partly for his personal and partly for business use of his employer. In
January, 2006 he remained admitted in the hospital for 10 days and the employer reimbursed hospitalization charges of Rs.
25,000.
Property Income (per month)
He owns 5 shops, which are rented out. His income and expenses in this behalf are as under:
Rs.
Annual rent of 4 shops 96,000

Conceptual Approach to Taxes _________________________________519


Chapter 25 Solved Past Papers Income Tax Numericals of ICMAP Stage IV - (2003 to 2015)

Shop No. 5 remained occupied for 7 months at a rent of Rs. 1,200 per month.
It was vacated through court order for which Mr. Asghar incurred Rs. 6,000 as
legal expenses
Ground rent 2,000
Collection charges paid 6,000
Property tax (This amount includes Rs. 5,000 of last year's tax not paid 10,000
2005.)
Agricultural Income 60,000
Dividend received on investment in shares of a public limited company (gross) 20,000
The employer has deducted tax at source from salary amounting to Rs. 12,000.

Required: You being a tax consultant of Mr. Asghar Abbas, calculate his taxable income and tax payable along
with this payable along with his tax return.
Solution
Mr Asghar Abbas
Computation of taxable income and tax liability
For the tax year 2016

INCOME FROM SALARY U/S 12 Rs. Rs.


Basic salary (Rs. 18,000 x 12 months) 216,000
House rent allowance (Rs. 10,000 x 12 months) 120,000
Utility allowance (Rs. 6,000 x 12 months) 72,000
Medical allowance (Rs. 3,000 x 12 months) 36,000
Less: Exempt upto 10% of basic salary U/C 139 (Note - 1) 21,600 14,400
Conveyance allowance (Rs. 5,000 x 12 months) 60,000
Hospitalization charges reimbursed (Note- 1) 25,000
(Assumed that it is not in accordance with the terms of employment) 507,400

INCOME FROM PROPERTY U/S 15

Rent of 4 shops 96,000


Rent of shop No. 5 (1,200 x 7) 8,400
104,400
Less: Admissible deductions U/S 15A
Legal expenses (6,000)
1/5th fixed repair allowance (20,880)
Ground rent (2,000)
Collection charges paid (6,000)
Property tax (Excluding Rs. 5,000 of last year's tax not paid) (5,000)
(39,880)
Net income from property chargeable to tax 64,520

INCOME FROM OTHER SOURCES U/S 39

Agricultural income (exempt u/s 41 & assumed tax paid under provincial law) 60,000 -
Taxable income 571,920

COMPUTATION OF TAX LIABILITY:

As the salary income of the indiviual constitute more than 50% of the total income hence tax under salary slab is
computed as under.
Tax liability under NTR:
Tax on Rs. 571,920 [2,000 + 2% x (571,920 - 500,000)] 5,596
Tax liability under SBI as FTR

520_______________________________ Conceptual Approach to Taxes


Chapter 25 Solved Past Papers Income Tax Numericals of ICMAP Stage IV - (2003 to 2015)

Tax on dividend income 2,000


Total tax liability 7,596
Less: Tax paid / deducted at source:
On salary 12,000
On dividend income (although not given in question) 2,000
Balance tax refundable (6,404)

(Note-1) If the medical reimbursement of medical expeses to employee are provided alongwith medical allowance as per
terms of employment then the 10% exemption for medical allowance shall not be allowed however the medical re-
imbursement shall be totally exempt from tax provided the hospital / clinic NTN and bills are provided by the employee.

Winter - 2005 Q. 5
Following are the details of Mr. Shakeel for the financial year ended June 30, 2005 who is employed with the Institute of
Computer Sciences as Professor. He joined the Institute on July 1, 2004. Previously he worked with Institute of computer
studies. He was required to give 3 months' notice to his previous employer to terminate his employment. He served notice
period for 1 month and his current employer paid the notice pay equivalent to 2 months' salary to the previous employer
amounting to Rs. 150,000. Other details for the year are as follows:

Salary Income
Rs. per month
Pay 90,000
House rent allowance 40,500
Utilities 9,000

No company maintained car was provided to him. However, 175 litres petrol per month was given to him @ Rs. 50
per litre.

Investments

Investment in shares of listed companies 175,000

House loan

Payment of interest to Commercial Bank on house loan 250,000

Charitable Donation: He paid Rs. 50,000 to non-profit organization.

Required: Compute taxable income and tax liability of Mr. Shakeel for tax year 2005. Prepare and present all
necessary workings.

Solution

Mr. Shakeel - Resident


Computation of taxable income and tax liability
For the tax year 2016
Rs. Rs.
INCOME FROM SALARY U/S 12

Pay (Rs. 90,000 x 12 months) 1,080,000


House rent allowance (Rs. 40,500 x 12 months) 486,000
Utilities (Rs. 9,000 x 12 months) 108,000
Petrol from employer (175 x 50 x 12) (assumed not for offical performance) 105,000
Employee's obligation paid by employer 150,000
Taxable salary income 1,929,000

COMPUTATION OF TAX LIABILITY:


As there is only salary income hence tax liability for salaried individual in computed as under.

Tax on Rs. 1,929,000 [137,000 + 17.5% x (1,929,000 - 1,800,000)] 159,575

Tax credit on investment in shares and insurance U/S 62


Tax credit shall be allowed on lower of:

Conceptual Approach to Taxes _________________________________521


Chapter 25 Solved Past Papers Income Tax Numericals of ICMAP Stage IV - (2003 to 2015)

- Actual amount of investment i.e. Rs. 175,000


- 20% of taxable income i.e. Rs. 385,800
- Rs. 1,000,000

Tax credit = (175,000 x 159,575 / 1,929,000) 14,477

Tax credit on mark up on house loan U/S 64 (No more available) -

Tax credit on donation U/S 61


Tax credit shall be allowed on lower of:

- Actual amount of donation i.e. Rs. 50,000


- 30% of taxable income i.e. Rs. 578,700

Tax credit = (50,000 x 159,575 / 1,929,000) 4,136


18,613
Balance tax payable 140,962

Summer - 2005 Q. 5

Following are the details of income of Mr. A. Rehman for the financial year ended June 30, 2004, who is employed with a
company as Senior Manager.
Salary income:
Pay Rs. 60,000 per month
House rent allowance Rs. 27,000 per month
Utilities Rs. 8,000 per month
He was provided with a company maintained car of 800 cc, partly for business and partly for his personal use. The cost of
the car to the company was Rs. 500,000.
Rental income:
Rs.
Annual letting value of property 372,000
(including Rs. 120,000 for furniture and fixtures).
During the year property remained vacant for 4 months 8,000
Ground rent paid 200,000
Business income 200,000
Capital gains:
Cost of shares of unlisted companies (bought in 1986) 500,000
Sale proceeds of shares 600,000
Investments:
Investment in shares of listed companies 180,000
Premium paid for life annuity eligible u/s 63 45,000
Required:
Compute taxable income & tax liability of Mr. A. Rehman for the year. Prepare & present all necessary workings.

Solution
Mr. A. Rehman - Resident
Computation of taxable income and tax liability
Tax year 2016
Rs. Rs.
INCOME FROM SALARY U/S 12
Pay (Rs. 60,000 x 12 months) 720,000
House rent allowance (Rs. 27,000 x 12 months) 324,000
Utilities (Rs. 8,000 x 12 months) 96,000
Conveyance (500,000 x 5%) U/R 5 25,000
1,165,000
INCOME FROM BUSINESS (asumed under NTR) U/S 18 200,000

522_______________________________ Conceptual Approach to Taxes


Chapter 25 Solved Past Papers Income Tax Numericals of ICMAP Stage IV - (2003 to 2015)

INCOME FROM PROPERTY U/S 15

8 months letting value of property [ (372,000 - 120,000) x 8/12] 168,000


Less: 1/5th of rent as repair allowance (33,600)
Less: Ground rent (200,000) (65,600)

INCOME FROM OTHER SOURCES U/S 39

Rent of furniture and fixture (assumed for 8 months) 120,000

CAPITAL GAINS U/S 37


Sale proceeds of shares 600,000
Less: cost of shares 500,000
100,000
Less: 25% exempt as sold after one year from the date of
purchase u/s 37 25,000 75,000
Taxable income 1,494,400

COMPUTATION OF TAX LIABILITY:

As the salary income of the indiviual constitute more than 50% of the total income hence tax under salary slab is
computed as under.

Tax liability under NTR


Tax on Rs. 1,494,400 [79,500 + 12.5% x (1,494,400 - 1,400,000)] 91,300
Tax credit on investment in shares and insurance U/S 62
Tax credit shall be allowed on lower of:
- Actual amount of investment i.e. Rs. 180,000
- 20% of taxable income i.e. Rs. 298,880
- Rs. 1,000,000
Tax credit = (180,000 x 91,300 / 1,494,400) 10,997
Tax credit on contribution to approved pension fund U/S 63
Tax credit shall be allowed on lower of:
- Actual amount of contribution i.e. Rs. 45,000
- 20% of taxable income i.e. Rs. 298,880
Tax credit = (45,000 x 91,300 / 1,494,400) 2,749
13,746
Balance tax payable 77,554
Winter - 2004 Q. 5
Mr. Baqar, Mr. Hadi, and Mr. Mikdad are members of an Association of Persons, (AOP) which is doing business under the
name of M/s BHM Associates. As per their agreement, Mr. Baqar is entitled to receive 5% interest on his capital employed,
besides his share from profit. Mr. Hadi is entitled to receive commission of 1% in recognition of his efforts to promote sales.
The profit is to be distributed equally amongst the members. During the year ended June 30, 2004, the books of accounts of
M/s BHM Associates showed following results:

Rs.
Sales 4,500,000
Gross profit 1,800,000
Selling and admin expenses 1,520,000

Further Notes are as under:

1. An asset, which has a written down value of Rs. 100,000 on July 1, 2003 was disposed of for Rs. 150,000 on May 30,
2004. Neither depreciation nor gain on its disposal was recorded in the books of accounts (Assets are depreciated at rates
given in the third schedule to the Income Tax Ordinance, 2001).

2. Interest on capital employed and commission on sales were properly recorded in the books of accounts and paid to Mr.

Conceptual Approach to Taxes _________________________________523


Chapter 25 Solved Past Papers Income Tax Numericals of ICMAP Stage IV - (2003 to 2015)

Baqar and Mr. Hadi.


3. Provision for bad debts charged to profit and loss account was 6% of sales. During the year actual bad debts written off
amounted to Rs. 150,000.

4. Capital employed by each member of AOP is as under:


Rs.
Mr. Baqar 1,000,000
Mr. Hadi 200,000
Mr. Mikdad 300,000

5. Other incomes of members were as under

(A) Mr. Baqar received salary of Rs. 300,000 (inclusive of house rent allowance of Rs. 120,000) from another company
during the year. The employer deducted Rs. 21,000 tax at source from salary. He also earned profit of Rs. 20,000 on his
bank deposits on which 10% tax was withheld by the bank.

(B) Mr. Hadi in addition to his commission and share of AOP income has also received agricultural income of Rs. 120,000
during the year. His income from his own buniness for the year ended June 30, 2004 was Rs. 100,000.

(C) Mr. Mikdad during the year supplied goods to government departments worth Rs. 1,500,000 on which Income Tax @
3.5% was withheld by those departments
Required:
Work out the following:
(i) Taxable income of M/s BHM Associates (AOP), for the year, 2004.
(ii) Taxable income of each of its members and their tax liabilities for the tax year, 2004.
Solution
M/s BHM Associates - Resident AOP
Computation of taxable income and tax liability
For the tax year 2016 Rs.

INCOME FROM BUSINESS U/S 18

Sales 4,500,000
Cost of sales 2,700,000
Gross profit 1,800,000
Selling and admin expenses 1,520,000
Net profit before tax 280,000

Add:
Interest on capital to Mr. Baqar ( Rs. 1,000,000 x 5% ) 50,000
Commission to Mr. Hadi ( Rs. 4,500,000 x 1% ) 45,000
Gain on sale of fixed assets ( 150,000 - 100,000 ) 50,000
( no depreciation shall be charged in the year of disposal)
Provision for bad debts (6% of sales) 270,000
415,000
Taxable income of AOP 695,000

COMPUTATION OF TAX LIABILITY:

Tax liability under NTR:


Tax on Rs. 695,000 [7,000 + 10% x (695,000 - 500,000)] 26,500
Divisible Income of AOP:
Taxable income under NTR 695,000
Tax on income under NTR 26,500
Divisible income 668,500
Calculation of share of profit from AOP
TOTAL Mr. Baqar Mr. Hadi Mr. Mikdad

524_______________________________ Conceptual Approach to Taxes


Chapter 25 Solved Past Papers Income Tax Numericals of ICMAP Stage IV - (2003 to 2015)

Share 3 1 1 1
Commission 45,000 45,000
Interest on capital employed 50,000 50,000
Balance (divided in partners) 573,500 191,167 191,167 191,167
668,500 241,167 236,167 191,167
Computation Of Taxable Income and Tax Liability Of Members
Mr. Baqar Mr. Hadi Mr. Mikdad
INCOME FROM SALARY U/S 12
Salary 300,000 - -

INCOME FROM BUSINESS U/S 18 - 100,000 -

INCOME FROM OTHER SOURCES U/S 39

Agricultural income (exempt under section 41) -


Taxable income 300,000 100,000 -
Share from AOP 241,167 236,167 -
Taxable income for rate purpose 541,167 336,167 -
Note: Share from AOP shall not be included in the income of Mr. Mikdad as he has no source of income
chargeable to tax under NTR.
Note: Tax rates for salaried individuals shall be applied on taxable income of Mr. Baqar, Tax rates for
non-salaried individual shall be applied on taxable income of Mr. Hadi.
Computation of tax liability: Mr. Baqar Mr. Hadi Mr. Mikdad
Tax liability under NTR:
[2,000 + 5% x (540,167 - 500,000)] 4,008 - -

Tax on income excluding share from AOP 2,222 - -


Tax liability under FTR:

Tax on profit on debt U/S 151 (Rs. 20,000 x 10%) 2,000 - -


Tax on sale of goods U/S 153 (Rs. 1,500,000 x 4.5%) - - 67,500
2,000 - 67,500
Total tax liability 4,222 - 67,500
Less: tax deducted at source
On salary U/S 149 21,000 - -
Profit on debt U/S 151 2,000 - -
on supply of goods U/S 153 - - 67,500
23,000 - 67,500
Balance tax refundable (18,778) - -
Summer - 2004 Q. 5
Mr. Qais Mansoor is Director-cum-Company Secretary of M/s Badar Salam and Co. Ltd., since the year 1990.
His monthly remunerations are as follows:
Rs.
Basic salary 20,000
House rent allowance 8,333
Utility allowance 2,000
Medical allowance 2,000
32,333
He was provided maintenacne cost of his private car used wholly for the company business on actual basis aggregating Rs.
20,000. He received bonuses equivalent to three basic salaries, plus two basic salaries as special merit rewards during the
year.

The company disbursed funeral expenses of his parents in the amount of Rs. 20,000 and also medical costs on birth of his
twin sons in the sum of Rs. 100,000 latter being as per employment terms.

Conceptual Approach to Taxes _________________________________525


Chapter 25 Solved Past Papers Income Tax Numericals of ICMAP Stage IV - (2003 to 2015)

The company has also provided him with free furnished accommodation costing Rs. 200,000 per annum. The company also
paid his tax liability of Rs. 20,000.
He was decorated with the President's Award, in August 2002 and March 2003 worth Rs. 500,000.
He earned capital gains on sale of listed shares (Rs. 20,000) and on sale of land (Rs. 100,000).
Tax deducted from salary Rs. 40,000
He paid following amounts evidenced by receipts bearing payees N.T.Ns, wherever, applicable:
1. School fees @ Rs. 3,000 per month, for each of his two daughters.
2. Fee to personal silicitor and tax adviser Rs. 20,000.
3. Prior year income and penalties Rs. 50,000.
4. Donation to approved institutions Rs. 500,000.
5. Purchase of second car for Rs. 1,000,000 for family use.
Required: As a tax consultant you are required to calculate total income, and tax liability of Mr. Qais Mansoor for tax year
2003.
Solution
Mr. Qais Mansoor - Resident
Computation of taxable income and tax liability
For the tax year 2016
Rs. Rs.
INCOME FROM SALARY U/S 12
Basic salary (Rs. 20,000 x 12 months) 240,000
Bonus (20,000 x 3) 60,000
House rent allowance (Rs. 8,333 x 12 months) 99,996
Utility allowance (Rs. 2,000 x 12 months) 24,000
Medical allowance (taxable as medical re-imbursment separately provided) (Rs. 2,000 x 12) 24,000
Reimbursement of official expenses (exempt) -
Special merit reward (20,000 x 2) 40,000
Personal expenses paid by company (Funeral expenses) 20,000
Accommodation:
Higher of Rs. 200,000, or
45% of B.S i.e. Rs.108,000 200,000
Tax liability paid by employer 20,000
727,996
CAPITAL GAINS U/S 37

Capital gain on sale of shares of listed company (SBI)


(0% rate on the assumtion of holding period of shares is more than four years)
Gain on sale of land (not taxable on assumption of holding period is more than two years) -

INCOME FROM OTHER SOURCES U/S 39

President's award (exempt under 2nd schedule) -


Taxable income 727,996

COMPUTATION OF TAX LIABILITY:

As the salary income of the indiviual constitute more than 50% of the total income hence tax under salary slab is
computed as under.
Tax on Rs. 727,996 [2,000 + 5% x (727,996 - 500,000)] 13,400

Tax credit on donation U/S 61


Tax credit shall be allowed on lower of:
- Actual amount of donation i.e. Rs. 500,000
- 30% of taxable income i.e. Rs. 218,399
Tax credit = (218,399 x 13,400 / 727,996) 4,020

526_______________________________ Conceptual Approach to Taxes


Chapter 25 Solved Past Papers Income Tax Numericals of ICMAP Stage IV - (2003 to 2015)

Total tax liability 9,380


Less: Tax deducted from salary 40,000
Tax paid by employer 20,000
60,000
Balance tax refundable (50,620)
Note: There is no treatment of school fee of children, fee to solicitor and tax adviser, prior year income and penalties and
purchase of car.
Summer - 2003 Q. 5
Mr. Musaddiqe Noor is a consultant in a group of companies. He derived following income during the income year July 1,
2002 to June 30, 2003:

Particulars Rs.
Per month
(i) Salary income
Basic salary 20,000
House rent allowance 8,000
Utility allowance 1,000
Medical allowance 1,000

He is also provided with a 1,000 cc Car, which is partly used for company's business. He has also been granted with a
housing loan of Rs. 500,000 on which no profit/interest has been charged.

In addition to above, he also received gratuity of Rs. 75,000 from his previous employers during the year. The gratuity fund is
not approved by the Commissioner of Income Tax or CBR.

Tax deducted at source from salary amounted to Rs. 15,000.

(ii) Property income Rs.


Rent from a house letout (per month) 10,000
He incurred following expenses on this property during the year:
Repairs 30,000
Collection charges: 7% of rent
Ground rent 10,000
Property tax 15,000
Rent-sharing with housing finance company (per month) 3,000

He received a deposit of Rs. 200,000, not adjustable against rent, out of which he refunded Rs. 100,000 to previous
tenant, who vacated the house after 3 years' tenancy.

(iii) Other income Rs.


Profit on PLS Bank account (net of 10% tax withheld) 9,000
Commission from insurance Company and from sale of plots
(net of 5% taxes withheld) 19,000
Lecturing and examination services fees from professional
institutes (net of 5% tax withheld) 19,000

Required:

As a tax consultant you are required to compute Mr. Musaddique's total income and his income tax liability for the tax year
2003.
Solution
Mr. Musaddiqe Noor - Resident
Computation of tax liability
For the tax year 2016
Rs. Rs.
INCOME FROM SALARY U/S 12
Basic salary (Rs. 20,000 x 12 months) 240,000

Conceptual Approach to Taxes _________________________________527


Chapter 25 Solved Past Papers Income Tax Numericals of ICMAP Stage IV - (2003 to 2015)

House rent allowance (Rs. 8,000 x 12 months) 96,000


Utility allowance (Rs. 1,000 x 12 months) 12,000
Medical allowance (Rs. 1,000 x 12 months) 12,000
Less: Exempt upto 10% of basic salary U/C 139 24,000 -

Conveyance (Value no given hence 5% of Rs.1,000,000 assumed) U/R 5 50,000


Interest free loan from employer (As the amount of loan is Rs. 500,000 hence
nothing shall be added as benefit in the salary income of employee U/S 13(7) -
Un-approved gratuity 75,000
Less: Exempt upto lower of:
Rs. 75,000, OR
50% of the amount receivable i.e. 37,500 37,500 37,500
435,500
INCOME FROM PROPERTY U/S 15

Rent (with share of HBFC) 120,000


Un-adjustable advance [ 200,000 - (100,000 / 3) / 10] / 10 17,000
Total rent 137,000

Less admissible expenses


Repairs (restricted to 1/5th of rent chargeable to tax) 27,400
Collection & admin. charges (restricted to 6% of rent chargeable to tax) 8,220
Ground rent 10,000
Property tax 15,000
Rent-sharing with housing finance company (per month) 36,000
Balance amount chargeable to tax 96,620 40,380

INCOME FROM OTHER SOURCES U/S 39

Lecturing and examination services fees (18,000 x 100 / 90) 20,000


Taxable income 495,880

COMPUTATION OF TAX LIABILITY:

As the salary income of the individual constitute more than 50% of the total income hence tax under salary slab
is computed as under.
Tax liability under NTR
Tax on Rs. 495,880 [0 + 2% x (495,880 - 400,000)] 1,918

Less proportionate tax on consultancy services


(1,918 / 495,880 x 20,430) 77
Balance tax under NTR 1,840
Proportionate tax on consultacny services as above (D) 77
Minimum tax on services
Rs. 20,000 @ 10% (E) 2,000
Add: Higher of (D) and (E) shall be included in tax under NTR 2,000
Total tax liability under NTR 3,840
Tax liability under FTR
Profit on PLS account (9,000 x 10 / 90 ) 1,000
Tax on commission @ 10% ( 18,000 x 10 / 90) 2,000
3,000
Total tax liability under NTR and FTR 6,840
Less: Tax paid / deducted at source
On salary 15,000
On profit on PLS account 1,000
On commission ( 20,000 x 10% already deducted on gross receipts) 2,000

528_______________________________ Conceptual Approach to Taxes


Chapter 25 Solved Past Papers Income Tax Numericals of ICMAP Stage IV - (2003 to 2015)

On services ( 20,000 x 10% as per given rate on gross receipts) 2,000


20,000
Balance tax refdundable (13,160)
Winter - 2003 Q. 6
Akeel Hospital is an unregistered partnership firm owned by Dr. Raees and Lady Doctor Mrs. Ramis. You being a tax
practitioner have been approached to prepare the Income Tax Return of the firm from following revenue account and other
relevant data as provided to you for the tax year 2003:
Akeel Hospital
Revenue Account
Year 2003
Rs. Rs.
Salaries 1,200,000 Gross In-patient receipts
Hospital supplies 300,000 (before tax deductions) 2,500,000
Electricity 200,000
Communication 100,000 O.P.D collections
Insurance 60,000 (before tax deductions) 500,000
Water 20,000
Repair and maintenance 20,000 Sale of scrap 100,000
Depreciation on Fixed
assets (excluding burnt
out equipment) 100,000
Property taxes 50,000
Radiology/Lab Expenses 100,000
Professional fees paid 50,000
Expenses on free weekly
clinic for community 100,000
Unsupported payments 50,000
Penalties for non-observance
of good environmental regulations. 50,000
Net profit 700,000
3,100,000 3,100,000

Other informaton / data:


(i) Depreciation includes Rs. 20,000 on a car completely in personal use of partners.
(ii) Salaries include drawings of Rs. 100,000 made by each of the partners.
(iii) A Philanthropist donated Rs. 1.0 million through cheque for construction of special cancer ward.
(iv) Taxes withheld by corporate clients aggregated Rs. 200,000.
(v) Mr. Waqar, a friend, extended an interest-free loan of Rs. 100,000 to the hospital during the year.

(vi) A foreign patient, being satisfied with Hospital's best quality care gave a cash gift of Rs. 150,000 for further improving the
Hospital's services. This amount has not been accounted for so far.

(vii) A surgical equipment costing Rs. 100,000 was accidentally burnt out, while dacoits took away Rs. 50,000 from the safe-
lockers. Both items are fully insured and the claim has been fully admitted by the insurance Company.

(viii) The Hospital has added 10 renowned companies to the panel with expected increase of Rs. 2,000,000 in future
revenue.
(ix) Salaries include excess perquisites of Rs. 80,000.
Required: Work out Hospital's total income and tax liability for tax year 2003.
Solution
Akeel Hospital - Resident AOP
Computation of taxable income and tax liability

Conceptual Approach to Taxes _________________________________529


Chapter 25 Solved Past Papers Income Tax Numericals of ICMAP Stage IV - (2003 to 2015)

For the tax year 2016


Rs.
INCOME FROM BUSINESS U/S 18
Net profit before tax 700,000
Add: Depreciation on personal car 20,000
Partners drawings (Rs. 100,000 x 2) 200,000
Cash gift not accounted for as deemed income u/s 39 150,000
Unsupported payments 50,000
Penalties for non-observance of good environmental regulations 50,000
470,000
Taxable income 1,170,000

COMPUTATION OF TAX LIABILITY:

The tax liability shall be higher of tax on taxable income or Minimum tax under 153.
Tax on Rs. 1,170,000
[32,000 + 15% x (1,170,000 - 750,000)] (A) 95,000
Less proportionate tax on receipts covered under minimum
tax liability Rs. 95,000 / 1,170,000 x 1,070,000 (Related to (B) (86,880) 8,120
income from services) (Balance tax related to sale of scrap)

OR
Minimum tax liability u/s 153
Gross receipts 3,000,000
10% of gross receipts (C) 300,000
Hence higher of (B) or (C) is to be added under NTR, hence 300,000
308,120
Less: Tax withheld by corporate clients 300,000
Balance tax payable 8,120
Notes
1. It is assumed that tax depreciation and accounting depreciation are same.
2. Interest on loan and donation have not been accounted for as the same are liability for the hospital to construct
Special Cancer Ward and return the loan from friend. Assumed both were being received through cross
cheques otherwise the same shall be treated as income u/s 39.
3. As loss incurred by burning of surgical equipments and by theft was fully insured, hence no gain or loss has
been considered for the purpose of computation of taxable income.
4. There is no treatment of excess perquisites included in salaries, as this concept is no more applicable.
5. There is no impact of reveune to be increased in future due to new clients induction.
6. Sale of scrap has not been shown as income from other sources as already included in the net profit.
7. No trunover tax has been computed as the revenue of the AOP's turnover does not exceeds Rs.50 million.
Summer - 2003 Q. 4
Being a tax consultant you have been provided with the following information in respect of Mr. A.D Chughtai, a senior
manager of a local company for the period 1st July, 2002 to 30th April, 2003 (Tax year, 2004):
Rs.

Basic pay / wages 210,000


Houes rent 115,500
Medical allowance 4,800
Cost of living allowance 7,860
Utilities 31,500
Orderly / Servant allowance 30,000
Bonus / ex-gratia 70,000

530_______________________________ Conceptual Approach to Taxes


Chapter 25 Solved Past Papers Income Tax Numericals of ICMAP Stage IV - (2003 to 2015)

Company car 1300 cc


(Partly used for company's business)
Leave fare assistance 17,500
Employer's contribution to provident fund 21,786
Employer's contribution to pension fund 27,300

Income tax deducted u/s 149. 60,000


In addition to the above you have been provided with the following data:

i) Dividend income 17,500


(Withholding tax deducted Rs. 1,750, Zakat deducted Rs. 250)
ii) Profit on PLS Account 20,000
(Withholding tax deducted Rs. 2,000, Zakat deducted Rs. 1,520)
iii) Professional fee received 10,000
(Withholding tax deducted Rs. 500)
iv) School fee paid for two children 25,000
(Receipts show National Tax Number)
Legal expenses (consultant fee) 12,500
(Receipts show National Tax Number)
vi) There is no time scale for this position
Required: Work out the taxable income and tax liability of Mr. A. D. Chughtai for the Tax year, 2004

Solution
Mr. A.D. Chughtai
Computation of taxable income and tax liability
Tax year 2016
Rs. Rs.
INCOME FROM SALARY U/S 12
Basic pay / wages 210,000
House rent 115,500
Medical allowance 4,800
Less: exempt upto 10% of basic salary U/C 139 21,000 -
Cost of living allowance 7,860
Utilities 31,500
Orderly / Servant allowance 30,000
Bonus / ex-gratia 70,000
Company car 1300 cc (5% of assumed value of Rs. 1,200,000) U/R 5 60,000
Leave fare assitance 17,500
Employer contribution provident fund 21,786
Less: exempt upto lower of:
Rs. 100,000, OR
10% of (basic salary + dearness allowance) i.e 21,000 21,000 786
Employer's contribution to pension fund (assumed not maintained by employer) 27,300
570,446
INCOME FROM OTHER SOURCES U/S 39
Professional fee 10,000
Total income 580,446
Less Zakat paid on profit on PLS account and dividend receipts (Rs. 250 + 520) 1,770
Taxable income 578,676

COMPUTATION OF TAX LIABILITY:

As the salary income of the individual constitute more than 50% of the total income hence tax under salary slab
is computed as under.

Conceptual Approach to Taxes _________________________________531


Chapter 25 Solved Past Papers Income Tax Numericals of ICMAP Stage IV - (2003 to 2015)

Tax liability under NTR


Tax on Rs. 578,676 [2,000 + 5% x (578,676 - 500,000)] (A) 5,934
Less proportionate tax on professional services (B) 103
Rs.5,934 / 578,676 x 10,000 5,831
Mimimum tax u/s 153
Tax deductible (professional income services Rs.10,000 x 10%) (C) 1,000
Higher of (B) or (C) 1,000
6,831
Tax liability under FTR
Tax on dividend U/S 150 1,750
Tax on profit on PLS account U/S 151 2,000
3,750
Total tax liability 10,581
Less: Tax deducted at source
On dividend income 1,750
On profit on PLS account 2,000
Tax deducted on professional fee 1,000
Tax deducted u/s 149 60,000
64,750
Balance tax refundable (54,169)
Summer - 2003 Q. 5
Dr. A. A. Qureshi a medical practitioner, furnishes his Receipt and Payment Account for the period 1st July, 2002 to 30th
April, 2003 (Tax year, 2004).

Payments Rs. Receipts Rs.

Rent of clinic 24,000 Consultation fees 450,000


Household expenses 96,000 Visiting fees 100,000
Purchase of motor car 300,000 Remuneration from articles
Purchase of surgical equipments 40,000 published in magazines 12,000
Advance income tax 60,000 Rental income 72,000
Salary to assistant 36,000 Gifts from patients 30,000
Car running expenses 30,000
Property tax 12,000
Depreciation of motor car 80,000
Stationery 5,000
Utilities 25,000

Required:
Compute the income for the relative tax year and tax thereon after taking into account the following facts:

i) Two-third of car running expenses are in connection with personal use.


ii) Depreciation on car should be charged according to the rules.
iii) Investment in Defense Saving Certificates at Rs. 30,000.
Solution
Dr. A. A. Qureshi - Resident
Computation of taxable income and tax liability
For the tax year 2016
Rs. Rs.
INCOME FROM BUSINESS U/S 18
Consultation fees (Note - 1) 450,000
Less: Expenses
Rent of clinic 24,000
Household expenses (Not allowed being personal) -

532_______________________________ Conceptual Approach to Taxes


Chapter 25 Solved Past Papers Income Tax Numericals of ICMAP Stage IV - (2003 to 2015)

Purchase of motor car (Capital nature payment) -


Purchase of surgical equipments (Capital nature payment) -
Salary to assistant 36,000
Car running expenses (30,000 x 1 / 3 related to business) 10,000
Property tax 12,000
Depreciation:
(Motor car (300,000 x 15% x 1/3 business) U/S 22 15,000
Surgical equipments (initial 40,000 x 25%) U/S 23 10,000
Surgical equipments (normal 30,000 x 15%) U/S 22 4,500
Stationery 5,000
Utilities 25,000
141,500
Net profit from business 308,500
INCOME FROM PROPERTY U/S 15
Property income (assumed after allowable deductions) 72,000
INCOME FROM OTHER SOURCES U/S 39
Visiting fees 100,000
Remuneration from articles in magazines 12,000
Gifts from patients (Not chargeable to tax because of personal nature liability) -
112,000
Total taxable income including property income taxable under NTR as SBI 492,500

COMPUTATION OF TAX LIABILITY:


As the taxpayer is a non salaried person hence tax liability is computed as under:
Tax on Rs. 492,500 [0 + 7% x (492,500 - 400,000)] 6,475
Less: Advance income tax paid 60,000
Balance tax refundable (53,525)

Note-1 The question has been solved on the assumption that no tax on consultation fee has been deducted during the year

othewise the same shall constitute as minimum tax liability U/S 153 of the Income Tax Ordinance, 2001.

Note - 2 Investment in DSC's shall be accounted for in the personal wealth statement of the taxpayer.

Conceptual Approach to Taxes _________________________________533


Preliminary Chapter-01

Chapter

1 PRELIMINARY

(FOR CAF-6 AND ICMAP STUDENTS)


Covered under this chapter:

- Section 1of the Sales Tax Act, 1990


- All definitions u/s 2 of the Sales Tax Act, 1990
- Past papers theoretical questions of ICMAP & CA Mod C

Short title, extent and commencement [u/s 1]


This Act may be called the Sales Tax Act, 1990.
It extends to the whole of Pakistan.
It shall come into force on such date as the Federal Government may, by notification in the official Gazette, appoint.

Sales tax definitions [u/s 2]


The Sales Tax Act, 1990 u/s 2 defines meanings to certain terms and phrases are:
1. Active taxpayer [u/s 2(1)]
means a registered person who does not fall in any of the following categories, namely:-
(a) who is blacklisted or whose registration is suspended or is blocked in terms of section 21;
(b) who fails to file the return under section 26 by the due date for two consecutive periods;
(c) who fails to file an Income Tax return under section 114 or statement under section 115, of the
Income Tax Ordinance, 2001, by the due date; and
(d) who fails to file two consecutive monthly or an annual withholding tax statement under section 165
of the Income Tax Ordinance, 2001.
2. Appellate tribunal [u/s 2(1A)] (Same as given in Income Tax Chapter 1)
3. Appropriate officer [u/s 2(2)]
'appropriate officer' means an officer of Inland Revenue authorised by the Board by notification in the official Gazette
to perform certain functions under this Act;
4. Arrears [u/s 2(2A)]
"arrears means, on any day, the sales tax due and payable by the person before that day but which has not yet been
paid;
5. Associates (associated persons) [u/s 2(3)]
"associates (associated persons)" means, -
(i) where two persons associate and the relationship between the two is such that one may reasonably be
expected to act in accordance with the intentions of the other, or both persons may reasonably be expected to
act in accordance with the intentions of a third person;
(ii) two persons shall not be associates solely by reason of the fact that one person is an employee of the other or
both persons are employees of a third person;

Conceptual Approach to Taxes 535


Preliminary Chapter-01

(iii) the following shall be treated as associates, namely:


(a) "an individual and a relative of the individual;
(b) members of an association of persons;
(c) a member of an association of persons and the association, where the member, either alone or together
with an associate or associates, controls 50% or more of the rights to income or capital of the
association;
(d) a trust and any person who benefits or may benefit under the trust;
(e) a shareholder in a company and the company, where the shareholder, either alone or together with an
associate or associates, controls either directly or through one or more interposed persons-
50% or more of the voting power, rights to dividends or rights to capital; and
(f) two companies, where a person, either alone or together with an associate or associates, controls either
directly or through one or more interposed persons -
(g) having in both companies 50% or more of the voting power, rights to dividends or rights to capital.
(iv) two persons shall not be associates where the CIR is satisfied that neither person may reasonably be
expected to act in accordance with the intentions of the other.
(v) In this clause, "relative in relation to an individual, means-
(a) an ancestor, a descendant of any of the grandparents, or an adopted child, of the individual, or of a
spouse of the individual; or
(b) a spouse of the individual or of any person specified in sub-clause (a).
6. Association of persons [u/s 2(3A)] (Same as given in Income Tax Chapter 1)
7. Banking company [u/s 2(3AA)] (Same as given in Income Tax Chapter 1)
8. Board [u/s 2(4)] (Same as given in Income Tax Chapter 1)
9. Chief commissioner [u/s 2(4A)]
Chief Commissioner" means a person appointed as the chief Commissioner Inland Revenue u/s 30
10. Commissioner [u/s 2(5)]
'CIR' means the Commissioner of Inland Revenue appointed u/s 30;
11. Common taxpayer identification number [u/s 2(5A)]
'common taxpayer identification number means the registration number or any other number allocated to a registered
person.
12. Company [u/s 2(5AA)]
(Same as given in Income Tax Chapter 1) except that provincial government, local government and a small
company are not included in the definition of a company under the Sales Tax Act.
13. Cottage Industry [U/s 2(5AB)]
cottage industry" means a manufacturer whose annual turnover from taxable supplies made in any tax period during
the last twelve months ending any tax period does not exceed Rs.5,000,000 or whose annual utility (electricity, gas
and telephone) bills during the last twelve months ending any tax period do not exceed Rs.800,000;
14. CREST [u/s 2(5AC)]
means the computerized program for analyzing and cross-matching of sales tax returns, also referred to as
Computerized Risk-based Evaluation of Sales Tax;
15. Computerized system [u/s 2(5AAA)]
computerized system" means any comprehensive information technology system to be used by the Board or any
other office as may be notified by the Board, for carrying out the purposes of this Act;
16. Customs act [u/s 2(6)]
Customs Act' means the Customs Act, 1969 and where appropriate all rules and notifications made under that Act
17. Defaulter [u/s 2(6A)]
'defaulter' means a person and, every director, or partner of the company, of the firm, and includes guarantors or
successors, who fail to pay the arrears

536 Conceptual Approach to Taxes


Preliminary Chapter-01

18. Default surcharge [u/s 2(6B)]


default surcharge" means the default surcharge levied u/s 34 as below;
if a registered person does not pay the tax due or any part thereof, claims a tax credit, refund or makes an adjustment
which is not admissible to him, or incorrectly applies the rate of zero per cent to supplies made by him, he shall, in
addition to the tax due, he pay default surcharge.
19. Distributor [u/s 2(7)]
'distributor' means a person appointed by a manufacturer, importer or any other person for a specified area to
purchase goods from him for further supply and includes a person who in addition to being a distributor is also
engaged in supply of goods as a wholesaler or a retailer;
20. Document [u/s 2(8)]
'document' includes any electronic data, computer programmes, computer tapes, computer disks, micro-films or any
other medium for the storage of such data
21. Due date [u/s 2(9)]
'due date' in relation to the furnishing of a return u/s 26 and section 26AA means the 15th day of the month following
the end of the tax period, or such other date as the Board may specify;
22. E-intermediary [u/s 2(9A)]
"e-intermediary" means a person appointed as e- intermediary u/s 52A for filing of electronic returns and such other
documents as may be prescribed by the Board from time to time, on behalf of a person registered.
23. Establishment [u/s 2(10)]
'establishment' means an undertaking, firm or company, whether incorporated or not, an association of persons or an
individual;
24. Exempt supply [u/s 2(11)]
'exempt supply' means a supply which is exempt from tax u/s 13;
25. Firm [u/s 2(11A)] (Same as given in Income Tax Chapter 1)
26. Goods [u/s 2(12)]
'goods' include every kind of movable property other than actionable claims, money, stocks, shares and securities;
27. Importer [u/s 2(13)]
'importer' means any person who imports any goods into Pakistan;
28. Input tax [u/s 2(14)]
"input tax", in relation to a registered person, means:
(a) tax levied on supply of goods to the person;
(b) tax levied on the import of goods by the person;
(c) Federal Excise duty in sales tax mode on goods and services to the persons.
(d) Provincial sales tax levied on services rendered or provided to the person.
29. Kibor [u/s 2(14A)] (Same as given in Income Tax Chapter 1)
30. Local inland revenue offices [u/s 2(15)]
'Local Inland Revenue Office' means the office of Superintendent of Inland Revenue or such other office as the Board
may, by notification in the official Gazette, specify;
31. Manufacture or produce [u/s 2(16)]
'Manufacture' or 'produce' includes -
(a) any process in which an article singly or in combination with other articles, materials, components, is either
converted, changed, transformed or reshaped that it becomes capable of being put to use differently or
distinctly and includes any process necessary to the completion of a manufactured product;
(b) process of printing, publishing, lithography and engraving; and
(c) process and operations of assembling, mixing, cutting, diluting, bottling, packaging, repacking or preparation of
goods in any other manner;

Conceptual Approach to Taxes 537


Preliminary Chapter-01

32. Manufacturer or producer [u/s 2(17)]


'manufacturer' or 'producer' means a person who engages, in the production or manufacture of goods with or without
of the ownership of raw material used for such production; and shall include:
(a) a person who by any process or operation assembles, mixes, cuts, dilutes, bottles, packages, repackages or
prepares goods by any other manner;
(b) an assignee or trustee in bankruptcy, liquidator, executor, or curator or any manufacturer or producer and any
person who disposes of his assets in any fiduciary capacity; and
(c) any person, firm or company which owns, holds, claims or uses any patent, proprietary, or other right to goods
being manufactured, whether in his or its name, or on his or its behalf.
Only such person shall be treated as manufacturer-cum-exporter who owns or has his own manufacturing facility to
manufacture or produce the goods to be exported;
33. Officer of inland revenue [u/s 2(18)]
"Officer of Inland Revenue" means an officer appointed u/s 30;
34. Open market price [u/s 2(19)]
'open market price means the consideration in money which that supply or a similar supply would generally fetch in
an open market;
35. Output tax [u/s 2(20)]
'output tax", in relation to a registered person, means-
(a) tax levied under this Act on a supply of goods, made by the person;
(b) tax levied under the Federal Excise Act, 2005 in sales tax mode as a duty of excise on the manufacture or
production of the goods, or the rendering or providing of the services, by the person;
(c) Provincial sales tax levied on services rendered or provided by the person;
36. Person [u/s 2(21)] (Same as given in Income Tax Chapter 1)
37. Prescribed [u/s 2(22)]
'Prescribed' means prescribed by rules made under this Act;
38. Provisional sales tax [u/s 2(22A)]
means tax levied under provincial laws or laws relating to Islamabad Capital Territory,
39. Registered office [u/s 2(23)]
'registered office' means the office or other place of business specified by the registered person in the application
made by him for registration under this Act or through any subsequent application to the Commissioner Inland
Revenue.
40. Registration number [u/s 2(24)]
means the number allocated to the registered person for the purpose of this Act;
41. Registered person [u/s 2(25)]
'registered person means a person who is registered or is liable to be registered under this Act:
Provided that a person liable to be registered but not registered under this Act shall not be entitled to any benefit
available to a registered person under any of the provisions of this Act or the rules made there under;
42. Retail price [u/s 2(27)]
'retail price', with reference to the Third Schedule, means the price fixed by the manufacturer, inclusive of all duties,
charges and taxes (other than sales tax) at which any particular brand or variety of any article should be sold to the
general body of consumers or, if more than one such price is so fixed for the same brand or variety, the highest of
such price:
Provided that the Board may through a general order specify zones or areas for the purposes of determining highest
retail price for any brand or variety of goods.
43. Retailer [u/s 2(28)]
'retailer' means a person, supplying goods to general public for the purpose of consumption:
Provided that any person, who combines the business of import and retail or manufacture or production with retail,
shall notify and advertise wholesale prices and retail prices separately, and declare the address of retail outlets;

538 Conceptual Approach to Taxes


Preliminary Chapter-01

44. Return [u/s 2(29)]


'return' means any return required to be furnished under Chapter-V of this Act;
45. Sales tax [u/s 2(29A)]
"sates tax" means
(a) the tax, additional tax, or default surcharge levied under this Act;
(b) a fine, penalty or fee imposed or charged under this Act; and (c) any other sum payable under the provisions
of this Act or the rules made there under;
46. Sales tax account [u/s 2(29AA)]
sales tax account" means an account representing the double entry recording of sales tax transactions in the books of
account;
47. Schedule [u/s 2(30)]
'Schedule' means a Schedule appended to this Act;
48. Similar supply [u/s 2(31)]
'similar supply", in relation to the open market price of goods, means any other supply of goods which closely or
substantially resembles the characteristics, quantity, components and materials of the aforementioned goods;
49. Special audit [u/s 2(31A)]
'special audit' means an audit conducted u/s 32A;
50. Special judge [u/s 2(32)]
'Special Judge' means the Special Judge appointed u/s 37C of the Act and till such appointment is made the Special
Judge appointed u/s 185 of the Customs Act;
51. Supply [u/s 2(33)]
supply" means a sale or other transfer of the right to dispose of goods as owner, including such sale or transfer under
a hire purchase agreement, and also includes
a. putting to private, business or non-business use of goods produced or manufactured in the course of taxable
activity for purposes other than those of making a taxable supply;
b. auction or disposal of goods to satisfy a debt owed by a person;
c. possession of taxable goods held immediately before a person ceases to be a registered person; and
d. in case of manufacture of goods belonging to another person, the transfer or delivery of such goods to the
owner or to a person nominated by him:
Provided that the Federal Government, may by notification in the official Gazette, specify such other transactions
which shall or shall not constitute supply;
Supply chain [u/s 2(33A)]
means the series of transactions between buyers and sellers from the stage of first purchase or import to the stage of
final supply;
52. Tax [u/s 2(34)]
"tax", unless the context requires otherwise, means sales tax;
53. Taxable activity [u/s 2(35)]
Means any economic activity carried on by a person whether or not for profit, and includes:
(a) an activity carried on in the form of a business, trade or manufacture;
(b) an activity that involves the supply of goods, the rendering or providing of services, or both to another person;
(c) a one-off adventure or concern in the nature of a trade; and
(d) anything done or undertaken during the commencement or termination of the economic activity, but does not
include the activities:
(a) of an employee providing services in that capacity to an employer;
(b) carried on by an individual as a private recreational pursuit or hobby; and
(c) carried on by a person other than an individual which, if carried on by an individual, would fall in sub-clause (b)
above.

Conceptual Approach to Taxes 539


Preliminary Chapter-01

54. Tax fraction [u/s 2(36)]


'tax fraction' means the amount worked out in accordance with the following formula;-

a
100 a

('a' is the rate of tax specified in section 3);


55. Tax fraud [u/s 2(37)]
'tax fraud' means knowingly, dishonestly or fraudulently and without any lawful excuse (burden of proof of which
excuse shall be upon the accused)
(i) doing of any act or causing to do any act; or
(ii) omitting to take any action or causing the omission to take any action, including the making of taxable supplies
without getting registration under this Act; or
(iii) falsifying or causing falsification the sales tax invoices,
in contravention of duties or obligations imposed under this Act or rules or instructions issued there under with the
intention of understating the tax liability or underpaying the tax liability for two consecutive tax periods or overstating
the entitlement to tax credit or tax refund to cause loss of tax;
56. Taxable goods [u/s 2(39)]
'taxable goods' means all goods other than those which have been exempted u/s 13;
57. Tax invoice [u/s 2(40)]
'tax invoice' means a document required to be issued u/s 23;
58. Taxable supply [u/s 2(41)]
'taxable supply' means a supply of taxable goods made by an importer, manufacturer, wholesaler (including dealer),
distributor or retailer other than a supply of goods which is exempt u/s 13 and includes a supply of goods chargeable
to tax at the rate of zero per cent u/s 4;
59. Tax period [u/s 2(43)]
'tax period' means a period of one month or such other period as the Federal Government may, by notification in the
official Gazette, specify;
60. Time of supply [u/s 2(44)]
"time of supply," in relation to:
(a) a supply of goods, means time at which goods are delivered or made available to the recipient of the supply or
the time when any payment is received by the supplier in respect of that supply, whichever is earlier;
(b) a supply of goods under hire purchase agreement, means the time at which the agreement is entered into; and
(c) services, means the time at which the services are rendered or provided:

Provided that in respect of sub-clause (a), (b) or (c), where any part payment is received,

(i) for the supply in a tax period, it shall be accounted for in the return for that tax period; and
(ii) in respect of exempt supply, it shall be accounted for in the return for the tax period during which the
exemption is withdrawn from such supply;
61. Trust [u/s 2(44A)] (Same as given in Income Tax Chapter 1)
62. Unit trust [u/s 2(44AA)] (Same as given in Income Tax Chapter 1)
63. Value of supply [u/s 2(46)]
value of supply means:-
In respect of a taxable supply, the consideration in money including all Federal and Provincial duties and taxes, if any,
which the supplier receives from the recipient for that supply but excluding the amount of tax:

540 Conceptual Approach to Taxes


Preliminary Chapter-01

Nature of supply Rule for determination of value of supply


(a) Supply is in kind and partly in money, supply between Open market price less amount of tax
associated persons, supply on instalments and supply
other than taxable goods to registered person for
processing
(b) Supply on discounted price Discounted price less amount of tax
(c) Supply of special nature of transaction (where value to Open market price
determine is difficult)
(d) Supply of imported goods Value determined by Customs Authorities + duties +
Federal Excise Duty
(e) Supply has not been correctly declared in the invoice Value determined by the Valuation Committee
comprising trade and the Commissioner Inland
Revenue representatives
(f) Supply with reference to retail tax Price of taxable goods - amount of retail tax

The Board has the power to fix the value of any imported goods or taxable supplies. However, if the import or supply
is made at a value higher than the value fixed by the Board then the actual value shall be considered.
64. Whistleblower [u/s 2(46A)]
means whistleblower as defined in section 72D of the Sales Tax Act, 1990.
65. Wholesaler [u/s 2(47)]
'wholesaler' includes a dealer and means any person who carries on, whether regularly or otherwise, the business of
buying and selling goods by wholesale or of supplying or distributing goods, directly or indirectly, by wholesale for
cash or deferred payment or for commission or other valuable consideration or stores such goods belonging to others
as an agent for the purpose of sale; and includes a person supplying taxable goods to a person who deducts income
tax at source under the Income Tax Ordinance, 2001; and
66. Zero rated supply [U/s 2(48)]
'zero-rated supply' means a taxable supply which is charged to tax at the rate of zero percent u/s 4.
Difference between Exempt Supply and Zero-Rated Supply
Under Sales tax Act, a person is not required to pay tax on exempted and zero rated supplies, however both differ
with each other on the following points.
S. No. Point of Distinction Exempt Supply Zero-Rated Supply

1. Taxability Not taxable Taxable

2. Registration under Sales Tax Act Not required Required

3. Input tax credit Not allowed Allowed

4. Maintenance of records under the Sales Tax Act Not required Compulsory

5. Filing of return under Sales Tax Act. Not required Required

6. Invoicing requirements Invoices are raised No sales tax invoice shall be


raised

Conceptual Approach to Taxes 541


Preliminary Chapter-01

MULTIPLE CHOICE QUESTIONS


Q.1. __________ means an officer of Inland Revenue authorised by the Board by notification in the official Gazette to
perform certain functions under this Sales Tax Act.
(a) appropriate officer
(b) Commissioner Inland Revenue
(c) Chief Commissioner Inland Revenue
(d) deputy officer
Q.2. _______ means, on any day, the sales tax due and payable by the person before that day but which has not yet been
paid;
(a) surcharge
(b) arrear
(c) additional tax
(d) none of above
Q.3. Associates (associated persons) means:
(a) where two persons associate and the relationship between the two is such that one may reasonably be
expected to act in accordance with the intentions of the other, or both persons may reasonably be expected to
act in accordance with the intentions of a third person;
(b) two persons shall not be associates solely by reason of the fact that one person is an employee of the other or
both persons are employees of a third person;
(c) a trust and any person who benefits or may benefit under the trust;
(d) all of above
Q.4. ________ means the registration number or any other number allocated to a registered person.
(a) NTN
(b) CNIC
(c) common taxpayer identification number
(d) all of above
Q.5. The term company under the Sales Tax Act does not include ___________
(a) provincial government
(b) local government
(c) individual
(d) all of above
Q.6. cottage industry" means a manufacturer whose annual turnover from taxable supplies made in any tax period during
the last twelve months ending any tax period does not exceed _____.
(a) Rs. 5,000,000
(b) Rs. 6,000,000
(c) Rs. 1,000,000
(d) Rs. 100,000
Q.7. ______ means any comprehensive information technology system to be used by the Board or any other office as may
be notified by the Board, for carrying out the purposes of this Act.
(a) electronic tax register
(b) computerized system
(c) ERP
(d) SAP
Q.8. ______ means a person and, every director, or partner of the company, of the firm, and includes guarantors or
successors, who fail to pay the arrears.
(a) defaulter

542 Conceptual Approach to Taxes


Preliminary Chapter-01

(b) associates
(c) third party
(d) representative
Q.9. _________means a person appointed by a manufacturer, importer or any other person for a specified area to
purchase goods from him for further supply and includes a person who in addition is also engaged in supply of goods
as a wholesaler or a retailer;
(a) supplier
(b) vendor
(c) distributor
(d) creditor
Q.10. ______ includes any electronic data, computer programmes, computer tapes, computer disks, micro-films or any
other medium for the storage of such data
(a) voucher
(b) ledger
(c) report
(d) document
Q.11. 'Due date' in relation to the furnishing of a return u/s 26 and section 26AA means the ____ day of the month following
the end of the tax period, or such other date as the Board may specify;
(a) 15th day
(b) 20th day
(c) 10th day
(d) none of above
Q.12. ______ means a person appointed for filing of electronic returns and such other documents as may be prescribed by
the Board from time to time, on behalf of a registered person.
(a) tax house
(b) inspector
(c) e-intermediary
(d) Chief Commissioner Inland Revenue
Q.13. _____________ means an undertaking, firm or company, whether incorporated or not, an association of persons or
an individual;
(a) establishment
(b) organization
(c) corporation
(d) all of above
Q.14. 'goods' include every kind of _______ other than actionable claims, money, stocks, shares and securities.
(a) intangible items
(b) immovable property
(c) moveable property
(d) none of above
Q.15. _____ means any person who imports any goods into Pakistan;
(a) exporter
(b) importer
(c) trader
(d) none of above

Conceptual Approach to Taxes 543


Preliminary Chapter-01

Q.16. "input tax", in relation to a registered person, means:


(a) tax levied on supply of goods to the person;
(b) tax levied on the import of goods by the person;
(c) Federal Excise duty in sales tax mode on goods and services to the persons.
(d) Provincial sales tax levied on services rendered or provided to the person.
(e) all of above
Q.17. _________ means the office of Superintendent of Inland Revenue or such other office as the Board may, by
notification in the official Gazette, specify;
(a) establishment
(b) e-intermediary
(c) local body
(d) Local Inland Revenue Office
Q.18. 'Manufacture' or 'produce' includes -
(a) any process in which an article singly or in combination with other articles, materials, components, is either
converted, changed, transformed or reshaped that it becomes capable of being put to use differently or
distinctly and includes any process necessary to the completion of a manufactured product;
(b) process of printing, publishing, lithography and engraving;
(c) process and operations of assembling, mixing, cutting, diluting, bottling, packaging, repacking or preparation of
goods in any other manner;
(d) all of above
Q.19. ____ means the consideration in money which that supply or a similar supply would generally fetch;
(a) open market price
(b) fair price
(c) arms length price
(d) none of above
Q.20. 'output tax", in relation to a registered person, means-
(a) tax levied under this Act on a supply of goods, made by the person;
(b) tax levied under the Federal Excise Act, 2005 in sales tax mode as a duty of excise on the manufacture or
production of the goods, or the rendering or providing of the services, by the person;
(c) Provincial sates tax levied on services rendered or provided by the person; (
(d) All of above
Q.21. ____ means the office or other place of business specified by the registered person in the application made by him
for registration under this Act or through any subsequent application to the Commissioner Inland Revenue.
(a) head office
(b) registered office
(c) main office
(d) none of above
Q.22. 'registered person means a person who is ____________.
(a) registered
(b) liable to be registered
(c) person having NTN
(d) both a and b
Q.23. _________, with reference to the Third Schedule, means the price fixed by the manufacturer.
(a) retail price
(b) open market price
(c) fair price

544 Conceptual Approach to Taxes


Preliminary Chapter-01

(d) net price


Q.24. ____ means a person, supplying goods to general public for the purpose of consumption.
(a) manufacturer
(b) retailer
(c) defaulter
(d) none of above
Q.25. sales tax means
(a) the tax, additional tax, or default surcharge levied under this Act;
(b) a fine, penalty or fee imposed or charged under this Act;
(c) any other sum payable under the provisions of this Act or the rules made there under; (
(d) all of above
Q.26. ________ means an account representing the double entry recording of sales tax transactions in the books of
account;
(a) sales tax account
(b) sales account
(c) purchases tax account
(d) none of above
Q.27. ______, in relation to the open market price of goods, means any other supply of goods which closely or substantially
resembles the characteristics, quantity, components and materials of the aforementioned goods;
(a) open market supply
(b) similar supply
(c) taxable supply
(d) exempt supply
Q.28. supply" means a sale or other transfer of the right to dispose of goods as owner, including such sale or transfer
under a hire purchase agreement, and also includes
(a) putting to private, business or non-business use of goods produced or manufactured in the course of taxable
activity for purposes other than those of making a taxable supply;
(b) auction or disposal of goods to satisfy a debt owed by a person;
(c) possession of taxable goods held immediately before a person ceases to be a registered person:
(d) in case of manufacture of goods belonging to another person, the transfer or delivery of such goods to the
owner or to a person nominated by him:
(e) all of above
Q.29. 'tax fraud' means knowingly, dishonestly or fraudulently and without any lawful excuse (burden of proof of which
excuse shall be upon the accused)
(a) doing of any act or causing to do any act; or
(b) omitting to take any action or causing the omission to take any action, including the making of taxable supplies
without getting registration under this Act; or
(c) falsifying or causing falsification the sales tax invoices,
(d) all of above
Q.30. 'tax period' means a period of _____ or such other period as the Federal Government may, by notification in the
official Gazette, specify;
(a) one month
(b) two months
(c) three months
(d) twelve months

Conceptual Approach to Taxes 545


Preliminary Chapter-01

Q.31. In case of supply on discounted price, value of supply shall be _______.


(a) open market price less amount of tax
(b) open market price
(c) discounted price less amount of tax
(d) all of above
Q.32. ____________ means a taxable supply which is charged to tax at the rate 0%.
(a) zero-rated supply
(b) exempt supply
(c) nil supply
(d) none of above

ANSWERS
1 (a) 2 (b) 3 (d) 4 (c) 5 (d)
6 (a) 7 (b) 8 (a) 9 (c) 10 (d)
11 (a) 12 (c) 13 (a) 14 (c) 15 (b)
16 (e) 17 (d) 18 (d) 19 (a) 20 (d)
21 (b) 22 (d) 23 (a) 24 (b) 25 (d)
26 (a) 27 (b) 28 (e) 29 (d) 30 (a)
31 (c) 32 (a)

546 Conceptual Approach to Taxes


Preliminary Chapter-01

ICMAP PAST PAPERS THEORETICAL QUESTIONS


Q. No. 5 (a) Spring 2013 Define the following terms in the light of the Sales Tax Act, 1990:
(i) Cottage Industry
(ii) Output Tax
(iii) Time of Supply

Q. No. 5 (a) February 2013 Define the following terms under section (2) of the Sales Tax Act, 1990:
(i) Person
(ii) Input tax
(iii) Distributor
Q.4 (a) APRIL 2012 Define the following terms as given in the Sales Tax Act, 1990:
(i) Firm
(ii) Open market price

Q. NO. 4 (a) WINTER 2010 Define the following terms under the Sales Tax Act, 1990:
(i) Person
(ii) Cottage Industry

Q. NO. 6 (a) SUMMER 2010 Define the following under Sales Tax Act, 1990:
(i) Distributor
(ii) Documents

Q. NO. 7(a) Autumn 2008 Explain the concept of Value of Supply under the Sales Tax Act, 1990.
Q. NO. 5 (a) SUMMER 2008 Define the following terms with reference to the STA, 1990:
(i) Cottage industry
(ii) Manufacture or Produce

Q. NO. 7 (a) WINTER 2006 White short note on the following in terms of the STA, 1990.
(i) Arrears Section 2(2A)
(ii) E-Intermediary-Section 2(9A)
(iii) Retail price-Section 2(27)

Q. NO. 3 (b) SUMMER 2005 Define the following terms under the Sales Tax Act, 1990. 1- Tax fraud 2- Time of supply 3-
Retail price

Q. NO. 3 (a) SUMMER 2004 Define the term Associated Person under Sales Tax Act, 1990.

Conceptual Approach to Taxes 547


Preliminary Chapter-01

CA MOD C PAST PAPERS THEORETICAL QUESTIONS


Q. NO. 11 Spring 2014 Under the Sales Tax Act, 1990, Taxable activity means any economic activity carried on by
a person whether or not for profit. You are required to specify the activities that are specifically included and excluded
from the above definition.

Q.2 (a) Spring 2011 Samad Corporation (SC) supplies specialized material to various industrial concerns. The company has
entered into following transactions during the month of February 2011.
(i) Supply of material costing Rs. 3 million to AB Limited (ABL). It has been agreed that ABL would settle the
transaction by paying Rs. 1.5 million in cash and the balance amount by way of allowing SC to use ABLs import
quota. The market price of the supply is Rs. 3.5 million.
(ii) Supply of material to DM Limited (DML) at a discounted price of Rs. 6.8 million. Due to particular relationship, DML
has been allowed a special discount of 15% as against the normal business practice of 8%.
(iii) Supply of 20 tons of material, falling under third schedule, to BML at a wholesale price of Rs. 138,000 per ton. The
retail price of the material is Rs. 150,000 per ton.
Required: In each of the above situation, advise the management about the value of supply on which sales tax would be
levied.

Q. NO. 9(a) Spring 2005 Define the following with reference to the Sales Tax Act, 1990:-
(a) Associated person.
(b) Manufacturer or producer.
(c) Taxable supply.

Q. NO. 11 Autumn 2004 Define the following with reference to the Sales Tax Act, 1990?
(a) Distributor
(b) Input-tax
(c) Manufacture
(d) Similar supply.

Q. NO. 10 Autumn 2003 Define the following terms in the light of Sales Tax Act, 1990.
(a) Manufacture or produce.
(b) Supply
(c) Taxable activity

Q. NO. 10 Spring 2003 Explain the term Manufacture as used in the Sales Tax Act 1990.

548 Conceptual Approach to Taxes


Registration Chapter-02

Chapter

2 REGISTRATION

Section Rule Topic covered


For CAF-6 and ICMAP Students
14 Registration
Rules regarding registration & De registration
3 Application
4 Requirement of registration
5 Application for registration
5A Temporary registration
6 Compulsory registration
7 Change in the particulars of registration
8 Transfer of registration
9 Option to file application with Commissioner Inland Revenue
10 Cancellation of multiple registrations
11 De-registration
21 12 Blacklisting & suspension of registration
12A Non-active taxpayer
12B Restoration as an active taxpayer
MCQs with solutions
ICMAP & CA Mod C past papers theoretical questions
In this chapter
CRO stands for Central Registration Office LRO stands for Local Registration Office
RTO stands for Regional Tax Offices LTU stands for Large Taxpayers Unit
1. Registration [U/S 14]:
Registration will be required for such persons and be regulated in such manner and subject to rules prescribed by the
Board.
2. Rules regarding registration and de-registration:
2.1 Application [Rule 3]

The provisions of this Chapter shall apply to the following persons, namely:--
Rules of registration shall apply to the following persons, namely a person:

(a) required to be registered under the Act; (d) who is already registered and requires a change
in the name, address or other particulars of
registration;
(b) required for duty or tax under any other Federal (e) who is blacklisted or whose registration is
law or Provincial law; suspended; and
(c) who is subject to compulsory registration; (f) who is required to be de-registered.

2.2 Requirement of registration [Section 14]:


(1) Every person engaged in making taxable supplies in Pakistan, including zero-rated supplies, in the course or
furtherance of any taxable activity carried on by him, falling in any of the following categories, if not already
registered, is required to be registered under this Act, namely:-
(a) a manufacturer who is not running a cottage industry;

Conceptual Approach to Taxes 549


Registration Chapter-02

(b) a retailer who is liable to pay sales tax under the Act or rules made there under, excluding such retailer
required to pay sales tax through his electricity bill under sub-section (9) of section 3;
(c) an importer;
(d) an exporter who intends to obtain sales tax refund against his zero rated supplies;
(e) a wholesaler, dealer or distributor; and
(f) a person who is required, under any other Federal law or Provincial law, to be registered for the
purpose of any duty or tax collected or paid as if it were a levy of sales tax to be collected under the Act;
(2) Persons not engaged in making of taxable supplies in Pakistan, if required to be registered for making imports or
exports, or under any provisions of the Act, or any other Federal law, may apply for registration.
(3) The registration under this Act shall be regulated in such manner as the Board may, by notification in the official
Gazette, prescribe.
2.3 Application for registration [Rule 5]:
A person required to be registered under the Act shall, before making any taxable supplies, apply on the
computerized system through owner, authorized member or partner or authorized director, as the case may
be, in the Form STR-1, as annexed to these rules. Such application shall specify the RTO in whose jurisdiction
the registration is sought, as per criteria given below, namely in case of a:
(a) In case of listed or unlisted Public Ltd. The place where the registered office is located
Company:
(b) In case of other companies: (i) If the company is primarily engaged in manufacture
or processing, the place where the factory is
situated; and
(ii) If the company is primarily engaged in business
other than manufacture or processing, the place
where main business activities are actually carried
on;
(c) Person not incorporated: area where the business is actually carried on;
(d) Person not incorporated, having business where his manufacturing unit is located:
premises and manufacturing units at
different places:
(e) The Federal Board of Revenue may transfer where the place of business or registered office or
the registration of any registered person to a manufacturing unit is located.
jurisdiction:
(f) Jurisdiction of Large Taxpayers Unit: shall remain as specified by the Board:

SUBMISSION OF 1. CNIC of all owner, members partners or directors, as the case may be, and the
SCANNED COPIES OF representative (if any), and in case of non- resident, their passports;
THE DOCUMENTS
WITH THE
2. in case of a company or registered AOP, the Registration or Incorporation Certificate,
APPLICATION
along-with Form III or Form A as prescribed in the Companies Ordinance, 1984.
3. in case of a partnership, the partnership deed and Statement of Affairs;
4. bank account certificate issued by the bank, in the name of the business;
5. lease or rent agreement, if the premises is on rent, along-with CNIC of the owner of
the premises;
6. ownership documents of the premises, such as registered sale deed or registered
transfer deed;
7. latest utility bills (electricity, gas, land line telephone, and post paid mobile phones, as
the case may be); and
8. list of machinery installed, in case of manufacturer.
9. distribution certificate from the principal showing distributorship or dealership, in case of
distributor or dealer;
10. balance sheet/statement of affairs/equity of the business;

550 Conceptual Approach to Taxes


Registration Chapter-02

11. particulars of all branches in case of multiple branches at various locations; and
12. particulars of all franchise holders in case of national or international franchise.
PERSONA VISIT The applicant being the owner, authorized member or partner or authorized director, as
the case may be, shall visit the concerned RTO for biometric verification along with all those
documents Sales Tax Rules, 2006 specified above which have not been submitted through
computerized system.
SUBMISSION OF Subject to above documents the applicant shall also submit the following to the computerized
ELECTRONICALLY system through the electronic application prescribed by the Board for the purpose, namely: -
GPS TAGGED
PHOTOGRAPHS a) GPS-tagged photographs of the business premises, office equipment, electricity
meter and gas meter;
b) in case of manufacturer, in addition to clause (a), GPS -tagged photographs of
factory premises, machinery, industrial electricity or gas meter installed; and
c) in case of wholesaler, in addition to clause (a), GPS -tagged photographs of
the business premises and go-down.
Incomplete applications shall not be entertained by the computerized system.
CLAIM OF INPUT ON Where an applicant has unsold or unused stocks of tax-paid inputs on which he
UNUSED STOCK desires to claim the benefit of section 59 of the Act, he shall declare such stocks in a
statement in the Form set out as STR-4, to be appended with his application for registration.
ACCEPTANCE OR The application shall be processed by the computerized system and if found complete in
REJECTION OF all respects, shall be assigned a risk score. In case the application is found low risk,
APPLICATION FOR registration shall be issued by the computer system and certificate shall be sent to the
ISSUANCE OF applicant by courier service. The high risk cases shall, for further inquiry and scrutiny
REGISTRATION of documents, be sent to the Commissioner Inland Revenue, designated in the RTO
CERTIFICATE for the purpose.
Where a person, who has furnished a Form for registration, discovers any omission or
wrong statement therein, or notices a subsequent change in any information,
particulars, annexures, statements, documents or data already furnished, he may, without
prejudice to any liability incurred by him under any provision of the Act, furnish a revised
Form for registration.
VERIFICATION OF In case the person applying for registration as manufacturer is sharing the premises, he shall
MANUFACTURING provide evidence of
FACILITY
(a) demarcation of manufacturing premises for registration, and
(b) installation of sub-meter by the relevant utility company, in case he does not have
independent industrial utility connection but is using electricity or gas through sub-
meter.

3. Temporary registration [Rule 5A]:

Where a person files application for sales tax registration as a manufacturer without having installed
machinery, for the purpose of import of machinery to be installed by him, temporary registration as manufacturer
shall be allowed to him for a period of sixty (60) days subject to furnishing of the complete list of machinery to be
imported along with Bill of Lading (BL) or Goods Declaration (GDs) in lieu of applicable requirements prescribed in
rule 5.
The temporary registration shall be issued by the computerized system within seventy two (72) hours of filing of the
complete application.
After receiving temporary registration, the person shall be allowed to import plant, machinery and raw
materials, etc. as a manufacturer, subject to submission to the customs authorities of a post-dated cheque equal to
the difference in duties and taxes to be availed as a manufacturer.
In case the applicable requirements prescribed in rule 5 are not fulfilled within sixty (60) days of issuance of
the temporary registration, such temporary registration shall be disabled and the post-dated cheques submitted
shall be enchased.

A person holding temporary registration shall file monthly return in the form STR-7, but shall not issue a sales tax
invoice and if such invoice is issued, no input tax credit shall be admissible against such invoice.
No sales tax refund shall be paid to the person during the period of temporary registration and the
amount of input tax may be carried forward to his returns for subsequent tax.

Conceptual Approach to Taxes 551


Registration Chapter-02

4. Compulsory registration [Rule 6]:

if a person, who is required to be registered under the Act, does not apply for registration and the CIR or
any other officer, as may be authorized by the Board, after such inquiry as deemed appropriate, is
satisfied that such person is required to be registered, he shall issue notice to such person in the Form
set out in Form STR-6.
In case the CIR receives a written reply from the said person within the time specified in the above notice,
contesting his liability to be registered, the CIR shall grant such person opportunity of personal hearing, if
so desired by the person, and shall thereafter pass an order whether or not such person is liable to be
registered compulsorily. Copy of the said order shall invariably be provided to that person. Where the CIR passes
the order for compulsory registration, he shall cause the said person to be registered through computerized
system.
Where the person to whom a notice as given above, does not respond within the time specified in the
notice, the CIR shall cause to compulsorily register the said person through computerized system under
intimation to the said person through courier service.
A person registered compulsorily as above is required to comply with all the provisions of the Act and rules made
there under from the date of compulsory registration, and in case of failure to do so, the CIR having jurisdiction
may issue notice U/S 25 of the Act for production of records or documents and appearance in person to
assess the amount of sales tax payable U/S 11 of the Act, and take any other action as required under the law
against such person:

Provided that if it is subsequently established that a person was not liable to be registered but was wrongly
registered under .this rule due to inadvertence, error or misconstruction, the CIR shall cause to cancel his
registration through the computerized system. In case of such cancellation of registration, such person shall
not be liable to pay any tax, default surcharge or penalty under the Act or rules made there under, subject to the
conditions, limitations and restrictions prescribed U/S 3B of the Act.
5. Change in the Particulars of registration [Rule 7]:

In case there is a change in the name, address or other particulars as stated in the registration certificate,
the registered person shall notify the change in the Form STR-l to the computerized system , within fourteen
(14) days of such change.
The change of business category as 'manufacturer' shall be allowed subject to fulfilment of all applicable
requirements as specified in rule 5.

In case of approval of the change applied for, a revised registration certificate shall be issued through
computerized system, which shall be effective from the date the person applied for the change.
The CIR may, based on available information or particulars and after making such inquiry as he may deem
necessary and after providing reasonable opportunity of being heard to a person, by an order in writing, make
modifications in registration of the person.

6. Transfer of registration [Rule 8]

The Board may, in accordance with rule 5 or otherwise, by an order, transfer the registration of a
registered person from the jurisdiction of one LTU or RTO to another.
On transfer of registration,--

(a) all the records and responsibilities relating to such registered person shall be transferred to the
LTU or RTO, in whose jurisdiction the registration has been so transferred;
(b) notwithstanding the actions already taken, being taken or otherwise pending immediately before
the transfer in respect of such registered person under any of the provisions of the Act or the
rules made there under in the LTU or RTO from where his registration has been transferred, the
LTU or RTO, in whose jurisdiction the registration is so transferred shall exercise the jurisdiction
over such person in the manner as if it always had such jurisdiction.
In case of transfer of registration as above, the Board shall issue intimation letter to the registered person
along with copy to concerned LTU or RTO.
In case a registered person intends to shift his business activity from the jurisdiction of one LTU or RTO to
another, or he has any other valid reason for such transfer, he shall apply to the Board for transfer of his
registration along with Form STR-I. The Board shall follow the procedure as provided above.

552 Conceptual Approach to Taxes


Registration Chapter-02

7. Option to file application with Commissioner Inland Revenue [Rule 9]


A person who is unable to file application for registration or change in particulars of registration directly in
computerized system may submit the prescribed application and required documents to the concerned CIR RTO,
which shall ensure entry of the application and documents in computerized system within three (3) days.
8. Cancellation of multiple registrations [Rule 10]
In case a person holds multiple sales tax registrations, he shall retain only one registration and surrender
all other registrations under intimation to concerned CIR at RTO.
Provided that the Board may, subject to such conditions as it may deem appropriate, allow or allocate a
person separate registration for manufacturing units located in different LTU or RTO.
The tax liabilities against the registration cancelled as above shall be transferred against the registration retained
and in case of such registrations being in different LTU or RTO, the CIR having jurisdiction over cancelled
registrations shall ensure that tax arrears files are transferred to the LTU or RTO, having jurisdiction over the
registration so retained.
9. De-registration [Rule 11]
Every registered person who ceases to carry on his business or whose supplies become exempt from tax, or
who ceases to remain registered shall apply to the CIR having jurisdiction for cancellation of his registration in
Form STR-3, and the CIR, on such application or on its own initiative, may issue order of deregistration or
cancellation of the registration of such person from such date as may be specified, but not later than
ninety (90) days from the date of such application or the date all the dues outstanding against such person
are deposited by him, whichever is later and such person shall caused to be de registered through
computerized system accordingly.
The CIR, upon completion of any audit proceedings or inquiry which may have been initiated consequent
upon the application of the registered person for de-registration, shall complete the proceedings or inquiry
within ninety (90) days from the date of application and direct the applicant to discharge any outstanding liability
which may have been raised therein by filing a final return U/S 28:
Provided that the person applying for de-registration shall not be de-registered unless he provides record for the
purpose of audit or inquiry.
If a registered person fails to file tax return for six (6) consecutive months, the CIR, without prejudice to
any action that may be taken under any other provision of the Act, after issuing a notice in writing and
after giving an opportunity of being heard to such person, shall issue order of de-registration of such person
and the computerized system shall be caused to de-register the person accordingly.
The obligations and liabilities of the person whose registration is cancelled above relating to the period
when he conducted business as a registered person shall not be affected by the fact that his registration has
been cancelled or that he has ceased to be a registered person.
10. Blacklisting and suspension of registration [Rule 12]
Where the CIR or Board has reasons to believe that the registered person is to be suspended or blacklisted, in
order to ensure that the LTUs and RTOs follow a uniform policy for suspension and blacklisting of sales tax
registered persons U/S 21(2) of the Act and for subsequent proceedings in such cases, the following
procedure shall be followed, namely:-
(a) SUSPENSION
Where a CIR, having jurisdiction, is satisfied that a registered person has issued fake invoices, evaded
tax or committed tax fraud, registration of such person may be suspended by the CIR through the
system, without prior notice, pending further inquiry. The basis for such satisfaction may inter alia
include the following, namely:
(A) non-availability of the registered person at the given address;
(B) refusal to allow access to business premises or refusal to furnish records to an authorized
Inland Revenue Officer;
(C) abnormal tax profile, such as taking excessive input tax adjustments, continuous carry-
forwards, or sudden increase in turnover;
(D) making substantial purchases from or making supplies to other blacklisted or suspended
person;
(E) non-filing of sales tax returns;
(F) on recommendation of a commissioner of any other jurisdiction;

Conceptual Approach to Taxes 553


Registration Chapter-02

(G) any other reason to be specified by the CIR;


the suspension of registration shall take place through a written order of the CIR concerned,
giving reasons for suspension. This order shall be endorsed to the registered person concerned,
all other LTUs/RTOs, the FBRs computer system, the STARR computer system and the Customs
Wing computer system for information and necessary action as per law;
a registered person who does not file sales tax return for six consecutive months shall be caused to be
suspended through the system without any notice;
in cases, where the buyers and suppliers of any such person, whose registration is being suspended,
belongs to another LTU/ RTO, and these buyers / suppliers are also required to be suspended, the
CIR shall intimate the Chief Commissioner of the concerned LTU/RTO in whose jurisdiction such
buyers/suppliers fall, in writing explaining the complete facts of the case and the reasons on the basis
of which these buyers/suppliers are to be suspended, to initiate proceedings for
suspension/blacklisting of the buyers/suppliers;
no input tax adjustment/refund shall be admissible to the registered person during the currency of
suspension. Similarly, no input tax adjustment/refund shall be allowed to any other registered
persons on the strength of invoices issued by such suspended person (whether issued prior to or after
such suspension), during the currency of suspension;
the CIR shall, within seven (7) days of issuance of order of suspension, issue a show cause
notice (through registered post or courier service) to the registered person to afford an
opportunity of hearing with fifteen (15) days of the issuance of such notice clearly indicating that he
will be blacklisted, in case
(A) there is no response to the notice;
(B) he has not provided the required record;
(C) he has not allowed access to his business record or premises; and
(D) any other reason specified by the CIR;
in case show cause notice is not issued within seven days of the order of suspension, the order
of suspension shall become void ab-initio;
in case of non-availability of the suspended person at the given address, the notice may be
affixed on the main notice Board of the LTU/RTO;
on receipt of the reply to the notice and after giving an opportunity of hearing to the registered person, if
the CIR is satisfied, he may order for revoking of suspension of the registered person;
(b) BLACKLISTING
in case, after giving an opportunity of hearing, the offence is confirmed, the CIR shall issue an
appealable self-speaking order for blacklisting of the registered person, and shall proceed to take
legal and penal action under the relevant provisions of the Act;
the order of blacklisting shall contain the reasons for blacklisting, the time period for which any refund
or input tax claimed by such person or by any other registered person on the strength of invoices
issued by him from the date of his registration shall be inadmissible, any recovery to be paid or
penalties to be imposed;
the order of blacklisting shall be issued within ninety days of the issuance of the notice of hearing. In
case, the order of blacklisting is not issued within this time period the suspension of registered person
shall become void ab-initio;
copies of the order shall be endorsed to the registered person concerned, all other LTUs/RTOs, the
FBR/PRAL computer system, the STARR computer system and the Customs Wing computer system.
Each LTU/RTO shall circulate all such lists to their refund sections, audit sections and other concerned
staff to ensure that the order is implemented in letter and spirit by all concerned;
all LTUs / RTOs shall further circulate the copies of the order along with a computer system-
generated list of invoices issued by the blacklisted persons as referred to in the preceding
clause, to all officers of Inland Revenue having jurisdiction over the registered persons who
have claimed credit of input tax or refund on the strength of the invoices issued by the said
blacklisted persons; and
the officer of Inland Revenue receiving the aforesaid list shall issue show-cause notice U/S 11
and section 21(3) of the Act to a registered person for rejecting the input tax or refund claimed against
the invoices so circulated and further proceed to decide the matter as per law through a self-speaking
appealable order and after affording a reasonable opportunity of being heard to such person, in the
manner as provided above.
554 Conceptual Approach to Taxes
Registration Chapter-02

11. Non-active taxpayer [Rule 12A]


A registered person who does not fulfil any of the conditions prescribed in section 2(1) of the Act shall
automatically become a non -active taxpayer and his name shall be removed from the active taxpayers list
maintained by the Board.
A non-active taxpayer shall not be entitled to
(a) file Goods Declarations for import or export;
(b) issue sales tax invoices;
(c) claim input tax or refund; or
(d) avail any concession under the Act or rules made there under.
No person, including government departments, autonomous bodies and public sector organizations, shall make any
purchases from a non-active taxpayer.
In case of entry of an invoice issued by a non-active taxpayer by any registered buyer in Annexure-A of his return, a
message shall appear to the effect that the supplier is a non-active taxpayer and no input tax credit shall be
admissible against such invoice.
12. Restoration as an active taxpayer [Rule 12B]
A non-active taxpayer may be restored as active taxpayer, if
(a) the registered person files the return or statement along with payment of any tax due under the Act or
Income Tax Ordinance, 2001;
(b) the RTO or LTU having jurisdiction, on satisfying itself after conducting such audit or other
investigation as may be necessary, recommends to the Board for restoration; and
(c) the Board issues an order to such effect.

Conceptual Approach to Taxes 555


Registration Chapter-02

MULTIPLE CHOICE QUESTIONS


Q.1 Registration through application is one in which a person who is not liable to be registered under the Sales Tax Act,
submits ________________.
(a) Sales tax return
(b) application for registration
(c) income tax return
(d) both a and b
Q.2 Compulsory registration is made without the application by the __________.
(a) Concerned person
(b) tax department
(c) supplier of the concerned person
(d) all of the above
Q.3 A commercial exporter who intends to obtain sales tax refund against the ______ is liable to be registered under the
Sales Tax Act.
(a) Exempt supplies
(b) taxable supplies
(c) zero rated supplies
(d) all of the above
Q.4 A person liable to be registered shall make an application to the ___ after making taxable supplies.
(a) Local Registration Office
(b) Computerization registration
(c) Regional Tax Office
(d) Pakistan Revenue Automation Limited
Q.5 A corporate person has an option to apply for the transfer of registration to the ___ having jurisdiction where the place
of business is located instead of his registered office.
(a) Local Registration Office
(b) CRO
(c) Regional Tax Office
(d) Pakistan Revenue Automation Limited
Q.6 Where a person holds such a stock in respect of which the input tax was paid prior to his registration then he should
also provide the detail of _______.
(a) Unsold or unused stock
(b) opening stock
(c) purchases made
(d) sale of such stock
Q.7 The verification of application for registration may be conducted through _____.
(a) Local Registration Office
(b) CRO
(c) Regional Tax Office
(d) Commissioner Inland Revenue
Q.8 The application for the registration is made only through electronic means provided by the ____.
(a) Local Registration Office
(b) CRO
(c) Regional Tax Office
(d) Board

556 Conceptual Approach to Taxes


Registration Chapter-02

Q.9 In case of approval of change in the registration from the ____ shall issue a revised registration certificate which will
be effective from the date of approval of change.
(a) Local Registration Office
(b) CRO
(c) Regional Tax Office
(d) Board
Q.10 The CIR is empowered to transfer the registration of a registered person from ____________.
(a) One collector to another
(b) Large Taxpayer Unit to Regional Tax Office
(c) both a or b
(d) None of the above
Q.11 On transfer of registration, it is not compulsory to transfer all the record and liabilities of the registered person to
the______ or Large Taxpayer Unit or regional tax office.
(a) Collector
(b) Large Taxpayer Unit
(c) Regional Tax Office
(d) all of the above
Q.12 A registered person may apply to Local Registration Office for de-registration / cancellation of his sales tax
registration _________________;
(a) on ceasing to carry on his business
(b) where supplies become exempt from tax
(c) where total taxable turnover during the last twelve months remains below the limit of Rs.5 million
(d) all of the above
Q.13 On application for de-registration or the Local Registration Office may recommend to the CRO to cancel the
registration of such person from such date as may be specified, but not later than 90 days _______.
(a) from the date of such application
(b) from the date all the dues outstanding against such person are deposited by him.
(c) from the date of such application or the date all the dues outstanding against such person are deposited by
him, whichever is the later.
(d) none of the above
Q.14 If a registered person fails to file tax return for six consecutive months, ____________________ recommend to the
central registration office for cancellation of the registration after satisfying that no tax liability is outstanding against
such person
(a) the Local Registration Office may, after issuing a notice in writing
(b) without giving an opportunity of being heard to such person
(c) the Local Registration Office may, after issuing a notice in writing and after giving an opportunity of being
heard to such person
(d) all of the above
Q.15 After the blacklisting of a registered person, the invoices issued ______________by such person shall not be
entertained for the purposes of sales tax refund or input tax credit.
(a) after blacklisting
(b) before blacklisting
(c) after and before blacklisting
(d) none of the above

Conceptual Approach to Taxes 557


Registration Chapter-02

ANSWERS

1 (b) 2 (a) 3 (c) 4 (b) 5 (a)

6 (a) 7 (a) 8 (d) 9 (b) 10 (c)

11 (a) 12 (d) 13 (c) 14 (c) 15 (c)

558 Conceptual Approach to Taxes


Registration Chapter-02

ICMAP PAST PAPERS THEORETICAL QUESTIONS


Q. NO. 5 (b) SUMMER 2008 What are the rules for de-registration of a registered person under the Sales Tax Law?
Q. NO. 7 (b) SUMMER 2006 Explain the procedure for De-registration under the Sales Tax Act, 1990.

Conceptual Approach to Taxes 559


Registration Chapter-02

CA MOD C PAST PAPERS THEORETICAL QUESTIONS


Q.NO.5 (a) Autumn 2014 Under the Sales Tax Act, 1990 and Rules made thereunder:
(i) List the persons who are required to be registered.

Q.8 Spring 2013 Under the Sales Tax Rules, 2006


(a) Identify the situations in which a registered person is liable to be deregistered.
(b) State the steps involved in case of de-registration of a person on his own initiative.

Q.7 (a) Autumn 2011 Under what circumstances, a registered person becomes liable to be de-registered under Sales Tax
Act, 1990. Also state the procedures for deregistration as enumerated in the Sales Tax Rules, 2006.
Q. NO. 7(a) Autumn 2009 Identify the situations under which a person registered under the Sales Tax Act, 1990 is liable to
be de-registered.
Q. NO. 7(b) Autumn 2009 Briefly explain procedure for de-registration as specified by the Sales Tax Rules, 2006.

Q. NO. 7(c) Autumn 2009 Comment whether the following persons are required to be registered under the Sales Tax Act,
1990.
(i) Mr. Yahya is a wholesaler and his annual business turnover is Rs. 4.9 million.
(ii) Mr. Fazal is operating a general store and his monthly average turnover is Rs. 0.4 million.
(iii) Mr. Ishaq is planning to import raw materials for business use. The annual imports are estimated at Rs. 3.0 million.
(iv) Mr. Pervez is a commercial exporter. All his business purchases are either exempt supplies or from unregistered
suppliers.
(v) Mr. Farooq is a distributor of consumer goods and his annual turnover is Rs. 15 million.
(vi) Mr. Rafiq is a manufacturer of candles. His turnover in last twelve tax periods was below Rs. 5 million. His utility bills
during the same period were Rs. 550,000.
Q. NO. 6(b) Spring 2008 Narrate the circumstances under which, it becomes obligatory for a person to get registered under
the STA, 1990.

Q. NO. 6(c) Spring 2008 Under what circumstances can a person registered under the Sales Tax Act, 1990, be de-
registered?
Q. NO. 7(b) Spring 2007 Registered persons under the sales tax rules 2006 (b) List down the persons who are required to
be registered under the Sales Tax Rules, 2006.
Q. NO. 8(c) Autumn 2007 What are the circumstances in which a person registered under the Sales Tax Act, 1990 may
apply for cancellation of his registration?
Q. NO. 12 Spring 2005 State under what circumstances the Collector of Sales-tax is empowered to black list a registered
person?

Q. NO. 11(a) Autumn 2005 Identify the persons required to be registered under the Sales Tax Act 1990.

Q. NO. 11(b) Autumn 2005 Under what circumstances is a registered person liable to be de-registered? Also explain the
procedure for de-registration.

Q. NO. 11(c) Autumn 2005 Explain the provisions relating to black listing and suspension of registration.

560 Conceptual Approach to Taxes


Sales Tax Returns Chapter-03

Chapter

3 SALES TAX RETURNS

Sr. Section Rule Topic covered


For CAF- 6 AND ICMAP Students
1. 14 Filing of Returns
2. 15 Receipt of return by the Bank
3. 16 Payment of service charges to the Bank
4. 17 Filing of annual sales tax return
5. 18 Electronic filing of return
6. 26 13 Return
Sales tax return
Revision of return
27 Special returns
28 Final return
7. 29 Return deemed to have been made
MCQs with solutions
ICAP Mod C past papers theoretical questions

In this chapter
CIR stands for Commissioner Inland Revenue
RP stands for Registered Person

Returns means any return required to be furnished under Sales tax Act.
1. RETURN
All registered persons required to file a return u/s 26 of the Safes Tax Act, 1990 according to the following rules:
2. Filing of Returns [Rule 14]
Every RP shall file the return, along with all its annexures, in accordance with the instructions and manner as
specified in rule 18 covered as under. Where a RP operates in different sectors for which different dates of filing of
return have been prescribed, such person shall file a single return for all sectors by such date applicable to his major
activity.
3. Receipt of return by the Bank [Rule 15]
The Bank official shall ensure that the particulars entered in all the three copies of the return are identical and that the
amount deposited by the RP tallies with the amount indicated as "TOTAL SALES TAX PAYABLE" in the return, and
shall thereafter sign and stamp the return indicating the date of payment of tax and submission of tax return.
The Bank shall forward the original copy of the return to the concerned department of Sales Tax and second copy
with the computer generated receipt shall be delivered to the RP and third copy shall be retained by the Bank for
record purposes.
In case of payment through cheque, pay order or bank draft, and shall issue a provisional acknowledgement receipt.
In cases where the payments are received through pay order or bank draft, the bank shall affix two stamps on the
return indicating the date on which the cross payment instrument was received and the date on which the instrument
was cleared for payment by transfer. In case the banking instrument is not cleared such person shall be liable to pay
default surcharge and penalties for late payment of sales tax.

Conceptual Approach to Taxes 561


Sales Tax Returns Chapter-03

4. Payment of service charges to the Bank [Rule 16]


The Bank shall charge Rs.10 per return (including nil returns) as service charges from the Collectorates having
jurisdiction in the area where the bank branches are located.
5. Filing of annual sales tax return [Rule 17]:
Every RP, being a private or public limited company, shall file annual sales tax return, for a financial year by the 30th
September of the following financial year, with the Collector having Jurisdiction,
6. Electronic filing of return [Rule 18]:
Every RP required to file sales tax return or special return or other statement shall file such a return or statement
electronically in the manner as specified by the Board.
A RP filing returns electronically, shall make payment of the amount of sales tax due, if any, in any of the designated
branches of the National Bank of Pakistan on the prescribed payment challan or through electronic payment system
devised for this purpose.

Monthly ST return has to be filed by a registered person on the due date. The term due date is defined in section 2(9)
of the Act in the following manner:
"due date", in relation to the furnishing of a return u/s 26 and section 26AA means the 15th day of the month
following the end of the tax period, or such other date as the Board may, by notification in the official Gazette,
specify;
Therefore, the taxpayers are required to file the monthly ST return by 15th of the month following the end of the tax
period. Rule 18(3) of the ST Rules, 2006 states that in case the tax is deposited on the due date, then the return can
be filed up till 18th of next month.
7. Return [U/S 26 and Rule 13]:
7.1 Sales tax return:
Every RP shall furnish a true and correct return in the prescribed form to a designated bank or any other office
specified by the Board, indicating the purchases and the supplies made during a tax period, the tax due and
paid and such other information, as may be prescribed:
The Board may require any person or class of persons to submit return on quarterly basis, annually basis in
addition to monthly return;

The return filed electronically on the web or any magnetic media or any other computer readable media shall
also be deemed to be a return, and the Board may make rules for determining eligibility of the data of such
returns and e-intermediaries who will digitize the data of such returns and transmit the same electronically
under their digital signatures.

If there is a change in the rate of tax during a tax period, a separate return in respect of each portion of tax
period showing the application of different rates of tax shall be furnished.
A RP may, subject to approval of the CIR having jurisdiction, file a revised return within 120 days of the filing of
return as above or special return under section 27, to correct any omission or wrong declaration made therein.
7.2 Revision of return:
A RP may, subject to approval of the concerned CIR, file a revised return within 120 days of the filing of return
to correct any omission or wrong declaration.
If a RP wishes to file revised return voluntarily along with deposit of the amount of tax short paid or amount of
tax evaded along with default surcharge, whenever it comes to his notice, before receipt of notice of audit,
no penalty shall be recovered from him:

If a RP wishes to deposit the amount of tax as pointed out by the officer of Inland Revenue during the
audit, or at any time before issuance of the show cause notice, he may deposit the evaded amount of tax,
default surcharge and 25% of the penalty payable under section 33 along with the revised return:
If a RP wishes to deposit the amount after issuance of show cause notice, he shall deposit the evaded
amount of sales tax, default surcharge and full amount of the penalty payable under section 33 along with
the revised return and thereafter, the show cause notice, shall stand abated.
8. Special returns [U/S 27]:
In addition to the return specified u/s 26"

562 Conceptual Approach to Taxes


Sales Tax Returns Chapter-03

a RP shall furnish special return within such date indicating information such as quantity manufactured or
produced, purchases made, goods supplied or payment of arrears made, etc, for such period as the Board
may specify; and
the CIR may require any person whether, registered or not, to furnish a return on his behalf or as a
representative.
9. Final return [U/S 28]:
If a person applies for de-registration, he shall before such de-registration, furnish a final return to the CIR in the
specified form in such manner and at such time as directed by the CIR.
10. Return deemed to have been made [U/S 29]:
A return to be made on behalf of a person by his duly appointed representative shall, for all purposes, be deemed to
have been made by such person or under his authority unless proved to the contrary.
11. Summary of returns and their filing dates is given hereunder:

SR. FILER UNDER N ATURE OF SALES DUE DATE


SECTION / TAX RETURN
UNDER RULE
1(a) REGISTERED PERSON U/S 26 & 26AA Monthly 15th of next month following the tax period
WHERE NO T AX (including
PAYMENT IS INVOLVED production
monthly return,
[ OTHERTHAN ( B) TO ( D)
where applicable)
AS UNDER]

(b) REGISTERED PERSON U/R 18(3) Monthly 18th of next month following the tax period
WHERE ST DEPOSITED (including
ON DUE D ATE production
monthly return,
[ OTHER THAN ( A) ABOVE
where applicable)
& ( C) & ( D) AS UNDER
(c) PETROLEUM U/S 26 Monthly 25th of next month following the tax period
EXPLORATION & [Read with (including
PRODUCTION SRO production
COMPANIES 88(I)/2002 monthly return,
dated 11/02- where applicable)
2002]
(d) M ANUF ACTURERS OF U/R 58I of Monthly 28th of next month following the tax period
MILD STEEL , STEEL special (including
MELTERS, RE- ROLLERS procedure production
& SHIP BREAKERS Rules, 2007 monthly return,
where applicable)
2. REGISTERED PERSON U/S 26 Change in rate of Separate returns for each applicable tax
[Read with tax rate in the next month following the tax
Rule 13] period
3. CNG STATIONS Special Quarterly 15th of month next following each quarter
Procedure
Rules, 2007
4. REGISTERED OR U/S 27 Special On the date specified by the CIR its notice
UNREGISTERED PERSON calling for such return.
5. PERSON APPLIED FOR U/S 28 Final On the date specified by the CIR
DE- REGISTRATION

6. COMPANY (P RIVATE OR U/R 17 Annual 30th of September following the year end
PUBLIC LTD .)

Conceptual Approach to Taxes 563


Sales Tax Returns Chapter-03

MULTIPLE CHOICE QUESTIONS


Q.1. Normally sales tax return is filed by due date on ____________basis.
(a) Monthly
(b) Quarterly
(c) Semi-annually
(d) Annually
Q.2. The bank shall charge as service charges from the collectorate Rs___________:
(a) 20
(b) 10
(c) 30
(d) 15 per return
Q.3. The revised sales tax return can be filed within________________ of filing of original return, subject to approval of
Commissioner Inland Revenue having jurisdiction.
(a) 120 days
(b) 60 days
(c) 30 days
(d) None of the above
Q.4. Every registered person, being a private or public limited company, shall file annual sales tax return, for a financial
year by the ______________of the following financial year.
(a) 30th September
(b) 15th October
(c) 31st December
(d) All of above
Q.5. Where a registered person operates in different sectors for which different dates of filing of return have been
prescribed, such person shall file a single return for all sectors by such date applicable to _____.
(a) individuals
(b) companies
(c) his major activity
(d) none of above
Q.6. In case of non-furnishing of information required by Board, the penalty shall be of Rs____________.
(a) 25,000
(b) 10,000
(c) 5,000
(d) None of above
Q.7 A person who is registered under the sales tax Act is required to file a ______ with the sales tax authorities.
(a) Sales tax return
(b) Income tax return
(c) detail of sales
(d) detail of purchases
Q.8 In case of no activity during the tax period, registered person shall_____________.
(a) not file return
(b) file null return
(c) file nil business return
(d) file an application for deregistration

564 Conceptual Approach to Taxes


Sales Tax Returns Chapter-03

Q.9 In case of any change in the rate of tax during a tax period,________________.
(a) changed rate shall be applicable on period before such change
(b) changed rate shall have no effect on period after such change
(c) one return before change and one return after change shall be filed
(d) None of above
Q.10 A person filing sales tax return shall deposit the amount of sales tax with __________.
(a) Income tax department
(b) designated bank
(c) Central Registration Office
(d) Local Registration Office
Q.11 Every registered person is required to file the sales tax returns in ________on E-portal.
(a) triplicate
(b) duplicate
(c) singly
(d) none of the above
Q.12 The sales tax returns are to be filed in the designated branches of _________________.
(a) State Bank of Pakistan
(b) National bank of Pakistan
(c) Silk bank
(d) None of above
Q.13 Every private company is also required to file ____________ return in addition to monthly sales tax returns.
(a) annual
(b) quarterly
(c) both a or b
(d) All of the above
Q.14 The electronic filing of sales tax returns is required from __________.
(a) companies
(b) AOPs
(c) individuals
(d) All of the above
Q.15 The Board has an authority to require ________ to furnish such summary or detail as may be notified by it.
(a) companies
(b) AOPs
(c) individuals
(d) any person
Q.16 A retail sales tax return filed by a retailer uptill ______________.
(a) 15th of the same month
(b) 18th of the next month
(c) 10th of the same month
(d) 20 of the next month
Q.17 A ________ is in lieu of monthly sales tax return or retail tax return.
(a) statement of sales
(b) detail of output tax

Conceptual Approach to Taxes 565


Sales Tax Returns Chapter-03

(c) declaration of sales tax


(d) special return
Q.18 A person applying for de-registration shall furnish a _________.
(a) special return
(b) final return
(c) nil return
(d) all of the above
Q.19 A sales tax return filed by the _________shall deemed as return filed by the taxpayer himself.
(a) authorized representative
(b) any person
(c) both a and b
(d) none of the above
Q.20 Nil return is filed by a person where there is no _____________during the tax period.
(a) sales
(b) purchases
(c) both a or b
(d) sales tax payable

MULTIPLE CHOICE ANSWERS


1 (a) 2 (b) 3 (a) 4 (a) 5 (c)

6 (b) 7 (a) 8 (b) 9 (c) 10 (b)

11 (a) 12 (b) 13 (a) 14 (d) 15 (d)

16 (b) 17 (d) 18 (b) 19 (a) 20 (d)

566 Conceptual Approach to Taxes


Sales Tax Returns Chapter-03

ICMAP PAST PAPERS THEORETICAL QUESTIONS


Q.NO. 5 (b) March 2015 XYZ Industries (pvt.) Limited has been established for many years and registered under the Sales
Tax Act, 1990. The Chief Financial Officer (CFO) of the company, Mr. Akram observed that there is often delay and
discrepancies in filing of sales tax return due to inexperienced staff regarding the sales tax matters. He requested the
Sales Tax Advisor of the company to train the staff of the sales tax department.
Required:
Being the Sales Tax Advisor, guide the staff regarding the following queries in the light of the Sales Tax Act, 1990:
(iii) Is there any provision in the Sales Tax Act, 1990 for a registered person to file a revised sales tax return?
Elaborate in the light of section 26(3) of the Sales Tax Act, 1990.

Conceptual Approach to Taxes 567


Sales Tax Returns Chapter-03

MODULE C THEORETICAL PAST PAPERS QUESTIONS


Q.NO.5 (a) Autumn 2014 Under the Sales Tax Act, 1990 and Rules made thereunder:
(ii) Change in rate of tax during a tax period.

Q. NO. 10(b) Spring 2014 Under the provisions of Sales Tax Act, 1990:
Discuss the conditions under which a registered person may file a revised return.
Q. No. 8 (b) Autumn 2013 Discuss the rules relating to filing of electronic return under the Sales Tax Rules, 2006.

Q. No. 8 (a) Autumn 2012 Under the provisions of Sales Tax Act, 1990 and Rules made there under, identify the last date
for filing of sales tax return in each of the following cases:
(i) One 1 August 2012, Sara registered herself under the Sales Tax Act, 1990.
(ii) Fatima filed the return for the month of July 2012 on 10 August 2012. She wants to revise her return to correct certain
errors.
(iii) Amna Engineering Limited (AEL) is registered under the Sales Tax Act, 1990. AEL wants to file annual sales tax
return for the financial year ended 30 June 2012.
(iv) Abida wants to deregister herself with effect from 30 September 2012.

Q.NO. 7(a) Spring 2009 Discuss the provisions of the Sales Tax Act, 1990 with regard to the following:
(i) Filing of return
(ii) Voluntary revision of return
(iii) Revision of return during or after issuance of a notice of audit.

Q. NO. 10 Spring 2006 Explain the provisions of section 26 of Sales Tax Act, 1990 with regard to the following:
(a) Change in rate of tax during a tax period
(b) Voluntary revision of return
(c) Revision of return during audit or after issuance of a show cause notice by the department.

Q. NO. 11 Spring 2006 Which date shall be considered as the date of payment of sales tax in the following cases:
(a) Payment through cash or cheque
(b) Payment through pay order or bank draft

568 Conceptual Approach to Taxes


Records and Book Chapter-04

Chapter

4 RECORDS AND BOOKS

Sr. Section Topic covered


For CAF-6 AND ICMAP Students
1. 22 Records
2. 23 Tax invoices
3. 24 Retention of record & documents for 6 years
4. 25 Access to records & documents etc.
5. 25A Drawing of samples
6. 25AA Transaction between associates
MCQs with solutions
ICMAP & CA mod C past papers theoretical questions
In this chapter
CIR stands for Commissioner Inland Revenue
RP stands for Registered Person
Book keeping and invoicing requirements (U/s 22 to 25AA):
1. Records (U/s 22)
A RP making taxable supplies shall maintain and keep at his business premises or registered office in English or
Urdu language the following records of goods purchased, imported and supplies (including zero-rated and exempt
supplies) made by him or by his representative in such form and manner for ascertainment of his tax liability during a
tax period:
(a) records of supplies made shall indicate the description, quantity and value of goods, name and address of the
person to whom supplies were made and the amount of the tax charged;
(b) records of goods purchased shall show the description, quantity and value of goods, name, address and
registration number of the supplier and the amount of the tax on purchases;
(c) records of goods imported shall show the description, quantity and value of goods and the amount of tax paid
on imports;
(d) records of zero-rated and exempt supplies.
(da) double entry sales tax accounts;
(e) invoices, credit notes, debit notes, bank statements, banking instruments inventory records, utility bills, salary
and labour bills, rental agreements, sale purchase agreements and lease agreements;
(ea) record relating to gate passes, inward or outward, and transport receipts; and
(f) such other records as may be specified by the Board:
The persons paying retail tax shall keep such record as may be specified by the Board.
The Board may specify for any class of taxable persons or any other person registered under this Act to keep such
other records for the purposes of this Act
The Board may require, a RP or class of registered persons to declare and use only as many number of business
bank accounts to make or receive payments on account of purchase and sale transactions for the purpose of this Act
or rules.
The Board may specify for any class of taxable persons registered to use such electronic fiscal cash registers as are
approved by the Board.
The Board may prescribe the procedure or software for electronically maintenance of records, filing of sales tax
returns or refunds and for any other matter by a person or class of such persons.
The registered persons, whose accounts are subject to audit under the Companies Ordinance, 1984, shall be
required to submit a copy of the annual audited accounts, alongwith a certificate by the auditors certifying the
payment of due tax by the registered person.

Conceptual Approach to Taxes 569


Records and Book Chapter-04

2. Tax invoices [U/S 23]:


A RP making a taxable supply shall issue a serially numbered tax invoice at the time of supply of goods containing
the following particulars, namely:-
(a) name, address and registration number of the supplier;
(b) name, address and registration number of the recipient;
(c) date of issue of invoice;
(d) description and quantity of goods;
(e) value exclusive of tax;
(f) amount of sales tax; and
(g) value inclusive of tax:
The Board may specify such modified invoices for different persons or classes of persons:
The person shall not issue more than one tax invoice for a taxable supply.
No person other than a RP or a person paying retail tax shall issue an invoice.
A RP making a taxable supply may, subject to such conditions as specified by the Board may issue invoices to
another RP electronically, the manner and procedure for regulating the issuance and authentication of tax invoices.
3. Retention of record and documents for six years [U/S 24]:
A person, who is required to maintain any record or documents under this Act, shall retain the record and documents
for a period of 6 years after the end of the tax period to which such record or documents relate or till such further
period the final decision in any proceedings for assessment, appeal, revision, reference, petition and any proceedings
before an alternative Dispute Resolution Committee is finalized.
4. Access to records and documents etc. [U/S 25]:
A person who is required to maintain any record or documents under this Act or any other law shall, as and when
required by CIR, produce record or documents and provide access to such data which are in his or his representative
possession or control.
The officer of Inland Revenue authorized by the CIR, on the basis of the record, obtained, may once in a year,
conduct audit:
Where the CIR is satisfied that such registered person is involved in tax fraud or evasion of tax, he may authorize an
officer of Inland Revenue, not below the rank of Assistant Commissioner Inland Revenue, to conduct an inquiry or
investigation.
Further nothing shall bar the officer of Inland Revenue from conducting audit of the records of the registered person if
the same were earlier audited by the office of the Auditor-General of Pakistan.
After completion of the audit under this section or any other provision of this Act, the officer of Inland Revenue may,
after obtaining the registered person's explanation on all the issues raised in the audit shall pass an order u/s 11.
Explanation: In order to remove doubt, it is declared that the powers of the Board, Commissioner or officer of Inland
Revenue under this section are independent of the powers of the Board under section 72B and nothing contained in
section 72B restricts the powers of the Board, CIR or officer of Inland Revenue to have access to premises, stocks,
accounts, records, etc. under these sections or to conduct audit under these sections.
If a RP wishes to deposit the amount of tax short paid or amount of tax evaded along with default surcharge
voluntarily, whenever it comes to his notice, before receipt of notice of audit, no penalty shall be recovered from him:
If a RP wishes to deposit the amount of tax short paid or amount of tax evaded along with default surcharge during
the audit, or at any time before issuance of show cause notice, he may deposit the evaded amount of tax, default
surcharge and 25% of the penalty payable:
If a RP wishes to deposit the amount of tax short paid or amount of tax evaded along with default surcharge after
issuance of show cause notice, he shall deposit the evaded amount of tax, default surcharge and full amount of the
penalty payable and thereafter, the show cause notice, shall stand abated.
4.1 Drawing of samples [U/S 25A]:
Where an authorized officer of Inland Revenue considers it necessary to take a sample of any goods or raw materials
to determine the value of goods and sales tax liability or for any other reason, he may remove a minimum quantity to
enable him for proper examination or analysis to be made. At the time of taking the sample the person in possession
of the goods shall be informed and given the opportunity to sign the representative samples, so drawn and the same
shall be against a proper receipt a copy each of which shall be kept in the record by the registered person and the
relevant sales tax department.
4.2 Transaction between associates [U/S 25AA]:
The CIR or an officer of Inland Revenue may, in respect of any transaction between persons who are associates,
determine the transfer price of taxable supplies between the persons on the basis of fair market value.

570 Conceptual Approach to Taxes


Records and Book Chapter-04

MULTIPLE CHOICE QUESTIONS


Q.1. Companies which are also registered under the sales tax act are required to submit the copy of audited
accounts______________:
(a) Annually.
(b) Semi-annually
(c) Quarterly
(d) Monthly.
Q.2. Tax invoice shall be issued by______________:
(a) A registered person
(b) A non-registered person
(c) Person paying retail tax
(d) Both a and c
Q.3. The records and documents which are required to be maintained under Sales Tax Act, shall be retained for the period
of____________:
(a) 2 years
(b) 3 years
(c) 5 years
(d) 6 years after the end of relevant tax year.
Q.4 The registered person should maintain the record at his ___________.
(a) business premises
(b) registered office
(c) head office
(d) both a or b
Q.5 After sales tax registration, the record of _________ is not necessary to be maintained.
(a) exempt supplies
(b) zero rated supplies
(c) sales to non-registered person
(d) None of the above
Q.6 ____ can specify to the taxpayer, bank accounts which shall be used making or receiving payments and making
payment of tax due.
(a) Central Registration Office
(b) Board
(c) taxpayer
(d) Local Registration Office
Q.7 Records can also be maintained on _______ in such a form and manner as prescribed by the board.
(a) Computer
(b) electronic cash register
(c) none of the above
Q.8 Where the records are kept on computer, it is also required to keep the _______ of such records.
(a) hard copy
(b) back up
(c) hard disk
(d) all of the above

Conceptual Approach to Taxes 571


Records and Book Chapter-04

Q.9 The law requires that every registered person shall issue a tax invoice at the time of ________________.
(a) Purchase of goods
(b) making supply of goods
(c) making exempt supply
(d) making supply at discounted price
Q.10 The tax invoice shall also contain value of goods ___________.
(a) inclusive of sales tax
(b) exclusive of sales tax
(c) sales tax
(d) all of the above
Q.11 Tax invoice cannot be issued ___________.
(a) manually
(b) in duplicate
(c) in triplicate
(d) none of the above
Q.12 A sales tax officer may have access to the ______ kept by the taxpayer as and when required.
(a) stock
(b) records
(c) plant and machinery
(d) computer
Q.13 An audit by the tax officer generally conducted _________ in a year as necessary to verify the records obtained by
the taxpayer.
(a) as many times
(b) once
(c) two times
(d) None of the above
Q.14 The officer appointed for the purpose of investigation about the records kept should not be below the rank of
_________.
(a) Commissioner Inland Revenue
(b) Chief Commissioner Inland Revenue
(c) Assistant Commissioner
(d) None of the above
Q.15 Where a show-cause notice has been issued, _________ of penalty shall be recovered from the taxpayer
(a) 25%
(b) 50%
(c) 75%
(d) 100%
Q.16 Where an authorized officer of Inland Revenue considers it necessary to take a sample of any goods or raw materials
to determine the value of goods and sales tax liability or for any other reason, he may remove a ___________quantity
to enable him for proper examination or analysis to be made.
(a) minimum
(b) maximum
(c) average
(d) none of the above

572 Conceptual Approach to Taxes


Records and Book Chapter-04

Q.17 The Commissioner Inland Revenue or an officer of Inland Revenue may, in respect of any transaction between
persons who are associates, determine the transfer price of taxable supplies between the persons on the basis of
_______________.
(a) Fair Market Value
(b) Net Realisable Value
(c) market value
(d) Discounted value

ANSWERS
1 (a) 2 (a) 3 (d) 4 (d) 5 (d)

6 (b) 7 (a) 8 (b) 9 (b) 10 (d)

11 (a) 12 (b) 13 (b) 14 (c) 15 (d)

16 (a) 17 (a)

Conceptual Approach to Taxes 573


Records and Book Chapter-04

ICMAP PAST PAPERS THEORETICAL QUESTIONS


Q. NO. 5 (a) WINTER-2009 Describe the records required to be maintained by registered persons u/s 22 of the Sales Tax
Act, 1990.

Q. NO. 5 (b) SUMMER-2007 Describe the records, which are required to be maintained by a registered person u/s 22 of
Sales Tax Act, 1990

Q. NO. 8 WINTER-2003 Describe in detail the books keeping and invoicing requirements as required u/s 22 to 24 of Sales
Tax Act, 1990.

574 Conceptual Approach to Taxes


Records and Book Chapter-04

CA MOD C PAST PAPERS THEORETICAL QUESTIONS


Q.8 (a) Autumn 2013 State the provisions of Sales Tax Act, 1990 relating to maintenance and retention of records by a
registered person making taxable supplies.

Q. NO. 7(a) Spring 2010 State the provisions of Sales Tax Act, 1990 relating to maintenance and retention of records by a
registered person making taxable supplies.

Q. NO. 7(a) Spring 2008 A registered person making a taxable supply has to issue a tax invoice at the time of supply of
goods. What are the particulars that are required to be mentioned on the invoice?

Q. NO. 6(a) Autumn 2008 Mr. Kazim has recently started a business and has been registered under the Sales Tax Act,
1990. You are required to explain to him the provisions of Sales Tax Act, 1990 relating to maintenance and retention of
records.

Q. NO. 8(a) Autumn 2007 Every registered person making a taxable supply is required to issue a tax invoice at the time of
supply of goods. List down the particulars which are required to be contained in a sales tax invoice.

Q. NO. 8(b) Autumn 2006 Provisions relating to maintenance of records. List down the records that are required to be
maintained. Minimum period for which the records are required to be retained A registered person is required to maintain
certain records under the Sales Tax Act, 1990. You are required to:
(i) explain the provisions relating to maintenance of records.
(ii) list down the records that are required to be maintained.
(iii) specify the minimum period for which the records are required to be retained.

Q. NO. 14 Autumn 2004


(a) Discuss the salient features of a tax invoice?
(b) Who is permitted to issue a tax invoice?
(c) State the period for which a person is required to maintain records for the purpose of sales tax.

Q. NO. 12 Autumn 2003 List down the records that are required to be maintained by a person registered under the Sales
Tax Act, 1990 and for what period the records are to be maintained?

Q. NO. 11(b) Spring 2003 What are the minimum information required to be given on Sales Tax invoice.

Q.13 Autumn 2002 The finance manager of ABC Ltd requests for an advice on Tax Invoice, please draft a suitable reply in
the light of Sales Tax Act, 1990.

Conceptual Approach to Taxes 575


Records and Book Chapter-04

576 Conceptual Approach to Taxes


Scope and Payment of Tax Chapter-05

Chapter

5 SCOPE & PAYMENT OF TAX

(FOR CAF-6 and ICMAP Students)


Section Rule Topic
3 Scope of tax
3B Collection of excess tax, etc.
4 Zero rating
5 Change in the rate of tax
6 Time and manner of payment
7 Determination of tax liability
24 & 25 Apportionment of input tax
8 Tax credit not allowed
8A Joint and several liability of registered persons in supply chain where tax unpaid
8B Adjustable input tax
9 19, 20, 21 & 22 Debit and credit note
23 Destruction of goods
10 Refund on input tax
11 Assessment of tax
11A Short paid amounts recoverable without notice
13 Exemptions
49 Sales of taxable activity or transfer of ownership
53 Estate of deceased person
54 Estate in bankruptcy
55 Removal of difficulties
56 Service of orders, decisions, etc
57 Rectification of mistake
58 Liability of tax in the case of private companies or business enterprises
59 Tax paid on stocks acquired before registration
60 Powers to deliver certain goods without payment of tax
61A Repayment of tax to persons registered in Azad Jammu and Kashmir
68 Liability of the registered person for the acts of his agent
69 Issuance of duplicate of sales tax documents
73 Certain transactions not admissible
71 Special procedure
74 Condonation of time-limit
75 Application of the provisions of Act 1969 to Sales Tax

Part - II for CA Mod F and ICMAP students


Commercial importers
Retailers
3(8) Sales tax for CNG stations
Sales tax on services

MCQs with solutions


ICMAP and CA Mod C past papers theoretical questions

Conceptual Approach to Taxes 577


Scope and Payment of Tax Chapter-05

PART I (For CAF-6 and ICMAP students)


1. Scope of tax [U/S 3]:
Sales tax shall be charged, levied and paid at the rate of 17% on the value of:
(a) taxable supplies made by a registered person
(b) goods imported into Pakistan
Further Tax

Subject to the provision of section 8(6) or any notification issued there under, where taxable supplies are made to a
person who has not obtained registration number, there shall be charged, levied and paid a further tax at the rate of
2% of the value in addition to the rate specified in this section:
The Federal Government may, subject to such conditions and restrictions as it may impose, by notification in
the official Gazette, declare that in respect of any taxable goods, the tax shall be charged, collected and paid in
such manner and at such higher or lower rate or rates as may be specified in the said notification.
Tax on production capacity basis
The Board may, by notification in the official Gazette, in lieu of levying and collecting tax on taxable supplies, levy
and collect tax
(a) on the production capacity of plants, machinery, undertaking, establishments or installations producing or
manufacturing such goods; or
(b) on fixed basis, as it may deem fit, from any person who is in a position to collect such tax due to the nature of
the business.
The liability to pay the tax shall be:
(a) in the case of supply of goods, of the person making the supply, and
(b) in the case of goods imported into Pakistan, of the person importing the goods.
(c) The Federal Government may fix a lower or higher rate of sales tax in addition to 17%.
Federal Government vide SRO 657 dated 11-07-2013 has notified that the import and supply of second-hand
and worn clothing shall be charged to sales tax @ 5%.
Liability of Sales Tax on Purchases by Exporters
The Federal Government vide SRO 401(I)/2001 dated June 18, 2001 has notified that where the goods are supplied
to or received by a person as exporter under the provisions of Duty and Tax Remission for Export Rules, 2001 then
sales tax in respect of such goods shall be payable by the person receiving the supply instead of the person making
the supply if such person is unable to account for the goods in the manner prescribed under the Rules.
According to sales tax schedule the rates are given as follows:
Third Schedule (Complete schedule provided separately) 17% of retail price
Fifth Schedule (Complete schedule provided separately) Zero percent and
Sixth Schedule (Complete schedule provided separately) Exempted supplies
Eighth Schedule (Complete schedule provided separately) At specified rates
Ninth Schedule (Complete schedule provided separately) At specified rates
1.1 Taxable supplies made by a registered person [Section 3(2)(a)]
Taxable-supplies specified in the Third Schedule shall be charged to tax at the rate of 17% of the retail price with
sales tax shall be legibly, prominently and indelibly printed or embossed by the manufacturer on each item:
Provided that the Federal Government may exclude any taxable supply from the said Schedule,
Goods specified in 3rd schedule are given below:
Fruit juices and vegetable juices, ice cream, aerated waters or beverages, syrups and squashes, cigarettes, toilet
soap, detergents, shampoo, toothpaste, shaving cream, perfumery and cosmetics, tea, powder drinks, milky drinks,
toilet paper and tissue paper, spices sold in retail packing bearing brand names and trademarks, shoe polish and
shoe cream. Household electrical goods, including air conditioners, refrigerators, deep freezers, televisions, recorders
and players, electric bulbs, tube-lights, fans, electric irons, washing machines, telephone sets, Fertilizers, cement sold
in retail packing, Tiles sold in retail packing, Biscuits, confectionery, chocolates, toffees and candies.
Taxable supplies made by registered person [Section 3(2)(aa)]
Goods specified in the Eight Schedule (provided separately) shall be charged to tax at such rates and subject to such
conditions and limitations as specified therein.

578 Conceptual Approach to Taxes


Scope and Payment of Tax Chapter-05

Taxable supplies made by registered person [Section 3(B)]


Notwithstanding anything contained in sub sections (1) and (3), sales tax on the import and supply of the goods
specified in the Ninth Schedule (provided separately) to this Act shall be charged, collected and paid at the rates, in
the manner, at the time, and subject to the procedure and conditions as specified therein or as may be prescribed,
and the liability to charge, collect and pay the tax shall be on the persons specified therein.
1.2 Deduction of tax by withholding agents [U/S 3(7)]:
The Federal Government may, by notification in the Official Gazette, specify any person or class of persons as
withholding agent for the purpose of deduction and deposit of tax at specified rates and under such conditions or
restrictions as prescribe in this behalf. The following persons are termed as withholding agents:
Non applicability of sales tax special procedure (withholding) rules, 2007 on the following:
Electrical energy, natural gas, petroleum product as supplied by petroleum production and exploration companies,
Mild steel product, product made from sheets of iron or non-steel alloy, paper in rolls or sheets, plastic products
including pipes, vegetable ghee, cooking oil and telecommunication services.
Responsibilities of a withholding agent
The withholding agent, intending to make purchases of taxable goods, shall indicate in an advertisement or notice for
this purpose that the sales tax to the extent as provided in these rules shall be deducted from payment to be made to
the supplier.
A withholding agent shall deduct an amount equal to 20% of the total sales tax shown in the sales tax invoice issued
by the supplier and make payment of the balance amount to him as per illustration given below,--
Example: Value of taxable supplies excluding sales tax Rs. 1,000
Sales tax chargeable @ 17% Rs. 170
Sales tax deductible by the withholding agent Rs. 34 (i.e. Rs. 170 x 20%)
Sales tax payable by the withholding to the supplier Rs. 136 (i.e. Rs. 170 - Rs. 34)
Balance amount payable to the supplier by the withholding agent Rs. 1,136 (Rs. 1,000 + Rs. 136);
Purchases from un-registered persons: All withholding agents shall make purchases of taxable goods from a
person duly registered person, provided that under unavoidable circumstances and for reasons to be recorded in
writing, purchases are made from unregistered persons, the withholding agent shall deduct sales tax at 17% unless
otherwise specified of the gross value of taxable supplies made to him from the payment due to the supplier.
Advertisements on television and radio: Where the registered person who receives advertisement services, in
Pakistan or abroad, shall deduct the amount of sales tax as mentioned in the invoice and where the sales tax amount
not specified in the invoice then the sales tax at the rate applicable rate on advertisement services.
2. Collection of excess tax, etc. [U/s 3B]
Any person who has collected any tax which was not payable or which is in excess of the tax actually payable then
such amount shall be deemed to be an arrear of tax payable to the Federal Government and no claim for refund shall
be allowed in this respect.
Notwithstanding anything contained in any law or judgment of a court, including the Supreme Court and a High Court,
any amount payable to the Federal Government under sub section (1) shall be deemed to be an arrear of tax or
charge payable under this Act and shall be recoverable accordingly and any claim for refund in respect of such
amount shall neither be admissible to the registered person nor payable to any court of law or any person under
direction of the court.
The burden of proof that the incidence of tax has been or has not been passed to the consumer shall be on the
person collecting the tax.
3. Zero rating [U/S 4]
U/s 3 of sales tax act, the following goods shall be charged to tax at the rate of zero per cent:
(a) goods exported, or the goods specified in the Fifth Schedule;
(b) supply of stores and provisions for consumption aboard a conveyance proceeding to a destination outside
Pakistan;
(c) such other goods as notified by the Federal Government:
The Federal Government vide SRO 343(I)/2002 dated June 15, 2002 has notified that the goods exempted
under section 13 shall be charged to tax @ 0% on their export by manufacturers who make local supplies of
both taxable and exempt goods.
(d) such other goods as may be specified by the Board as are supplied to a registered person or class of
registered persons engaged in the manufacture and supply of goods at reduced rate of sales tax.

Conceptual Approach to Taxes 579


Scope and Payment of Tax Chapter-05

Examples of zero rated items:


Supply to diplomats, diplomatic missions and privileged persons
Supply of locally produced plant and machinery (as notified by the government), raw materials and
components for further manufacture of goods in EPZ.
Supply of duty free shops and to airlines, PNSC etc for use in aircrafts / ships proceeding outside Pakistan.
Import or supply made to Gawadar Special Economic Zone excluding vehicles.
Packing materials used for zero rated supplies.
Electric and gas consumed by manufacturer-exporters.
Other items specified under various notifications e.g. SRO 670 dated 18-07-2013 specified certain zero rated
goods subject to certain conditions including exercise books, pens, pencils, cheese, butter, bicycles etc.
However this section shall not apply in respect of supply of the goods which:
(i) are exported, but have been or are intended to be re-imported into Pakistan; or
(ii) have been entered for export but are not exported; or
(iii) have been exported to a country specified by the Federal Government (export of cigarettes to Afghanistan,
Iran or China [SRO 1232(I)/90 dated 01-12-1990] and export to Afghanistan by land route [SRO 190 dated 02-
04-2002].
4. Change in the rate of tax (u/s 5)
If there is a change in the rate of tax-
(a) a taxable supply made by a registered person shall be charged to tax at such rate as is in force at the time of
supply;
(b) imported goods shall be charged to tax in case the goods are:
(i) entered for home consumption, on the date on which a goods declaration is presented;
(ii) cleared from warehouse, on the date of goods declaration for clearance.
Provided that where a goods declaration is presented in advance of the arrival of the conveyance by which the goods
are imported, the tax shall be charged as is in force on the date on which the manifest of the conveyance is delivered.
Provided further that if the tax is not paid within seven (7) days of the presenting of the goods declaration under
section 104 of the Customs Act, the tax shall be charged at the rate as in force on the date on which tax is actually
paid.
5. Time and manner of payment (u/s 6):
The tax in respect of goods imported into Pakistan shall be charged and paid at the same time as if it were a customs
duty.
The tax in respect of taxable supplies shall be paid at the time of filing the return.
Notwithstanding anything contained in any other law for the time being in force, including but not limited to the
Protection of Economic Reforms Act, 1922, and notwithstanding any decision or judgement of any forum, authority or
court whether passed, before or after the promulgation of the Finance Act, 1998, referred to in sub section (1) shall be
incorporated in and shall be deemed to have always been so incorporated in this Act and no person shall be entitled
to any exemption from or adjustment of or refund of tax on account of the absence of such a provision in this Act, or
in consequence of any decision or judgement of any forum, authority or court passed on that ground or on the basis
of the doctrine of promissory estoppels or on account of any promise or commitment made or understanding given
whether in writing or otherwise, by any government department or authority.
The tax due on taxable supplies shall be paid by any of the following modes, namely:
(i) through deposit in a bank designated by the Board; and
(ii) through such other mode and manner specified by the Board.
6. Determination of tax liability (u/s 7):
6.1 Taxable supplies or services:
Subject to the provisions of section 8 and 8B for determining the tax liability in respect of taxable supplies made
during a tax period, a registered person shall subject to the provisions of section 73, be entitled to deduct input tax
paid or payable during the tax period for the purpose of taxable supplies made, or to be made, by him from the output
tax excluding the amount of further tax under sub section 3(1A) that is due from him in respect of that tax period and
to make such other adjustments of debit and credit notes as are specified in section 9. Where a registered person did
not deduct input tax within the relevant period, he may claim such tax in the return for any of the six succeeding tax
periods.

580 Conceptual Approach to Taxes


Scope and Payment of Tax Chapter-05

6.2 Restriction on deduction of input tax from output tax:


A registered person shall not deduct input tax from output tax unless,-
(i) He holds a tax invoice in his name and registration number;
(ii) in case of imported goods, he holds bill of entry or goods declaration, duly cleared under the Customs Act,
1969;
(iii) in case of goods purchased in auction, he holds a treasury challan, registration number, showing payment of
sales tax;
7. Apportionment of input tax (Rule 24 and 25):
Apportionment of input tax shall be used by registered persons who make taxable and exempt supplies
simultaneously.
Input tax on wholly taxable supplies shall be admissible.
Input tax paid wholly to exempt supplies shall not be admissible.
The input tax incurred for making both exempt and taxable supplies shall be apportioned according to the following
formula, namely;
Value of taxable sup plies
Residual input tax on taxable supplies = x Residual input tax
Value of taxable exempt sup plies
Monthly adjustment of input tax claimed by a registered person shall be treated as provisional adjustment and at the
end of each financial year, the registered person shall make final adjustment on the basis of taxable and exempt
supplies made during that year.
Any input tax adjustment claimed wrongfully on account of incorrect application of above formula shall be punishable
under the respective provisions of law irrespective of the fact that the claim was provisional.
8. Tax credit not allowed (U/S 8 & 21):
A registered person shall not be entitled to reclaim or deduct input tax paid on -
(a) the goods or services to be used other than for taxable supplies;
(b) any other goods or services notified by the Federal Government;
The Federal Government vide SRO 488(I)/2003 dated June 12, 2003 has notified the following goods that a
registered person cannot supply to unregistered person under the Sales Tax Act, 1990:
- Polypropylene granules (type yarn grade falling under PCT Heading 3902-1000);
- Sugar (supply of wholesalers and dealers);
- Air-conditioning, chilling plants and humidification plants;
- Cranes; and
- Propane storage tank, heat exchanger and gas separator.
Note: Above restriction shall not apply on the aforesaid last three items if the same are supplied to retail sale
to ultimate consumer, Government Organization and persons not liable to income tax but are liable to deduct
tax at source under the Income Tax Ordinance, 2001.
The Federal Government vide SRO 490(I)/2004 dated June 12, 2004 has notified the following goods on
which a registered person cannot claim input tax:
- Vehicles falling in chapter 87 of the First Schedule to the Customs Act, 1969;
- Food, beverages, garments, Fabrics etc and consumption of entertainments;
- Gifts and giveaways and
- Supply of electricity and gas to residential colonies of registered persons.
- Building materials including cement, bricks, paint etc.
- Office equipment (excluding electronic fiscal cash registers), furniture and fixtures
- Electric and gas appliances, pipes, fitting, wires, cables, electrical and sanitary fittings
- Crockery, cutlery etc
Note: If a registered person shall acquire the aforesaid items a stock in trade than he can claim the input tax
on such goods.
The Federal Government vide SRO 549(I)/2006 dated June 05, 2006 has notified that if a person is making
taxable supply of locally produced coal then such person shall not be allowed to claim input tax adjustment.

Conceptual Approach to Taxes 581


Scope and Payment of Tax Chapter-05

(c) the goods on which extra tax shall be charged;


The Federal Government may collect extra tax on any specified items up to a maximum of 17% in addition to
the normal sales tax.
Examples of extra tax:
Extra tax is levied @ 2% on certain specified goods such as air conditioners, refrigerators, deep freezers,
televisions etc. supplied by manufacturers and importers.
Extra tax is levied @ 5% on supplies of electric power and natural gas to unregistered or inactive persons
having industrial or commercial connections and whose bill in any month exceeds Rs. 15,000.
(d) the goods or services in respect of which sales tax has not been deposited in the Government treasury by the
respective supplier;
(e) purchases in respect of which a discrepancy is indicated by CREST or input tax of which is not verifiable in the
supply chain;
(f) fake invoices;
(g) purchases made by such registered person, who fails to furnish the information required by the Board.
(h) goods and services acquired for personal use or non business consumption;
(i) goods used in, or permanently attached to, immoveable property, such as building and construction materials,
paints, electrical and sanitary fittings, pipes, wires and cables, but excluding pre-fabricated buildings and such
goods acquired for sale or re-sale or for direct use in the production or manufacture of taxable goods;
(j) vehicles falling in Chapter 87 of the First Schedule to the Customs Act, 1969, parts of such vehicles, electrical
and gas appliances, furniture, furnishings, office equipment (excluding electronic registers), but excluding such
goods acquired for sale or re-sale.
(j) services in respect of which input tax adjustment is barred under the respective provincial sales tax law;
(k) import or purchase of agricultural machinery or equipment subject to sales tax at the rate of 7% under Eighth
Schedule to this Act; and
(l) from the date to be notified by the Board, such goods and services which, at the time of filing of return by the
buyer, have not been declared by the supplier in his return.
Under section 21(3) During the period of suspension of registration, the invoices issued by such person shall not be
entertained for the purposes of sales tax refund or input tax credit, and once such person is blacklisted, the refund or
input tax credit claimed against the invoices issued by him, whether prior or after such blacklisting, shall be rejected
through a self speaking appealable order and after affording an opportunity of bearing heard.
If a registered person deals in taxable and non-taxable supplies, he can reclaim only such proportion of the input tax
as is attributable to taxable supplies. [SRO 1307(I)1997 dated December 20,1997]
No person other than a registered person shall make any deduction or reclaim input tax in respect of taxable supplies
to be made by him.
No input tax credit shall be allowed to the persons who paid fixed tax.
9. Joint and several liability of registered persons in supply chain where tax unpaid (U/S 8A)
Where a registered person receiving a taxable supply from another registered person is in the knowledge or has
reasonable grounds to suspect that some or all of the tax payable in respect of that supply or any previous or
subsequent supply of the goods supplied would go unpaid of which the burden to prove shall be on the department,
such person as well as the person making the taxable supply shall be jointly and severally liable for payment of such
unpaid amount of tax:
The Board may exempt any transaction or transactions from the provisions of this section.
10. Adjustable input tax (U/S 8B):
A registered person shall not be allowed to adjust input tax in excess of 90% of the output tax for that tax period:
Tax charged on the acquisition of fixed assets shall be fully adjustable against the output tax in the month of
acquisition.
The Board may exclude any person or class of persons from this section.
A registered person may be allowed adjustment or refund of input tax on fulfilment of the following conditions, in case:
(i) whose accounts are subject to audit under the Companies Ordinance, 1984, upon furnishing a statement
along with annual audited accounts, duly certified by the auditors, showing value additions less than the limit
prescribed above; or
(ii) other registered persons, as notified by the Board.

582 Conceptual Approach to Taxes


Scope and Payment of Tax Chapter-05

The adjustment or refund of input tax if any to a registered person shall be made on yearly basis in the second month
following the end of the financial year.
The Board may notify any other limit of input tax adjustment for any person or class of persons.
Any auditor found guilty of misconduct in furnishing the certificate shall be referred to the Council for disciplinary
action.
Non applicability of section 8B [SRO 647(I)/2007, dated 27-06-2007]:
Provision of section 8B(1) shall not apply to the following registered persons:
1. Person registered in electric energy sector;
2. Oil marketing companies and petroleum refineries;
3. Fertilizers manufacturers;
4. Manufacturers consuming raw materials chargeable to sales tax @ 19.5% or 21% provided that the value of
such raw materials exceed 50% of value of all taxable purchases in a tax period;
5. Wholesalers-cum-retailers (covered under Chapter XII of Sales Tax Special Procedures Rules, 2007);
6. Commercial importers provided the value of imports subjected to 2% value addition tax under Chapter X of the
Sales Tax Special Procedures Rules, 2007, exceeds 50% of the value of all taxable purchases in a tax period.
7. Persons making zero-rated supplies provided value of such supplies exceed 50% of value of all taxable supplies
in a tax year;
8. Distributors and wholesalers;
9. Gas distribution companies;
10. Solvent extracting units of edible oils;
11. Telecommunication services; and
12. Pakistan Steel, Bin Qasim, Karachi
11. Debit and credit note [U/S 9]
Where a registered person has issued a tax invoice in respect of a supply made by him and as a result of cancellation
of supply or return of goods or a change in the nature of supply or change in the value of the supply or some such
event the amount shown in the tax invoice or the return needs to be modified, the registered person may, issue a
debit or credit note and make adjustment against output tax in the return.
Sales tax rules for debit and credit note [Rule 19, 20, 21 and 22]
Buyer on goods returns Issue a debit note
Supplier on receipt of returned goods Issue a credit note
The goods may be returned along with the debit note within 180 days from the date of supply however the collector
on the request of suppliers may extend this period for further 180 days.
The debit and credit notes shall be issued in duplicate and contain the information regarding the description, quantity
and the value of goods, based on the invoices issued at the time of supply, the amount of sales tax paid and the
number and date of original tax invoice. The adjustment of input and output taxes shall be available in the tax period
in which the goods are returned. The supplier shall reduce the amount of output tax in his return for the tax period in
which he received the goods.
Rule for destruction of goods [Rule 23]
Where goods are returned on the ground that those are unfit for consumption and need to be destroyed by the
suppliers, then along-with other formalities an approval of collector of sales tax shall be obtained before the goods are
destroyed in the presence of an officer not below the rank of Assistant Collectorate deputed by the collector.
Input tax on goods subsequently destroyed: A registered person is entitled to reclaim input tax paid on goods
which were subsequently destroyed and were not meant for use Lahore High Court in the case of Mayfair Spinning
Mills Ltd PTCL 2002 CL.115.
Input tax on wastage of Raw Materials during Manufacturing: Circular 1 of 1989 clarifies that such input is
reclaimable. However, if the wastage is such that can be sold then the same shall be considered as a by-product and
chargeable to sales tax unless specifically exempt.
12. Refund of input tax [U/S 10]
If the input tax paid by a registered person exceeds the output tax on account of zero rated local supplies or export
made during that tax period, the excess amount of input tax shall be refunded to the registered person not later than
45 days of filing of refund.

Conceptual Approach to Taxes 583


Scope and Payment of Tax Chapter-05

In case of excess input tax against supplies other than zero-rated or exports, such excess input tax may be carried
forward to the next tax period, along with the input tax as is not adjustable u/s 8B, and shall be treated as input tax for
that period.
If a registered person is liable to pay any tax, default surcharge or penalty payable under any law the refund of input
tax shall be made after adjustment of unpaid outstanding amount of tax.
Where a person has claimed input tax credit or refund which was not admissible to him, the proceedings against him
shall be completed within 60 days and may be extended for further 60 days but the same in no case may be
extended by Additional CIR for nine months.
13. Assessment of tax and recovery of tax not levied or short-levied or erroneously refunded [U/S 11]:
1- If a person is required to file a tax return fails to file the return for a tax period by the due date or pays an
amount which, for some miscalculation is less than the amount of tax actually payable,
an officer Inland Revenue shall, after a notice to show cause to such person, make an order for assessment of
tax, including imposition of penalty and default surcharge in accordance with section 33 and 34:
However if in the above case a person files the return after the due date and pays the amount of tax payable
along with default surcharge and penalty, the notice to show cause and the order of assessment shall abate.
2- If a person has not paid the tax due on supplies made by him or has made short payment or has claimed input
tax credit or refund which is not admissible under this Act for reasons other than those specified as above,
An officer Inland Revenue shall, after a notice to show cause to such person, make an order for assessment of
tax actually payable by that person or determine the amount of tax credit or tax refund which he has unlawfully
claimed and shall impose a penalty and charge default surcharge in accordance with section 33 and 34.
3- Where by reason of some collusion or a deliberate act any tax or charge has not been levied or made or has
been short-levied or has been erroneously refunded, the person liable to pay any amount of tax or charge or
the amount of refund erroneously made.
shall be served with a notice requiring him to show cause for payment of the amount specified in the notice.
4- Where, by reason of any inadvertence, error or misconstruction, any tax or charge has not been levied or
made or has been short-levied or has been erroneously refunded, the person liable to pay the amount of tax or
charge or the amount of refund erroneously made
shall be served with a notice requiring him to show cause for payment of the amount specified in the notice:
Provided that, where a tax or charge has not been levied as above, the amount of tax shall be recovered as
tax fraction of the value of supply.
5- No order under this section shall be made by an Officer Inland Revenue unless a notice to show cause is
given within 5 years, of the relevant date, to the person in default specifying the grounds on which it is
intended to proceed against him and the officer of Sales Tax shall take into consideration the representation
made by such person and provide him with an opportunity of being heard:
Provided that order under this section shall be made within 120 days of issuance of show cause notice or
within such extended period as the CIR may, for reasons to be recorded in writing, fix provided that such
extended period shall in no case exceed 90 days:
Provided further that any period during which the proceedings are adjourned on account of a stay order or
Alternative Dispute Resolution proceedings or the time taken through adjournment by the petitioner not
exceeding 60 days shall be excluded from the computation of the period specified above.
(6) Where a registered person fails to file a return, an Officer Inland Revenue not below the rank of Assistant
Commissioner shall subject to such conditions as specified by the Board, determine the minimum tax liability
of the registered person.
(7) For this section, the expression "relevant date" means--
(a) the time of payment of tax or charge as provided u/s 6; and
(b) in a case where tax or charge has been erroneously refunded, the date of its refund.
13.1 Short paid amounts recoverable without notice [U/S 11A]
Where a registered person pays the amount of tax less than the tax due as indicated in his return, the short paid
amount of tax along with default surcharge shall be recovered from such person by stopping removal of any goods
from his business premises and through attachment of his business bank accounts, without giving him a show cause
notice. No penalty shall be imposed unless a show cause notice is given to such person.
14. Exemption [U/S 13(1)]
Supply of goods or import of goods specified in the Sixth Schedule as specified by the Federal Government, is
exempt from tax as follows:

584 Conceptual Approach to Taxes


Scope and Payment of Tax Chapter-05

(a) pursuant to the approval of the Economic Coordination Committee of Cabinet, whenever circumstances
exist to take immediate action for the purposes of national security, natural disaster, national food
security in emergency situations, protection of national economic interests in situations arising out of
abnormal fluctuation in international commodity prices, removal of anomalies in taxes, development of
backward areas and implementation of bilateral and multilateral agreements;
The exemption from tax may be allowed from any previous date specified in the notification.
Items specified in 6th Schedule are exempt from levy of tax however, where a person does not desire to avail any
tax exemption, he may, after voluntary registration, opt. to pay sales tax as normal sales tax payer.

The Federal Government shall place before the National Assembly all notifications issued under this section in
a financial year.
Any notification issued, shall, if not earlier rescinded, stand rescinded on the expiry of the financial year in which
it was issued.
15. Sales of taxable activity or transfer of ownership [U/S 49]
(A) To a non registered person: In case of termination of taxable activity or part thereof or its sale or transfer of
ownership to a non-registered person, the possession of taxable goods or part thereof by the registered
person shall be deemed to be a taxable supply and the registered person shall be required to account for and
pay the tax on the taxable goods held by him;
Provided that if the tax payable by such registered person remains unpaid, the amount of unpaid tax shall be
the first charge on the assets of the business and shall be payable by the transferee of business.
(B) To a registered person: In the case of sale or transfer of ownership of a taxable activity or part thereof to
another registered person as an on-going concern, sales tax chargeable on taxable goods or part thereof shall
be accounted for and paid by the registered person to whom such sale is made or ownership is transferred.
16. Estate of deceased person [U/S 53]
The tax liability of a deceased registered person under the Act shall be the first charge on his estate in the hands of
his successors.
17. Estate in bankruptcy [U/S 54]
(1) If a registered person is declared bankrupt, the tax liability under this Act shall pass on to the estate in
bankruptcy if it continues to operate the business.
(2) If tax liability is incurred by an estate in bankruptcy, the tax is deemed to be a current expenditure in the
operations of the estate in bankruptcy and shall be paid before the claims preferred by other creditors are
settled.
18. Removal of difficulties [U/S 55]
If any difficulty arises in giving effect to the provisions of this Act or the rules made or notifications issued there under,
the Board may through a general order or otherwise, issue instructions-or directions, not inconsistent with the
provisions of this Act, for such actions to be taken by an Officer Inland Revenue or any other person as it considers
necessary or expedient for the purpose of removing the difficulty.
19. Service of orders, decisions, etc. [U/S 56]
(1) Any notice, order or requisition required to be served on a resident individual, other than in a representative
capacity shall be treated as properly served on the individual if-
(a) personally served on the individual or, in the case of an individual under a legal disability or a non-
resident individual, the representative of the individual;
(b) sent by registered post or courier service to the place of business or to the individual's usual or last
known address in Pakistan: or
(c) served on the individual in the manner prescribed for service of a summons under the Code of Civil
Procedure, 1908.
(2) Any notice, order or requisition required to be served on any person, other than a resident individual, for the
purposes of this Act, shall be treated as properly served on the person if-
(a) personally served on the representative of the person;
(b) sent by registered post or courier service to the person's registered office or address for service of
notices under this Act, in Pakistan, or where the person does not have such office or address, the notice
is sent by registered post to any office or place of business of the person in Pakistan; or

Conceptual Approach to Taxes 585


Scope and Payment of Tax Chapter-05

(c) served on the person in the manner prescribed for service of a summons under the Code of Civil
Procedure, 1908.
(3) Where an AOP is dissolved, any notice, order or requisition required to be served on the association may be
served on any person who was the principal officer or a member of the association immediately before such
dissolution.
(4) Where, business stands discontinued, any notice, order or requisition required to be served on the person
discontinuing the business may be served on the person personally or on any individual who was the person's
representative at the time of discontinuance.
(5) The validity of service of a notice issued shall not be called into question after the notice has been complied
with in any manner
Agreement for the Exchange of information [U/S 56A]
The Federal Government may enter into bilateral or multilateral agreements with provincial governments or
with governments of foreign countries for the exchange of information, including electronic exchange of
information, with respect to sales tax imposed under this Act or any other law of Pakistan and under the
corresponding laws of such countries and may, by notification in the official Gazette, make such provisions as
may be necessary for implementing such agreements.
The provisions of section 107 of the Income Tax Ordinance, 2001 shall, mutatis mutandis, apply to the
provisions of this section.
Disclosure of information by public servant [Section 56B]
Any information acquired under any provision of this Act or in pursuance of a bilateral or multilateral
agreement or tax information exchange agreement shall be confidential and no public servant shall disclose
any such information, except as provided under section 216 of the Income Tax Ordinance, 2001.
The provisions of section 216 of Income Tax Ordinance, 2001, shall, mutatis mutandis, apply to the provisions
of this section.
Prize schemes to promote tax culture [Section 56C]
The Board may prescribe prize schemes to encourage the general public to make purchases only from
registered persons issuing tax invoices.
Rectification of mistake [U/s 57]
The officer Inland Revenue, Commissioner, the Commissioner (Appeals) or the Appellant Tribunal may by an order
in writing to rectify any mistake apparent in any order passed by him or from the record on his or its own motion or
any mistake brought to his or its notice by a taxpayer or, in the case of the Commissioner (appeals) or the Appellate
Tribunal, the Commissioner.
No rectification order under this section which has the effect of increasing an assessment, reducing a refund or
otherwise applying adversely to the taxpayer shall be made unless the taxpayer has been given a reasonable
opportunity of being heard.
Where a mistake apparent on the record is brought to the notice of the officer of Inland Revenue, Commissioner or
Commissioner (Appeals), as the case may be, and no order has been made, before the expiration of the financial
year next following the date on which the mistake was brought to their notice, the mistake shall be treated as rectified
and all the provisions of this Act shall have effect accordingly.
No order under this section shall be made after five years from the date of order sought to be rectified.
20. Liability for payment of tax in the case of private companies or business enterprises [U/S 58]
Where any private company or business enterprise is wound up and any tax chargeable on the company or business
enterprise, whether before, or in the course of, or after its liquidation, in respect of any tax period cannot be recovered
from the company or business enterprise, every person who was a owner of, or partner in, or director of, the company
or business enterprise during the relevant period shall, jointly and severally with such persons, be liable for the
payment of such tax.
21. Tax paid on stocks acquired before registration [U/S 59]
The tax paid on goods purchased by a person who is subsequently required to be registered u/s 14 due to new
liabilities or levies or gets voluntary registration shall be treated as input tax, provided that such goods were
purchased by him from a registered person against an invoice issued during a period of 30 days before making an
application for registration and constitute his verifiable unsold stock on the date of compulsory registration or on
the date of application for registration or for voluntary registration:

586 Conceptual Approach to Taxes


Scope and Payment of Tax Chapter-05

Provided that where a person imports goods, the tax paid by him thereon during a period of 90 days before making
an application for registration shall be treated as an input tax subject to the condition that he holds the bill of entry
relating to such goods and also that these are verifiable unsold or un-consumed stocks on the date of compulsory
registration or on the date of application for registration or for voluntary registration.
22. Powers to deliver certain goods without payment of tax [U/S 60]
The Federal Government may authorise the import of goods or class of goods, without payment of the whole or any
part of the tax payable thereon to the following persons, namely:-
(i) registered importers importing such goods temporarily with a view to subsequent exportation;
(ii) registered manufacturer-cum-exporters who import raw materials and intermediary products for further
manufacture of goods meant for export.
23. Repayment of tax to persons registered in Azad Jammu and Kashmir [U/S 61A]
The Board may authorize the repayment in whole or in part of the input tax paid on any goods acquired in or imported
into Pakistan by the persons registered in Azad Jammu and Kashmir as are engaged in making of zero-rated
supplies.
24. Liability of the registered person for the acts of his agent [U/S 68]
When any person is expressly or impliedly authorised by a registered person to be his agent for all or any of the
purposes of this Act, the registered person shall be responsible for the act done by his agent.
25. Issuance of duplicate of sales tax documents [U/S 69]
An officer Inland Revenue not below the rank of Assistant Commissioner may, on payment of one hundred rupees,
issue an attested duplicate of any sales tax document as is available with the department or has been filed made
there under to a relevant registered person applying for the same.
26. Certain transactions not admissible [U/S 73]
(1) Payment of the amount for a transaction exceeding value of Rs.50,000, excluding payment against a utility bill,
shall be made by a crossed cheque drawn on a bank or by crossed bank draft or crossed pay order or any
other crossed banking instrument showing transfer of the amount of the sales tax invoice in favour of the
supplier from the business bank account of the buyer:
Provided that online transfer of payment from the business account of buyer to the business account of
supplier as well as payments through credit card shall be treated as transactions through the banking channel,
subject to the condition that such transactions are verifiable from the bank statements of the respective buyer
and the supplier.
(2) The buyer shall not be entitled to claim input tax credit, adjustment or deduction, or refund, repayment or draw-
back or zero-rating of tax under this Act if payment for the amount is made otherwise than as stated above,
provided that payment in case of transaction on credit is so transferred within one hundred and eighty days of
issuance of the tax invoice.
(3) The amount transferred in terms of this section shall be deposited in the business bank account of the
supplier, otherwise the supplier shall not be entitled to claim input tax credit, adjustment or deduction, or
refund, repayment or draw-back or zero-rating of tax under this Act.
Explanation - For the purpose of this section, the term "business bank account" shall mean a bank account utilized
by the registered person for business transactions, declared to the CIR in whose jurisdiction he is registered through
Form STR 1 or change of particulars in registration database.
27. Special procedure [U/S 71]
(1) The Federal Government may, by notification in the official Gazette, prescribe special procedure for scope and
payment of tax, registration, book keeping and invoicing requirements and returns, etc; in respect of such
supplies as may be specified therein.
(3) The trade enrolment Certificate Schemes immediately in force before the commencement of the Finance Act,
1999, shall be deemed to be validly made under this Act.
28. Condonation of time-limit [U/S 74]
Where any time or period has been specified within which any application is to be made or any act or thing is to be
done, the Board may in any case or class of cases, permit such application to be made or such act or thing to be
done within such time or period as it may consider appropriate:
Provided that the Board may, by notification in the official Gazette, and subject to such limitations or conditions as
may be specified therein, empower any Commissioner Inland Revenue to exercise the powers under this section in
any case or class of cases.

Conceptual Approach to Taxes 587


Scope and Payment of Tax Chapter-05

29. Application of the provisions of act 1969 to sales tax [U/S 75]
The Federal Government may, by notification in the official Gazette, declare that any of the provision of the Customs
Act, 1969, relating to the levy of, and exemption from, customs duties, draw-back of duty, warehousing, confiscation,
and procedure relating to offences and appeals shall, with such modifications and alterations as it may consider
necessary or desirable to adapt them to the circumstances, be applicable in regard to like matters in respect of the
tax imposed by section 3.

PART II (For CA Mod F and ICMAP students)


30. Commercial Importers / Other importers [Section 3 read with Sales tax Special Rules 58A to 58E]:
A commercial importer shall pay sales tax on import value @ 17% in the normal manner. However, sales tax on
account of minimum value addition shall be collected at import stage @ 3% of the value of goods imported in addition
to the sales tax paid in the normal manner.
The commercial importer shall charge sales tax @ 17% in the normal manner from his customers and the sales tax
paid on import value including 3% shall form part of input tax and claimable against output tax for determining his net
liability.
The excess of input tax, if any, over output tax shall be carried forward to the next tax period. However, the refund of
excess input tax over output tax in respect of such commercial imports shall not be allowed to a registered person.
Exclusions: Provided that the value addition tax shall not be charged on,-
(i) the goods as are imported by a manufacturer for in house consumption;
(ii) the POL products, imported by an Oil Marketing Company for sale in the country, whose prices are
regulated under a special pricing arrangement by the Government of Pakistan or by regulatory authority
working under the Government of Pakistan; and
(iii) registered service providers importing goods for their in house business use or for furtherance of their
taxable activity and not intended for further supply.
31. Retailers {Rule 3 to 10 of Sales Tax Special Procedure Rules, 2007]
See separate chapter on special Procedure Rules, 2007.
32. Sales tax for CNG stations [U/S 3(8)]:
Notwithstanding anything contained in any law or notification made there under, in case of supply of natural gas to
CNG stations, the Gas Transmission and Distribution Company shall charge sales tax from the CNG stations at the
rate of 17% of the value of supply to CNG consumers, as notified by the Board from time to time, but excluding the
amount of sales tax, as provided in clause (46) of section 2.
33. Sales tax on services
Sales tax on certain services is leviable at various rates on specified services under Provincial Sales Tax Ordinances
including:
1. Services provided or rendered by hotels, clubs and caterers;
2. Advertisement on TV and radio, other than few exceptions such as advertisement by a Government Agency
for health and education.
3. Services provided or rendered by custom agents
4. Courier services
5. Other specified services.
Input tax credit can be claimed by the service providers paid on purchases, imports or utilities subject to certain
limitations.
Similarly, sales tax paid on services by the recipient of services can be claimed as input tax if the services are used
against taxable supplies.

588 Conceptual Approach to Taxes


Scope and Payment of Tax Chapter-05

MULTIPLE CHOICE QUESTIONS


Q.1. The sales tax is charged, levied and paid at the rate of___________.
(a) 16%
(b) 15%
(c) 17%
(d) None of the above
Q.2. Sale tax is charged___________.
(a) Directly to the manufactures of goods
(b) Indirectly to the ultimate consumers of goods
(c) Indirectly to the retailers
(d) All of the above
Q.3. Sales tax on imports for manufacture is included in the___________.
(a) Input tax
(b) Output tax
(c) Both a and b
(d) None of the above
Q.4. While calculating the tax on imports, the addition should be made in the value of imports assessed by custom
authorities in respect of___________ on imports.
(a) Custom duty
(b) Federal excise duty
(c) Invoice value
(d) Both a and b
Q.5. Extra tax on specified goods under the special procedure may be charged at the rate of___________.
(a) 2%
(b) 5%
(c) 20%
(d) 17%
Q.6. Where a registered person is making taxable as well as exempt supplies, then credit of input tax is allowed only in
respect of___________.
(a) Taxable supplies
(b) Exempt supplies.
(c) Total supplies
(d) None of the above.
Q.7. Withholding agent shall deduct an amount out of the total sales tax showed in the tax invoice issued by the supplier,
equal to___________.
(a) 1/10th of total sales tax
(b) 1/5th of total sales tax
(c) 1/6th of total sales tax
(d) None of the above
Q.8. It is necessary for the withholding agents to purchase taxable goods from_______.
(a) registered person
(b) non-registered person
(c) Both a and b
(d) None of the above
Q.9. Where under unavoidable circumstances purchases by the withholding agent are made from an unregistered person,
the sale tax deducted by the withholding agent shall be___________.

Conceptual Approach to Taxes 589


Scope and Payment of Tax Chapter-05

(a) 5%
(b) 10%
(c) 17%
(d) 1/5th of the tax payable by the supplier
Q.10. The withholding agent shall issue a certificate to the supplier showing therein_____.
(a) The name and registration number of the supplier
(b) Description of goods purchase and the amount of sales tax deducted from the supplier
(c) Both a and b
(d) None of the above
Q.11. While making the payment of sales tax, the supplier shall claim___________.
(a) The credit of input tax
(b) The credit of sales tax deducted by the withholding agent
(c) Both a and b
(d) None of the above
Q.12. The non-applicability of the provision of sales tax special procedure (withholding) rule is not applicable in respect of
companies engaged in___________.
(a) Distribution of gas electricity
(b) Providing telephone services, including mobile phone services
(c) Both a and b
(d) None of the above
Q.13. A retailer having annual turnover less than Rs. 5 million in respect of taxable supplies made by him___________.
(a) Is not required to pay sales tax
(b) Shall pay turnover tax as per slab rates.
(c) Shall pay the tax irrespective of the quantum of supplies made by him
(d) none of the above
Q.14. A retailer paying sales tax on electricity bill basis is__________.
(a) not required to be registered under the Sales Tax Act, 1990
(b) required to be registered under the Sales Tax Act, 1990
(c) optional for the retailer that he may be registered or not.
(d) none of the above
Q.15. Goods exported out of Pakistan are charged to tax at the rate of___________.
(a) 10%
(b) 15%
(c) 0%
(d) 5%
Q.16 Sales tax in respect of imported goods is charged at the time of ___________.
(a) sale of such goods
(b) import of goods
(c) on raising of invoice
(d) All of the above
Q.17 Sales tax for taxable supplies made in Pakistan is paid at the time of ___________.
(a) on raising of invoice
(b) on receipt against sales
(c) filing of return
(d) All of the above
Q.18 Sales tax on purchases made from registered persons is subtracted from the ________with limitations, where
applicable.

590 Conceptual Approach to Taxes


Scope and Payment of Tax Chapter-05

(a) output tax


(b) input tax
(c) income tax
(d) sales tax refund
Q.19 Sales tax on sales to non-registered person is _____________.
(a) Output tax
(b) Input tax
(c) Exempt
(d) Zero rated
Q.20 Commercial invoice is issued by a ____________.
(a) non-registered person
(b) registered person
(c) commercial importer
(d) exporter
Q.21 _________ means price fixed by the manufacturer or importer inclusive of all charges and taxes including sales tax.
(a) sale price
(b) retail price
(c) total price
(d) consumer price
Q.22 Where goods falling under 3rd schedule of the Sales Tax Act, 1990 are sold by manufacturer at discount then sale
price for the purpose of sales tax shall be equal to________________.
(a) discounted price
(b) retail price
(c) purchase price
(d) none of above
Q.23 Extra tax charged shall __________ as input tax against output tax.
(a) be available
(b) not available
(c) None of the above
Q.24 The _________ may specify any person or a class of persons as withholding agent for the purposes of deduction and
deposit of tax.
(a) Provincial government
(b) Federal government
(c) Board
(d) Commissioner Inland Revenue
Q.25 Withholding agent means ____________________.
(a) Govt. departments
(b) autonomous bodies
(c) persons registered as exporters
(d) All of the above
Q.26 Under ____________ circumstances goods can be purchased by withholding agents from unregistered persons.
(a) special
(b) unavoidable
(c) any
(d) None of above.
Q.27 The tax liability of a deceased registered person under the Sales Tax Act shall be the _____ charge on his estate in
the hands of his successors.

Conceptual Approach to Taxes 591


Scope and Payment of Tax Chapter-05

(a) first
(b) last
(c) none of above
Q.28 The registered supplier shall issue sales tax invoice for all _____ supplies made to the withholding agent.
(a) exempt supplies
(b) zero rated supplies
(c) taxable
(d) all of above
Q.29 A person falling in the definition of cottage industry is _________to be registered under the Sales Tax Act, 1990.
(a) not required
(b) required
(c) optional
(d) none of the above
Q.30 In case of retailers who are not required to get themselves registered, the sales tax paid on electricity bills by them
shall discharge their tax liabilities under the ________________.
(a) Sales Tax Act
(b) Income Tax Ordinance
(c) FED Act
(d) Both a and b
Q.31 The Govt. may recover the tax deducted by a person as ___________.
(a) arrears of tax
(b) penalty
(c) default surcharge
(d) none of above
Q.32 The goods which have been entered for export, although not exported are treated as____ supplies.
(a) taxable
(b) exempt
(c) zero rated
(d) none of the above
Q.33 Zero rated supply are ____________.
(a) exempt supplies
(b) taxable supplies
(c) none of the above
(d) all of the above
Q.34 The goods, which have been exported to a country specified by the _________ are not included in the zero rated
supply.
(a) Securities and Exchange Commission of Pakistan
(b) Federal Government
(c) Chief Commissioner Inland Revenue
(d) Commissioner Inland Revenue
Q.35 The credit of input tax is available on __________ supplies.
(a) taxable supplies
(b) zero rated supplies
(c) exempt supplies
(d) both a and b

592 Conceptual Approach to Taxes


Scope and Payment of Tax Chapter-05

Q.36. An officer Inland Revenue not below the rank of Assistant Commissioner may, on payment of Rs. ___, issue an
attested duplicate of any sales tax document as is available with the department or has been filed made there under
to a relevant registered person applying for the same.
(a) 200
(b) 100
(c) 300
(d) 400
Q.37. The ________ may, by notification in the official Gazette, prescribe special procedure for scope and payment of tax,
registration, book keeping and invoicing requirements and returns, etc; in respect of such supplies as may be
specified therein.
(a) Board
(b) Federal Government
(c) Central Registration Office
(d) none of above
Q.38. Payment of the amount for a transaction exceeding value of Rs._______, excluding payment against a utility bill, shall
be made by a crossed cheque.
(a) 50,000
(b) 75,000
(c) 100,000
(d) 150,000
Q.39. A registered retailer (having credit or debit card machines) shall deposit the sales tax along-with return on _______.
(a) monthly
(b) quarterly
(c) annually
(d) semi-annually
Q.40. The sales tax on account of minimum value addition shall be levied and collected at import stage on goods imported
for other than in house use @ ___% of the value of goods in addition to the tax chargeable under the Act.
(a) 6
(b) 5
(c) 4
(d) 3
Q.41. Any person who has collected any tax which was not payable or which is in excess of the tax actually payable then
such amount shall be deemed to be an _____ payable to the Federal Government and no claim for refund shall be
allowed in this respect.
(a) arrear of tax
(b) default surcharge
(c) additional tax
(d) none of above
Q.42. The burden of proof that the incidence of tax has been or has not been passed to the consumer shall be on the
person _____________.
(a) paying the tax
(b) collecting the tax
(c) both a and b
(d) none of above
Q.43. If there is a change in the rate of tax, a taxable supply made by a registered person shall be charged to tax at such
rate as is in force at the __________.
(a) time of supply
(b) time of purchase
(c) time of payment of goods

Conceptual Approach to Taxes 593


Scope and Payment of Tax Chapter-05

(d) none of above


Q.44. If there is a change in the rate of tax, imported goods shall be charged to tax on the date on which a goods
declaration is presented if goods are _________.
(a) cleared from warehouse
(b) entered for home consumption
(c) both a and b
(d) none of above
Q.45. Where a person imports goods, the tax paid by him thereon during a period of __________ days before making an
application for registration shall be treated as an input tax subject to fulfilment of certain conditions.
(a) 70
(b) 80
(c) 90
(d) none of above
Q.46. The tax in respect of goods imported into Pakistan shall be charged and paid at the same time as if it were a
__________.
(a) sales tax
(b) custom duty
(c) value addition tax
(d) none of above
Q.47. The tax in respect of taxable supplies shall be paid at the time of __________.
(a) sale of goods
(b) purchase of goods
(c) filing of return
(d) export of goods
Q.48. Where a registered person did not deduct input tax within the relevant period, he may claim such tax in the return for
any of the ___ succeeding tax periods.
(a) 6
(b) 7
(c) 8
(d) 9
Q.49. If a person fails to file the return or pays an amount which is less than the amount of tax actually payable, an officer
Inland Revenue shall, after a notice to show cause to such person, make an order for assessment of tax, including
imposition of ________.
(a) penalty
(b) default surcharge
(c) both a and b
(d) none of above
Q.50. Where a registered person pays the amount of tax less than the tax due as indicated in his return, the short paid
amount of tax along with default surcharge shall be recovered from such person by ____
(a) stopping of removal of goods from business premises
(b) through attachment of his business bank accounts
(c) both a and b
(d) none of above
Q.51. Supply of goods or import of goods specified in the ___th Schedule as specified by the Federal Government is
exempt from tax.
(a) 6
(b) 5
(c) 4
(d) 7

594 Conceptual Approach to Taxes


Scope and Payment of Tax Chapter-05

Q.52. The Commissioner Inland Revenue or an Officer Inland Revenue may, in respect of any transaction between persons
who are associates, determine the transfer price of taxable supplies between the persons as is necessary to reflect
the ______ of supplies in an arm's length transaction.
(a) cost value
(b) fair market value
(c) net realizable value
(d) none of above
Q.53. In case of termination of taxable activity or part thereof or its sale or transfer of ownership to a non-registered person,
the possession of taxable goods or part thereof by the registered person shall be deemed to be a ________ supply.
(a) taxable
(b) exempt
(c) zero rated supply
(d) none of above
Q.54. The tax liability of a deceased registered person under the Act shall be the ______ charge on his estate in the hands
of his successors.
(a) last
(b) first
(c) second
(d) none of above
Q.55. If a registered person is declared bankrupt, the tax liability under this Act shall pass on to the ______ if it continues to
operate the business.
(a) legal representative
(b) successor
(c) estate in bankruptcy
(d) none of above
Q.56. If any difficulty arises in giving effect to the provisions of this Act or the rules made or notifications issued there under,
the _______ may through a general order or otherwise, issue instructions-or directions, not inconsistent with the
provisions of this Act, for such actions to be taken by an Officer Inland Revenue or any other person as it considers
necessary or expedient for the purpose of removing the difficulty.
(a) Board
(b) Commissioner Inland Revenue
(c) Appellate Tribunal Inland Revenue
(d) none of above
Q.57. Where an AOP is dissolved, any notice, order or requisition required to be served on the association may be served
on _____________ immediately before such dissolution.
(a) principal officer
(b) member
(c) both a and b
(d) none of above
Q.58. Where, business stands discontinued, any notice, order or requisition required to be served on the person
discontinuing the business may be served on the _________ at the time of discontinuance.
(a) person personally
(b) persons representative
(c) both a and b
(d) none of above
Q.59. Where any private company or business enterprise is wound up and any tax chargeable on the company or business
enterprise, cannot be recovered from the company or business enterprise, every person who was a owner of, or
partner in, or director of, the company or business enterprise during the relevant period shall, ________, be liable for
the payment of such tax.
(a) jointly and severally

Conceptual Approach to Taxes 595


Scope and Payment of Tax Chapter-05

(b) individually
(c) none of above
Q.60. The tax paid on goods purchased by a person who is subsequently required to be registered or gets voluntary
registration shall be treated as input tax, provided that such goods were purchased by him from a registered person
against an invoice issued during a period of ______ days before making an application for registration.
(a) 60
(b) 30
(c) 90
(d) 120

ANSWERS
1 (c) 2 (b) 3 (a) 4 (d) 5 (a)
6 (a) 7 (b) 8 (a) 9 (c) 10 (c)
11 (c) 12 (c) 13 (a) 14 (a) 15 (c)
16 (b) 17 (a) 18 (a) 19 (a) 20 (a)
21 (b) 22 (b) 23 (b) 24 (b) 25 (d)
26 (c) 27 (a) 28 (d) 29 (a) 30 (a)
31 (a) 32 (a) 33 (b) 34 (b) 35 (d)
36 (b) 37 (b) 38 (a) 39 (a) 40 (d)
41 (a) 42 (b) 43 (a) 44 (b) 45 (c)
46 (b) 47 (c) 48 (a) 49 (c) 50 (c)
51 (a) 52 (b) 53 (a) 54 (b) 55 (c)
56 (a) 57 (c) 58 (c) 59 (a) 60 (b)

596 Conceptual Approach to Taxes


Scope and Payment of Tax Chapter-05

ICMAP PAST PAPERS THEORETICAL QUESTIONS


Q.NO. 5 (a) March 2015 In view of the provisions of Sales Tax Laws:
(i) Identify the circumstances under which the debit and credit notes are required to be issued in respect of supply
made by a registered person.
(ii) Specify the manner and modes in which sales tax due on imported or taxable supplies shall be paid.

Q. NO. 5 (a) March 2015 (b) XYZ Industries (pvt.) Limited has been established for many years and registered under the
Sales Tax Act, 1990. The Chief Financial Officer (CFO) of the company, Mr. Akram observed that there is often delay and
discrepancies in filing of sales tax return due to inexperienced staff regarding the sales tax matters. He requested the
Sales Tax Advisor of the company to train the staff of the sales tax department.

Required:
Being the Sales Tax Advisor, guide the staff regarding the following queries in the light of the Sales Tax Act, 1990:
(i) In cases of any short payment of sales tax, how the company can avoid and reduce imposition of penalty under
section 33 of the Sales Tax Act, 1990?
(ii) Under which situations, XYZ Industries (pvt.) limited may be held liable to pay default surcharge in addition to the
sales tax due on it?

Q.NO. 5(a) August 2014 M/s. Sultan Limited is contemplating ways to increase its sales. Assume you are a Tax
Consultant and the management of the company is seeking your advice on the sales tax implications. Discuss the
provisions of the Sales Tax Act, 1990 in respect of the sales tax and valuation of the goods under the following schemes:
(i) For a certain range of products, it is being proposed to provide sample packs Free of Cost to the customers.
(ii) A mix of products X, Y and Z is proposed to be sold at a concessional rate as a Package Deal.
(iii) Sales of certain products are intended to be introduced under the hire purchase/ installment mode. However, an
additional issue raised in this regard that the rate of sales tax will be changed subsequently.

Q.NO. 6(a) August 2014 Ajmal Traders is engaged in the retail business. They are not registered with the sales tax
authorities. They intended to get themselves registered as a retailer. However, before getting themselves registered, they
have certain questions.
Required:
Being the companys Tax Advisor, you are required to answer the following queries in the light of the relevant provisions of
the Sales Tax Act, 1990:
(i) What is the threshold of the value of supplies upon which registration with the sales tax authorities
is compulsory?
(ii) Whether zero-rated and exempted supplies will be considered as a part of value of supplies for the purpose
of charging and collecting sales tax?
(iii) Can a retailer adjust input tax or claim refund of sales tax?
(iv) At what dates, quarterly sales tax returns are required to be filed by the retailer?
(v) Whether issuance of invoice is mandatory for supplies made by the retailer? How such invoices will be
generated?
Q. No. 5(b) February 2014
Briefly state in the light of the provisions of the Sales Tax Act, 1990 whether the following persons can reclaim input tax?
(i) Alpha Private Limited is a renowned company established in 1990. It is registered under the said Act and deals in
taxable and non-taxable supplies.
(ii) Mr. Amir is a non-registered person under the said Act, deals in taxable supplies only.
Q. No. 5(c) February 2014
Shalimar Private Limited (SPL) manufactures and supplies household electrical goods to Altamash Enterprises (AE). Both
the companies are registered under the Sales Tax Act, 1990. During the month of January, 2014 SPL had supplied 30
washing machines to AE. However, AE decided to return 16 washing machines to SPL due to sub-standard quality.
Required: Under the provision of the Sales Tax Rules, 2006 describe the procedure to be followed by Altamash
Enterprises (AE) for returning the goods.
Q. No. 6 (a) (i) Spring 2013
Apparently an exempt supply and a zero-rated supply are look-alike. Under both the cases a person is not
required to pay tax under the Sales Tax Act, 1990. However, these two types of supplies differ with each other
on many points.
Differentiate exempt supply and zero-rated supply with reference to the following:
Taxability

Conceptual Approach to Taxes 597


Scope and Payment of Tax Chapter-05

Registration under the Sales Tax Act, 1990


Credit of input
Maintenance of records under the Sales Tax Act,1990
Filling of return under the Sales Tax Act,1990
Q. No. 6 (a) (ii) Spring 2013
Where any goods are returned by the buyer on the ground that the same are unfit for consumption and are
required to be destroyed by the supplier. What shall be the requirement for destroying such goods under the Sales
Tax Rules, 2006? Is buyer entitled to claim input tax on such goods?
Q. No. 5(b) February 2013
Mr. Waqar and Mr. Tanveer are registered persons under the sales Tax Act 1990, Mr. Waqar purchases goods from Mr.
Tanveer. Mr. Waqaar has reasonable grounds to suspects that Mr. Tanveer has not been depositing the sales tax paid by
Mr. Waqar on the supplly of goods by Mr. Tanveer. Advise Mr. waqar about the following:
(i) What is the liability of Mr, waqar in the above scenario?
(ii) Who has the power to declare the transactions exempt from any liability in case where the tax remains unpaid?
Q. No. 4 (a) August 2012
As per section 2(48) of Sales Tax Act, 1990 zero-rated supply means a taxable supply which is charged to tax at the rate of
zero percent under section (4). On what grounds rate of zero percent is applicable under section 4 of the Sales Tax Act,
1990?
Q. No. 4 (b) August 2012
Where a person who is required to file a tax return fails to file the return f or a tax period by the due date, an officer of (Inland
Revenue) shall, after a notice to show cause to such person, make an order for assessment of tax in accordance with
sections 33 and 34 of the Sales Tax Act, 1990. What is the time limitation of passing an order related to assessment of tax
under section 11 of the Sales Tax Act, 1990?
Q.4 (b) (i) SUMMER 2012
U/s 23 of the Sales Tax Act, 1990, a registered person making a taxable supply is required to issue a serially numbered tax
invoice at the time of supply of goods containing certain particulars. What are these particulars?
(ii) Can the above mentioned invoice at (b) (i) be issued by an unregistered person?
Q. NO. 4 (b) WINTER 2010
(i) What are the goods that shall be charged to tax at 0% under the Sales Tax Act, 1990?
(ii) What are the exceptions to the above Rule?
Q. NO. 5 (b) WINTER 2009
An officer of Sales Tax obtains authority u/s 38 of the Sales Tax Act, 1990 for access to premises, stocks and records.
Explain the legal provisions in this regard.
Q. NO. 5 (a) SUMMER 2009
What are the provisions of the Sales Tax Law regarding refund of input tax?
Q. NO. 5 (b) SUMMER 2009
Differentiate between zero rates supply and exempt supply, showing major distinctions
Q. NO. 5 (a) WINTER 2007
What are the requirements and contents of a sales tax invoice u/s 23 of Sales Tax Act, 1990?
Q. NO. 5 (a) SUMMER 2007
Explain the term Short paid amount recoverable without notice as per Section 11A of Sales Tax Act, 1990.
8 (b) SUMMER-2006
How the tax deposited at the time of receipt of advance is adjusted, when supply or part thereof is cancelled or changed,
under Sales Tax Special Procedure Rules, 2005?
Q. NO. 7 (b) WINTER 2006
Describe the procedure for recovery of tax not levied or short levied or erroneously refunded u/s 36

598 Conceptual Approach to Taxes


Scope and Payment of Tax Chapter-05

Q. NO. 8 (a) WINTER 2005


Describe the conditions under which a person can claim input tax on the stock acquired before registration under Sales Tax
Act, 1990.
Q. NO. 3 (a) SUMMER 2005
Explain the circumstances under which registered person is not entitled to re-claim or deduct input tax.
Q. NO. 4 WINTER 2004
Explain briefly the provisions of Section 38, 40 and 40A of the Sales Tax Act, 1990 concerning access of authorized officers
to premises stock accounts and records and searches to be made.
Q. NO. 6 (a) SUMMER 2004
What do you understand by Posting of Sales Tax Officer u/s 40B of the Sales Tax Act, 1990
Q. NO. 7 SUMMER 2003
Describe the scope of Sales Tax u/s 3 of the Sales Tax Act, 1990.

Conceptual Approach to Taxes 599


Scope and Payment of Tax Chapter-05

CA MOD C PAST PAPERS THEORETICAL QUESTIONS


Q.NO.8 Spring 2015 Saleem is registered under the Sales Tax Act, 1990 and is engaged in the business of export and
distribution of electronic appliances.
Required:
Under the provisions of the Sales Tax Act, 1990 and Rules made thereunder, advise Saleem on the following matters:
(a) any six situations in which input tax is not allowed to be adjusted against the output tax liability.
(b) exports which are outside the purview of zero rating.
(c) eligibility for a refund if input tax is paid in excess of the output tax payable for the month.
(d) concept of provisional and final adjustment in relation to Apportionment of input tax.

Q.NO.5 (b) Autumn 2014 There are certain food items in the inventory of XY Limited (XYL) which were returned by
the customers after the expiry date. Specify the procedure which must be followed under the Sales Tax Rules, 2006 if XYL
wishes to destroy these items.

Q. NO. 10(a) Spring 2014 Under the provisions of Sales Tax Act, 1990:
Identify the situations under which a debit or credit note may be issued by a registered person.
Q.7 (a) Autumn 2012 Identify the goods that shall be charged at the rate of zero per cent under the Sales Tax Act, 1990.
(b) List the situations in which the type of goods identified in (a) above would not be eligible for zero rating.
Q.8 (b) Autumn 2012 While carrying the sales tax audit of haleema, the Officer of Inland Revenue identified a deficiency
in the amount of sales tax deposited by her. She acknowledged this deficiency but failed to deposit the balance amount.
Determine Haleemas liability in the above situation. Also explain whether it would have been to her advantage if she had
paid the amount before issuance of the show cause notice.
Q. 8 Spring 2012 Mr. Zamarrud is engaged in the manufacture and sale of taxable as well as zero-rated products.
Required: As a tax consultant, advise Ms. Zamarrud on the following matters:
(a) The conditions that need to be satisfied for the adjustment of input tax against the output tax liability.
(b) Any seven situations in which input tax is not allowed to be adjusted against the output tax liability.
(c) The remedy available to her if she fails to adjust input tax in the period in which it is paid.
Q.2 (b) Spring 2011 List down the particulars to be mentioned on the debit note issued by the supplier in the event of
change in the value of supply, under the Sales Tax Rules, 2006.
Q.7 (b) Autumn 2011 Mr. Gohar has recently been registered under the Sales Tax Act, 1990. He is engaged in the export
and distribution of consumer products. Before filing the first return, he wishes to obtain advice on the following matters:
(i) Eligibility for a refund if input tax paid is in excess of the output tax payable for the month.
(ii) Consequences of non-payment of the entire amount of tax due as indicated in the return.
(iii) Concept of provisional and final adjustment.
Required: Comment on each of the above matters. (08 marks)
Q. NO. 7(b) Spring 2010 List the type of exports which are outside the purview of zero rating.
Q. NO. 7(b) Spring 2009 With reference to the Sales Tax Act, 1990, identify the situations under which a registered person
shall not be entitled to claim or deduct input tax.
Q. NO. 6(b) Autumn 2008 Certain food items supplied by Pakistan Distributors (Pvt.) Ltd. (PDL) have been returned by
the customers after the expiry date. PDL wishes to destroy them. Specify the procedure which would have to be followed in
this regard.
Q. NO. 7(b) Spring 2008 Sales Tax Act, 1990 places certain restrictions on adjustment of input tax. You are required to
explain the related provisions in respect of the following:
(i) Extent of restriction on admissibility of input tax;
(ii) The conditions under which the amount of input tax which had been so restricted may subsequently be allowed;

600 Conceptual Approach to Taxes


Scope and Payment of Tax Chapter-05

(iii) Treatment of sales tax paid on acquisition of fixed assets.


Q. NO. 7(a) Autumn 2007 How would the input tax on raw material be determined and claimed where a registered person
is engaged in making taxable as well as exempt supplies?
Q. NO. 7(b) Autumn 2007 A contract has been signed on May 20, 2006 by Mr. Pervez and Mr. Farooq. Both of them are
registered persons. Under the contract, Mr. Pervez will supply branded computers to Mr. Farooq within two months. At the
time of entering into contract, the goods were exempt from sales tax. Through the Finance Act, 2006, the Government
withdrew this exemption with effect from July 1, 2006. 25% advance was paid on signing of contract and balance on the
delivery of goods on July 15, 2006. Advise Mr. Pervez on the chargeability of sales tax.
Q. NO. 7(a) Spring 2007 There shall be charged, levied and paid a tax known as sales tax at the rate of 15% of the value
of taxable supplies made by a registered person in the course or furtherance of any taxable activity carried on by him; and
goods imported into Pakistan.
Explain the term taxable supply and taxable activity used in above statement describing the scope of sales tax.
Q. NO 12 Spring 2006 With reference to Sales Tax Rules 2005 relating to the apportionment of input tax, explain the
following:
(a) Residual input tax (b) Manner of computation of residual input tax credit on taxable supplies (c) Concepts of provisional
and final adjustment.
Q. NO. 7(a) Autumn 2006 Where goods supplied are returned, the supplier is required to issue credit note. The buyer
responds by issuing a debit note.
You are required to specify the particulars to be mentioned on the debit note issued by the buyer.
Q. NO. 7(b) Autumn 2006 Describe the circumstances under which a registered person shall not be able to claim or
deduct input tax?
Q. NO. 8(a) Autumn 2006 What is the difference between zero-rated and exempt supplies?
Q. NO. 9 Spring 2006 List down the exports which are outside the purview of zero rating u/s 4 of the Sales Tax Act, 1990.
Q. NO. 10 Autumn 2005 In the monthly sales tax return filed under the Sales Tax Act, 1990, input tax paid for the month is
adjusted against output tax payable. You are requested to comment on the following issues relating to input / output
adjustment Can input tax paid in prior periods be claimed in a monthly tax return.
(a) What would happen, if output tax paid exceeds input tax payable for the month.
(b) What recourse is available if output tax for the month inadvertently disclosed at a lesser amount
(c) Can input tax paid in prior periods be claimed in a monthly tax return.
(d) What would happen, if input tax paid exceeds output tax payable for the month.
(e) What recourse is available if output tax for the month is inadvertently disclosed at a lesser amount.
Q. NO. 10 Spring 2005 What impact would a change in rate of Sales-tax have in case of:
(a) Supply of goods. (b) Import of goods.
Q. NO. 11 Spring 2005 What are the precondition of claiming input tax against output tax by a registered person?
Q. NO. 11 Spring 2004 Briefly explain the uses of debit and credit notes under the Sales Tax Act. 1990
Q. NO. 12 Autumn 2004 Discuss the treatment provided under the STA, 1990 in respect of any excess tax collected from
the customer.
Q. NO. 13 Autumn 2004 Describe the category of goods on which sales tax is chargeable at Zero rate?
Q. NO. 6 (b) SUMMER 2004 Describe the following provisions under Sales Tax Act, 1990
1- Debit and Credit Notes u/s-9
2- Exemption from Penalty and Additional Tax u/s 34A.
Q. No. 8(a) Autumn 2003 Distinguish between the concept of zero rating and exemption.
Q. No. 12 Summer 2002 Mr. Omar has recently registered himself under Sales Tax Act. He has written a letter enquiring
the following in the light of the Sales Tax Act:
- Whether taxable supplies can be sold at discounted price.
- Whether sales tax is payable in discounted price or normal price,
- Whether discounts can be given at varying rates
Please draft a suitable reply.

Conceptual Approach to Taxes 601


Scope and Payment of Tax Chapter-05

Q. No. 13 Summer 2002 Explain the incidence of sales tax, if any, for exporters.
Q.11 Spring 2002 Explain the term " time of supply" with reference to the Sales Tax Act 1990.
Q.11 Autumn 2002 Elaborate the term input tax as defined in the Sales Tax Act,1990.
Q. No. 12 Autumn 2002
a. What is the significance of Third schedule to the Sales Tax Act,
b. Define retail price in the context of third schedule to the Sales Tax Act,
c. Whether trade discount allowed on products covered under the third schedule will affect sales tax levied on such
products. Give reasons in support of your answer.

602 Conceptual Approach to Taxes


Practice Questions with Solutions Chapter-06

Chapter

6 PRACTICE QUESTIONS WITH SOLUTIONS

Note: All the under stated questions have been solved under the Sales Tax Act effective from July 1st 2015.
The following points are very important for students before solving the problems of sales tax.
Q.1 Briefly explain in which of the following cases a person is liable to be registered and also calculate tax payable on
turnover, if any:
Sr. No. Particulars
1 Mr. Aslam is an importer; his taxable turnover during last 12 months is Rs. 5,000,001.
2 A wholesaler having taxable turnover during last 12 months Rs. 400,000.
3 A manufacturer having taxable turnover during last 12 months Rs. 4,000,000.
4 A manufacturer having taxable turnover during last 12 months Rs. 5,000,000.
5 A manufacturer having taxable turnover during last 12 months Rs. 5,000,001.
6 Mr. Amir is a manufacturer; his taxable turnover during last 12 months was Rs. 2,500,000 and his utility
bills during last 12 months was
a) Rs. 600,000
b) Rs. 800,000
c) Rs. 900,000
7 Mr. Zahid is a Retailer; his taxable turnover during last 12 months is less than Rs. 5 million.
8 Taxable turnover of Mr. Akram during last 12 months is Rs. 400,000, he is a distributor.
9 Mr. Usman is a Retailer; his taxable turnover during last 12 months is Rs. 15 million.
10 A person wants to import goods, whether he is required to get himself registered or not.
11 Cottage Industry
12 Mr. Bilal is a manufacturer; detail regarding his turnover during last 12 months is as follows:
Local sales 3,500,000
Exports (Zero rated supplies) 2,500,000
13 Mr. Kamran is a manufacturer, information regarding last 12 months is given below:
Exports (Zero rated supplies) 3,000,000
Electricity bills 850,000
14 Mr. Ehsan is a Commercial exporter and wants to claim refund.

Solution of Q.1:
Important note: This solution for requirement of registration has been made under section 14 of the Sales Tax Act,
1990.

Sr. No. Particulars


1 An importer is required to be registered irrespective of its turnover.
Sales tax payable (5,000,001 x 17%) = 850,000
2 A wholesaler is required to be registered irrespective of its turnover.
Sales tax payable (400,000 x 17%) = 68,000
3 In this case manufacturer is not liable to be registered as his taxable turnover is less than Rs. 5,000,000.
(It is assumed that his utility bills during last 12 months do not exceed Rs. 800,000, otherwise he shall be
liable to be registered.)
4 In this case manufacturer is not liable to be registered as his taxable supplies do not exceed Rs.
5,000,000. (It is assumed that his utility bills during last 12 months do not exceed Rs. 800,000, otherwise
he shall be liable to be registered.)

Conceptual Approach to Taxes 603


Practice Questions with Solutions Chapter-06

5 In this case manufacturer is liable to be registered as his taxable turnover is more than Rs.5,000,000.
Sales tax payable (5,000,001 x 17%) = 850,000
6 A In this case manufacturer is not required to be registered as he falls in the definition of cottage
industry. (cottage industry is not liable to be registered)
(Cottage industry means a manufacturer whose annual taxable turnover during the last 12 months
ending any tax period does not exceed Rs.5 million or whose annual utility (electricity, gas and
telephone) bills during the last 12 months ending any tax period do not exceed Rs.800,000.
B In this case manufacturer is not required to be registered as he falls in the definition of cottage
industry i.e. his utility bills do not exceed Rs.800,000. (cottage industry is not liable to be registered)
(Cottage industry means a manufacturer whose annual taxable turnover during the last 12 months
ending any tax period does not exceed Rs.5 million or whose annual utility (electricity, gas and
telephone) bills during the last 12 months ending any tax period do not exceed Rs.800,000.
C In this case manufacturer is required to be registered as his utility bills exceed Rs. 800,000.
Sales tax payable (2,500,000 x 17%) 425,000
7 Mr. Zahid is a retailer (other than dealing only in exempt supplies) hence liable to be registered as there is
limit on taxable supplies.
8 A distributor is required to be registered irrespective of its turnover.
Sales tax payable (400,000 x 17%) = 68,000
9 A retailer (other than exempt supplies) is liable to be registered as there is limit on taxable supplies.
Sales tax shall be payable under normal procedure u/s 3 of the Sales Tax Act, 1990.
10 An importer is required to be registered irrespective of its turnover.
11 Cottage industry is not required to be registered.
12 Total taxable supplies (including zero rated supplies) of the manufacturer is 6,000,000, so he is liable to
be registered.
Sales tax payable (3,500,000 x 17%) = 595,000
13 Mr. Kamran is required to be registered as he is a manufacturer and his utility bills during last 12 months
are more than Rs. 800,000.
However he is not required to pay any sales tax as his sales are zero rated.
14 A commercial exporter is not required to be registered, however if he wants to claim refund of input tax
then he must get himself registered.

Q.2: Compute sale tax liability of Mr. Aslam (registered manufacturer) for the month of July 2015 from following
information.
Rs.
Sales to registered persons 650,000
Purchases from registered persons 300,000
Purchases from non-registered persons 100,000
Solution:
Output tax:
On sales to registered persons U/S 3 (Rs. 650,000 x 17%) 110,500
Less: Input tax
On purchases from registered person (300,000 x 17%) 51,000
On purchases from non-registered persons (Note-1) -
Total input tax 51,000
Limitation on input tax upto 90% of output tax u/s 8B of the
Sales the Sales Tax Act, 1990 (Rs. 110,500 x 90%) 99,450
Input tax is fully admissible as it is less than 90% of output tax. ______
Sales tax payable 59,500
(Note 1) As purchases from non-registered persons are without sales tax invoices hence the same shall be
without sales tax u/s 23(2) of the Sales Tax Act, 1990, therefore the question of adjustment of input tax in not
applicable.
Q.3 Explain section 8B of the Sales Tax Act, 1990.

604 Conceptual Approach to Taxes


Practice Questions with Solutions Chapter-06

Solution:
A registered person shall not be allowed to adjust input tax in excess of 90% of the output tax for that tax period:
Tax charged on the acquisition of fixed assets shall be fully adjustable against the output tax in the month of
acquisition.
The Board may exclude any person or class of persons from this section.
A registered person may be allowed adjustment or refund of input tax on fulfilment of the following conditions, in
case:
(i) whose accounts are subject to audit under the Companies Ordinance, 1984, upon furnishing a statement along
with annual audited accounts, duly certified by the auditors, showing value additions less than the limit prescribed
above; or
(ii) other registered persons, as notified by the Board.
The adjustment or refund of input tax if any to a registered person shall be made on yearly basis in the second
month following the end of the financial year.
The Board may notify any other limit of input tax adjustment for any person or class of persons.
Any auditor found guilty of misconduct in furnishing the certificate shall be referred to the Council for disciplinary
action.
Q.4: Following information has been provided by Mr. Zohaib registered as commercial importer for the month of August
2015.
Rs.
Invoice price of imported goods 500,000
Value for custom duty 550,000
Custom duty 50,000
FED (Federal Excise Duty) 30,000

Compute sales tax payable under following two situations


a) If taxable supplies 800,000
b) If taxable supplies 700,000

Solution (a):

Output tax:
On taxable supplies U/S 3 (Rs. 800,000 x 17%) 136,000

Less Input tax:


On commercial imports U/S 3 (Rs. 630,000 x 17%) (Note 2) 107,100
Additional tax paid on commercial imports U/R 58B(1) (Rs. 630,000 x 3%) 18,900
126,000
Balance sales tax payable 10,000
Notes:
1. Restriction of 90% of output tax is not applicable in case of commercial importer under SRO 647(I)/2007 dated
June 27, 2007.
2. Import value u/s 2(46)(iii)(d) for the purposes of sales tax is as under:
Rs.
Value for custom duty 550,000
Add: Custom duty 50,000
Add: Federal Excise duty 30,000
630,000

Conceptual Approach to Taxes 605


Practice Questions with Solutions Chapter-06

Solution (b):
Output tax:
On taxable supplies (Rs. 700,000 x 17%) 119,000
Less input tax:
On commercial imports U/S 3 (630,000 x 17%) (Note 2) 107,100
Additional tax paid on commercial imports U/R 58B(1) (Rs. 630,000 x 3%) 18,900
126,000
Balance sales tax excess paid (Read with note 3) (7,000)
Notes:
1. Restriction of 90% of output tax is not applicable in case of commercial importer under SRO 647(I)/2007 dated
June 27, 2007.
2. Import value u/s 2(46)(iii)(d) for the purposes of sales tax is as under:
Rs.
Value for custom duty 550,000
Add: Custom duty 50,000
Add: Federal Excise duty 30,000
630,000
3. It is important to note that if commercial imports stock is totally sold then there should be no refund as it is
minimum value addition case however if imported stock is not fully sold then there may be excess payment as
per return however there will be no refund in minimum tax case. Further if the value addition is in excess of the
minimum value addition then the taxpayer has to pay the sales tax on the value addition in excess of total sales
tax paid at import stage U/R 58C of Sales Tax Special Procedure Rules, 2007.
Q.5: Mr. Mohsin is a registered manufacturer of goods falling under third schedule. Information about his sales and
purchases for the month of August 2015 is as follows:
Sales (after 10% discount) 45,000
Taxable purchases 30,000
Required: Calculate sale tax payable by Mr. Mohsin.
Solution:
Goods falling under 3rd schedule are chargeable to tax @ 17% at retail price. If discount is allowed on these goods
then sales tax shall be calculated on the retail price and not on the discounted price.
Output tax:
Tax on sales (45,000 x 100/90 x 17%) 8,500
Input tax:
Tax on purchases (30,000 x 17%) 5,100
Full input tax is allowed as it is less than 90% of output tax of Rs. 7,650. ____
Balance sales tax payable 3,400
Q.6: Calculate output tax on following supplies

Sr. #
1 Taxable supplies Rs. 232,000 inclusive of sales tax
2 Local taxable supplies Rs. 116,000 without sales tax

Solution:

(Note 1) When value of sales are given including sales tax, then value of sales tax shall be calculated by using tax fraction
formula U/S 2(36) i.e. Sales tax = value including sales tax x 17 / 117. [Rs. 232,000 x 17/117 = 33,709]
(Note 2) Output tax on local taxable supplies without sales tax [Rs. 116,000 x 17% = 19,720]

Q.7 Mr. Kamran is a registered manufacturer under the Sales tax Act, 1990. Data regarding his business for the month of
July 2015 is as follows:

606 Conceptual Approach to Taxes


Practice Questions with Solutions Chapter-06

Sr. # Rs.
1 Taxable supplies to registered persons 500,000
2 Taxable supplies to non-registered persons (including the amount of Sales Tax) 250,000
3 Exempted sales (made from exempt purchases only) 300,000
4 Supplies made to DTRE registered persons 200,000
5 Zero rated supply 450,000
6 Taxable purchases from registered persons 130,000
7 Purchases (for exempt supplies only) 50,000
8 Sales tax paid on factory gas bill consumed in labor residential colonies 2,500
9 Sales tax paid on factory electricity bills (Sales tax registration number is printed on bills) 5,600
10 Sales tax paid on factory telephone bills (Sales tax registration number is printed on bills) 500
11 Sales tax receivable (previous month) 200

Required: Calculate sales tax payable.

Solution:
Output tax Rs.
On taxable supplies to registered persons U/S 3 (Rs. 500,000 x 17%) 85,000
On taxable supplies to non-registered persons U/S 3 (Rs. 250,000 x 17/117) (Note - 2) 36,325
th
On supplies to DTRE registered persons [U/S 4 read with 5 Schedule] 0
On zero rates supplies [U/S 4 read with 5th Schedule] 0
121,325
Input tax
On taxable purchases used only for taxable local supplies (Note - 3) 11,566
On factory electricity bills (Note - 4) 5.600
On factory telephone bills (Note - 4) 500
On factory gas bill of labor colonies (Note 5) 0
Previous months b/f 200
Actual admissible input tax (A) 17,866
90% of output tax (Rs. 121,325 x 90%) (B) (Note - 3) 109,193
Less Admissible input tax: Lower of (A) or (B) 17,866
Sales tax payable 103,459

Refundable in respect of zero rated supplies & DTRE supplies (Rs. 3,241 + 7,293] 10,534
(Note - 1)
Input tax on purchases from registered persons (130,000 x 17%) 22,100
Total input tax for apportionment 22,100

Apportionment of taxable supplies input tax


[U/R 25 of the Sales Tax Rules, 2006] Supplies Input tax
Rs. Rs.
Local supplies to registered persons 500,000
Local supplies to non-registered persons (250,000 x 100/117) 213,675
Total local supplies 713,675 11,566
Supplies related to DTRE and zero rated 650,000 10,534
1,363,675 22,100

Conceptual Approach to Taxes 607


Practice Questions with Solutions Chapter-06

(Note 2) It has been assumed that 2% further tax u/s 3(1A) is not applicable on local taxable supplies to unregistered
persons by virtue of SRO 648(I)/2013 dated July 09, 2013 otherwise further tax shall be accounted for and paid separately
without adjustment of the same against input tax / refund of the registered person and further it shall also not be considered
for the computation of 90% limitation on output tax.
(Note - 3) As the zero rated supplies (including sales to DTRE registered persons) are less than 50% of all taxable supplies
under SRO 647(I)/2007 dated June 27, 2007, therefore 90% limitation is applicable U/S 8B of the Sales Tax Act, 1990.
(Note - 4) The question has been solved on the assumption that sales tax used on factory electricity and telephone bills has
been used only for taxable supplies (other than zero rated supplies) hence no apportionment of the same has not been
made.
(Note 5) Input tax paid on gas bills of labor residential colonies is not admissible input tax under SRO 490(I)/20014 dated
June 12, 2004.
(Note - 6) The question has been solved on the assumption that exempt supplies have been made only from exempt
purchases and without use of any sales tax of other inputs, hence the same has been fully accounted for in the input tax
without any apportionment.

Q.8 Mr. Kamran is a registered manufacturer under the Sales Tax Act, 1990. Data regarding his business for August 2015 is
as follows

1 Taxable turnover to registered persons 400,000


2 Taxable turnover to non-registered persons 150,000
3 Credit note issued for taxable supplies 50,000
4 Debit note issued for taxable supplies 20,000
5 Supplies to DTRE registered persons 20,000
6 Zero rated supply 45,000
7 Taxable purchases from registered persons 250,000
8 Exempted purchases 40,000
9 Sales tax paid on electricity bills 7,500
(Sales tax registration number is printed on bills)
10 Sales tax paid on telephone bills 8,500
(Sales tax registration number is printed on bills)

Required: Calculate sales tax payable.


Solution
Output tax Rupees
On taxable turnover to registered persons U/S 3 (Rs. 400,000 x 17%) 68,000
On taxable turnover to non-registered persons U/S 3 (Rs. 150,000 x 17%) 25,500
On supplies to DTRE registered persons [U/S 4 read with 5th Schedule] 0
On zero rates supplies [U/S 4 read with 5th Schedule] 0
On debit note issue in respect of sales (Rs. 20,000 x 17%) (Note 4) 3,400
On credit note issue in respect of sales U/R (Rs. 50,000 x 17%) (Note 5) (8,500)
88,400
Input tax
On local supplies (A) 52,317
90% of output tax (Rs. 88,400 x 90%) (without 2% further tax) (B) 79,560
Less: Admissible input tax: lower of (A) or (B) 52,317
Sales tax payable 36,083
2% further sales tax payable on supplies to un-registered persons (Note 3) 3,000
Sales tax refundable in respect of zero rated supplies & DTRE supplies
(Rs.4,280 + Rs. 1,903) 6,183
Note-1:
Taxable purchases from registered persons (Rs. 250,000 x 17%) 42,500
Sales tax paid on electricity bills 7,500
Sales tax paid on telephone bills 8,500
Total input tax to be apportioned 58,500

608 Conceptual Approach to Taxes


Practice Questions with Solutions Chapter-06

Apportionment of taxable supplies input tax


[U/R 25 of the Sales Tax Rules, 2006] Supplies Input tax
Rs. Rs.
Local supplies 550,000 52,317
Supplies related to DTRE and zero rated 65,000 6,183
615,000 58,500
(Note- 2) As the zero rated supplies (including sales to DTRE registered persons) are less than 50% of all taxable supplies
under SRO 647(I)/2007 dated June 27, 2007, therefore 90% limitation is applicable U/S 8B of the Sales Tax Act, 1990.
(Note 3) It has been assumed that 2% further tax is applicable on local taxable supplies to unregistered persons and the
same shall be accounted for and paid separately without adjustment of the same against input tax / refund of the registered
person and further it shall also not be considered for the computation of 90% limitation on output tax.
(Note 4) It has been assumed that debit note issued by supplier of the Sales tax Act is fulfilling the conditions of Rule 22 of
Sales Tax Rules, 2006 that has resulted into increase in value of taxable supplies.
(Note 5) It has been assumed that credit note issued by supplier U/S 9 of the Sales tax Act is fulfilling the conditions of
Rule 22 of Sales Tax Rules, 2006 that has resulted into decrease in value of taxable supplies.
Q.9 Mr. Khan is a registered manufacturer under the Sales Tax Act, 1990. Data regarding his business for the month
of July 2015 is as follows:
Rs.
1. Taxable turnover to registered persons 350,000
2. Taxable turnover to non-registered persons 140,000
3. Sales to retailers 50,000
4. Supplies to Government hospital (consisting of 30 beds) 150,000
5. Supplies to Government hospital (consisting of 60 beds) 130,000
6. Zero rated supply 45,000
7. Taxable purchases from registered persons 160,000
8. Taxable purchases from non-registered persons 25,000
9. Sales tax paid on electricity bills 10,000

Required: Calculate sales tax payable.

Solution Rs. Rs.


Output tax
On taxable turnover to registered persons U/S (Rs. 350,000 x 17%) 59,500
On taxable turnover to non-registered persons U/S (Rs. 140,000 x 17%) 23,800
Sales to retailers (Rs. 50,000 x 17%) 8,500
Supplies to Government / charitable hospital (30 beds) (Rs. 150,000 x 17%) 25,500
Supplies to Government / charitable hospital
(60 beds exempt under 6th Schedule) (Note 1) 0
th
Zero rated supplies [U/S 4 read with 5 Schedule] 0
117,300
Input tax
On taxable local supplies (working) (A) 29,674
90% of output tax (Rs. 117,300 x 90%) (without 2% further tax) (B) 105,570

Less: Admissible input tax: lower of (A) or (B) 29,674


Sales tax payable 87,626
2% further sales tax payable on supplies to un-registered persons (Note 3) 2,800
Sales tax refundable for zero rated supplies 1,935

Working
Taxable purchases from registered persons (Rs. 160,000 x 17%) 27,200
Taxable purchases from non-registered persons (Note 3) 0
Sales tax paid on electricity bills 10,000
Total input tax to be apportioned 37,200

Conceptual Approach to Taxes 609


Practice Questions with Solutions Chapter-06

Apportionment of residual input tax Residual input


[U/R 25 of the Sales Tax Rules, 2006] Supplies tax
Rs. Rs.

Taxable local supplies 690,000 29,674


Exempt supplies 130,000 5,591
Zero rated supplies 45,000 1,935
865,000 37,200
Note:
1. Supplies to Government / charitable hospital consisting of 50 or more beds are exempt under sixth Schedule to the
Sales tax Act, 1990.
2. As the zero rated supplies are less than 50% of all taxable supplies under SRO 647(I)/2007 dated June 27, 2007,
therefore 90% limitation is applicable U/S 8B of the Sales Tax Act, 1990.
3. It has been assumed that 2% further tax is applicable on local taxable supplies to unregistered persons and the same
shall be accounted for and paid separately without adjustment of the same against input tax / refund of the registered
person and further it shall also not be considered for the computation of 90% limitation on output tax.
4. As purchases from non-registered persons are without sales tax invoices hence the same shall be without sales tax
u/s 23(2) of the Sales Tax Act, 1990, therefore the question of adjustment of input tax in not applicable.

Q.10 Mr. Hashmi is registered as wholesaler-cum-retailer under the Sales Tax Act, 1990. Data regarding his business is
given below for the month of August 2015:
Rs.
1 Taxable supplies to registered person 400,000
2 Taxable supplies to non-registered person 28,000
4 Taxable purchases from registered person 150,000
5 Taxable purchases from non-registered person 290,000
6 Taxable purchases from wholesaler 60,000
7 Sales tax paid on PTCL dues 6,000
8 Sales tax paid on electricity bill 5,000
9 General sales tax paid on Sui Gas bill 6,000
Note: Sales tax registration number is printed on utility bills. Calculate his sales tax liability for August 2015.

Solution
Rs.
Output tax
On taxable supplies to registered person U/S 3 (Rs. 400,000 x 17%) 68,000
On taxable supplies to non-registered person U/S 3 (Rs. 28,000 x
17%) (Note 2) 4,760
72,760
Less: Input tax
On taxable purchases from registered person (Rs. 150,000 x 17%) 25,500
On taxable purchases from non-registered person (Note 3) 0
On taxable purchases from wholesaler (Rs. 60,000 x 17%) 10,200
On PTCL dues (Note 4) 0
On electricity bills paid (Note 4) 0
On sui gas bills paid (Note 4) 0
Total input tax 35,700
Balance sales tax payable 37,060
(Note 1) As it is case of a wholesaler-cum-retailers under SRO 647(I)/2007 dated June 27, 2007, therefore 90% limitation
is not applicable U/S 8B of the Sales Tax Act, 1990.
(Note 2) It has been assumed that 2% further tax is not applicable on local taxable supplies to unregistered persons under
SRO 648(I)/2013 dated July 09, 2013 otherwise further tax shall be accounted for and paid separately without adjustment of
the same against input tax / refund of the registered person and further it shall also not be considered for the computation of
90% limitation on output tax.
(Note - 3) As purchases from non-registered persons are without sales tax invoices hence the same shall be without sales
tax u/s 23(2) of the Sales Tax Act, 1990, therefore the question of adjustment of input tax in not applicable.
(Note - 3) A registered person is not entitled to claim or deduct the input tax paid on electricity, telephone and gas bills that
are not being the direct constituent and integral part of the taxable goods produced, manufactured or supplied during the
course or in the furtherance of any taxable activity under SRO 1307(I)/97 dated December 20, 1997.

610 Conceptual Approach to Taxes


Practice Questions with Solutions Chapter-06

Q.11 Mr. Usman is a registered commercial importer. Data for his business for the month of August 2015 is as follows:
Rs.
1 Taxable supplies to registered person 402,130
2 Taxable supplies to non-registered person 94,013
(including the amount of sales tax)
3 Supplies to DTRE registered person 192,993
4 Zero rated supply 607,785
5 Taxable purchases from registered person 209,812
6 Exempted purchases 35,912
7 Imported goods 337,100
8 Sales tax paid on gas bill- consumed 18,000
9 Sales tax paid on electricity bills 20,000
(Sales tax registration number is printed on)
10 Previous month credit b/f 2,000

Required: Calculate sales tax payable.

Solution
Rs.
Output tax
On taxable supplies to registered person U/S 3 (Rs. 402,130 x 17%) 68,362
On taxable supplies to non-registered person U/S 3 (Rs. 94,013 x 17
/ 117) (Note 4) 13,660
Supplies to DTRE registered person [U/S 4 read with 5th Schedule] 0
Zero rated supplies [U/S 4 read with 5th Schedule] 0
82,022
Input tax
On apportioned local supplies (working attached) 38,759
Of previous month credit b/f 2,000
Balance sales tax payable 41,263
Sales tax refundable in respect of zero rated supplies / DTRE supplies
(Rs.15,504 + 48,825) 64,329

Working
Taxable purchases from registered person (Rs. 209,812 x 17%) 35,668
Exempted purchases 0
Imported goods U/S 3 (Rs. 337,100 x 17%) 57,307
Additional tax on commercial imports (Rs. 337,100 x 3%) 10,113
Sales tax paid on gas bill- consumed (Note 5) 0
Sales tax paid on electricity bills (Note 5) 0
Total input tax to be apportioned 103,088

Apportionment of taxable supplies input tax


[U/R 25 of the Sales Tax Rules, 2006] Supplies Input tax
Rs. Rs.
Taxable supplies to registered person 402,130
Taxable supplies to non-registered person (94,013 x 100/117) 80,353
Total taxable local supplies 482,483 38,759
Supplies related to DTRE and zero rated 800,778 64,329
1,283,261 103,088

Note: Limitation of 90% of output tax U/S 8B shall not be applicable because of fulfillment of the following two conditions
under SRO 647(I)/2013 dated June 27, 2013.
1. Commercial imports are more than 50% of total taxable purchases.
2. Zero rated supplies (including sales to DTRE registered persons) are more than 50% of all taxable supplies.
Although input sales tax credit is available with the registered person to be considered for 90% limitation but where no
such limitation is applied the same shall be fully adjusted against the ultimate sales tax liability of the person. The
unadjusted balance of sales tax if any shall be carried forward for future adjustment.
4. It has been assumed that 2% further tax u/s 3(1A) is not applicable on local taxable supplies to unregistered persons
by virtue of SRO 648(I)/2013 dated July 09, 2013 otherwise further tax shall be accounted for and paid separately
without adjustment of the same against input tax / refund of the registered person and further it shall also not be
considered for the computation of 90% limitation on output tax.

Conceptual Approach to Taxes 611


Practice Questions with Solutions Chapter-06

5. A registered person is not entitled to claim or deduct the input tax paid on gas and electricity bills that are not being
the direct constituent and integral part of the taxable goods produced, manufactured or supplied during the course or
in the furtherance of any taxable activity under SRO 1307(I)/97 dated December 20, 1997.

Q.12 Mr. Bilal Idrees is a registered manufacturer under the Sales Tax Act, 1990. Data regarding his business is given
below:
Rs.
1 Taxable supplies to registered person 500,000
2 Taxable supplies to non-registered person 180,000
3 Sales to retailers 50,000
4 Supplies donated to recognized institution 15,000
5 Taxable purchases from registered person 220,000
6 Taxable purchases from non-registered person 190,000
7 Purchased raw material (used on taxable and exempt supplies) 200,000
8 Withholding tax deducted on sales 10,000
9 Sales tax paid on PTCL dues 6,000
10 Sales tax paid on factory electricity bills 5,000
Note: Sales tax registration number is printed on utility bills.
Required: Calculate his sales tax liability for the month of September 2015.

Solution
Rs.
Output tax
On taxable supplies to registered person U/S 3 (Rs. 500,000 x 17%) 85,000
On taxable supplies to non-registered person U/S 3(Rs. 180,000 x
17%) (Note - 2) 30,600
Sales to retailers U/S 3 (Rs. 50,000 x 17%) 8,500
th
Supplies donated to recognized institution (Exempt under 6 Schedule) 0
124,100
Input tax
On taxable supplies (working attached) (A) 80,741
90% of output (Rs. 124,100 x 90%) (B) 111,690
Less: Admissible input tax: lower of (A) or (B) 80,741
43,359
Withholding tax on supplies 10,000
Balance sales tax payable 33,359
Working
Taxable purchases from registered person (Rs. 220,000 x 17%) 37,400
Taxable purchases from non-registered person (Note - 3) -
Purchased raw material (used taxable and exempt supplies) (Rs.
200,000 x 17%) 34,000
Sales tax paid on PTCL dues 6,000
Sales lax paid on factory electricity bill 5,000
Total input tax to be apportioned 82,400

Apportionment of residual input tax Residual input


[U/R 25 of the Sales Tax Rules, 2006] Supplies tax
Rs. Rs.

Taxable supplies 730,000 80,741


Exempt supplies 15,000 1,659
745,000 82,400

612 Conceptual Approach to Taxes


Practice Questions with Solutions Chapter-06

(Note - 1) It is assumed that all the purchases are used for both taxable as well as exempt supplies
(Note 2) It has been assumed that 2% further tax u/s 3(1A) is not applicable on local taxable supplies to unregistered
persons by virtue of SRO 648(I)/2013 dated July 09, 2013 otherwise further tax shall be accounted for and paid separately
without adjustment of the same against input tax / refund of the registered person and further it shall also not be considered
for the computation of 90% limitation on output tax.
(Note 3) As purchases from non-registered persons are without sales tax invoices hence the same shall be without sales
tax u/s 23(2) of the Sales Tax Act, 1990, therefore the question of adjustment of input tax in not applicable.
(Note 4) As it is case of a manufacturer and none of the clauses under SRO 647(I)/2007 dated June 27, 2007 is applicable
therefore 90% of output tax limitation on input is applicable U/S 8B of the Sales Tax Act, 1990.

Q.13 Mr. Muhammad Shan is a registered manufacturer under the Sales Tax Act, 1990. Data regarding his business for
the month of August 2015 is as follows.
Rs.
1 Taxable supplies to registered persons on credit basis 3,500,000
2 Taxable supplies to non-registered persons from exempted purchases (with Sales Tax) 250,000
3 Sales to retailers 150,000
4 Exempted supplies (Out of purchases from non-registered persons) 600,000
5 Supplies to DTRE registered person 400,000
6 Zero rated supply 900,000
7 Supplies made for personal use (including the amount of sales tax) 120,000
8 Taxable purchases from registered on credit basis 300,000
9 Purchases from unregistered persons 250,000
10 Imported goods 150,000
11 Acquisition of fixed assets purchased from non-registered persons 800,000
12 Sales tax paid on gas bill consumed in residential colonies 20,000
13 Sales tax paid on electricity bills 50,000
14 Sales tax paid on telephone bills 50,000
15 Sales tax credit 20,000
Required: Calculate sales tax payable.

Solution
Rs.
Output tax
On taxable turnover to registered persons (Rs.3,500,000 x 17%) 595,000
On taxable supplies to non-registered persons (Rs. 250,000 x 17 /
117) (Note - 2) 36,325
On supplies to retailers (Rs. 150,000 x 17%) 25,500
Supplies to DTRE registered persons (U/S 4 read with 5th Schedule) 0
On zero rated supplies (U/S 4 read with 5th Schedule) 0
On exempted supplies (U/S 13 read with 6th Schedule) 0
On supplies made for personal use (Rs.120,000 x 17 / 117) 17,436
674,261
Input tax
On local supplies (Working attached) 116,248
Previous months credit 20,000
(A) 136,248
90% of output (Note - 1) (B) 606,349
Less: Admissible input tax: lower of (A) or (B) 136,248
Sales tax payable 538,013
Working
Sale tax refundable in respect of zero rated supplies / DTRE supplies attached 39,114

Working
Taxable purchases from registered on credit basis (Rs. 300,000 x 17%) 51,000
Exempted purchases 0
Imported goods (Rs. 150,000 x 17%) 25,500
Acquisition of fixed assets purchased from non-registered persons (Note 3) 0
Sales tax paid on gas bill consumed in residential colonies (Note 4) 0
Sales tax paid on factory electricity bills 50,000
Sales tax paid on factory telephone bills 50,000
Total input tax to be apportioned 176,500

Conceptual Approach to Taxes 613


Practice Questions with Solutions Chapter-06

Apportionment of residual input tax


[U/R 25 of the Sales Tax Rules, 2006] Supplies Residual input tax
Rs. Rs.

Local taxable supplies (Rs.3,900,000 Sales tax Rs. 36,325) 3,863,675

Total taxable local supplies 3,863,675 116,248


Exempted sales [Rs.600,000 + (120,000 x 100/177)] (including
supplies for personal use) 702,564 21,138
Supplies related to DTRE and zero rated 1,300,000 39,114
5,866,239 176,500
Notes:
1. As the zero rated supplies (including sales to DTRE registered persons) are less than 50% of all taxable supplies
under SRO 647(I)/2007 dated June 27, 2007, therefore 90% limitation is applicable U/S 8B of the Sales Tax Act,
1990.
2. It has been assumed that 2% further tax u/s 3(1A) is not applicable on local taxable supplies to unregistered persons
by virtue of SRO 648(I)/2013 dated July 09, 2013 otherwise further tax shall be accounted for and paid separately
without adjustment of the same against input tax / refund of the registered person and further it shall also not be
considered for the computation of 90% limitation on output tax.
3. As purchase of fixed assets are from non-registered persons are without sales tax invoices hence the same shall be
without sales tax u/s 23(2) of the Sales Tax Act, 1990, therefore the question of adjustment of input tax in not
applicable.
4. Input tax paid on gas bills of labor residential colonies is not admissible input tax under SRO 490(I)/20014 dated June
12, 2004.

614 Conceptual Approach to Taxes


Chapter 07 Solved Past Papers Sales Tax Numericals of CA Mod- C

Chapter

07
SOLVED PAST PAPERS SALES TAX NUMERICALS OF
MOD-C (2003 TO 2015)
Note: All the following questions have been solved under the Sales Tax Act effective from July 1st 2015.

Q.NO.7 Spring 2015 Bashir is registered under the Sales Tax Act, 1990 and is engaged in the business
of export and supply of consumer goods. Following information has been extracted from his records for
the month of February 2015.

Rupees
Supplies
To registered persons 25,980,000
To unregistered persons 2,500,000
Exempt supplies 1,874,000
Export to USA 2,000,000
Purchases
Purchases from registered person 21,710,000
Import of a machine 2,500,000

Following additional information is also available:


(i) supplies to registered persons include goods amounting to Rs. 300,000 which were supplied to an
associated company at a special discount of 25%.
(ii) input tax amounting to Rs. 55,900 was paid in January, 2015 but inadvertently it could not be claimed in
the return for January 2015.
(iii) a registered supplier had supplied goods worth Rs. 500,000 to Bashir in February 2015. However,
Bashir did not receive the sales tax invoice from the supplier.
(iv) the imported machine was put into operation during February, 2015.
(v) sales tax credit of Rs. 410,000 is to be brought forward from January 2015.
Sales tax is payable at the rate of 17%. All the above amounts are exclusive of sales tax, wherever
applicable.
Required:
Under the provisions of the Sales Tax Act, 1990 and Rules made thereunder, compute sales tax

payable/refundable and input tax credit to be carried forward, if any, for tax period February 2015.

Solution

Mr. Bashir
Computation of Sales tax payable / refundable
For the period of February 2016

Output Tax
Sales to registered person 4,433,600
Sales to unregistered person 425,000

Conceptual Approach to Taxes_______________________________________________


_____________________________615
Chapter 07 Solved Past Papers Sales Tax Numericals of CA Module C

Exempt Supplies -
Export 2,000,000 x 0% -
4,858,600

Input tax
Residual Input tax W-1 3,175,291
Not claimed input tax 55,900
Sales tax credit 410,000
3,641,191
or 90% of 4,858,600 4,372,740 3,641,191
1,217,409
Add: Further Sales Tax 50,000
Less: Input tax on fixed assets (374,268)
Sales tax payable with return 893,140

Sales tax refundable 248,395

Apportionment of Input tax


Taxable Supplies Turnover Input tax Fixed Assets
Reg taxable 26,080,000 2,897,537 341,530
Unregd 2,500,000 277,755 32,739
Own Use - - -
wholly exempted 1,874,000 208,205 24,541
DTRE Person - -
Zero Rated 2,000,000 222,204 26,191
32,454,000 3,605,700 425,000

Q.NO.6 Autumn 2014 Ali Trading Company (ATC) is registered under the Sales Tax Act, 1990 and is engaged in the
business of manufacture and supply of consumer goods. Following information has been extracted from the
records of ATC for the month of August 2014.

Rupees
Supplies
Local supplies to wholesalers 14,500,000
Local supplies to distributors 10,254,980
Exports 18,650,000
Local supplies to registered retailers 980,000
Supply of exempted goods 5,500,000

Purchases
Local purchases from registered persons 50,982,000
Local purchases from un-registered persons 9,200,000

Following additional information is also available:


(i)Supplies amounting to Rs. 540,000 were returned by registered retailers.
(ii)An early settlement discount of Rs. 250,000 was given to local distributors.
(iii) An amount of Rs. 500,000 was received from Imran Associates, representing 25% advance payment in respect
of supply of a special order. ATC will supply this order in November 2014.
(v) Goods pledged with a bank, were disposed of by the bank for Rs. 4 million in satisfaction of debt owed
by ATC.
(vi)Sales tax credit brought forward from previous month amounted to Rs. 854,700.

616 Conceptual Approach to Taxes


Chapter 07 Solved Past Papers Sales Tax Numericals of CA Mod- C

(vii) Proper debit and credit notes have been issued wherever necessary.

Sales tax is payable at the rate of 17%. All the above figures are exclusive of sales tax.

Required:
Under the provisions of the Sales Tax Act, 1990 compute sales tax payable/refundable and input tax
credit to be carried forward, if any, for August 2014.

Solution

Ali Trading Corporation


Computation of Sales tax payable / refundable
For the tax period August 2015

Output Tax Rs. Rs.

Local supplies to wholesalers 14,500,000 2,465,000


Local supplies to distributors 10,254,980 1,743,347
Exports 18,650,000 -
Local supplies to registered retailers 980,000 166,600
Supply of exempted goods 5,500,000 -
Advance received 500,000 85,000
Goods disposed pledged with bank 4,000,000 680,000
Return 540,000 (91,800)
- 5,048,147
Input tax
Input tax admissible 4,782,618
Sales tax credit brought forward 854,700
5,637,318
or 90 % of 5,048,147 4,543,332
(4,543,332)
Sales tax payable with return 504,815

Sales tax to be carried forward 1,093,986

Sales tax refundable on zero rated supplies 2,999,694

Input tax admissible to be apportioned


Purchases from registered persons 50,982,000 8,666,940

Apportionment of Input tax:

Taxable Supplies 29,734,980 4,782,618


Exports 18,650,000 2,999,694
Exempted goods 5,500,000 884,628
53,884,980 8,666,940

Q. NO. 9 Spring 2014 Zaheer is registered under the Sales Tax Act, 1990. He is engaged in the manufacture and

supply of spare parts. Following information has been extracted from the records for the month of February 2014.

Local purchases of raw-material: Rupees

Conceptual Approach to Taxes_______________________________________________


_____________________________617
Chapter 07 Solved Past Papers Sales Tax Numericals of CA Module C

from registered seppliers 23,000,000


from un-registered seppliers 9,000,000
Supplies of manufactured goods:
local taxable supplies to registered persons 12,000,000
local taxable supplies to un-registered persons 4,000,000
local exempt supplies to registered persons 3,000,000
exports to Malaysia 11,000,000

Following additional information is also available:


(i) Raw materials of Rs. 1.5 million were purchased from a registered supplier. Invoice was received on 15

November 2013. However, the input tax on this invoice could not be claimed in the relevant period.
(ii) Taxable supplies amounting to Rs. 1.2 million were returned by different customers. Proper debit /
credit notes were raised in respect of such supplies.
(iii) Raw materials purchased from registered suppliers include an amount of Rs. 2.5 million against which
100% advance was paid in the month of January 2014. However, due to a dispute, sales tax invoice was
delayed and was received by the company after filing of return.
(iv) Sales tax credit of Rs. 1.2 million was brought forward from previous month.

Sales tax is payable at the rate of 17%. All the above figures are exclusive of sales tax.

Required:
Compute the sales tax payable by or refundable to Zaheer along with input tax to be carried forward, if any, in the
sales tax return for the month of February 2014.

Solution: Mr. Zaheer


Computation of Sales tax
For the month of February 2016

Rs. Rs.
Output tax on:

Supplies to registered persons U/S 3 (12,000,000 x 17%) 2,040,000


Less: credit note (U/S 9 read with Rule 22) (1,200,000 x 17%) (204,000)
Supplies to un-registered persons U/S 3 ( 4,000,000 x 17%) 680,000
Exempt supplies (U/S 13 read with 6th Schedule) -
Zero rated exports (U/S 4 read with 5th Schedule) -
(A) 2,516,000
Input tax on:

Input tax (W - 1) 2,221,333


Brought forward 1,200,000
(B) 3,421,333

90% of output tax (N - 1) (Rs. 2,720,000 x 90%) (C) 2,264,400


Less: Admissible input tax: Lower of (B) or (C) (D) 2,264,400
Balance sales tax payable [A less D] 251,600
2% further sales tax on supplies to unregistered persons (N - 2) 80,000

Sales tax carried forward [B less C] 1,156,933


Tax Refund (W - 1) 1,527,167

618 Conceptual Approach to Taxes


Chapter 07 Solved Past Papers Sales Tax Numericals of CA Mod- C

Working of residual input tax:

Purchase from regitered person ( 23,000,000 x 17 %) 3,910,000


Raw material on which input not claimed ( 1,500,000 x 17%) 255,000
4,165,000
(W-1)

APPORTIONMENT OF RESIDUAL INPUT TAX:


Supplies Residual input tax
Rs. Rs.

Supplies taxable to registered persons 12,000,000


Supplies taxable to un-registered persons 4,000,000
16,000,000 2,221,333
Supplies - exempt (N - 3) 3,000,000 416,500
Supplies export 11,000,000 1,527,167
30,000,000 4,165,000

(N - 1) As the zero rated supplies are less than 50% of all taxable supplies under SRO 647(I)/2007 dated June 27,
2007, therefore 90% limitation is applicable U/S 8B of the Sales Tax Act, 1990.

(N - 2) It has been assumed that 2% further tax is applicable on local taxable supplies to unregistered persons hence
further tax shall be accounted for and paid seperately without adjustment of the same against input tax / refund
of the registered person and further it shall also not be considered for the computation of 90% limitation on
output tax.

(Note - 3) A registeed person is not entitled to claim input tax attirbutable to exempt supplies under section 8(2) of the
Sales tax Act, 1990.

(Note - 4) A registred person is not entitled to deduct input tax from output tax for taxable supply without holding a sales
tax invoice under section 7(2)(i) of the Sales tax Act, 1990.
Where a registered person has not deducted input tax in the relevant tax period, he may claim such tax in the

return for any of the six subsequent tax periods under proviso of section 7(1) of the Sales Tax Act, 1990.
In view of the above no adjustment of 100% advance made against purchases in the preceeidng tax period has
been made in the current tax period.

Q.No. 7 Autumn 2013

Faizan is registered under the Sales Tax Act, 1990 and is engaged in the business of manufacture and supply of
engineering goods. Following information has been extracted from his records for the month of August 2013.

(i) Supplies made during the month were as follows:

Discounts
Gross amount Net amount
allowed
-------------------- Rupees --------------------
Local supplies to registered persons
- Noori Limited 16,000,000 800,000 15,200,000
- Soori Limited 4,000,000 400,000 3,600,000
Local supplies to unregistered persons 4,200,000 210,000 3,990,000
Exports to Jordan 6,000,000 - 6,000,000

(ii) Faizan normally allows 5% discount to all its customers. However, as a special case, a discount of 10%
was allowed to Soori Limited. All the discount were shown on the invoice.

Conceptual Approach to Taxes_______________________________________________


_____________________________619
Chapter 07 Solved Past Papers Sales Tax Numericals of CA Module C

(ii) Supplies worth Rs. 617,500 (net of discount ) were returned by Noori Limited. Proper debit and credit
notes were issued in this regard.

(iv) Following purchases were made during the month:


Rupees
Local materials from registered persons 27,000,000
Local materials from unregistered persons 3,000,000

(v) Records indicate that a pump and a motor were given to Fizan's friend, free of cost. The list price of the
pump and motor was Rs. 33,000.

(vi) Faizan is required to pay a penalty of Rs. 10,000 under the Sales Tax Act, 1990 on account of certain
defects in the maintenance of records.

(vii) Sales tax credit brought forward from previous month amounted to Rs. 850,280.

(viii) Sales tax is payable at the rate of 17% All the above figures are exclusive of sales tax.

Required:
Compute the sales tax payable by or refundable to Faizan along with input tax to be carried forward, if any, in the sales
tax return for the month of August 2013.

Solution

Mr Faizan
Sales Tax Liability
Tax Period: August 2015

Rupees
Output tax:
Sales tax
Rs. Rate of sales tax
Rs.
on local supplies to registered persons
- Noori Limited U/S 3 15,200,000 17% 2,584,000
- Soori Limited (value after 5% discount) U/S 3 3,800,000 17% 646,000
On local supplies to unregistered persons (N - 4) 3,990,000 17% 678,300
On exports to Jordan [U/S 4 read with 5th Schedule] 6,000,000 0% -
On goods given to Friend, free of cost U/S 3 33,000 17% 5,610
Less: sale return [U/S 9 read with Rule 22] 617,500 17% (104,975)
29,640,500 3,808,935

Input tax:

On taxable supplies (N - 2) 3,641,097


Sales tax credit brought forward 850,280
(A) 4,491,377

90% of output tax(N - 3) (Rs.3,808,935 x 90%) (B) 3,428,042

Less: Admissible input tax: Lower of (A) or (B) 3,428,042


Balance sales tax payable for the period 380,894
Add: penalty 10,000
Total sales tax / penalty payable 390,894
2% further sales tax on supplies to un-registered persons (N - 4) 79,800

Sales tax carried forward [(A) - (B)] 1,063,336

Sales tax refundable on zero rated supplies (W-2) 948,903

(N - 1) Input tax
Input tax on Local purchases from registered persons (Rs. 27,000,000 x 17%) 4,590,000

620 Conceptual Approach to Taxes


Chapter 07 Solved Past Papers Sales Tax Numericals of CA Mod- C

(N - 2) Apportionment of input tax on taxable supplies


[U/R 25 of the Sales Tax Rules, 2006] Supplies Input tax
Rs. Rs.
Local taxable (Gross) 23,023,000 3,641,097
Exports 6,000,000 948,903
29,023,000 4,590,000

(N - 3) As the zero rated supplies are less than 50% of all taxable supplies under SRO 647(I)/2007 dated June 27,
2007, therefore 90% limitation is applicable U/S 8B of the Sales Tax Act, 1990.

(N - 4) It has been assumed that 2% further tax is applicable on local taxable supplies to unregistered persons hence
further tax shall be accounted for and paid seperately without adjustment of the same against input tax / refund
of the registered person and further it shall also not be considered for the computation of 90% limitation on
output tax.

Q.No. 7 (b) Spring 2013

Mr. Clever a manufacturer of household appliances, is registered under the Sales Tax Act, 1990. Following
information has been extracted from its records for the month of February 2013:

Supplies Rupees
Local supplies of manufactured good to registered persons 26,860,000
Local supplies of manufactured good to unregistered persons 3,550,000
Local supplies of zero-rated goods 1,250,000
Exports to Malaysia 15,000,000
Local purchases
Registered persons 40,550,000
Unregistered persons 5,000,000

Following additional information is also available:

(i) Supplies worth Rs. 1,300,000 were returned by different registered persons. Proper debit/ credit
notes were raised within the specified time.

(ii) Local purchase from registered person include and invoice Re. 60,000 which was issued in the name
of Mr. Clever's uncle.

(iii) A new machine amounting to Rs. 3,000,000 was imported from china and put into operation during
the same month.
(iv) Sales tax credit of Rs. 410,000 was brought forward from pervious month.
Sales tax is payable at the rate fo 16%. All the above amounts are exclusive of sales tax.

Required:
Compute the sales tax payable by /refundable to Mr. Clever along with input tax to be carried forward, if any, in the
sales tax return for the month of February 2013.

Solution
Sales tax
Output tax Rs. Rs.

Local taxable supplies to registered persons U/S 3 26,860,000 4,566,200


Local taxable supplies to unregistered persons (Note - 2) 3,550,000 603,500
Zero rated supplies - local [U/S 4 read with 5th Schedule] -
Zero rated supplies - export [U/S 4 read with 5th Schedule] 1,250,000 -
Exports to Malaysia [U/S 4 read with 5th Schedule] 15,000,000 -
Total output tax 5,169,700

Conceptual Approach to Taxes_______________________________________________


_____________________________621
Chapter 07 Solved Past Papers Sales Tax Numericals of CA Module C

Less sales returns [U/S 9 read with Rule 22] 1,300,000 (221,000)
4,948,700
Input tax
Input tax apportioned to local taxable supplies (Working given) 4,818,479
Add sales tax credit brought forward 410,000

Total input (A) 5,228,479


90% of output tax (N - 1) (Rs. 4948,700 x 17%] (B) 4,453,830
Less: Admissible input tax: lower of (A) or (B) 4,453,830
Input tax on fixed assets (30,410,000 / 46,660,000 x 510,000) 332,385
Sales tax payable 162,485
2% further sales tax on supplies to un-registered persons (N - 2) 71,000

Input to be carried forward [(A) without fixed assets admissible input - (B)]
[Rs.4,896,094 - Rs.4,453,830] 442,264

Sales tax refundable in respect of exports U/S 10(1) (Working attached) 2,574,821

Input tax to be apportioned:


Purchases from registered persons
(Rs. 40,550,000 - Rs.60,000 = 40,490,000) x 17% 6,883,300
Purchases from unregistered persons [Rs. 5,000,000] (N - 3) -
Fixed assets (machine) [Rs. 3,000,000 x 17%] = (N - 1) 510,000
Total input tax to be apportioned 7,393,300

Apportionment of input tax of taxable supplies Supplies Input tax


[U/R 25 of the Sales Tax Rules, 2006] Rs. Rs.

Local supplies Allowable 30,410,000 4,818,479


Zero rated Refundable 16,250,000 2,574,821
46,660,000 7,393,300

(Note - 1) As the zero rated supplies are less than 50% of all taxable supplies under SRO 647(I)/2007 dated June 27,
2007, therefore 90% limitation is applicable U/S 8B of the Sales Tax Act, 1990. The said limitation is not
applicable on input tax paid on acquistion of fixed assets.

(Note - 2) It has been assumed that 2% further tax u/s 3(1A) applicable on local taxable supplies to unregistered persons
under SRO 648(I)/2013 dated July 09, 2013 hence further tax shall be accounted for and paid seperately
without adjustment of the same against input tax / refund of the registered person and further it shall also not be
considered for the computation of 90% limitation on output tax.

(Note - 3) As purchases from non-registered persons are without sales tax invoices hence the same shall be without
sales tax u/s 23(2) of the Sales Tax Act, 1990, therefore the question of adjustment of input tax is not
applicable.

Q.NO. 9 Autumn 2012

Zainab is registered under the Sales Tax Act, 1990 and is engaged in the manufacture and supply of Products A and B.
Following information has been extracted from her records for the month of August 2012:

Product A Product B
Rs. Rs.
Supplies
Local supplies 5,350,000 1,010,000
Export to Thailand 2,550,000 3,950,000
Purchases
Local materials from registered persons 6,000,000
Local materials from unregistered persons 850,000

622 Conceptual Approach to Taxes


Chapter 07 Solved Past Papers Sales Tax Numericals of CA Mod- C

Additional information

(i) Product A is exempt from the charge of sales tax


(ii) Sales tax credit brought forward from previous month amounted to Rs. 262,500.
(iii) Substandard supplies worth Rs. 150,000 were returned to the registered vendors and proper debit and credit notes were
issued.
(iv) A purchase invoice dated 5 February 2012 amounting to Rs. 100,000 had not been claimed inadvertently. This oversight
was detected in the month of August 2012.

Required:

In the light of Sales Tax Act, 1990 and Rules made thereunder, calculate the following for the month of August 2012:
(a) Sales tax payable / refundable
(b) Input tax to be carried forward, if any

Solution

(a)
Sales tax
Rs. Rs.
Output tax
Local taxable supplies [U/S 3 (Rs. 1,110,000 x 17%] 188,700
Local exempt supplies [U/S 13 read with 6th Schedule] -
Exports (zero rated) [U/S 4 read with 5th Schedule] -
Total output tax 188,700

Less: Input tax allocated to taxable supplies (Note - 1) 86,633


Input tax brought forward from previous month 262,500
Total input tax 349,133

Input tax to be carried forward (160,433)

(b) Sales tax refundable on export sales U/S 10(1) (Note - 1) 507,311

(Note -1) Apportionment of residual input tax


[U/R 25 of the Sales tax Rules, 2006] Supplies Residul input
tax
Rs. Rs.
Local taxable supplies (Product B) 1,110,000 86,633
Local exempt supplies (Product A) (Note - 3) 5,350,000 417,556
Exports (zero rated) 6,500,000 507,311
12,960,000 *1,011,500

* Input tax on purchases from registered persons i.e. [(6,000,000-150,000+100,000) x 17%] = 1,011,500

(Note - 2) As the zero rated supplies are more than 50% of all taxable supplies under SRO 647(I)/2007 dated June 27,
2007, therefore 90% limitation is not applicable U/S 8B of the Sales Tax Act, 1990.

(Note - 3) A registeed person is not entitled to claim input tax attirbutable to exempt supplies under section 8(2) of the
Sales tax Act, 1990.

Q.NO. 6 Autumn 2011

Mr. Agha is registered under the Sales Tax Act, 1990. He is engaged in the supply of household appliances and has
provided you the following information for the month of August 2011:

(i) Supplies made during the month were as follows:


Rupees
Local taxable supplies to registered persons 35,500,000
Local taxable supplies to unregistered persons 1,700,250
Exports to USA and Canada 25,500,000
Supplies of exempt goods 5,235,000

Conceptual Approach to Taxes_______________________________________________


_____________________________623
Chapter 07 Solved Past Papers Sales Tax Numericals of CA Module C

Goods worth Rs. 1,500,000 were returned by a registered person. Proper debit/credit notes have been issued in
this regard.

(ii) Following purchases were made during the month:


Rupees
Purchases from registered persons 54,550,000
Purchases from unregistered persons 10,600,000

Goods purchased from unregistered persons were exclusively used for making taxable supplies. An amount of Rs.750,000
is payable to a registered person since February 01, 2011.

(iii) Sales tax credit of Rs. 610,000 has been brought forward from previous month.
(iv) All the above amounts are exclusive of sales tax.
(v) Agha is also required to pay a penalty of Rs. 10,000 under the Sales Tax Act, 1990 on account of certain defects in the
maintenance of records.

Required:
Compute the sales tax payable/(refundable) by/to Mr. Agha along with input tax to be carried forward, if any, in the sales tax
return for the month of August 2011.

Solution
Rs. Rs.
Output tax
Local taxable supplies to registered persons U/S 3 (Rs. 35,550,000 x 17%) 6,043,500
Taxable supplies to unregistered persons U/S 3 (Rs.1,700,250 x 17%) (Note - 2) 289,043
Exports to USA and Canada [U/S 4 read with 5th Schedule] -
Supplies of exempt goods [U/S 13 read with 6th Schedule] -
Sales tax in respect of goods returned [U/S 9 read with Rule 22] (255,000)
Total output tax 6,077,543

Input tax
Input tax apportioned to local taxable supplies (Note - 1) 5,008,202
Add sales tax credit b/f 610,000
Total input (A) 5,618,202
90% of output tax (B) 5,469,788
Less: Admissible input tax: lower of (A) or (B) 5,469,788
Sales tax payable 607,754

Input tax to be carried forward (A) Less (B) 148,414


Sales tax refundable in respect of exports U/S 10(1) (Note - 1) 3,433,019

Input tax to be apportioned:


Purchases from registered persons (Rs. 54,550,000 x 17%) 9,273,500
Purchases from unregistered persons (Note - 4) -
Less : sales tax inadmissible U/S 73 (Note - 5) (127,500)
Total input tax to be apportioned 9,146,000

(Note - 1) Apportionment of residual input tax Supplies Residul input


[U/R 25 of the Sales tax Rules, 2006] tax
Rs. Rs.
Local supplies 37,200,250 5,008,202
Exports 25,500,000 3,433,019
Exempt supplies (Note - 6) 5,235,000 704,779
67,935,250 9,146,000

(Note - 2) As the zero rated supplies are less than 50% of all taxable supplies under SRO 647(I)/2007 dated June 27,
2007, therefore 90% limitation is applicable U/S 8B of the Sales Tax Act, 1990.

(Note - 3) It has been assumed that 2% further tax u/s 3(1A) is not applicable on local taxable supplies to unregistered
persons under SRO 648(I)/2013 dated July 09, 2013 otherwise further tax shall be accounted for and paid
seperately without adjustment of the same against input tax / refund of the registered person and further it shall

624 Conceptual Approach to Taxes


Chapter 07 Solved Past Papers Sales Tax Numericals of CA Mod- C
seperately without adjustment of the same against input tax / refund of the registered person and further it shall
also not be considered for the computation of 90% limitation on output tax.

(Note - 4) As purchases from non-registered persons are without sales tax invoices hence the same shall be without
sales tax u/s 23(2) of the Sales Tax Act, 1990, therefore the question of adjustment of input tax is not
applicable.

(Note - 5) A registeed person is not entitled to claim input tax U/S 73 where the payment against invoices exceeding Rs.

50,000 has not been made within 180 days of the issuance of tax invoices unless condonaion of time limit

approval from the of Board has been obtained U/S 74 of the Sales Tax Act, 1990.

(Note - 6) A registeed person is not entitled to claim input tax attirbutable to exempt supplies under section 8(2) of the
Sales tax Act, 1990.

Q.NO. 5 Spring 2011

Maroof Engineering Limited (MEL) is registered under the Sales Tax Act, 1990. The company is engaged in the
manufacture & supply of spare parts. Following information has been extracted from the records of MEL for February 2011
month.
Rupees
Purchases:
Local Material:
from registered suppliers 15,000,000
from un-registered suppliers 8,000,000
Supplies:
Manufactured goods:
local taxable supplies to registered persons 10,000,000
local taxable supplies to un-registered persons 3,000,000
export to Taiwan 10,000,000
exempt goods 2,000,000

Following additional information is also available.


(i) Purchases from registered suppliers include an amount of Rs. 1.0 million which was invoiced on May 15, 2010. The
input tax on this invoice could not be claimed in the relevant period.

(ii) Material purchased from un-registered suppliers was exclusively used for making taxable supplies.
(iii) Goods worth Rs. 500,000 were returned by different customers. Proper debit/credit notes were raised within the
specified period.

(iv) A new machinery of Rs 2.4 million was purchased and put to use during the same month.

(v) Rs. 20,000 was paid to a courier company for delivering gifts to MELs high value customers.
(vi) MELs purchases from registered suppliers include material worth Rs. 2 million against which an advance was paid in
the month of January 2011. However, due to a dispute, sales tax invoice was delayed and was received by the company
after filing of return.

(vii) Parts worth Rs. 15,000 were delivered to the CEO for his personal use, free of cost.
(viii) Sales tax credit of Rs. 50,000 was brought forward from previous month.

All the above amounts are exclusive of sales tax.

Required:
(a) Compute the sales tax payable/refundable.
(b) Input tax credit to be carried forward, if any.

Solution
Rs. in 000 Rs. in 000
Taxable Value Sales Tax
Sales Tax Credit (Input Tax)

Local purchases:
From registered persons (Rs. 15.0 m Rs. 3.0 m) 12,000 2,040

Conceptual Approach to Taxes_______________________________________________


_____________________________625
Chapter 07 Solved Past Papers Sales Tax Numericals of CA Module C

From un-registered persons (Note - 5) 8,000 -


Fixed assets (Machinery) (Note - 2) 2,400 408
Courier charges (Not admissible being not related to production / purchases) 20 -

Input tax attributable to both taxable and exempt goods 2,448


Less: Inadmissible / un-adjustable input tax (Note - 1) (1,174)
Input tax for the month 1,274
Add previous month credit brought forward 50
Accumulated credit 1,324

Sales Tax Debit (output tax)

Domestic supplies of manufactured goods:


to registered persons U/S 3 10,000 1,700
to unregistered persons U/S 3 (Note - 2) 3,000 510
Export to Taiwan [U/S 4 read with 5th Schedule] 10,000 -
Exempt goods [U/S 13 read with 6th Schedule] 2,000 -
Parts provided to the CEO U/S 3 15 3
Output tax for the month 2,213

Less: Sales return [U/S 9 read with Rule 22] 500 85


Debit for the month 2,128

On local supplies [Rs. 1,324 - Rs. 212 (408/ 25,015 x 13,015)] (A) 1,112
90% of output tax (Rs. 2,128 x 90%)= (B) 1,915

Less: Admissible input tax: lower of (A) or (B) 1,112


Input portion related to fixed assets used on taxable supplies without 90% limitation 212
Balance sales tax payable 804

Sales tax refund on export sales U/S 10(1) (Note - 1) 979

(Note - 1) Apportionment of residaul input tax


[U/R 25 of the Sales Tax Rules, 2006]
Rs. in 000
Total residual input tax 2,448

Allocation of residual input tax


[U/S 25 of the Sales Tax Rules, 2006] Residual input tax
Rs. Rs.

Taxable supplies 13,015 1,274


Exempt supplies (Note - 4) 2,000 196
Zero rated supplies 10,000 979
25,015 2,448

(Note - 2) As the zero rated supplies are less than 50% of all taxable supplies under SRO 647(I)/2007 dated June 27,
2007, therefore 90% limitation is applicable U/S 8B of the Sales Tax Act, 1990. The said limitation is not
applicable on input tax paid on acquistion of fixed assets.

(Note - 3) It has been assumed that 2% further tax u/s 3(1A) is not applicable on local taxable supplies to unregistered
persons under SRO 648(I)/2013 dated July 09, 2013 otherwise further tax shall be accounted for and paid
seperately without adjustment of the same against input tax / refund of the registered person and further it shall
also not be considered for the computation of 90% limitation on output tax.

(Note - 4) A registeed person is not entitled to claim input tax attirbutable to exempt supplies U/S 8(2) of the Sales Tax
Act, 1990.

(Note - 5) As purchases from non-registered persons are without sales tax invoices hence the same shall be without

626 Conceptual Approach to Taxes


Chapter 07 Solved Past Papers Sales Tax Numericals of CA Mod- C

sales tax u/s 23(2) of the Sales Tax Act, 1990, therefore the question of adjustment of input tax is not
applicable.

Q.NO. 6 Autumn 2010

Abdul Ghaffar is registered as a manufacturer, under the Sales Tax Act, 1990. He carried out the following activities during
the month of August 2010:

Rs. in 000
Supplies
Manufactured goods
Local - taxable goods 22,000
Local - exempt goods 3,000
Exports 5,000
Commercial goods 14,000

Purchases
Local purchases of raw material 8,000
Import of raw material 17,000
Commercial import of finished goods 10,000

Other relevant information is as follows:

(i) All the above amounts are exclusive of sales tax.

(ii) Commercial imports are stated at C and F value and are subject to customs duty at the rate of 15%.
(iii) In July 2010, an amount of Rs. 365,000 was carried forward as sales tax credit.

(iv) Sales tax is payable @ 16% except commercial imports which are charged @ 19%.

Required: Compute the following for the month of August 2010:


(a) Sales tax payable / refundable. (b) Input tax credit to be carried forward, if any.

Solution
Rs. in '000' Rs. in '000' Rs. in '000'
SALES TAX CREDIT (INPUT TAX) Taxable value Sales tax Sales tax

SALES TAX DEBIT (OUTPUT TAX)

On domestic supplies (manufactured goods U/S 3) 22,000 22,000 3,740


On exempt goods [U/S 13 read with 5th Sche.] 3,000 3,000 -
On supplies of imported goods U/S 3 14,000 14,000 2,380
On exports [U/S 4 read with 5th Schedule] 5,000 5,000 -
6,120
INPUT TAX

Commercial imports @ 20% (10,000 x 1.15) 11,500 2,300


Imports for domestic consumptions @ 17% 17,000 2,890
Local purchases @ 17% 8,000 1,360
Less: Inadmissible import & exempt supplies (Note - 1) (1,133)
Input tax for the month 5,417
Previous month credit brought forward 365
Accumulated credit (A) 5,782

90% of output tax (Rs. 6,120 x 90%) (B) 5,508

Less: Admissible input tax: Lower of (A) or (B) 5,508


Balance sales tax payable (Rs. 6,120 - Rs. 5,508) 612

Sales tax credit carried forward (A) less (B) 274

Sales tax refundabe on export sales U/S 10(1) (Note - 1) 708

Conceptual Approach to Taxes_______________________________________________


_____________________________627
Chapter 07 Solved Past Papers Sales Tax Numericals of CA Module C

(Note - 1) Apportionment of residual input tax Supplies Residual input tax


[U/R 25 of the Sales tax Rules, 2006] Rs. Rs.

Imports for domestic consumption 17,000 2,890


Local purchases 8,000 1,360
25,000 4,250

Total sales other than sales of commercial goods 30,000

Inadmissible input tax


Exempt supplies (3,000 / 30,000 x 4,250) 425
Exports (5,000 / 30,000 x 4,250) (Note - 3) 708
1,133

(Note - 2) As the zero rated supplies are less than 50% of all taxable supplies under SRO 647(I)/2007 dated June 27,
2007, therefore 90% limitation is applicable U/S 8B of the Sales Tax Act, 1990.

(Note - 2) As commercial imports subject to 3% additional tax are less than 50% of all taxable puchases under SRO

647(I)/2007 dated June 27, 2007, therefore 90% limitation is applicable U/S 8B of the Sales Tax Act, 1990.

(Note - 3) A registeed person is not entitled to claim input tax attirbutable to exempt supplies U/S 8(2) of the Sales Tax
Act, 1990.

Q.NO. 8 Spring 2010

Mr. Kaleem is registered under the Sales Tax Act, 1990 as a manufacturer as well as a commercial importer. He has
provided you the following information for the month of February 2010:
Rs. in million
Export sales manufactured goods 30
Local sales of exempt manufactured goods 20
Taxable supplies manufactured goods 120
Taxable supplies commercial imports 60

Purchases
Local purchases of raw material from:

Registered persons 160


Unregistered persons 50
Commercial imports 40

All the above amounts are exclusive of sales tax. Commercial imports have been stated at C and F value and are subject to

customs duty at the rate of 10%. There was no stock of commercial imports at the beginning or end of the month.

Required: Compute the sales tax liability of Mr. Kaleem along with input tax to be carried forward (if any) in his sales tax

return for the month of February 2010. (Ignore the feect of minimum value addition in case of commercial imports).

Solution
Rs. Rs.
Sales tax liability:
Output tax local manufactured taxable supplies U/S 3 (Rs. 120,000,000 x 17%) 20,400,000
Output tax local supplies out of commercial imports U/S 3 (Rs. 60,000,000 x 17%) 10,200,000
30,600,000

Input tax on commercial imports (44,000,000 x 17% without value addition) 7,480,000
Input tax against local taxable supplies (Note - 1) 19,200,000
(A) 26,680,000
90% of output tax (Rs. 30,600,000 x 90%) (B) 27,540,000
Less admissible input tax: lower of (A) or (B) 26,680,000

628 Conceptual Approach to Taxes


Chapter 07 Solved Past Papers Sales Tax Numericals of CA Mod- C

Balance sales tax payable 3,920,000

Sales tax refundable on exports U/S 10(1) (Note - 1) 4,800,000

Input tax:
Input tax on purchases from registered persons (Rs. 160,000,000 x 17%) 27,200,000
27,200,000

(Note - 1) Apportionment of residual input tax Supplies Residul input


[U/R 25 of the Sales tax Rules, 2006] tax
Rs. Rs.
Taxable local supplies 120,000,000 19,200,000
Exports 30,000,000 4,800,000
Exempt supplies (Note - 3) 20,000,000 3,200,000
170,000,000 27,200,000

(Note - 2) As the zero rated supplies are less than 50% of all taxable supplies under SRO 647(I)/2007 dated June 27,
2007, therefore 90% limitation is applicable U/S 8B of the Sales Tax Act, 1990.

(Note - 3) A registeed person is not entitled to claim input tax attirbutable to exempt supplies U/S 8(2) of the Sales Tax
Act, 1990.

Q. No. 8(b): Autumn 2009

Mr. Asif is registered under the Sales Tax Act, 1990. Following information for August 2009 month has been extracted from
his business records:

i. Supplies made during the month were as follows:


Rupees
To registered persons 5,000,000
To unregistered persons 3,000,000
Export supplies 11,000,000
Exempted supplies 2,000,000

ii. Goods costing Rs.8,000,000 were purchased from registered persons.


iii. Goods purchased from unregistered persons amounting to Rs.2,000,000 and were used exclusively for making
taxable supplies.

Required: Compute the sales tax payable and/or to be carried forward by Mr. Asif in the return for the month of August
2009.

Solution

Sales tax liability:


Rs. Rs.
Output tax U/S 3 (Rs. 8,000,000 x 17%) (Note - 2) 1,360,000
Less: input tax against taxable local supplies (Note - 1) 518,095
Balance sales tax payable 841,905

Sales tax refundable on exports U/S 10(1) (Note - 1) 712,381

Input tax:
Input tax on purchases from registered persons (8,000,000 x 17%) 1,360,000
1,360,000
(Note - 1) Apportionment of residual input tax
[U/R 25 of the Sales tax Rules, 2006] Supplies Residual input
tax
Rs. Rs.
Taxable local supplies 8,000,000 518,095
Exports 11,000,000 712,381

Conceptual Approach to Taxes_______________________________________________


_____________________________629
Chapter 07 Solved Past Papers Sales Tax Numericals of CA Module C

Exempt supplies (Note - 4) 2,000,000 129,524


21,000,000 1,360,000

(Note - 2) As the zero rated supplies are more than 50% of all taxable supplies under SRO 647(I)/2007 dated June 27,
2007, therefore 90% limitation is not applicable U/S 8B of the Sales Tax Act, 1990.

(Note - 3) It has been assumed that 2% further tax u/s 3(1A) is not applicable on local taxable supplies to unregistered
persons under SRO 648(I)/2013 dated July 09, 2013 otherwise further tax shall be accounted for and paid
seperately without adjustment of the same against input tax / refund of the registered person and further it shall
also not be considered for the computation of 90% limitation on output tax.

(Note - 4) A registeed person is not entitled to claim input tax attirbutable to exempt supplies U/S 8(2) of the Sales Tax
Act, 1990.

Q.NO.8 Spring 2009


Mr. Azad is a registered person and engaged in the manufacture of consumable goods. The following for the month of
February 2009 is available:

Value
excluding Sales Tax @ Value including
Sales Tax 16% Sales Tax
Rupees Rupees Rupees
Sales
- Taxable 6,000,000 960,000 6,960,000
- Exempt 4,000,000 0 4,000,000
10,000,000 960,000 10,960,000
Purchases
- Raw materials 9,200,000 1,472,000 10,672,000
- Fixed assets 1,700,000 272,000 1,972,000
10,900,000 1,744,000 12,644,000

Required: Work out sales tax liability of Mr. Azad along with input tax to be carried forward (if any) in his sales
tax return.

Solution

Sales tax liability: Rs. Rs.

Output tax 1,020,000


Input tax against taxable local supplies (Note - 1) (A) 938,400
90% of output tax (Rs.1,020,000 x 90%) (Note - 3) (B) 918,000
Less admissible input tax: lower of (A) or (B) 918,000
Balance sales tax payable 102,000

Input tax on fixed assets ( 289,000 x 6,000,000 / 10,000,000 )


(Restriction of 90% of output tax is not applicable, therefore considered separately) 173,400
Excess input tax paid c/f (71,400)

Sales tax credit carried forward (A) less (B) 20,400

Input tax:
Input tax on raw material 1,564,000
1,564,000

(Note - 1) Apportionment of residual input tax Supplies Residul input


[U/R 25 of the Sales tax Rules, 2006] tax
Rs. Rs.
Taxable local supplies 6,000,000 938,400
Exempt supplies (Note - 2) 4,000,000 625,600
10,000,000 1,564,000

630 Conceptual Approach to Taxes


Chapter 07 Solved Past Papers Sales Tax Numericals of CA Mod- C

(Note - 2) A registeed person is not entitled to claim input tax attirbutable to exempt supplies U/S 8(2) of the Sales Tax
Act, 1990.

(Note - 3) As the none of the clause of SRO 647(I)/2007 dated June 27, 2007 is applicable, therefore 90% limitation is
applicable U/S 8B of the Sales Tax Act, 1990.

Q.NO. 8 Spring 2007:

Mr. Adam is a registered person and engaged in the supply of various types of appliances for last many years. He has
provided you the following information for the month of February 2007:

i. Supplies made during the month amount to Rs. 95 million. Details of supplies made are as follows:
Rs. in million

Exports 50
Exempt supplies 10
Supplies to registered person 30
Supplies to unregistered person 5

ii. During the month, he has made an adjustment of Rs. 500,000 through credit note in a registered
persons balance.

iii. Following purchases were made during the month:

- From registered persons 45


- From unregistered persons 15

iv. All goods purchased from unregistered persons are exclusively used for making taxable supplies.

v. An amount of Rs. 3,000,000 is payable to a registered person since December 20, 2006. The input tax on the
purchase as accounted for in the relevant tax period.

vi. Mr. Adam is also required to pay the following:


Rupees

Arrears 500,000
Surcharge 70,000
Penalty 30,000
Compute Mr. Adams sales tax liability.

Solution

Sales tax liability:


Output tax U/S 3 (Rs. 35,000,000 x 17%) (Note - 3) 5,950,000
Less: credit note adjustment [U/S 9 read with Rule 22] (Rs. 500,000 x 17%) 85,000
5,865,000
Input tax (Note - 1) 2,818,421
3,046,579

Arrears 500,000
Surcharge 70,000
Penalty 30,000
Payable 600,000
Sales tax payable 3,646,579

Sales tax refundable on exports U/S 10(1) (Note - 1) 4,026,316

Input tax:
Input tax (45,000,000 x 17%) 7,650,000
7,650,000

(Note - 1) Apportionment of residual input tax Supplies Residul input


[U/R 25 of the Sales tax Rules, 2006] tax
Rs. Rs.

Conceptual Approach to Taxes_______________________________________________


_____________________________631
Chapter 07 Solved Past Papers Sales Tax Numericals of CA Module C

Taxable local supplies 35,000,000 2,818,421


Exports 50,000,000 4,026,316
Exempt supplies (Note - 4) 10,000,000 805,263
95,000,000 7,650,000

(Note - 2) As the zero rated supplies are more than 50% of all taxable supplies under SRO 647(I)/2007 dated June 27,
2007, therefore 90% limitation is not applicable U/S 8B of the Sales Tax Act, 1990.

(Note - 3) It has been assumed that 2% further tax u/s 3(1A) is not applicable on local taxable supplies to unregistered
persons under SRO 648(I)/2013 dated July 09, 2013 otherwise further tax shall be accounted for and paid
seperately without adjustment of the same against input tax / refund of the registered person and further it shall
also not be considered for the computation of 90% limitation on output tax.

(Note - 4) A registeed person is not entitled to claim input tax attirbutable to exempt supplies U/S 8(2) of the Sales Tax
Act, 1990.

Q.NO. 12 Autumn 2005

Following information is extracted from the records of M/s Rainbow Enterprises (Private) Limited. The information pertains
to the month of July 2005:

Rupees
Supplies to registered person 5,000,000
Supplies to unregistered person 1,500,000
Export Supplies 3,000,000
Purchase from registered suppliers 4,000,000
Purchase from unregistered suppliers 1,000,000
Sales of exempt goods 1,000,000

Examination of creditors ledger reveals that an amount of Rs.100,000 is still outstanding on account of the purchase made
from a registered supplier on January 12, 2005. The input tax on the said purchase was accounted for in the relevant tax
period.

Goods purchased from unregistered suppliers are exclusively used for making taxable supplies.

Required: Determine the amount of sales tax liability.

Solution

Sales tax liability: Sales tax


Rs. Rs.
Output tax U/S 3 (Rs. 6,500,000 x 17%) (Note - 3) 1,105,000
Input tax against taxable local supplies (Note - 1) (A) 410,429
90% of output tax (Rs.1,105,000 x 90%) (A) 994,500
Less admissible input tax: lower of (A) or (B) 410,429
Balance sales tax payable 694,571

Sales tax refundable on exports U/S 10(1) (Note - 1) 189,429

Input tax:
Input tax on purchases from registered persons (4,000,000 x 17%) 680,000
Less: Sales tax on purchase outstanding for more than 180 days (100,000 x 17%) 17,000
663,000

(Note - 1) Apportionment of residual input tax Supplies Residul input


[U/R 25 of the Sales tax Rules, 2006] tax
Rs. Rs.
Taxable local supplies 6,500,000 410,429
Exports 3,000,000 189,429
Exempt supplies (Note - 4) 1,000,000 63,143
10,500,000 663,000

(Note - 2) As the zero rated supplies are less than 50% of all taxable supplies under SRO 647(I)/2007 dated June 27,

632 Conceptual Approach to Taxes


Chapter 07 Solved Past Papers Sales Tax Numericals of CA Mod- C
2007, therefore 90% limitation is applicable U/S 8B of the Sales Tax Act, 1990.

(Note - 3) It has been assumed that 2% further tax u/s 3(1A) is not applicable on local taxable supplies to unregistered
persons under SRO 648(I)/2013 dated July 09, 2013 otherwise further tax shall be accounted for and paid
seperately without adjustment of the same against input tax / refund of the registered person and further it shall
also not be considered for the computation of 90% limitation on output tax.

(Note - 4) A registeed person is not entitled to claim input tax attirbutable to exempt supplies U/S 8(2) of the Sales Tax
Act, 1990.

Q.NO.10 Spring 2004

Following is the pertinent data relating to sales tax return of the company:

Output tax

Input tax Tax at the rate Further tax at the


claimable of 15% rate of 3% u/s 3(IA)
Rupees Rupees Rupees
Jul-03 1,050,000 1,210,000 310,000
Aug-03 1,450,000 1,175,750 150,000
Sep-03 1,130,000 1,375,000 225,000
Oct-03 1,200,100 1,300,750 175,050
Nov-03 1,050,000 1,250,700 210,000

(a) You are required to compute the sales tax payable alongwith the monthly sales tax return for the tax period July 2003,
August 2003 and September 2003 with brief explanatory notes, where relevant.

(b) With reference to data in (a) above, assuming that on October 20, 2003 it was found that input tax claim relating to tax
period July 2003 amounting to Rs. 50,000 was inadvertently not claimed in the sales tax return filed for that period. On
November 10, 2003, it was also found out that there was another input tax claim relating to tax period September 2003
amounting to Rs.25,000 which was not claimed in the monthly return filed for that period.

You are required to advise as to whether such unclaimed amounts could be claimed under the Sales Tax Act, 1990. If your
answer is in affirmative, then briefly explain the procedure for claiming such amounts.

(c) With reference to data (a) above, & disregarding the errors given in (b) above], assuming that in the month of November
2003, it was found that output tax of Rs. 1,300,750 shown in the monthly return for tax period October 2003 was infact Rs.
1,350,750 i.e. short declared by Rs. 50,000.

You are required to briefly explain the remedy, if any, available in the STA, 1990 to account for this error.

Solution
The question has been solved by assuming that the rate of output tax given in the question is 17% instead of 15%.

(a) September August July

Output tax (A) 1,375,000 1,175,750 1,210,000

Actual Input tax 1,130,000 1,450,000 1,050,000


Input tax brought forward 391,825 - -
Total input tax (B) 1,521,825 1,450,000 1,050,000

90% of output tax (90% x A above) (Note - 3) (C) 1,237,500 1,058,175 1,089,000
Less admissible input tax: lower of (B) or (C) 1,237,500 1,058,175 1,050,000
Balance sales tax payable 137,500 117,575 160,000
Input tax carried forward 284,325 391,825 -

Effect of further tax under section 3(IA) has not been taken into account as the same can not be adjusted against output tax
and further the additional sales to unregistered persons tax has been reduced from 3% to 1% with effect from tax year
2014.

Conceptual Approach to Taxes_______________________________________________


_____________________________633
Chapter 07 Solved Past Papers Sales Tax Numericals of CA Module C

(Note - 3) As the none of the clause of SRO 647(I)/2007 dated June 27, 2007 is applicable, therefore 90% limitation is
applicable U/S 8B of the Sales Tax Act, 1990.

(b)

Under section 7(1) it is provided that where a registered person did not deduct input tax within the relevant period, he may
claim such tax in the return for any of the six succeeding tax periods therefore both the input may be claimed in the monthly
sales tax returns for the tax period October.

(c)

Under section 11A of the Sales Tax Act, 1990. where a registered person pays the amount of tax less than the tax due as

indicated in his return, the short paid amount of tax alongwith default surcharge shall be recovered from such person and no

penalty under section 33 shall be imposed unless a show cause notice is given to such person.

Q. NO. 9 Spring 2003

Star Enterprises has submitted the following data for the month of March 2003
Rupees
Total Sales-registered 1,000,000
Total Sales-Unregistered 5,000,000
Export Sales 2,500,000
Exempt Supplies 500,000
Gross Purchases-from Registered suppliers 6,500,000
Gross Purchases-from Unregistered suppliers 500,000
Purchase Return-to Registered suppliers 650,000

Required: You are required to compute sales tax liability of Star Enterprises for the month of March 2003.

Solution

Sales tax liability: Sales tax


Rs. Rs.
Output tax U/S 3 (Rs. 6,000,000 x 17%) (Note - 4) 1,020,000
Input tax against taxable local supplies (Note - 1) (A) 663,000
90% of output tax (Rs.1,120,000 x 90%) (B) 918,000
Less admissible input tax: lower of (A) or (B) 663,000
Balance sales tax payable 357,000

Sales tax refundable against exports u/s 10(1) (Note - 1) 276,250

Input tax:
On taxable supplies (Rs. 6,500,000 x 17%) 1,105,000
Less: Sales tax on purchase returns [U/S 9 read with Rule 22] (Rs. 650,000 x 17%) 110,500
994,500

(Note - 1) Apportionment of residual input tax Supplies Residul input


[U/R 25 of the Sales tax Rules, 2006] tax
Rs. Rs.
Taxable local supplies 6,000,000 663,000
Exports 2,500,000 276,250
Exempt supplies (Note - 3) 500,000 55,250
9,000,000 994,500

(Note - 2) As the zero rated supplies are less than 50% of all taxable supplies under SRO 647(I)/2007 dated June 27,
2007, therefore 90% limitation is applicable U/S 8B of the Sales Tax Act, 1990.

(Note - 3) A registeed person is not entitled to claim input tax attirbutable to exempt supplies U/S 8(2) of the Sales Tax
Act, 1990.

(Note - 4) It has been assumed that 2% further tax u/s 3(1A) is not applicable on local taxable supplies to unregistered
persons under SRO 648(I)/2013 dated July 09, 2013 otherwise further tax shall be accounted for and paid

634 Conceptual Approach to Taxes


Chapter 07 Solved Past Papers Sales Tax Numericals of CA Mod- C
persons under SRO 648(I)/2013 dated July 09, 2013 otherwise further tax shall be accounted for and paid
seperately without adjustment of the same against input tax / refund of the registered person and further it shall
also not be considered for the computation of 90% limitation on output tax.

Conceptual Approach to Taxes_______________________________________________


_____________________________635
Recovery of Arrears and Refund Chapter-08

Chapter

8 RECOVERY OF ARREARS and REFUND

Section Rule Topic covered


For ICMAP & CA Mod F students
48 Recovery of arrears of tax
Recovery rules
70 Application
71 Initiation of recovery action
72 Stoppage of clearance and selling of business premises
73 Demand notice
74 Attachment and sale of property
75 Master registers to be maintained by the referring authority and the recovery Officer
76 Power to require information to be furnished
77 Mode of service of notice
78 Disposal of proceeds of execution
79 Ruling regarding disputed matters
80 Property liable to attachment and sale in execution
81 Objections and investigations, thereof
82 Removal of attachment on satisfaction of cancellation of a demand note
83 Officer entitled to attach and sell
84 Adjournment or stoppage of sale
85 Defaulter not to interfere with attached property
86 Prohibition against bidding or purchase by officer
87 Assistance for action
Application
Expeditious processing and payment of refunds
MCQs with solutions

FOR ICMAP & CA MOD F STUDENTS


Recovery of arrears of tax (U/s 48)
1. Where any amount of tax is due from any person, the Officer Inland Revenue may:
a) Deduct the amount from any money owing to person from whom such amount is recoverable and which may
be at the disposal or in the control of such officer or any officer of Income Tax , custom or central excise
department
b) Require by a notice in writing any person who holds or may subsequently hold any money for or on account of
the person from whom tax may be recoverable to pay to such officer the amount specified in notice;
c) Stop removal of any goods from the business of such person till such time the amount of tax is paid or
recovered in full;
d) Require by a notice in writing any person to stop clearance of imported goods or manufactured goods or attach
bank accounts;
e) Seal the business premises till such time the amount of tax is paid or recovered in full;
f) Attach and sell or sell without attachment any movable or immovable property of the registered person from
whom tax is due;
g) Recover such amount by attachment and sale of any movable or immovable property of the guarantor, person,
company, bank or financial institution where they fails to make payment under such guarantee, bond or
instrument.

Conceptual Approach to Taxes 637


Recovery of Arrears and Refund Chapter-08

2. If any arrears of tax, default surcharge, penalty or any other amount which is adjudged or payable by any person and
becomes irrecoverable, the Board or any authorized officer, may write of the arrears in the manner prescribed by the
Board.
3. The officer Inland Revenue shall have the same powers for the recovery of tax, penalty or any other demand raised
under this Act a Civil Court has for the purpose of recovery of an amount due under a decree.

RECOVERY RULES
Application (Rule 70) These provisions shall apply to recoveries made u/s 48 recovery of arrears of the Act.
Initiation of recovery action (Rule 71)
1. On expiry of 30 days from the date on which the Govt. dues are adjudged, the referring authority shall deduct the
amount from any money owing to the person from whom such amount is recoverable and which may be at the
disposal or in the control of such officer.
2. In case the government dues are not fully recovered, the referring authority may;
(a) Serve a notice to the sales Tax, custom, federal excise and income tax officers in the form as set out in STR-
16 to deduct the government dues from any money owing to the defaulter which may be under their control
and a copy of such notice shall be endorsed to the defaulter.
(b) Require by a notice in writing, any person or organization who holds or may subsequently hold, any money for
or on account of the defaulter, to pay to such officer the amount specified in the notice.
(c) Require, by notice in writing, the custom officers to stop the clearance of any goods imported by defaulter; and
(d) Attach the bank account of defaulter
Further if the concerned person request then the collector may recover the dues in such installments as he may deem
proper either before or after the initiation of recovery proceedings.
Further in case a registered person pays tax less than the due tax, the referring authority may directly proceed to recover the
remaining tax. He may, after serving a notice for the payment in 3 days, do so by the attachment of the bank accounts of
defaulter or through stoppage of clearance from the business premises.
Stoppage of clearance and selling of business premises (Rule 72)
In case the government dues are not recovered the referring authority shall serve upon defaulter a notice to inform him that if
he shall not pay his dues in full then he shall be disallowed to remove the goods from his business premises with effect from
the date specified in notice.
Further if the govt. dues still remain unpaid, the referring authority shall seal the business premises of the defaulter till such
time the dues are paid or recovered in full.
If the referring authority is satisfied that the defaulter is likely to conceal, remove or dispose off the whole or any part of such
of his movable or immovable property, as shall be liable to attachment in the process of recovery and that the realization of
government dues in consequence be delayed or obstructed, he may at any time after the issue of the notice direct for
reasons to be recorded in writing, execution of the notice by ignoring the specified time limit.
The referring authority may, if he deems fit, publish such notice in one or more newspapers circulated in district of normal
residence of defaulter.
Demand notice (Rule 73)
If the referring authority fails to recover the dues then he shall issue a demand note to the recovery officer and shall specify
therein the details of Government dues. He shall certify that the formalities have been completed and there exist no bar or
stay order against the proposed recovery.
Attachment and sale of property (Rule 74)
The Recovery Officer, on the receipt of demand note, shall serve upon the defaulter a notice. His movable or immovable
property shall stand attached and subsequently shall be sold if the recovery is not otherwise affected.
Master registers to be maintained by the referring authority and the recovery Officer (Rule 75)
The referring authority and Recovery Officer shall maintain master register in which they shall enter every notice, order and
demand note serially. They shall also authenticate all entries by affixing their signature and seal thereon.
They shall exchange their information for completion of corresponding entries in the master registers of both offices in the
form of a monthly return which shall after filling the respective columns by the concerned office.

638 Conceptual Approach to Taxes


Recovery of Arrears and Refund Chapter-08

Power to require information to be furnished (Rule 76)


The referring authority or the Recovery Officer may, by requisition in writing, require any person or organization to furnish
any information, required for the proceedings.
Mode of service of notice (Rule 77)
All notices or orders, unless otherwise specifically provided, shall be served;
(a) By tendering or sending by registered post or courier services, to the person or his agent at his last known address;
(b) By affixing it on the notice board in the office of recovery officer where it cannot be served in the manner as provided
in clause (a).
Disposal of proceeds of execution (Rule 78)
The following is liable to attachment and sale in execution of a notice, namely: lands, houses or other buildings, goods, bank
notes, Government securities, bonds or other securities for money, cheques, bills of exchange, hundies, promissory notes,
shares in corporation and save as hereinafter mentioned, all other saleable property movable or immovable.
Ruling regarding disputed matters (Rule 79)
Any question arising between the referring authority and the defaulter or there is representative, relating to the execution of a
notice or discharge or satisfaction of a demand note, duly issued or relating to the confirmation or setting aside by an order
of a sale held in execution of such notice, shall be determined by the recovery officer, before whom such question arises.
Property liable to attachment and sale in execution (Rule 80)
Objections and investigations, thereof (Rule 81)
1. The recovery officer shall proceed to investigate if the objection is raised on the ground that such property is not liable to
attachment or sale in execution of notice.
2. The Recovery Officer shall reject the objection summarily if he believes that the objection is raised to delay the
proceedings.
3. Pending investigation, the recovery officer may adjourn proceedings upon such terms as to security or otherwise as he
may deem fit.
4. The objector shall produce evidence to prove the legitimacy of the objection, failing which the recovery officer shall
reject the objection.
Removal of attachment on satisfaction of cancellation of a demand note (Rule 82)
When the government dues are paid to the recovery officer or the demand note is cancelled, the attachment shall be
deemed to be withdrawn. If the defaulter so desired such withdrawal shall be proclaimed at his expense and a copy of the
proclamation shall be affixed in the manner provided for a proclamation of sale of immovable property.
Officer entitled to attach and sell (Rule 83)
The attachment and sale of movable or immovable property may be made by such officer as the recovery officer may direct
in each case of recovery.
Adjournment or stoppage of sale (Rule 84)
1. The recovery officer or any officer, conducting any sale by recording his reason, may adjourn any sale proceedings to a
specified day and hour.
2. Every sale shall be stopped if, before the lot is knocked down, the amount due is tendered to the officer conducting the
sale or proof is given to his satisfaction that the amount has been paid to9 the recovery officer, who ordered the sale.
Defaulter not to interfere with attached property (Rule 85)
Where a notice has been served the defaulter or his representative in interest shall not sell, mortgage, charge, lease or
otherwise deal with any property belonging to him except with the permission of the Recovery Officer.
Prohibition against bidding or purchase by officer (Rule 86)
Either directly or indirectly, any officer or other person having any duty to perform in connection with any sale shall not bid
for, acquire or attempt to acquire any interest in the property being sold.
Assistance for action (Rule 87)
1. An officer authorized to attach or sell any property or charged with any duty to be performed may take along with him a
contingent of sales tax staff and sepoys, armed or otherwise, for any assistance he may require in the performance of
his duties.
2. In addition to sub-rule (1), such officer may apply to the officer in charge of the nearest police station for such assistance
as may be necessary in the discharge of his duties.

Conceptual Approach to Taxes 639


Recovery of Arrears and Refund Chapter-08

REFUND
Application (Rule 26)
This shall apply to all refund claims filed by--
a) registered manufacturer-cum-exporters and commercial exporters whose all or part of supplies are zero rated.
b) registered persons whose goods to be supplied are chargeable to sales tax at the rate of zero percent, claim input tax.
c) registered persons claiming refund of the excess amount of input tax.
d) registered persons whose claim input tax on supplies used in the export of goods and
e) local supply of which is exempt.
f) persons claiming refund within one year (section 66) and diplomats, diplomatic missions and privileged persons and
organizations who purchase goods or services on payment of tax and are otherwise entitled to receive zero-rated
supply.
Expeditious processing and payment of refunds (Rule 26A)

The refund claims as provided in this rule shall be processed and paid in the manner as provided and all other claims shall
be processed and paid in the manner as prescribed after rule 26A.
Refunds under this rule shall be allowed to the registered manufacturers-cum-exporters of Regional Tax Office, Lahore from
tax period April, 2010. From tax period July, 2010 all registered manufacturer-cum-exporters of other Regional Tax Offices /
Large Taxpayer Units will be allowed refund under this rule.
Refunds shall be allowed to the Active Taxpayers, at the time of processing by the IT System of Board, as per Active
Taxpayers List displayed at Board's website.
Registered persons claiming refund will electronically submit refund claim in requisite data in RCPS format through Board
web portal by using the user-id, password and pin code allotted to them at the time of e-Enrolment.
The registered person claiming refund shall maintain and keep all the paper documents relating to the refund claim, such as
invoices, credit notes, debit notes, goods declarations, bank credit advice, etc. in his office instead of submitting to the
concerned Regional Tax Office or Large Taxpayers' Unit.
Risk Management System (RMS) of Board IT System shall process the refund claims within two working days of electronic
submission of refund claim in the RCPS format and take further action as follows:
a) In case the refund claim is cleared fully or partially, electronic advice will be issued to the concerned Regional Tax
Office/Large Taxpayer Unit and the registered person about the refund amount cleared by the Risk Management
System (RMS) for payment; and

b) In case the refund claim is not cleared by Risk Management System (RMS), an electronic intimation will be issued to the
registered person in this regard and his case will be processed in the prescribed manner (7) Concerned Regional Tax
Office/Large Taxpayer Unit will arrange issuance of cheque for the amount cleared by Risk Management System (RMS)
within 7 working days of the receipt of electronic advice.
The electronic copy of refund claim will be forwarded for post refund audit after the issuance of cheque for the amount
cleared by RMS.
Establishment of Refund Division and posting of officers (Rule 27)
There shall be established a CSTRO under the Board for centralized payment of refund amount to such claimants and from
such date as the Board may specify.
There shall be established a Refund Division in each Collectorate of Sales Tax to receive, process and settle the refund
claims filed under these rules.
There shall be posted an officer not below the rank of an Assistant Collector of Sales Tax, as nominated by the Collector to
be the officer-in-charge of the Refund Division.
There shall be established a Post Refund Division in each Collectorate of Sales Tax headed by an officer not below the rank
of an Assistant Collector of Sales Tax to audit the refund claims processed and sanctioned by the Refund Division.
Filing of refund claim (Rule 28)
When the claimant shall file the monthly sales tax return with all supportive document including requisite data in the format
or software (RCPS), shall be treated as refund claim has been received.
Provided that no refund claim shall be entertained if the claimant fails to furnish the claim on the prescribed software (RCPS)
along-with the supportive documents within 120 days of the filing of return:
Provided further that the period of one hundred and twenty days as aforesaid, in case of a commercial exporter, shall be
reckoned from the date when the BCA is issued to him by the concerned Bank.

640 Conceptual Approach to Taxes


Recovery of Arrears and Refund Chapter-08

Provided also that if a claimant is exporting goods manufactured by him as well as the goods purchased in the same state,
in the same tax period, the period of 120 days shall be reckoned from date of filing of return or the date of issuance of BCA,
whichever is later.
The manufacturer-cum-exporters, who are registered as limited companies, having annual turnover more than 100 million
rupees and whose refund claim on inputs consumed in zero-rated supplies excluding building material and utilities is less
than 1% of the value of exports and local zero-rated sales, shall have the option to file refund claim electronically provided
their suppliers are also filing return along with details of sale and purchases electronically.
If the supportive documents are not submitted to the officer-in-charge within the specified time, the Collector of Sales Tax
having jurisdiction may extend the time limit for a further 60 days on a written request from the claimant justifying the
reasons for delay in submission of such documents or data on RCPS.
The Board may, through a General Order or otherwise, prescribe the date, manner and procedure for electronic filing of
sales tax refund claims by the registered persons filing their monthly returns electronically.
Scrutiny and processing of refund claim (Rule 29)
On submission of a refund claim, the Refund Receipt Section shall confirm that the claim is complete in all respects, after
which it shall be loaded in the system for assigning the claim a unique identification number.
After assigning the unique identification number, the CREST shall cross match the data on soft copy with the data available
in the system and process the claim by applying the risk parameters and generate analysis report indicating the admissible
amount as well as the amount not validated on the basis of automated risk criterion along-with the objections raised by the
system.
The processing officer shall forward the claim file along-with the analysis report to the officer-in-charge for further necessary
action.
Where the Processing Officer or the officer-in-charge is of the opinion that any further inquiry or audit is required in respect
of amount not cleared by the CREST or for any other reason to establish genuineness and admissibility of the claim, he may
make or cause to be made such inquiry or audit as deemed appropriate, after seeking approval from the concerned
Additional Collector and inform the refund claimant accordingly.
Sanction and payment of refund claim (Rule 30)
On receipt of analysis Report and refund payment order for the amount verified by CREST and found admissible by the
processing officer, the officer in-charge shall sanction the amount so determined and issue the Refund Payment Order
(RPO).
The officer-in-charge shall transmit the Refund Payment Order electronically and in respect of claim filed manually, forward
the original copy thereof to the treasury officer of the Collectorate. The treasury officer shall make payment of refund through
a cross cheque in favour of the refund claimant, indicating his declared account number and Bank name. The crossed
cheque shall also be counter signed by an authorized co-signatory.
The Additional Collector shall reconcile the refund cheques issued by the treasury officer of the Collectorate during a month
with the Bank scrolls received from State Bank of Pakistan and record the outcome of such reconciliation in the system.
Where any cheque is returned back by the State Bank of Pakistan due to any reason, the treasury officer shall cancel such
cheque, if required and attach such cancelled cheque with the respective counter foil of the cheque book.
From such date to be notified by the Board, the officer-in-charge shall electronically transmit the RPO to the treasury officer
in the CSTRO under his digital signatures and retain a copy thereof in the Refund Division for record.
The treasury officer in CSTRO and the co-signatory designated by the Board in this regard shall issue the cheque for the
sanctioned amount as mentioned in the RPO.
The CSTRO shall also prepare a statement of payment advice for the concerned Bank on a daily basis, for direct transfer of
the refund amount to the declared Bank account of the claimant, under intimation to the CSTRO, the concerned Collectorate
of Sales Tax as well as the claimant.
Scrutiny and processing of refund claims filed by manufacturers of specified goods (Rule 31)
Refund of sales tax paid inputs, local supply of manufacturing of goods which has been zero-rated, shall be processed and
sanctioned through the Fast Track Channel within 15 days from the date of filing of refund claim.
The refund of sales tax paid on utilities and the goods imported directly by the claimant and the input tax incurred on
acquiring furnace oil from Oil and Gas Marketing Companies for power generation and consumption thereof in the
manufacture of zero-rated goods, shall be sanctioned upon validation by the automated system. Refund of input tax paid in
respect of other inputs, if validated by the CREST, shall also be sanctioned in the same manner.
Refund of tax paid verifiable inputs used in the export of goods, local supply of which is exempt under the Act or any
notification, shall be paid.

Conceptual Approach to Taxes 641


Recovery of Arrears and Refund Chapter-08

Scrutiny and processing of refund claims relating to commercial exporters (Rule 32)
When the commercial exporters made exports and claimed refund of sales tax paid input then refund shall be sanctioned
only after verification of supportive documents and approval through CREST.
Extent of payment of refund claim (Rule 33)
When the claimants claim refund under these rules then refund shall be sanctioned only to the extent of input tax paid on
actual purchases or imports used in manufacturing of goods to be exported or supplied at the rate of 0%.
Refund of excess input tax not relating to zero-rated supplies (Rule 34)

(1) The refund of excess unadjusted input tax relating to supplies other than zero-rated shall be claimed and sanctioned
in the cases mentioned below, namely:-
(a) the manufacturers of fertilizers, electric power producers and electric power distribution companies may claim
refund of excess input tax over output tax in any tax period;
(b) registered persons in plastic, paper and steel sectors whose inputs are subject to sales tax at 20% or 17.5%
ad valorem and their final product is subject to tax at 15% ad valorem may claim refund of excess input tax if
the same is not adjusted within a minimum consecutive period of three months;
(c) registered persons who are not able to adjust input tax in excess of 90% of output tax in view of restriction in
section 8B of the Act, may file refund claim as under,-
(i) in case of registered persons whose accounts are subject to audit under the Companies Ordinance,
1984, after the end of their accounting year; and
(ii) in case of other registered persons, after the end of financial year;
(d) all other registered persons, not covered by clauses (a) to (c) above, may claim refund of excess input tax, if
the same is not adjusted within a minimum consecutive period of twelve months:
Provided that the amount of refund claim in all such cases shall not exceed the excess of total input tax over the total
output tax, as declared in the relevant returns, for the period in respect of which the claim has been filed and shall not
include any excess input tax declared prior to the said period.
(2) The registered person shall file application for refund claim along with data prepared through RCPS, providing the
following information, namely:--
(i) name and registration number of the claimant;
(ii) period of claim;
(iii) amount of claim; and
(iv) a statement along with annual audited accounts as envisaged in clause (i) of subsection (2) of section 8B of
the Act, if applicable:
Provided that the application for claim shall be filed within the period specified in rule 28 after the filing of return for
the last month in the period of claim.
(3) The refund of excess input tax under this chapter shall be filed, processed and sanctioned in the manner as provided
in rules 29 and 30.
(4) The refund of excess input tax excluding the cases of claims by registered persons, whose accounts are subject to
audit under the Companies Ordinance, 1984 as referred to in section 8B(2) of the Act, shall be sanctioned as found
admissible after a departmental audit of records maintained by the registered person and after a certificate is
recorded by the sales tax officers auditing the records that actual value addition during the period involved was not
found sufficient to require a net payment of tax for the reasons mentioned in the audit report:
Provided that in case of refund claim post-refund audit shall be conducted after the close of financial year and the
auditors shall report on the aspect of value addition in their audit report.
(5) The refund claimant shall ensure that the input tax involved in the refund claim is not shown as outstanding credit in
the returns for the tax periods subsequent to the period of claim.
(6) The refund of excess input tax under this rule shall not be claimed where the same has already been claimed or paid
under any other notification issued by the Federal Government or the Board.
Responsibility of the claimant (Rule 35)
The claimant shall provide supportive document and data on prescribed electronic format and then apply for the automated
processing of refund claims. He shall be responsible for any mis-declaration or submission of incorrect information and shall
be liable for penal action besides recovery of the amount erroneously refunded along with default surcharge.

642 Conceptual Approach to Taxes


Recovery of Arrears and Refund Chapter-08

Post-sanction audit of refund claims (Rule 36)


(1) When the refund claim is disposed of then the Officer-in-charge shall forward the relevant file to the Post Refund
Audit Division for post sanction audit and scrutiny. The relevant file shall include verification of input tax payment by
respective suppliers.
(2) The officer-in-charge of Post Refund Audit Division shall send his findings to the concerned Refund Division for
further necessary action, as required under the law.
Action on inadmissible claims (Rule 37)
At the time of issuing RPO, the officer-in-charge shall, after finding the inadmissible or unverified claim or any part thereof,
issue a notice requiring the claimant to show cause as to why the claim or any pert thereof should not be rejected and as to
why the claimant should not be proceeded against.
Supportive documents (Rule 38)
(1) The refund claimant shall submit the refund claim in computer diskette in the prescribed format or software along with
the following documents namely:--
(a) input tax invoices or goods declaration for import in respect of which refund is being Claimed;
(b) output tax invoices and summary of invoices for local zero-rated goods;
(c) goods declaration for export (quadruplicate copy) indicating Mate Receipt number with date or airway bill or
railway receipt or postal receipt besides the examination report endorsed on the reverse side thereof by the
Customs Officers, in case of claims by persons other than manufacturer-cum exporters of goods zero-rated in
a notification.
Provided that in case of imports or exports processed through PACCS, submission of goods declaration shall not be
required and verification regarding import, or as the case may be export, in such cases shall be carried out by cross-
matching of the declarations with the data available in the system.
(a) copy of House and Master bill of lading and airway bill or as the case may be, railway receipt in token of
verification of the goods taken out of Pakistan; and
(b) statement of the tax paid inputs, in respect of which refund is claimed by the claimants other than the
manufacturers of goods zero-rated for local supplies, in the formal set out below:
(2) If the Custom Authorities issued the duty drawback order, the commercial exporter shall submit its copy and Bank
Credit Advice as well in addition to the documents specified above.
(3) In case the claimant file the refund claim shall submit an application for refund with particulars indicating his name,
address, registration number, the amount of sales tax refund claimed and reasons for seeking such refund along with
following documents, namely:--
(a) input tax invoices in respect of which refund is claimed;
(b) proof of payment of input tax claimed as refund; and
(c) copy of the relevant order on the basis of which refund is claimed.
When it is verified that no adjustment or refund of input tax has been claimed earlier
The refund claimed shall be sanctioned after verifying that no adjustment or refund of input tax has been claimed earlier and
that the goods have been duly accounted for in the inventory records and the invoices claimed are validated by the CREST.
In case of claims by diplomats, diplomatic missions and privileged persons and organizations, they shall submit original
exemption order or certificate or Board Booklet and original sales tax invoice. The refund shall be sanctioned after making
necessary endorsements on these documents to the effect that the refund has been paid against the same.
Miscellaneous and savings (Rule 39)
In case the refund was paid in excess of the amount due, such excess shall be recovered along with default surcharge
besides any other penal action that may be taken under the Act.
A registered person, committed tax fraud, shall be finalized after detailed scrutiny of all partners in the supply chain to
establish the forward and backward linkages and after verifying input tax payment by them.
The existing Sales Tax Treasury Offices functioning in the Collectorates shall continue to pay sales tax refund till such time
the CSTRO is established and, accordingly, any reference to CSTRO shall be construed as a reference to such existing
Treasury Offices.
The admissible refund claims received with supportive documents up to the date of commencement of these rules shall be
sanctioned and paid in accordance with the provisions of the Sales Tax Refund Rules, 2002, or the Chapter V of the Sales
Tax Rules, 2005, or the Sales Tax Refund on Zero-Rated Supply Rules, 2006, whichever is applicable.

Conceptual Approach to Taxes 643


Recovery of Arrears and Refund Chapter-08

Processing of refund claims in Large Taxpayers Units (Rule 39A)


The refund claimant registered in a Large Taxpayers Unit, desirous of availing facility, shall file a refund application to the
Collector of Sales Tax having jurisdiction along with the following documents, namely:--
(a) an undertaking affirming the accuracy and genuineness of refund; and
(b) a revolving bank guarantee valid for at least 120 days issued by a scheduled bank, to the satisfaction of Collector of
Sales Tax (Large Taxpayers Unit), of an amount not less than amount of refund claimed.

The refund claim shall be processed as follows:--


(a) where the claimant has filed documents, the Collector shall process and allow 50% of the refund of input tax within 5
working days of receipt thereof;
(b) within 15 days after the sanctioning of the claim as at (a) the claimant shall file a complete refund claim along with the
supportive documents and soft copy on the prescribed format, which shall be scrutinized in the Large Taxpayers
Unit, and objections, if any, related to the refund claim shall be conveyed to the claimant within 7 days of the receipt
of the claim;
(c) subject to the provisions of this rule, the refund claim shall be finalized by the Large Taxpayer Unit within 15 days of
the filing of complete refund claim and
(d) where an amount of refund on account of input credit document or otherwise is unverified, the Collector may require
extension in bank guarantee covering such amount as deemed fit.
In case any amount already sanctioned and paid is found inadmissible or remains unverified after 6 months of the bank
guarantee, the same shall be recovered within 7 days by encashment of the bank guarantee to the extent of inadmissible
amount besides other legal action.
The Collector of Sales Tax shall notify an officer, not below the rank of an Assistant Collector, as focal person in the Large
Taxpayer Unit to liaise with other Collectorates regarding the problems or objections encountered on account of purchases
and supplies of the refund claimant for speedy solution thereof.

644 Conceptual Approach to Taxes


Recovery of Arrears and Refund Chapter-08

MULTIPLE CHOICE QUESTIONS


Q.1. Where any amount of tax is due from any person, the CIR may _______.
(a) Deduct the amount from any money owing to person from whom such amount is recoverable and which may
be at the disposal or in the control of such officer or any officer of Income Tax , custom or central excise
department
(b) Require by a notice in writing any person who holds or may subsequently hold any money for or on account of
the person from whom tax may be recoverable to pay to such officer the amount specified in notice;
(c) (c) Stop removal of any goods from the business of such person till such time the amount of tax is paid or
recovered in full;
(d) all of above
Q.2. On expiry of ___ days from the date on which the Govt. dues are adjudged, the referring authority shall deduct the
amount from any money owing to the person from whom such amount is recoverable and which may be at the
disposal or in the control of such officer.
(a) 50
(b) 40
(c) 30
(d) 60
Q.3. In case the government dues are not recovered the referring authority shall serve upon defaulter a notice to inform
him that if he shall not pay his dues in full then he shall be ____ to remove the goods from his business premises with
effect from the date specified in notice.
(a) disallowed
(b) allowed
(c) ordered
(d) none of above
Q.4. If the referring authority fails to recover the dues then he shall issue a ___ to the recovery officer and shall specify
therein the details of Government dues.
(a) demand notice
(b) notice of deregistration
(c) show cause notice
(d) none of above
Q.5. The Recovery Officer shall serve upon the defaulter a notice and his ____ shall stand attached and subsequently
shall be sold if the recovery is not otherwise affected.
(a) moveable property
(b) immovable property
(c) both a and b
(d) stock in trade
Q.6. The referring authority and Recovery Officer shall maintain _______ in which they shall enter every notice, order and
demand note serially.
(a) register of notices
(b) register of defaulters
(c) master register
(d) none of above
Q.7. The referring authority or the Recovery Officer may, by requisition in writing, require any person or organization to
furnish ______, required for the proceedings.
(a) any information
(b) information about sales and purchases only
(c) sales tax return
(d) none of above

Conceptual Approach to Taxes 645


Recovery of Arrears and Refund Chapter-08

Q.8. All notices or orders, unless otherwise specifically provided, shall be served___________;
(a) By tendering or sending by registered post or courier services, to the person or his agent at his last known
address;
(b) By affixing it on the notice board in the office of recovery officer where it cannot be served in the manner as
provided in clause (a).
(c) through email
(d) both a and c

Q.9. Any question arising between the referring authority and the defaulter or there is representative, shall be determined
by ___________, before whom such question arises.
(a) Commissioner Inland Revenue
(b) Appellate Tribunal Inland Revenue
(c) Recovery officer
Q.10. The Recovery Officer shall reject the objection summarily if he believes that the objection is ___.
(a) causing hardship to taxpayer
(b) raised to delay the proceedings
(c) raised by his subordinate authority
(d) none of above
Q.11. When the government dues are paid to the recovery officer or the demand notice is cancelled, the attachment shall
be deemed to be ___.
(a) withdrawn
(b) effective
(c) valid
(d) none of above
Q.12. When the claimant shall file the monthly sales tax return with all supportive document including requisite data in the
format or software (______), shall be treated as refund claim has been received.
(a) RCS
(b) RCCS
(c) RCPS
(d) Board
Q.13. Provided that no refund claim shall be entertained if the claimant fails to furnish the claim on the prescribed software
along-with the supportive documents within ___ days of the filing of return:
(a) 120
(b) 60
(c) 100
(d) 80
Q.14. The period of 120 days shall be reckoned from date of filing of return or the date of issuance of ___ whichever is later.
(a) RCPS
(b) BCA
(c) Demand notice
(d) none of above
Q.15. If a claimant is exporting goods manufactured by him as well as the goods purchased in the same state, in the same
tax period, the period of 120 days shall be reckoned from date of filing of return or the date of issuance of BCA,
_____.
(a) which-ever is earlier
(b) whichever is later
(c) none of above

646 Conceptual Approach to Taxes


Recovery of Arrears and Refund Chapter-08

Q.16. The manufacturer-cum-exporters, who are registered as limited companies, having annual turnover more than ___
million rupees and whose refund claim on inputs consumed in zero-rated supplies excluding building material and
utilities is less than 1% of the value of exports and local zero-rated sales, shall have the option to file refund claim
electronically provided their suppliers are also filing return along with details of sale and purchases electronically.
(a) 500
(b) 200
(c) 100
(d) 10
Q.17. If the supportive documents are not submitted to the officer-in-charge within the specified time, the Collector of Sales
Tax having jurisdiction may extend the time limit for a further ___ days on a written request from the claimant
justifying the reasons for delay in submission of such documents or data on RCPS.
(a) 60
(b) 70
(c) 80
(d) 90
Q.18. Refund of sales tax paid inputs, local supply of manufacturing of goods which has been zero-rated, shall be
processed and sanctioned through the Fast Track Channel within ___ days from the date of filing of refund claim.
(a) 20
(b) 30
(c) 15
(d) 10
Q.19. When the commercial exporters made exports and claimed refund of sales tax paid input then refund shall be
sanctioned only after verification of supportive documents and approval through ____.
(a) Refund payment order
(b) Federal Board of Revenue
(c) RCPS
(d) CREST
Q.20. When the claimants claim refund under these rules then refund shall be sanctioned only to the extent of input tax paid
on actual purchases or imports used in manufacturing of goods to be exported or supplied at the rate of ___ %.
(a) 16
(b) 0
(c) 21
(d) 1

ANSWERS

1 (d) 2 (c) 3 (a) 4 (a) 5 (c)


6 (c) 7 (a) 8 (d) 9 (c) 10 (b)
11 (a) 12 (c) 13 (a) 14 (b) 15 (b)
16 (c) 17 (a) 18 (c) 19 (d) 20 (b)

Conceptual Approach to Taxes 647


Recovery of Arrears and Refund Chapter-08

648 Conceptual Approach to Taxes


Offences and Penalties Chapter-09

Chapter

9 OFFENCES & PENALTIES

Section Topic covered


For CA Mod F and ICMAP Students
33 Offences and penalties
34 Default Surcharge
34A Exemption from penalty and default surcharge
37 Power to summon persons to give evidence and produce documents in inquiries
37A Power to arrest and prosecute
37B Procedure to be followed on arrest of a person
37C Special Judges
37D Cognizance of offences by Special Judges
37E Special Judge, etc. to have exclusive jurisdiction
37G Transfer of cases
37H Place of sittings
37I Appeal to the High Court
38A Power to call for information
38B Obligation to produce documents and provide information
40C Monitoring or tracking by electronic or by other means
MCQs with solutions
ICMAP past papers theoretical questions

(For CA MOD F AND ICMAP students)


Offences and penalties (U/s 33): The Sales tax Act, 199, levied various punishments in the form of default surcharge,
penalties, prosecutions and imprisonment on certain actions, omissions, abstinence or defaults. The various defaults under
the Sales tax Act, 1990 along with punishments applicable on the same are as under.

Section of the Act to which


Offences Penalties
offence has reference
1. Where any person fails to furnish a Such person shall pay a penalty of 26
return within the due date. Rs.5,000:

Provided that in case a person files a return


within 10 days of the due date, he shall pay
a penalty of Rs.100 for each day of default.

2. Any person who fails to issue an invoice Such person shall pay a penalty of Rs.5,000 23
when required under this Act. or 3% of the amount of the tax involved,
whichever is higher.

3. Any person who un-authorizedly issues Such person shall pay a penalty of 3, 7 and 23
an invoice in which an amount of tax is Rs.10,000 or 5% of the amount of the tax
specified. involved, whichever is higher.

4. Any person who fails to notify the Such person shall pay a penalty of 14
changes of material nature in the Rs.5,000.
particulars of registration of taxable
activity.

Conceptual Approach to Taxes 649


Offences and Penalties Chapter-09

5. Any person who fails to deposit the Such person shall pay a penalty of 3, 6, 7 and 48
amount of tax due or any part thereof in Rs.10,000 or 5% of the amount of the tax
the time or manner laid down under this involved, whichever is higher:
Act or rules or order made thereunder.

Provided that, if the amount of tax or any


part thereof is paid within 10 days from the
due date, the defaulter shall pay a penalty of
Rs.500 for each day of default:

Provided further that no penalty shall be


imposed when any miscalculation is made
for the first time during a year:

Provided further that if the amount of tax


due is not paid even after the expiry of a
period of 60 days of issuance of the notice
for such payments by an officer of Inland
Revenue not blow the rank of the Assistant
Commissioner Inland Revenue, the
defaulter shall, further be liable, upon
conviction by a Special Judge, to
imprisonment for a term which may extend
to 3 years, or with fine which may extend to
amount equal to the amount of tax involved,
or with both.

6. Any person who repeats erroneous Such person shall pay a penalty of Rs.5,000 7 and 26
calculation in the return during a year or 3% of the amount of the tax involved,
whereby amount of tax less than the actual whichever is higher.
tax due is paid.

7. Any person who is required to apply for Such person shall pay a penalty of 14
registration under this Act fails to make an Rs.10,000 or 5% of the amount of tax
application for registration before making involved, whichever is higher:
taxable supplies.

Provided that such person who is required


to get himself registered under this Act, fails
to get registered within 60 days of the
commencement of taxable activity, he shall,
further be liable, upon conviction by a
Special Judge, to imprisonment for a term
which may extend to 3 years, or with fine
which may extend to an amount equal to the
amount of tax involved, or with both.

8. Any person who fails to maintain records Such person shall pay a penalty of 22 and 24
required under this Act or the rules made Rs.10,000 or 5% of the amount of tax
there under. involved, whichever is higher:

9. Where a registered person who, without 25


any reasonable cause, in non-compliance
with the provisions of section 25,-

(a) fails to produce the record on receipt of


first notice; such person shall pay a penalty of Rs.5,000;

(b) fails to produce the record on receipt of such person shall pay a penalty of
second notice; and Rs.10,000; and

(c) fails to produce the record on receipt of such person shall pay a penalty of
third notice. Rs.50,000.

650 Conceptual Approach to Taxes


Offences and Penalties Chapter-09

10. Any person who fails to furnish the such person shall pay a penalty of 26
information required by the Board through Rs.10,000; and
a notification issued under sub-section (5)
of section 26.

11. Any person who,-

(a) submits a false or forged document to Such person shall pay a penalty of 2(37) and general
any officer of Inland Revenue; or Rs.25,000 or 100% of the amount of the tax
involved, whichever is higher. He shall,
further be liable, upon conviction by a
Special Judge, to imprisonment for a term
which may extend to 1 year, or with fine
which may extend to an amount equal to the
amount of tax involved, or with both.

(b) destroys, alters, mutilates or falsifies


the records including a sales tax invoice; or

(c) knowingly or fraudulently makes false


statement, false declaration, false
representation, false personification or
issues or uses a document which is forged
or false.

12. Any person who denies or obstructs Such person shall pay a penalty of 25, 38 , 38A and 40B
the access of an authorized officer to the Rs.25,000 or an 100% of the amount of tax
business premises, registered office or to involved, whichever is higher. He shall,
any other place where record are kept, or further be liable upon conviction by a
otherwise refuses access to the stocks, Special Judge, to imprisonment for a term
records or fails to present the same when which may extend to 5 years, or with fine
required under section 25, 38 ,38A or 40B. which may extend to an amount equal to the
loss of tax involved, or with both.

13. Any person who commits, causes to Such person shall pay a penalty of 2(37)
commit or attempts to commit the tax Rs.25,000 or 100% of the amount of tax
fraud, or abets or connives in involved, whichever is higher. He shall,
commissioning of tax fraud. further be liable, upon conviction by a
Special Judge, to imprisonment for a term
which may extend to 5 years, or with fine
which may extend to an amount equal to the
loss of tax involved, or with both.

14. Where any person violates any Such person shall pay a penalty of 48
embargo placed on removal of goods in Rs.25,000 or 10% of the amount of the tax
connection with recovery of tax. involved, whichever is higher. He shall,
further be liable, upon conviction by a
Special Judge, to imprisonment for a term
which may extend to 1 year, or with fine
which may extend to amount equal to the
amount of tax involved, or with both.

15. Any person who obstructs the Such person shall pay a penalty of 31 and general
authorized officer in the performance of his Rs.25,000 or one hundred per cent of the
official duties. amount of the tax involved, whichever is
higher.

16. Any person who fails to make payment Such person shall pay a penalty of Rs.5,000 73
in the manner prescribed under section 37 or 3% of the amount of tax involved,
of this Act. whichever is higher.

17. Any person who fails to fulfill any of the Such person shall pay a penalty of Rs.5,000 71 and general
conditions, limitation or restrictions or 3% of the amount of the tax involved,
prescribed in a Notification issued under whichever is higher.
any of the provisions of this Act.

Conceptual Approach to Taxes 651


Offences and Penalties Chapter-09

18. Where any officer of Inland Revenue Such officer of Inland Revenue shall be General
authorized to act under this Act, acts or liable, upon conviction by a special Judge,
omits or attempts to act or omit in a to imprisonment for a term which may
manner causing loss to the Inland extend to 3 years, or with fine which may
Revenue or otherwise abets or connives in extend to amount equal to the amount of tax
any such act. involved, or with both.

19. Any person who contravenes any of Such person shall pay a penalty of Rs.5,000 General
the provisions of this Act for which no or 3% of the amount of the tax involved,
penalty has, specifically, been provided in whichever is higher.
this section.

21. Where any person repeats an offence Such person shall pay twice the amount of General
for which a penalty has, specially, been penalty provided under the Act for the said
provided in this Act. offence.

22. Any person who,-

(a) knowingly and without lawful authority Such person shall pay a penalty of 50A
gains access to or attempts to gain access RS.25,000 or 100% of the amount of tax
to the computerized system; or involved, whichever is higher. He shall,
further be liable, upon conviction by the
Special Judge, to imprisonment for a term
which may extend to 1 year, or with fine
which may extend to an amount equal to the
loss of tax involved, or with both.

(b) unauthorizedly uses or publishes or


otherwise disseminates information
obtained from the computerized system; or

(c) falsifies any record or information


stored in the computerized system; or

(d) knowingly or dishonestly damages or


impairs the computerized system; or

(e) knowingly or dishonestly damages or


impairs any duplicate tape or disc or other
medium on which any information obtained
from the computerized system is kept or
stored; or

(f) unauthorizedly uses unique user


identifier of any other registered user to
authenticate a transmission of information
to the computerized system; or

(g) fails to comply with or contravenes any


of the condition prescribed for security of
unique user identifier.

Default Surcharge (U/s 34)


1. If a registered person does not pay the tax or any part of tax, whether due to fraud or otherwise or claims a tax credit
refund or makes an adjustment which is inadmissible or incorrectly applies the rate of 0% to supplies made by him,
shall be liable to pay the tax due and default surcharge at the rate below:
(a) In the case of person liable to pay amount of tax or charge or refund erroneously made, shall pay default
surcharge @ rate of KIBOR plus 3% p.a. of the amount of tax due or refund made.
(b) In the case the person has committed tax fraud shall pay default surcharge @ rate of 2% per month of the
amount of tax evaded or amount of refund fraudulently claimed until the total liability including default
surcharge is paid.
2. Tax due does not include the amount of penalty. In the calculation of default surcharge, following should be
considered;

652 Conceptual Approach to Taxes


Offences and Penalties Chapter-09

(a) In the case of inadmissible input tax credit or refund, the period of default surcharge shall be started from
the date of adjustment of inadmissible input tax credit or refund is received.
(b) In the case of non-payment of tax or part thereof, the period of default surcharge shall be started from
the 16th day of month (following the due date of the tax period to which the default relates) to the day
preceding the date on which tax due is actually paid.
Exemption from penalty and default surcharge (U/s 34A)
The Federal Government (by notification) or the board (by order), published in Gazette in writing, may exempt any person
or class of person from payment of whole or part of penalty or default surcharge on conditions and limitation as
specified therein.
Power to summon persons to give evidence and produce documents in inquiries under this Act (u/s 37)
1. Any Officer of Inland Revenue shall have powers to summon any person whose attendance or any other thing he
considers necessary to produce evidence or document in any inquiry.
2. Any person summoned, shall be bound to attend either in person or by an authorized agent as directed by Officer
of Inland Revenue.
3. Provide that person who is exempt from personal appearance in a court u/s 132 and 133 of the Code of Civil
Procedure, shall not be required to appear in person.
4. Any inquiry before an Officer Inland Revenue shall be deemed to be a judicial proceeding within the meaning of sec.
193 and 228 of the Pakistan Penal Code Act, 1860.
Power to arrest and prosecute (U/s 37A)
1. An Officer Inland Revenue (at least an Assistant Commissioner) or any other officer of equal rank authorized by
the Board, who believes that any person has committed a tax fraud or any offence warranting prosecution on the
basis of material evidence, may cause arrest of such person.
2. All arrests made under this Act shall be carried out in accordance of Code of Criminal Procedure, 1898
3. Where any person has committed a tax fraud or any offence (warranting prosecution), the CIR may, either before
or after the institution of any proceedings for recovery of tax, compound the offence if such person pays the tax due
along with default surcharge and penalty as determined.
4. In the case of company, suspected of tax fraud, every director or officer of that company whom the authorized
officer believes is personally responsible for actions of company contributing the tax fraud shall be liable to arrest.
Further any arrest shall not absolve the company from the liabilities of payment of tax, default surcharge and
penalty.
Procedure to be followed on arrest of a person (U/s 37B)
1. The sales tax officer shall intimate the fact of arrest of person to Special Judge who may direct him to produce the
arrested person at such time and place and on such date as considers expedient and such officer shall act
accordingly.
2. The person arrested shall be, within 24 hours of such arrest excluding the time necessary for the journey, produced
before the Special Judge, if there is no Special Judge within a reasonable distance then to the nearest Judicial
Magistrate.
3. Any person arrested may (on the request), after pursuing the record or after giving the prosecution an opportunity of
being heard, admit or refuse to admit him to bail on his executing bond, with or without sureties and direct is detention
at such place as he deems fit.
4. Magistrate may, after authorizing the detention of person produced before him, direct to fix the date and time of his
production in such custody at such place and for such period as he considers necessary or proper for his earliest
production before the Special Judge, or direct him to be forthwith taken to, and produced before Special Judge, and
he shall be so taken.
5. Special Judge or Judicial Magistrate may preclude from remanding any such person in the custody of the Sales Tax
Officer who makes a request in writing to that effect and the Special Judge or Judicial Magistrate, after pursuing
record and hearing such person, deems necessary to make such order for the completion of inquiry or investigation
and such custody shall not exceed 14 days in any case.
6. The Sales Tax Officer shall record the fact of arrest and other relevant particulars in the register and shall
immediately proceed and if he completes inquiry within 24 hours of his arrest, excluding the time required for journey,
he may make a request for his further detention in his custody.
7. While holding an inquiry, the Sales Tax Officer shall exercise same powers as are exercisable by an officer in charge
of a police station under the Code of Criminal Procedure, 1898.

Conceptual Approach to Taxes 653


Offences and Penalties Chapter-09

8. Sales Tax Officer, after holding an inquiry, shall release that person on his executing a bond with or without sureties
as he found no sufficient evidence or reasonable grounds for suspicion. He shall direct a person to appear, as and
when required before the Special Judge and make a report about his discharge to Special Judge and shall make a
full report of case to his immediate superior.
9. The Special Judge may, after the perusal of record of inquiry and hearing the prosecution, agree with such report and
discharge the accused and if he believes that there is sufficient grounds against such person proceed with his trial
and direct the prosecution to produce evidence.
10. The Sales Tax Officer empowered to hold inquiry shall maintain a Register of Arrests and Detention in the
prescribed form. He shall enter the name and particulars of every person arrested under this section together with the
time and date of arrest, details of the information received, the details of things, goods or documents, recovered from
his custody, the name of witnesses and the explanation given by him and the manner in which the inquiry has been
conducted from day to day in such register. The Special Judge may direct such officer to produce such register or
authenticated copies of its aforesaid entries.
11. Any Magistrate of the first class may record any statement or confession during inquiry under this Act, in accordance
with the provisions of sec. 164 of Code of Criminal Procedure, 1898.
12. The Federal Government by notification may authorize any other officer working under the Board to exercise the
powers and perform the functions of a Sales Tax Officer on some conditions imposed by him.
Special Judges (U/s 37C)
1. The Federal Government (by notification) may appoint as many Special Judges as he considers necessary and
shall specify the headquarters of each Special Judge and the territorial limits within which he shall exercise
jurisdiction.
2. No person shall be appointed as Special Judge unless he is or has been a session judge.
Cognizance of offences by Special Judges (U/s 37D)
1. A Special Judge may, within the limits of his jurisdiction, take cognizance of any offence punishable under this Act:
(a) Upon a report in writing made by an officer of Inland revenue or any other officer authorized by Federal
Government;
(b) Upon receiving a complaint or information of facts constituting such offence made or communicated by any
person;
(c) Upon his own knowledge acquired during any proceeding before him under this Act or any other law for the
time being in force.
2. The Special Judge shall proceed with the trial of the accused upon the receipt of report.
3. Upon the receipt of a complaint or information or by own knowledge, the Special Judge may
4. After conducting such inquiry or considering the report of such Magistrate or officer, if Special Judge believe that:
(a) There is no sufficient grounds for proceedings, he may dismiss the complaint
(b) There are sufficient grounds for proceedings; he may proceed against the person.
5. A Special Judge or a Magistrate or an officer, holding inquiry, may hold inquiry, as early as possible, u/s 202 of Code
of Criminal Procedure, 1898.
Special Judge, etc to have exclusive jurisdiction (U/s 37E)
(a) No court other than the Special Judge having jurisdiction, shall try an offence punishable under this Act;
(b) No other court or officer, except in the manner and to the extent specifically provided for in this Act, shall exercise any
power or perform any function under this Act;
(c) No court other than High court, shall entertain, hear or decide any application, petition or appeal under the aforesaid
Code;
(d) No court other than Special Judge or High court shall entertain any application, petition or pass any order or give any
direction under the aforesaid Code.
Transfer of cases (U/s 37G)
1. In the case of more than 1 Special Judge appointed the High Court or in the case of 1 Special Judge the Federal
Government (by order in writing) direct the transfer, at any
2. When the case is transferred to a Special Judge then he is not bound to recall or rehear any witness whose the
evidence has been recorded before the transfer and may act upon the evidence already recorded or produced before
the court which tried the case before the transfer.

654 Conceptual Approach to Taxes


Offences and Penalties Chapter-09

Place of sittings (U/s 37H)


A Special Judge may hold sittings at headquarters or at any other place as the case may be.
Appeal to the High Court (U/s 37I)
1. Any person, including the Federal Government, the Board, the CIR or Director of Intelligence and Investigation or any
other officer authorized by the board, aggrieved by any order passed or decision made by a Special Judge under this
Act or Code of Criminal Procedure, 1898, may within 60 days from the date of an order or decision, prefer an appeal
to the High Court.
2. The provisions of the Limitation Act, 1908, shall apply to an appeal preferred under sub-section (1).
Power to call for information (U/s 38A)
The CIR may, by notice in writing, require any person, including a banking company, to furnish such information or such
statement for investigation or inquiry in case of tax fraud as the case may be. Further the CIR may require any regulatory
authority to provide information regarding the licenses and authorizations issued by it.
Obligation to produce documents and provide information (Sec. 38B)
(1) Any person required to maintain the record under the Act, on demand by an officer, not below the rank of an Assistant
CIR, by notice in writing, as and when specified in the notice, shall-
(a) produce for examination, such documents or records which the Officer Inland Revenue considers necessary or
relevant to the audit, inquiry or investigation under the Act;
(b) allow the officer of Inland Revenue to take extracts from or make copies of such documents or records; and
(c) appear before the Officer Inland Revenue and answer any question put to him concerning the documents and
records relating to the audit or inquiry or investigation referred to in clause(a) above.
(2) An Officer Inland Revenue conducting an audit, inquiry or, as the case may be, an investigation may require in writing
any person, department, company or organization to furnish such information as is held by that person, department,
company or organization, which, in the opinion of the Officer Inland Revenue is relevant to such audit, inquiry or
investigation.
(3) The Board may require, in writing, any person, department, company or organization, as the case may be, to provide
any information or data held by that person, department, company or organization, which, in the opinion of the Board,
is required for purposes of formulation of policy or administering the Customs, Sales Tax, Federal Excise or Income
Tax.
(4) Every person, department, company or organization shall furnish the information requisitioned by the Board or the
Officer Inland Revenue within the time specified in the notice issued by the Board or, as the case may be, the officer
of Inland Revenue.
Monitoring or tracking by electronic or by other means (Sec. 40C)
(1) Subject to such conditions, restrictions and procedures, as it may deem fit to impose or specify, the Board may, by
notification in the official Gazette, specify any registered person or class of registered persons or any good or class of
goods in respect of which monitoring or tracking of production, sales, clearances, stocks or any other related activity
may be implemented through electronic or other means as may be prescribed.
(2) From such date as may be prescribed by the Board, no taxable goods shall be removed or sold by the manufacturer
or any other person without affixing tax stamp, banderole, stickers, labels, barcodes, etc. in any such form, style and
manner as may be prescribed by the Board in this behalf.
(3) Such tax stamps, bandroles, stickers, labels, barcodes etc., shall be acquired by the registered person referred in sub
section (2) from a licensee appointed by the Board for the purposes against price approved by the Board, which shall
include the cost of equipment installed by such licensee in the premises of the said registered person.

Conceptual Approach to Taxes 655


Offences and Penalties Chapter-09

MULTIPLE CHOICE QUESTIONS


Q.1. Where any person fails to furnish a return within 10 days of the due date, such person shall pay a penalty of
Rs.____________.
(a) 5,000
(b) 10,000
(c) 100
(d) 2,000
Q.2. Any person who fails to issue an invoice when required under this Act, such person shall pay a penalty of
Rs._______ or 3% of the amount of the tax involved, whichever is higher.
(a) 5,000
(b) 10,000
(c) 100
(d) 2,000
Q.3. Any person who un-authorizedly issues an invoice in which an amount of tax is specified, such person shall pay a
penalty of Rs.10,000 or ___ % of the amount of the tax involved, whichever is higher.
(a) 5
(b) 6
(c) 7
(d) 10
Q.4. Any person who fails to notify the changes of material nature in the particulars of registration of taxable activity, Such
person shall pay a penalty of Rs.____.
(a) 10,000
(b) 5,000
(c) 6,000
(d) 7,000
Q.5. Any person who repeats erroneous calculation in the return during a year whereby amount of tax less than the actual
tax due is paid, such person shall pay a penalty of Rs.5,000 or ___ % of the amount of the tax involved, whichever is
higher.
(a) 5
(b) 6
(c) 3
(d) 10
Q.6. Any person who fails to maintain records required under this Act or the rules made there under, such person shall pay
a penalty of Rs.10,000 or ______% of the amount of tax involved, whichever is higher:
(a) 5
(b) 4
(c) 3
(d) 2
Q.7. In the case of person liable to pay amount of tax or charge or refund erroneously made, shall pay default surcharge
@ rate of KIBOR plus ____% p.a. of the amount of tax due or refund made.
(a) 3
(b) 6
(c) 9
(d) 10
Q.8. In the case the person has committed tax fraud shall pay default surcharge @ rate of ___% per month of the amount
of tax evaded or amount of refund fraudulently claimed until the total liability including default surcharge is paid.
(a) 1
(b) 2
(c) 3
(d) 4

656 Conceptual Approach to Taxes


Offences and Penalties Chapter-09

Q.9. In the case of non-payment of tax or part thereof, the period of default surcharge shall be started from the ____ day of
month (following the due date of the tax period to which the default relates) to the day preceding the date on which
tax due is actually paid.
(a) 16th
(b) 20th
(c) 15th
(d) 10th
Q.10. ______, published in Gazette in writing, may exempt any person or class of person from payment of whole or part of
penalty or default surcharge on conditions and limitation as specified therein.
(a) Federal Government
(b) Federal Board of Revenue
(c) both a or b
(d) none of above
Q.11. All arrests made under this Act shall be carried out in accordance of ____.
(a) Code of Criminal Procedure, 1898
(b) Companies Ordinance, 1984
(c) Income Tax Ordinance, 2001
(d) Sales Tax Act, 1990
Q.12. _____ (by notification) may appoint as many Special Judges as he considers necessary and shall specify the
headquarters of each Special Judge and the territorial limits within which he shall exercise jurisdiction.
(a) Board
(b) Federal Government
(c) Commissioner Inland Revenue
(d) Securities and Exchange Commission of Pakistan
Q.13. No person shall be appointed as Special Judge unless he is or has been a ______.
(a) Session judge
(b) Civil judge
(c) Advocate of Supreme Court
(d) None of above
Q.14. Any person, including the Federal Government, the Board, the Commissioner Inland Revenue or Director of
Intelligence and Investigation or any other officer authorized by the board, aggrieved by any order passed or decision
made by a Special Judge under this Act or Code of Criminal Procedure, 1898, may within ___ days from the date of
an order or decision, prefer an appeal to the High Court.
(a) 180
(b) 120
(c) 60
(d) 150
Q.15. A Special Judge may hold sittings at ___________or at any other place as the case may be.
(a) Headquarters
(b) Supreme Court
(c) High court
(d) Appellate Tribunal Inland Revenue

ANSWERS

1 (c) 2 (a) 3 (a) 4 (b) 5 (c)


6 (a) 7 (a) 8 (b) 9 (a) 10 (a)
11 (a) 12 (b) 13 (a) 14 (c) 15 (a)

Conceptual Approach to Taxes 657


Offences and Penalties Chapter-09

ICMAP PAST PAPERS THEORETICAL QUESTIONS


Q. No. 6(a) February 2013 What are the penalties of the following defaults under the provisions of the Sales Tax Act, 1990?
(i) Any person who fails to furnish a return within due date under section 26 of the Act.
(ii) Any person who fails to issue an invoice when required under section 23 of the Act,
(iii) Any person who fails to make payment in the manner prescribed under section 73 of the Act.

Q.4 (c) APRIL 2012 What are the penalties in respect of the following offences u/s 33 of the Sales Tax Act, 1990?
(i) A person who fails to maintain records required u/s 22 and 24 of this Act or the rules made there under.
(ii) Any person who obstructs an authorized officer in the performance of his official duty in general or in respect of
Section 31 of the Act.
(iii) What penalties shall be applicable in respect of the following contraventions of the law:
(a) Failure in furnishing a return;
(b) Non-payment of tax or failure in payment of tax; and
(c) Concealment of Income.

658 Conceptual Approach to Taxes


Appeals Chapter-10

Chapter

10 APPEALS

Section Topic covered


For CA Mod F & ICMAP Students
45A Powers of Board and Commissioner to call for records
45B Appeals
46 Appeals to Appellate Tribunal
47 Reference to High Court
47A Alternative dispute resolution
MCQs with solutions

(For CA MOD F AND ICMAP students)


Powers of Board and Commissioner Inland Revenue (CIR) to call for records (U/s 45A)
1. The Board may, by itself or otherwise, call for and examine the record of any departmental proceedings under this Act
or rules to satisfy himself as to the legality and propriety of decision or order passed by the Officer of Inland Revenue,
It may pass such order as it may think fit.
Further any order shall not be passed which impose or enhance any penalty or fine resulting in the greater amount
than the originally levied unless give an opportunity of showing cause or of being heard to the affected person.
2. Proceedings shall not be initiated if an appeal u/s 45B or 46 is pending.
3. No order shall be made after the expiry of 5 years from the date of original decision or order of sub-ordinate officer.
4. The CIR may call for and examine the record of any departmental proceedings to satisfy him-self as to the legality
and propriety of decision or order passed by the Officer of Inland Revenue (OIR). It may pass such order as it may
think fit.
Appeals (U/s 45B)
1. Any person other than the sales tax department aggrieved by the decision or order passed u/s 10, 11, 25, 36 and 66
by an OIR may within 30 days of the date of receipt of such decision or order prefer appeal to the CIR (Appeals).
Provided that an appeal preferred after the expiry of 30 days may be admitted by the CIR (Appeals) if he is satisfied
that the appellant has sufficient cause for not preferring the appeal within the specified period.
Provided further that appeal shall be accompanied by a fee of Rs.1,000 to be paid in such manner as the board may
prescribed.
Where in a particular case, the CIR (Appeals) is of the opinion that the recovery of tax levied under this Act, shall
cause undue hardship to the taxpayer, he, after affording opportunity of being heard to the CIR or OIR against whose
order appeal has been made, may stay the recovery of such tax for a period not exceeding 30 days in aggregate.
2. The CIR (Appeals) shall give an opportunity of being heard to both parties and may pass such order as he thinks fit,
confirming, varying, altering, setting aside or annulling the order or decision appealed against.
The CIR (Appeals) shall pass an order within 120 days from the date of filing of appeal and he may extend that period
for reasons to be recorded in writing. The extended period shall not exceed the 60 days in any case.
Provided further that any period during which the proceedings are adjourned on account of a stay order or alternative
dispute resolution proceeding or the time taken through adjournment by the petitioner not exceeding 30 days shall be
excluded from the computation of aforesaid periods.

Conceptual Approach to Taxes 659


Appeals Chapter-10

3. In deciding an appeal, the CIR (Appeals) may make such further inquiry as may be necessary provided he shall not
remand the case for denovo consideration.
Appeals to Appellate Tribunal (U/s 46)
1. Any person including an OIR at least an Additional CIR, aggrieved by any order passed by-
(a) The CIR (Appeals),
(b) The CIR through adjudication or under any of the provisions of this Act or rules,
(c) The Board.
may, prefer an appeal to Appellate Tribunal within 60 days of receipt of such order or decision.
2. The Appellate Tribunal may admit, hear and dispose of the appeal in accordance with the section 131 and 132 of the
Income Tax Ordinance, 2001 and rules made there under.
3. All appeals and proceedings under this act pending before the customs, excise and sales tax appellate tribunal
constituted u/s 194 of the Customs Act, 1969 shall stand transferred to the appellate tribunal constituted u/s 130 of
the income tax ordinance, 2001 w. e. f 28th day of October 2009.
Reference to High Court (U/s 47)
1. The aggrieved person or any OIR at least an Additional CIR,, authorized by the CIR, may prefer an application in the
prescribed form along with a statement of the case to the High Court, stating any question of law arising out of such
order. The application shall be made within 90 days of the communication of the order of the Appellate Tribunal.
2. The statement to the High Court shall set out the facts, the determination of the Appellate Tribunal and the question
of law which arises out of its order.
3. The High Court may proceed to hear the case if, on an application, satisfied that a question of law arises out of the
order as above.
4. A reference to the High Court shall be heard by a bench consisting of at least two judges of the High Court and in
respect of reference.
5. The High Court upon hearing a reference shall decide the question of law raised and deliver judgment specifying the
grounds and the order of Tribunal shall stand modified accordingly. The Court shall send a copy of the judgment
under the seal of the Court to the Appellate Tribunal.
6. The cost of any reference to High Court shall be in the direction of the Court.
7. The tax shall be payable in accordance with the Appellate Tribunal order irrespective of reference has been made to
High Court.
Provided that if the amount of tax is reduced as a result of the judgment in the reference by the high court and
amount of tax found refundable by the high court, the high court may on application by an additional CIR authorized
by the CIR within 30 days of receipt of the judgment of the high court, that he intends to seek leave to the supreme
court, make an order authorizing the CIR to postpone the refund until the disposal of the appeal by the supreme
court.
8. Where an order, by the High Court, has stayed the recovery of tax, shall cease to have effect after 6 months of the
date of order unless the High Court decide the appeal or withdraw such order earlier.
9. Section 5 of the Limitation Act, 1908 shall apply to an application made to the High Court.
10. The person other than the Additional Commissioner authorized by the CIR shall pay a fee of Rs.100 with an
application.
11. Where any reference or appeal was filed with the approval of CIR by the officer of lower rank then the CIR and the
reference or appeal is pending before an appellate forum for the court, such reference or appeal shall always be
deemed to have been so filed by the CIR.
Alternative dispute resolution (U/s 47A)
1. Any registered person aggrieved in connection with any dispute pertaining to:
(a) The liability of tax against the registered person or admissibility of refunds, as the case may be;
(b) The extent of waiver of default surcharge and penalty;
(c) The quantum of input tax admissible in terms of section 7(3).
(d) Relaxation of any procedural or technical irregularities and condonation of any prescribed time limitation;
(e) Any other specific relief required to resolve the dispute,

660 Conceptual Approach to Taxes


Appeals Chapter-10

May apply to the board for the appointment of a committee for the resolution of any hardship or dispute mentioned in
detail in the application which is under litigation in any court of law or an appellate authority, except in the cases
where first information (FIRs) have been lodged under the act or criminal proceedings initiated or where the
interpretation of question of law having larger revenue impact in the opinion of the Board is involved, may apply to the
Board for the appointment of a committee for the resolution of dispute in appeal and only such application may be
entertained for dispute resolution under the provisions of the section.
2. After examination of the application of a registered person, the Board may appoint a committee within 30 days of
receipt of such application. Such committee shall consist of an officer of Inland Revenue not below the rank of
Additional Commissioner and 2 persons from the notified penal consisting of retired judges not below District and
Sessions judge, Chartered or Cost accountant, advocates, representative of trade bodies or associations or any other
reputable taxpayers, for the resolution of dispute.
3. The committee constituted above shall examine the issue and may if deems fit, conduct enquiry, seek expert opinion,
direct any OIR or any other person to conduct an audit and shall make recommendation within 90 days of its
constitutions in respect of the dispute. If the committee fails to make recommendation within the said period, the
board shall dissolve the committee and constitute a new committee which shall decide the matter within a further
period of 90 days. If after the expiry of that period the dispute is not resolved, the matter shall be taken up by the
appropriate forum for decision.
4. On the recommendation of the committee, the Board may pass such order, as it may deem appropriate within 45
days of recommendations of committee.
5. The Chairman Board and a Member nominated by him may, on the application of an aggrieved person, for reasons to
be recorded in writing, and on being satisfied that there is an error in order or decision, pass such order as may be
deemed just and equitable.
6. The registered person may make payment of sales tax and other duty and taxes as determined by the board in its
order and such order of the board shall be submitted before the forum, tribunal or the court where the matter is
subjudice for consideration of order as deemed appropriate.
7. The Board by notification in the official gazette may make rules for carrying out the purposes of this section.

Conceptual Approach to Taxes 661


Appeals Chapter-10

MULTIPLE CHOICE QUESTIONS


Q.1. ____ may, by itself, call for and examine the record of any departmental proceedings under this Act.
(a) Board
(b) Appellate Tribunal Inland Revenue
(c) Commissioner Inland Revenue
(d) Chief Commissioner Inland Revenue
Q.2. No order shall be made after the expiry of ___ years from the date of original decision or order of sub-ordinate officer.
(a) 6
(b) 5
(c) 7
(d) 10
Q.3. An appeal preferred after the expiry of 30 days may be admitted by the ____if he is satisfied that the appellant has
sufficient cause for not preferring the appeal within the specified period.
(a) Commissioner Inland Revenue
(b) Chief Commissioner Inland Revenue
(c) Commissioner Inland Revenue (Appeals)
(d) Board
Q.4. Appeal shall be accompanied by a fee of ____to be paid in such manner as the board may prescribed.
(a) Rs.4,000
(b) Rs.3,000
(c) Rs.2,000
(d) Rs.1,000
Q.5. The Commissioner Inland Revenue (Appeals) shall pass an order within 120 days from the date of filing of appeal
and he may extend that period for reasons to be recorded in writing. The extended period shall not exceed the ___
days in any case.
(a) 70
(b) 60
(c) 120
(d) 30
Q.6. Application to the High Court shall be made within ___ days of the communication of the order of the Appellate
Tribunal.
(a) 90
(b) 100
(c) 120
(d) 365
Q.7. A reference to the High Court shall be heard by a bench consisting of at least ___ judges of the High Court and in
respect of reference.
(a) 3
(b) 2
(c) 4
(d) 5
Q.8. On the recommendation of the alternative dispute resolution, the Board may pass such order, as it may deem
appropriate within ___ days of recommendations of committee.
(a) 45
(b) 100

662 Conceptual Approach to Taxes


Appeals Chapter-10

(c) 120
(d) 365
Q.9. The person other than the Additional Commissioner authorized by the CIR shall pay a fee of Rs.___ with an
application to High Court.
(a) 200
(b) 100
(c) 150
(d) 300
Q.10. Any person including an officer of Inland Revenue at least an Additional Commissioner, aggrieved by any order
passed by the Commissioner Inland Revenue (Appeals), the Commissioner Inland Revenue through adjudication or
under any of the provisions of this Act or rules, the Board may, prefer an appeal to Appellate Tribunal within ___ days
of receipt of such order or decision.
(a) 60
(b) 70
(c) 80
(d) 90

ANSWERS

1 (a) 2 (b) 3 (c) 4 (d) 5 (b)


6 (a) 7 (b) 8 (a) 9 (b) 10 (a)

Conceptual Approach to Taxes 663


Appeals Chapter-10

664 Conceptual Approach to Taxes


Sales Tax Special Procedure Rules, 2007 Chapter-11

Chapter

11 SALES TAX SPECIAL PROCEDURE


RULES, 2007

Arrangement of Rules

SR. RULES PARTICULARS


1. 1 Short title, application and commencement
2. 2 Definitions
3. 3 TO 10 Special Procedure for payment of Sales tax by Retailers
4. 11 TO 18 Special Procedure for payment of Sales tax on Electric Power
5. 18A TO 18C Special Procedure for collection and payment of extra tax on supplies of electric power and
natural gas consumed by unregistered and inactive persons
6. 19 TO24 Special Procedure for collection and payment of sales tax on natural gas
7. 25 TO 28 Special Procedure for supply of sugar to Trading Corporation of Pakistan (TCP)
8. 29 TO 35 Special Procedure for Persons Providing or Rendering Services Subject to Sales Tax under the
Provincial Laws
9. 36 TO 37 Advertisement on Television and Radio
10. 38 TO 39 Customs Agents and Ship Chandlers
11. 39A Services Provided by Stevedores
12. 40 TO 44 Special Procedure for collection and payment of Sales Tax from the Oil Marketing Companies
(Sharing of Product)
13. 45 TO 53 Special Procedure for collection and payment of Sales Tax by Vehicle Dealers
14. 54 TO 58 Special Procedure for Processing of Refund Claims filed by the persons engaged in Making Zero
rated supply of Ginned Cotton
15. 58A TO 58E Special Procedure for payment of Sales tax by Importers
16. 58F TO 58MC Special Procedure for payment of Sales Tax by Steel Melters, Re-rollers and Ship Breakers
17. 58N TO 58R Special Procedure for payment of Sales Tax by Wholesale Cum Retail Outlets
18. 58RA Miscellaneous
19. 58S TO 59 Special Procedure for payment of Extra Tax on specified Electric Home Appliances

(For CA MOD F AND ICMAP students)


Notification No. S.R.O. 480(1)/2007, Islamabad, the 9th June, 2007,- In exercise of the powers conferred by section 71 of
the Sales Tax Act, 1990, read with clauses (9) and (46) of section 2, sections 3 and 4, sub-section (2) of section 6, section 7,
section 7A, clause (b) of sub-section (1) of section 8, clause (a) of sub-section (2) of section 13, sub-sections (2A) and (3) of
section 22, sections 23 and 60 thereof, the Federal Government is pleased to make the following rules, namely:--
THE SALES TAX SPECIAL PROCEDURES RULES, 2007

1. Short title, application and commencement.-


(1) These rules may be called the Sales Tax Special Procedures Rules, 2007.
(2) They shall apply to such persons as are specified in the respective Chapter.
(3) These shall come into force with effect from the 1st day of July, 2007.

Conceptual Approach to Taxes 665


Sales Tax Special Procedure Rules, 2007 Chapter-11

CHAPTER I PRELIMINARY

2. Definitions.-
(1) In these Rules, unless there is anything repugnant in the subject or context,-
i. "Act" means the Sales Tax Act, 1990;
ii. Annex means an Annex to these rules;
iii. "NEPRA" means the National Electric Power Regulatory Authority established u/s 3 of the Regulation of
Generation, Transmission and Distribution of Electric Power Act, 1997;
iv. "CNG station" means any place or premises from where Compressed Natural Gas (CNG) is supplied to,
or filled in cylinders or tankers;
v. "Collectorate" means the office of the Collector of Sales Tax having jurisdiction and includes the Large
Taxpayers' Unit (LTU) and the Regional Tax Office (RTO), where the offices of Income Tax, Sales Tax
and Federal Excise are co-located, and the word "Collector" shall be construed accordingly;
vi. "commission", in case of a car dealer, means the amount payable by the
vii. consumer to the dealer for the purpose of intermediating sale, booking, delivery or other related
services or activities in respect of a vehicle and includes any other amount charged from a consumer or
seller over and above the price of the vehicle;
viii. "consumer", in relation to Chapter III, means a person or his successor-in- interest who purchases or
receives electric power for consumption and not for delivery or resale thereof to others and includes a
person who owns or occupies a premises where electric power is supplied;
ix. "courier service" means delivery of documents, goods or articles utilizing the services of a person, either
directly or indirectly, to carry or accompany such documents, goods or articles for consideration;
x. "distribution", in relation to Chapter III, means the ownership, operation, management or control of
distribution facilities for the movement or delivery or sale to consumers of electric power but shall not
include the ownership, operation, management and control of distribution facilities located on private
property and used solely to move or deliver electric power to the person, owning, operating, managing
and controlling those facilities or to tenants thereof shall not constitute distribution;
xi. "Fiscal Electronic Cash Register" or "FECR" means an electronic cash register with fiscal memory
(black box), fiscal screw and seal, capable of simultaneously printing second copy (record copy) that
contains all information in addition to that on the first paper roll (customer copy) and having two
displays, one for operator and the other for customer;
xii. "gas bill" means the bill of charges issued by the gas transmission and distribution companies to their
consumers pertaining to a tax period for natural gas supplied by them;
xiii. "generation", in relation to Chapter III, includes the ownership, operation, management or control of
generation facilities for delivery or sale of electric power and not solely for consumption by the person
owning, operating, managing and controlling those facilities;
xiv. "HUBCO" means the Hub Power Company Limited;
xv. "IPP" means an Independent Power Producer established in private sector
xvi. operating under a license issued by the NEPRA for the purpose of generation, transmission, distribution
and sale of electric power, and governed by various Implementation Agreements executed between the
Islamic Republic of Pakistan and such Independent Power Producer and includes HUBCO and KAPCO;
xvii. jeweller means any person engaged in the supply of ornaments as a manufacturer, wholesaler or
retailer, but does not include a zargar;
xviii. JIMCO means joint installation of the oil marketing companies at Mehmood Kot, District Gujrat,
Punjab;
xviiia. National or international chain stores includes a chain of more than on retail outlets having the same
brand name or trade name or trade mark or logo, engaged in the retail sale of goods and operating
under a single or joint ownership or as a franchise or any other arrangement;
xix. "KAPCO" means the Kot Addu Power Company Limited;
xx. "KESC" means the Karachi Electric Supply Corporation;
xxi. "natural gas" means the gas obtained from bore-holes and wells whether unmixed or mixed with
artificial gas consisting primarily of hydrocarbons whether gaseous or in liquid form which are not oils
and includes liquefied petroleum gas (LPG) and compressed natural gas (CNG);

666 Conceptual Approach to Taxes


Sales Tax Special Procedure Rules, 2007 Chapter-11

xxii. "OMC" means the oil marketing company and includes Shell Pakistan Limited, Chevron Pakistan
Limited and Pakistan State Oil (PSO);
xxiii. "private sector project" means a facility for generation, transmission or distribution of electric power
constructed, owned, managed or controlled by any one or more organizations or companies
incorporated under the Companies Ordinance, 1984 (XLVIII of l984);
xxiv. "product sharing" means acquiring of a product by one OMC from another OMC on loan basis, without
payment of price under an arrangement of returning the product of the same description by the former
to the latter, within such time as may be agreed between them;
xxv. "public sector project" means a facility for generation, transmission or distribution of electric power
constructed, owned, managed or controlled by the Federal Government, a Provincial Government, a
local authority or anybody owned or controlled by any such Government or authority;
xxvi. (xxiii-a) "stevedore" means a person, company or commercial concern engaged in loading and
unloading of cargo, including bulk cargo, from ships, whether mechanically or otherwise, and whether or
not licensed by the respective port authorities;
xxvii. "TCP" means the Trading Corporation of Pakistan;
xviiia. "taxable services" means the services chargeable to sales tax under the respective Provincial law, and
include all such services, utilities or facilities, by whatever name called, which are provided or rendered
by a service provider to his clients or customers or members;
xxviii. "Terminal Operator" means the company or person managing the affairs of joint installation (JIMCO) at
Mehmood Kot, District Gujrat;
xxix. "value of taxable services", in relation to hotels and courier services, means the gross amount charged
or the consideration in money including all Federal and Provincial levies, if any, which a service provider
receives from the clients or customers or members for providing or rendering taxable services, but
excluding the amount of sales tax:
Provided that in case the consideration for providing a taxable service is in kind or is partly in kind
and partly in money or the service provider and recipient or client are associated persons and the
service is provided for no consideration or for a consideration which is lower than the open market
value, the value of taxable service shall mean the open market value for providing the taxable
service, excluding the amount of tax:
Provided further that value of taxable service in relation to clubs for the purpose of levy of sales tax
shall not include consideration received on account of membership fees, refundable deposit or
security unless the same is deducted or adjusted in full or in part as settlement or recovery of dues
for services;
xxx. "vehicles" include all types of vehicles covered under Chapter 87 of the Pakistan Customs Tariff other
than headings 87.12, 87.15 and 87.16 thereof, as are generally used for the transportation of persons or
goods including three and two wheelers; and
xxxi. "zargar" means any person who is engaged in the making of ornaments or carrying out any related
process on labour charge basis and is not involved in the sale of ornaments to ordinary consumers.
(2) The words and expressions used, but not defined herein, shall have the same meanings as are assigned to
them in the Act.
(3) All provisions of any other rules made under the Act, in so far as they are not in consistent with these rules
shall, mutatis mutandis, apply to the registered persons operating under these rules.
CHAPTER II
SPECIAL PROCEDURE FOR PAYMENT OF SALES TAX
BY RETAILERS UNDER RULE 3 TO 10 OF SPECIAL PROCEDURE RULES 2007:
Application U/R 3
The provisions of this Chapter shall apply to all persons who make supplies from retail outlets to end consumers, including
jewelers and wholesalers cum retailers, whether registered or not, who shall be deemed to be retailers in respect of such
supplies for the purpose of this chapter and also to persons making supplies of electric power to retailers;
Provided that the provisions of this chapter shall not be applicable to the following registered persons, namely:-
(i) vehicle dealers paying sales tax in the manner prescribed in Chapter VIII;
(ii) registered retailers exclusively making supplies of goods specified in Chapter XIII, on which extra tax has already
been paid in the manner prescribed therein.

Conceptual Approach to Taxes 667


Sales Tax Special Procedure Rules, 2007 Chapter-11

Registration U/R 4
Notwithstanding anything contained in clause (b) of rule 4 of the Sales Tax Rules, 2006, retailers falling in any of the
following categories shall be required to be registered as a retailer under the Act, in the manner specified in Chapter I of the
Sales Tax Rules, 2006.
(a) a retailer operating as a unit of national or international chain of stores;
(b) a retailer operating in an air conditioning shopping mall, plaza or centre, excluding kiosks;
(c) a retailer who has a credit or debit card machine;
(d) a retailer whose cumulative electricity bill during the immediately preceding twelve consecutive months exceeds
rupees six hundred thousand (Rs. 600,000): and
(e) a wholesalers cum retailer, engaged in bulk import and supply of consumer goods on wholesale basis to the retailers
as well as on retail basis to the general body of the consumers:
Provided that the provisions of this Chapter shall remain applicable to retailers who do not obtain registration:
Provided further that the retailers operating as a unit of a franchise or any other arrangement of a national or
multinational chain of stores, shall obtain a separate registration as distinct from their principal.
Retailers required to pay tax on standard rate U/R 5
(1) Retailers specified in rule 4 shall pay sales tax at the rate specified in sub-section (1) of section 3 of the Act and shall
observe all the applicable provisions of the Act and rules made there under, including the requirement to file monthly
sales tax returns in the manner prescribed in Chapter II of the Sales Tax Rules, 2006:
Provided that the retailers making supplies of finished goods of the five sectors specified in Notification No. S.R.O.
1125(1)/2011, dated the 31st December, 2011 shall pay sales tax in respect of such supplies at the rates prescribed
in the said Notification.
(2) Subject to rule 8, retailers specified in rule 4 shall be required to install and operate Fiscal Electronic Cash Registers
(FECRs), and to issue invoices only there from to their customers.
(3) Retailers shall provide seamless and real-time access of their FECRs data to the Board and also allow on-site
physical inspection as and when authorized by the commissioner Inland Revenue having jurisdiction.
(4) While determining the taxable supplies made by jewelers, a jeweler shall be entitled to exclude the value of gold or
silver used in the jewelry supplied, provided that such assessable value for the purpose of taxable supply is not less
than 10% of the actual sale price excluding the amount of tax.
Other retailers shall pay sale sax through electricity bills U/R 6: -

(1) Retailers not falling in the categories specified in sub-rule(1) of rule 5, shall be charged sales tax through their
electricity bills by the persons making supplies of electric power, at the rates specified in sub-section (9) of section 3
of the Act, in the manner as specified hereunder, which shall be in addition to the tax charged on supply of electricity
under sub-section (1), (1A) and (5) of section 3 of the Act,
(2) Every person making supplies of electric power shall charge and collect sales tax at the rates specified in sub-section
(9) of section 3 of the Act, from every retailer having a commercial electricity connection:
Provided that sales tax under sub-section (9) of section 3 of the Act shall not be charged in cases where the person
making supplies of electric power receives a written order from the commissioner of Inland Revenue of the effect that-
(a) The consumer is not engaged in any retail business; of
(b) The consumer is already registered and paying sales tax through monthly sales tax returns.
(3) The amount of sales tax charged from retailers shall be shown separately in the electricity bill or invoice issued by the
supplier of electric power,
(4) The supplier of electric power shall collect and pay the amount of sales tax from retailers in the manner as prescribed
in chapter III.
Conditions and limitations U/R 7.-
(1) The amount of sales tax charged and collected though the electricity bill in terms of rule 6 shall not be adjustable by
the supplier of electric power and shall be paid by him in full into the Treasury.
(2) The tax paid through electricity bill by a retailer as prescribed in rule 6, shall be construed as the discharge of final tax
liability for the purpose of sales tax and he shall not be entitle for and input tax adjustment or refund there from.
Issuance of invoice or cash memo U/R 8; -
Every retailer operating under rules 5 shall issue serially numbered invoices or, as the case may be, cash memos in respect
of each supply made by him, manually or through electronic cash register, and from such date as may be specified by the
Board, the invoices shall be issued through Fiscal Electronic Cash Register.

668 Conceptual Approach to Taxes


Sales Tax Special Procedure Rules, 2007 Chapter-11

Payment of sale tax and filing of return U/R 9.


(1) Every retailer operating under rule 5 shall deposit the sales tax due along with his return on monthly basis in the
manner prescribe in Chapter II of the Sales Tax Rules, 2006.
(2) A retailer operating under rule 6 shall not be required to file monthly sales tax return.
Audit or scrutiny of record U/R 10.
(1) A retailer operating under rule 5 shall be subject to audit as per normal procedure.
(2) Retailers operating under rule 6 shall not be subject to audit provided they are properly paying the sales tax as
specified in sub-section (9) of section 3 of the Act through their electricity bills.

CHAPTER III
SPECIAL PROCEDURE FOR COLLECTION AND PAYMENT OF SALES
TAX ON ELECTRIC POWER
11. Application.- The provisions of this Chapter shall apply for collection and payment of sales tax on electric power
imported, generated, produced, transmitted and supplied by electricity generation, transmission and distribution
companies licensed under the Regulation of Generation, Transmission and Distribution of Electric Power Act, 1997,
including their distributors, dealers and agents, or by any other person dealing in importation, generation, production,
transmission, distribution and supply of electric power.
12. Registration.- Every electricity generation, transmission and distribution company licensed by NEPRA, including a
distributor, dealer and agent of such company, an Independent Power Producer, a Public Sector Project, Private
Sector Project, or any other person dealing in importation, generation, production, transmission, distribution and
supply of electric power shall, if not already registered, obtain registration in the manner prescribed in Chapter I of the
Sales Tax Rules, 2006.
13. Levy and collection of sales tax.-
(1) Every person, referred to in the preceding rule, who supplies electric power shall charge and collect sales tax
at the rate specified in sub-section (1) of section 3 of the Act.
(2) Subject to sub-rule (3), sales tax on electric power shall be levied and collected at the following stages,
namely:-
(a) in case of its importation, the responsibility to pay sales tax shall be of the importer, and the value
thereof shall be the value as determined u/s 25 or, as the case may be, section 25B of the Customs Act,
1969, including the amount of customs-duties and duty of excise duties levied thereon; and
(b) in case of generation, transmission, distribution and supply of electric power by a public sector project
like WAPDA a private sector project including an IPP, a Captive Power Unit or any other person, the
responsibility to collect sales tax shall be of the person making the supply, and the value shall be the
price of' electric power including all charges, surcharges excluding the amount of late payment
surcharge, rents, commissions and all duties and taxes whether local, Provincial or Federal, but
excluding the amount of sales tax, as provided in clause (46) of section 2 of the Act:
Provided that in case of electric power supplied by WAPDA, the additional charge of Rs.0.10 per kWh,
collected on account of Neelum Jehlum Hydro Power Development Fund shall not be included in value for
determination of sales tax payable.
(3) In case of an IPP, HUBCO, KAPCO or WAPDA Hydroelectric Power, the value of supply shall be the amount
received by such IPP or, as the case may be, HUBCO, KAPCO or WAPDA Hydroelectric Power, on account of
Energy Purchase Price only and any amount in excess of Energy Purchase Price received on account of
Capacity Purchase Price, Energy Price Premium, Excess Bonus, Supplemental Charges, etc., shall not be
deemed as a component of the value of supply:
Provided that in case WAPDA or KESC disputes any amount, WAPDA or, as the case may be, KESC, shall
issue a certificate showing such amount and the tax involved therein and such certificate shall be deemed to
be a Credit Note for the IPP for the purposes of section 9 of the Act, and shall be accounted for in the return for
the tax period in which such Credit Note is issued:
Provided further that in case an IPP, for the like reasons, receives any amount from WAPDA or KESC in
respect of supply made during any pervious tax period, tax on such amount shall be accounted for in the return
for the period in which it is received.
14. Filing of returns and deposit of sales tax.-
(1) In case of WAPDA and KESC, sales tax levied and collected under rule 13 during a tax period shall be
deposited on 'accrual basis' i.e. the amount of sales tax actually billed to the consumers or purchasers for the
tax period.

Conceptual Approach to Taxes 669


Sales Tax Special Procedure Rules, 2007 Chapter-11

(2) WAPDA and KESC shall submit the monthly return as prescribed under section 26 of the Act, by the 21st day
of the month following the month in which the electric power bill or invoice has been raised. The tax due shall
be deposited in the Government Treasury under the relevant head "B0234l-Sales Tax" along with the
prescribed return under Chapter II of the Sales Tax Rules, 2006.
(3) In case of an IPP, the due date for the purpose of filing monthly sales tax return and for payment of sales tax
shall be the 25th day of the month following the month to which the sales tax invoice relates.
(4) Any person other than an IPP, WAPDA or KESC who supplies electric power shall file a monthly sales tax
return under section 26 of the Act and Chapter of the Sales Tax Rules, 2006, and deposit the amount of sales
tax payable for the tax period by the due date.
15. Determination of sales tax liability in respect of WAPDA and KESC.-
(1) Any person, except WAPDA and KESC, which supplies electric power shall be entitled to claim admissible
input tax adjustment in the manner specified in section 7 of the Act, read with sections 8 and 8B thereof.
(2) WAPDA and KESC shall be entitled to claim admissible input tax adjustment against sales tax paid on their
taxable purchases made in the month immediately preceding the tax period.
(3) The WAPDA shall henceforth be entitled to claim input tax paid by it on price differential of Low Sulphur
Furnace Oil (LSFO) and High Sulphur Furnace Oil (HSFO) to PSO on behalf of KAPCO for generation of
electricity by KAPCO, subject to the condition that PSO mentions this apportionment on the invoices issued to
M/s. KAPCO & WAPDA in each transaction and the same is verifiable from the accounts of both WAPDA and
KAPCO.
16. Input tax adjustment for registered consumers.-
(1) In case of registered consumers, the electric power bill issued by electric power distribution company shall be
treated as a tax invoice as defined in clause (40) of section 2 of the Act.
(2) The registered consumers shall be entitled to claim input tax adjustment against such invoice after the bill has
been paid, as per the provisions of section 7, 8 and 8B of the Act provided the bill contains registration number
and address of the business premises declared to the Collector by such consumer.
17. Record keeping and invoicing.-
(1) Every person who supplies electric power shall maintain records as prescribed under section 22 of the Act or a
notification issued there under.
(2) Every person who supplies or distributes electric power shall print in his bill or invoice, as the case may be,
registration number of the consumer, if applicable, the rate and the amount of sales tax required to be charged
by him under sub-section (1) of section 3 of the Act.
(3) Every person who supplies electric power and using computerized accounting system may issue a computer
generated sales tax invoice and keep his record on the computer in the prescribed format.
18. Penalty.-
(1) Non-issuance of electric power bill for a tax period or any inordinate delay in the issuance of such bill by the
electric power transmission and distribution companies or by any registered person engaged in the supply of
electric power shall be liable to penalties under the relevant provisions of the Act.
(2) If the tax is not paid within the due date or in the manner as provided under this Chapter, the registered person
shall be liable to pay default surcharge and such other penalties prescribed in the Act.

CHAPTER IVA
SPECIAL PROCEDURE FOR COLLECTION AND PAYMENT OF
EXTRA TAX ON SPPLIES OF ELECTRIC POWER AND NATRUAL GAS
CONSUMED BY UNREGISTERED AND INACTIVE PERSONS
18A. Application The provisions of this chapter shall apply to supplies of electric power and natural gas consumed by
persons having industrial or commercial connections.
18B. Mode and manner of collection.-
(1) Every person supplying electric power of natural gas, shall charge and collect extra tax at the rate notified by
the Federal Government, from every consumer having an industrial or commercial connection, where the bill
for a month is in excess of rupees fifteen thousand, and the consumer either has not provided his sales tax
registration number to the supplier or his name is not shown as active on the Active Taxpayers List (ATL)
maintained by the Federal Board of Revenue.
(2) The amount of extra tax shall be shown separately in the bill or invoice for electric power or natural gas issued
by the supplier.
(3) The supplier shall collect and pay the amount of extra tax in the manner prescribed in Chapter III and IV, as
the case may be.

670 Conceptual Approach to Taxes


Sales Tax Special Procedure Rules, 2007 Chapter-11

18C. Conditions and limitations.-


(1) The amount of extra tax shall not be adjustable by the supplier or the consumer in their returns, and shall be
paid in full by the supplier into the Treasury.
(2) Where a person claims that he has a sales tax registration number, the supplier of electric power or natural
gas, as the case may be, shall require him to produce the sales tax registration certificate, and shall verify from
the Active Taxpayers List maintained by the Federal Board of Revenue that the person is actually registered
and is appearing as active thereon. The supplier shall also confirm that the name, address and other
particulars appearing on the registration certificate or Active Taxpayers List, as the case may be, are the same
as that of the electric power or natural gas connection.
(3) A person having multiple places of business shall ensure that all such places of business are properly
declared and entered on his registration certificate and Active Taxpayers List.
(4) After a person produces sales tax registration certificate in his name, and he is verified as active on the Active
Taxpayer List, the supplier shall incorporate the sales tax registration number in his billing system so that it is
printed on future bills. Thereafter, the supplier shall stop charging and collecting the extra tax from such
person.
(5) The supplier shall again start charging and collecting extra tax from the consumer from the month in which he
is de-registered from sales tax or he does not remain active on the Active Taxpayers List.
CHAPTER IV
SPECIAL PROCEDURE FOR COLLECTION AND PAYMENT OF SALES TAX ON NATURAL GAS

19. Application.- The provisions of this Chapter shall apply for collection and payment of Sales Tax on Natural Gas
including Compressed Natural Gas (CNG) and Liquefied Petroleum Gas (LPG) imported, produced, transmitted and
supplied by gas well-head companies and gas transmission and distribution companies licensed under the Natural
Gas Rules, 1960, including their distributors, dealers, sales agents, retailers or by any other person hereinafter called
the "person" for the purposes of this Chapter and dealing in importation, production or distribution and supply of
Natural Gas including Compressed Natural Gas and Liquefied Petroleum Gas.
20. Levy and collection of sales tax.-
(1) Every person who supplies natural gas shall be liable to registration and shall charge and pay sales tax at the
rate specified in sub-section (1) of section 3 of the Act.
(2) Sales tax on natural gas shall be levied and collected at the following stages and in the following manners,
namely:-
(a) in case of its importation, the responsibility to pay sales tax shall be of the importer who shall pay in the
manner prescribed in sub-section (1) of section 6 of the Act, and the value thereof shall be the value as
determined u/s 25 or 25B of the Customs Act, 1969, read with section 31A thereof, including the
amount of customs-duties and Federal excise duties levied thereon;
(b) in case of production and supply from the bore-holes and wells, the person responsible to charge and
pay sales tax shall be the person making the supply at the bore-holes or the well-heads. The value for
the purposes of levy of sales tax shall include price of natural gas, charges, rents, commissions and all
duties and taxes, local, Provincial and Federal but excluding the amount of sales tax, as provided in
clause (46) of section 2 of the Act;
(c) in case of supply of natural gas by a gas transmission and distribution company, the person responsible
to charge, collect and deposit sales tax shall be the gas transmission and distribution company and the
value for the purpose of tax shall be the total amount billed including price of natural gas, charges
excluding the amount of late payment surcharge, rents, commissions and all duties and taxes, local,
Provincial and Federal, but excluding the amount of sales tax as provided in clause (46) of section 2 of
the Act:
Provided that in case of supply of natural gas to CNG stations, the Gas transmission and distribution
company shall charge sales tax at the rate of 9% of the value in addition to the tax chargeable under
section 3 of the Act or a notification issued there under:
Provided that CNG stations, if not already registered, shall obtain registration under Chapter I of the
Sales Tax Rules, 2006, and shall also file quarterly sales tax return in the manner given in rule 7; and
(d) in case of supply of LPG, the person responsible to charge, collect and deposit sales tax shall be the
person who is a manufacturer, dealer, distributor or a retailer of LPG and the value of LPG for the
purposes of levy of sales tax shall include price of LPG, charges, rents, commissions and all duties and
taxes, local, Provincial and Federal, but excluding the amount of sales tax as provided in clause (46) of
section 2 of the Act.
(3) If the supplies are made free of charge or for some other consideration or a consideration which is lower than
the billed or invoiced prices, the sales tax shall be charged as if it were supplied at open market price in terms
of sub-clause (a) of clause (46) of section 2 of the Act.

Conceptual Approach to Taxes 671


Sales Tax Special Procedure Rules, 2007 Chapter-11

21. Determination of tax liability. - While determining his tax liability, the person supplying or distributing natural gas
shall be entitled for input tax credit for the tax paid on his purchases for making taxable supplies against output tax
payable subject to the limitations and restrictions imposed under sections 7, 8 and 8B of the Act and the notifications
issued there under:
Provided that the gas distribution companies may deduct input tax paid by them on purchase of natural gas as is
subsequently supplied by them in Azad Jammu and Kashmir from the output tax.
22. Record keeping and invoicing.-
(1) Every person supplying or distributing natural gas shall issue a serially numbered sales tax invoice for every
supply made by him.
(2) The bill or invoice issued by the person supplying or distributing natural gas shall, inter alia, indicate the rate
and amount of sales tax required to be charged by him under sub-section (1) of section 3 of the Act:
Provided that the monthly gas bill or invoice issued to a registered consumer shall also contain registration
number of that consumer, and such bill or invoice shall be deemed to be tax invoice in terms of section 23 of
the Act.
(3) The registered consumers shall be entitled to claim input tax adjustment against such invoice after the bill has
been paid, as per the provisions of sections 7, 8 and 8B of the Act, subject to the condition that the bill
contains registration number and address of the business premises declared to the Collector by such
consumer.
(4) The registered persons supplying natural gas using computerized accounting system may, issue computer-
generated sales tax invoices and keep their record on computer in the prescribed format.
(5) The registered person supplying natural gas shall maintain records as prescribed u/s 22 of the Act, including
record of daily stocks and sales, stating therein the quantity and value of the gas supplied and the amount of
sales tax charged thereon, provided that the gas transmission and distribution companies shall not be required
to maintain records of daily stocks and sales.
23. Filing of monthly return.- Every person supplying or distributing natural gas shall submit monthly return as
prescribed in the Act. The tax due shall be deposited in the Government Treasury under the relevant head "B02341-
Sales Tax" by the 15th day of the month following the month in which the gas has been supplied:
Provided that in case of gas supplied by gas companies to its consumers directly and charges are billed on a monthly
basis, the date shall be the 15th day of the second month following the month in which supplies were made.
24. Penalty.
(1) Non-issuance of gas bill or invoice for a tax period or any inordinate delay in the issuance of such bill by the
person engaged in supplying or distributing natural gas shall be liable to penalties under the relevant
provisions of the Act.
(2) If the tax is not paid within the date due as provided under this Chapter, the registered person supplying or
distributing natural gas shall be liable to pay default surcharge and such other penalties prescribed in the Act.

CHAPTER V
SPECIAL PROCEDURE FOR SUPPLY OF SUGAR TO TRADING CORPORATION OF PAKISTAN (TCP)
25. Application. - The provisions of this Chapter shall be applicable in case of supply of sugar by the registered
manufacturers of sugar to the TCP for further supply or export thereof.
26. Manner of payment of tax-
(1) Upon successful grant of tender for purchase of sugar, TCP will only pay the value of supply of sugar to the
sugar mills excluding the amount of sales tax against a Commercial Invoice issued by the mills.
(2) At the time of removal of sugar from the mill premises, the mill will issue a sales tax invoice in favour of TCP
who will accordingly pay to the mill the amount of sales tax due on the quantity being removed from the sugar
mill.
(3) In the event of removal of sugar by TCP for export purposes, the mill will issue a zero-rated tax invoice,
against which no sales tax shall be payable.
27 Relevant tax period.- The mill will show the value of sugar sold to TCP and the tax chargeable thereon in the
monthly tax return as well as in its supply register relating to the tax period in which the sales tax invoice has been
issued by the mill in favour of TCP.
28. Monthly statement by TCP.- TCP shall submit a monthly statement to the Collector in the format set out at Annex-A,
which shall be used by the Collector for cross verification of the supplies declared by the sugar mills as having been
made to the TCP.

672 Conceptual Approach to Taxes


Sales Tax Special Procedure Rules, 2007 Chapter-11

CHAPTER VI
SPECIAL PROCEDURE FOR PERSONS PROVIDING OR RENDERING SERVICES
SUBJECT TO SALES TAX UNDER THE PROVINCIAL LAWS
29. Application. - The provisions of this Chapter shall apply for collection and payment of sales tax by the persons
providing or rendering services chargeable to sales tax under the respective Provincial laws.
30. Registration.- Every service provider, providing or rendering taxable services to its customers or clients or members,
if not already registered, shall obtain registration in the manner prescribed in Chapter I of the Sales Tax Rules, 2006.
31. Levy and collection of sales tax.- A service provider, providing or rendering taxable services to customers, clients
or members shall charge, collect and pay sales tax at the rate as provided in the respective Provincial Sales Tax
Ordinances, 2000 or the Islamabad Capital Territory (Tax on Services) Ordinance, 2001, as the case may be.
32. Filing of return and deposit of sales tax.
(1) A service provider, providing or rendering taxable services shall file return in accordance with the procedure
laid down in section 26 of the Act read with Chapter II of the Sales Tax Rules, 2006.
(2) The tax due shall be deposited in the designated branch of National Bank of Pakistan under the relevant head
"B02366-Sales Tax on Services collected on behalf of Provincial Governments", in the manner as provided in
the aforesaid Chapter II.
(3) In case a service is provided or rendered over a period of time and bill is to be issued on completion of service,
time of supply shall be the time when service is completed or the payment, or consideration in money, in
respect thereof is received whichever is earlier.
33. Determination of tax Liability.- While determining his tax liability, a service provider shall be entitled to claim input
tax credit for the tax paid on account of taxable purchases or imports made and utilities like telephone (excluding
mobile telephone), gas and electricity consumed in providing taxable services, against his output tax liability, subject
to the conditions, limitations and restrictions prescribed under sections 7, 8 and 8B of the Act and the rules or
notifications issued there-under; and subject to fulfillment of the conditions laid down u/s 73 of the Act.
34. Invoicing.
(1) A service provider, providing .or rendering taxable services shall issue serially numbered sales tax invoices to
its customers or clients or members, for the services provided or rendered, containing all the particulars as
prescribed u/s 23 of the Act:
Provided that the customers or clients or members who have been extended credit facility by a service
provider, may, for the taxable services provided or rendered during the month, be issued serially numbered
sales tax invoices at the end of each month.
(2) A service provider using computerized accounting system may issue computer generated sales tax invoice
containing all the prescribed entries.
35. Specific Provisions.- The specific provisions relating to particular categories of service providers are contained in
Part 1 to 3 of this Chapter.
PART-I
ADVERTISEMENTS ON TELEVISION AND RADIO
36. Scope and value.-
(1) In relation to advertisements, the expression "taxable services" means the services in respect of
advertisements
(a) broadcast or telecast by TV or radio stations based in Pakistan;
(b) booked in Pakistan for broadcasting or telecasting on TV or radio stations based abroad, whether or not
possessing landing rights in Pakistan; and
(c) transmitted on closed circuit T.V. or cable T.V. network.
(2) "Value of taxable service" for the purposes of levy of sales tax shall be the total consideration in money
received or the gross amount charged by a service provider from his clients for broadcasting or telecasting of
any advertisement on radio or television, including all Federal and Provincial levies but excluding the amount
of sales tax.
37. Input tax adjustment by the c1ient.- A registered person (client) whose advertisement is released on radio or
television, and to whom the sales tax invoice is issued and routed through the advertising agency, can claim input tax
adjustment for the amount of tax paid on account of release of advertisement on radio or television subject to the
observance and fulfillment of following conditions, namely:
(a) payments for all such advertisements are made by such registered person through Banking channels in such
manner that payment against a particular invoice is easily verified;
(b) all invoices issued by the service provider are in accordance with the specimen invoice set out at Annex-B;
and
Conceptual Approach to Taxes 673
Sales Tax Special Procedure Rules, 2007 Chapter-11

PART-2
CUSTOMS AGENTS AND SHIP-CHANDLERS
38. Scope and levy in relation to Customs agents.
(1) In relation to Customs agents, value of taxable service for the purposes of levy of sales tax shall be the total
consideration or charges received by a Customs agent for providing and rendering the service, excluding the
amount of sales tax. It shall not include considerations received on account of transportation charges,
demurrage, wharf age, customs-duties, excise duty, sales tax, provincial duties or taxes, toll taxes, municipal
charges, port charges, handling charges, packing charges, labour payment and such other reimbursable
expenses which a Customs agent pays on behalf of his clients against a proper receipt or invoice or bill.
(2) The sales tax registration number along with license number of the Customs agent shall be quoted on the
'Goods Declaration' or the drawback or refund claim, as the case may be.
39. Scope and levy, in relation to ship-chandlers. -- In relation to ship- chandlers, value of taxable services for the
purposes of levy of sales tax, shall be total consideration received or the gross amount charged by a ship-chandler for
providing or rendering the taxable services, including all Federal and Provincial levies but excluding the amount of
sales tax. It shall not include consideration received on other accounts such as transportation charges, toll taxes,
municipal charges, port charges, handling charges, packing charges and labour charges, which a ship-chandler pays
on behalf of his clients against a proper receipt or bill.
PART - 3
SERVICES PROVIDED BY STEVEDORES

39A. Tax liability of stevedores.- (1), (2) & (3) omitted


(4) A stevedore shall issue serially numbered sales tax invoice as required under section 23 of the Act.
(5) Every person registered as stevedore shall file monthly sales tax return in the manner as prescribed in
Chapter II of Sales Tax Rules, 2006.
(6) The cases or disputes relating to the stevedores operating under these rules shall be dealt with in the Large
Taxpayers Unit, Karachi.
CHAPTER VII
SPECIAL PROCEDURE FOR COLLECTION AND PAYMENT OF SALES TAX FROM
THE OIL MARKETING COMPANIES (SHARING OF PRODUCT)
40. Application.- The provisions of this Chapter shall apply for the collection and payment of sales tax from the oil
marketing companies (OMCs) against sharing of taxable petroleum products, herein after referred to as the product in
this Chapter, whether imported or otherwise, which are stored at joint installation (JIMCO), located at Mehmood Kot,
District Gujrat, by or on behalf of OMCs.
41. Sharing of product.-
(1) The OMCs shall be entitled to share their products without payment of sales tax at JIMCO.
(2) No sales tax invoice shall be issued for the product shared between OMCs, provided that the OMCs shall not
be barred from adhering to an internal invoicing system for the purpose of stock sharing.
(3) The OMC which has borrowed the product from another OMC shall return the product of the same description
within the time agreed between them.
42. Register for stock sharing.-
(1) Each OMC, benefiting from stock sharing facility under these rules, shall maintain, or cause to be maintained,
a separate register for recording movements of stocks under sharing arrangements between OMCs.
(2) The stock sharing register, maintained under sub-rule (1), shall contain such information about credit and debit
of the shared or returned stocks as is necessary to identify the movement of such stocks between the
concerned OMCs.
(3) The Terminal Operator shall certify the bona fides of all the credit and debit entries make in the stock sharing
register by 10th of the each month following the month to which the entries relate.
(4) The stock sharing register, duly certified by the Terminal Operator as aforesaid, shall be produced to the Sales
Tax Department, as and when required for inspection, audit or any other authorized purpose.
43. Tax Liability.-
(1) The OMC which has given a product to another OMC on stock sharing basis, shall be entitled to avail input tax
adjustment as provided under the Act and the rules made there under.
(2) The OMC, which has taken a product from another OMC, shall pay sales tax on its subsequent supply or sale
to the consumers, without claiming any input tax adjustment thereon.

674 Conceptual Approach to Taxes


Sales Tax Special Procedure Rules, 2007 Chapter-11

(3) The OMC, to whom a product taken on stock sharing basis is returned, shall pay sales tax on its supply or sale
to the buyer or consumer and input tax adjustment thereon shall be admissible, if not already availed.
44. Miscellaneous.-
(1) The stock of a product moved for exchange under these rules shall not be required to be declared on the sales
tax return unless finally supplied or sold on payment of sales tax.
(2) The OMC, which has taken any stock of a product on sharing basis under these rules, shall not normally
charge the price, over and above the price which would have been fetched by such stock had it been supplied
or sold by the lending OMC.
(3) No adjustment, refund or remission of sales tax shall be allowed under any circumstances on account of
variation or difference of the sales price of the exchanged stocks.
CHAPTER VIII
SPECIAL PROCEDURE FOR COLLECTION AND PAYMENT OF SALES TAX BY VEHICLE DEALERS
45. Registration.-
(1) All vehicle dealers shall be required to be registered under the Act who are engaged or otherwise deal in the
sale of locally manufactured vehicles and all types of imported vehicles, whether new or old or used, on the
basis of commission or otherwise, whether or not such dealer is appointed or authorized by the manufacturer
or importer of vehicles.
(2) All dealers shall within seven days of coming into force of this Chapter declare to the Collector of Sales Tax
having jurisdiction, full particulars of his dealers and the Collector shall ensure that 'no such dealer of vehicles
falling in his jurisdiction remains unregistered.
46. Booking of vehicles.-
(1) No vehicle shall be booked by the concerned manufacturer or importer through a dealer unless the particulars
of such dealer and the concerned buyer are clearly mentioned in the relevant booking documents.
(2) The aforesaid condition shall not apply in case of vehicles imported under Personal Baggage, Transfer of
Residence or Gift Scheme.
47. Invoicing.-
(1) Subject to sub-rule (2) each dealer shall issue a sales tax invoice in the name of the consumer or buyer, in
case the manufacturer or dealer has issued invoice in the name of the dealer:
Provided that in case of motorcycles, the manufacturer shall supply the same to his dealer and the dealer shall
issue invoice in the name of the buyer or consumer.
(2) Where the vehicle is invoiced directly to customer through a dealer, the dealer shall issue a delivery advice-
cum-invoice as specified in the form set out at Annex-C indicating, inter alia, the amount and the sales tax, if
any, charged thereon by the dealer over and above the price indicated in the invoice issued by the assembler,
or as the case may be, the importer, directly in the name of the consumer. Such delivery advice-cum-invoice
shall be handed over to the buyer at the time of delivery of the vehicle along with the invoice issued by the
manufacturer or importer.
48. Declaration of commission.-
(1) Each manufacturer or as the case may be, importer of vehicles shall declare to the Collector of Sales Tax
having jurisdiction, the rates of commission payable to his dealers in case of each category, make and model
of vehicle. Any change or alteration made therein shall be communicated to the Collector within seven days.
(2) Nothing in sub-rule (1) shall prohibit the Collector to ascertain or verify the accuracy of the declared rates or
amounts of commissions and other information supplied under any of the provisions of this Chapter.
49. Input tax adjustment.- Subject to such conditions, limitations and restrictions, as are imposed by sections 7, 8 and
8B of the Act and the rules or notifications issued there under and subject to fulfillment of the conditions laid down
under section 73 of the Act, the dealers shall be entitled to input tax adjustment against their output tax liability.
50. Determination of tax liability.-
(1) A dealer shall not be required to pay sales tax on such amounts of commission on which tax has been paid by
the manufacturer or importer on whose behalf vehicles is sold by such dealer provided that in case any
amount is received over and above such commission, the obligation to pay tax shall be of the dealer. Such
amounts and commissions not previously charged to sales tax shall be declared in the value of taxable
supplies in the return.
(2) In case of vehicles exchanged without involvement of any cash payment between the dealers exclusively for
subsequent sale at their respective ends, tax shall be paid only at the time of their actual sale to the public.

Conceptual Approach to Taxes 675


Sales Tax Special Procedure Rules, 2007 Chapter-11

51. Filing of return and payment of tax.- Each dealer shall file monthly sales tax return in the manner as provided in
Chapter II of the Sales Tax Rules, 2006.
52. Records to be maintained.- Each dealer shall keep proper record of all purchases, sales and tax invoices including
import documents and such other records as required to be maintained under section 22 of the Act.
53. Miscellaneous.- Where so requested by the Collector, the authority competent to register the vehicles shall furnish
information about the vehicles on which sales tax has been paid under these rules.
CHAPTER X
SPECIAL PROCEDURE FOR PAYMENT OF SALES TAX BY IMPORTERS
58A. Application.- The provisions of this Chapter shall apply to imports of all taxable goods as are chargeable to tax under
section 3 of the Act or any notification issued there under.
58B. Payment of sales tax on account of minimum value addition.-
(1) The sales tax on account of minimum value addition (hereinafter referred to as value addition tax in this
Chapter), shall be levied and collected at import stage on goods as specified aforesaid at the rate of three per
cent of the value of goods in addition to the tax chargeable under section 3 of the Act or a notification issued
there under:
Provided that the value addition tax shall not be charged on.-
(i) the goods as are imported by a manufacturer for in-house consumption;
(ii) the POL products, imported by an Oil Marketing Company for sale in the country, whose prices are
regulated under a special pricing arrangement by the Government of Pakistan or by a regulatory
authority working under the Government of Pakistan; and
(iii) Registered service providers importing goods for their in-house business use or for furtherance of their
taxable activity and not intended for further supply.
(2) The value addition tax paid at import stage shall form part of input tax, and the importer shall deduct the same
from the output tax due for the tax period, subject to limitations and restrictions under the Act, for determining
his net liability. The excess of input tax over output tax shall be carried forwarded to the next tax period as
provided in section 10 of the Act.
58C. Tax not to be refunded.-
(1) In no case, the refund of excess input tax over output tax, which is attributable to tax paid at import stage, shall
be refunded to a registered person.
(2) The registered person, if also dealing in goods other than imported goods, shall be entitled to file refund claim
of excess carried forward input tax for a period as provided in section 10 or in a notification issued there under
by the Board after deducting the amount attributable to the tax paid at import stage i.e. sum of amounts paid
during the claim period and brought forward to claim period. Such deducted amount may be carried forward to
subsequent tax period.
58D. Treatment of existing stocks of commercial importers.- The closing stocks of imported goods held by commercial
importers on 30th June 2008 on which additional sales tax at two per cent was paid at import stage shall be disposed
of under the provisions of this Chapter as in force before 1st July, 2008. The differential amount payable, in case tax
charged was higher than that paid at import stage, shall be paid on the monthly return as arrears of tax.
58E. Filing of return and audit.-
(1) The importers paying value addition tax under this Chapter shall file monthly return as provided in Chapter II of
the Sales Tax Rules, 2006.
CHAPTER XI
SPECIAL PROCEDURE FOR PAYMENT OF SALES TAX BY STEEL MELTERS,
RE-ROLLERS AND SHIP BREAKERS
58F. Application.- The provisions of this Chapter shall apply to
(a) steel melting units, steel re-rolling units, composite unites of melting and re-rolling and composite units having
complete facility of metling, re-rolling and MS cold drawing, whether operating on electric power, natural gas or
any other source of energy and regardless of the type of electricity connection;
(aa) importers of re-meltable iron and steel scrap falling under PCT Headings 7204.3000, 7204.4100 and
7204.4990, and of waste and scrap of compressor falling under PCT heading 7204.4940;
(ab) local suppliers of re-meltable iron and steel scrap;
(b) supplies of electric power and natural gas to units specified in clause (a);

676 Conceptual Approach to Taxes


Sales Tax Special Procedure Rules, 2007 Chapter-11

(c) furnaces or steel mills operated by sugar mills or other persons using self-generated electricity from bagasse
or other means;
(d) Pakistan Steel Mills operated by sugar mills or other persons using self generated electricity from bagasse or
other means;
(e) Pakistan Steel Mills, Karachi Peoples Steel Mills, Karachi and Heavy Mechanical Complex; and
(f) Ship breakers.
58G. Registration.- Every person specified in rule 56F shall, if not already registered, shall obtain registration in the
manner prescribed in Chapter I of the Sales Tax Rules, 2006.
58H. Payment of tax.-
(1) Every steel-melter, steel re-roller, composite unit of melting, re-rolling and MS cold drawing and composite
unite of steel melting and re-rolling having a single electricity meter, excluding units operated by sugar mills or
other persons using self generated electricity shall pay sales tax at the rate of nine rupees per unit of electricity
consumed for the production of steel billets, ingots and mild steel (MS) products excluding stainless steel,
which will be considered as their final discharge of sales tax liability:
Provided that the rates of sales tax on the basis of electricity consumption prescribed in sub-rules (1) and (2)
shall only be applicable to unites consuming electric power supplied by public sector electricity distribution
companies and M/s K-Electric Limited.
(2) Payment of tax by steel melters, re-rollers, composite units of melting, re-rolling and MS cold drawing and
composite unites of melting and re-rolling shall be made through electricity bills along with electricity charges:
Provided that in case the due amount of sales tax mentioned in sub-rule (1) is not mentioned in the electricity
bill issued to any steel melter or re-roller, composite units of melting, re-rolling and MS cold drawing or
composite unit of melting and re-rolling, the said melter or re-roller, composite unit of melting, re-rolling and MS
cold drawing or composite unit shall deposit the due amount of tax for the relevant tax period at the rate of nine
rupees per unit of electricity consumed excluding the amount of sales tax already paid on the electricity bill
related to the said tax period through his monthly sales tax return.
(2A) Adjustable sales tax at the rate of Rs. 5,600 per metric ton shall be levied and collected on import of re-
meltable iron and steel scrap falling under PCT headings 7204.3000, 7204.4100 and 7204.4990,
whereas non-adjustable sales tax Rs. 5,600/- per metric ton shall be levied and collected on import of
waste and scrap of compressors falling under PCT heading 7204.4940:
Provided that further local supplies of such imported waste and scrap of compressor shall not be subject to
sales tax.
(2B) Local supplies of re-meltable iron and steel scrap shall be charged to sales tax at the rate of Rs. 5,600 per
metric ton.
(2C) Steel melters may obtain adjustment of the sales tax paid on imported remeltable iron and steel scrap,
against the sales tax payable through their electricity bills, in the manner prescribed by the Board through a
general order.
(3) In case of default in payment of sales tax by the due date mentioned on the electricity bill, besides other legal
action by the concerned RTO or LTU, the concerned electric supply company shall disconnect the electricity
connection of the unit.
(3A) The Commissioner of Inland Revenue may, if he considers it expedient in the interest of revenue, collect sales
tax directly from steel-melters and re-rollers at the rates prescribed in sub-rule (1) or sub-rule (2), as the case
may be.
In case of such direct collection of sales tax, the commissioner shall issue adjustment certificate to the
electricity distribution company, which shall adjust the amount of sales tax so paid in the electricity bills of the
registered person.
(4) Ship breakers shall pay sales tax at the rate of Rs. 8,000 per metric tonne of re-rollable scrap supplied by
them at the time of import. The quantity of re-rollable scrap shall constitute 70.5% of the total LDT of the ship
imported for breaking.
(5) The Customs Collectorate shall clear the goods declaration of ship for breaking on payment of sales tax along
with other Government dues.
(6) Pakistan Steel Mills, Karachi, Heavy Mechanical Complex, Taxila and Peoples Steel Mills, Karachi shall pay
sales tax on their products under sub-section (1) of section 3 of the Act read with section 7 and section 8B
thereof.
(7) Steel melters and re-rollers, except Pakistan Steel Mills, Heavy Mechanical Complex and Peoples Steel Mills,
paying sales tax on fixed rates through electricity bills shall not be entitled to any input tax adjustment except
as provided in second proviso to sub-rule (2C).

Conceptual Approach to Taxes 677


Sales Tax Special Procedure Rules, 2007 Chapter-11

58Ha. Steel melters and re-rollers operating on self-generation basis.-


(1) Subject to permission by the Chief Commissioner, the facility to pay sales tax liabilities on the basis of gas bill
shall be allowed to the registered persons who have requisite permission, for producing electricity with the help
of gas generators, from the gas distribution companies or Oil and Gas Regulatory Authority or any other
Government authority authorized to grant such permission.
(2) Steel melters and re-rolling mills producing electricity with the help of gas generators shall discharge their
sales tax liability on the basis of the gas bill for the relevant month as per the following formula:-
Sales tax payable = HM3(or hundred cubic meter) x Rs. 2,138 less sales tax paid on gas bill:
Provided that adjustment shall be allowed as provided in second proviso to sub-rule (2) of rule 58H.
(3) Steel melters and re-rolling mills operating on self-generated electricity shall discharge their tax liability on
monthly basis, in the following manner:-
Sales tax payable = mill size (in inches) x Rs. 58,446:
Provided that if a steel melters and re-rolling mill operating on self generation basis remains closed for seven
or more days consecutively during a tax period, the registered person shall inform through telephone or fax to
the respective commissioner and the representative of the association prior to the closure of the mill. A survey
report shall accordingly be prepared by the monitoring committee comprising of one or more inland revenue
officers nominated by the concerned commissioner and representative of Pakistan Steel Re-Rolling Mills
Association and tax liability of the said mill shall be determined on the basis of above formula for the number of
days the mill remains in operation during the month.
58Hb. Steel mills operated by sugar mills or other persons using self-generated electricity
(1) Sugar mills or any other persons operating steel melting or steel re-rolling mills using self generated electricity
produced from baggasse or other means except those specified in Rule 58Ha, shall pay sales tax on the steel
products manufactured by them at the rate specified in sub section (1) of section 3 of the Act, and shall
observe all the applicable provisions of the Act.
(2) Such sugar mills or other persons shall-
(a) declare to the commissioner having jurisdiction their installed transformer capacity for steel melting and
re-rolling, which would be subject to verification; and
(b) install a tamper proof electricity meter on the transformer used for steel melting or re-rolling, along-with
a check meter outside the mills premises on the recommendations and under supervision of one
representative each from the RTO or LTU having jurisdiction and the electricity distribution company
operating in the area.
(3) In case of failure to comply with the requirements of sub-rule (2) within thirty (30) days of the commencement
of this rule or prior to commencement of operations of a new unit beside any other legal action the respective
RTO or LTU shall invoke the provisions of section 40B of the Act to monitor production, supplies and stocks so
that sales tax payable on the steel products being manufactured and supplied may be properly assessed and
recovered.
58 I. Invoices and returns.-
(1) Sales tax invoices shall be issued by the registered persons for the products or category specified in column
(2) of Table -1 below, at the rates mentioned against each in column (3) of the said Table, namely:-
Amount of sales tax to be
Sr. No. Invoices issued by and for or to
mentioned on the invoices
(1) (2) (3)
1. By steel melters or composite units of metling, re-rolling Rs.8,047 per metric ton.
and MS cold drawing to registered re-rollers
2. By steel re-rollers using ingots or billets of steel melters Rs.9,217 per metric ton.
or composite unites of melting, re-rolling and MS cold
drawing to registered persons
3. Re-rollers using billets of Pakistan Steel Mills or Heavy Rs. 8,092 per metric ton.
Mechanical Complex or Peoples Steel Mills or
imported billets to registered persons
4. Re-rollers using ship-plates and re-rollable scrap as Rs. 9,170 per metric ton.
raw material shall issue sales tax invoices to registered
persons
5. By re-rollers to unregistered persons Rs. 1,170 per metric ton.
6. By persons supplying imported MS products to ur- Rs. 8,526 per metric ton.
registered persons
7. By persons supplying imported MS products, to un- Rs. 1,170 per metric ton.
registered persons

678 Conceptual Approach to Taxes


Sales Tax Special Procedure Rules, 2007 Chapter-11

(2) Every steel-melter, composite unites of melting, re-rolling and MS cold drawing and steel re-roller paying sales
tax under these rules shall submit a copy of electricity bill showing payment of tax due duly authenticated by
the concerned Association along with a copy of sales tax return to the Commissioner having jurisdiction.
(8) The due date for filing of return shall be the 28th day of the month following the tax period to which the
electricity or gas bill relates.
58J. Records.-
Every person paying sales tax under these rules shall be required to maintain records specified under section 22 of
the Act.
58K. Values of steel products.-
The items specified in column (2) of the Table below shall be assessed for the purpose of sales tax on the values
fixed in column (4) thereof:-
TABLE
SNo. Description HS Code Value
(1) (2) (3) (4)
1. Billets supplied by Pakistan Steel Mills, Heavy Respective Rs.47,600/- PMT
Mechanical Complex and Peoples Steel Mills Headings
2. Imported billets -do- US$ 514 PMT
3. Re-rollable scrape supplied by ship breakers -do- Rs. 47,059/- PMT
58L. Responsibility of All Pakistan Steel Melters and All Pakistan Steel Rerollers Associations.-
The All Pakistan Steel Melters Association and All Pakistan Steel Re-rollers' Association shall be responsible to
ensure that the steel melters and re-rollers pay sales tax in the manner specified in these rules, and in case of non-
compliance, the Association shall actively assist the concerned Commissioner for enforcement and recovery of sales
tax due along with default surcharge calculated thereon, besides any other proceedings that may be initiated against
the defaulting steel-melter or steel re-roller under the Act. All Pakistan Steel Melters Association and All Pakistan
Steel Re-rolling Mills Association shall be authorized to authenticate the paid electricity bills of steel melters and steel
rerollers paying sales tax under these rules. The Associations shall be responsible to maintain unit-wise record of
sales tax paid by all steel melters and re-rollers on monthly basis. Every case of default in payment of sales tax shall
be reported by the President of the concerned Association to the concerned Commissioner or any other officer
nominated by the Board within seven days after the due date for payment of electricity bill.
58M. Monitoring Committee.-
A monitoring committee comprising of officers of Inland Revenue, representatives of concerned Associations and any
other person as may be nominated by the Board shall be constituted through a General Order to monitor the
collection of sales tax under these rules on monthly basis.
58MA. Option to pay sales tax on ad valorem basis.-
(1) The steel melters and re-rollers may opt to pay sales tax on ad valorem basis at the rate specified in sub-
section (1) of section 3 of the Act after deduction of input tax paid on their inputs subject to limits and
conditions as specified under the Act or notifications issued thereunder. Such melters and re-rollers shall
discharge their liability in the manner as indicated below, namely:-
(a) by the 25th of June each year such registered persons shall submit in writing to the Commissioner
having jurisdiction their irrevocable option to pay sales tax on ad valorem basis for the coming financial
year and the option so exercised shall remain enforce till the end of the financial year; and
(b) the commissioner shall coordinated with the electricity distribution companies to ensure that sales tax is
charged from such registered person in his electricity bill on the rate specified in sub section (1) of
section 3 of the Act which shall be adjustable against output tax payable on taxable supplied made by
such person, subject to the applicable provisions of the Act and rules made there under.
(2) The records maintained by registered persons opting to pay sales tax under this rule shall be subjected to
periodical audits.
58MB. Treatment for units engaged in exports.-
Subject to permission of Commissioner, concerned, the option to exclude the sales tax amount as specified in sub
rule (1) of rule 58H from the electricity bill shall be available to steel units exporting more than fifty percent of their
production.
58MC. Treatment for composite units.-
Steel melters and re-rollers who also supply products or products other than billets, ingots, MS cold drawing products
and re-rolled MS products shall follow standard sales tax procedure. The fixed taxes and values prescribed under this
Chapter shall not be applicable to supplies of such registered persons.

Conceptual Approach to Taxes 679


Sales Tax Special Procedure Rules, 2007 Chapter-11

CHAPTER XII
SPECIAL PROCEDURE FOR PAYMENT OF SALES TAX BY WHOLESALE-CUM-RETAIL OUTLETS

58N. Application.-
The provisions of this Chapter shall apply to such chains of wholesale-cum-retail outlets, engaged in bulk import and
supply of consumer goods on wholesale basis to the retailers as well as on retail basis to the general body of
consumers and who maintain their records electronically.
58Q. Supplies to diplomats and diplomatic missions and refund of tax collected.-
(1) In case the supplies are made by the wholesaler-cum-retailers to diplomats and diplomatic missions, the same
shall be charged to sales tax at zero rate provided an exemption certificate issued by Ministry of Foreign
Affairs is provided mentioning the description and quantity of goods to be purchased.
(2) The invoice issued against zero-rated supplies as aforesaid shall mention the reference number and date of
the exemption certificate.
(3) In case the supplies to a diplomat or diplomatic mission have been charged to sales tax at a rate other than
zero, the wholesaler-cum-retailer may refund the amount charged after preparation of a credit note mentioning
the particulars of the invoice and the exemption certificate.
58RA. Miscellaneous.
(1) The wholesaler-cum-retailer operating under this Chapter shall issue a sales tax invoice for the goods subject
to extra tax under Chapter XIII, if supplied to a registered person, for the purpose of claiming input tax
adjustment by the buyer.
(2) The provisions of section 73 of the Act shall not affect the admissibility of input tax adjustment where the
wholesaler-cum-retailer receives consideration in cash against the supplies made by him.
CHAPTER XIII
SPECIAL PROCEDURE FOR PAYMENT OF EXTRA SALES TAX ON SPECIFIED
ELECTRIC HOME APPLIANCES
SPECIAL PROCEDURE FOR PAYMENT OF EXTRA SALES TAX ON SPECIFIED 2[GOODS]
58S. Application.

The provision of this Chapter shall apply to the supplies of goods specified in the following Table, hereinafter referred
to in this Chapter as the specified goods, namely:-
Table
S. No. Specified Goods
1 Households electric goods, including air conditioners refrigerators, deep freezers, televisions,
recorders and players, electric bulbs, tube-lights, fans, electric irons, washing machines and
telephone sets.
2 Household gas appliances, including cooking range, ovens, geysers and gas heaters.
3 Foam or spring mattresses and other foam products for household use.
4 Auto-parts and accessories.
5 Lubricating oils, brake fluids, transmission fluids, and other vehicular fluids and maintenance
products.
6 Tyres and tubes.
7 Storage batteries.
8 Arms and Ammunitions.
9 Paints, distempers, enamels, pigments, colours, varnishes, gums, resins, dyes, glazes, thinners,
blacks cellulose lacquers and polisher sold in retail packing.
10 Tiles.
11 Biscuits, confectionery, chocolates, toffees and candies.]
58T. Mode, manner and rate applicable for payment of extra amount of tax.-
(1) Extra amount of sales tax at the rate of 2% of value of supplies shall be levied and collected on the
supplies of all specified 5[ ] goods by manufacturers and importers in addition to the tax payable under
sub-sections (1) and (2) of section 3 of the Act, as the case may be.
(2) Extra amount of sales tax so charged and collected by the above listed registered person shall
be declared in the monthly return against relevant supplies and shall be deposited without any
adjustment against the same.

680 Conceptual Approach to Taxes


Sales Tax Special Procedure Rules, 2007 Chapter-11

(3) The supplier of specified1[ ] goods shall mention the extra mount of sale tax charged under this chapter
separately on the sales tax invoice to be issued by them.
(4) The said registered persons shall charge the said extra sales tax even if they have paid any tax relating to
value addition at import stage.
(5) The specified 1[ ] goods on which extra sales tax has been paid in the aforesaid manner shall be exempt
from payment of sales tax on subsequent supplies including those as made by a retailer.
(6) The retailers operating under chapter II shall be entitled to deduct value of supplies subject to extra tax
under this Chapter from their turnover for the purpose of payment of sales tax under the said Chapter,
However, they shall pay sales tax at a rate specified in Chapter II which is based on their total turnover.
(7) If a registered person, other than a retailer, who buys the specified 1[ ] goods on payment of extra sales
tax under this chapter, also deals in sale and purchase of other goods, he shall discharge his liability in
respect of such other goods under sub-section (1) of section 3 and other relevant provisions of the Act
and shall also the be entitled to input tax adjustment only in respect of taxable supplies of such other
goods.
(8) A registered person who is engaged exclusively in purchase and sale of specified 1[ ] goods and
purchases the same on payment of extra sales tax, shall file quarterly sale tax return, in the manner
prescribed in rule7.
59. Repeal.- The Sales Tax Special Procedure Rules, 2006 are hereby repealed.
Monthly statement by
Trading Corporation of Pakistan
Total Value Sugar exported
Total Qty
Name of (excluding
S. No. purchased Qty
sugar mill sales tax ) Value (Rs.)
(Kgs.) (Kgs.)
(Rs.)
(1) (2) (3) (4) (5) (6)

Sugar supplied in local market Sales tax Date of Payment of


Qty (Kgs.) Value (Rs.) involved (Rs) Price/Value (Rs.) Sales tax (Rs.)
(7) (8) (9) (10) (11)
SPECIAL PROCEDURE FOR THE GOOS PRECIFIED IN
No. 12 OF THE FIFTH SCHEDULE TO THE ACT
58U. Application.-
the provision of this Chapter shall apply to manufacturers of goods specified against S. No. 12 of the Fifth Schedule
of the Act.
58V. Conditions and limitations for availing zero-rating facility.-
(1) Zero- rating of goods specified against S. No. 12 of the Fifth Schedule to the Act shall be subject to
determination of input-output ratios of the manufacturer by the Input-Output Co-efficient Organization
(hereinafter referred to as IOCO) if not already determined under an earlier concessionary notification
issued for such goods.
(2) For zero-rating of the import and local procurement of raw materials, packing materials, subcomponents,
components, bus-assemblies and assemblies required for the manufacture of goods specified in S. No. 12 of
the Fifth Schedule to the Act, the following conditions and procedures shall be observed, namely;-
(a) a registered manufacturer of the goods specified against S. No. 12 of the Fifth Schedule, having
suitable in- house facilities (hereinafter referred to as the applicant), shall submit and application to the
commissioner Inland Revenue having jurisdiction along with the complete list of his annual requirement
of input he intends to import or purchase for the manufacture of such goods, in the format prescribed in
Annex-F to these Rules;
(b) the commissioner may approve the declaration of input-output ratio of the applicant in the format
prescribed as Annex- G to these Rules, without physical verification in case the input-output ratio of
the applicant has already been determined by LOCO under and earlier notification issued for such
goods or the declared input-output ratio and input requirements are in accordance with prevailing
industry averages;
(c) in case the commissioner is not satisfied with the declared input-output ratios because of their being
prima facie not in accordance with prevailing industry averages and the input-output ratios of the
applicant have not already been determined by LOCO, he may, after provisionally allowing quantity
required for six months, make a reference to LOCO for final determination thereof. After receipt of report

Conceptual Approach to Taxes 681


Sales Tax Special Procedure Rules, 2007 Chapter-11

from LOCO the Commissioner shall then determine the annual quantitative entitlement of input and
grant final approval for zero-rated purchases or imports. In case of non-receipt of report from LOCO
within four months of the application being forwarded by the Commissioner, he may provisionally allow
another six months quantity to the applicant, provided he is satisfied from the records that the previously
imported or purchased inputs are being properly consumed in the manufacture of goods specified
against S. No. 12 of the Fifth Schedule to the Act;
(d) in case of input goods to be imported by the applicant, the authorized officer of Inland Revenue shall
furnish all relevant information online to the Pakistan customs computerized System as per Annex H
to these rules against a specific user ID and password obtained under section 155D of the Customs
Act, 1969 (IV of 1969):
(e) where a registered person supplies input goods to the applicant in terms of an approval granted under
clause (b) or (c) as the case may number of the buyer besides all the particulars as required under
section 23 of the Act;
(f) the applicant will be entitled to claim refund of input tax paid on utilities and other inputs which are
purchased by him on payment of sales tax, in terms of section 10 of the Act read with relevant
provisions of the Sales Tax Rules, 2006;
(g) the applicant shall maintain complete records of the inputs imported or purchased and the goods
manufactured there from;
(h) the input goods allowed under clause (b) or (c), as the case my be, shall be imported or purchased
before the expiry date of the approval, and shall be consumed within twelve months of the date of their
import or purchase;
(i) the applicant shall inform the concerned Commissioner Inland Revenue in writing about the
consumption of the imported or purchased input goods within ninety days of their consumption. The
indemnity bond shall be released on receipt of written confirmation regarding consumption of goods by
the applicant;
(j) in case the input goods are not consumed within the period allowed in the approval, the applicant shall
pay the amount of sales tax involved, or may seek extension from the commissioner Inland Revenue
under intimation to the Collector of Customs;
(k) the concerned Commissioner Inland Revenue, whenever he deems necessary but not more than once
in a calendar year, may get the records of the manufacturer audited. In case it is found that the inputs
have not been properly accounted for or consumed in the manufacture and supply of goods as
prescribed, the Commissioner may initiate proceedings for recovery of the sales tax involved on the
unaccounted inputs besides penal action under the relevant provisions of the Act; and
(l) under circumstances of exceptional nature and for reasons to be recorded in writing, the concerned
Commissioner may relax any of the conditions, if he is satisfied that such condition is detrimental to the
bona fide purposes of manufacturers business, subject to such surety or guarantee he may deem
appropriate to secure the sales tax and to ensure proper account and utilization of the imported or
locally procured goods.
CHAPTER XV
SPECIAL PROCEDURE FOR SALES TAX ON COTTONSEED OIL EXPELLED
BY OIL EXPELLING MILLS AND COMPOSITE UNITS
OF GINNING AND EXPELLING

58W. Application. The provisions of this Chapter shall apply to the persons engaged in supply
cottonseed as well as composite units of cotton ginning and expelling of oil from cottonseed.

58X. Scope and levy of tax.

The sales tax payable on supply of cottonseed oil shall be collected at the time of supply of cottonseed on the basis
of quantity of cottonseed supplied], or consumed in-house for expelling of oil by composite cotton ginning units.

58Y. Mode, manner and rate applicable for payment of sales tax.
(1) The amount of sales tax chargeable under rule 58X shall be levied and collected at the rate of Rs.6 per 40
kg at the time of supply of cottonseed by cotton ginners for in-house consumption, or to any other
registered or unregistered person for the purpose of oil extraction or expelling.
(2) All cotton ginners, if not already registered or required to be registered, shall obtain sales tax registration for
the purpose of these rules.
(3) The amount of sales tax so charged and collected by the cotton ginners shall be declared in the monthly
returns and shall be deposited as such without any input tax adjustment.
(4) The suppliers of cottonseed shall mention sales tax charged under this Chapter separately on the sales
tax invoice to be issued by them.

682 Conceptual Approach to Taxes


Sales Tax Special Procedure Rules, 2007 Chapter-11

(5) The oil expelling units using the cottonseed on which sales tax has been charged and collected in the
aforesaid manner shall be exempted from payment of sales tax on the supplies of oil cake produced from such
cottonseed.
(6) The ginner shall submit a certificate to the Commissioner having jurisdiction by the 15th day of the
month following the tax period for the quantity of cottonseed supplied to the growers for sowing purpose.

58Z. Monthly statement. Each ginning unit including a composite ginning unit, shall submit to the Commissioner of
Inland Revenue having jurisdiction, monthly statement of production and supply of ginned cotton, cottonseed and
cottonseed oil in the format set out in Annex-I, by the 15th day of the month following the tax period.

58ZA. Notice to be given by the ginning unit.A ginning unit, or as the case may be, a composite ginning unit
shall, at the time of commencement of ginning activity and at the time of closure thereof, inform the Commissioner of Inland
Revenue having jurisdiction within three days of such commencement or closure, as the case may be.

58ZB. Final statement to be furnished by the ginning unit.


(1) Each ginning unit including a composite ginning unit shall, within fifteen days of the cessation of
ginning activity, furnish to the Commissioner of Inland Revenue having jurisdiction, a statement
regarding production and supply of ginned cotton, cottonseed, cottonseed oil, oil cake and oil dirt, in the format
set out in Annex-J.
(2) Where the cotton ginner or the composite cotton ginning unit fails to furnish any statement or certificate as
required under this Chapter, he shall be liable for penal action as provided under serial No.17 of the
Table in section 33 of the Sales Tax Act, 1990.]

59. Repeal.The Sales Tax Special Procedure Rules, 2006 are hereby repealed.

Conceptual Approach to Taxes 683


Sales Tax Special Procedure Rules, 2007 Chapter-11

684 Conceptual Approach to Taxes


Sales Tax Special Procedure (Withholding) Rules, 2007 Chapter-12

Chapter

12 SALES TAX SPECIAL PROCEDURE


(WITHHOLDING) RULES, 2007

(For CA MOD F AND ICMAP students)

Sr. Rule Topic covered


For CAF-6 and ICMAP Students

1. 1
Short title, application and commencement
2. 2
Responsibility of a withholding agent
3. 3
Responsibility of the registered supplies
4. 4
Responsibility of the Collector
5. 5
Exclusions
6. 6
Application of other provision
7. ANNEX

MCQs with solutions


ICMAP & CA Mod C past papers theoretical questions
S.R.O. 660(1)/2007, Islamabad, the 30th June, 2007. In exercise of the powers conferred by sub-section (6) and sub-
section (7) of section 3 and sub-section (4) of section 7 of the Sales Tax Act, 1990, read with section 71 thereof, the Federal
Government is pleased to make the following rules, namely:-
Short title, application and commencement [Rule 1]
(1) These rules may be called the Sales Tax Special Procedure (Withholding) Rules, 2007.
(2) They shall apply to taxable goods and services as are supplied to following persons, hereby specified as withholding
agents, for the purpose of deduction and deposit of sales tax, namely:-
(a) federal and provincial government departments;
(b) autonomous bodies;
(c) public sector organizations;
(d) companies as defined in the Income Tax Ordinance, 2001, which is registered for sales tax, federal excise
duty or income tax;
(e) recipients of service of advertisement, who are registered for sales tax;
(f) Persons registered as exporters.
Explanation.- "withholding agent" includes the accounting office which is responsible for making payment against the
purchases made by a government department.
(3) They shall come into force on the 1st day of July, 2007.
Responsibility of a withholding agent [Rule 2]
(1) The withholding agent, intending to make purchases of taxable goods, shall indicate in an advertisement or notice for
this purpose that the sales tax to the extent as provided in these rules shall be deducted from the payment to be
made to the supplier.

Conceptual approach to Taxes 685


Sales Tax Special Procedure (Withholding) Rules, 2007 Chapter-12

(2) A withholding agent, other than a recipient of advertisement services, shall deduct an amount equal to one fifth of the
total sales tax shown in the sales tax invoice issued by a registered person and make payment of the balance amount
to him as per illustration given below,-
Example:
Value of taxable supplies excluding sales tax Rs. 1000
Sales tax chargeable @ 17% Rs. 170
Sales tax to be deducted by the withholding agent Rs. 34 (i.e. Rs. 170 5)
Sales tax payable by the withholding agent to the supplier Rs. 136 (i.e. Rs. 170- Rs.34)
Balance amount payable to the supplier by the withholding agent Rs. 1,136 (i.e. Rs. 1,000 + Rs.136)
(3) A withholding Agent, shall on purchase of taxable goods from unregistered persons, deduct sales tax at the
applicable rate of the value of taxable supplies made to him from the payment due to the supplier and unless
otherwise specified in the contract between the buyer and the supplier, the amount of sales tax for the purpose of this
rule shall be worked out on the basis of gross value of taxable supply.
(3A) A person mentioned in clause (e) of sub-rule (2) of rule 1, who receives advertisement services, provided or rendered
by a person based in Pakistan or abroad, shall deduct the amount of sales tax as mentioned in the invoice issued by
the service provider from the payment due to the service provider. In case the sales tax amount is not indicated on
the invoice, the recipient shall deduct sales tax at the applicable rate of the value of taxable services from the
payment due to the service provider.
(4) Where the purchases are made by a government department, the following procedure shall be observed, namely:-
(a) the Drawing and Disbursing Officer (DDO) preparing the bill for the accounting office shall indicate the amount
of sales tax withheld as prescribed above. The accounting office shall adopt the procedure as indicated below:
(i) in case of purchases made by a department under the Federal Government, the office of the
Accountant General of Pakistan Revenue shall account for the amount deducted at source during a
month under the Head of Account "B02341-Sales Tax" and send an intimation to the Chief
Commissioner, Regional Tax Office, Islamabad, by the 15th of the following month;
(ii) in case of purchases by departments under provincial or district governments, the Accountant General
of the province or the District Accounts Officer, as the case may be, shall credit the amount deducted at
source during a month to the head of account "GI2777- Sales Tax Deductions at Source under Sales
Tax Special Procedure (Withholding) Rules, 2007'. Cheque for the amount will be prepared by the
Accountant General or the District Accounts Officer, as the case may be, in the name of Collector
having jurisdiction by debit to the aforesaid head of account and sent to the Collector by the 15th of the
following month; and
(iii) where the purchases are made by the departments falling in purview of Military Accountant General,
the MAG shall account for the amount deducted at source during a month under the Head of Account
"B0234l-Sales Tax" and send intimation to the Chief Commissioner, Regional Tax Office, Rawalpindi,
by the 15th of the following month. The amount so deducted at source shall be reported by MAG office
to AGPR through civil exchange accounts; and
(b) the concerned Drawing and Disbursement Officer shall prepare the return in the form as in the Annexure to
these rules for each month and forward the same to the Collector having jurisdiction by the 15th of the
following month.
(5) In case of purchases, not covered by sub-rule (4) or sub -rule (6), the sales tax deducted at source shall be deposited
by the withholding agent in the designated branch of NBP under relevant head of account on sales tax return-cum-
payment challan in the form set out at Annexure to these rules, by 15th of the month following the month during which
the purchase has been made. The return-cum-payment challan shall be prepared and deposited with the bank in
triplicate and the bank shall send the original to the Collectorate of Sales Tax having jurisdiction, return the duplicate
to the depositor and retain the triplicate for its own record:
Provided that a single return-cum-challan can be filed in respect of all purchases for which the payment has been
made in a month.
(6) In case the withholding agent is also registered under the Sales Tax Act, 1990, or the Federal Excise Act, 2005, he
shall deposit the withheld amount of sales tax along with return filed for the month in which the purchase was madein
the manner as provided under Chapter II of the Sales Tax Rules, 2006, along with other tax liability and such person
shall not be required to file the return in the term as set out in the Annexure to these rules:
Provided that in case the withholding agent is not registered for sales tax or federal excise duty but holds a national
tax number assigned under the Income Tax Ordinance, 2001, he shall file the return, as set out in the Annexure to
these rules; electronically and deposit the amount deducted at source in the manner as provided for persons filing
returns electronically under rule 18 of the Sales Tax Rules, 2006:
Provided further that any other withholding agent may also opt to file the prescribed return electronically and deposit
the deducted amount in the manner as provided in this sub-rule.

686 Conceptual approach to Taxes


Sales Tax Special Procedure (Withholding) Rules, 2007 Chapter-12

(7) The withholding agent shall furnish to the Collector of Sales Tax having jurisdiction all such information or data as
may be requested by him for carrying out the purposes of these rules.
(8) A certificate showing deduction of sales tax shall be issued to the supplier by the withholding agent duly specifying
the name and registration number of supplier, description of goods and the amount of sales tax deducted.
Responsibility of the registered supplier [Rule 3]
(1) The registered supplier shall issue sales tax invoice as stipulated in section 23 of the Sales Tax Act, 1990, in respect
of every taxable supply made to a withholding agent.
(2) The registered supplier shall file monthly return as prescribed in the Sales Tax Rules, 2006, and shall adjust total
input tax against output tax under sections 7, 8 and 8B of the Sales Tax Act, 1990, taking due credit of the sales tax
deducted by the withholding agent, in the manner as prescribed in the return under Chapter II of the Sales Tax Rules,
2006.
Responsibility of the Collector [Rule 4]
(1) The Collector shall keep a list of all withholding agents falling in his jurisdiction and monitor payment of tax deducted
by withholding agents falling in his jurisdiction and shall also ensure that the return prescribed under these rules is
filed.
(2) The Collector shall ensure that the return received from the bank is duly fed in the computerized system as referred to
in clause (5AA) of section 2 of the Sales Tax Act, 1990.
(3) The Collector shall periodically ensure that the suppliers mentioned in the return filed by the withholding agents, as
fall under his jurisdiction, are filing returns u/s 26 of the Sales Tax Act, 1990, read with Chapter II of the Sales Tax
Rules, 2006, and are duly declaring the supplies made to withholding agents.
Exclusions [Rule 5]
The provisions of these rules shall not apply to the supplies of the following goods and services if made by a registered
person, namely:-
(i) Electrical energy;
(ii) Natural gas;
(iii) Petroleum products as supplied by petroleum production and exploration companies, oil refineries, oil marketing
companies dealers of motor spirit and high diesel; and
(iv) registered persons paying sales tax under Chapter XI of the Sales Tax Special Procedure Rules, 2007, except those
paying sales tax on ad valorem basis at standard rate;;
(v) vegetable ghee and cooking oil; and
(vi) telecommunication services.
(vii) goods specified in the third Schedule to the Sales Tax Act, 1990; and
(viii) supplies made by commercial importers who paid value addition tax on such at the time of import of prescribed under
chapter X of the Sales Tax Special Procedure Rules, 2007.
Application of other provisions [Rule 6]
All the provisions of the rules and notifications made or issued under the Sales Tax Act, 1990, shall apply to supplies as
aforesaid not inconsistent with the provisions of these rules.

Conceptual approach to Taxes 687


Sales Tax Special Procedure (Withholding) Rules, 2007 Chapter-12
Annex
See rule 2(4)
Government of Pakistan
Monthly Sales Tax Return for Withholding agents
__________________________________________________________________________________________
Withholding agents name & address month year
Period NTN / FTN
__________________________________________________________________________________________

DETAIL OF SALES TAX DEDUCTED DURING THE MONTH (attach additional sheets if required)
S. No. Name of supplier NTN No. of invoices Total Sales Tax Sales tax
charged deducted

TOTAL SALES TAX WITHHELD DURING THE MONTH


I, ________________________, holder of CNIC No. _______________________________in my capacity as
________________________certify that the information given above is/are correct, complete and in accordance with
Verification

the provisions of the Sales Tax Act, 1990, and Rules and Notifications issued thereunder.

Date (dd/mm/yy)_______________Stamp________________Signature____________________

Head of Account Amount Amount received (in words) ____________________


(in figures) ___________________
B02341-Sales Tax
For Bank use
Details of ST Paid

B02366 Sales Tax on services


Bank Officers Signature ______________________
B02367 FED in Sales Tax mode
Banks Stamp ___________Date dd/mm/yy _______

TOTAL AMOUNT DEPOSITED

688 Conceptual approach to Taxes


Sales Tax Special Procedure (Withholding) Rules, 2007 Chapter-12

ICMAP PAST PAPERS THEORETICAL QUESTIONS


Q. No. 5(a) February 2014 As per the Sales Tax Special Procedure (Withholding) Rules, 2007 answer the following:
(i) For the purpose of deduction and deposit of sales tax who can be regarded as with holding agents?
(ii) The withholding agent is required to issue a certificate to the supplier showing certain particulars. Enumerate
those particulars.
(b) (i) As per section 8(1) of the Sales Tax Act, 1990 in respect of which goods, a registered person is not
entitled to deducted of reclaim the input tax? Specify any five.
(ii) Briefly state in the light of the provisions of the Sales Tax Act, 1990 whether the following person can
reclaim input tax?
Alpha Private Limited is a renowned company established in 1990. It is registered under the said
Act and deals in taxable and non-taxable supplies.
Mr. Amir is a non-registered person under the said Act, deals in taxable supplies only

Conceptual approach to Taxes 689


Sales Tax Special Procedure (Withholding) Rules, 2007 Chapter-12

690 Conceptual approach to Taxes


Chapter 13 Solved Past Papers Sales Tax Numericals of ICMAP Stage IV - (2003 to 2015)

Chapter

13
SOLVED PAST PAPERS SALES TAX NUMERICALS OF ICMAP
STAGE IV - (2003 TO 2015)
Note: All the following questions have been solved under the Sales Tax Act effective from July 1st 2015,
except as provided in the Finance Act, 2015.

Q.No. 6(a) March 2015 Karven Limited is a company registered under the Sales Tax Act, 1990. The company is
engaged in the manufacturing, import and export of chemical products. Following activities were carried out by the
company dating the month of January 2015:

Rs. 000
Purchases:
Local:
Raw materials from registered suppliers 800,000
Raw materials from non-registered suppliers 400,000
Imports:
Invoice value (converted into pak Rupees) 150,000
Customs duty 37,500
Value inclusive of customs duty 187,500
Federal excise duty 6,250
Manufacturing and other costs 220,000
Supplies:
Sales to registered customers 1,400,000
Sales to non-registered persons (commercial/industrial Customers) 600,000
Sales of exempted supplies 400,000
Exports 220,000

Additional Information:
Sales tax of Rs. 50,000, Rs. 20,000 and Rs. 16,000 was paid in cash on account of electricity, gas and telephone bills
respectively, directly consumed for taxable activities.

Required:
In the light of the provisions of the Sales Tax Act, 1990, compute the net sales tax payable for the month of January
2015. Substantiate your answer with notes.

Solution:

Karvan Limited
Computation of Sales tax payable / refundable
For the period of January 2016

Output tax Rs. in '000'


Sales to registered customers 238,000
Sales to unregistered person 102,000
Sales of exempted supplies -

Conceptual Approach to Taxes_______________________________________________


_____________________________________691
Chapter 13 Solved Past Papers Sales Tax Numericals of ICMAP Stage IV - (2003 to 2015)

Exports -
340,000
Input tax
On taxabale supplies 128,960
On utilities bills ( 50,000 + 20,000 + 16,000) 86,000
A 214,960
or 90 % of 340,000 B 306,000
Lower of A and B (214,960)
Sales Tax payable 125,040
Add: 2% further sales tax on 400,000 8,000
Sales tax payable with return 133,040

Sales Tax refundable 25,792

Input tax for apportionment


Purchase from registered person 136,000
Imports (150,000 + 37,500 + 6,250) x 17% 32,938
168,938

Apportionment of input tax

Supplies to: Amount Input Tax


Registered person 1,400,000 90,272
Un-registered person 600,000 38,688
Exempted Supplies 400,000 25,792
Exports 220,000 14,186
2,620,000 168,938

Notes - 1
Manufacturing and other costs will be

Q.NO. 5(b) August 2014 Sitara Manufacturers (SM) deals in the taxable and exempted supplies. SM provided following
information for determination of its sales tax liability for the month of June, 2014.

SMmadepurchasesamountingRs.800,000fromElahi&Sonswhoisregisteredperson.
Mr.AhsaninvoicedRs.150,000toSMwithoutcharginganysalestax.
SMfurtherincurredmanufacturingandothercostamountingRs.150,000.
Outoftotalstock,SMsuppliedgoodsofworthRs.1,500,000toSidra&Co.,whichisaregistered
company under the Sales Tax Ac t, 1990.
In addition to above SM made exempted supplies of Rs. 600,000 and supplied goods of worth Rs. 500,000 to non-
registered person.
SMalsopaidsalestaxonelectricitybillamountingRs.18,000.
Required:
Compute sales tax liability of SM under the Sales Tax Act, 1990 for the month of June, 2014.

Solution

Sitara Manufacturing
NTN #
STRN #
Computation of Sales tax payable / refundable
For the month of June 2016

692____________________
___________________________________________________________Conceptual Approach to Taxes
Chapter 13 Solved Past Papers Sales Tax Numericals of ICMAP Stage IV - (2003 to 2015)
Output tax
Sales to registered person 1,500,000 255,000
Sales to un-registered person 500,000 85,000
Exempt supplies 600,000 -
340,000
Input tax
Input tax admissible W-1 118,462
or 90 % of 340,000 306,000 (118,462)
221,538
2% further tax on Rs. 500,000 10,000
Sales tax payable with return 231,538

Residual Input tax


Purchase from registered supplier 800,000 136,000
Electricity bill 18,000
154,000
Apportionment of Residual input tax
taxable supplies 2,000,000 118,462
Exempt supplies 600,000 35,538
2,600,000 154,000

Q .NO. 6 Spring 2014 Beta (Pvt.) Limited is engaged in imports, trading and local manufacturing of certain
taxable consumer goods including products like detergents which are subject to levy of sales tax on retail price
basis. During the month of December, 2013 the following information of the company is complied:

Sales: Rs. '000


Sales of locally manufactured taxable goods (other than detergents) 20,000
Sales of imported finished goods on value addition basis 2,750
Sales of imported detergents to wholesalers
[31,250 packets @ Rs. 240 each packet] 7,500
Sales of locally manufactured detergents to wholesalers
[30,000 packets @ Rs. 225 each packet] 6,750

Imports and Local Purchases:


Import of raw materials for in-house consumption - Taxable 10,000
Import of finished goods - other than detergents 2,000
Import of finished goods - detergents 1,000
Local purchases of raw materials - Taxable 8,750

Additional Information:

Retail price of detergents (imported and locally manufactured) is Rs. 250 per packet of 1.5 kilograms each.

Note: All above amounts are exclusive of sales tax.

Required: In the light of the provisions of the Sales Tax Act, 1990 and rules made there under, calculate the sales
tax liability of Beta (Pvt.) Limited for the month of December, 2013.

Solution:
Beta (Pvt.) Limited

Conceptual Approach to Taxes_______________________________________________


_____________________________________693
Chapter 13 Solved Past Papers Sales Tax Numericals of ICMAP Stage IV - (2003 to 2015)

Output tax on: Rs. '000

Sales of locally manufactured taxable goods (20,000,000 x 17%) 3,400


Sales of locally manufactured detergents [(30,000 products X Rs.250)] x 17% 1,275
Sales of imported detergents to wholesalers [(31,250 products X Rs.250)] x 17% 1,328
Sales of imported finished goods on value addition basis (2,750,000 x 17%) 468
Total Output Tax (A) 6,471

Input tax on:

Local purchase of raw materials 1,488


Import of raw materials for in-house consumption 1,700
Import of finished goods - other than detergents [Rs. 2(M) x 20%] 400
Import of finished goods - detergents [Rs. 1(M) x 20%] 200
Total input tax (B) 3,788

90% of output tax (Note - 1) (C) 5,823

Less admissible tax credit: lower of (B) or (C) (D) 3,788


Balance sales tax payable [A less D] 2,683

(N - 1) As the registered person given in this question is engaged in various types of activities and none of the
clause under SRO 647(I)/2007 dated June 27, 2007 is applicable therefore 90% of output tax limitation
on input is applicable U/S 8B of the Sales Tax Act, 1990.

(N - 2) 2% further sales tax u/s 3(1A) has not been charged as the supplies are made to registered wholesalers and
further final consumers (against supplies made) are not required to be registered under the Sales Tax Act,
1990.

Spring - 2013 Q. 5 b

Hassan Associates manufactures and supplies Product A and Product B. Hassan Associates is registered under the
Sales Tax Act, 1990. Following information has been extracted from its records for the month of May 2013:

Rupees in million
Purchase of raw material from registered person
To manufacture Product A 900
To manufacture Product B 300
Purchase of raw material from unregistered person
To manufacture Product A 150
To manufacture Product B 200
Import of raw material to manufacture Product A and B 450
Sale of Product A
To registered person 800
To unregistered person 250
Sale of Product B
To registered person 500
To unregistered person 150
Sale return during the month
Product A 50
Product B 30
Sales tax paid on electricity bill 15

Further information:

694____________________
___________________________________________________________Conceptual Approach to Taxes
Chapter 13 Solved Past Papers Sales Tax Numericals of ICMAP Stage IV - (2003 to 2015)
ProductBisexemptfromthesalestax.
SalestaxcreditbroughtforwardfrompreviousmonthamountedtoRs.25million.
AnimportbilldatedNovember10,2012amountingtoRs.25millionhadnotbeenclaimed
inadvertently. This oversight was detected during the month.
Salestaxispayableattherateof16%.Alltheaboveamountsareexclusiveofsalestax.

Required:
In the light of the Sales Tax Act, 1990 and rules made there under, calculate the following for the month of
May 2013;

Salestaxpayable/refundable
Inputtaxtobecarriedforward,ifany.

Solution:

HASSAN ASSOCIATES
SALES TAX LIABILITY
TAX PERIOD: MAY 2016

OUTPUT TAX Rs. "000" Rs. "000"

On total taxable supplies [(800+250) x 17%] (Note - 1) 179


Less: Return (50 x 17%) (9)
170
INPUT TAX

On taxable supplies only for product A U/S 3 153


On taxable supplies U/S 3 (W - 2) 54
Add: sales tax credit brought forward from previous month 25
(A) 232

90% of output tax (Note - 2) (B) 153

Less admissible tax credit: lower of (A) or (B) 153


Balance sales tax payable 17

Sale tax excess payment carry forward (A - B) 79

WORKING

W-1
Residual input tax :

Import for taxable & exempt goods ( 450 x 17%) 77


Sale tax on electricity bill 15
Less: Import bill un claimed ( 25 x 17%) (4)
Total residual input tax 87

W-2 Apportionment of residual input tax


[U/R 25 of Sales Tax Rules, 2006] Supplies Residual input tax
Rs. "000" Rs. "000"

Taxable supplies 1,050 54


Exempt supplies 650 33
Total supplies 1,700 87

(Note - 1) It has been assumed that 2% further tax u/s 3(1A) is not applicable on local taxable supplies to unregistered
persons under SRO 648(I)/2013 dated July 09, 2013 otherwise further tax shall be accounted for and paid
seperately without adjustment of the same against input tax / refund of the registered person and further it
shall also not be considered for the computation of 90% limitation on output tax.

Conceptual Approach to Taxes_______________________________________________


_____________________________________695
Chapter 13 Solved Past Papers Sales Tax Numericals of ICMAP Stage IV - (2003 to 2015)

(Note - 2) As it is case of a manufacturer and none of the clause under SRO 647(I)/2007 dated June 27, 2007 is
applicable therefore 90% of output tax limitation on input is applicable U/S 8B of the Sales Tax Act,
1990.

Autumn - 2013 Q. 5

Bashir corporation is registered with the sales tax department as manufacturer, exporter and the distributor, it has the
following transactions for the month of january, 2013:

Rupees
Purchases from registered persons 6,000,000
Purchases from non-registered persons 720,000
Exports 1,000,000

During a tax period the company supplied goods worth Rs.7,000,000. As per normal business practice, the
company sells the goods at a discount of 20 % of the retail price,

Required:
Compute the sales tax liability of Bashir corporation for the month of junuary 2013.

Solution:

BASHIR CORPORATION PVT LTD


COMPUTATION OF SALES TAX
FOR THE MONTH OF JANUARY 2016

OUTPUT TAX Rs.

Sales to registered person U/S 3 (N - 2) (Rs. 8,750,000 x 17 %) 1,487,500


Export [U/S 4 read with 5th Schedule] (N-2)
(Rs. 1,000,000 x 0%) -

Total output tax 1,487,500

LESS : input tax

Purchased from registered person ( 6,000,000 x 17%) 1,020,000


Purchased from un registered person (N - 3) -
1,020,000
Balance tax payable 467,500

(N - 1) As the zero rated supplies are less than 50% of all taxable supplies under SRO 647(I)/2007 dated June
27, 2007, therefore 90% limitation is applicable U/S 8B of the Sales Tax Act, 1990. Howeve the same
has not been shown as the available input tax is already less than 90% of the total output tax.

(N - 2)
Retail price of goods after discount 7,000,000

Retail price ( i.e sale price plus discount allowed )


(7,000,000 x 100 / 80) 8,750,000

(N - 3) As purchases from non-registered persons are without sales tax invoices hence the same shall be
without sales tax u/s 23(2) of the Sales Tax Act, 1990, therefore the question of adjustment of input tax
in not applicable.

Q.4 (c) August 2012

Pak Manufacturing Company Limited is engaged in manufacturing of both taxable and exempted supplies. Following are
the transactions for the month of June:

696____________________
___________________________________________________________Conceptual Approach to Taxes
Chapter 13 Solved Past Papers Sales Tax Numericals of ICMAP Stage IV - (2003 to 2015)
Rupees
Supplies to registered manufacturers 4,250,000
Supplies to non-registered retailers 3,250,000
Sales of exempted supplies 600,000
Purchases from registered persons 3,000,000
Purchases from non-registered persons 2,100,000
Import of raw material 1,750,000

Required:
Compute the sales tax liability of Pak Manufactur ing Company for the month of June.

Solution Q.4 (c) August 2012

Pak Manufacturing Company


Computation of sales tax liability
Rs.
Output tax
On supplies to registered manufacturers U/S 3 (Rs. 4,250,000 x 17%) 722,500
On supplies to non-registered retailers U/S 3 (Rs.3,250,000 x 17%) (N-1) 552,500
On sales of exempted supplies [U/S 6 read with 6th Schedule] -
1,275,000

Input tax allocated to taxable supplies (W-1) (A) 747,685

90% of output tax [Rs. 1,275,000 x 90%] (N - 3) (B) 1,147,500

Less: admissible input tax: Lower of (A) or (B) 747,685


Balance sales tax payable 527,315

(W-1)
Residual input tax
Purchases from registered persons (Rs. 3,000,000 x 17%) 510,000
Purchases from non-registered persons (N - 2) -
Import of raw material (Rs. 1,750,000 x 17%) 297,500
807,500

Allocation of residual input tax: [U/R 25 of the Sales Tax Rules, 2006]
Supplies Residual input tax
Rs. Rs.
Supplies to registered manufacturers 4,250,000 423,688
Supplies to non-registered retailers 3,250,000 323,997
Sales of exempted supplies 600,000 59,815
8,100,000 807,500

Input tax allocated to taxable supplies (W-1) (423,688 + 323,997) 747,685

(Note - 1) It has been assumed that 2% further tax u/s 3(1A) is not applicable on local taxable supplies to unregistered
persons under SRO 648(I)/2013 dated July 09, 2013 otherwise further tax shall be accounted for and paid
seperately without adjustment of the same against input tax / refund of the registered person and further it
shall also not be considered for the computation of 90% limitation on output tax.

(Note - 2) As purchases from non-registered persons are without sales tax invoices hence the same shall be
without sales tax u/s 23(2) of the Sales Tax Act, 1990, therefore the question of adjustment of input tax
in not applicable.

(Note - 3) As it is case of a manufacturer and none of the clause under SRO 647(I)/2007 dated June 27, 2007 is
applicable therefore 90% of output tax limitation on input is applicable U/S 8B of the Sales Tax Act,
1990.

Conceptual Approach to Taxes_______________________________________________


_____________________________________697
Chapter 13 Solved Past Papers Sales Tax Numericals of ICMAP Stage IV - (2003 to 2015)
Spring - 2012 Q. 7

Mr. Folad is registered under the Sales Tax Act, 1990 and is engaged in the business of manufacture and supply of
home appliances. Following information has been extracted from the records of Mr. Folad for the month of February
2012.
Rupees
Purchases Local
From registered suppliers 70,250,000
From un-registered suppliers 15,750,000

Supplies:
Local taxable supplies to registered persons 72,870,000
Local taxable supplies to un-registered persons 9,850,000
Exports to Canada and USA 12,700,000

Following additional information is also available:

(i) A new machine purchased for Rs. 12 million was commissioned into operations during February 2012.

(ii) Sub-standard supplies amounting to Rs. 4,500,000 were returned to vendors. Proper debit/credit notes were raised in
this regard.

(iii) Goods worth Rs. 7,200,000 were returned by different customers. Proper debit/credit notes were raised within the
specified period.

(iv) An amount of Rs. 820,000 on account of purchases made from a registered supplier is outstanding since July 2011.
The related input tax was accounted for in the relevant tax period.

(v) Sales tax credit brought forward from previous month amounted to Rs. 910,500.

Sales tax is payable at the rate of 16%. All the above figures are exclusive of sales tax.
Required:
Compute sales tax payable/refundable and input tax credit to be carried forward, if any.

Solution:

Mr. Folad
Computation of sales tax liability

Rs. Rs.
Output tax:

On local taxable supplies to registered persons U/S 3 12,387,900


On local taxable supplies to unregistered persons U/S 3 (Note 4) 1,674,500
On exports to Canada and USA [U/S 4 read with 5th Schedule]
Less sales returns (1,224,000)
12,838,400
Input tax:

On net local supples from registered persons (Note 2) less related to 7,351,347
fixed assets (Rs.9,119,832 - Rs. 1,768,485)
On purchases from un- registered persons -
Input tax b/f 910,500
(A) 8,261,847

90% of output tax (N - 1) (B) 11,554,560


Less: Admissible input tax Lower of (A) or (B) 8,261,847

Purchase of fixed asset related to local taxable supplies 1,768,485


(Rs. 2,040,000 / 95,420,000 x 82,720,000) (N - 1)
(C) 10,030,332

Balance sales tax payable [ Output tax - (C) ] 2,808,068

698____________________
___________________________________________________________Conceptual Approach to Taxes
Chapter 13 Solved Past Papers Sales Tax Numericals of ICMAP Stage IV - (2003 to 2015)

2% further sales tax payable on supplies to un-registered persons (N - 3) 197,000


Sales tax refundable related to export supplies (Note 3) (1,400,168)

(N - 1) As the zero rated supplies are less than 50% of all taxable supplies under SRO 647(I)/2007 dated June 27,
2007, therefore 90% limitation is applicable U/S 8B of the Sales Tax Act, 1990. The said limitation is not
applicable on input tax paid on acquistion of fixed assets.

(N - 2) Apportionment of taxable supplies input tax


[U/R 25 of Sales Tax Rules, 2006] Supplies Input tax
Rs. Rs.

Local supplies (gross) 82,720,000 9,119,832


Export sales 12,700,000 1,400,168
95,420,000 10,520,000

(N - 3) It has been assumed that 2% further tax u/s 3(1A) is applicable on local taxable supplies to unregistered
persons under SRO 648(I)/2013 dated July 09, 2013 hence the same shall be accounted for and paid
seperately without adjustment of the same against input tax / refund of the registered person and further it
shall also not be considered for the computation of 90% limitation on output tax.

Summer - 2011 Q. 4(b)

The following information relate to XYZ (Pvt) Ltd., for the month ended December 31, 2010:
Rs.
Taxable supply 5,000,000
Exempt supply 1,000,000
Zero-rated supply 2,000,000
Total supply 8,000,000

Purchase of raw materials used in taxable and zero-rated supply 4,500,000


Purchase of raw materials used in exempt supply 550,000

Required:
Compute the amount of sales tax liability / refund of the company.

Solution:

XYZ (Pvt) Ltd


Computation of sales tax liability for the month ended December 31, 2015

Rs.
Output tax:
On taxable supply U/S 3 (Rs. 5,000,000 x 17%] 850,000
On exempt supply [U/S 6 read with 6th Schedule] -
On zero rated supply [U/S 5 read with 5th Schedule] -
850,000
Input tax:
Purchase of raw materials used in taxable supplies (N - 2) 546,429
Purchase of raw materials used in exempt supply -
(A) 546,429

90% of output tax (N - 1) (B) 765,000 546,429


Less: Admissible input tax lowe of (A) or (B)
Balance sales tax liability 303,571

Sales tax refundable (Note 2) 218,571

(N - 1) As the zero rated supplies are less than 50% of all taxable supplies under SRO 647(I)/2007 dated June 27,
2007, therefore 90% limitation is applicable U/S 8B of the Sales Tax Act, 1990.

Conceptual Approach to Taxes_______________________________________________


_____________________________________699
Chapter 13 Solved Past Papers Sales Tax Numericals of ICMAP Stage IV - (2003 to 2015)

(N - 2) Apportionment of taxable supplies input tax:


[U/R 25 of the Sales Tax Rules, 2006]
Supplies Input tax
Rs. Rs.

Local supplies (net) 5,000,000 546,429


Zero rated supplies 2,000,000 218,571
7,000,000 765,000

Winter - 2008 Q. 5

Karsaz Limited is engaged simultaneously in manufacturing and supply of taxable as well as exempted goods. Summary
of its transactions for the month of October is given below:

Rupees Rupees
Purchase of goods to be used for taxable supplies 600,000
Purchase of goods to be used for exempt supplies 900,000
Purchase of goods to be used for both taxable and exempt supplies 3,000,000
Total input tax on all purchases 675,000
Supply of wholly taxable goods 1,500,000
Supply of wholly exempt goods 1,800,000

Supply of partly taxable and partly exempted goods:

Taxable supplies 3,750,000


Exempt supplies 750,000
4,500,000
Required:
Compute the sales tax liability of the company for the tax period. (Show proper workings)

Solution

Karsaz Limited
Computation of sales tax liability
Rs.
Output tax:
On supply of wholly taxable goods U/S 3 255,000
On supply of wholly exempt goods [U/S 6 read with 6th Schedule] -
On supply of partly taxable and partly exempted goods
Taxable supplies U/S 3 637,500
Exempt supplies [U/S 6 read with 6th Schedule] -
892,500
Input tax:
On purchase of goods to be used for taxable supplies (W-1) 90,000
On purchase of goods to be used for exempt supplies -
On share of taxable supplies in purchase of goods for both 375,000
taxable and exempt supplies (W - 2) (A) 465,000

90% of output tax (N - 1) (B) 803,250


Less admissible input tax lower of (A) or (B) 465,000
Balance sales tax liability 427,500

Working:

Total purchases (600,000 + 900,000 + 3,000,000) 4,500,000


Total input tax on all purchases 675,000

W - 1:
Tax on purchase of goods to be used for taxable supplies
(675,000 / 4,500,000 x 600,000) 90,000

700____________________
___________________________________________________________Conceptual Approach to Taxes
Chapter 13 Solved Past Papers Sales Tax Numericals of ICMAP Stage IV - (2003 to 2015)

W - 2:
Tax on purchase of goods for both taxable and exempt supplies
(675,000 / 450,0000 x 3,000,000) 450,000

This input tax of Rs. 450,000 shall be allowed proportionately:


(450,000 x 3,750,000 / (3,750,000 + 750,000) ) 375,000

(N - 1) As it is case of a manufacturer and none of the clause under SRO 647(I)/2007 dated June 27, 2007 is
applicable therefore 90% of output tax limitation on input is applicable U/S 8B of the Sales Tax Act,
1990.

Winter - 2004 Q. 6

Mr. Fakhar Rizvi is a commercial importer of taxable goods. During the month of July, 2004 he imported goods valuing
Rs. 1,500,000. On these imports he paid custom duty @ 10%, general sales @ 15% and income tax @ 6%. He also
opted to pay Sales Tax on value addition at the time of imports.

In addition to the above he provided following further information:

1. All imported goods were sold during the month of import i.e. July, 2004.

2. There were no opening or closing inventories.

3. His actual rate of value addition was 14%.

4. GST paid on utility bills during the tax period amounted to Rs. 5,000.

Required:

(i) Work out output tax, input tax and sales tax payable for the tax period of July, 2004.

(ii) Also state:


(A) who collects sales tax on imports and how?
(B) Last date of filing of sales tax return in above case.

Solution

(i)

Input tax: Rs.


On assessed value @ 17% 280,500
Additonal tax on value addition @ 3% paid at import stage 49,500
On utility bills paid (No entitlement to claim) -
Total input tax 330,000

Output tax:
Assessed value (N - 2) 1,650,000
14% value addition 231,000
Output tax @ 17% 1,881,000 319,770
319,770
Excss sales tax paid [ Output tax - Input tax ] (See note below) (10,230)

(N - 1) The importer is not entitled to claim refund on commercial imports sold at less than minimum value addition
that is presently 18.75%. Provided that all the available stock in sold by registered person.

(N - 2)

Assessed value
Import value 1,500,000
Custom duty @ 10% 150,000
Assessed value 1,650,000

(ii) (A)

Conceptual Approach to Taxes_______________________________________________


_____________________________________701
Chapter 13 Solved Past Papers Sales Tax Numericals of ICMAP Stage IV - (2003 to 2015)
Who collects sales tax on imports and how?

Sales tax on imports is collected by the Collector of Custom @ 17% on assesed value and 3% additional tax
of the same value as minimum value addition at the time of import clearance.

(ii) (B)

Last date of filing of sales tax return:

Last date of filing of sales tax return is 15-08-2015 as no sales tax is payable by the registered person.

Summer - 2004 Q. 8

M/s Safi Electronics are engaged in manufacturing of electronic goods and are registered under Sales Tax Act, 1990 as
Manufacturer-cum-Exporter. During the month of March, 2004 their Sales/Purchases data were recorded as under:
Rs.
(i) Local purchases:
(a) From registered persons 4,500,000
(b) from un-registered persons 1,200,000
(ii) Imports 2,300,000
(iii) Utility bills, (exclusive of GST Rs. 75,000) 500,000
(iv) Sales to Registered Persons 3,200,000
(v) Sales to Un-registered persons 3,600,000
(vi) Exports 3,000,000

Notes:

1. All the above figures are exlusive of sales tax paid or recovered.

2. The owner also took goods worth Rs. 200,000 for his private use.

3. Purchases include an invoice of Rs. 100,000 dated: 27-2-2004 which was not included in the Sales Tax Return for
February, 2004, due to it's late receipt.

4. Unadjusted imput tax carried forward from last month amounted to Rs. 45,000.

Required:
(i) Calculate Sales Tax payable by M/s Safi Electronics for the month of March 2004.

(ii) Give necessary explanations, on adjustments if any, to notes number 2, 3 and 4.


Solution

M/s Safi Electronics


Computation of sales tax payable
For the month of March 2016
Rs.
Output tax:
On sales to registered persons (Rs. 3,200,000 x 17%) 544,000
On sales to un-registered persons (Rs. 3,600,000 x 17%) (Note 5) 612,000
Exports [U/S 5 read with 5th Schedule] -
Goods taken for private use U/S 3 (Rs. 200,000 x 17%) (Note 1) 34,000
1,190,000
Input tax:
On purchases from registered persons (Rs. 4,500,000 x 17%) 765,000
On purchases from un-registered persons -
On imports used for manufacturing (Rs. 2,300,000 x 17%) 391,000
On utility bills paid 75,000
Unclaimed invoice of last month (Rs. 100,000 x 17%) (Note 2) 17,000
Unadjusted input tax of previous month (Note 3) 45,000
(A) 1,293,000

90% of output tax (N - 4) (B) 1,071,000


Sales tax payable [ Output tax - (lower of (A) or (B) ] 119,000

702____________________
___________________________________________________________Conceptual Approach to Taxes
Chapter 13 Solved Past Papers Sales Tax Numericals of ICMAP Stage IV - (2003 to 2015)
Sales tax credit c/f (A) less (B) 222,000

Explanations:

(N - 1) Goods taken for private use are chargeable to sales tax @ 17%.

(N - 2) Invoice not claimed in previous month can be claimed in six succeeding tax periods.

(N - 3) Previous month carry forward shall be included in the input tax.

(N - 4) As the zero rated supplies are less than 50% of all taxable supplies under SRO 647(I)/2007 dated June 27, 2007,
therefore 90% limitation is applicable U/S 8B of the Sales Tax Act, 1990.

(N - 5) It has been assumed that 2% further tax u/s 3(1A) is not applicable on local taxable supplies to unregistered
persons under SRO 648(I)/2013 dated July 09, 2013 otherwise further tax shall be accounted for and paid seperately
without adjustment of the same against input tax / refund of the registered person and further it shall also not be
considered for the computation of 90% limitation on output tax.

Conceptual Approach to Taxes_______________________________________________


_____________________________________703
Chapter 13 Solved Past Papers Sales Tax Numericals of ICMAP Stage IV - (2003 to 2015)

704____________________
___________________________________________________________Conceptual Approach to Taxes
Third, Fifth and Sixth Schedules Chapter-14

Chapter

14 THIRD, FIFTH AND SIXTH SCHEDULES

SCHEDULES
THE FIRST / SECOND / FORTH & SEVENTH SCHEDULES (Omitted)

(For CAF-6 AND ICMAP students)

THE THIRD SCHEDULE


See clause (a) of sub-section (2) of section 3 (Total 19 retail products)

SR. DESCRIPTION
1. Fruit juices & vegetable juices

2. Ice Cream

3. Aerated waters or beverages.

4. Syrups & squashes.


5. Cigarettes

6. Toilet soap

7. Detergents
8. Shampoo

9. Toothpaste
10. Shaving cream
11. Perfumery & cosmetics
12. Tea
13. Powder drinks
14. Milky drinks
15. Toilet paper & tissue paper
16. Spices sold in retail packing bearing brand names & trade marks
17. Shoe polish & shoe cream
18. Fertilizers
19. Cement sold in retail packing

Conceptual approach to Taxes 705


Third, Fifth and Sixth Schedules Chapter-14

THE FIFTH SCHEDULE


See section 4 [Total eleven (11) zero rated classes]
SR. HEADINGS EXPLANATION
1. (i) S UPPLY, REPAIR OR (a) a ship of gross tonnage of less than 15 LDT; nor
MAINTENANCE OF ANY SHIP
(b) a ship designed or adapted for use for recreation or pleasure.
WHICH IS NEITHER;

( II) S UPPLY, REPAIR OR


(a) an aircraft of weight-less than 8,000 kilograms; nor
MAINTENANCE OF ANY (b) an aircraft designed or adapted for use for recreation or pleasure.
AIRCRAFT WHICH IS
NEITHER; (iii) Supply of spare parts & equipment for ships & aircraft falling under (i) & (ii) above.

(iii) S UPPLIES OTHER THAN (iv) Supply of equipment & machinery for pilot age, salvage or towage services.
THE ABOVE [( FROM ( III ) (v) Supply of equipment & machinery for air navigation services.
TO ( VI)].
(vi) Supply of equipment & machinery for other services provided for the handling of ships
or aircraft in a port or Customs Airport.

2. S UPPLY TO DIPLOMATS , covered under various Acts, Orders, Rules, Regulations & Agreements passed by the
DIPLO MATIC MISSIONS , Parliament or issued or agreed by the Government of Pakistan.
PRIVILEGED PERSONS AND
PRIVILEGED ORGANIZATIONS

3. S UPPLIES TO DUTY FREE Provided that in case of clearance from duty free shops against various baggage rules
SHOPS issued under the Customs Act, 1969, the supplies from duty free shops shall be treated as
import for the purpose of levy of ST.

4. S UPPLIES OF RAW MAT ERIALS , For further manufacture of goods


COMPONENTS & GOODS TO
EXPORT PROCESSING ZONE
(EPZ)

5. S UPPLIES
OF SUCH LOCALLY - To petroleum & Gas sector Exploration & Production companies,
MANUFACTURED PLANT AND
MACHINERY
- Their contractors & sub-contractors
as may be specified by the Federal Government, by notification in the official Gazette,
subject to such conditions and restrictions as may be specified in such notification.

6. S UPPLIES OF LOCALLY (i) Plant and machinery, operated by power of any description, as is used for the
MANUFACTURED PLANT AND manufacture or production of goods by that manufacturer;
MACHINERY OF THE
(ii) Apparatus, appliances and equipments specifically meant or adapted for use in
FOLLOWING
conjunction with the machinery specified in clause (i);
SPECIFICATIONS , TO
MANUFACTURERS IN THE (iii) Mechanical and electrical control and transmission gear, meant or adapted for use in
E XPORT P ROCESSING conjunction with machinery specified in clause (i); and
Z ONE , SUBJECT TO THE (iv) Parts of machinery as specified in clauses (i), (ii) and (iii) identifiable for use in or with
CONDITIONS , such machinery.
RESTRICTIONS AND
Conditions, restrictions and procedures:-
PROCEDURE GIVEN BELOW ,
NAMELY :- (a) the supplier of the machinery is registered under the Act;
(b) proper bill of export is filed showing registration number;
(c) the purchaser of the machinery is an established manufacturer located in the Export
Processing Zone and holds a certificate from the Export Processing Zone Authority to
that effect;
(d) the purchaser submits an indemnity bond in proper form to the satisfaction of the
concerned CIR that the machinery shall, without prior permission from the said
Commissioner, not be sold, transferred or otherwise moved out of the Export
Processing Zone before a period of five years from the date of entry into the Zone;
(e) if the machinery is brought to tariff area of Pakistan, sales tax shall be charged on the
value assessed on the bill of entry; and
(f) breach of any of the conditions specified herein shall attract legal action under the
relevant provisions of the Act, besides recovery of the amount of sales tax along with
default surcharge and penalties involved.

706 Conceptual approach to Taxes


Third, Fifth and Sixth Schedules Chapter-14

7. S UPPLIES MADE TO Subject to the observance of procedures, restrictions and conditions prescribed therein.
EXPORTERS UNDER THE D UTY
AND TAX RE MISSION R ULES ,
2001

8. I MPORTS OR SUPPLIES MADE Excluding vehicles falling under heading 87.02 of the Pakistan Customs Tariff, subject to
TO G AWADAR S PECIAL such conditions, limitations and restrictions as the Board may impose.
E CONOMIC Z ONE

9. G OODS EXEMPTED UNDER If exported by a manufacturer.


SECTION 13

10. P ETROLEUM CRUDE OIL P ETROLEUM CRUDE OIL

11. RAW MAT ERIALS , If imported or purchases locally for use in the manufacturing of such plant and machinery
COMPONENTS , SUB as is chargeable to sales tax at the rate of zero percent subject to the condition that the
COMPONENTS AND PARTS importer or purchaser of such goods holds a valid sales tax registration showing his
registration category as manufacturer; and in case of import, all the conditions,
restrictions, limitations and procedures as are imposed by notification under section 19 of
the Customs Act, 1969 shall apply.

12. S PECIFIED GOODS The following goods and the raw materials, packing materials, sub components,
components, sub assemblies and assemblies imported or purchases locally for the
manufacture of the said goods subject to the conditions, limitations and restrictions as
specified in chapter XIV of the Sales Tax Special Procedure Rules, 2007:-
(i) Colours in sets
(ii) Writing, drawing and marking inks
(iii) Erasers
(iv) Exercise books
(v) Pencils sharpeners
(vi) Geometry boxes
(vii) Pens, ball pens, markers and porous tipped pens
(viii) Pencils including color pencils
(ix) Milk
(x) Preparations for infant use put up for retail sale
(xi) Fat filled mild
(xii) Bi cycles

Conceptual approach to Taxes 707


Third, Fifth and Sixth Schedules Chapter-14

THE SIXTH SCHEDULE


[See section 13(1)]
Table-1 (Imports or Supplies) (Total 96 classes)

SR. P RODUCT NAME

1. Live Animals & live poultry.


2. Meat of bovine animals, sheep & goat, excluding poultry & offal, whether or not fresh, frozen or otherwise, preserved.
3. Fish & crustaceans excluding live fish whether or not fresh, frozen or otherwise preserved.
4. Eggs including eggs for hatching.
5. Live plants including bulbs, roots & the like.
6. Edible vegetables including roots and tubers, except ware potato & onions whether fresh, frozen or otherwise
preserved (e.g. in cold storage) but excluding those bottled or canned.
7. Pulses.
8. Edible fruits excluding imported fruits (except fruits imported from Afghanistan) whether fresh, frozen or otherwise
preserved but excluding those bottled or canned.
9. Red chillies excluding those sold in retail packing bearing brand names & trademarks.
10. Ginger excluding those sold in retail packing bearing brand names and trademarks.
11. Turmeric excluding those sold in retail packing bearing brand names and trademarks.
12. Cereals & products of milling industry.
13. Seeds, fruit and spores of a kind used for sowing.
14. Cinchona bark.
15. Sugar beet.
16. Sugar cane
17. Edible oils & vegetable ghee, including cooking oil, on which FED is charged, levied & collected by a registered
manufacturer or importer as if it were a tax payable u/s 3 of the Act.
Explanation- Exemption of this entry shall not be available to distributors, wholesalers or retailers.
18. Fruit Juices, whether fresh, frozen or otherwise preserved but excluding those bottled, canned or packaged.
20. Poultry feed and Cattle feed including their all ingredients except soyabean meal & oil-cake of cottonseed.
21. Table salt including iodized salt excluding salt sold in retail packing bearing brand names & trademarks.
22. Glass bangles
23. Holy Quran, complete or in parts, with or without translation; Quranic Verses recorded on any analogue or digital
media; other Holy books-
24. Newsprint, newspapers, journals, periodicals, books but excluding directories.
25. Currency notes, bank notes, shares, stocks & bonds.
26. Silver, in un-worked condition.
27. Gold, in un-worked condition.
28. Monetary gold.
30. Dextrose & saline infusion giving sets along with empty non-toxic bags for infusion solution, Dextrose & saline
infusion giving sets. Artificial parts of the body, Intra-Ocular lenses and Glucose testing equipment.
31. Goods imported by various agencies of the United Nations, diplomats, diplomatic missions, privileged persons & privileged
organizations which are covered under various Acts and Orders, rules & regulations made there under; and agreements by
the Federal Government provided that such goods are charged to zero-rate of customs duty under Customs Act, 1969, and
the conditions laid therein

708 Conceptual approach to Taxes


Third, Fifth and Sixth Schedules Chapter-14

32. Import of articles of household and personal effects including vehicles & also the goods for donation to projects established
in Pakistan imported by any of the rulers of Gulf Sheikhdoms who is in possession of residential accommodation in Pakistan
and goods including vehicles by the UAE dignitaries as are listed in column (2) against heading No. 99.05 in column (1) of
the First Schedule to the Customs Act, 1969 for their personal use and for donation to welfare projects established in
Pakistan subject to the similar conditions as are envisaged for the purposes of applying zero-rate of customs duty on such
goods under the said Act.
33. Goods imported or supplied under grants-in-aid for which a specific consent has been obtained from the Board; supplies &
imports under agreements signed by the Government of Pakistan before the 30-06-1996, provided the agreements
contained the provision for exemption of tax at the time of signing of agreement.
34. Import of all goods received, in the event of a natural disaster or other catastrophe, as gifts & relief consignments, including
goods imported for the President's Fund for Afghan Refugees, relief goods donated for Afghan Refugees, gifts for
President's Fund for Assistance of Palestine and gifts received by Pakistani organizations from Church World Services or
the Catholic Relief Services subject to the similar conditions as are envisaged for the purposes of applying zero-rate of
customs duty under the Custom Act.
35. Articles imported through post as unsolicited gifts, subject to the same conditions as are envisaged for the purposes of
applying zero-rate of customs duty under the Customs Act, 1969.
36. Imported samples, subject to the same conditions a'. are envisaged for the purposes of applying zero-rate of customs duty
under the Customs Act, 1969.
37. Goods imported by or donated to hospitals run by the Federal Government or a Provincial Government; and non-profit
making educational & research institutions subject to the similar restrictions, limitations, conditions and procedures as are
envisaged for the purpose of applying zero-rate of customs duty on such goods under the Customs Act, 1969.
38. Goods supplied to hospitals run by the Federal Government or Provincial Government or charitable operating hospitals of
fifty (50) beds or more or the teaching hospitals of statutory universities of two hundred (200) or more beds.
39. Import of all such gifts as are received, and such equipment for fighting tuberculosis, leprosy, AIDS and cancer and such
equipment and apparatus for the rehabilitation of the deaf, the blind, crippled or mentally retarded as are purchased or
otherwise secured by a charitable non-profit making institution solely for the purpose of advancing declared objectives of
such institution, subject to the similar conditions as are envisaged for the purposes of applying zero-rate of customs duty
under the Customs Act, 1969.
40. Educational, scientific & cultural material imported from a country signatory to UNESCO Agreement or a country signatory to
bilateral commodity exchange agreement with Pakistan, subject to the same conditions as are envisaged for the purposes
of exemption under the Customs Act, 1969.
41. Import of replacement goods supplied free of cost in lieu of defective goods imported, subject to similar conditions as
are envisaged for the purposes of applying zero-rate of customs duty under the Customs Act, 1969.
43. Goods (including dry fruits imported from Afghanistan) temporarily imported into Pakistan, meant for subsequent exportation
charged to zero-rate of customs duty subject to the similar restrictions, limitations, conditions & procedures as are envisaged
for the purpose of applying zero-rate of customs duty on such goods under the Customs Act, 1969.
44. Import of ship stores, subject to the procedures, conditions & restrictions as may be specified by the Collector of Customs in
this behalf including those consignments of such stores that have been released without charging sales tax since 01-07-
1998, but excluding such consignments of ship stores as have been cleared on payment of ST.
45. Artificial kidneys, eye cornea, hemodialysis machines, hemodialyzers, A.V. fistula needles, hemodialysis fluids & powder,
blood tubing tines for dialysis & reverse osmosis plants for dialysis, double lumen catheter for dialysis, catheter for renal
failure patient & peritoneal dialysis solution & angioplasty equipment (balloons, catheters, wires and stents), subject to the
similar conditions and procedures as are envisaged for the purpose of applying zero-rate of customs duty on these goods
under the Customs Act, 1969.
46. Contraceptives & accessories thereof.

47. Goods produced or manufactured in & exported from Pakistan which are subsequently imported in Pakistan within 1 year of
their exportation, provided conditions of section 22 of the Customs Act, 1969 are complied with.

48. Personal wearing apparel & bonafide baggage imported by overseas Pakistanis & tourists, if imported under various
baggage rules & is exempt from Customs duties.

49. Goods & services purchased by non-resident entrepreneurs & in trade fairs & exhibitions subject to reciprocity & such
conditions & restrictions as may be specified by the Board.

50 Goods and services purchased by non-resident entrepreneurs and in trade fairs and exhibitions subject to reciprocity and
such conditions and restrictions as may be specified by the Board.

51 Uncooked poultry meat

52 Milk

Conceptual approach to Taxes 709


Third, Fifth and Sixth Schedules Chapter-14

53 Milk and cream, concentrated or containing added sugar or other sweetening matter, excluding that 04.01 and 04.02 sold in
retail packing under a brand name
54 Flavored milk, excluding that sold in retail packing under a brand name

55 Yogurt, excluding that sold in retail packing under a brand name

56 Whey, excluding that sold in retail packing under a brand name

57 Butter, excluding that sold in retail packing under a brand name

58 Desi ghee, excluding that sold in retail packing under a brand name

60 Cheese, excluding that sold in retail packing under a brand name

61 Processed cheese not grated or powdered, excluding that sold in retail packing under a brand name

61 Cotton seed

62 Frozen, prepared or preserved sausages and similar products of poultry meat or meat offal

63 Meat and similar products of prepared frozen or preserved meat or meat offal of all types including poultry meat and fish

64 Preparations for infant use, put up for retail sale

65 Fat filled milk

66 Colours in sets (Poster colours)


67 Writing, drawing and marking inks
68 Erasers
69 Exercise books
70 Pencil sharpeners
71 Energy saver lamps
72 Sewing machines of the household type
73 Bicycles
74 Wheelchairs
75 Vessels for breaking up
76 Other drawing, marking out or mathematical
calculating instruments (geometry box)
77 Pens and ball pens
78 Pencils including color pencils
79 Compost (non-chemical fertilizer) produced and supplied locally
80 Construction materials to Gawader Export Processing Zones investors and to Export Processing Zone G awader for
development of Zones infrastructure.
81 Raw and pickled hides and skins, wet blue hides and skins, finished leather, and accessories, components and trimmings, if
imported by a registered leather goods manufacturer, for themanufacture of goods wholly for export, provided that conditions,
procedures and restrictions laid down in rules 264 to 278 of the Customs Rules, 2001 are duly fulfilled and complied with.
82 Machinery, equipment and Respective materials imported either for headings exclusive use within the limits of Export Processing
Zone or for making exports there from, and goods imported for warehousing purpose in Export Processing Zone, subject to the
conditions that such machinery, equipment, materials and goods are imported by investors of Export Processing Zones, and all
the procedures, limitations and restrictions as are applicable on such goods under the Customs Act, 1969 (IV of 1969) and
rules made there under shall mutatis mutandis, apply.
83 Import and supply thereof, up to the year 2020, of ships of gross tonnage of less than 15 LDT and all floating crafts including tugs,
dredgers, survey vessels and other specialized crafts purchased or bare-boat chartered by a Pakistan entity and flying the
Pakistan flag, except ships or crafts acquired for demolition purposes or are designed or adapted for use for recreation or
pleasure purposes, subject to the condition that such ships or crafts are used only for the purpose for which they were procured,
and in case such ships or crafts are used for demolition purposes within a period of five years of their acquisition, sales tax
applicable to such ships purchased for demolition purposes shall be chargeable.
84 Substances registered as drugs under the Drugs Act, 1976 (XXXI of 1976) and medicaments as are classifiable under
Chapter 30 of the First Schedule to the Customs Act, 1969 (IV of 1969) except the following, even if medicated or
medicinal in nature, namely

710 Conceptual approach to Taxes


Third, Fifth and Sixth Schedules Chapter-14

(a) filled infusion solution bags imported with or without infusion given sets;
(b) scrubs, detergents and washing preparations;
(c) soft soap or no-soap soap;
(d) adhesive plaster;
(e) surgical tapes;
(f) liquid paraffin;
(g) disinfectants; and
(h) cosmetics andtoilet preparations
85 Raw materials for the basic manufacture of pharmaceutical active ingredients and for manufacture of pharmaceutical
products, provided that in case of import, only such raw materials shall be entitled to exemption which are liable to customs
duty not exceeding ten per cent advalorem, either under the First Schedule or Fifth Schedule to the Customs Act, 1969 or
under a notification issued under section 19 thereof.
86 Import of Halal edible offal of bovine animals.
87 Import and supply of iodized salt bearing brand names and trademarks whether or not sold in retail packing.
88 Components or sub-components of energy saver lamps, namely:-
(a) Electronic Circuit (b) Plastic Caps (Upper and Lower)
(c) Base Caps B22 and E27 (d) Tungsten Filaments
(e) Lead-in-wire (f) Fluorescent Powder (Tri Band Phosphor)
(g) Adhesive Additive (h) Al-Oxide Suspension
(i) Capping Cement (j) Stamp Pad Ink
(k) Gutter for Suspension
89 Goods imported temporarily with a view to subsequent exportation, as concerned by the Board, including passenger service
item, provision and stores of Pakistani Airlines.
90 The following items with dedicated use of renewable source of energy like solar and wind, subject to certification by the
Alternative Energy Development Board (AEDB), Islamabad:-
(a) Solar PV panels;
(b) LVD induction lamps;
(c) SMD, LEDs with or without ballast, with fittings andfixtures;
(d) Wind turbines including alternators and mast;
(e) Solar torches;
(f) Lanterns and related instruments;
(g) PV modules along with related components, including invertors, charge controllers and batteries.
91 White crystalline sugar
92 Following cardiology/cardiac surgery, neurovascular, electrophysiology, endosurgery, endoscopy, oncology, urology,
gynaecology, disposables and other equipment:-
A. ANGIOPLASTY PRODUCTS
1. Coronary Artery Stents
2. Drugs Eluting Coronary Artery Stents
3. Coronary Artery Dialatation Catheters (Balloons)
4. PTCA Guide Wire
5. PTCA Guiding Catheters
6. Inflation Devices/Priority Packs
B. ANGIOGRAPHY PRODCUTS
1. Angiography Catheters
2. Sheaths
3. Guide Wires
4. Contrast Lines
5. Pressure Lines

Conceptual approach to Taxes 711


Third, Fifth and Sixth Schedules Chapter-14

6. Manifolds
C. CONTRAST MEDIA FOR ANGIOGRAPHY/ ANGIOPLASTY
1. Angiography Accessories
2. ASD Closure Devices
3. ASD Delivery Systems
4. VSD Closure Devices
5. VSD Delivery System
6. Guide Wires
7. Sizing Balloons
8. Sizing Plates
9. PDA Closure Devices
10. PDA Delivery system
D. TEMPORARY PACEMAKER (with leads, connectors and accessories)
E. PERMANENT PACEMAKER (with leads, connectors and accessories)
F. HEART FAILURE DEVICES (with leads, connectors and accessories)
G. IMPLANTABLE CARDIOVERTES (with leads, connectors and accessories)
H. CARDIAC ELECTRO PHYSIOLOGY PRODUCTS
1. Electrophysiology catheters
2. Electrophysiology cables
3. Electrophysiology connectors
I. LEAR CARDIOLOGY PRODUCTS
1. Radioactive isotopes
2. Cold kits (Cardioloite MAA, DTPA etc)
J. CARDIAC SURGERY PRODUCTS
1. Oxygenators
2. Cannulas
3. Prosthetic Heart
4. Valves
5. Luminal Shunts for heart surgery
6. Artificial limbs and appliances
K. EQUIPMENT
1. Cardiac Angiography Machine
2. Echocardiography Machines
3. ETT Machines
4. Gamma Camera for Nuclear cardiology studies
L. PERIPHERAL INTERVENTIONS EQUIPMENT
Disposables and other equipment for peripheral interventions including stents (including carotid and wall stents), balloons,
sheaths, catheters, guide wires, filter wires coils, needles, valves (including rotating homeostatic valves), connecting cables,
inflation devices adaptors.
93 High Efficiency Irrigation Equipment. (If used for agriculture sector)
(1) Submersible pumps (up to 75 lbs and head 150 meters)
(2) Sprinklers including high and low pressure (center pivotal) system, conventional sprinkler equipment, water reel
traveling sprinkler, drip or trickle irrigation equipment, mint irrigation sprinkler system.
(3) Air release valves, pressure gauges, water meters, back flow preventers, andautomatic controllers.
94 Green House Farming and Other Green House Equipment consisting of plastic covering and mulch film, anti-insect net and
shade net.(If used for agriculture sector)
1. Tunnel farming equipment
2. Green houses (prefabricated).
95 Plant, machinery and equipment Respective imported for setting up fruit headings processing and preservation units in Gilgit-
Baltistan, Balochistan Province and Malakand Division subject to the same conditions and procedure as are applicable for
import of such plant, machinery and equipment under the Customs Act, 1969 (IV of 1969).

712 Conceptual approach to Taxes


Third, Fifth and Sixth Schedules Chapter-14

th
96 Plant, machinery and equipment imported for setting up industries in FATA upto the 30 June, 2019 subject to the same
conditions and procedure as are applicable for import of such plant machinery and equipment under the Customs Act,
1969 (IV of 1969).

Appliances for colostomy

Colostomy and urostomy bags

Tubular day lighting devices (TDDs)

Diagnostic kits or equipment, namely:- 3822.000


HIV Kits 4C Es Trionyx, 5C Cell control Lnormal, Bovine precision multi sera, Pregnancy test, DNA SSP DRB Generic IC,
Reticulocyte count (control) retic, C Control, Kit for vitamin B12 estimation, Ferritin kit, HEV (Hepatitis E virus), ID-DA Cell,
Urine Analysis Strips, Albumin beg, Cratinin sysi, Ring, Detektiion cups, ISE Standard, Alkaline phosphatase (Alb), Bilirubin
kit, HDL Cholesterol, Ck creatinin kinase (mb), Ck nac, Glulcose kit, Ammonia Modular, Lac, Ldh kit (lactate dehydrogenase
kit), Urea uv kit, Ua plus, Tina quant, Crp control, Aslo tin, Proteins, Lipids, HDL/LDL cholesterol, Protein kit, U, Control
Sera, Pac, Control, HCV, UIBC (Unsaturated iron binding capacity), U/CSF, Inorganic Phosphorus kit, Kit amplicon kit (for
PCR), Ige, Lc hsv, Oligo, NA/K/CL, Hcy, Standard [or calibrated], Hla B27, Liss Coombs, Typhoid kit, HCV amp, Urine test
strips, Strips for sugar test, Blood glucose test strips, Kits for automatic cell separator for collection of platelets, Elisa or Eclia
kit, PCR kits, Immunoblast (western blot test), I.C.T. (Immunochromatographic kit), CBC Reagent (For hematology
analyzer), Complete blood count reagent.

Blood Bag CPDA-1 with blood transfusion set pack in aluminum foil with set. Respective headings

Urine drainage bags Respective headings

Aircraft, whether imported or acquired on wet or dry lease

Maintenance kits for use in trainer aircrafts of PCT headings Respective headings

Spare parts for use in aircrafts, trainer aircrafts or simulators Respective headings

Machinery, equipment and tools for setting up maintenance, repair and overhaul (MRO) workshop by MRO company
recognized by Aviation Division Respective headings

Operational tools, machinery, equipment and furniture and fixtures on one-time basis for setting up Greenfield airports by a
company authorized by Aviation Division Respective headings

Aviation simulators imported by airline company recognized by Aviation Division

Import of plant, machinery and production line equipment used for the manufacturing of mobile phones by the local
manufacturers of mobile phones duly certified by the Pakistan Telecommunication Authority. Respective headings

Table 2
(Exempt Local Supplies only) (Total 14 classes)

SR. H EADING P RODUCT NAME


1. SUPPLY OF COTTON Supply of cottonseed exclusively meant for sowing purposes, subject to such conditions as
SEED FOR SOWING the Board may specify.
PURPOSES

2. SUPPLY OF LOC ALLY Supply of locally produced crude vegetable oil obtained from the locally produced seeds,
PRODUCED CRUDE except cooking oil, without having undergone any process except the process of washing.
VEGETABLE OIL

3. SUPPLIES MADE BY Supplies made by cottage industry;


COTT AGE INDUSTRY

4. IN HOUSE USE OF Raw material & intermediary goods manufactured or produced, & services provided or
MATERIAL & SERVICES rendered, by a registered person, consumed in-house for the manufacture of goods
BY RP subject to sales tax.
5. LIMITATIONS ON INPUT Supply of fixed assets against which input tax adjustment is not available under a
ST ON FIXED ASSETS notification issued in terms of clause (b) of sub-section (1) of section 8 of the Sales Tax
UNDER NOTIFICATION Act, 1990.
6. FOOD ITEMS Breads prepared in tandoors & bakeries, vermicillies, nans, chapattis, sheer mal, bun,
rusk.

Conceptual approach to Taxes 713


Third, Fifth and Sixth Schedules Chapter-14

7. FOODSTUFF COOKED OR and served in messes run on the basis of mutuality & industrial canteens for workers.
PREPARED IN- HOUSE

8. FOODSTUFF & OTHER in the flight kitchens & supplied for consumption on-board in local flights.
EATABLES PREPARED

9. AGRICULTURAL that are not subjected to any further process of manufacture.


PRODUCE OF P AKISTAN

10. S UPPLY OF WARE S UPPLY OF WARE POTATO & ONIONS


POTATO & ONIONS

11. E QUIPMENTS a. S PRINKLER EQUIPMENT


b. D RIP EQUIPMENT
c. S PRAY PUMPS AND NOZZLES

12. C OTTON R AW COTTON AND GINNED COTTON

Raw and pickled hides and skins, wet blue hides and skins

13. Supplies made by having annual turnover less than five million rupees even if their annual utility bill is more than
manufacturers of eight hundred thousand rupees.
marble and granite
th
14. B RICKS up to 30 June, 2018
th
15. Crushed stone up to 30 June, 2018

16. Poultry Feed, cattle 2306.3000, 2306.4900 and Respective headings


feed, sun flower feed,
meal, rape seed meal
and canola seed meat

NOTES:
1. For the purposes of this Schedule, for entries against which classification of headings or sub-headings has been specified,
exemption shall be admissible on the basis of description of goods as mentioned in column 2 of this Schedule Pakistan
Customs Tariff classification of headings is provided for ease of reference & commodity classification purposes only.
2. For the purposes of determining classification of any goods, the general rules for interpretation of the First Schedule to the
Customs Act, 1969 & Explanatory Notes to the Harmonized Commodity Description and Coding System (relevant version)
as amended from time to time shall be considered authentic source of interpretation.
3. For the purposes of exemption of Sales tax under table 1 under various serial numbers (where applicable) of this Schedule,
the definitions, restrictions, limitations, conditions and procedures and all the provisions of Chapter 99 of the First Schedule
to the Customs Act, 1969, for the purposes of applying zero-rate of customs duty shall, mutatis mutandis, apply and shall be
deemed and construed to be part of this Schedule

714 Conceptual approach to Taxes


Eighth and Ninth Schedules Chapter-15

Chapter

15 EIGHTH AND NINTH SCHEDULES

EIGHTH SCHEDULE
[See clause (aa) of sub-section (2) of section 3]

Heading Nos. of the Rate


S. First Schedule to the of
Description Condition
No. Customs Act, 1969 (IV Sales
of 1969) Tax
(1) (2) (3) (4) (5)
1 Soyabean meal 2304.0000 10%
2 Oil cake and other solid 2306.1000 5%
residues, whether or not
ground or in the form of
pellets
4 Oilseeds meant for Respective headings 5% Import thereof subject to the condition that the
sowing concerned department of the Division dealing with
the subject-matter of oil seed certificates that
imported seeds are fungicide and insecticide
treated and are meant for sowing.
5 Raw cotton and ginned Respective headings 5% On import
cotton
6 Plant and machinery not Respective headings 10% (i) On import of such plant and machinery by
manufactured locally and registered manufacturers post-dated
having no compatible local cheque(s) equal to the differential amount of
substitutes sales tax payable at import stage, shall be
submitted to the customs authorities, which
shall be returned on furnishing proof of
filing of first sales tax return after import of
such machinery, showing the import of such
machinery;
(ii) On import by commercial importers, good-for-
payment cheque, bank guarantee, pay
order or treasury challan showing deposit,
equal to the differential amount of sales tax
payable at import stage, shall be submitted
to the customs authorities which shall be
returned back or as the case may be
refunded, after evidence of subsequent
supply to registered manufacturers or
industrial users is furnished to the customs
authorities; tax payable at import stage, shall
be submitted to the customs authorities,
which shall be returned back, or as the case
may be, refunded, after evidence of
subsequent supply to registered
manufacturers orindustrial users isfurnished to
the customs authorities;

Conceptual approach to Taxes 715


Eighth and Ninth Schedules Chapter-15

(iii) (iii) Supply of such imported plant and


machinery by commercial importers to
unregistered persons or persons other than
manufacturers shall be liable to standard
rate of tax, and evidence to that effect shall
be produced to the customs authorities for
release of the aforementioned instruments
or refund of the amount paid at import
stage;
(iv) Subsequent supply of plant and machinery
imported or acquired by registered
manufacturers to unregistered persons
other than manufacturers shall be liable to
tax at standard rate; and
(v) The validity period of instruments furnished
under this provision shall not be less than
one hundred and twenty days.
Explanation. For the persons of this provision,
plant and machinery means such plant and
machinery as is used in the manufacturers or
production of goods.
7. Flavoured milk 0402.9900 10% Sold in retail packing under a brand name
8. Yogurt 0403.1000 Sold in retail packing under a brand name
9. Cheese 0406.1010 10% Sold in retail packing under a brand name
10. Butter 0405.1000 10% Sold in retail packing under a brand name
11. Cream 04.01 and 04.02 10% Sold in retail packing under a brand name
12. Desi ghee 0405.9000 10% Sold in retail packing under a brand name
13. Whey 04.04 10% Sold in retail packing under a brand name
14. Milk and cream, 0402.1000 10% Sold in retail packing under a brand name
concentrated and added
sugar or other sweetening
matter
15. Poultry feed, cattle feed, 2301.2090, 2305.0000, 5%
and their ingredients 2306.2000, 2306.4100,
except soya bean meal of 2306.5000, 2309.9010,
PCT heading 2304.0000 2309.9020, 2309.9090,
and oil-cake of cottonseed 2936.2100, 2936.2200,
falling under PCT heading 2936.2300, 2936.2400,
2306.1000 2936.2500, 2936.2600,
2936.2700, 2936.2800
and 2308.9000 (Guar
Meal), 2303.1000, (Corn
Gluton Feed/Meal),
2303.1000 (Residues of
starch manufacture and
similar residues),
3507.9000
(Enzymesother),
2302.1000 (Maize Bran),
2302.2000 (Rice Bran),
2302.3000 (Wheat
Bran), 2302.4000 (Other
Cereals), 2302.5000
(Bran of Leguminous
Plants), 2306.7000 (Oil-
cake and other solid
residues of Maize (corn)
germ), 2306.4900

716 Conceptual approach to Taxes


Eighth and Ninth Schedules Chapter-15

(Sesame Cake),
2306.9000 (Sesame
Meal/other Meal),
2842.1000 (Double or
complex silicates,
including
aluminosilicates whether
or not chemically
defined), 2301.2010
(Fish Meal), 0505.9000
(Poultry by product
Meal), and the following
items only of Feed
Grade:
2827.6000 (Potassium
Lodide), 2833.2990
(Manganese Sulphate),
2833.2600 (Zinc
Sulphate), 2817.4000
(Zinc Oxide), 2833.2500
(Copper Sulphate),
2833.2910 (Ferrous
Sulphate), 2915.5000
(Propionic acid, its salts
and esters), 2930.4000
(DL Methionine),
2930.4000 (Methionine
Hydroxy Analogue
(liquid)), 2922.4100
(Lysine Monohydro
Chloride /sulphate),
2923.2000 (Lecithins),
2923.9000 (Betafin),
2922.4290 (Arganine),
2934.9910
(Furazolidone),
2922.5000 (Threonine),
2835.2600 (Mono
Calcium Phosphate),
16. Incinerators of disposal of 8417.8000, 5%
waste management, 8430.2000 and
motorized weepers and 8479.8990
snow ploughs
17. Re-importation of foreign 99.18 5% Subject to similar conditions as are envisaged for
origin goods which were the purposes of customs duty under the Customs
temporarily exported out Act, 1969, and taxable value shall be the value
of Pakistan determined under PCT heading 99.18 of the said
Act increased by customs duty payable.
18. Reclaimed lead Respective headings 5% If supplied to recognized manufacturers of lead
and lead batteries
19. Waste paper 47.07 5% If supplied locally
20. Plant, machinery, Respective headings 5% The Alternative Energy Development Board
equipment and specific (AEDB), Islamabad shall certify in the prescribed
items used in production manner and format as per Annex-B, as given in the
of biodiesel Sixth Schedule, that the imported goods are
bonafide project requirement.
The goods shall not be sold or otherwise disposed
of within a period of five years of their import
except with the prior approval of the FBR and
payment of customs duties and taxes leviable at
the time of import

Conceptual approach to Taxes 717


Eighth and Ninth Schedules Chapter-15

21. Rapeseed, sunflower 1205.0000, 1206.0000 16% On import by solvent extraction industries
seed and canola seed
22. Soya bean seed 1201.1000 6% On import by solvent extraction industries, subject
to the condition that no refund of input tax shall be
admissible;
23. Second hand and worn 6309.0000 5%
clothing or footwear
25. Agricultural tractors 8701.9020 10%
26. Tillage and seed bed 8432.8010 7%
preparation equipment: 8432.2910
(i) Rotavator (ii) 8432.8090
Cultivator (iii)
8432.3090
Ridger
8432.8090
(iv) Sub soiler
8432.1010
(v) Rotary slasher
8432.1090
(vi) Chisel plow
8432.2990
(vii) Ditcher
8432.2100
(viii) Border disc
8432.2990
(ix) Disc harrow
8432.1090
(x) Bar harrow
8430.6900
(xi) Mould board
plow 8430.6900
(xii) Tractor rear or 8432.8090
front blade 8432.1090
(xiii) Land leveller 8432.8090
or land planer 8432.8090
(xiv) Rotary tiller 8701.9020
(xv) Disc plow 8430.6900
(xvi) Soil-scrapper
(xvii) K.R.Karundi
(xviii) Tractor
mounted trancher
(xix) Land leveller
27. Seeding or planting 7%
equipment:
(i) Seed-cumfertilizer drill 8432.3010
(wheat, rice barley, etc.)
(ii) Cotton or maize
8432.3090
planter with fertilizer
attachment 8432.3090
(iii) Potato planter
8432.3090 8432.3090
(iv) Fertilizer or manure
spreader or broadcaster 8432.4000
8432.4000
(v) Rice transplanter
8432.3090
(vi) Canola or sunflower 8432.3090
drill 8432.3010
(vii) Sugarcane planter 8432.3010
8432.3090
8432.3090
28. Irrigation, drainage and 7%
agro-chemical application
equipment:
8421.2100,
(i) Tubewells filters or
718 Conceptual approach to Taxes
Eighth and Ninth Schedules Chapter-15

strainers 8421.9990
(ii) Knapsack sprayers 8424.2010
(iii) Granular applicator 8424.2010
(iv) Boom or field 8424.2010
sprayers 8424.2010
(v) Self propelled sprayers 8424.2010
(vi) Orchard sprayer

29. (i) Harvesting, threshing 8433.5200 7%


and storage equipment: 8433.5200
(ii) Wheat thresher
(iii) Maize or groundnut 8433.5900
thresher or sheller
8433.5300
(iv) Groundnut digger
(v) Potato digger or
8433.5200
harvester
8433.5900
(vi) Sunflower thresher
(vii) Post hole digger
8433.4000
(viii) Straw balers
8433.5900
(ix) Fodder rake
(x) Wheat or rice reaper 8433.5900
8433.5900
(xi) Chaff or fodder cutter
(xii) Cotton picker
8433.5900
(xiii) Onion or garlic
harvester 8433.5200
(xiv) Sugar harvester 8433.5200
(xv) Tractor trolley or
forage wagon 8716.8090
(xvi) Reaping machines 8433.5900
(xvii) Combined 8433.5100
harvesters 8433.5900
(xviii) Pruner/shears
Covered under the codes
30. Post-harvest handling 7%
and processing &
miscellaneous
machinery:
(i) Vegetables and fruits
cleaning and sorting or 8437.1000
grading equipment 8433.4000
(ii) Fodder and feed
cube maker equipment
31. Pesticides and their 38.08 7% In case of supplies, no input tax credit shall be
active ingredients admissible, except that of the tax paid under this
registered by the serial number
Department of plant
protection under the
Agricultural pesticides
ordinance, 1971 (II of
1971,) stabilizers,
emulsifiers and solvents,
namely:
-Bata pinene/Agrotin 527 / 2902.1990
Terpenic derivative
Mixed xylene isomers 2902.4400
Naphthalene 2902.9010

Conceptual approach to Taxes 719


Eighth and Ninth Schedules Chapter-15

Solvesso -100, 150, 200 2902.9090


Ingredients for pesticides 2903.3040
Cadusafos Technical 2903.6900
Methanol (methyl alcohol) 2905.3200
Propylene glycol 2905.4900
(propans 1, 2-diol) 2906.2910
-Adhesives Polyvinyl 2906.2990
Acetate
Polyvinyl Alocohol 2912.1100
Ingredients for pesticides
Other Ingredients for 2914.2200
pesticides
Metnoxy 2 propanl 2914.2999
Methnan (formaldehyde)
Cyclo-hexanone and 2915.2400
Mthyl-cyco-hexanones 2916.3920
-Cyclohexanon 2917.3200
Cyclohexanone Mixed 2918.9010
petroleum Xylene (1,2 &
1,3 & 1,4 dimethyl
benzene and ethyle
benzene)
Acetic anthydride 2919.0010
Ingredients for pesticides
Dioctyl othophthalates 1919.0090
Ingredients for pesticides 2920.9020
Ingredients for pesticides 2919.0090
Other Ingredients for 2920.9020
pesticides
Endosulfan Technical 2920.9090
Material
Other Ingredients for 2921.1200
pesticides
Endosulfan Technical 2921.4110
Material

Other Ingredients for 2921.4390


pesticides
Diethylamine and its salts 2921.5110
Ingredients for pesticides 2922.1300
Other Ingredients for 2924.1990
pesticides
Ingredients for pesticides 2924.2930
Triethanolamine and its 2924.2990
salts
Dimethyl Formanide 2926.9030
(DMF)

720 Conceptual approach to Taxes


Eighth and Ninth Schedules Chapter-15

Ingredients for pesticides 2926.9050


Other Ingredients for 2926.9090
pesticides
Alpha cyano, 3- 2930.9010
phenoxybenzy1(-) cis,
trans 3- (2, -2dictord vinl)
2, 2 dimethyl
cyclopropsan carboxylate
(S) Alpha cyano, 3- 2930.9020
phenoxybenzyl (S) -2-(4,
choro phenyl)-3 mehlyl
bulyrate
Cypemethin, Alpha 2930.9030
cypemethin, Bate-
Cypermethrin, Zela-
Cypemethrin,
Lambda cylalothin. 2930.9040
Deltametnin,
Fenproparthin,
Esfenvalorate Bifenthrin
Technical Meterial-
Acetamiprid, Imidacloprid
Technical Meterial
Monomohypo,
Chlorothalonil Technical
Material-Bromoxynil
Technical Methorial
Other nitrite compounds- 2930.9050
Cyfluthnin, Beta
Cyfluthnin Technical
Metrial
2-N, N-Dimethyl amino -1 2930.9060
sodium thiosulphate, 3-
thiosulfaoump\opane
O, S-dimethyl 2930.9070
phosphoramidolhioate
S-S (2 dimethyl amino 2930.9090
(tribethylene) bis (thio
carbamate)
Diafethiuran technical 2930.9090
(iterbutyl) 3-2-6 disopropyl
(4- phenoxyphenyl)
thiourene
O-O diethyl O- (3,5,6 2931.0010
trichloro pyridinyl)
O- (4-bromo, 2-chloro 2931.0090
phenyl) o-ethyl s-
propyl(phosphorathioate)
O, Oduethyl O-(3,5,6- 2932.2920
trichloro 2- pyridyl)
phosphorothioate
Ingredients for pesticides 2932.9910
Other orgonosulpher 2932.9990
compounds
-Ethion, Methamidophos 2933.1900
Technical Material-
Dimethysulfoxid

Conceptual approach to Taxes 721


Eighth and Ninth Schedules Chapter-15

Ingredients for pesticides 2933.3930


Other Ingredients for 2933.5990
pesticides
Ingredients for pesticides 2933.5950
2,3 Dihydro 2-2 dimethyl- 2933.5990
7 benzo furanyl methyl-
carbamate
Other ingredients for 2933.6910
pesticides
-Carbosulfan Technical 2933.6940
Meterial
Fipronil 2933.6990
Ingredients for pesticides 2933.7910
Other Ingredients for 2933.7920
pesticides
Chlorpyrifos, Triazophos, 2933.9910
Diazinon Technical
Material
Other Ingredients for 2934.1010
pesticides
Pyrimethanine 2934.9920
Ingredients for pesticides 2938.9010
Atrazine Technical 2939.9910
Material
Isstin (lactam of lstic acid 2941.9050
1-Vinyl-2,-pynol-idone 2941.9090
-Triazophos Technical 3402.1110
Material 3402.1190
Ingredients for pesticides 3402.1290
Ingredients for pesticides 3402.1300
-Methyl benzimidazol-2
ylcarbamate,
Dicopper chloride 3402.1990
trihydroxide
Ingredients for pesticides
-Abamectin, Emanectin
Technical Material
Other Ingredients for
pesticides
Sulphonic acid (Soft)
Other surface active
agents
Catonic
Non lonic surface active
agents
Other organic surface
Active agents 3402.9000
Chemical preparations 3824.9099

722 Conceptual approach to Taxes


Eighth and Ninth Schedules Chapter-15

Table-2

Plant, machinery, equipment and apparatus, including capital goods, specified in column (2) of the Annexure below,
falling under the HS Codes specified in column (3) of that Annexure, shall be charged to sales tax at the rate of 5%
except goods mentioned in serial numbers 1, 4 and 5 of the Annexure which shall be at the rate of 10% subject to the
following conditions, besides the conditions specified in column (4) of the Annexure, namely:-
(i) the imported goods as are not listed in the locally manufactured items, notified through a Customs General Order
issued by the Board from time to time or, as the case may be, certified as such by the Engineering Development
Board.
(ii) the Chief Executive, or the person next in hierarchy duly authorized by the Chief Executive or Head of the importing
company shall certify in the prescribed manner and format as per Annex-A that the imported items are the company's
bonafide requirement. He shall furnish all relevant information Online to Pakistan Customs Computerized System
against a specific user ID and password obtained under section 155D of the Customs Act, 1969. In already
computerized Collectorates or customs stations where the Pakistan Customs Computerized System is not operational,
the Project Director or any other person authorized by the Collector in this behalf shall enter the requisite information
in the Pakistan Customs Computerized System on daily basis, whereas entry of the data obtained from the customs
stations which have not yet been computerized shall be made on weekly basis; and
(iii) in case of partial shipments of machinery and equipment for setting up a plant, the importer shall, at the time of arrival
of first partial shipment, furnish complete details of the machinery, equipment and components required for the
complete plant, duly supported by the contract, lay out plan and drawings.
Explanation.-In this Table the expression, capital goods mean any plant, machinery, equipment, spares and
accessories, classified in chapters 84, 85 or any other chapter of the Pakistan Customs Tariff, required for-
(a) the manufacture or production of any goods, and includes refractory bricks and materials required for setting
up a furnace, catalysts, machine tools, packaging machinery and equipment, refrigeration equipment, power
generating sets and equipment, instruments for testing, research and development, quality control, pollution
control and the like; or
(b) usein mining, agriculture, fisheries, animal husbandry, floriculture, horticulture, livestock, dairy and poultry industry.

Conceptual approach to Taxes 723


Eighth and Ninth Schedules Chapter-15

Annexure
PCT
S. No Description Conditions
heading
(1) (2) (3) (4)
1 Machinery and equipment for development of Respective Nil
grain handling and storage facilities.
Headings
2 Cool chain machinery and equipment. Respective Nil
Headings

3 Respective
1. Machinery, Machinery, equipment 1. This concession shall be available to those
materials, capital goods specialized Headings Mineral Exploration and Extraction Companies
vehicles (4x4 non luxury) i.e. single or or their authorized operators or contractors
double cabin pickups accessories, spares, who hold permits, licenses leases and who
chemicals and consumables meant for enter into agreements with the
mineral exploration phase. Government of Pakistan or a Provincial
Government.
2. Temporarily imported goods shall be
2. Construction machinery, equipment and
cleared against a security in the form of a
specialized vehicles, excluding passenger
post-dated cheque for the differential
vehicles, imported on temporary basis as
amount between the statutory rate of
required for the exploration phase.
customs duty and sales tax and the
amount payable under notification, along
with an undertaking to pay the customs
duty and sales tax at the statutory rates in
case goods are not re-exported on
conclusion of the project.
3. The goods shall not b sold or otherwise
disposed of without prior approval of the
FBR and the payment of customs duties
and taxes liable at the time of import.
These shall however be allowed to be
transferred to other entitled mining
companies with prior approval of the
Board.
4 Complete plants for relocated industries Respective Nil
headings
5 Machinery, equipment and other capital goods
meant for initial installation, balancing,
modernization, replacement expansion of oil
refining (mineral oil, hydro- cracking and other
value added petroleum products) petrochemical and
petrochemical downstream products including
fibers and heavy chemical industry, cryogenic
facility forethylene storage and heading.

724 Conceptual approach to Taxes


Eighth and Ninth Schedules Chapter-15

Annex-A

Header Information
NTN/FTN of importer Regulatory Authority No. Name of Regulatory Authority
(1) (2) (3)
Details of Input goods (to be filled by the chief executive of the Goods imported (Collectorate of Import)
importing company)

Date of CRN/Mach
Quantity imported
Custom Duty rate

Sales Tax Rate

CRN/Mach No.
Collectorate
(applicable)
Description

(applicable
HS Code

Quantity
Specs

WHT

UOM

No.
(4) (5) (6) (7) (8) (9) (10) (11) (12) (13) (14) (15)

CERTIFICATE. It is certified that the description and quantity mentioned above are commensurate with the project
requirement and that the same are not manufactured locally. It is further certified that the above items shall not be used for
any other purpose.

Signature of Chief Executive, or


the person next in hierarchy duly
authorized by the Chief Executive
Name ______________________
N.I.C. No. __________________________________

NOTE:- In case of clearance through Pakistan Customs Computerized System, the above information shall be furnished on
line against a specific user I.D. and password obtained under section 155D of the Customs Act, 1969.
Explanation.-
Chief Executive means.-
1. owner of the firm, in case of sole proprietorship; or
2. partner of firm having major share, in case of partnership firm; or
3. Chief Executive Officer or the Managing Director in case of limited company or multinational organization; or
4. Principal Officer in case of a foreign company.

Conceptual approach to Taxes 725


Eighth and Ninth Schedules Chapter-15

Header Information
NTN/FTN of importer Approvel No.
(1) (2)
Details of Input goods (to be filled by the authorized office of Goods imported (Collectorate of Import)
Regulatory Authority)

Date of CRN/Mach
Quantity imported
Custom Duty rate

Sales Tax Rate

CRN/Mach No.
Collectorate
(applicable)
Description

(applicable
HS Code

Quantity
Specs

WHT

UOM

No.
(3) (4) (5) (6) (7) (8) (9) (10) (11) (12) (13) (14)

CERTIFICATE Before certifying the above-authorized officer of the Regulatory Authority shall ensure that the goods are
genuine and bonafide requirement of the project and that the same are not manufactured locally.

Signature ________________________

Designation ________________________

NOTE:- In case of clearance through Pakistan Customs Computerized System, the above information shall be furnished on
line against a specific user I.D. and password obtained under section 155D of the Customs Act, 1969.]

726 Conceptual approach to Taxes


Eighth and Ninth Schedules Chapter-15

"NINTH SCHEDULE
[See sub-section (3B) of section 3]
TABLE
(1) (2) (3) (4) (5)
S. No. Description/Specification Sales tax on Sales tax Sales tax on supply (
of Goods import or local chargeable at the payable at time of
supply at the time time of registration supply by CMOs)
of import of IMEI number by
CMOs
1. Subscriber Identification - - Rs. 250
Module (SIM) Cards
2. A. Low Priced Cellular Rs. 300 Rs. 300 -
Mobile Phones or Satellite
Phones
i. All cameras: 2.0 mega-
pixels or less
ii. Screen size: 2.6 inches or
less
iii. Keypad
B. Medium Priced Rs. 500 Rs.500 -
Cellular Mobile
Phones or Satellite
Phones
i. One or two
cameras: between
2.1 to 10 mega-pixels
ii. Screen size:
between 2.6 inches
and 5.0 inches
iii. Micro-processor:
less than 2 GHZ
C. Smart Cellular Rs. 1,000 Rs. 1,000
Mobile Phones or
Satellite Phones
i.one or two cameras:
10 mega-pixels and
above
ii. Touch Screen: 5.0
inches and above
iii. 4GB or higher
memory
iv. Operating system
of the type of IOS,
Android V2.3,
Android Gingerbread
or higher, Windows 8
or Blackberry RIM
v. Micro-processor
2GHZ or higher, dual
core or quad core

Conceptual approach to Taxes 727


Eighth and Ninth Schedules Chapter-15

LIABILITY, PROCEDURE AND CONDITIONS

(i) In case of goods specified against S. No. 1 of the Table, the liability to charge, collect and pay sales tax shall be on
the Cellular Mobile Operator (CMO) at the time of supply. In case of the goods specified against S. No. 2, the liability
to pay sales tax at the time of import shall be on the importer and the liability to charge, collect and pay sales tax
payable on supplies shall be on Cellular Mobile Operator at the time of registering International Mobile Equipment
Identity (IMEI) number in his system.
(ii) The cellular Mobile Operators shall, if not already registered, obtain registration under the Sales Tax Act, 1990.
(iii) No IMEI shall be registered in his system by a Cellular Mobile Operator without charging and collecting the sales tax
as specified in the Table.
(iv) The Cellular Mobile Operator shall deposit, the sales tax so collected through his monthly tax return in the manner
prescribed in section 26 of the Sales Tax Act, 1990, and rules made there under.
(v) The Cellular Mobile Operator shall maintain proper records of all IMEI numbers registered for a period of six years,
and such records shall be produced for inspection, audit or verification, as and when required, by an authorized
officer of Inland Revenue.
(vi) The Pakistan Telecommunication Authority shall provide data regarding IMEI numbers registered with other Cellular
Mobile Operators to prevent double taxation on the same IMEI number in case of switching by a subscriber from one
operator to another, and to provide data regarding registration of IMEI numbers to the Board on monthly basis.

(via) The sales tax as indicated in column (3) of the table above shall be paid by the importer in case of imports and by the
manufacturer in case of locally manufactured cellular mobile phones.
(vii) No adjustment of input tax shall be admissible to the Cellular Mobile Operator or any purchaser of cellular mobile
phone against the sales tax charged and paid in terms of this Schedule.
(viii) The tax specified in column (4) of the Table shall be charged, collected and paid with effect from such date as may be
specified by the Board and the sales tax specified in column (3) shall stand withdrawn from the date so specified.
Note:- Notwithstanding anything contained in any other law for the time being in force, the levy, collection and payment of
sales tax under Notification No. S.R.O. 460(I)/2013 dated the 30th May, 2013, shall be always deemed to have been
lawfully and validly, levied, collected and paid.

728 Conceptual approach to Taxes


Capital Value Tax Chapter-01

Chapter

1 CAPITAL VALUE TAX

(For CA MOD F AND ICMAP students)


SCOPE OF CAPITAL VALUE TAX (CVT):
The tax on the capital value of assets is called the Capital Value Tax. This tax is payable on acquisition of an asset etc. by
every individual, Association of persons, Firm or Company.
WHEN THE CVT IS PAYABLE:
The CVT is payable on:

Acquisition on an asset;

Acquisition of a right to use an asset for more than 20 years or renewal of the lease or any premium paid thereon;
and

Acquisition of shares of a public company listed on a stock exchange in Pakistan.

Purchase of modaraba certificate or a registered instrument of redeemable capital as defined in the Companies
Ordinance, 1984
The acquisition of an asset or right to use asset may be by purchase; gift; exchange; power of attorney other than revocable
and time bound (not exceeding 60 days) executed between spouses, father and son or daughter, grand parents and grand
children, brother and sister or surrender or relinquishment of rights by owner.
The acquisition may be effected orally or by deed or obtained through court decree. The acquisition should not be by
inheritance, or gift from spouse, parents, grand parents, a brother and a sister. In case of a bank, the capital value tax shall
be paid when general power of attorney is used to sell the mortgaged property offered as collateral other than traded
security for obtaining loan.
Provided that in case of a bank, the capital value tax shall be paid when general power of attorney is used to sell the
mortgaged property offered as collateral other than traded security for obtaining a loan.
ADDITIONAL TAX:
Where any person fails to collect or having collected fails to pay the CVT, as required, he shall be personally liable to pay
the tax along with additional tax @ 15% per annum.
WHO WILL COLLECT CVT AND WHEN:
1. The person responsible for registering or attesting the transfer of an asset, in respect of which the tax is payable,
shall collect CVT at the time of registering or attesting the transfer.
2. In case of motor vehicle purchased from a manufacturer in Pakistan, the manufacturer shall collect CVT before
making the delivery of the vehicle.
3. In case of motor vehicle imported into Pakistan, the collector of customs, at the time of Customs clearance collect
CVT on the value of such vehicle as increased by customs duty, sales, income and any other charges payable before
removal of the vehicle from custom area.

Conceptual Approach to Taxes 729


Capital Value Tax Chapter-01

Description of Assets Rate of CVT (Percentage of the Capital Value)


1. IMMOVABLE PROPERTY
1.1 Residential immovable property, (other than
flats), situated in urban area, measuring at least of 500
square yards or one kanal (whichever is lower) and
more:
(i) Where the value of the property is recorded 2% of the recorded value or Rs.100 per square yard of the
landed area whichever is higher.
(ii) Where the value of the property is not recorded. Rs.100 per square feet of the landed area.
(iii) Where the property is a constructed property. Rs.10 per square feet of the constructed area in addition to
the value computed as above.
1.2 Residential flats of any size situated in an urban
area:
(i) Where the value of the property is recorded. 2 % of the recorded value or Rs. 100 per square feet of the
covered area whichever is higher.
(ii) where the value of the property is not recorded. Rs. 100 per square feet of the covered area.
1.3 Commercial Immovable property of any size situated
in an urban area :
(i) where the value of the property is recorded 2 % of the recorded value or Rs.100 per square yard of the
landed area whichever is higher.
(ii) where the value of the property is not recorded. Rs. 100 per square feet of the landed area
(iii) where the property is a constructed property. Rs. 10 per square feet of the constructed area in addition to
the value computed as above
2. OTHERS
2.1 (i) Purchase of modaraba certificate or 0.02% of the purchase value
(ii) Any instrument of redeemable capital as defined
in the companies ordinance, 1984
2.2 Purchase of shares of a shares of a public company 0.01% of the purchase value
listed on a stock exchange in Pakistan
2.3 motor vehicle not plying for hire
(i) Imported The landed cost as determined by the custom authorities,
(ii) Purchased from a manufacturer in Pakistan The price paid by the purchaser.
(iii) Others The value as declared by the transferee.

EXPLANATION.-" For the purposes of this section, the expressions


(a) "development authority" means an authority formed by or under any law for the purposes of development of an
area and includes any authority, society, agency, trust, association or institution declared as development authority by
the CBR by a NOG; and
(b) "registration authority" means the person responsible for registering or attesting the transfer of the asset or of the
right to use thereof for more than twenty years, and in the case of a development authority or a co-operative society,
its principal officer.
(c) "Urban Area" means area within the limits of-
i. the Islamabad Capital Territory;
ii. a Cantonment Board;
iii. the rating areas as defined under the Urban Immoveable Act, 1958 as in force in Punjab NWFP, Sindh and
Balochistan except where rate u/s 117 of the respective provincial local government ordinance is zero and
iv. such areas the FG of revenue may, from time to time, by Notification in the Official Gazettee (NOG), specify.

730 Conceptual Approach to Taxes


Capital Value Tax Chapter-01

OTHER PROVISIONS:
- The proceeds of the tax collected under CVT shall be credited to the Federal Consolidated Fund under the head
specified by the Federal Government.
- The Commissioner Inland Revenue, on an application by assesses may revise any order made for CVT.
- The CBR may, by Notification in the Official Gazette, make provisions relating to the collection and recovery of, any
other matter relating to the CVT.
- The Federal Government G may, by Notification in the Official Gazette, exempt, any person or class of persons or
asset or class of assets from the CVT subject to such conditions as may be specified in the notification.
Provided that with a view to facilitate to the process of privatization, the sale of assets of Kot Addu Power Station
shall be exempt from payment of CVT with effect from 27-06-1996.

Conceptual Approach to Taxes 731


Capital Value Tax Chapter-01

ICMAP PAST PAPERS THEORECTICAL QUESTIONS

Q.6(c) Spring 2014 The tax on the capital value of the assets is called the Capital Value Tax. Identify the situations where
capital value tax would be payable.

732 Conceptual Approach to Taxes


Definitions Chapter-01

Chapter

1 DEFINITIONS

(For CA MOD F AND ICMAP students)


1. Short title, extent and commencement.- (1) This Act may be called the Federal Excise Act, 2005, (2) It extends to
the whole of Pakistan (3) It shall come into force on 01-07-2005.

2. Definitions. - In this Act, unless there is anything repugnant in the subject or context,

(1) "adjudicating authority" means any authority competent to pass any order or decision under this Act or the
rules made there under, but does not include the Board or Appellate Tribunal;

(2) "adjustment" means deduction of amount of duty paid on goods used in the manufacture or production of
other goods from the amount of duty payable on such other goods in the prescribed manner;

(3) "Appellate Tribunal" means Appellate Tribunal Inland Revenue established under section 130 of the ITO,
2001;

(4) "Board" means the Federal Board of Revenue established under the Central Board of Revenue Act, 1924 (IV
of 1924) and on the commencement of the Federal Board of Revenue Act, 2007, the Federal Board of
Revenue established under section 3 thereof.

(4A) "Chief Commissioner" means a person appointed as Chief Commissioner Inland Revenue under section
29;"

(5) "Commissioner" means an person appointed as Commissioner Inland Revenue under section 29 of this Act;

(6) Commissioner" means any means of transport used for carrying goods or passengers such as vessel,
aircraft, vehicle or animal etc.:

(7) "default surcharge" means surcharge levied under section 8;

(8) "Distributor" means a person appointed by a manufacturer in or for a specified area to purchase goods from
him for sale to a wholesale dealer in that area;

(8a) "due date", in relation to furnishing a return under section 4, means the 15th day of the month following the
end of the month, or such other date as the Federal Government may, by notification in the official Gazette,
specify.

(8b) "dutiable goods" means all excisable goods specified in First Schedule except those which are exempt
u/s16;

(8c) "dutiable supply" means a supply of dutiable goods made by a manufacturer other than a supply of goods
which is exempt under section 16 of the Act;

(8d) "dutiable services" means all excisable services specified in the First Schedule except those which are
exempt under section 16 of the Act;

(9) "duty" means any sum payable under the provisions of this Act or the rules made there under and includes
the default surcharge and the duty chargeable at the rate of zero per cent;

(9a) "duty due" means duty in respect of supplies made or services provided or rendered during a month and shall
be paid at the time of filing of return;

(10) "establishment" includes an undertaking, firm or company, whether incorporated or not, an association of
persons and an individual:

manufactured, or wherein or in any part of which any manufacturing process connected with the production of
the part of which goods are

Conceptual Approach to Taxes 733


Definitions Chapter-01

(11) "factory" means any premises, including the precincts thereof, wherein or in any goods is being carried on
or is ordinarily carried on;

(12) "Officer of Inland Revenue" means any officer appointed by the Board as officer of Inland Revenue under
section 29 or any person including an officer of the Provincial Government invested by the Board with any of
the powers of an officer of Inland Revenue under this Act or rules made there under;

(12a) "franchise" means an authority given by a franchiser under which the franchisee is contractually or otherwise
granted any right to produce, manufacture, sell or trade in or do any other business activity in respect of goods
or to provide service or to undertake any process identified with franchiser against a fee or consideration
including royalty or technical fee, whether or not a trade mark, service mark, trade name, logo, brand name or
any such representation or symbol, as the case may be, is involved

(13) "goods" means goods leviable to excise duty under this Act or as specified in the First Schedule and includes
goods manufactured or produced in non-tariff area and brought for use or consumption to tariff area;

(14) "goods insurance" includes fire, marine, theft, accident and other such miscellaneous insurance;

(15) "import" and "export" mean respectively bringing into, and taking out of Pakistan by sea, land or air and
shall be deemed to have always been so defined;

(15a) "KIBOR" means Karachi Inter Bank Offered Rate prevalent on first day of each quarter of the financial year;"

(16) "manufacture" includes,-

(a) any process incidental or ancillary to the completion of a manufactured product;

(b) any process of re-manufacture, remaking, reconditioning or repair and the processes of packing or repacking
such product, and, in relation to tobacco, includes the preparation of cigarettes, cigars, cheroots, biris,
cigarette and pipe or hookah tobacco, chewing tobacco or snuff or preparation of unmanufactured tobacco
drying, cutting and thrashing of raw tobacco, and the word "manufacturer" shall be construed accordingly and
shall include,-

(i) any person who employs hired labour in the production or manufacture of goods; or

(ii) any person who engages in the production or manufacture of goods on his own account if such goods
are intended for sale; and

(iii) any person who, whether or not he carries out any process of manufacture himself or through his or any
other person, gets any process of manufacture carried out on his behalf by any person who is not in his
employment;

Provided that any person so dealing in goods shall be deemed to have manufactured for all purposes of this Act,
such goods in which he deals in any capacity whatever;

(16a) "non-fund banking services" includes all non-interest based services provided or rendered by the banking
companies or non-banking financial institutions against a consideration in the form of a fee or commission or
charges;

(17) "non-tariff area" means Azad Jammu and Kashmir, Northern Areas and such other territories or areas to
which this Act does not apply;

(18) "person" includes a company, an association, a body of individuals, whether incorporated or not, a public or
local authority, a Provincial Government or the Federal Government;

(19) "prescribed" means prescribed under this Act or by rules made there

(19a) "franchise" means an authority given by a franchiser under which the franchisee is contractually or otherwise
granted any right to produce, manufacture, sell or trade in or do any other business activity in respect of goods
or to provide service or to undertake any process identified with franchiser against a fee or

consideration including royalty or technical fee, whether or not a trade mark, service mark, trade name, logo,
brand name or any such representation or symbol, as the case may be, is involved;

(20) "registered person" means a person who is registered or is required to be registered under this Act provided
that a person who is not registered but is required to be registered shall not be entitled to any benefit or
privilege under this Act or rules made there under, unless he is registered and such benefit and privilege,
unless allowed by Board, shall be confined to period of registration;

734 Conceptual Approach to Taxes


Definitions Chapter-01

(21) "sale" and "purchase" with their grammatical variations and cognate expressions, mean any transfer of the

possession of goods or rendering and providing of services by one person to another in the ordinary course of
trade or business for cash or deferred payment or other consideration;

(21a) "sales tax mode" means the manner of collection and payment under the Sales Tax Act, 1990, and rules
made there under, of the duties of excise chargeable under this Act specified to be collected and paid as if
such duties were tax chargeable under section 3 of the said Act and all the provisions of that Act and rules,
notifications, orders and instructions made or issued there under shall, mutatis mutandis, apply to the excise
duty so chargeable; and

(22) "Schedule" means the schedule appended to this Act;

(23) "services" means services, facilities and utilities leviable to excise duty under this Act or as specified in the
First Schedule read with Chapter 98 of the Pakistan Customs Tariff, including the services , facilities and
utilities originating from Pakistan or its tariff area or terminating in Pakistan or its tariff area;

(23a) "supply" includes sale, lease or other disposition of goods and shall include such transaction as the Federal
Government may notify in the official Gazette from time to time;

(24) "tariff area" means area other than the non-tariff area;

(24A) "whistleblower" means whistleblower as defined in section 42D of the Federal Excise Act, 2005;

(25) "wholesale dealer" means a person who buys or sells goods wholesale for the purpose of trade or
manufacture, and includes a broker or commission agent who, in addition to making contracts for the sale or
purchase of goods for others, stocks such goods belonging to others as an agent for the purpose of sale; and

"Zero-rated" means duty of Federal excise levied and charged at the rate of zero per cent under section 5 of
this act.

Conceptual Approach to Taxes 735


Definitions Chapter-01

ICMAP LAST YEARS QUESTIONS FROM TAX YEAR 2003 TO 2014 ON


FEDERAL EXCISE ACT, 2005
Q.NO. 6 (b) March 2015 Define the following terms as per section (2) of the Federal Excise Act, 2005:
(i) Wholesale dealer.
(ii ) Distributor.

Q.NO.6(b) August 2014 Define the following terms as per section (2) of the Federal Excise Act, 2005:
(i) Franchise.
(ii) Property Developers or Promoters
Q. No. 6(b) Spring 2014 Define the following terms as per section (2) of the Federal Excise Act, 2005:
(i) Establishment
(ii) Non-Fund Banking Services
Q. No. 6(b) Spring 2013 Define the term sales tax mode as provided under section 2(21A) of Federal Excise Act,
2005. Under what circumstances federal excise duty is payable in sales tax mode as per section 7 of the
Federal Excise Act, 2005? Explain any three
Q. No. 7(a)(ii) February 2013 Define the following terms under the provisions of the Federal excise Act, 2005:
- Adjustment
- Duty due
Q.6 (c) APRIL 2012 In the light of the Federal Excise Act, 2005, define the following terms:
(i) Establishment
(ii) Conveyance
(iii) Import and export
6 (b) SUMMER-2009 Under the Federal Excise Act, 2005, explain the following:
(i) Basis of duty charged
(ii) Establishment and Franchise
6 (a) SUMMER-2008 Define the following terms as used in the Federal Excise Act, 2005:
(i) Factory
(ii) Sale and Purchase
6 (a) WINTER-2007 Define the following terms as used in Federal Excise Act, 2005
(i) Non-tariff area
(ii) Establishment
(iii) Import and export
6 (a) SUMMER-2007 What is meant by due date as per section 2(8a) of the Federal Excise Act, 2005?

8 (b) WINTER-2006 Define the following in terms of the Federal Excise Act, 2005
(i) Duty due section 2(9a)
(ii) Goods section 2(13)
(iii) Zero rated section 2(26)

736 Conceptual Approach to Taxes


Levy, Collection and Payment of Duty Chapter-02

Chapter

2 LEVY, COLLECTION AND


PAYMENT OF DUTY

Sr. No. Section Topic covered


For CAF-6 and ICMAP Students
1 3 Duties specified in the First Schedule to be levied
2 4 Filing of return and payment of duty etc.
3 5 Zero rate of duty and drawback of duty etc.
4 6 Adjustment of duties of excise
5 7 Application of the provisions of the Sales Tax Act, 1990
6 8 Default surcharge
9 Liability for payment of duty in the case of private companies or business enterprises or in sale of
7
business ownership
8 10 Applicable value and rate of duty
9 11 Collection of excess duty etc.
10 12 Determination of value for the purposes of duty
11 13 Registration
12 14 Recovery of unpaid duty or of erroneously refunded duty or arrears of duty, etc.
13 14A Short paid amounts recoverable
14 15 Application of the Customs Act, 1969 to Federal excise duties
15 16 Exemptions
16 17 Records
17 18 Invoices

(For CA MOD F AND ICMAP students)


1. Duties specified in the First Schedule to be levied (U/S 3)
(1) Subject to the provisions of this Act and rules made there under, there shall be levied and collected in such
manner as may be prescribed duties of excise on;
(a) goods produced or manufactured in Pakistan;
(b) goods imported into Pakistan;
(c) such goods as the FG may, by notification in the official Gazette, specify, as are produced or
manufactured in the non-tariff areas and are brought to the tariff areas for sale or consumption therein;
and
d) services provided in Pakistan including the services originated outside but rendered in Pakistan;
at the rate of 15% ad valorem except the goods and services specified in the First Schedule, which shall be
charged to FED as, and at the rates, set-forth therein.
(2) Duty in respect of goods imported into Pakistan shall be levied and collected in the same manner and at the
same time as if it were a duty of customs payable under the Customs Act, 1969 and the provisions of the said
Act including section 31A thereof shall apply.
(3) The Board may, by notification in the official Gazette, in lieu of levying and collecting under sub-section (1)
duties of excise on goods and services, as the case may be, levy and collect duties,

Conceptual Approach to Taxes 737


Levy, Collection and Payment of Duty Chapter-02

(a) on the production capacity of plants, machinery, undertakings, establishments or installations producing
manufacturing such goods; or
(b) on fixed basis, as it may deem fit, on any goods or class of goods or on any services or class of
services, payable by any establishment or undertaking producing or manufacturing such goods or
providing or rendering such services.
(3A) Subject to the provision of sub-section (3) of section 6 or any notification issued there under, where excisable
goods and services are supplied to a person who has not obtained registration number, the Federal
Government may, by notification in the official Gazette, charge, levy and collect, on the excisable goods and
services specified in that notification, a further duty at the rate of two per cent of the value in addition to the
rate specified in sub-sections (1), (3), (4) and (5) of this section.
(4) The FG may levy and collect duty on any class or classes of goods or services by notification in the official
Gazette at such higher or lower rate or rates as may be specified in such notification.
(5) The liability to pay duty shall be-
(a) in case of goods produced or manufactured in Pakistan, of the person manufacturing or producing such
goods;
(b) in case of goods imported into Pakistan, of the person importing such goods;
(c) in case of services, provided where services are rendered by the person out of Pakistan, the recipient
of such services in Pakistan shall be liable to pay Duty;
(d) in case of goods produced or manufactured in non-tariff areas and brought to tariff areas for sale or
consumption therein, of the person bringing or causing to bring such goods to tariff areas.
2. Filing of return and payment of duty etc. (U/S 4)
(1) For every month, a registered person shall furnish not later than the due date a true and correct return in such
manner and form as may be prescribed by the board by notification in the official Gazette.
(2) Duty due for the dutiable supplies made or services rendered during a month shall be deposited by the
registered person in the designated branch of the bank at the time of filing of his return as stated above:
Provided that FBR may, by notification in the official Gazette, prescribe any other manner of depositing the
duty.
(3) If during any month, there is a change in the rate of duty, a separate return showing the application of different
rates of duty shall be used in respect of each portion of such month.
(4) A registered person may, subject to approval of the CIR having jurisdiction, file a revised return within 120
days of the filing of return as above, to correct any omission or wrong declaration made therein.
(5) The FBR may, by notification in the official Gazette, require any person or class of person for any goods or
class of goods to furnish such summary or details of particulars pertaining to imports, purchases, utilization,
consumption, production, sales or disposal of such goods during any month or months in such formate and
manner as may be specified and provisions of this subsection shall apply equally in respect of services.
(6) The FBR may by an order, specify the manner and procedure for filing of return for the purpose of this Act or
rules made there under and for payment of duty by electronic means. The Board may specify the manner and
procedure for the submission, receipt and transmission of any information for the purpose of this Act or rules
made there under by electronic means.
(7) Every amount of duty due from any person on any other account shall also be deposited on the prescribed
return in the bank branch designated and in the same manner as aforesaid.
(8) The Board may, by rules made under this Act, prescribe a composite return.";
3. Zero rate of duty and drawback of duty etc. (U/S 5)
(1) Without considering the provisions of section 3, the goods exported out of Pakistan or such goods as may be,
by a notification in the official Gazette, specified by the FG shall be charged to duty at the rate of 0% and
adjustment of duty in terms of section 6 shall be admissible on such goods.
(2) The FBR may, by notification in the official Gazette, grant drawback of duty paid on any goods used in the
manufacture of any goods manufactured in and exported out of Pakistan, or shipped as provisions or stores
for consumption on board a ship or aircraft proceeding to a destination outside Pakistan, at such rate or rates
and subject to such conditions and limitations as may be specified in the notification.
(3) The Board may, by notification in the official Gazette, prohibit the payment of drawback, refund or adjustment
of duty upon the exportation of goods or any specified goods or class of goods to any specified foreign port or
territory.

738 Conceptual Approach to Taxes


Levy, Collection and Payment of Duty Chapter-02

4. Adjustment of duties of excise (U/S 6)


(1) For the purpose of determining net liability of duty in respect of any goods, the duty already paid on goods
specified in the First Schedule and used directly as input goods for the manufacture or production of such
goods shall be deducted from the amount of duty calculated on such goods.
(2) Adjustment of duty of excise as above shall be admissible only if a person registered under this Act holds a
valid proof to the effect that he has paid the price of goods purchased by him including the amount of duty and
received the price of goods sold by him including the amount of duty through banking channels including
online payment whether through credit card or otherwise.
The Board may, by a notification in the official Gazette, disallow or restrict whole or part of the amount of or
otherwise regulate the adjustment of duty in respect of any goods or class of goods.
5. Application of the provisions of the Sales Tax Act, 1990 (U/S 7)
In case of goods specified in the Second Schedule or such services as may be specified by the Board through a
notification in the official Gazette the duty shall be payable in sales tax mode, whereby
(1) (a) a registered person manufacturing or producing such goods or providing or rendering such services
shall be entitled to deduct input tax paid during the tax period from the amount of duty of excise due
from him on such goods or services in respect of that tax period;
(b) a registered person shall be entitled to deduct the amount of duty of excise paid or payable by him on
such goods or services as are acquired by him during a tax period from the output tax due from him in
respect of that tax period;
(c) a registered person supplying such goods or providing or rendering such services shall be entitled to
deduct duty of excise paid or payable on such goods or services as are acquired by him during the tax
period from the amount of duty of excise due from him on such goods manufactured or produced or
services as are provided or rendered by him during that period; and
(d) a person shall be entitled to deduct duty of excise paid or payable, on such goods or services as are
acquired by him during a month, from the amount of duty of excise due from him on such goods
manufactured or produced or services as are provided or rendered by him, during that month. Such
services as are provided or rendered by him, during that month.
(2) The FG may, by notification in the official Gazette, declare that any of the provisions of the STA, 1990, relating
to the levy of and exemption from sales tax, registration, book keeping and invoicing requirements, returns,
offences and penalties, appeals and recovery of arrears shall, with such modifications and alterations as it
may consider necessary or desirable to adapt them to the circumstances, be applicable in regard to like
matters in respect of the duty leviable under this Act.";
Explanation. For the purposes of this section, the expressions "input tax", "output tax" and "tax period" shall
have the same meanings as are assigned to them in the Sales Tax Act, 1990
6. Default surcharge (U/S 8)
If a person does not pay the duty due or any part thereof within the prescribed time or receives a refund of duty or
drawback or makes an adjustment which is not admissible to him, he shall, in addition to the duty due, pay default
surcharge at the rate of "KIBOR plus 3% p. a." of the duty due, refund of duty or drawback.
Explanation.-For the purpose of this section,
(a) the period of default shall be reckoned from the date following the due date on which the duty was payable to
the preceding day on which the duty is actually paid; and
(b) in case of inadmissible adjustment or refund of duty or drawback, the period of default shall be reckoned from
the date of such adjustment or as the case may be, refund of duty or drawback is received.";
7. Liability for payment of duty in the case of private companies or business enterprises or in sale of business
ownership (U/S 9)
(1) Without considering anything contained in any other law for the time being in force, where any private
company or business enterprise is closed or discontinued or otherwise ceases to exist and any amount of duty
chargeable on the company or business enterprise, whether before, or in the course of, or after its liquidation
cannot be recovered from the company or business enterprise, every person who was a owner of, or partner
in, or director of, the company or business enterprise shall, jointly and severally with such person, be liable for
the payment of such duty.
(2) In the case of sale or transfer of ownership of a business or part thereof involving any charge of duty to
another person as an ongoing concern, the chargeable duty shall be paid by the person to whom such sale is
made or ownership is transferred provided that if any amount of duty payable by such person remain unpaid
amount of duty shall be the first charge on the assets of the business and shall be payable by the transferee of
business.

Conceptual Approach to Taxes 739


Levy, Collection and Payment of Duty Chapter-02

Provided that no business enterprise or a part thereof shall be sold or transferred unless the outstanding duty
is paid and a NOC in this behalf from the Commissioner concerned is obtained.
(3) In case of termination of a business or part thereof involving any outstanding charge of duty, a person
terminating such business or part thereof shall be required to account for and pay the outstanding charge of
duty as if no such termination has taken place.
8. Applicable value and rate of duty (U/S 10)
The value and the rate of duty applicable to any goods or services shall be the value, retail price, tariff value and the
rate of duty in force,-.
(a) in the case of goods, on the date on which the goods are supplies for export or for home consumption;
(b) in the case of services, on the date on which the services are provided or rendered; and
(c) in the case of goods produced or manufactured outside the areas to which this Act has been applied and
brought to such areas for sale or consumption therein, the date on which the goods are brought to the areas.
9. Collection of excess duty etc (U/S 11)
Every person who for any reason whatever has collected or collects any duty, which is not payable as duty or which is
in excess of the duty actually payable and the incidence of which has been passed on to the consumer, shall pay the
amount so collected to the FG and all the provisions of this Act or rules made there under shall apply for the recovery
of such amount and claim for refund of any such amount paid or recovered shall not be admissible on any ground
whatever.
10. Determination of value for the purposes of duty (U/S 12)
(1) Where any goods are liable to duty under this Act at a rate dependent on their value, duty shall be assessed
and paid on the basis of value as determined in accordance with section 2(46) of the STA, 1990, excluding the
amount of duty payable thereon."
(2) Where any services are liable to duty under this Act at a rate dependent on the duty shall be paid on total
amount of charges for the services including the ancillary facilities or utilities, if any irrespective whether such
services have been rendered or provided on payment of charge or free of charge or on any confessional basis.
(3) Where any goods are chargeable to duty at the import stage, duty will be assessed and paid on the value
determined in accordance u/s of the Customs Act, 1969 including custom duties payable thereon.
(4) Where any good is chargeable to a duty on the basis of retail price, duty thereon shall retail price fixed by the
manufacturer, inclusive of all duties, charges and taxes, other than sales collected u/s 3 of the STA, 1990, at
which any particular brand or variety should be sold to the general body of consumers or, if more than one
such price is so fixed for the variety, the highest of such price and such retail price shall, unless otherwise
directed by the Board, be legibly, prominently and indelibly indicated on each good, packet, container,
package, cover or label of such goods:
Provided that where so and as specified by the Board, any goods or class of goods liable to duty on local
production as percentage of retail price, the provisions of this sub-section shall equally in case such goods are
imported from abroad:
Provided that the Board may through a general order specify zones or areas only for the purpose of
determination of highest retail price of any brand or variety of goods.
(5) The Board may fix the minimum price of any goods or class of goods, for the purpose of levying and collecting
of duty and duty on such goods shall be paid accordingly:
Provided that, where the price at which the goods or class of goods are sold, is higher than the price fixed by
the Board, the duty shall, unless otherwise directed by the Board, be levied and collected at such higher price.
11. Registration (U/S 13)
(1) Any person engaged in the production or manufacture of goods or providing or rendering services liable to
duty of excise under this Act shall, unless otherwise specified, be required to obtain registration in the
prescribed manner regardless of his annual turnover or volume of sales of such goods or services.
(2) Where a person who is already registered under the STA, 1990, shall not be required to take separate
registration for excise purpose and his sales tax registration shall be deemed to be a registration for the
purpose of this Act:
Provided that provisions of the STA, 1990, including those relating to exemption threshold shall not apply
where a person obtains or is liable to obtain registration for the purposes of this Act but does not have or is not
liable to registration under the STA, 1990.

740 Conceptual Approach to Taxes


Levy, Collection and Payment of Duty Chapter-02

12. Recovery of unpaid duty or of erroneously refunded duty or arrears of duty, etc. (U/S 14)
(1) Where any person has not levied or paid any duty or has short levied or short paid such duty or where any
amount of duty has been refunded erroneously, such person shall be serviced with notice requiring him to
show cause for payment of such duty provided that such notice shall be issued within 5 years from the
relevant date.
(2) The OIR, empowered in this behalf, shall after considering the objections of the person served with a notice to
show cause as above, determine the amount of duty payable by him and such person shall pay the amount so
determined along with default surcharge and penalty as specified by such officer under the provisions of this
Act;
Provided that an order under this section shall be made within 120 days of issuance of show cause notice or
within such extended period as the CIR may, for reasons to be recorded in writing, fix, provided that such
extended period shall in no case exceed 60 days:
Provided further that any period during which the proceedings are adjourned on account of a stay order or;
ADR proceedings or the time taken through adjournment by the petitioner not exceeding 30 days shall be
excluded from the computation of the periods specified in the first proviso.
(3) Where any amount of duty levied and penalty imposed or any other amount payable under this Act is due from
any person, such amount or sum shall be recovered in such manner as is prescribed under this Act or rules
made there under.
(4) Notwithstanding anything contained under any other law for the time being in force, where any business or
activity involving liability to charge, levy and pay duty under this Act is sold, discontinued or liquidated, the
amount of unpaid or recoverable duty shall be the first charge on the assets of the business.
Explanation: For the purpose of this section, refund includes drawback of duty and] the expression "relevant
date" means the date on which the payment of duty was due as above and in case where any amount of duty
has been erroneously refunded, the date of its refund.
13. Short paid amounts recoverable (U/S 14A)
Notwithstanding the provisions of this Act or the rules made there under, here a registered person pays the amount of
duty less than the duty due as indicated in his return, the short paid amount of duty along with default surcharge shall
be recovered from such person by stopping removal of any goods from his business premises and through
attachment of his business bank accounts without prejudice to any other action under this Act or the rules made there
under:
Provided that no penalty under this Act or rules made there under shall be imposed unless a show cause notice is
given to such person.
14. Application of the Customs Act, 1969 to Federal excise duties (U/S 15)
The FG may, by notification in the official Gazette, declare that any or all of the provisions of the Customs Act, 1969,
shall, with such modifications and alterations it may specify, consider necessary or desirable to adapt them to the
circumstances, be applicable in regard to like matters in respect of the duties levied by section 3 and 8.
15. Exemptions (U/S 16)
(1) All goods imported, produced or manufactured in Pakistan and services provided or rendered except such
goods and services as are specified in the First Schedule shall be exempt from whole of excise duties levied
under section 3:
Provided that goods and services specified in the Third Schedule shall be exempt from duty subject to such
conditions and restrictions, if any, specified therein and no adjustment in terms of section 6 shall be admissible
in respect of goods exempt from duty of excise whether conditionally or otherwise.
(2) The FG may pursuant to the approval of the Economic Coordination Committee of Cabinet, whenever
circumstances exist to take immediate action for the purposes of national security, natural disaster,
national food security in emergency situations, protection of national economic interests in situations
arising out of abnormal fluctuation in international commodity prices, removal of anomalies in duties,
development of backward areas and implementation of bilateral and multilateral agreements;
(3) Notwithstanding the provisions as above, the FG or the Board may, by a notification in the official Gazette, for
reasons to be recorded, exempt any person or class of persons from payment of the whole or part of the
default surcharge imposed u/s 8 and penalties subject to the conditions or limitations as may be specified in
such notification.
(4) The Federal Government shall place before the National Assembly all notifications issued under this
section in a financial year.

Conceptual Approach to Taxes 741


Levy, Collection and Payment of Duty Chapter-02

(5) Any notification issued, shall, if not earlier rescinded, stand rescinded on the expiry of the financial year in
which it was issued.
16. Records (U/S 17)
(1) Every person registered for the purposes of this Act shall maintain and keep for a period of 6 years "or till such
further period the final decision in any proceedings including proceedings for assessment, appeal, revision,
reference, petition and any proceedings before an ADRC finalized," at his business premises or registered
office in English or Urdu language the following records of excisable goods purchased, manufactured and
cleared (including those cleared without payment of excise duty) by him or by his agent acting on his behalf in
such form and manner as would permit ready ascertainment liability of duty, namely:
(a) records of clearances and sales made indicating the description, quantity and value of goods, and
address of the person to whom sales were made and the amount of the duty charged;
(b) records of goods purchased showing the description, quantity and value of goods, name, address and
registration number of the supplier and the amount of the duty, if any, on purchases;
(c) records of goods cleared and sold without payment of duty;
(d) records of invoices, bills, accounts, agreements, contracts, orders and other allied business matter
(da) record relating to gate passes, inward or outward, and transport receipts;
(e) records of production, stocks and inventory;
(f) records of imports and exports; and
(g) such other records as may be specified by the Board.
(2) For any person or class of persons registered under this Act, or for any goods or class of goods the board may
specify or prescribe;
(a) to keep any other records for the purposes of this Act;
(b) to use such electronic fiscal cash registers as may be approved by the Board; and
(c) the procedure or software for electronic maintenance of records and filing of statements, documents or
information by any person or class of persons,
(3) Above provisions shall apply mutatis mutandis on services provided or render person registered under this
Act.
17. Invoices (U/S 18)
(1) A person registered under this Act shall issue for each transaction a serially numbered at the time of clearance
or sale of goods, including goods chargeable to duty at the rate of zero percent, or providing or rendering
services containing the following particulars, namely:
I. name, address and registration number of the seller;
II. name, address and registration number of the buyer; date of issue of the invoice;
III. description and quantity of goods or as the case may be, description of services;
IV. value exclusive of excise duty;
V. amount of excise duty; and
VI. value inclusive of excise duty.
(2) Where a registered person is also engaged in making supplies taxable under the STA, 1990, such person
shall not be required to issue a separate invoice for excise purposes and the amount of excise duty and other
related information may in such cases be mentioned on the invoice issued for sales tax purposes.
(3) The Board may, by notification in the official Gazette, specify such modified invoices for different persons or
classes of persons as it may deem necessary.
(4) The Board may, by notification in the official Gazette, specify goods in respect of which a copy of the invoice
shall be carried or accompanied with the conveyance during their transportation or movement in such manner
and subject to such conditions as may be specified in this behalf either in such notification or otherwise.
(5) The Board may, by notification in the official Gazette, specify the goods or services in respect of which sales
invoice shall be issued electronically and prescribe the manner and procedure therein.

742 Conceptual Approach to Taxes


Levy, Collection and Payment of Duty Chapter-02

ICMAP LAST YEARS QUESTIONS FROM TAX YEAR 2003 TO 2012 ON


FEDERAL EXCISE ACT, 2005
Q. No. 7(a) (i) February 2013 A person registered under the Federal Excise Act, 2005 is required to issue invoice for each
transaction at the time of clearance of goods, List down the particulars of the said invoice as per section 18(1) of the Federal
Excises Act, 2005.
Q.6 (a) August 2012 Alpha Private Limited (APL) wanted to adjust the Federal Excise duty paid on the raw materials
purchased against the account of duty required to pay on its finished products.
Under sect ion 6 of the Federal Excise Act, 2005 explain the conditions which Alpha Private Limited is required to fulfil for
the adjustments of duty?
Q.6 (a) SUMMER 2012 Section 3(1) of the Federal Excise Act, 2005 enumerates the types of goods and services on
which duties of excise specified in the First Schedule are levied. What are those goods and services?
4 (b) SUMMER-2010 Define the term invoice and its contents under the Federal Excise Act, 2005.
6 (b) WINTER-2009 How would Federal Excise Officer proceed for recovery of arrears of duty or any other sum under the
Federal Excise Act. 2005?
6 (a) WINTER-2008
(i) Explain the provision relating to Special Excise Duty with reference to section 3A of the Federal Excise Act, 2005.
(ii) Discuss Default Surcharge with reference to section 8 of the Federal Excise Act, 2005.
6 (b) SUMMER-2007 Explain the provisions of Section 8 of the Federal Excise Act, 2005, for levy of default surcharge.
3 (b) WINTER-2005 Explain adjustment of Excise duty as has been provided in Federal Excise Act, 2005.

Conceptual Approach to Taxes 743


Levy, Collection and Payment of Duty Chapter-02

744 Conceptual Approach to Taxes


Offences and Penalties Chapter-03

Chapter

3 OFFENCES AND PENALTIES

Sr. No. Section Topic covered


For CAF-6 and ICMAP Students
1 19 Offences, penalties, fines and allied matters
2 33 Appeals to Commissioner (Appeals)
3 34 Appeals to the Appellate Tribunal and Reference to High Court
4 34A Reference to High Court
5 35 Powers of Board or Commissioner to pass certain orders
6 36 Power to rectify mistakes in orders
7 37 Deposit, pending appeal, of duty demanded or penalty levied
8 38 Alternative dispute resolution
9 39 Exclusion of time taken for copy

(For CA MOD F AND ICMAP students)

1. Offences, penalties, fines and allied matters (U/S 19)


(1) Any person who fails to file or files an incorrect return within the period specified in sub-section (1) of section 4
or fails to make payment or makes short payment of duty on any account, shall pay a penalty of Rs.5,000 in
case of non-filing of return and Rs.10,000 or 5% of the duty involved whichever is higher in case of short
payment of duty due in addition to the amount due from him and without prejudice to other liabilities which may
be determined against him or action which may be taken against him under this Act and rules made there
under;
Provided that where a person files the return within 15 days after the due date he shall pay a penalty of Rs.100
for each day of default"; and
(2) Any person who:
(a) makes, orally or in writing, or signs any declaration, certificate or other document required by this Act or
rules made there under or by any officer of Inland Revenue (OIR) to do so, which is untrue or incorrect
in any particular or which is incomplete by omitting any material particular there from;
(b) counterfeits or falsifies, or uses, when counterfeited or falsified, any document which is or may be
required under this Act or rules made there under or any document used in the transaction of any
business or
(c) matter relevant to this Act or rules made there under; and
fails or refuses to give or produce to the OIR any information or document required to be given or
produced under this Act or rules made there under;
shall be guilty of an offence and for every such offence shall be liable to fine which may extend to
twenty thousand rupees and in case of offence under clause (b), the fine may extend to one hundred
thousand rupees
and he shall be punishable with imprisonment for a term which may extend to five years or with both.

Conceptual Approach to Taxes 745


Offences and Penalties Chapter-03

(2) Any person who,


(a) illegally removes, stores, keeps, or withdraws or in any way assists or is concerned in the illegal
'removal or withdrawal of any goods in the manner other than the manner prescribed under this Act or
rules made there under;
(b) is in any way concerned in conveying, removing, depositing or dealing with any goods with intent to
defraud the Government of any duty of excise due thereon, or to violate any of the provisions of this Act
or rule made there under;
(c) is in any way concerned in any fraudulent evasion or attempt at fraudulent evasion of excise;
(d) claims, takes or avails adjustment of duty not admissible under this Act or the rules made and
(e) is in any way concerned in the manufacture of any dutiable goods in contravention of the this Act or
rules made there under;
shall be guilty of an offence and for each such offence, shall be liable to fine which may extend to
Rs.50,000 or five times of the duty involved, whichever is higher and to punishment with imprisonment
which may extend to five years or both.
(3) Any person who, without the approval of the Commissioner, directly or otherwise destroy erases or otherwise
manipulates data stored in or used in connection with a computer or otherwise uses a computer, the purpose
or effect of which is to reduce, avoid or evade any liability to duty of excise
Which would otherwise have been imposed by this Act, or to defeat any provisions of this Act or rules made
there guilty of an offence and shall be liable to fine which may extend to Rs.75,000 or ten times of the duty
involved, whichever is higher and to punishment with imprisonment which may extend to five years or both.
(4) If any person obtains, attempts to obtain or abets in obtaining, or does anything whereby t there might be
obtained by another person, any amount by way of refund or drawback of any duty in respect of services not
lawfully payable or allowable in respect thereof or which is greater than the amount allowable, he shall be
guilty of an offence and shall be liable to fine which may extend to Rs.100 or five times of duty involved,
whichever is higher and to punishment with imprisonment which may extend to five years or both.
(5) Any person who obstructs any OIR or other public servant or any person acting in his aid or assistance, or duly
employed for the prevention of offences under this Act or rules made there under in the execution of his duty
or in the due seizing of any goods liable to seizure under this Act or rules made there under, shall be guilty of
an offence and for each such offence, shall be liable to fine which may extend to Rs.50,000 or five times of
duty involved, whichever is higher and to punishment which may extend to 3 years or both.
(6) Where an offence under this Act or rules made there under has been committed by a company, firm, or other
body of persons, any person who at the time of the commission of the offence was a director, manager, or
other similar officer or a partner of the company, firm, or other body of persons or was purporting to act in that
capacity shall be deemed to be guilty of that offence unless he proves that the offence was committed without
his consent or connivance and that he exercised all such diligence to prevent the commission of the offence as
he ought to have exercised, having regard to the nature of his functions in that capacity and to all the
circumstances.
(7) Unless otherwise legally proved, where any person is liable under this Act to any penalty or punishment for
any act, omission, neglect or default, he shall be liable to the same punishment, penalty or forfeiture for every
such act, omission, neglect or default of any agent or employee.
(8) Where any goods are chargeable to duty on the basis of retail price under this Act and the retail price is not
indicated on the goods "and in case of cigarettes, retail price, health warning and name of the manufacturer is
not mentioned" in the manner specified therein or in the rules made there under, the duty shall be charged at
the rate of 50% ad valorem in case of cigarettes, and 40% ad valorem in case of goods other then cigarettes:
Provided that in cases where a registered person does not intend to print retail price for any genuine reasons,
he may voluntarily pay duty on the said higher rates as applicable to him and declare his duty payments in the
return accordingly and other provisions of this section shall not apply in such cases.
(9) Where any person is engaged in the manufacture or production of cigarettes in the manner contrary to this Act
or rules made there under or otherwise evades duty of excise on cigarettes or is engaged in the manufacture
or production of counterfeited cigarettes, the machinery, equipments, instruments or devices used in such
manufacture or production shall, after outright confiscation, be destroyed in such manner as may be approved
by the Commissioner and no person shall be entitled to any claim on any ground whatsoever, or be otherwise
entitled to any compensation in respect of such machinery or equipments, instruments or devices and such
confiscation or destruction shall be without prejudice to any other penal action which may be taken under the
law against the person or in respect of the cigarettes or vehicles involved in or otherwise linked or connected
with the case.

746 Conceptual Approach to Taxes


Offences and Penalties Chapter-03

(10) Any goods in respect of which any of the provisions of this Act or rules made or notifications issued there
under has been contravened shall be liable to confiscation along with the conveyance, if any, in which such
goods are laden or have been or being carried and all confiscations in this regard shall vest with the Federal
Government.
(11) Any person who attempts to commit any offence punishable under this Act, or abets the commission of the
offence, shall be liable to the punishment provided for the offence.
2. Appeals to Commissioner (Appeals) (U/S 33)
(1) Any person other than OIR aggrieved by any decision or order passed under this Act or the rules made there
under by OIR up to the rank of Additional CIR, other than a decision or order or notice given or action taken for
recovery of the arrears of duty under this Act or rules made there under may within 30 days of receipt of such
decision or order prefer appeal there from to the CIR(Appeals).
(1A) Where in a particular case, the Commissioner (Appeals) is of the opinion that the recovery of tax levied under
this Act, shall cause undue hardship to the taxpayer, he, after affording opportunity of being heard to the
Commissioner or officer of Inland Revenue against whose order appeal has been made, may stay the
recovery of such tax for a period not exceeding thirty days in aggregate.
(2) The CIR(Appeals) may, after giving both parties to the appeal an opportunity of being heard, pass such order
as he thinks fit, confirming, varying, altering, setting aside or annulling the decision or order appealed against:
"Provided that such order shall be passed not later than 120 days from the date of filing of appeal or within
such extended period, not exceeding 60 days, as the Collector (Appeals) may, for reasons to be recorded in
writing, extend.": "Provided further that any period during which the proceeding are adjourned on account of
stay order or ADR proceedings or the time taken through adjournment by the petitioner not exceeding 30 days
shall be excluded for the computation of these period."
(3) In deciding an appeal, the CIR (Appeals) may make such further inquiry as may be necessary provided that he
shall not remand the case for de novo consideration.
3. Appeals to the Appellate Tribunal and Reference to High Court (U/S 34)
(1) Any person or OIR aggrieved by any of the following orders may within 60 days of the receipt of such orders
file appeal to the ATIR against such orders,-
(a) an order passed by the CIR(Appeals); and
(b) an order passed by the Board or the CIR u/s 35:
Provided that the ATIR shall decide the appeal filed under this sub-section within 6 months of its filing of
appeal. Officer of lower rank than the CIR, and the reference or appeal is pending before appeal forum or the
Court, such reference or appeal shall be deemed to have been filed and shall be taken to have been always so
filed by the Commissioner.
(2A) The Appellate Tribunal may admit, hear and dispose of the appeal in accordance with procedure laid down in
section 131 and 132 of Income Tax Ordinance 2001 and rules made there under.
(2B) AII Appeals and proceedings under this act pending before the Customs, Excise and Sales Tax Appellate
Tribunal constituted under section 194 of the Customs Act, 1969, shall stand transferred to the Appellate
Tribunal constituted under section 130 of Income Tax Ordinance, 2001 with effect from the date of
promulgation of the Finance (amendment) Ordinance, 2009.
4. Reference to High Court (U/S 34A)
(1) Within ninety days of the communication of the order of the Appellate Tribunal under sub-section (2A) of
section 34, the aggrieved person or the Commissioner may prefer an application, in the prescribed form along
with a statement of the case, to the High Court, stating any question of law arising out of such order.
(2) The statement to the High Court (HC) referred to in sub-section (1), shall set out the facts, the determination of
the Appellate Tribunal and the question of law which arises out of its order.
(3) Where, on an application made under sub-section (1), the HC is satisfied that a question of law arises out of the
order referred to in sub-section (1), it may proceed to hear the case.
(4) A reference to the HC under this section shall be heard by a Bench of not less than two judges of the HC and
in respect of the reference, the provisions of section 98 of the Code of Civil Procedure, 1908 shall apply.
(5) The HC upon hearing a reference under this section shall decide the question of law raised by the reference
and pass judgment thereon specifying the grounds on which such judgment is based and the Tribunal's order
shall stand modified accordingly. The Court shall send a copy of the judgment under the seal of the Court to
the Appellate Tribunal.

Conceptual Approach to Taxes 747


Offences and Penalties Chapter-03

(6) Notwithstanding that a reference has been made to the HC, the tax shall be payable in accordance with the
order of the Appellate Tribunal.
(7) Where recovery of tax has been stayed by the HC by an order, such order shall cease to have effect on the
expiration of a period of 6 months following the day on which it was made unless the appeal is decided or such
order is withdrawn by the HC earlier.
(8) Section 5 of the Limitation Act, 1908 shall apply to an application made to the HC under sub-section (1).
(9) An application under sub-section (1) by a person other than the Commissioner shall be accompanied by a fee
of Rs.100.
5. Powers of Board or Commissioner to pass certain orders (U/S 35)
(1) The Board or the Commissioner within his jurisdiction, may suo moto, or otherwise call for and examine the
records of any proceedings under this Act for the purpose of satisfying itself or, as the case may be, himself as
to the legality or propriety of any decision or order passed by a subordinate officer and may pass such order
as it or he may think fit.
(2) No order confiscating goods of greater value or enhancing any fine, or imposing or enhancing any penalty, or
requiring payment of any duty not levied or short-levied shall be passed under sub-section (1) unless the
person affected thereby has been given an opportunity of showing cause against it and of being heard in
person or through a counsel or other person duly authorized by him.
(3) No record of any proceedings relating to any decision or order passed by any officer of Inland Revenue shall
be called for or examined under subsection (1) after the expiry of two years from the date of such decision or
order.
Explanation.- For the purpose of sections 35, 45 and 46 and for removal of doubt, it is declared that the
powers of the Board, Commissioner or officer of Inland Revenue under these sections are independent of the
powers of the Board under section 42B and nothing contained in section 42B restricts the powers of the Board,
Commissioner or officer of Inland Revenue under these sections or to conduct audit under these sections.
6. Power to rectify mistakes in orders (U/S 36)
The Federal Government, the Board or any OIR may rectify any mistake which is apparent from the record in any
order passed by it or him under any of provisions of this Act or the rules made there under, on its or his own motion or
on an application made by a person affected by the order within 3 years of the passing of such order provided that no
such rectification which has the effect of enhancing any penalty or fine or requiring the payment of a greater amount
of duty shall be made unless the person affected by the proposed rectification has been given an opportunity of being
heard.
7. Deposit, pending appeal, of duty demanded or penalty levied (U/S 37)
(1) Where in any appeal, the decision or order appealed against relates to any duty demanded or penalty imposed
under this Act, the person desirous of appealing against such decision or order shall, pending appeal, deposit
the duty demanded or the penalty imposed provided that the Appellate Tribunal or Commissioner (Appeals)
may in any particular case dispense with such deposit subject to such conditions as it may deem fit to impose
so as to safeguard the interest of revenue.
(2) The order for such dispensation under sub-section (1) shall cease to have effect on the expiration a period of 6
months following the date on which order for dispensation was passed or until the order of dispensation is
withdrawn earlier or the case is finally decided earlier by the Appellate Tribunal or Commissioner (Appeals).
(3) Notwithstanding sub-sections (1) and (2), the Appellate Tribunal or Commissioner (Appeals) may direct that,
pending decision on the appeal, the duty demanded or penalty imposed, along with the default surcharge
payable under this Act, be paid by the appellant in suitable installments spread over a period not exceeding six
months from the date of such direction:
Provided that where a person has, at the time of filing appeal, deposited fifteen per cent of the liability covered
under the decision or order appealed against, he shall not be required to separately seek stay against recovery
and stay in such a case shall commence from the date of payment of such 15% amount and shall remain valid
till the expiry of a period of 6 months or till the decision of the appeal, whichever is earlier unless the case is
decided in his favour and the amount so paid is claimed to have become due for refund.
8. Alternative dispute resolution (U/S 38)
(1) Notwithstanding any other provisions of this Act, or the rules made there under, any registered person
aggrieved in connection with any dispute pertaining to;
(a) the liability of excise duty against the registered person or, as the case may be, admissibility of refunds;
(b) the extent of waiver of default surcharge and penalty; (c) the confiscation of goods;

748 Conceptual Approach to Taxes


Offences and Penalties Chapter-03

(d) relaxation of any procedural or technical irregularities and condonation of any prescribed time limitation;
and
(e) any other specific relief required to resolve the dispute, may apply to the Board for the appointment of a
committee for the resolution of any hardship or dispute mentioned in detail in the application and if
dispute is under litigation in any Court of law or an Appellate authority, except in the cases where first
information reports ( F.I.R's ) have been lodged under this Act or criminal proceedings initiated or where
interpretation of question of law having larger revenue impact in the opinion of the Board is involved,
may apply to the Board for the appointment of a Committee for the resolution of dispute in appeal and
only such application may be entertained for dispute resolution under the provisions of this section.
(2) The Board may, after examination of the application of a registered person, appoint a committee within 30
days of receipt of such application in the Board, consisting of an OIR, not below the rank of an Additional
Commissioner and two persons from the notified panel consisting of retired Judges not below District and
Sessions Judge, chartered or cost accountants, advocates, representatives of trade bodies or associations, or
any other reputable taxpayers, for the resolution of dispute.
(3) The committee constituted under sub-section (2) shall examine the issue and may if it deems fit, conduct
inquiry, seek expert opinion, direct any OIR or any other person to conduct an audit and shall make
recommendations within 90 days of its constitution in respect of the dispute. If the committee fails to make
recommendations within the said period the Board shall dissolve the committee and constitute a new
committee which shall decide the matter within a further period of 90 days. If after the expiry of the period the
dispute is not resolved the matter shall be taken up by the appropriate forum for decision."
(4) The Board may, on the recommendation of the Committee, pass such order, as it may deem appropriate.
(4A) Notwithstanding anything contained in sub-section (4), the Chairman, FBR, and a member nominated by him,
may, on the application of an aggrieved person, for reasons to be recorded in writing and on being satisfied
that there is an error in order or decision, pass such order as may be deemed just and equitable.";
(5) The registered Board may, make payment of duty and other taxes as determined by the Board in its order
under sub-section (4), and such order of the Board shall be submitted before the forum, tribunal or the Court
where the matter is sub-judice, for consideration and orders as deemed appropriate.
(6) The Board may, by notification in the official Gazette, make rules for carrying out the purposes of this section."
9. Exclusion of time taken for copy (U/S 39)
In computing the period of limitation specified for an appeal or application the day on which the order complained was
served, and if the party preferring the appeal or making the application was not furnished with a copy of the order
when the notice of the order was served upon him, the time requisite for obtaining a copy of such order shall be
excluded.

Conceptual Approach to Taxes 749


Offences and Penalties Chapter-03

750 Conceptual Approach to Taxes


Federal Excise Rules, 2005 Chapter-04

Chapter

4 FEDERAL EXCISE RULES, 2005

Sr. No. Rule Topic covered


For CAF-6 and ICMAP Students
1 7 Liability of duty
2 8 No refund of duty erroneously levied or paid, unless claimed within one year
3 9 Payment of duty not to be postponed
4 10 No default surcharge for holidays
5 15 Declaration of business premises and equipment
6 16 Storage of excisable goods
7 17 Daily account of production, clearance and balances of excisable goods
8 32 Drawback of duty paid on goods exported
9 33 Drawback of duty on exported goods which are made from excisable goods
10 34 Pecuniary competence to sanction drawback or refund

(For CA MOD F AND ICMAP students)


Assessment and collection of duty
1. Liability of duty (U/R 7)
Every person who produces or manufactures any excisable goods or provides or renders any excisable services shall
pay duty due on such goods or services on such date and in such manner as is prescribed under the Act or these
rules provided that the duty in respect of goods imported into Pakistan shall be charged and collected in the same
manner and at the same time as if it were a duty of customs payable under the customs Act, 1969 (IV of 1969).
2. No refund of duty erroneously levied or paid, unless claimed within one year (U/R 8)
No duty or part thereof which has been paid or overpaid through inadvertence, error or misconstruction, shall be
refunded unless a Written claim is lodged to the Collector within one year from the date of such payment.
3. Payment of duty not to be postponed (U/R 9)
"Under no circumstances whatsoever, any registered person shall on his own or otherwise defer or postpone the
payment of duty on the pretext or ground that he has not received the price inclusive of duty or the amount of duty
from a person to whom he has sold excisable goods or rendered or provided excisable services.
4. No default surcharge for holidays (U/R 10)
In situations where any amount of duty due to be paid on a given day is not posited owing to holiday, no default
surcharge shall be paid or required to be paid if such amount of duty is paid on the next working day.
Manufacturing of goods and removal thereof
5. Declaration of business premises and equipment (U/R 15)
Every manufacturer or service provider in respect of dutiable goods or services shall, at the time of applying for
registration, declare in writing all details of his business premises including godowns along with the name, address,
copy of National Identity Card, details of plant, machinery, raw materials, dutiable or other goods to be produced or
dutiable or other services to be provided and any change in these particulars shall be intimated to the Collector within
fifteen days of the change.

Conceptual Approach to Taxes 751


Federal Excise Rules, 2005 Chapter-04

6. Storage of excisable goods (U/R 16)


(1) Every manufacturer of excisable goods shall maintain a separate store-room or any other place of storage on
his registered manufacturing premises and deposit in such room or place or place all excisable goods
manufactured or produced by him.
(2) No duty paid goods or goods other than excisable goods shall be deposited in such store room or place of
storage.
7. Daily account of production, clearance and balances of excisable goods (U/R 17)
(1) every manufacturer shall maintain a production account in appropriate manner and shall enter daily in such
account the description, quantity and rating of all excisable and other goods which are:-
(a) Manufactured or produced in his factory;
(b) removed on payment of duty from the factory for home consumption;
(c) removed without payment of duty from factory for export outside Pakistan;
(d) issued for use in the manufacture of other goods within the factory or reprocessing or remanufacture;
(e) goods sent for value addition or further processing to other manufacturing premises; and
(f) in balance at the close of each day.
(2) In the case of removal of exempted goods, or of excisable goods on payment of duty, or without payment of
duty for export, the manufacturer shall maintain accounts, separately for each type of such removal.
(3) The manufacturer shall also maintain records of such inputs used in connection with manufacturing of goods
within the factory like electricity, fuel (used in manufacturing process) gas, telephone, wages of labour and
salaries and other emoluments of staff and rent paid for the premises.
Export drawback and refund of duty
8. Drawback of duty paid on goods exported (U/R 32)
(1) Subject to the conditions and limitations contained in this chapter, drawback of duty paid on any excisable
goods may be granted by the Collector of Federal excise or the officer authorized by him in this behalf, if such
goods are exported.
(2) The board may withdraw or disallow grant of drawback of duty paid on any excisable goods.
(3) The claim for the drawback of duty shall be lodged, within three months of the date on which the goods were
exported, with the Collector in whose jurisdiction the person is registered.
(4) The drawback may be granted by the Collector or the authorized officer if he is satisfied that duty had actually
been paid on the goods which were exported and that the goods were duly exported in accordance with the
prescribed procedure.
(5) No drawback under this rule shall be admissible if the goods, after removal from the factory on payment of
duty, were subjected to any further process of manufacture or otherwise tampered with after such removal and
before export.
9. Drawback of duty on exported goods which are made from excisable goods (U/R 33)
(1) The Central Board of Revenue may, by notification in the official Gazette, grant drawback of duty paid on any
goods used in the manufacture of any goods exported out of Pakistan, except those specified by the Board
under sub-rule (2) of rule 32 at such rate or rates and subject to such conditions and limitations as may be
specified in the notification.
(2) No drawback of Federal excise duty shall be granted in the following cases, namely:
(a) goods exported from Pakistan by land route except as declared exportable against drawback of Federal
excise duty as per conditions prescribed under the relevant Export Policy;
(b) export of consumer goods to any country, in retail packings bearing the retail price in Pakistani rupees;
and
(c) export of excisable goods in retail packing not printed, in bold letters, with the words:-
(i) "NOT FOR SALE IN PAKISTAN" or such other code as the Federal Board of Revenue may, for
reasons to be recorded in writing, approve;
(ii) "FOR EXPORT ONLY"; or MANUFACTURED FOR ________ (the name of their customer).

752 Conceptual Approach to Taxes


Federal Excise Rules, 2005 Chapter-04

(3) A person desiring to be granted a drawback of duty under sub-rule (1) in respect of goods in the manufacture
of which excisable goods have been used and which are to be exported shall make an application in
quadruplicate signed by him or his authorized agent to the Board declaring therein the name and address of
his business, the description, quantity and value of excisable and non-excisable goods used, the rate and
amount of excise duty levied and the value of goods for export.
(4) On the receipt of an application under sub-rule (3), the Board may cause such surveys or enquiries to be
made as it deems necessary to enable it to decide whether any drawback should be granted and if so, at what
rate or rates and from what date.
(5) In order to obtain payment of drawback the applicant shall produce before the officer authorized by the
Collector of Federal excise in this behalf, the shipping documents certifying the export of the consignment.
After satisfying himself that the claim is in order, officer shall sanction the payment of the drawback in
accordance with the relevant notification and these rules.
(6) If any of the particulars entered in the application submitted under this rule is found to be incorrect, either
before or after the export of goods, the applicant shall be liable:
(a) to a penalty under the Act and these rules for each breach of any provision of this rule;
(b) to refund to the Government the sums received by him as drawback; and
(c) to be deprived of the benefit of such drawback for a period of one year.
(7) No drawback shall be granted if the claim for drawback is filed after one hundred and twenty days of the
exportation or of the publication of notification, whichever is later.
10. Pecuniary competence to sanction drawback or refund (U/R 34)
The claims for refund or drawback .if duty of excise shall be decided by the following officers of Federal excise,
namely:
Assistant collector Not exceeding rupees two hundred thousand
Deputy collector Not exceeding rupees one million
Additional collector Unlimited

Conceptual Approach to Taxes 753

You might also like